100% found this document useful (4 votes)
4K views1,196 pages

Physics For IIT - JEE & All Other Engineering Examinations, - Ashwani Kumar Sharma - II, 1, 2019 - Wiley India - 9789389307245 - Anna's Archive

Physics for IIT-JEE and All Other Engineering Examinations: Mechanics II covers important concepts in circular motion and collisions required for the IIT-JEE exam. The book provides thorough theory explanations, solved examples, and practice problems. Key topics include angular displacement and velocity, centripetal acceleration and force, Coriolis force, banking of roads, dynamics of circular motion, and analysis of 1D and 2D collisions. Daily practice problems at the end help students prepare for the exam.
Copyright
© © All Rights Reserved
We take content rights seriously. If you suspect this is your content, claim it here.
Available Formats
Download as PDF, TXT or read online on Scribd
100% found this document useful (4 votes)
4K views1,196 pages

Physics For IIT - JEE & All Other Engineering Examinations, - Ashwani Kumar Sharma - II, 1, 2019 - Wiley India - 9789389307245 - Anna's Archive

Physics for IIT-JEE and All Other Engineering Examinations: Mechanics II covers important concepts in circular motion and collisions required for the IIT-JEE exam. The book provides thorough theory explanations, solved examples, and practice problems. Key topics include angular displacement and velocity, centripetal acceleration and force, Coriolis force, banking of roads, dynamics of circular motion, and analysis of 1D and 2D collisions. Daily practice problems at the end help students prepare for the exam.
Copyright
© © All Rights Reserved
We take content rights seriously. If you suspect this is your content, claim it here.
Available Formats
Download as PDF, TXT or read online on Scribd
You are on page 1/ 1196

es

ludst
TM

nc
I at nse

IIT-JEE Qu
L stio
e

II
II I IT - J E E &
Physics for IIT - JEE (Mechanics-II) is designed for the aspirants of JEE and
other engineering examinations. All Other Engineering Examinations
The volumes contain to the point theory and numerous solved numericals along
with practice tests having subjective type, objective type, matrix matching and
integer type questions which provide the aspirants a thorough understanding of
the subject and help prepare them for any type of problem asked in IIT - JEE.
II
Key Features
Develop and hone the problem Mechanics I 20%
solving technique
Abundant tips, short-cut methods
for solving specific problems Oscillations, Waves, Heat
and Thermodynamics 17%
Plenty of conceptual and
application-based numericals with Electrostatics and
detailed solutions and free-body Magnetism 23%
diagrams for better understanding
Optics and Modern Physics 20%

978-93-89307-24-5
Sharma Ashwani Kumar Sharma
` 925/-
Distributed by:
9 789389 307245 TM
Physics for IIT-JEE
&
All Other Engineering Examinations

MECHANICS II

As per NTA Syllabus


Physics for IIT-JEE
&
All Other Engineering Examinations

MECHANICS II
As per NTA Syllabus

Ashwani Kumar Sharma


Ex Sr Faculty, Kota
©Copyright 2019 I.K. International Pvt. Ltd., New Delhi-110002.

This book may not be duplicated in any way without the express written consent of the publisher,
except in the form of brief excerpts or quotations for the purposes of review. The information
contained herein is for the personal use of the reader and may not be incorporated in any commercial
programs, other books, databases, or any kind of software without written consent of the publisher.
Making copies of this book or any portion for any purpose other than your own is a violation of
copyright laws.

Limits of Liability/disclaimer of Warranty: The author and publisher have used their best efforts in
preparing this book. The author make no representation or warranties with respect to the accuracy or
completeness of the contents of this book, and specifically disclaim any implied warranties of
merchantability or fitness of any particular purpose. There are no warranties which extend beyond the
descriptions contained in this paragraph. No warranty may be created or extended by sales
representatives or written sales materials. The accuracy and completeness of the information provided
herein and the opinions stated herein are not guaranteed or warranted to produce any particulars
results, and the advice and strategies contained herein may not be suitable for every individual.
Neither Dreamtech Press nor author shall be liable for any loss of profit or any other commercial
damages, including but not limited to special, incidental, consequential, or other damages.

Trademarks: All brand names and product names used in this book are trademarks, registered
trademarks, or trade names of their respective holders. Dreamtech Press is not associated with any
product or vendor mentioned in this book.

ISBN: 978-93-89307-24-5

EISBN: 978-93-89698-47-3

Edition: 2019
Preface

It is with great pleasure that I place before you the Physics for IIT-JEE: Mechanics II (the
second in the series) which carries 20% weightage in IIT-JEE.
 Ŗ #NNKORQTVCPVEQPEGRVUCPINGRTQDNGOUUJQTVEWVOGVJQFUVTKEMUCPFVKRUJCXGDGGP
RTQXKFGF
 Ŗ 6JGVJGQTGVKECNRCTVJCUVQVJGRQKPVVJGQT[HQNNQYGFD[UGXGTCNV[RGUQHEQPEGRVWCN
UQNXGF PWOGTKECN GZCORNGU YJKEJ IKXG VJG UVWFGPV C VJQTQWIJ MPQYNGFIG QH VJG
VQRKE
 Ŗ #V VJG GPF QH GCEJ EJCRVGT VJGTG KU C RTCEVKEG VGUV UGV UQ VJCV UVWFGPV ECP CPCN[\G
VJGOUGNXGU

Useful Tricks while Preparing for IIT-JEE


s 3TUDY THEORY PART TWICE OR THRICE TO MAKE IT CRYSTAL CLEAR
s 7HILE SOLVING A NUMERICAL PROBLEM IF YOU FACE PROBLEM AGAIN STUDY THE THEORY PART OR
visit the conceptual solved problem where you will get a hint to solve that numerical.
s &OCUS ON SELF STUDY AND MAKE A ROUTINE AND FOLLOW IT STRICTLY
s $O NOT STRAIN YOURSELF MUCH ON THE EVE OF THE EXAMINATION
Ashwani Kumar Sharma
Ex Sr Faculty, Kota
Contents

Preface .....................................................................................................................................................v

 %KTEWNCT/QVKQP 1
Angular Displacement 1
Angular Velocity 1
Angular Acceleration Denoted by a 2
Relationship Between Angular and Linear Variables 2
Angular Velocity in General 4
Relative Angular Velocity 4
Acceleration in Circular Motion 8
Centripetal Acceleration 9
Centripetal Force 11
Centrifugal Force 11
Analysis of Conical Pendulum 13
What is Coriolis Force? 17
Death Wall or Rotor 21
Motion of Cyclist on Circular Road 22
Banking of Roads and Rails 22
Motion on a Plane Circular Path 24
Why There is Overturning? 25
Dynamics of a Circular Motion in a Vertical Circle 27
Formulae Used in Dynamics of a Circular in a Vertical Circle 29
Daily Practice Problems for JEE Main and Advance 104

 %QNNKUKQPU 133


What is a Collision? 133
Collisions 133
Elastic Collision in One Dimension 135
Coefficient of Restitution 137
viiiപŽŶƚĞŶƚƐ

Elastic Oblique Collision or Elastic Collision in Two Dimensions 158


Centre of Mass 175
Newton’s Second Law for System of Particles 175
Centre of Mass of n-Particle 177
Centre of Mass of a Rigid Body 177
Centre of Gravity 181
Centroid 182
Tilting of a Body 196
Stability of Equilibrium 203
Daily Practice Problems for JEE Main and Advance 300

 4QVCVKQP 349


Particle 349
What is a Rigid Body? 349
Equations of Motion of a Rotating Body 350
What Happens When Wheel in Contact Rotates? 352
Torque or Moment of Force 358
Couple 362
What is Instantaneous Axis of Rotation? 370
What is Moment of Inertia or Rotational Inertia? 372
What is Radius of Gyration? 373
Important Theorems of Moment of Inertia 373
Angular Momentum 401
Rotational Kinetic Energy and Moment of Inertia 408
Rotational Work 409
Angular Impulse 410
Analogy Between Linear and Rotational Motions 417
Rotation About a Moving Axis 425
Rolling Motion 445
Accelerated Pure Rolling 464
Direction of Friction in Case of Translation and Rotation Combined 524
What is Percussion? 548
Daily Practice Problems for JEE Main and Advance 586

 )TCXKVCVKQP 652


Kepler’s Laws 652
Newton’s Law of Gravitation 654
The Two Shell Theorems 655
Derivation of Kepler’s Laws 656
Intensity of Gravitational Field 662
Variations in Acceleration Due to Gravity 665
Variation of g with Altitude (Height) 665
ŽŶƚĞŶƚƐപix

Variation of g with Depth 667


Gravitational Potential 675
Escape Velocity 683
Motion of Satellite in a Circular Orbit 688
Orbital Velocity 689
Time Period of Revolution 690
Energy of a Satellite 691
Binding Energy 691
Geostationary Satellite (GSS) 692
Black Hole 695
Weightlessness 701
Reduction of Two-Body Problem to One-Body Problem 705
Daily Practice Problems for JEE Main and Advance 795

 /GEJCPKECN2TQRGTVKGUQH/CVVGT 817


Introduction 817
Intermolecular Forces 818
Origin of Intermolecular Forces 818
Types of Stress and Strain 821
What is Meant by Elastic Limit? 823
Hooke’s Law 823
Different Types of Modulii of Elasticity 823
Elastic Hysteresis 825
Poisson’s Ratio 833
Elastic Potential Energy of a Strained Body: Strain Energy 839
Torsion 842
Twisting of a Shaft is Same as Twisting Couple on Cylinder 843
Surface Tension 848
Examples Based on Surface Tension 849
Molecular Theory of Surface Tension 849
Surface Energy and Surface Tension 851
Work Done in Increasing the Area of the Surface Film 851
Pressure Difference 852
Angle of Contact 857
Capillary Rise 858
Concept of Capillarity 861
Viscosity 879
Factor Affecting Viscosity 882
Viscosity Versus Friction 882
Poiseuille’s Equation 882
Daily Practice Problems for JEE Main and Advance 937
xപŽŶƚĞŶƚƐ

 *[FTQUVCVKEUQT(NWKF5VCVKEU 968


Introduction 968
Mass Density and Specific Weight 969
Thrust or Normal Force of a Liquid 972
Pressure 972
Variation of Liquid Pressure with Depth 974
Pascal’s Law 975
Applications of Pascal’s Law 977
Measuring Pressure 981
Hydrostatic Forces on Surfaces 991
Buoyant Force and Buoyancy 999
Archimedes’ Principle 999
Relative Density 1001
Principle of Floatation 1001
Fluid Dynamics 1030
Reynolds Number 1032
Ideal Fluid 1034
Equation of Continuity 1034
Energy of a Flowing Fluid 1035
Bernoulli’s Principle 1036
Derivation of Bernoulli’s Equation 1036
Applications of Bernoulli’s Equation 1039
Speed of Efflux or Torricelli’s Law of Efflux 1044
Time of Emptying a Tank 1048
Time of Emptying (or Filling) a Tank with Inflow 1048
Siphon Tube 1049
Daily Practice Problems for JEE Main and Advance 1134
1 CHAPTER

Circular Motion

ANGULAR DISPLACEMENT
The angle swept by the particle moving a circular path is called Dl
angular displacement. If a particle travels a distance Ds, then angular
displacement Dq

Dq = Dl r
r
l
or q = simply. VVIP formula
r
The SI unit angular displacement is radian. It is a vector quantity, Fig. 1.1
provided it to be small.

ANGULAR VELOCITY z

Consider a particle moving along a circle.


r r
Let
 it undergo an angular displacement Here, w = w k
Dq in time Dt. We can define the average
angular velocity of the particle as r
w
 r y
 Dq v2
r
w = v1
Dt
The instantaneous angular velocity Dq
 
 Dq  dq
w = lim or w =
Dt Æ0 Dt dt x v1 = v2
Angular velocity is a vector quantity Fig. 1.2
whose direction is along the axis of rotation
and given by the right hand screw rule. Its SI unit is rad/s. If a particle makes n cycles or
revolutions in one second, then
w = n ¥ 2p or w = 2pn
1
Since n = , where T is time to complete one revolution
T
2p
\ w =
T
2പWŚLJƐŝĐƐĨŽƌ//dͲ:͗DĞĐŚĂŶŝĐƐ//

ANGULAR ACCELERATION DENOTED BY a


If the angular velocity of a rotating body is z
not constant, then the body has an angular r
acceleration. Let w1 and w2 be the angular Here, a = akˆ

velocities at time t1 and t2 respectively. The r


a
average angular acceleration
   r
 w 2 - w 1 Dw Dw
a av = = .
t2 - t1 Dt r
y
v2
The instantaneous angular acceleration r
 
 Dw dw r
v1
a = lim =
DtÆ0 Dt dt
dq x v1 = v 2
As w = ;
dt d 2q
a= 2 Fig. 1.3
dt
Angular acceleration is a vector quantity, its direction is along the axis of rotation. Its SI
unit is rad/s2 with dimension T –2.

RELATIONSHIP BETWEEN ANGULAR AND LINEAR VARIABLES


Let in time Dt the arc distance travelled by particle be Ds, then we have
Dl = r Dq

or Dl = r Dq r Dl

Dt Dt Dq
or v = rw O

Differentiating the above equation w.r.t. time, we get tangential ac-


celeration Fig. 1.4
dv dw
at = =r
dt dt
or at = ra.
In the uniform circular motion, speed of the particle remains constant hence, at = 0. (tangential
acceleration)
z
z

r
w r
r w
a r
a
y
y

v
r r v
a1 at
x
x
(a) Particle is speeding up (b) Particle is slowing down

Fig. 1.5
ŝƌĐƵůĂƌDŽƟŽŶപ3

 r
Angular Velocity (w ) in Vector Form w

Consider a particle P revolving around an axis with an angular


 
velocity w. Let r be the position vector of the particle with respect r
v
r sin q
to any point Q on the axis. Since the particle describes a circle
of radius (r sin q) lying in the plane perpendicular to the axis of
rotation, the magnitude of linear velocity
v = w r sin q r
r
 
and its direction is perpendicular to the plane containing r and w.
q
By right hand screw rule, we can write
   Q
v =w¥r
Fig. 1.6
Change in Position (Linear Displacement)
If particle changes its position from P to Q in time t, then displacement
   
PQ = Dr = r2 - r1 Q
r
  r2 r
or Dr = |r2 - r1|= r12 + r22 - 2r1 r2 cos q Dr
q
For r1 = r2 = r r P
r1
Dr = r 2 + r 2 - 2r 2 cos q = 2r 2 (1 - cos q )
q
Dr = 2r sin
2
Fig. 1.7
Change in Velocity
Change in velocity after traversing angle q
   r
Dv = v 2 - v1 v2
r
  v1
or Dv = |v 2 - v1|= v12 + v22 - 2v1v2 cos q r
q
r2
For uniform speed v1 = v2 = v = wr
q r
2 2 2 r1
Dv = v + v - 2v cos q
È 2q˘
= 2v 2 (1 - cos q ) ÍÎ1 - cos q = 2 sin 2 ˙˚
q Fig. 1.8
or Dv = 2v sin
2
If two particles are moving on same circle or different circles in same plane with different
angular velocities w1 and w2 respectively, the angular velocity of 2 relative to 1 will be
  
w 21 = w 2 - w 1
or w 21 = w 2 – w 1.
4പWŚLJƐŝĐƐĨŽƌ//dͲ:͗DĞĐŚĂŶŝĐƐ//

Time taken by 2 to complete one revolution w.r.t. particle 1 is given by


v2 v1
2p 2p 2p
T = = = . r2 r1
w 21 w 2 - w 1 2p
-
2p
T1 T2
T1 T2
\ T =
T2 - T1
Fig. 1.9

ANGULAR VELOCITY IN GENERAL


Consider the situation, in which a particle is not on a circular path. v
Suppose the particle D at any instant is going along the tangent of the 
r q
curve. Let its speed be v and its direction of motion makes an angle q O
 D
with the radius vector r relative to D (see Fig. 1.10).
The angular velocity of particle D is defined as
The velocity component perpendicular to OD Fig. 1.10
w =
OD
v^
or w = ...(1)
r
Here, v^ = v sin q
v sin q rv sin q
\ w = =
r r2
|r ¥ v|
or w = ...(2)
r2

RELATIVE ANGULAR VELOCITY


1. Suppose two particles D1 and D2 are moving with speeds v1 and v2 as shown in
Fig. 1.11.

v2
q2
sin (v2 sin q 2 – v1 sin q 1)
v2
q2

D2 D2

v1
nq
1
si  
v1 r r
q1

D1 D1

Fig. 1.11
ŝƌĐƵůĂƌDŽƟŽŶപ5

We wish to find angular velocity of particle 2 with respect to 1. It is


  
w 21 = w 2 - w 1
v^ 2 - v^ 1
or w21 =
r
È v sin q 2 - v1 sin q1 ˘
or w21 = Í 2 ˙˚ ...(1)
Î r
2. Consider two particles D1 and D2 which are moving with veloci- (90° – q)
   
ties v1 and v 2 . At an instant their position vectors are r1 and r2
relative to the origin. Let q be the angle between their position
 
vectors, then angular velocity w of D2 with respect to D1 is given  D1
v2
v1  q
by r1 D2
     q 
v 2 - v1 = w ¥ ( r2 - r1 ) r2
 
Multiply both side by ( r 2 - r1)
        
or ( r2 - r1 ) ¥ (v 2 - v1 ) = ( r2 - r1 ) ¥ {w ¥ ( r2 - r1 )}
Fig. 1.12
By using vector triple product, we can write
             
( r2 - r1 ) ¥ (v 2 - v1 ) = {( r2 - r1 ) ◊ ( r2 - r1 )}w - {( r2 - r1 ) ◊ w }( r2 - r1 )
...(1)
  
As w is perpendicular to the plane of r2 - r1,
        
\ {( r2 - r1 ) ◊ w } = 0 and {( r2 - r1 ) ◊ ( r2 - r1 )} =|( r2 - r1 )|2
Now Eq. (1) becomes    
 ( r - r ) ¥ (v - v )
w = 2 1  22 1 ...(2)
|r2 - r1|
If ĥ is the unit vector perpendicular to the plane of the motion, then
       
 ( r ¥ v 2 + r1 ¥ v1 - r2 ¥ v1 - r1 ¥ v 2
w = 2   ...(3)
|r2 - r1|2
For particles on circular paths,
  =rv h  and
r2 ¥ v 2 = r2 v2 sin 90∞ h 2 2
  =rv h .
r1 ¥ v1 = r1v1 sin 90∞ h 1 1
   = r v cos q h and
r2 ¥ v1 = r2v1 sin (90∞ + q )h 2 1
   = r v cos q h
r1 ¥ v 2 = r1v2 sin (90∞ - q )h 1 2
   
 È r v + r v - ( r v + r v )cos q ˘ 
\ w = Í 2 2 21 1 2 2 1 1 2 ˙h ...(4)
ÍÎ r1 + r2 - 2r1r2 cos q ˙˚
Important Cases
1. When D2 is nearest to D1, q = 0° and Eq. (4) reduces to,
v2 - v1
w=
r2 - r1
6പWŚLJƐŝĐƐĨŽƌ//dͲ:͗DĞĐŚĂŶŝĐƐ//

2. When D2 is farthest to D1, q = 180° and cos q = – 1 v2 r


r2
v2 + v1 v1 r
\ w= r1
r2 + r1 q=0
3. For w to be zero,
Èv r + v r ˘
cos q = Í 2 2 1 1 ˙
Î v1r2 + v2 r1 ˚ Fig. 1.13

WORKED PROBLEMS
Problem 1.1 A particle is spinning with an angular velocity of 4 rad/s about an axis parallel
 passing through the point i + 3j - k
to 3j - k  . Find the velocity of the particle at the point
  
4i - 2j + k .
 then, n
 be the unit vector in the direction of 3j - k =

3j - k (3j - k
)
Solution Let n =
  32 + (-1)2 10
\ angular velocity of the particle w = w n
4
=  ) rad/s.
(3j - k
10
The position vector of the point with reference to point (i + 3j - k )

r = ( 4i - 2j + k  ) - (i + 3j - k
)
= 3i - 5j + 2k  m.
  
Hence, linear velocity v = w ¥ r
4
=  ) ¥ (3i - 5j + 2k
(3j - k )
10
-4   
= (i + 3j + 9k) m/s. Ans.
10
Problem 1.2 A rigid body is moving parallel to x-axis with constant speed v at a distance
a from the axis. Find its angular velocity about an axis passing through the origin O, at the
instant when radial vector of the rigid body makes angle q with the x-axis.
Solution The velocity of the particle perpendicular to the radial y

vector r is v sin q. Also r sin q = a.
Thus, by the definition v
r 90° – q
v r
w = ^ a
v sin q
r q
o x
v sin q v
= = sin 2 q .
( a/sin q ) a Fig. 1.14

Problem 1.3 Two balls of mass M = 9 g and m = 3 g are attached by


threads AD and OB whose combined length is l = 1 m to a vertical m M
axis O (Fig. 1.15) and are set in rotational motion in a horizontal plane B
O
A
about this axis with a constant angular velocity w . Determine the ratio
of the lengths AO and OB for which the tensions in the threads will Fig. 1.15
be the same. Disregard the weight of the threads.
ŝƌĐƵůĂƌDŽƟŽŶപ7

Solution If l1 and l2 are the lengths of the parts OB and AO respectively, then for same
tension in both the threads we can write
T = mw 2 l1 ...(1)
and T = Mw 2 l2 ...(2)
l1 M
\ =
l2 m
l1 9
or = =3 ...(i)
l2 3
Also, l1 + l2 = 1 ...(ii)
From the above equations, we get
l1 = 0.75 m and l2 = 0.25 m Ans.
Problem 1.4 Two bodies A and B are moving as shown in Fig. 1.16. At this moment of time,
find the angular speed of A relative to B.

vA vA sin qA

vB sin qB vB

qA qB
vA cos qA A B vB cos qB
r

Fig. 1.16

Solution We know that [vAB]y = vAy – vBy


= vA sin qA – vB sin qB
[vAB ]y
and w =
AB
È v sin q A - vB sin q B ˘
=Í A ˙˚ in clockwise direction.
Î r
As there is no separation between the velocity component along x-axis, thus, there is no
specific rotation due to (vAB)x = vA cos qA – vB cos qB.
Problem 1.5 A solid body starts rotating about a stationary axis with an angular acceleration
b = at, where a = 2.0 ¥ 10–2 rad/s2. How soon from the beginning of rotation will the total
acceleration vector of an arbitrary point of the body form an angle a = 60° with its velocity
vector?
(a) 7 sec (b) 4 sec (c) 2 sec (d) 6 sec
dw
Solution Given: b = = at; at = br = atr
dt
\ dw = (at) dt
8പWŚLJƐŝĐƐĨŽƌ//dͲ:͗DĞĐŚĂŶŝĐƐ//

On integrating, we get
at 2 v
w = ai
2 an
a 2t 4 r
\ a n = w 2r =
4

As at is along velocity vector v, so Fig. 1.17
a
tan 60° = n
at

a 2t 4 r/4 at 3
or 3 = =
atr 4
1 1
È4 ¥ 3 ˘3 È 4 ¥ 3 ˘3
\ t =Í ˙ =Í -2 ˙
=7s Ans.
Î a ˚ Î 2 ¥ 10 ˚

Ans. (a) is correct.

Unit Vector along the Radius and the Tangent


 
Let us draw a unit vector OP = r along the outward radius and a unit vector PQ = q along
the tangent in direction of increasing q. We call r̂ radius unit vector and q the tangential
unit vector. Thus,
 y
OP = OP cos q i + OP sin q j (as OP = 1)
\ r̂ = cos q i + sin q j
Q q̂
Similarly,
 P
PQ = PQ sin q i + PQ cos q j (as PQ = 1) r̂
q = - sin q i + cos q j q
x
O

As r and q̂ are mutually perpendicular,
So rˆ ◊ qˆ = 0.
Fig. 1.18
ACCELERATION IN CIRCULAR MOTION
Uniform Circular Motion
If the particle is moving along a circular path with constant speed is called uniform circular
motion. In motion along circular path, the velocity of particle always changes due to change

in direction of motion. Due to which there is an acceleration, which is called normal or radial
v2
acceleration. Its magnitude is found to be .
r
Non-Uniform Circular Motion
If a particle is moving along a circular path with variable speed, its velocity changes due to
change in direction of motion and also by change in magnitude of the velocity. In such a case
there are two perpendicular accelerations:
ŝƌĐƵůĂƌDŽƟŽŶപ9

(i) Normal or Radial Acceleration/Centripetal Acceleration (an )


v2
This acceleration is due to change in direction of motion and is given by an = .
r
(ii) Tangential Acceleration (at)
dv
This acceleration is due to change in magnitude of velocity and is given by at = = a r.
Thus, resultant acceleration dt
  
a = a n + at a at

 Ê v2 ˆ
a = - Á ˜ r̂ + ÊÁ ˆ˜ q
dv
or
Ë r¯ Ë dt ¯ O an

and a = an2 + at2



Here it must be noted that at governs the magnitude of v
where an its direction of motion so that
Fig. 1.19
If an = 0 and at = 0
a Æ 0 motion is uniform translatory
If an = 0 but at π 0
a Æ at motion is accelerated translatory
If an π 0 but at = 0
a Æ an motion is uniform circular
If an π 0 but at π 0

a= an2 + at 2 motion is non-uniform circular.

CENTRIPETAL ACCELERATION r
v
P
Method I: Let us consider a particle moving along a circular path of
radius r with constant angular speed w. Its position vector at any time r an
q
t is given by
  O
r = OP
= OP r

or r = r(cos q i + sin q j) ...(i)
Fig. 1.20
Differentiating Eq. (i) with respect to time, we get velocity

 dr dq dq
v = = r(- sin q i + cos q j)
dt dt dt
Ê dq ˆ
= r Á ˜ ( - sin q i + cos q j)
Ë dt ¯

or v = rw ( - sin q i + cos q j) ...(ii)
10പWŚLJƐŝĐƐĨŽƌ//dͲ:͗DĞĐŚĂŶŝĐƐ//

We know that,
dq
= w and ( - sin q i + cos q j) = q
dt
 
\ v = rw q and|v|= w r .
Acceleration can be obtained by differenting Eq. (ii). Thus,

 dv d
an = = [rw ( - sin q i + cos q j)]
dt dt
È dq  dq ˘
= rw Í - cos q i - sin q j
Î dt dt ˙˚
Ê dq ˆ
= - rw Á ˜ [cos q i + sin q j]
Ë dt ¯
È dq ˘
= - rw 2 r̂ ÍÎ dt = w ˙˚
  v2
or a n = - w 2 r and an = w 2 r =
r

Here a n is called normal acceleration and its direction is towards the centre of circular
path.
The minus sign shows that the acceleration is towards the origin (centre of circle) known
as centripetal acceleration.
Method-II: Let us consider a particle which traverses angle Dq in time Dt, with reference to
the coordinate axes shown in Fig. 1.21, we have,
Dq Dq
vPx = v sin and vPy = v cos
2 2
Dq Dq
and VQx = - v sin , VQy = v cos
2 2
\ Dvy = vQy – vPy y
Dq Dq
= - v cos - v cos =0 v vy
2 2
and Dvx = vQx – vPx Q
–vx
Dq Dq Dq
= - v sin - v sin = - 2v sin . Dq/2
2 2 2 x
Dq/2 vy v
Dq Dq
As Dq is small, so sin ª
2 2 vx
Dq P
\ Dvx = – 2v = - vDq ...(i)
2 Fig. 1.21
Dividing Eq. (i) by Dt, we get centripetal acceleration
Dvx Dq
an = =-v = - v w = -(w r )w = -w 2 r .
Dt Dt
The minus sign tells us that the acceleration is towards the origin (centre of circle).
ŝƌĐƵůĂƌDŽƟŽŶപ11

CENTRIPETAL FORCE
For a particle moving in a circular path, it has an acceleration towards r
the centre of path. According to Newton’s second law the particle must v
experience a net force towards the centre of path. This force is called
O
centripetal force. If an is the centripetal acceleration, then centripetal F n an
force is given by
 
Fn = ma n
mv 2 Fig. 1.22
or Fn =
r
Centripetal force may be gravitational force, tension, frictional or Coulomb’s force.
For example: A body is rotating in a circle with the help of a string. Here tension in the string
is the centripetal force.

O
T

Fig. 1.23

For example: Motion of the earth around the sun. Here gravitational force is the centripetal
force.

r
v

Sun Earth
F F

Fig. 1.24

Important:
When force acts always perpendicular to velocity, the resulting motion will be uniform
circular motion.
r
v
CENTRIFUGAL FORCE
Consider a block of mass m placed on a table top at a distance r Sun Earth
F F
from its centre. Suppose the table rotates with constant angular
velocity w and block remains at rest with respect to table. Let us
first analyse the motion of the block relative to an observer on
the ground (inertial frame). In this frame the block is moving in Fig. 1.25
12പWŚLJƐŝĐƐĨŽƌ//dͲ:͗DĞĐŚĂŶŝĐƐ//

v2
a circular of radius r. It, therefore, has an acceleration
towards the centre. The resultant
r
mv 2
force on the block must be towards the centre and its magnitude is . In this frame the
forces on the block are: r

(i) Weight mg
(ii) Normal reaction N
(iii) Frictional force f by the table.
Thus, we have, N = mg ...(i)
and by Newton’s second law of motion
mv 2
f = ...(ii)
r

w w

N N
r f mv 2/r
f
mg mg

(a) FBD of the block in inertial frame (b) FBD of the block in non-inertial frame

Fig. 1.26

Now observe the same block in a frame attached with the rotating table. The observer
here finds that the block is at rest. Thus, the net force on the block in this frame must be
zero. The weight and normal reaction balance each other but frictional force, f acts on the
block towards the centre of the table. To make the resultant zero, a pseudo force must be
mv 2
assumed which acts on the block away radially outwards and has a magnitude . This
r
pseudo force is called centrifugal force. In this frame the forces on the block are:
(i) Weight mg
(ii) Normal reaction N
(iii) Frictional force f
mv 2
(iv) Centrifugal force
r
Thus, we have SFvertical = 0
or N – mg = 0 ...(i)
and SFradius = 0
mv 2
or f– =0 ...(ii)
r
ŝƌĐƵůĂƌDŽƟŽŶപ13

Important
We have got the same result in non-inertial frame as we have got in inertial frame. It should
be remembered that the centrifugal force is taken into account only in rotating frame (non-
inertial frame).

ANALYSIS OF CONICAL PENDULUM


Consider a conical pendulum of length l. It is made to rotate about a vertical axis. Suppose
string of the pendulum makes an angle q with the axis. Its motion can be studied in two
frames. These are: inertial frame and rotating frame.

w w

q q

l 

q T T
mw 2r q r
r
mg mg
(a) (b)

Fig. 1.27

(a) In inertial frame of reference, we have


T sin q = mw2r ...(i)
and T cos q = mg ...(ii)
From Eqs. (i) and (ii), we get
w 2r
tan q =
g

w 2 l sin q
or tan q =
g
g
w =
l cos q

(b) In non-inertial frame, we have


S Fradius = 0
or T sin q – mw2r = 0 ...(i)
and SFV = 0
T cos q – mg = 0 ...(ii)
14പWŚLJƐŝĐƐĨŽƌ//dͲ:͗DĞĐŚĂŶŝĐƐ//

From Eqs. (i) and (ii), we get


w 2 r w 2 l sin q
tan q = =
g g
g
or w =
l cos q

Linear velocity of the bob, v = gr tan q


Angular velocity of the bob,
g g g
w = tan q = =
r h l cos q
Time period of revolution:
l cos q h
TP = 2p = 2p
g g

l2 - r 2 r
= 2p = 2p …(iii)
g g tan q
If q = 90°, the pendulum becomes horizontal and from Eqs. (i), (ii) and (iii) we get
v = •, T = • and Tp = 0 which is practically not possible.
r
Work Done in Circular Motion v

(i) In uniform circular motion, the only force is centripetal force, 90°
which acts perpendicular to the velocity. So the rate of doing O
Fn
work or power
dW  
P= = Fn ◊ v = Fn v cos 90∞ = 0 .
dt
(ii) In non-uniform circular motion, there are normal and tangential Fig. 1.28
forces. The rate of doing work r
v
dW   
P= = (Fn + Ft ) ◊ v
dt Ft
    90°
= Fn ◊ v + Ft ◊ v = Fn v cos 90∞ + Ft v cos 0∞ O
Fn
or P = Ft v,
where Ft is the tangential force which comes into play due
to tangential acceleration in non-uniform circular motion and
given by Ft = mat. Fig. 1.29

WORKED PROBLEMS
Problem 1.6 A biker is riding a bike with a speed of 27 km/h. As he approaches a circular
turn on the road of radius 80 m, he applies the brakes and reduces his speed at the constant
rate 0.5 m/s2. What is the magnitude and direction of the net acceleration of the biker on
the circular turn?
ŝƌĐƵůĂƌDŽƟŽŶപ15

Solution This is an example of non-uniform circular motion,


and therefore it has two accelerations:
v
Centripetal acceleration
an
v 2 (27 ¥ 5/18)2
an = = a
r 80 q
2 at
= 0.70 m/s
and tangential acceleration
at = 0.5 m/s2 Fig. 1.30
\ Total acceleration
a= an2 + at2

= 0.70 2 + 0.52 = 0.86 m/s2.


If q is the angle between the total acceleration and the velocity of the biker, then
an 0.7
tan q = =
at 0.5
or q = tan–1 (1.4) Ans.
y
Problem 1.7 A body moves in a circle of radius 4.0 cm clock- Q
wise at constant speed of 2 cm/s. If x and y are unit accel-
eration vectors along x-axis and y-axis respectively, find the
acceleration of the particle at the instant half way between P
and Q. Refer to Fig. 1.31. (NCERT)
O X
P
Solution The centripetal acceleration

v 2 22
a= = = 1 cm/s2
r 4
If R is the midpoint of the arc PQ, then –POR = 45° Fig. 1.31
Thus, if ax and ay are the components of the acceleration y
along x-axis and y-axis respectively, then Q
 1  ax
a x = a cos 45∞(- x ) = -
R
x
2 a
ay
45°
 1 
a y = a sin 45∞(- y ) = -
x
and y O P
2
  
Hence, a = ax + ay
1  
= - (x + y ) Ans.
2 Fig. 1.32

Problem 1.8 Kinetic energy K of a particle moving along a circle of radius R depends on
the distance s as K = as2, where a is a constant. What is the force acting on the particle?
16പWŚLJƐŝĐƐĨŽƌ//dͲ:͗DĞĐŚĂŶŝĐƐ//

1 2
Solution Given that mv = as2
2
2 as2
or v2 = ...(i)
m
Here speed depends on s, so there are normal and tangential forces both acting on the
particle.
mv 2 2 as2
Normal force Fn = = ...(ii)
R R
and tangential force Ft = mat
dv dv
= m = mv
dt ds
On differentiating Eq. (i) w.r.t. s, we get,

d 2 d Ê 2 as2 ˆ
(v ) =
ds ds ÁË m ˜¯
dv 2a
2v = ¥ 2s
ds m
dv 2as
or v =
ds m
2 as
\ Ft = m ¥ = 2 as
m
Now resultant force on the particle
2
Ê sˆ
F = Fn2 + Ft2 = 2 as 1 + Á ˜ Ans.
Ë R¯

Problem 1.9 A body of mass m is moving in a circular path of constant radius r such that,
its centripetal acceleration ac is varying with time t as ac = K2rt2, where k is a constant. What
is the power delivered to the particle by the forces acting on it? (IIT-JEE, 1994)
Solution Given that,
a c = k 2r t 2
v2
or = k2rt2
r
\ v = krt …(i)
dv d(krt)
Tangential acceleration, at = = = kr
dt dt
and Ft = mat = mkr
Power delivered, P = Ft v cos q = mkr ¥ krt cos 0°
= mk2r2t. Ans.
ŝƌĐƵůĂƌDŽƟŽŶപ17

w
     ǫ ȍ Ȏ
G. Coriolis, a French mathematician, found the theory of force.
This force arises in a rotating frame when a particle moves with r
respect to the frame. Coriolis force is in addition to centrifugal vt
force. Its direction depends on the direction of velocity of the
particle.
Consider a body moving with a uniform tangential speed v
with respect to a rotating table. The angular velocity of rotation
of the table is w and particle is at a distance r from the centre of
the table. The velocity of the body w.r.t. ground frame.
vT = (vt + wr). Fig. 1.33 Particle is moving
on a rotating table
The centripetal acceleration of the body
(vt + w r )2 vt2
a= = + w 2 r + 2w vt
r r
Thus, the centripetal force F = ma
mvt2
= + m(w 2 r + 2w vt ) .
r
mvt2
1. If the table was not rotating (w = 0) the body has the only force, F = in inertial
frame. r

Thus, due to rotating of the table the body experiences a pseudo force (mw2r + 2 mw vt).
If the body is at rest w.r.t. table vt = 0, then the only pseudo force is mw2r.
Thus, on a moving particle on a rotating table an extra pseudo force 2mw vt comes to
act which is called Coriolis force. Its direction is perpendicular to the direction of vt.
The expression of Coriolis force FCoriolis = 2mwv is free from r, so it does not depend
on the position of the particle but depends on the speed of the body.

vt 2mwvt
w r w
mw 2r + 2mwvt mw 2r
v
{
{

Centri- Coriolis
fugal
Pseudo force

(a) Body is moving along the sense (a) Body is moving opposite to the
of rotation of the table sense of rotation of the table

Fig. 1.34

2. The body is moving with uniform radial velocity vr with respect to rotating table. Here
centrifugal force is mw2r radially outward and Coriolis force is 2mwvr perpendicular
to vr.
18പWŚLJƐŝĐƐĨŽƌ//dͲ:͗DĞĐŚĂŶŝĐƐ//

Coriolis force
2mwv r

w w vr mw 2r
mw 2r
vr Centrifugal force

2mwv r
Coriolis force

(a) Body is moving radially towards (b) Body is moving radially away from
the centre of table the centre of table

Fig. 1.35

WORKED PROBLEMS
Problem 1.10 A table with smooth horizontal surface is w = 3 rad/s m
placed in a cabin which moves in a circle of a large radius R
= 100 m, with w = 3 rad/s (see Fig. 1.36). A smooth pulley of 2m
small radius is fastened to the table. Two masses m and 2 m
Fig. 1.36
placed on the table, are connected through a string going over
the pulley. Initially, the masses are held by a person with the strings along the outward radius
and then the system is released from rest (with respect to the cabin). Find the magnitude of
the initial acceleration of the mass m as seen from the cabin.
Solution By Newton’s second law, we can write T
mw2R
(2m)w2R – T = (2m)a ...(i)
(2m)w2R
and T – mw2R = ma ...(ii) T
After simplifying, above equation, we get Fig. 1.37
2
a = 300 rad/s
Problem 1.11 A closed chain D of mass m = 0.36 kg is attached to a vertical rotating shaft
by means of thread (Fig. 1.38) and rotates with a constant angular velocity w = 35 rad/s.
The thread forms an angle q = 45° with the vertical. Find the distance between the chain’s
centre of gravity and the rotation axis, and the tension of the thread.

Fig. 1.38
ŝƌĐƵůĂƌDŽƟŽŶപ19

Solution For circular motion of the chain


T sin q = mw2r
and T cos q = mg q

w 2r
\ tan q = q T
g r

g tan q mg
or r = Ans.
w2 Fig. 1.39
Problem 1.12
(i) A motor car is going on a bridge which is convex of radius R with constant speed v.
Find normal reaction when it is at an angular position q from the vertical.
(ii) A motor car is going on a concave bridge of radius R with constant speed v. Find
normal reaction when it is at an angular position q from the vertical.
Solution
(i) FBD is shown in Fig. 1.40. By Newton’s second law, we have
mv 2 N v
mg cos q – N =
R
q
mv 2
\ N = – mg cos q
R mg
q
With the decrease in q, N will decrease and so it will
be least at the top of the bridge.
mv 2 Fig. 1.40
It is N =
R
(ii) FBD of the motor car is shown in Fig. 1.41. By Newton’s second law, we have
mv 2 N
N – mg cos q =
R
q
mv 2
\ N = + mg cos q. q
R
With the decrease in q, N increases and so it will be
maximum at the bottom of the bridge. It is v

mv 2 mg
N = +mg
R
Fig. 1.41
Problem 1.13 A sleeve A can slide freely along a smooth rod bent in the shape of a half
circle of radius R. The system is set in rotation with a constant angular velocity w about a
vertical axis OO’. Find the angle q corresponding to the steady position of the sleeve.
20പWŚLJƐŝĐƐĨŽƌ//dͲ:͗DĞĐŚĂŶŝĐƐ//

Solution If q is the angle corresponding to steady position, then


N sin q = mw2 r = mw2 (R sin q) O
2
or N = mw R ...(i) N cosq
R
and N cos q = mg ...(ii) q N
Solving Eqs. (i) and (ii), we get mw2r
N sinq A
g w
cos q = (for g < w2 R) O¢ mg
w 2R
If g > w2 R, then steady position will correspond to q = 0. Fig. 1.42 FBD of sleeve in
rotating frame

Problem 1.14 A disc rotates in horizontal plane z


with a constant angular velocity w = 6.0 rad/s w
about a vertical axis passing through its centre.
A small body of mass m = 0.50 kg moves along
a diameter of the disc with a velocity v = 50 N
cm/s which is constant relative to the disc. Find
the force that the disc exerts on the body at the
mw2r y-axis
moment when it is located at the distance r = 30
90°
cm from the rotating axis.
Solution The body will experience three mutu- 2mwv
mg
ally perpendicular forces in the rotating table. x-axis

(a) Centrifugal force mw2 r. Fig. 1.43


(b) Normal reaction N = mg.
(c) Coriolis force 2mw v.
The resultant force = (mw 2 r )2 + (mg )2 + (2mw v)2 = 8 N Ans.

Problem 1.15 A tabletop with smooth horizontal frictionless surface is turning at an


angular speed w about its axis. A groove is made on the surface along a radius and a particle
is gently placed inside the groove at a distance a from the centre. Find the speed of the
particle with respect to the table as its distance from the centre becomes b.
Solution In a turning table the force along the radius of the table is the centrifugal force.
Thus, the acceleration of a particle when it is at a distance x from the centre is
a = w2 x w
dv
or v = w2 x
dx
or v dv = w2 x dx mw2x
Integrating the above equation, we get x
v b

Ú v dv = Úw
2
xdx
0 a

Fig. 1.44
ŝƌĐƵůĂƌDŽƟŽŶപ21

v b
v2 2x2
= w
2 0
2 a

or v = w (b – a2)
2 2 2

\ v = w b 2 - a2 Ans.
Problem 1.16 A metal ring of mass m and radius R is placed on a smooth horizontal table
and is set rotating about its own axis in such a way that each part of the ring moves with a
speed v. Find the tension in the ring. (IIT Roorkee, 1974)
Solution Take a small part of the ring which subtends Dq
Dq
an angle Dq at the centre of the ring. Let T be the tension T cos T cos
2 Dq Dq 2
in the ring. The forces act on this part in the plane of 2 Dq
2 2T sin
rotation are shown in Fig. 1.45. The mass of the small Dq Dq 2
m
part of ring Dm = Dq . T
2 2
T
2p
Dq
The centripetal force on this part is 2T sin . By
Newton’s second law, we have 2 T T

Dq Dmv 2
2T sin = Fig. 1.45
2 R
As Dq is small,
Dq Dq
\ sin 
2 2
The above equation reduces to
2
Ê Dq ˆ Êm ˆv
2T Á = ◊ Dq
Ë 2 ˜¯ ÁË
2p
˜¯
R
mv 2
or T = Ans.
2p R

DEATH WALL OR ROTOR w


Consider a rotor, a hollow vertical cylindrical device which rotates
about its axis and a person rests against the inner wall. As the person
R
is at rest w.r.t. rotor, so
N = mw2R ...(i)
and frictional force
f
f = mg ...(ii)
N
Minimum Speed of Rotation of Rotor: At the minimum speed, the
friction is limiting one, that is F = ms N. mg
or mg = ms (mw2R)
g
\ w = wmin =
us R Fig. 1.46
22പWŚLJƐŝĐƐĨŽƌ//dͲ:͗DĞĐŚĂŶŝĐƐ//

MOTION OF CYCLIST ON CIRCULAR ROAD


When a cyclist goes around a circular road, a centripetal N R
force is required. The frictional force between the tyres
and the ground provides the necessary centripetal force.
To get rotational equilibrium he leans from the vertical.
Let q be the angle, the contact force on tyres makes with
vertical. Then to prevent skidding, we have
q
man £ flim
flim
O
mv 2
or £ ms N
r mg
£ ms (mg) r

fi v £ m s rg

Again we have Fig. 1.47


mv 2
flim = ...(i)
r
and N = mg ...(ii)
Dividing Eq. (i) by Eq. (ii), we get
flim v2
=
N rg
flim
As = tan q
N
v2
tan q =
rg

BANKING OF ROADS AND RAILS


On circular path, if frictional force is not enough to provide the required centripetal force,
then outer edge of the road is raised with respect to inner edge of the road. This is called
banking of the road.
Consider a car of mass m going on a circular path of radius r with a speed v. The outer
edge of the road is raised by h with respect to the inner edge. Geometrically, banking of road
(also known as superelevation) is defined as
h
tan q =
b
h
for small q, tan q = ,
W
where W is the width of the road.
Now let tendency of car be to slip up (this corresponds to maximum speed), then friction
will act down the elevation.
ŝƌĐƵůĂƌDŽƟŽŶപ23

N
q N Vertical

O f W h
q Horizontal
r b q

mg
f = msN

mg

Fig. 1.48

Simplified FBD assuming car as a point object Fig. 1.48.


Along vertical direction, we have
SFv = 0
or N cos q = mg + ms N sin q
or N cos q – ms N sin q = mg ...(i)
And along horizontal direction by Newton’s second law

mv 2
N sin q + ms N cos q = ...(ii)
r
Dividing Eq. (ii) by Eq. (i), we have
sin q + m s cos q v2
=
cos q - m s sin q rg

Again dividing numerator and denominator each by cos q, we get

È m s + tan q ˘ 2
vmax
Í ˙ = ...(1)
Î 1 - m s tan q ˚ rg

If tendency of the car is to slip down the plane, it corresponds to minimum speed of the
car, then
È m s - tan q ˘ vmin
2

Í 1 + m tan q ˙ = ...(2)
Î s ˚ rg
So,
(i) If tendency of the car is to slip up, then

m s + tan q v2
= max fr
1 - m s tan q rg q

Fig. 1.49
24പWŚLJƐŝĐƐĨŽƌ//dͲ:͗DĞĐŚĂŶŝĐƐ//

(ii) If tendency of the car is to slip down, then fr

m s - tan q v2
= min
1 + m s tan q rg q
Fig. 1.50
(iii) When there is no friction between the tyres and the road, ms = 0, then we have

v2
tan q =
rg
(Bending of cyclist from vertical)

MOTION ON A PLANE CIRCULAR PATH


Consider a car whose centre of gravity is at a N1 N2
height h from the road and separation between
the wheels is 2a. Car is going with a constant
speed v on a circular path of radius r. mv 2
r
For the vertical equilibrium of the car h
N1 + N2 = mg ...(i) O
m sN 1 msN2
The maximum value of centripetal force that a a
can be available is
r
f = f1 + f2 = mstatic N1 + mstatic N2
mg
= mstatic (N1 + N2) = mstatic mg ...(ii)
Fig. 1.51
Skidding: To prevent skidding, we have
mv 2
£f
r
£ mstatic mg

or v £ mstatic rg

\ vmax = mstatic rg
Now taking moment of all the forces acting on the car’s centre of gravity (or any other
point) we have
mstatic (N1 + N2) ¥ h + N1 ¥ a – N2 ¥ a = 0 ...(iii)
v2
From Eq. (i), N2 = mg – N1 and from Eq. (ii) we have mstatic = . Substituting these values
in Eq. (iii) and after simplification, we get rg
mg Ê v hˆ
2
N1 = Á 1- ...(iv)
2 Ë rga ˜¯

mg Ê v2 h ˆ
and N2 = Á 1+ ...(v)
2 Ë rga ˜¯
ŝƌĐƵůĂƌDŽƟŽŶപ25

WHY THERE IS OVERTURNING?


With the increase in speed of the car normal reaction N2 increases
while N1 decreases. For a particular value of speed v = vmax, N1
becomes zero, and the car is then about to overturn. Thus, to
prevent overturning, N1 ≥ 0.
N1 = 0

mg Ê v hˆ
2 Overturning of
or Á 1- ≥0
rga ˜¯
the car
2 Ë

rga
or v £
h
Fig. 1.52
WORKED PROBLEMS
w
Problem 1.17 A uniform rod of length l and mass
m rotates uniformly with an angular velocity w in a
horizontal plane about a vertical axis passing through T T + dT
one of its ends. Determine the tension in the rod as a
function of the distance x from the rotation axis. x
dx
(IIT Roorkee, 1976)
Fig. 1.53
Solution Choose a small element of width dx at a
m
distance x from one end of the rod. The mass of the element dm = dx. Let T be the tension
l
in the rod at a distance x. By Newton’s second law for the motion of element of mass dm,
we have
T – (T + dT) = (dm)w2x
Êm ˆ
or – dT = Á dx˜ w 2 x
Ë l ¯
Integrating the above equation, we get
0 l
mw 2
l Úx
- Ú dT = xdx
T

mw 2 2 l
or -|T|T0 = |x |x
2l
mw 2 2
or –(0 – T) = (l - x 2 )
2l
mw 2 2
or T = (l - x 2 ) Ans.
2l
mw 2 l
Special case: At x = 0, T = Tmax =
2
At x = l/2 T =
mw l
8
26പWŚLJƐŝĐƐĨŽƌ//dͲ:͗DĞĐŚĂŶŝĐƐ//

Problem 1.18 A particle of mass m describes a horizontal circle on the smooth inner surface
of a conical funnel as shown in Fig. 1.54. If the height of the plane of the circle above the
vertex is 9.8 cm, find the speed of the particle.

9.8 cm

Fig. 1.54

Solution For circular motion of the particle


mv 2 N
N sin a =
r
r a
and N cos a = mg
h
v2 a
\ tan a = mg
rg

or v = (r tan a ) g
Fig. 1.55
= hg = 0.098 ¥ 9.8 = 0.98 m/s

Problem 1.19 A conical pendulum, a thin uniform rod of length l and mass m, rotates
uniformly about a vertical axis with angular velocity w (the upper end of the rod is hinged).
Find the angle q between rod and the vertical.
Solution Choose an element of the rod of width dx
at a distance x from the hinge. Mass of the element, w

m q
dm = dx. The centrifugal force on this element x
l
dF = (dm) w2 (x sin q). dmw2(x sin q)
Its moment of force about the hinge
l/2
dt = dF ¥ x cos q
= (dm) w2 (x sin q) (x cos q) mg

Fig. 1.56
ŝƌĐƵůĂƌDŽƟŽŶപ27

Ê sin 2q ˆ
= ÊÁ dxˆ˜ w 2 x 2 Á
m
Ë l ¯ Ë 2 ˜¯
mw 2
= sin 2q x2 dx ...(i)
2l
For the moment of force of whole length of rod, integrating (i)
l
mw 2
t = sin 2q Ú x 2 dx
2l 0

mw 2 l 2
= sin 2q ...(ii)
6
In the rotating frame, apart from other forces the centrifugal force also act. For rotational

equilibrium of the rod, we have  t = 0.
Taking moment of all the forces the about hinge and putting their algebraic sum zero,
we get
l mw l 2
mg sin q = sin 2q
2 6
3g
or cos q = Ans.
2w 2 l
DYNAMICS OF A CIRCULAR MOTION IN A VERTICAL CIRCLE
Suppose a body of mass m tied to the one end
H
of the string and made to rotate in a vertical
circle of radius r as shown in Fig. 1.57. Let u be
the velocity of the body at its lowest position.
Its velocity for any angular position q is v.
M
Height of the body, h = r – r cos q = r (1 – cos q). O T
q r
By the third equation of motion r cos q
T v
v2 = u2 = 2gh N
P
= u2 – 2gr (1 – cos q) h = r (1 – cos q) q
L mg cos q
or v = u2 - 2 gr (1 - cos q ) ...(i)
mg
Tension in the string;
by Newton’s second law Fig. 1.57
2
mv
T – mg cos q =
r
mv 2
\ T = mg cos q +
r
u2 - 2 gr (1 - cos q )
or T = mg cos q + m ...(ii)
r
28പWŚLJƐŝĐƐĨŽƌ//dͲ:͗DĞĐŚĂŶŝĐƐ//

From right angled DOPN,


ON r - h
cos q = =
OP r

(i) For q = 0° and 180°


At the highest position of the body, q = 180°.
Thus, the tension in the string at the highest
position of the body
È u2 - 2 gr (1 - cos 180∞) ˘ T=0
TH = mg cos 180° + m Í ˙
Î r ˚
È u2 - 4 gr ˘
or TH = – mg + m Í ˙
ÍÎ r ˙˚
O
To complete the circle TH ≥ 0
Fig. 1.58
È u2 - 4 gr ˘
or – mg + m Í ˙≥0
ÍÎ r ˙˚
\ u ≥ 5gr .
Thus, the minimum velocity required at lowest position of the body to complete the
circle is 5gr . The corresponding tension in the string; from Eq. (ii), Put q = 0° and u
= 5gr.
We get, TL = 6 mg
The velocity at the highest position of the body corresponds to u = 5gr ;

From Eq. (i), we get v = 5 gr - 2 gr (1 - cos 180∞)

or v = gr
The most critical position at which body can leave the circle is its highest position.
Thus, the critical velocity will be vc = gr .
v c > 2gr
(ii) For q = 90°:
From Eq. (i), we get v = 5 gr - 2 gr (1 - cos 90∞) TH = 0

= 3gr

m( 3 gr )2 T = 3 mg
and T = mg cos 90° + = 3 mg. v c = 3 gr
r v c = 3gr T = 3 mg
Critical Circle

v = u2 - 2 gr (1 - cos q ) and
TL = 6 mg
2
m [u - 2 gr (1 - cos q )]
T = mg cos q + u = 5 gr
r
Fig. 1.59
ŝƌĐƵůĂƌDŽƟŽŶപ29

Important:
It can be shown that, TL – TH = 6 mg.

Condition for Oscillations or to Leave the Circle


(i) For u £ 2gr H
Let us consider a body attached to a string of length r. It u > 2gr
is projected from its lowest position with a velocity M

u= 2gr . u = 2gr
O
The body will go to a height;
r u < 2gr
v2 = u2 – 2gh

or 0 = ( 2 gr )2 – 2gh
u
fi h = r. L

For u < 2gr , h < r. Fig. 1.60

Thus, for velocity u £ 2gr , the body cannot cross the position M and thereafter return
back towards initial position and due to inertia cross the mean position (L) and will
oscillate about L.
(ii) For 2 gr < u < 5 gr
As the velocity u > 2gr , the body will cross the position M but cannot go up to H,
because (u < 5gr ). So the body will leave the circle somewhere between M and H
and will follow parabolic path.

For Your Knowledge


s &OR A BODY LOOPING THE LOOP THE FOLLOWING CONDITIONS HOLD GOOD
vL ≥ gr , vM ≥ 3gr , vH ≥ gr

and TL ≥ 6mg , TM ≥ 3mg , TH ≥ 0.


s %VEN IF A BODY IS PROJECTED WITH A MINIMUM VELOCITY OF 5gr from the lowest point,
its velocity at the highest point will be gr and not zero.
s 4HE BODY WILL OSCILLATE ABOUT THE LOWEST POINT OF THE VERTICAL CIRCLE IF v < 2gr .
s &OR THE BODY TO LEAVE THE CIRCLE 2 gr < vL < 5 gr .
s TL – TH = 6 ¥ Weight of the body.

FORMULAE USED IN DYNAMICS OF A CIRCULAR IN A VERTICAL CIRCLE


1. Velocity of the body at any point at a height h from the lowest point,

v= u2 - 2 gh
30പWŚLJƐŝĐƐĨŽƌ//dͲ:͗DĞĐŚĂŶŝĐƐ//

2. Tension in the string at any point,


m 2
T= (u – 3gh + gr)
r
3. Tension at the lowest point,
m 2
TL = (u + gr)
r
4. Tension at the highest point,
m 2
TH = (u – 5gr)
r
5. Difference in tensions at the highest and lowest points,
TL – TH = 6 mg
6. Minimum velocity at the lowest point for looping the vertical loop,
vL = 5gr
7. Velocity at the highest point for looping the loop,
vH = gr

Units Used
Radius r and height h are in metre, velocities u and v are in ms–1, tensions T, TL and TH are
in newton.

WORKED PROBLEMS
Problem 1.20 In a rotor, a hollow vertical cylindrical structure rotates ........................ .
.................................................................................................................
..................................... ..
about its axis and a person rests against the inner wall. At a particular
speed of the rotor, the floor below the person is removed and the person
fs
hangs resting against the wall without any floor. If the radius of the
rotor is 2 m and the coefficient of static friction between the wall and N
the person is 0.2, find the minimum speed at which the floor may be
removed. Take g = 10 m/s2.
mg
Solution The situation is shown in Fig. 1.61.
When the floor is removed, the forces on the person are:
Fig. 1.61
(a) wight mg downward,
(b) normal force N due to the wall, towards the centre, and
(c) friction force fs, parallel to the wall, upwards.
The person is moving in a circle with a uniform speed, so its acceleration is v2/r towards
the centre.
Newton’s law for the horizontal direction (2nd law) and for the vertical direction (1st
law) give
N = mv2/r ...(i)
and fs = mg ...(ii)
ŝƌĐƵůĂƌDŽƟŽŶപ31

For the minimum speed when the floor may be removed, the friction is limiting one and
so equals ms N. This gives
ms N = mg
m s mv 2
or = mg [using Eq. (i)]
r
rg 2m ¥ 10 m/s 2
or v = = = 10 m/s.
ms 0.2

Problem 1.21 Two blocks each of mass M are connected to


the ends of a light frame as shown in Fig. 1.62. The frame is M
M
rotated about the vertical line of symmetry. The rod breaks
if the tension in it exceeds T0. Find the maximum frequency
with which the frame may be rotated without breaking the
rod.
Solution Consider one of the blocks. If the frequency
of revolution is f, the angular velocity is w = 2p f. The
l
acceleration towards the centre is v2/l = w2 l = 4p2f 2 l. The
only horizontal force on the block is the tension of the rod. Fig. 1.62
At the point of breaking, this force is T0. So from Newton’s
second law,
T0 = M ◊ 4p 2 f 2 l
1/2
1 È T0 ˘
or, f = ÍÎ Ml ˙˚
2p

Problem 1.22 Figure 1.63 shows a rod of length 20 cm pivoted w


near an end and is made to rotate in a horizontal plane with L
a constant angular speed. A ball of mass m is suspended by
a string also of length 20 cm from the other end of the rod. q L
If the angle q made by the string with the vertical is 30°, find
the angular speed of the rotation. Take g = 10 m/s2.
Solution Let the angular speed be w. As is clear from the
Fig. 1.63, the ball moves in a horizontal circle of radius L + L m
sin q where L = 20 cm. Its acceleration is, therefore, w2 (L + L
Fig. 1.63
sin q) towards the centre. The forces on the bob are (Fig. 1.63).
(a) the tension T along the string, and
(b) the weight mg.
Resolving the forces along the radius and applying Newton’s second law,
T sin q = mw2L (1 + sin q) ...(i)
Applying Newton’s first law in the vertical direction,
T cos q = mg ...(ii)
32പWŚLJƐŝĐƐĨŽƌ//dͲ:͗DĞĐŚĂŶŝĐƐ//

Dividing Eq. (i) by Eq. (ii),


w 2 L (1 + sin q )
tan q =
g
g tan q (10 m/s 2 )(1/ 3 )
or w2 = =
L (1 + sin q ) (0.20 m)(1 + 1/2)
or w = 4.4 rad/s.
Problem 1.23 A bucket containing water is tied to one end of a rope of length 2.5 m and
rotated about the other end in a vertical circle in such a way that the water in it does not
spill. What is the minimum velocity of the bucket at which this happens and how many
rotations per minute is it making then? Take g = 10 cm–2. (BITSAT, 2007)
Solution Water in the bucket will not spill if the centripetal force is equal to the weight
of water.
mv 2
= mg or v = rg
r
But r = 2.5 m, g = 10 ms–2

\ v = 2.5 ¥ 10 = 5 ms–1.

v 5
Angular speed, w = = = 2 rad s–1
r 2.5
w 2 60
Frequency of rotation, v = = rps = rpm. Ans.
2p 2p p

Problem 1.24 A stone of mass 0.3 g is tied to one end of a string 0.8 m long and rotated
in a vertical circle. At what speed of the ball will the tension in the string be zero at the
highest point of the circle? What would be the tension at the lowest point in this case? Given
g = 9.8 ms–2. (WBJEE, 2010)
Solution When tension is zero at the highest point, speed is minimum. It is given by

vH = gr = 9.8 ¥ 0.8 = 2.8 ms–1


As TL – TH = 6 mg
\ TL = 6 ¥ 0.3 ¥ 9.8 – 0 = 17.64 N.
Problem 1.25 In a circus, the diameter of the globe of death is 20 m. From what minimum
height must a cyclist start in order to go around the globe successfully?
Solution When the cyclist rolls down the incline,
Loss in PE = Gain in KE
1
mgh = mv2
2
\ Velocity gained, v= 2gh
For looping the loop, the minimum velocity at the lowest point should be 5gr .
ŝƌĐƵůĂƌDŽƟŽŶപ33

\ 2gh = 5gr

5 5
or h = r = ¥ 10 = 25 m.
2 2 [∵ Diameter = 20 m]
Problem 1.26 A weightless thread can bear tension upto 3.7 kg wt. A stone of mass 500 g
is tied to it and revolves in a circular path of radius 4 m in a vertical plane. If g = 10 cm–2
then what will be the maximum angular velocity of the stone?
Solution Here Tmax = 3.7 kg wt = 3.7 ¥ 10 = 37 N,
m = 500 g = 0.5 kg, r = 4 m
mv 2
As Tmax = + mg
r
mv 2
\ = Tmax – mg = 37 – 0.5 ¥ 10 = 32
r
32 ¥ r 32 ¥ 4
or v2 = = = 256 or v = 16 ms–1
m 0.5
v 16
Hence, w = = = 4 rads–1
r 4

Problem 1.27 The speed given at the bottom in a circular motion is 3gl. Find speed at an
angle 60°. Also find speed when the ball is at horizontal level. (OJEE, 2013)

3gl

Fig. 1.64

Solution To find speed we should use


v2 = u2 + 2ah
Here, a = – g; u = 3gl
h is the vertical displacement l cos q 60° l
Ê 1ˆ 1
h = l(1 – cos q) = l(1 – cos 60°) = l Á 1 - ˜ = .
Ë 2¯ 2 h = l – l cos q
h
Ê ˆ
1 h = l (1 – cos q)
\ v2 = 3gl – 2g Á ˜ fi v = 2gl
Ë 2¯
Fig. 1.65
34പWŚLJƐŝĐƐĨŽƌ//dͲ:͗DĞĐŚĂŶŝĐƐ//

Speed when the ball is at horizontal level:


v2 = 3gl – 2gl (the vertical displacement is l).
\ v = gl .

Problem 1.28 When should be the speed at bottom such that a ball doing circular motion
is able to reach the horizontal level?
Solution Let the speed be v0
v2 = u2 + 2as
The speed at the horizontal level will become zero.

0 = v02 – 2gl fi v0 = 2gl .

Problem 1.29 The speed given at bottom is 6gl . Find speed and tension at an angle 120°.
Also find speed and tension at top.

6gl

Fig. 1.66
Solution v2 = u2 + 2ah
3l
At 120° h = l + l sin 30° = (h is the vertical displacement).
2
30°
Ê 3l ˆ
v2 = 6gl – 2g Á ˜ \ v= 3gl h
Ë 2¯ l
Tension at this position
Components of mg force are taken. 6gR
2
mv
T + mg cos 60° = Fig. 1.67
l
mg 3 mgl
T+ =
2 l
mg 5 mg
T = 3 mg – =
2 2
Speed and tension at top:
v2 = u2 + 2ah or v2 = 6gl – 2g(2l)
\ v= 2gl
ŝƌĐƵůĂƌDŽƟŽŶപ35

For tension:
mv 2
mg + T = fi mg + T = 2 mg
l
\ T = mg
Problem 1.30 A small body of mass m slides along a P
frictionless loop. If the body starts from rest at P, what
is the resultant force on it at Q? The height of P with
respect to the loop bottom is 5R. At what height above
h
the bottom of the loop, should the block be released
so that the force it exerts against the track at the top R
of the loop is equal to its weight? Q

Solution The speed of the body at Q


Fig. 1.68
v = 2 g ( 5 R - R)
P
= 8gR .
The normal force on body at Q

mv 2 m ¥ 8 gR N v
Fn = = = 8 mg
R R mg Q
O N
and Ft = mg.

The resultant force F = Fn2 + Ft2 = 65 mg Fig. 1.69


If v¢ is the speed of the body at the top, then
mv ¢ 2
N + mg =
R
Given, N = mg
mv ¢ 2
\ mg + mg =
R

fi v¢ = 2gR
Now from conservation of mechanical energy, we have
1
mgh = mg(2R) + mv¢ 2
2
1
= mg(2R) + m ¥ 2 gR
2
\ h = 3R Ans.
Problem 1.31 A ball suspended by string swing in a vertical plane so that its acceleration
values in the extreme and the lowest position are equal. Find the string deflection angle in
the extreme position.
36പWŚLJƐŝĐƐĨŽƌ//dͲ:͗DĞĐŚĂŶŝĐƐ//

Solution Suppose q is the required angle. At extreme position the velocity of the ball is
v2
zero, thus normal acceleration an = = 0, and tangential acceleration at = g sin q. At mean
l
position the velocity of the ball

v= 2g (l - l cos q ) . q
v2
The normal acceleration at this position an = l cos q l
l
2g (l - l cos q )
= = 2g(1 – cos q)
l g sin q v=0
and tangential acceleration q
h = l – l cos q
at = g sin q = 0. g
Thus, total acceleration at mean position
Fig. 1.70
a = 2g(1 – cos q)
According to the given condition, we have
g sin q = 2g(1 – cos q)
After solving, we get q = 53°. Ans.
Problem 1.32 A simple pendulum is oscillating with angular displacement 90°. For what
angle with the vertical the acceleration of the bob directed horizontally?
v2
Solution At q = 90°, v = 0, an = = 0 at any angular position q, h = l cos q. The velocity
r
of the body at angular position q is
v2 = 0 + 2g(l cos q) ...(i) q
l
l cos q
The normal acceleration of the bob, an

v2 a q
an =
l q
at
g
2gl cos q
= = 2g cos q
l Fig. 1.71

and tangential acceleration of the bob, at = g sin q.


As the resultant acceleration direction horizontally, so
an
tan q =
at

sin q 2g cos q
or =
cos q g sin q

or 2 cos2 q = sin2 q = (1 – cos2 q)


1 Ê 1 ˆ
fi cos2 q = and q = cos–1 Á Ans.
3 Ë 3 ˜¯
ŝƌĐƵůĂƌDŽƟŽŶപ37

A v0 B
Problem 1.33 A small block slides with velocity 0.5 gr on the
C
horizontal frictionless surface as shown in the Fig. 1.72. The block q
leaves the surface at point C. Calculate angle q in Fig. 1.72. r

O
Solution v2 = v02 + 2gh = (0.5 gr )2 + 2gr(1 – cos q)
Fig. 1.72
= ( 2.25 gr – 2gr cos q) h = r – r cos q
At the time of leaving contact, N = 0 = r(1 – cos q)
h

mv 2
\ mg cos q = = 2.25 mg – 2mg cos q v
r q
r cos q
r mg
2.25 3 q
\ cos q = =
3 4

\ q = cos–1 (3/4) Ans.


Fig. 1.73
Problem 1.34 A particle is suspended from a fixed point by v
a string of length 5 m. It is projected from the equilibrium q
position with such a velocity that the string slackens after the P
particle has reached a height 8 m above the lowest point. Find 5m q
the velocity of the particle, just before the string slackens. Find 3m
also, to what height the particle can rise further?
4m
(EAMCET, 2011) 8m
5m mg
Solution At P,
T =0
mv 2 v2
\ mg cos q = or g cos q = Fig. 1.74
R R
Ê 3ˆ v2
or (9.8) Á ˜ =
Ë 5¯ 5
\ v = 5.42 m/s Ans.
Problem 1.35 The sphere at A is given a downward velocity v0 of magnitude 5 m/s and
swings in a vertical plane at the end of a rope of length l = 2 m attached to a support at
O. Determine the angle q at which the rope will break, knowing that it can withstand a
maximum tension equal to twice the weight of the sphere.
B

A l
O
q

v0

Fig. 1.75
38പWŚLJƐŝĐƐĨŽƌ//dͲ:͗DĞĐŚĂŶŝĐƐ//

Solution v2 = v02 + 2 gh = v02 + 2 gR sin q q


= (5)2 + 2 ¥ 10 ¥ 2 sin q
h
= (25 + 40 sin q) 2 mg

mv 2 v
Now, 2 mg – mg sin q = 90° – q
R
v2
or 2g – g sin q =
R mg

25 + 40 sinq Fig. 1.76


or 2 ¥ 10 – 10 sin q =
2
Solving this equation, we get
Ê 1ˆ
q = sin–1 Á ˜ Ans.
Ë 4¯

Problem 1.36 The bob of the pendulum shown in Fig. 1.77 describes an arc of circle in a
vertical plane. If the tension in the cord is 2.5 times the weight of the bob for the position
shown. Find the velocity and the acceleration of the bob in that position.

2.0 m
30°

Fig. 1.77

mv 2
Solution 2.5 mg – mg cos 30° =
r
q
v2 v2
\ 1.63 g = =
r 2 T = 2.5 mg
\ v = 5.66 m/s v
2 2
Fnet = (2.5 mg ) + (mg ) + (2)(2.5 mg )(mg )cos 150∞ 150°
30°

= 1.7 mg
Fnet mg
anet = = 1.7 g ª 16.75 m/s2 Ans.
m Fig. 1.78
ŝƌĐƵůĂƌDŽƟŽŶപ39

Problem 1.37 A track consists of two circular parts ABC and CDE of equal radius 100 m and
joined smoothly as shown in Fig. 1.79. Each part subtends a right angle at its centre. A cycle
weighing 100 kg together with the rider travels at a constant speed of 18 km/h on the track.
(a) Find the normal contact force by the road on the cycle when it is at B and at D.
(b) Find the force of friction exerted by the track on the tyres when the cycle is at B, C
and D.
(c) Find the normal force between the road and the cycle just before and just after the
cycle crosses C.
(d) What should be the minimum friction coefficient between the road and the tyre, which
will ensure that the cyclist can move with constant speed? Take g = 10 m/s2.

B
A C E
D

Fig. 1.79

Solution
(a) Force diagrams at B and D are shown in Fig. 1.80.

Fcf
n

n Fcf mg
mg

at B at D

Fig. 1.80
cf = centrifugal force
For B: 2
(5)
n = mg – Fcentrifugal force = 100 ¥ 10 – 100 ¥ = 975 N
100
mv 2
[∵ Fcf = ; v = 18 kmph = 5 ms–1]
R
For D:
(5)2
n = mg + Fcf = 100 ¥ 10 + 100 ¥ = 1025 N
100
f
(b) At B and D, no friction acts. Because there is no additional
tangential force acting on the bicycle. At C, a component of
mg acts along the track. This component is counterbalanced by
friction.
mg sin 45°
Thus, mg
f = mg sin 45 = 100 ¥ 10 ¥ (0.7071) = 707 N Fig. 1.81
40പWŚLJƐŝĐƐĨŽƌ//dͲ:͗DĞĐŚĂŶŝĐƐ//

(c) Force on bicycle just before C is shown in Fig. 1.82.


n + Fcf = mg cos q
Fcf
n = mg cos q – Fcf
2
100 ¥ 5 n
= 100 ¥ 10 ¥ cos 45 –
100 C
= 707 – 25 = 682 N mg cos 45°
Similarly, it can be shown
Fig. 1.82
nD = mg cos q + Fcf = 732 N
(d) The minimum frictional force as the tendency of the cyclist to skid is max just before
point C where radius of curvature charge its direction.
mnmin = mg sin 45°
= mnmin = m (682)
This force must balance the tangential component of force.
m(682) = 702 or m = 1.037
Problem 1.38 A person stands on a spring balance at the equator.
(a) By what fraction is the balance reading less than his true weight?
(b) If the speed of earth’s rotation is increased by such an amount that the balance reading
is half the true weight, what will be the length of the day in this case?
Solution wapp = mgeff = mg – mw2R
\ Dw = mw2R
Dw w 2 R
Friction required = =
w g
For earth,
2p
w= ; R = 6400 ¥ 103 m; g = 9.81
86400
Substituting values,
Dw
= 3.5 ¥ 10–3
w
For part (b) of the question,
Dw 1 w 2R g
= = or w =
w 2 g 2R

2p g 2R
or = or T = 2p g = 2 hr.
T 2R

Problem 1.39 A motorcycle has to move with a constant speed on an overbridge which
is in the form of a circular arc of radius R and has a total length L. Suppose the motorcycle
starts from the highest point.
(a) What can its maximum velocity be for which the constant with the road is not broken
at the highest point?
ŝƌĐƵůĂƌDŽƟŽŶപ41

1
(b) If the motorcycle goes at speed times the maximum found in part (a), where will
2
it lose the contact with the road?
(c) What maximum uniform speed can it maintain on the bridge if it does not lose contact
anywhere on the bridge?
Fcf
Solution
mg sin a
(a) To remain in contact at highest point, vmax = Rg
A mg cos a B
a
vmax Rg
(b) v = = a mg
2 2 q q R
Let contact is broken at angle a . Then,
mv 2 m Rg
mg cos a = Fcentrifugal force = or mg cos a = ◊ Fig. 1.83
R R 2
1 p
or cos a = or a = rad
2 3
(c) To not lose contact at end point
mv 2 L mv 2 L
mg cos q = or mg cos = or v= Rg cos
R 2R R 2R

Problem 1.40 Figure 1.84 shows a light rod of length l rigidly attached to a small heavy
block at one end and a hook at the other end. The system is released from rest with the rod
in a horizontal position. There is a fixed smooth ring at a depth h below the initial position
of the hook and the hook gets into the ring as it reaches there. What should be the minimum
value of h so that the block moves in a complete circle about the ring?

Fig. 1.84

Solution The velocity gained by the block after falling from height h

v= 2gh
Velocity required to complete vertical circular motion

v= 2gR
So, 2gh = 2gR or h=R=l
42പWŚLJƐŝĐƐĨŽƌ//dͲ:͗DĞĐŚĂŶŝĐƐ//

Problem 1.41 The bob of a pendulum at rest is given a sharp hit to impart a horizontal
velocity 10gl where l is the length of the pendulum. Find the tension in the string when:
(a) the string is horizontal,
(b) the bob is at its highest point, and
(c) the string makes an angle of 60° with the upward vertical.
Solution Tension at the lowest point
Ê v2 ˆ Ê 10 gl ˆ
T = m Á 0 + g˜ = m Á + g ˜ = 11 mg
ËR ¯ Ë l ¯
The velocity at horizontal point is v
mv 2
v2 = v02 – 2 Rg = 10 gl – 2 gl = 8 gl \ T= = 8 mg
l
The velocity at the topmost point is v¢, where
v¢2 = v02 – 4gl = 10 gl – 4 gl = 6 gl
Ê v¢ 2 ˆ
So, T = mÁ - g ˜ = m(6 g - g ) = 5 mg
Ë R ¯
The velocity when string makes an angle q with vertical v≤, where
v≤ = v02 – 2gl (1 + cos q)
q
Ê 1ˆ
= 10 gl – 2gl Á 1 + ˜
Ë 2¯
= 10gl – 3gl = 7gl
Tension at angle q Fig. 1.85
Ê v ¢¢ 2 ˆ Ê 7 gl 1ˆ
T = mÁ - g cos q ˜ = m Á - g ◊ ˜ = 6.5 mg
Ë R ¯ Ë l 2¯

Problem 1.42 A block of mass m moves on a horizontal circle against the wall of a cylindrical
room of radius R. The floor of the room on which the block moves is smooth but the friction
coefficient between the wall and the block is m. The block is given an initial speed v0. As a
Function of the speed v write:
(a) the normal force by the wall on the block,
(b) the frictional force by the wall, and
(c) the tangential acceleration of the block.
Ê dv dv ˆ
(d) Integrate the tangential acceleration Á = v ˜ to obtain the speed of the block after
Ë dt ds ¯
one revolution.
Solution
mv 2
(a) n = normal reaction = Fc =
r
mmv 2
(b) f = mn =
r
ŝƌĐƵůĂƌDŽƟŽŶപ43

f mv2
(c) at = =-
m r
dv mv 2 v dv 2p r m ds
(d) Now, v
ds
= -
r
or Úv
0 v
=Ú -
0 r
v m
or ln = – 2pr = – 2p m or v = v0 e–2pm
v0 r

Problem 1.43 A nail is located certain distance vertically below the point of suspension
of a simple pendulum. The pendulum bob is released from the position where the string
makes the angle 60° from the vertical. Calculate the distance of the nail from the point of
suspension such that the bob performs revolutions with the nail as the centre. Assume the
length of the pendulum is 1 m.
Solution The velocity of the bob at its lowest position;
by the third equation of motion 60°
l l=1m
v2 = 0 + 2 g(l – l cos 60°) = 2g ¥
2 d
or v = gl = g ¥ 1 ...(i)

= g. Nail
l – l cos q
Let d be the distance of the nail from the point of
suspension. The bob will have to complete the circle of
radius r = 1 – d. Fig. 1.86
For the bob to just perform the revolutions about the nail, the minimum speed at the
lowest position must be
5gr = 5 g(1 - d) ...(ii)
Equating Eqs. (i) and (ii), we get

g = 5 g(1 - d)
4
After solving, d = = 0.80 m Ans.
5
Problem 1.44 A particle of mass m slides from the top of the surface of a sphere of radius
R. It loses contact and strikes the ground. At what depth below the top the particle will lose
contact with the surface? At what horizontal distance from
N
the initial position the body strikes the ground? h P

Solution Let the particle lose the contact at point P. At q v


point P normal reaction by the contact surface becomes mg
zero. If v is the velocity of the particle at P, then we have
v2 = 0 + 2gh

or v = 2gh ...(i) R sin q


x

Fig. 1.87
44പWŚLJƐŝĐƐĨŽƌ//dͲ:͗DĞĐŚĂŶŝĐƐ//

By Newton’s second law, we have


mv 2
mg cos q – N =
R
At P, N = 0,
mv 2 m(2 gh)
\ mg cos q = =
R R
2h
fi cos q = ...(ii)
R
R-h
But from the geometry, cos q =
R
2h R-h R
Therefore, = or h =
R R 3

R-h 2
and cos q = =
h 3

5
and sin q =
3
2 gR
Velocity at point P becomes v = 2 gh =
3
Vertical component of velocity at P = v sin q and horizontal component of velocity is v
cos q.
R 5R
Height of P from ground = 2R – h = 2R – = .
3 3
If t is the time taken by the particle to reach the ground from P, then by second equation
of motion
5R 1
= v sin qt + gt2 ...(iii)
3 2
and x = v cos q t ...(iv)
Solving Eqs. (iii) and (iv), we get
x = 0.71 R
Thus, horizontal distance from A at which the particle strikes is
5
= R sin q + x = R ¥ + 0.71R = 1.5R Ans.
3

Problem 1.45 A small heavy block is attached to the lower end of a light rod of length l
which can be rotated about its clamped end. What minimum horizontal velocity should the
block be given so that it moves in a complete circle? (BITSAT, 2006)
ŝƌĐƵůĂƌDŽƟŽŶപ45

Solution As rod is a rigid member, so it can take compression H


(T < 0). The velocity of the block at its highest position can be zero to
just cross this position. 
Let the block be given a velocity v at its lowest position, then by
third equation of motion, we have
vH2 = vL2 – 2gh 
or 0 = v – 2g ¥ 2l
2

L
fi v = 2 g Ans.
Fig. 1.88
Problem 1.46 A round cone A of mass m = 3.2 kg and half angle a = 10° rolls uniformly
without slipping along a round conical surface B so that its apex O remains stationary. The
centre of gravity of the cone A is at the same level as the point O and at a distance 1 = 17 cm
from it. The cone’s axis moves with angular velocity w. Find:
(a) the static frictional force acting on the cone A, if w = 1.0 rad/s; and
(b) At what values of w the cone A will roll without sliding, if the coefficient of friction
between the surface is equal m = 0.25?
Solution
(a) For the equilibrium of the cone in the vertical N
direction
a
N cos a + fr sin a = mg ...(i) O a A
l
The cone rolls without slipping, and its centre
of mass O describes a circle or radius l. One can fr
write
mg
fr cos a – N sin a = mw2 l ...(ii)
Solving above equations, we get
fr = (mg sin a + mw2l cos a) ...(iii)
(b) For pure rolling, the rolling friction is less than Fig. 1.89
or equal to the limiting friction
fr £ mN
Solving Eqs. (i) and (iii), we get
N = mg sin a – mw2l sin a.
Substituting this value in fr £ mN, we get

g ( m - tan a )
w£ Ans.
 (1 + m tan a )

Problem 1.47 A small body of mass m tied to a non-stretchable thread moves over a smooth
horizontal plane. The other end of the thread is being drawn into a hole O shown in Fig. 1.90
with a constant velocity. Find the thread tension as a function of the distance r between the
body and the hole if at r = r0 the angular velocity of the thread is equal to w0.
46പWŚLJƐŝĐƐĨŽƌ//dͲ:͗DĞĐŚĂŶŝĐƐ//

Solution The thread tension T is equal to the centrifugal force mw2r. Since the net torque
due to all the forces acting on the body of mass m is zero, and therefore the angular
momentum must be constant.

O
r T mw2r

Fig. 1.90

\ mw0 r02 = mw r2
or w = mw 0 r2/mr2 = w0 r02/r2
Tension in the string will be equal to the centrifugal force
\ T = mw2r = m(w0r2/r2)2r
or T = mw02 r04/r3 Ans.
Problem 1.48 A tube of length L is filled completely with an
incompressible liquid of mass M and closed at both ends. The tube
is then rotated in a horizontal plane about one of its ends with a L
uniform angular velocity w. Find the force exerted by the liquid at O
the other end. (IIT-JEE, 1992)
Solution The mass of the element of width dx,
m Fig. 1.91
dm = dx
L
By Newton’s second law, we can write
(F + dF) – F = (dm) w2x
Êm ˆ
or dF = Á dx˜ w 2 x F
ËL ¯ F + dF
x
L dx
mw 2
or Ú (dF) = L Ú0
xdx
Fig. 1.92
mw 2 L
F = Ans.
2
Problem 1.49 A small particle of mass m initially at A (Fig. 1.93), slides down a frictionless
surface AEB. When the particle is at point C, show that the angular velocity and the force
Ê 2g sina ˆ
exerted by the surface are w = ÁË ˜ and F = 3 mg sin a.
R ¯
ŝƌĐƵůĂƌDŽƟŽŶപ47

A B
a

C
E

Fig. 1.93

Solution The speed of the particle at C, A


a
v = 2gR sin a R sin a N
v a v
\ w == 2 g sin a/R C
R
mg
For the motion of the particle, we have
mv 2 Fig. 1.94
N – mg sin a =
R
mv 2
\ N = mg sin a +
R
m (2 gR sin a )
= mg sin a +
R
= 3 mg sin a Ans.
Problem 1.50 A device (Fig. 1.95) consists of a smooth L-shaped rod located in a horizontal
plane and a sleeve A of mass m attached by a weightless spring to a point B. The spring
stiffness is equal to k. The whole system rotates with a constant angular velocity w about
a vertical axis passing through the point O. Find the elongation of the spring. How is the
result affected by the rotation direction?

O
B B

w R F
r q

A w N A

mg
(a) (b)

Fig. 1.95

Solution Different forces are shown in Fig. 1.95(b). Resolving these forces along tangent
and normal directions, we have
N sin q + F cos q = mw2 r ...(1)
N cos q = F sin q or N = F sin q/cos q ...(2)
Further r cos q = (l + Dl),
48പWŚLJƐŝĐƐĨŽƌ//dͲ:͗DĞĐŚĂŶŝĐƐ//

where l = initial length.


or r = (l + Dl)/cos q ...(3)
Substituting the values of N and r from Eqs. (2) and (3) in Eq. (1), we get
F sin q
¥ sin q + F cos q = m w2 (l + Dl)/cos q
cos q
F sin2 q + F cos q = mw2 (l + Dl)
or F = m w2 (l + Dl)
Now, kDl = mw2 l + mw2 Dl
or Dl (k – mw2) = mw2 l
mw 2l
or Dl =
k - mw 2
È l ˘
or Dl = Í 2 ˙
ÍÎ (k/m w ) - 1 ˙˚
There will be no effect on changing the direction.
A A
Problem 1.51 A weightless rod of length l carries
first a mass 2 m at its end and then two equal masses
m, one secured at the end and other in the middle of
the rod (Fig. 1.96). The rod can revolve in a vertical
l B
plane around the point A. What horizontal velocity m
must be imparted to the end of the rod C in the
first and second cases to deflect it to the horizontal
position?
C
Solution For the first case, by conservation of me- C
2m m
chanical energy, we have
1 (a) (b)
(2m)v12 = (2m)gl
2 Fig. 1.96
\ v1 = 2gl
In the second case, If v2 is the velocity given to mass at C, then velocity of mass at B will
v
be 2 . Thus, by conservation of mechanical energy, we have
2
2
1 1 Êv ˆ l
mv22 + m Á 2 ˜ = mgl + mg
2 2 Ë 2¯ 2
\ v2 =
12 Ans.
gl
5
m w
Problem 1.52 A block of mass m is sitting on a rotating
P
frictionless wedge. The wedge rotates with constant angular
velocity w around the axis shown in Fig. 1.97. h

Calculate the value of w such that the block stays at constant q


height h (express your answer in terms of g, h, q). Fig. 1.97
ŝƌĐƵůĂƌDŽƟŽŶപ49

Solution The velocity of point P on the wedge. Velocity of wedge at point P = wr = w ¥


h
.
tan q
Drawing a free-body diagram for the block.
N

w
r
P
h mg
q

Fig. 1.98

Resolving forces as indicated in step 2.4


h h
= tan q fi =r
r tan q
N cos q – mg = 0 (y-direction)
N sin q = mw2r
w 2r w 2h g tan 2 q g
tan q = = fi w2 = fi w= tan q
g g tan q h h
Problem 1.53 What is the speed required to negotiate the turn shown in Fig. 1.99. Frictionless
and radius of curvature R and banking q. (BCECE, 2003)
Solution
N cos q
N
e O
sid q v
ut
v

O N sin q
C
R
e
sid f
In
mg q
Inside

Fig. 1.99

N cos q – mg = 0 N cos q = mg
mv 2 mv 2
N sin q = mg tan q =
R R

v = Rg tan q .
50പWŚLJƐŝĐƐĨŽƌ//dͲ:͗DĞĐŚĂŶŝĐƐ//

Problem 1.54 A simple pendulum is constructed by attaching a bob of mass m to a string


of length L fixed at its upper end. The bob oscillates in a vertical circle. It is found that the
speed of the bob is v when the string makes an angle a with the vertical. Find the tension
in the string and the magnitude of net force on the bob at the instant.
Solution
(i) The forces acting on the bob are: d

(a) the tension T, and T


(b) the weight mg.
As the bob moves in a circle of radius L with centre at O. d
mg sin d
2 mg mg cos d
mv
A centripetal force of magnitude is required towards O.
L Fig. 1.100
This force will be provided by the resultant of T and mg cos a.
Thus,

mv 2 Ê v2 ˆ
or T – mg cos a = T = m Á g cosa + ˜
L Ë L¯
2
 2 Ê mv ˆ
2
v4
(ii) |Fnet| = (mg sin a ) + Á ˜ = m g 2 sin 2 a + 2
Ë L ¯ L

Problem 1.55 A wheel rotates around a stationary axis so that the rotation angle f = at2,
where a = 0.20 rad/s2. Find the total acceleration a of the point A at the rim at the moment
t = 2.5 s if the linear velocity of the point A at this moment v = 0.65 m/s.
Solution Given: f = at2
df d 2f
Angular velocity, w = = 2 at ; a = 2 = 2 a
dt dt
We know that v = wr
or v = (2at)r
v 0.65
\ r = = = 0.65 m
2 at 2 ¥ 0.2 ¥ 2.5
Tangential acceleration at = ar
= 2ar = 2 ¥ 0.20 ¥ 0.65
= 0.26 m/s2
v2
Normal acceleration an =
r
0.652
= = 0.65 m/s2
0.65

Total acceleration a = at2 + an2 = 0.7 m/s2


ŝƌĐƵůĂƌDŽƟŽŶപ51

Problem 1.56 The 4 kg block (Fig. 1.101) is attached to the vertical A


rod by means of two strings. When the system rotates about the
axis of the rod, the two strings are extended as indicated in Fig. 1.25 m
P
1.101. How many revolutions per minute must the system make 2m O 4 kg
in order that the tension in the upper string is 60 N?
What is tension in the lowest string? 1.25 m

Solution The forces acting on block P of mass 4 kg are shown B


in Fig. 1.102. If q is the angle made by strings with the vertical, Fig. 1.101
T1 and T2 tension in strings for equilibrium in the vertical direction
T1 cos q = T2 cos q + mg A
(T1 – T2) cos q = mg q T1 cos q

1 4È OA 1 ˘
cos q = = ∵ cos q = = O
1.25 5 ÍÎ AP 1.25 ˙˚
P
q
T2 cos q
mg 5 mg 5 mg
\ T1 – T2 = = = ¥ 4 ¥ 9.8 = 49 N
cos q 4 4
Fig. 1.102
Given, T1 = 60 N
T2 = T1 – 49 = 60 N – 49 N = 11 N
The net horizontal force (T1 sin q + T2 sin q) provides the necessary centripetal force m w2 r.
(T1 + T2 )sin q
\ (T1 + T2) sin q = m w2r fi w2 =
mr
3
sin q = 1 - cos 2 q = 1 - ( 4/5)2 =
5
r = OP = 1.252 - 12 = 0.75
3
(60 + 11)
\ w2 = 5 = 14.2 w= 14.2 = 3.768 rad/s
4 ¥ 0.75
w 3.768
Frequency revolution = = = 0.6 rev/s or 36 rev/min
2p 2 ¥ 3.14

Problem 1.57 A particle is moving along a vertical circle of radius r = 20 m with a constant
vertical circle of radius r = 20 m with a constant speed v = 31.4 m/s as shown in Fig. 1.103.
Straight line ABC is horizontal and passes through the centre of the circle. A shell is fired
from point A at the instant when the particle is at C. If distance AB is 20 3 m and the shell
(2n - 1)2
collides with the particle at B. Then prove tan q = . When n is an integer. Further,
show that smallest value of q is 30°. 3

Solution As at the time of firing of the shell, the particle was at C and the shell collides with
it at B, therefore the number of the revolutions completed by the particle is odd multiple of
half, i.e., (2n – 1)/2, where n is an integer.
52പWŚLJƐŝĐƐĨŽƌ//dͲ:͗DĞĐŚĂŶŝĐƐ//

Let T be the time of the period of the particle, then


2p r 2 ¥ 3.14 ¥ 20
T= = = 4 seconds
v 31.4
If t is the time of the flight of the shell, then
t = time of [(2n – 1)/2] revolutions of the particle
(2n - 1)
= ¥ 4 = 2(2n – 1) second
2
2u sin q
For a projectile, the time of flight is given by t=
g
2u sin q
Hence, = 2(2n – 1) ...(i)
g
u2 sin 2q
The range of the projectile is given by R=
g
u2 sin 2q
Hence, = 20 3 ...(ii)
g
2
(2n - 1)
From Eqs. (i) and (ii) tan q =
3
(2n - 1)2
For q to be smallest, n = 1, so tan q =
3

Problem 1.58 Figure 1.103 shows top view of a circular rotating table, rotating with speed
w. Two particles connected by a string are kept on two mutually perpendicular radii.
Coefficient of friction is m. What can be the maximum angular speed of the table so that the
particles do not slip on it?
mrw2 (Pseudo
force)
m Y

r T
T q X
mrw2 45° f
m
r w q
(Pseudo
force) f

(a) (b)

Fig. 1.103

Solution We will solve this problem in the reference frame of table. Friction force is static,
therefore it is variable. Let friction f act at an angle q as shown in Fig. 1.103.
fmax = mN
In the impending state of motion,
SFy = mrw2 – (f cos q + T cos 45°) = 0 ...(1)
SFx = f sin q – T sin 45° ...(2)
ŝƌĐƵůĂƌDŽƟŽŶപ53

From Eqs. (1) and (2) we eliminate T, to obtain


Ê 1 1 ˆ
mrw2 = f (sin q + cos q) = f 2 Á sin q + cos q ˜
Ë 2 2 ¯
= mmg 2 sin (45° + q)
2m g
or w2 = sin ( 45∞ + q )
r
Since maximum value of sin (45° + q) = 1,

2m g
therefore, wmax. =
r

Problem 1.59 A ball of mass M is swung around in a circle around l


on a light spring which has spring constant k. The ball describes a q
horizontal circle a distance h above the floor. The stretched spring has M
a length l and makes an angle q with the vertical as shown in Fig.
1.104. Neglect air resistance. h

(a) In terms of only the given quantities, what is the magnitude of


the force F that the spring exerts on the mass M?
Fig. 1.104
(b) In terms of F, k and l, what is the natural length l0 of the spring
i.e., the length of the spring when it is not stretched?
(c) In terms of F, l, M and q, what is the speed v of the ball?
(d) At some instant of time, the spring breaks. The ball moves a horizontal distance x
before it hits the floor. In terms of v, h, and g, what is x?
Solution
Concept: When a particle moves in a circle perpendicular forces along y-axis balance out.
Towards the horizontal circle centripetal acceleration acts therefore that must be a resultant
force. Note that along the length of spring case are not balanced because this direction has
component acceleration.

y-axis
T T cos q Component of
q acceleration
ac
q ac

mg
(a) (b)

Fig. 1.105

Mg
(a) Fsp cos q = Mg fi Fsp =
cos q
54പWŚLJƐŝĐƐĨŽƌ//dͲ:͗DĞĐŚĂŶŝĐƐ//

(b) Fsp = KDl


Fsp Mg q l
Dl = =
K K cos q
Fsp
l0 = l – Dl r =/ sin q
Mg
l0 = l –
K cos q Fig. 1.106
2
mv
(c) Fsp sin q =
r
mg mv 2
sin q =
cos q l sin q

gl sin 2 q
v =
cos q
1 2 2h
(d) h = gt fi t=
2 g
x = vt
2h
x = v
g

Problem 1.60 A block of mass 25 kg rests on a horizontal floor


(m = 0.2). It is attached by a 5 m long horizontal rope to a peg fixed on 5m
the floor. The block is pushed along the ground with an initial velocity
of 10 m/s so that it moves in a circle around the peg. Find:
Fig. 1.107
(a) Tangential acceleration of the block.
(b) Speed of the block at time t.
(c) Time when tension in rope becomes zero.
Solution
(a) Tangential acceleration is the retardation produced by the friction
a = – f/m = – mmg/m
at = – 0.2 ¥ 10 = – 2 m/s2
dv
(b) = at = – 2 10 m/s
dt 5m
v t

Ú dv = -2 Ú dt
10 0
Fig. 1.108
v – 10 = – 2t
v = 10 – 2t
(c) Tension in the rope will become zero when centripetal acceleration becomes zero, i.e.,
when speed becomes zero
v=0 fi 10 – 2t = 0 fi t = 5 sec.
ŝƌĐƵůĂƌDŽƟŽŶപ55

Problem 1.61 A particle of mass m moves along the internal smooth surface of a vertical
cylinder of radius R. Find the force with which the particle acts on the cylinder wall if at
the initial moment of time its velocity equals v0 and forms an angle a with the horizontal.
Concept
In this case the particle moves on a circular path whose centre moves
downwards with gravitational acceleration with initial velocity v0 sin a.
Since, the surface is smooth. So, no force is present along the tension
of circular path. So, the magnitude of a component of velocity along the
N
tangent of circular path remains constant. a
\ From Fig. 1.109, the magnitude of component of velocity along tangent is v0
vt = v0 cos a
vt2 v02 cos 2 a
\ Centripetal acceleration, an = =
R R Fig. 1.109

Normal reaction force N on the particle provides the required centripetal acceleration.
mvt2
\ N =
R

mv02 cos 2 a
\ N =
R
O
Problem 1.62 A is a fixed point at a height h above a perfectly A
inelastic smooth horizontal plane. A light inextensible string of
length L (> h) has one end attached to A and the other to a heavy
L
particle. The particle is held at the level of O, with the string taut, h
and released from rest. Find the height of the particle above the
plane when it is next instantaneously at rest.
Solution The situation is shown in Fig. 1.111. The velocity of Smooth
the particle before collision with the plane u = 2gh . It has two Fig. 1.110
components; one parallel to plane which is u cos q and other
perpendicular to plane which is u sin q. After collision u sin q gets absorbed by the plane.
At the left side of the mean position the component of velocity which will raise the particle
becomes u cos2 q. Thus, by conservation of mechanical energy, we can write
1
m (u cos2 q)2 = mgh¢
2
1 qq
or m u2 cos4 q = mgh¢
2 h L
4 (u cos q )cos q
1 Ê hˆ
or m( 2 gh )2 Á ˜ = mgh¢ u cos q
2 Ë L¯
u cos q
u
h5 u sin q
\ h = Ans.
L4 Fig. 1.111
56പWŚLJƐŝĐƐĨŽƌ//dͲ:͗DĞĐŚĂŶŝĐƐ//

Problem 1.63 A block is kept inside a hemispherical bowl rotating with angular velocity w.
Inner surface of the bowl is rough, coefficient of friction is m. The block is kept at a position
where radius makes an angle q with the vertical. What is the range of the angular speed for
which the block will stay at the given position?
Y X

Observer
N mrw 2cos q
w
q

mrw 2
q (Pseudo force)
R r = R sin q
q
mrw 2 sin q

mg cos q
mg sin q mg

(a) (b)

Fig. 1.112

Solution We analyse this problem in the reference frame of the bowl. As angular velocity
is increased the centrifugal force will increase. When the component of centrifugal force,
tangential to surface mr w2 cos q will increase, the block will have a tendency to slip upwards.
In this case friction force will act downwards. Similarly, at low angular speed the block has
a tendency to slip downwards and friction force will act upwards.
Impending Motion Upwards:
SFx = mrw2 cos q – mg sin q – mN = 0 ...(1)
SFy = N – mg sin q – mrw sin q = 0
2
...(2)
Substitute N from Eq. (2) in Eq. (1) to obtain
1/2
È g(sin q - m cos q ) ˘
wmax = Í ˙
Î R sin q (cos q + m sin q ) ˚
Impending Motion Downwards:
SFx = mrw2 cos q + mN – mg sin q = 0 ...(3)
SFy = N – mg sin q – mrw2 sin q = 0 ...(4)
Substitute N from Eq. (4) in Eq. (3) to obtain
1/2
È g(sin q - m cos q ) ˘
wmin = Í ˙
Î R sin q (cos q + m sin q ) ˚
Therefore, the block will remain stationary relative to bowl if its angular speed lies in the
range wmin. < w < wmax.
Students are advised to write equations for block in the ground reference frame also and
verify the similarity of results in both the approaches.
ŝƌĐƵůĂƌDŽƟŽŶപ57

Problem 1.64 A particle suspended from the ceiling by


inextensible light string is moving along a horizontal circle
2m
of radius 1.5 m as shown. The string traces a cone of height
2 m. The string breaks and the particle finally hits the floor
O
(which is xy plane 5.76 m below the circle) at point P. Find
the distance OP. 1.5 m 5.76 m
O
Solution Let the string breaks when the particle is 1.5 m x
right of point O and direction of its velocity v is along y-axis.
y
mv 2 P
T sin q =
r Fig. 1.113
and T cos q = mg
\ v = gr tanq T
Now time to reach the floor, q
2 h2 v2
t=
g r
\ Before it hits the floor,
Dy = vt Fig. 1.114
= 2 h2 r tanq
r
where tan q =
h1

r2
Dy = 2 h2
h1

144 (1.5)2
= 2¥ ¥
25 2
18
= m = 3.6 m
5
Its position from O, when it hits the floor = 1.5i + 3.6j

OP = (1.5)2 + (3.6)2 = 3.9 m

Problem 1.65 A small block B is supported by a turntable. The friction r


coefficient between the block and surface is m. A
(a) If turntable rotates at constant angular speed w, what can the
maximum angular speed w be for which the block does not slip?
(b) If the angular speed is increased uniformly from rest with an
angular acceleration a, at what speed will the block slip?
(c) If the turntable rotates in such a way that the block undergoes a
constant tangential acceleration, what is the smallest interval of
time in which the block can reach the speed v?
Fig. 1.115
58പWŚLJƐŝĐƐĨŽƌ//dͲ:͗DĞĐŚĂŶŝĐƐ//

Solution
(a) In Fig. 1.116, the circular path of the block is shown. The only force directed towards
the centre is Ffriction.
Since angular speed is constant, the block has centripetal acceleration only.
From Newton’s second law,
mv 2
SFx = Ffriction = max = ...(1)
r
SFy = N – mg = may = 0 ...(2)
Since Ffriction, max. = ms N from Eq. (2) we substitute N into Eq. (1).
mv 2
msmg =
r
Thus, vmax. = m s gr = r wmax.
or wmax. = m s g/r
(b) When the turntable rotates with angular v w N Y
acceleration, the block has centripetal as well as r
tangential accelerations. ac x
Ffriction
  
Therefore, a R = ac + at mg

 (a) Top view (b) Side view


|a R| = act + at2
Fig. 1.116
= (w 2 r )2 + (ra )2
Resultant acceleration of the block is parallel to the surface of turntable. The only force
that is parallel to the surface is force of friction.
So, SFx = Ffriction, max.
= msN = max

= m ac2 + at2

= m (w 2 r )2 + (ra )2 ...(3)
and SFy = N – mg = 0 ...(4)
Form Eqs. (3) and (4) aR a t
v
ms mg = m (w 2 r )2 + (ra )2 aC
2 2 2 2
or (w r) = (msg) – (ra) Top view
Ffriction N
1/4
ÈÊ m g ˆ 2 ˘
or w = ÍÁ s ˜ - a 2 ˙
ÍÎË r ¯ ˙˚ Ffriction
mg
Side view

Fig. 1.117
ŝƌĐƵůĂƌDŽƟŽŶപ59

(c) From equation of kinematics,


v = v0 + at
If block starts from rest, v0 = 0.
v
So, t =
a
where a = at = ra
When the block is on the verge of slipping,

v = ( m s g )2 - (ra )2

( m s g )2 - (ra )2
Therefore, t =
(ra )

Problem 1.66 A chain of mass m forming a circle of radius R is slipped on a smooth round
cone with half-angle q. Find the tension of the chain if it rotates with a constant angular
velocity w about a vertical axis coinciding with the symmetry axis of the cone.
Discussion
Every element of chain moves on a circular path. So, every element of the chain experiences
centripetal force. We consider a small element Dm on the chain making angle Dq at the centre
(shown in Fig. 1.118).

Dq Dq Dq Dq
T cos T cos
DN sin q 2 2 2 2
q q
DN Dq Dq
T
2 2
q
DN cos q T T

Dq
2T sin
2 T

Dmg Dq
T sin
2

(a) (b)

Fig. 1.118

From Fig. 1.118


DN sin q = Dmg ...(i)
Net force towards the centre is
Dq
2T sin DN cos q = DmRw2
2
Dq
or DN cos q = 2T sin – DmRw2 ...(ii)
2
60പWŚLJƐŝĐƐĨŽƌ//dͲ:͗DĞĐŚĂŶŝĐƐ//

From Eqs. (i) and (ii), we get


Dq
2T sin - DmR w 2
cot q = 2
Dmg

But Dq is very small.


Dq Dq
\ sin =
2 2

T Dq - DmR w 2
∵ cot q =
Dmg
Here Dm makes an angle Dq at the centre. The total mass of chain is m.
\ The mass of chain per unit angle is
m
l=
2p
m mDq
\ Dm = lDq = Dq =
2p 2p

Ê mDq ˆ
T Dq - Á Rw 2
Ë 2p ˜¯
\ cot q =
Êm ˆ
ÁË Dq ˜ Rw 2
2p ¯
m
\ T = (Rw 2 + g cot q)
2p

Problem 1.67 A cyclist rides along the circumference of a circular horizontal plane of radius
R, the friction coefficient being dependent only on distance r from the centre O of the plane
as k = k0 (1 – r/R), where k0 is a constant. Find the radius of the circle with the centre at the
point along which the cyclist can ride with the maximum velocity. What is this velocity?
(IIT-JEE, 1975)
Concept
In the case of uniform circular motion, net force on the body provides centripetal acceleration.
Solution In this case, normal reaction on the cyclist system is balanced by weight. But force
of friction provides centripetal acceleration.
\ N = mg ...(i)
2
mv
= kN
r
mv 2
or = kmg
r
Ê rˆ
Given, k = k0 Á 1 - ˜
Ë R¯
v2 Ê rˆ
or = k0 Á 1 - ˜ g
r Ë R¯
ŝƌĐƵůĂƌDŽƟŽŶപ61

Ê r2 ˆ
\ v2 = k0 Á r - ˜ g
Ë R¯
For vmax. v2 is also max.
d(v 2 ) R
\ =0 fi r=
dr 2
Ê r2 ˆ
∵ v2 = k0 Á r - ˜ g
Ë R¯

Ê R (- R/2)2 ˆ
\ v2max = k0 Á - g
Ë2 R ˜¯

1
\ vmax = k0 Rg
2

Problem 1.68 A disc of radius R has a light pole fixed w


perpendicular to the disc at the circumference which, in turn,
R
,has a pendulum of length R attached to its other end as shown 30°
in Fig. 1.119. The disc is rotated with a constant angular velocity R
w. The string is making an angle 30° with the rod. Then the
angular velocity w of disc is
1/2
Ê 3g ˆ Ê 3g ˆ
1/2

(a) Á ˜ (b) Á ˜
Ë R ¯ Ë 2R ¯ Fig. 1.119
1/2 1/2
Ê g ˆ Ê 2g ˆ
(c) Á (d) Á
Ë 3R ˜¯ Ë 3 3R ˜¯

Solution The bob of the pendulum moves in a circle w


3R
of radius (R + R sin 30r) = . 30° T cos 30°
2
Ê 3R ˆ
Force equations: T sin 30° = m Á ˜ w 2 T sin 30°
Ë 2 ¯
T cos 30° = mg mg

3 w 2R 1 2g
fi tan 30°= = fi w=
2 g 3 3 3R Fig. 1.120
\ (d) is the right answer. 2400
Problem 1.69 A table fan rotating at a speed of 2400 rpm is switched
Rev/min

off and the resulting variation of the rpm with time is shown in
600
Fig. 1.121. The total number of revolutions of the fan before it comes to
rest is (BITSAT, 2014) 0 8 16 24
(a) 420 (b) 280 t(s)
(c) 190 (d) 16800 Fig. 1.121
62പWŚLJƐŝĐƐĨŽƌ//dͲ:͗DĞĐŚĂŶŝĐƐ//

(rev/s) 40
Solution The corresponding (rev./sec), graph is given in Fig. 1.122.
Area under this curve gives the total number revolutions.
10
1 1
D= (8) (30) + (10 ¥ 8) + (16) (10) = 280 revolutions.
2 2 0 8 24
t(s)
\ (b) is the right answer. Fig. 1.122
Problem 1.70 A particle is projected horizontally from the top of a cliff of height H with a
speed 2gH . The radius of curvature of the trajectory at the instant of projection will
(WBJEE, 2013)
(a) H/2 (b) H (c) 2H (d) μ.
 
Solution Since, g ^ v ; radial acceleration ar = g
We know ar = v2/r
v2
fi = g where r is the radius of curvature.
r
2gH
fi =g (∵ v = 2gH )
r
fi r = 2H
\ Hence, (c) is the right answer.
Problem 1.71 A spotlight S rotates in horizontal plane with a constant angular velocity of
0.1 rad/s. The spot of light P moves along the wall at a distance of 3 m. What is the velocity
of the spot when q = 45°? (IIT-JEE, 1987)
vp
Solution The situation is shown in Fig. 1.123. From geometry O
d = x tan q x q
d
or x = d cot q
dx dq q
or = – d cosec2 q
dt dt w
dx Fig. 1.123
or = – d cosec2 q w
dt
or vp = – d(2) (0.1) = – 0.6 ms–1
Problem 1.72 A wet open umbrella is held upright and is rotated about the handle at a
uniform rate of 21 revolutions in 44 sec. If the rim of the umbrella has a circle 1 metre in
diameter and the height of the rim above the floor is 1.5 metres, find where the drops of
water spun off the rim hit the floor.
Solution Angular speed of umbrella
21 ¥ 2p
w =
44
21 ¥ 2 22
= ¥ = 3 rad s–1
44 7
ŝƌĐƵůĂƌDŽƟŽŶപ63

Velocity of water drops of the rim


1
v = wR = 3 ¥ = 1.5 ms–1
2
The water particle will become horizontal projectile after R
leaving the rim.

2H 2 ¥ 1.5
R¢ = v = 1.5 = 0.8 m H
g 10
S
Thus, distance from the centre using Pythagoras’ theorem

S= R2 + R¢ 2 = 0.52 + 0.8 2 = 0.94 m
Fig. 1.124

Problem 1.73 A point moves along the arc of a circle of radius R. Its velocity depends on
the distance covered as v = b s where b is constant. Find the resultant acceleration and the
angle a between the acceleration vector and velocity vector as a function of s.
Solution v=b s or v2 = b 2 s
dv dv b2
2v = b2 or v = = at
ds ds 2
v2 b 2s
Also, ac = =
R R
2 2
Ê b 2 ˆ Ê b 2s ˆ
Now, ar = ac2 + at2 = Á ˜ + Á ˜
Ë 2¯ Ë R ¯

ac b 2 s/R 2s
Direction tan a = = =
at b2 R
2
Problem 1.74 A horizontal plane supports a stationary
vertical cylinder of radius R and a disc A attached to the
cylinder be a horizontal thread AB of length l0. An initial
velocity v0 is imparted to the disc as shown in Fig. 1.125. v0
How long will it move along the plane till it strikes the
cylinder? The friction is assumed to be absent. R l0

Solution In the process the disc remains constant, because B


Top view
A
tension force acts perpendicular to velocity. Thus, we have
to find the total length of the path, which then divided by Fig. 1.125
speed v0 will give the time of motion.
For any angle q, l = (l0 – Rq)
\ ds = ldq = (l0 – Rq)dq
64പWŚLJƐŝĐƐĨŽƌ//dͲ:͗DĞĐŚĂŶŝĐƐ//

Total length of the path ds


( l0/R )
s = Ú (l0 - Rq )dq dq l
0 dq
l0/R v0
Rq 2 q
= l0q -
2 l0
0
2 Fig. 1.126
Ê l ˆ R(l0/R)
= l0 Á 0 ˜ -
Ë R¯ 2
l02
=
2R
s l2
Thus, time t = = 0 Ans.
v0 2Rv0
P
Problem 1.75 A particle P is moving on a circle under the action of only
one force acting always towards fixed point O on the circumference. Find F
2 q
dq Ê dq ˆ O
ratio of and Á ˜ . C
dt 2 Ë dt ¯

Solution
Concept: Determine tangential and normal component of force F. Apply Fig. 1.127
SFn = mRw 2

SFt = mRt
F cos q = mw2R ...(1)
F sin q = mar ...(2)
Angular velocity w of line joining P and C is
d(2q ) dq
w = =2
dt dt
dq Ê w ˆ
= Á ˜ and tangential acceleration of particle about
dt Ë 2 ¯
F sin q
ar =
m
Ê F sinq ˆ F sinq
aT = Ra = Á
Ë m ˜¯
;a=
mR
È dq ˘
d Í2 ˙
d 2q d 2q a
= Î
dw dt ˚
a = =2 2 , 2 =
dt dt dt dt 2
ŝƌĐƵůĂƌDŽƟŽŶപ65

P P
q
F F cos q
F sin q
q 2q 2q
O
C C

Fig. 1.128

d 2q F sinq
= ...(3)
dt 2 2mR
From Eq. (1) 1/2
dq w 1 Ê F cos q ˆ
= = ¥Á ˜
dt 2 2 Ë mR ¯
2 2
Ê dq ˆ Êwˆ 1 F cos q
ÁË ˜¯ = ÁË ˜¯ = ...(4)
dt 2 4 mR

d 2q
dt 2 = F sin q ¥ 4 mR = 2 tan q
2
Ê dq ˆ 2mR F cos q
ÁË ˜¯
dt

Problem 1.76 A vertical frictionless semicircular track of


radius 1 m is fixed on the edge of a movable trolley shown 1m
in Fig. 1.129. Initially, the system is at rest and a mass m
is kept at the top of the track. The trolley starts moving to a
1.3 m
2g
the right with a uniform horizontal acceleration a = .
9
The mass slides down the track, eventually losing contact
with it and dropping to the floor 1.3 m below the trolley. Fig. 1.129

(a) calculate the angle q at which it loses contact with the trolley, and
(b) the time taken by the mass to drop on the floor, after losing contact.
Solution N

(a) In accelerated frame the FBD of the particle is shown ma q


v q
in Fig. 1.130.
mg
In radial direction, we have a
mv 2
mg cos q – (N + ma sin q) = ...(i)
R

Fig. 1.130
66പWŚLJƐŝĐƐĨŽƌ//dͲ:͗DĞĐŚĂŶŝĐƐ//

Along tangent of the path


mg sin q + ma cos q = mat
Ê vdv ˆ
= mÁ ...(ii)
Ë ds ˜¯
Here, ds = Rdq. For losing contact N = 0.
After solving the above equations, we get the required answer, q = 36.8°. Ans.
(b) The vertical displacement
s = (R cos q + 1.3)
uy = v sin q
If t is the time to hit the floor, then
1 2
s = u yt + gt
2
After substituting the given values and simplifying, we get
t = 0.41 s Ans.
Problem 1.77 Figure 1.131 shows a pendulum of length l A
suspended at a distance x vertically above a peg.
(a) The pendulum bob is deflected through an angle q and l q x
then released. Find the speed of the bob at the instant
E
shown in Fig. 1.131. B
(b) The pendulum is released when q = 90°. For what x f
(position of peg) will the pendulum complete the circle?
(c) The pendulum is released when q = 60°. What is the D
velocity of the bob as it passes position E. C Reference level

Solution Fig. 1.131

(a) As we have learnt earlier, the minimum velocity required at the lowermost point so
as to complete the circle is given by the expression

v= 5gl
From conservation of energy between position of release and position C,
KEA + UA = KEC + UC
1
0 + mgl = mv2 + 0
2
or v = 2gl
For just completing the circle,
v = 2 gl = 5 g(l - x )

3
or x = l
5
ŝƌĐƵůĂƌDŽƟŽŶപ67

(b) From conservation of energy between positions B and C,


KEB + UB = KEC + UC
1
0 + mg(l – l cos q) = mv02 + mg(l – x) (1 – cos f)
2
or v0 = [2gl (1 – cos q) – 2g(l – x) (1 – cos f)]1/2
(c) From conservation of energy between positions B and E,
KEB + UB = KEE + UE
1
0 + mgl (1 – cos 60°) = mvE2 + mg(l – x)
2
or vE2 = 2gx – 2gl cos 60°
1/2
È Ê l ˆ˘
or vE = Í 2 g Á x - ˜ ˙
Î Ë 2¯ ˚

Problem 1.78 Two particles of mass m and 2m, connected by a massless rod, slide on the
inside of a smooth circular ring of radius r, as shown in Fig. 1.132. If the assembly is released
from rest when q = 0, determine:
(a) the velocity of the particles when the rod passes the horizontal position, and
(b) the maximum velocity vmax of the particles.

q r
r 2m
r q
r 45°
2m
m
Reference
m level

Fig. 1.132

Solution
(a) In the absence of friction the energy of the system is conserved.
KEi + Ui = KEf + Uf
1 1
0 + 2mgr = mv 2 + (2m)v2 + mgr (1 – cos 45°) + 2mgr (1 – cos 45°) ...(1)
2 2
3 2
or v = 3gr cos 45° – gr ...(2)
2
or v = 0.865 gr
68പWŚLJƐŝĐƐĨŽƌ//dͲ:͗DĞĐŚĂŶŝĐƐ//

(b) At any general position q of the rod, the conservation of energy between initial position
and final position gives
1 1
2mgr = mgr (1 – cos q) + 2mgr (1 – sin q) + mv 2 + (2m)v2 ...(3)
2 2
3
or mv 2 = mgr cos q + 2mgr sin q – mgr ...(4)
2
2
or v2 = gr [cos q + 2 sin q – 1] ...(5)
3
For v to be maximum, the expression in bracket must be maximum,
d
i.e., (cos q + 2 sin q – 1) = 0
dq
or – sin q + 2 cos q = 0

(r – r sin q)
or tan q = 2 q

(r – r cos q)
q
2 1
From which we obtain sin q = and cos q =
5 5
Substituting these values in Eq. (5), we obtain Reference level

2 È 1 4 ˘ Fig. 1.133
v2 = gr Í + - 1˙
3 Î 5 5 ˚
or vmax = 0.908 gr
Problem 1.79 A pendulum bob is suspended on a flat car that moves with velocity v0. The
flat car is stopped by a bumper:
(a) What is the angle through which the pendulum swings?
(b) If the swing angle is q = 60° and l = 5 m, what was the initial speed of the flat car?

q l cos q
B
l
l h
v0
Reference A
level
(a) (b)

Fig. 1.134

Solution When the flat car collides with the bumper, due to inertia of motion the bob swings
forwards. No work is done by tension of string on the bob, therefore energy is conserved.
KEA + PEA = KEB + PEB
1
mv02 + 0 = 0 + mg (l – l cos q)
2
or v02 = 2gl(1 – cos q)
ŝƌĐƵůĂƌDŽƟŽŶപ69

= 4gl sin2 q/2 ...(1)


Ê v ˆ
or q = 2 sin–1 Á 0 ˜ ...(2)
Ë 2 gl ¯
On substituting numerical values
q = 30°, l = 10 m, g = 10 m/s2
We obtain
q
v = 2 gl sin
2
= (10) ¥ (10) sin 30∞ = 10 m/s

Problem 1.80 With what minimum speed v must a


small ball should be pushed inside a smooth vertical
tube from a height h so that it may reach the top of the d
tube? Radius of the tube is R.
R 2R
Solution vtop = v 2 - 2 g( 2R - h)
h v
To just complete the vertical circle vtop may be zero
d << R
\ 0 = v 2 - 2 g( 2R - h)
Fig. 1.135
or v = 2 g( 2R - h) Ans.

Problem 1.81 In Fig. 1.136, the mass m1 moves in a horizontal circle just like a conical
pendulum while the masses m2 and m3 hang vertically at rest. If the string between m2 and
m3 is cut at t = 0, find the instantaneous change in the tension of the string.

q l

r m1

m2

m3

Fig. 1.136

Solution For m1 to be at rest


T sin q = mw2r
= mw2l sin q
or T = mw2l ...(i)
70പWŚLJƐŝĐƐĨŽƌ//dͲ:͗DĞĐŚĂŶŝĐƐ//

and T cos q = m1g ...(ii)


Also, T = (m2 + m3)g ...(iii)
q l
When string between m2 and m3 is cut, the block m1 starts
T
accelerating along the string. Suppose T¢ is now the tension in the q
r
string. If a is the acceleration, then
T a
m1g – T¢cos q = m1a cos q ...(iv) m1g
m2
and T¢ – m2g = m2a ...(v)
After simplifying the above equations, we get m3

Èmm g˘
DT = T – T¢ = Í 1 3 ˙ Ans. Fig. 1.137
Î m1 + m2 ˚
Y
Problem 1.82 Two particles A and B start at the origin O
and travel in opposite directions along the circular path at
constant speeds vA = 0.7 m/s and vB = 1.5 m/s, respectively.
m
Determine the time when they collide and the magnitude of 5.0
the acceleration of B just before this happens.
Solution 1.5t + 0.7t = 2pR = 10p B
A
10p
\ t = = 14.3 s O
X
2.2 vB = 1.5 m/s vA = 0.7 m/s
vB2
a = = 0.45 m/s2 Fig. 1.138
R
m
Problem 1.83 A puck of mass m slides on a frictionless table while r

attached to a hanging cylinder of mass M by a cord through a hole in


the table. What speed keeps the cylinder at rest?

mv 2 mv 2 M M
Solution Fc = =T or v= = Mg or v= rg
r r m
Fig. 1.139
Problem 1.84 A bead can slide without friction on a circular hoop of radius
0.1 m in a vertical plane. The hoop rotates at a constant rate of 2 rev s–1 about
a vertical diameter, as in Fig. 1.140.
0.1 m
(a) Find the angle q at which the bead is in vertical equilibrium.
(b) Is it possible for the bead to “ride” at the same elevation as the centre q
of the hoop?
Solution Fig. 1.140
g g 10
(a) w2 = or cos q = = = 0.63
l cosq w 2l (2 ¥ 2p )2 ¥ 0.1
(b) No
ŝƌĐƵůĂƌDŽƟŽŶപ71

Problem 1.85 A small coin of mass 40 g is placed on the horizontal Vertical


surface of a rotating disc. The disc starts from rest and is given a
constant angular acceleration a = 2 rad/s2. The coefficient of static
r
friction between the coin and the disc is ms = 3/4 and coefficient of
kinetic friction is mk = 0.5. The coin is placed at a distance r = 1 m from
the centre of the disc. The magnitude of the resultant force on the coin
exerted by the disc just before it starts slipping on the disc is
(a) 0.2 N (b) 0.3 N
Fig. 1.141
(c) 0.4 N (d) 0.5 N
Solution The friction force on the coin just before the slipping will be
f = ms mg. Normal reaction on the coin, N = mg
The resultant reaction by the disk to the coin is

= N 2 + f 2 = (mg )2 + m s (mg )2 = mg 1 + m 2

9
= 40 ¥ 10–3 ¥ 10 ¥ 1+ = 0.5 N
16
\ (d) is the right answer. Fig. 1.142

Problem 1.86 A large mass M and a small mass m hangs at the two ends of the string that
passes through a smooth tube as shown in Fig. 1.143. The mass m moves around in a circular
path which lies in the horizontal plane. The length of the string from the mass m to the top
of the tube is l, and q is the angle; this length makes with the vertical. What should be the
frequency of rotation of the mass m so that M remains stationary. (IIT-JEE, 1978)

r m

Fig. 1.143

Solution For M to be stationary q

T = Mg ...(i) l
T
For the circular motion of m, we have
r q
T sin q = mw2r = mw2 l sin q
or T = mw2l ...(ii) T
mg
M
From Eqs. (i) and (ii), we get
Mg
Mg
w = Ans.
ml Fig. 1.144
72പWŚLJƐŝĐƐĨŽƌ//dͲ:͗DĞĐŚĂŶŝĐƐ//

Problem 1.87 If the system shown in Fig. 1.145 is rotated in w


a horizontal circle with angular velocity w. (g = 10 m/s2) Find:
m1
(a) the minimum value of w to start relative motion
between the two blocks.
(b) tension in the string connecting m1 and m2 when m2
slipping just starts between the blocks.
R
The coefficient of friction between the two masses is 0.5
and there is no friction between m2 and the ground. The Fig. 1.145
dimensions of the masses can be neglected. (Take R = 0.5 m, m1 = 2 kg, m2 = 1 kg)
Solution
(a) Force diagrams of m1 and m2 are as shown T
m1
T mm1g
m2
in Fig. 1.146.
mm1g
(Only horizontal forces have been shown)
Equations of motion are Fig. 1.146

T + mm1g = m1Rw2 ...(i)


T – mm1g = m2Rw2 ...(ii)
Solving Eqs. (i) and (ii) we have
2m1m g
w =
(m1 - m2 )R
Substituting the values, we have
wmin = 6.32 rad/s Ans.
2
(b) T = m2Rw + mm1g
= (1) (0.5) (6.32)2 + (0.5) (2) (10)
= 30 N Ans.
Problem 1.88 A point moves in a circle with a velocity of v = at where a = 0.50 m/s . Find 2

the total acceleration of the point at the moment when it covered the nth (n = 0.10) fraction
of the circle after the beginning of the motion.
Solution Given v = at
dv
Tangential acceleration, at = = a = 0.5 m/s2
dt
If t is the time to cover nth friction of circle, then
1 2
n(2pR) = at t
2
1/2
Ê 4p nR ˆ
t =Á
Ë a ˜¯ t
1/2
Ê 4p nR ˆ
The velocity at this time, v = at = a Á
Ë at ˜¯
ŝƌĐƵůĂƌDŽƟŽŶപ73

Normal acceleration
v 2 4p nRa 2
an = =
R at R
4p na 2 4p ¥ 0.10 ¥ 0.52
= =
at 0.5
= 0.625 m/s2

Total acceleration a = at2 + an2 = 0.8 m/s2


Problem 1.89 A block of mass m is constrained to move inside a ring of
v0
radius R which is on a frictionless table. At time t = 0, block is moving
along the inside of the ring (i.e., in the tangential direction) with velocity
R
v0. The coefficient of friction between the block and the ring is m. Find the
speed of the block at time t.
Solution
mv 2 Fig. 1.147
N =
R
fmax = mN
mmv 2
=
R
fmax
\ Retardation a =
m
mv 2
=
R
Ê dv ˆ mv 2
\ ÁË - ˜¯ =
dt R
v dv m t
or Úv0 v
2
= Ú dt
R 0
v0
or v = Ans.
mv t
1+ 0
R

Problem 1.90 A point moves along a circle with a velocity v = kt, where k = 1 m/s2. Find
the total acceleration of the point at the moment when it has covered the nth fraction of the
1
circle after the beginning of the motion, where n = .
10
ds
Solution v = = kt
dt
74പWŚLJƐŝĐƐĨŽƌ//dͲ:͗DĞĐŚĂŶŝĐƐ//

s t
or Ú0 ds = k Ú t dt
0

1 2
\ s = kt
2
For completion of nth fraction of circle,
s = 2prn
or t2 = (4pnr)/k ...(i)
dv
Tangential acceleration, aT = =k ...(ii)
dt
v 2 k 2t 2
Normal acceleration, aN = = ...(iii)
r r
or aN = 4pnk

\ a = ( aT2 + aN
2
) = [k2 + 16p2n2k2]1/2
= k [1 + 16p2n2]1/2
= 1 [1 + 16 ¥ (3.14)2 ¥ (0.10)2]1/2
= 0.16 m/s2

Problem 1.91 Two bodies A and B separated by a distance of 2R are moving counter-
clockwise along a circular path of radius R, each with a uniform speed v. If at time t = 0,
Ê 72 ˆ Ê v ˆ
2
A is given a constant tangential deceleration aq = Á ˜ Á , determine the time when B
Ë 25 ¯ Ë Rp ˜¯
collides with A.
Solution For the tangential motion of the particles, relative speed
ur = v – v = 0
and ar = (aq – 0) = aq
The relative separation s = pR.
From the second equation of motion,
1 v
s = urt + ar t 2
2
1
pR = 0 + aq t 2
2 v

2p R
\ t = Fig. 1.148
aq

2p R 5p R
= = Ans.
Ê 72 v ˆ 2 6v
Á 25 Rp ˜
Ë ¯
ŝƌĐƵůĂƌDŽƟŽŶപ75

Problem 1.92 Calculate the linear acceleration of a particle moving in a circle of radius
0.4 m at the instant when its angular velocity is 2 rad/s and its angular acceleration is 5
rad/s2.
Solution Normal acceleration of the particle,
an = w2r = 22 ¥ 0.4 = 1.6 m/s2
Tangential acceleration of the particle,
at = ar = 5 ¥ 0.4 = 2 m/s2 a
q
Resultant acceleration, at

a = an2 + at2 an

= 1.6 2 + 22 = 2.6 m/s2 Ans.


an 1.6
tan q = = = 0.8; q = 38°40¢ Ans. Fig. 1.149
at 2

Problem 1.93 A point moves along a circle with a velocity v = at, where a = 0.50 m/s2. Find
the total acceleration of the point at the moment when it covered the nth (n = 0.10) fraction
of the circle after the beginning of the motion.
Ans. a = 0.8 m/s2
Problem 1.94 A particle is suspended by a string of length l. It is projected with such a
velocity v along the horizontal such that after the string becomes slack it flies through its
initial position. Find v.
Solution Let the velocity be v¢ at B where the string becomes slack and the string makes
angle q with the horizontal by the law of conservation of energy.
1 1 B
mv 2 = mv ¢ 2 + mgl(1 + sin q ) ...(i)
2 2 q
O
or, v¢2 = v2 – 2gl (1 + sin q) ...(ii)
By the dynamics of circular motion
mv ¢ 2
mg sin q =
l
fi v¢2 = gl sin q ...(iii) vÆ
Fig. 1.150
From Eqs. (ii) and (iii) we get
\ gl sin q = v2 – 2gl (1 + sin q) ...(iv)
At B the particle becomes a projectile of velocity v¢ at 90 – q with the horizontal.
Here, ux = v¢ sin q and uy = v¢ cos q
ax = 0 and ay = – g
\ l cos q = v¢ sin qt
76പWŚLJƐŝĐƐĨŽƌ//dͲ:͗DĞĐŚĂŶŝĐƐ//

B
 cos q 1 cos q 1 12 cos 2 q
\t= and – l (1 + sin q) = v¢ cos q - g mg sin q
v ¢ sin q v ¢ sin q 2 v ¢ 2 sin 2 q q
mg cos q
mg
fi 2 sin3 q + 3 sin2 q – 1 = 0
1
\ sin q = is the acceptable solution
2
1 7 gl 7 gl
\ v2 = 2gl + 3gl ¥ = fiv=
2 2 2 A
v
(from Eq. (iv)) Fig. 1.151
Problem 1.95 Prove that a motor car moving over a convex bridge is lighter than the same
car resting on the same bridge.
Solution The motion of the motor car over a convex bridge AB is the R
motion along the segment of a circle AB (Fig. 1.152). The centripetal force
is provided by the difference of weight mg of the car and the normal B
A
reaction R of the bridge. mg

mv 2 mv 2
\ mg – R = or R = mg –
r r
Fig. 1.152
Clearly R < mg, i.e., the weight of the moving car is less than the weight
of the stationary car.
Problem 1.96 A body weighing 0.4 kg is whirled in a vertical circle with a string making
2 revolutions per second. If the radius of the circle is 1.2 m. Find the tension (a) at the top
of the circle, and (b) at the bottom of the circle.
Given: g = 10 ms–2 and p = 3.14.
Solution Mass, m = 0.4 kg,
1
Time period = second, radius, r = 1.2 m
2
2p
Angular velocity, w = = 4p rad s–1 = 12.56 rad s–1
1/2
(a) At the top of the circle,
mv 2
T = – mg = mrw2 – mg = m(rw2 – g)
r
= 0.4 (1.2 ¥ 12.56 ¥ 12.56 – 9.8)N = 71.2 N
(b) At the lowest point, T = m (rw2 + g) = 80 N

Problem 1.97 A system in which ring of mass M hangs from a thread


and two beads of mass m slides on it without friction. The beads are q
released simultaneously from the top of the ring and slides down in
3M
opposite sides. Show that the ring will start to rise, if m > . a ring of
2 mass M
Fig. 1.153
ŝƌĐƵůĂƌDŽƟŽŶപ77

Solution Let R be the radius of the ring


h = R(1 – cos q)
q
v2 = 2gh = 2gR (1 – cos q) h
mv 2 R
q v
= N + mg cos q q
R
or N = 2mg (1 – cos q) – mg cos q
N = 2mg – 3mg cos q
In the critical condition, tension in the string is zero and net
upward force on the ring
Fig. 1.154
F = 2N cos q = 2 mg (2 cos q – 3 cos2 q) ...(i)
dF
F is maximum when =0
dq
or – 2 sin q + 6 sin q cos q = 0
1
or cos q =
3
Substituting in Eq. (i)
Ê 1 1ˆ 2
Fmax = 2mg Á 2 ¥ - 3 ¥ ˜ = mg
Ë 3 9¯ 3
Fmax > Mg
2
or mg > Mg
3
3
or m > M Ans.
2
Problem 1.98 A turn of radius 20 m is banked for the vehicles going at a speed of 36 km/h.
If the coefficient of static friction between the road and the tyre is 0.4. What are the possible
speeds of a vehicle so that it neither slips down nor skids up? (g = 9.8 m/s2)

Ê v2 ˆ
Solution Banking angle, q = tan–1 Á ˜
Ë Rg ¯
36 km/h = 10 m/s

Ê 100 ˆ
\ q = tan–1 Á = 27°
Ë 20 ¥ 9.8 ˜¯
Angle of repose,
l = tan–1 (m) = tan–1 (0.4) = 21.8°
Since q > l, the vehicle cannot remain in the given position with v = 0. At rest it will slide
down. To find minimum speed, so that vehicle does not slip down, maximum friction will act
up the plane.
78പWŚLJƐŝĐƐĨŽƌ//dͲ:͗DĞĐŚĂŶŝĐƐ//

To find maximum speed, so that the vehicle does not skid N


up, maximum friction will act down the plane. mN
q
Minimum Speed q
Equation of motion are,
N cos q + mN sin q = mg ...(i) q
m 2 mg
N sin q – mN cos q = vmin ...(ii)
R
Fig. 1.155
Solving these two equations we get
vmin = 4.2 m/s Ans. N

Maximum Speed
q
Equations of motion are,
q
N cos q – mN sin q = mg ...(iii)
mN
m 2 q
N sin q + mN cos q = vmax ...(iv)
R mg
Solving these two equations, we have,
Fig. 1.156
vmax = 15 m/s Ans.

Problem 1.99 A smooth circular tube of radius R is fixed in a vertical plane. A particle is
projected from its lowest point with a velocity just sufficient to carry it to the highest point.
What is the time taken by the particle to reach the end of the horizontal diameter.
Solution Minimum velocity of particle at the lowest position to complete the circle should
be 4gR inside a tube.
So, u = 4gR

h = R (1 – cos q)
\ v2 = u2 – 2gh
or v2 = 4gR – 2gR(1 – cos q)
= 2gR(1 + cos q)
q
Ê qˆ v
or v2 = 2gR Á 2 cos 2 ˜ R
h = (R – R cos q)
Ë 2¯ h
h = (1 – cos q)
q u
or v = 2 gR cos
2 Fig. 1.157
From ds = v ◊ dt
q
We get R dq = 2 gR cos ◊ dt
2
ŝƌĐƵůĂƌDŽƟŽŶപ79

t 1 R p/2 Êqˆ
or Ú0 dt =
2 g Ú0
sec Á ˜ dq
Ë 2¯

p/2
RÈ Ê q qˆ˘
or t = ln Á sec + tan ˜ ˙
g ÍÎ Ë 2 2 ¯ ˚0

R
or t = ln(1 + 2 ) Ans.
g
mm
Problem 1.100 A circular tube of mass M is placed vertically on a hori-
zontal surface as shown in Fig. 1.158. Two small spheres, each of mass m, q
just fit in the tube, are released from the top. If q gives the angle between
radius vector of either ball with the vertical, obtain the value of the ratio
M/m if the tube breaks its contact with the ground when q = 60°. Neglect
any friction.
Solution Speed of each particle at angle q is, Fig. 1.158

v = 2gh (from energy conservation)


where, h = R (1 – cos q)
\ v = 2 gR(1 - cos q ) T¢

mv 2
N + mg cos q = T sin 30°
R
or N + mg cos q = 2mg (1 – cos q)

°
30
or N = 2mg – 3mg cos q ...(i)
T cos 30°
T
The tube breaks its contact with the ground when 2N cos q > Mg
Substituting, 2N cos q = Mg Fig. 1.159
or 4mg cos q – 6mg cos2 q = Mg
Substituting, q = 60°
3mg
2mg – = Mg
2
M 1
or = Ans.
m 2

Note: Initially, normal reaction on each ball will be radially outward and later it will be
radially inward, so that normal reactions on the tube is radially outward to break it off from
the ground.
80പWŚLJƐŝĐƐĨŽƌ//dͲ:͗DĞĐŚĂŶŝĐƐ//

Problem 1.101 A small sphere B of mass m is released from rest in 0.8 m


the position shown and swings freely in a vertical plane, first about O
O and then about the peg A after the cord comes in contact with the 30° B
peg. Determine the tension in the cord: 0.4 m A

(a) just before the sphere comes in contact with the peg; and
(b) just after it comes in contact with the peg.
Solution h = 0.8 sin 30° = 0.4 m
\ v2 = 2gh
(a) Just before, Fig. 1.160
2
mv
T1 – mg sin 30° = (R1 = 0.8 m)
R1
mg m(2 g )(0.4) 3mg
T1 = + = Ans.
2 0.8 2
(b) Just after,
mv 2
T2 – mg sin 30° = (R2 = 0.4 m)
R2
mg m(2 g )(0.4)
T2 = +
2 0.4
5mg
or T2 = Ans.
2

Problem 1.102 A particle of mass m is suspended by a string of length l from a fixed rigid
support. A sufficient horizontal velocity v0 = 3gl is imparted to it suddenly. Calculate the
angle made by the string with the vertical when the acceleration of the particle is inclined
to the string by 45°.
Solution h = l(1 – cos q)
v2 = v02 – 2gh = 3gl – 2gl (1 – cos q) = gl (1 + 2 cos q)
At 45° means radial and tangential components of acceleration are equal.
v2
\ = g sin q
l
or 1 + 2 cos q = sin q
p
Solving the equation we get, q = 90° or Ans.
2

Problem 1.103 A threaded rod with 12 turns/cm and diameter 1.18 cm is mounted
horizontally. A bar with a threaded hole to match the rod is screwed onto the rod the bar
and pins at 216 rev/min. How long will it take for the bar to move 1.50 cm along with rod?
ŝƌĐƵůĂƌDŽƟŽŶപ81

Solution As the distance between two consecutive threads on


the rod, i.e., pitch = (1/12) cm, so the number of rotations to be
made in moving a distance of 1.5 cm,
distance 1.5
n= = = 18
pitch (1/12)
So, angular displacement of the bar
q = 2pn = 2p ¥ 18 = 36p rad
And as angular speed of the bar
Fig. 1.161
w = 2p n = 2p ¥ 216
t 60
= 7.2 p rad/s
So, time taken by the bar to travel a distance of 1.5 cm on the rod (as q = wt with a = 0)
q 36p
t = = =5s
w 7.2p

Problem 1.104 Figure 1.162 shows a particle starting from point A, C


travelling up to B with speed v, then up to point C with speed 2v and
to A with speed 3v. Find the time taken in each part and average speed 120°
O B
during the entire path. Take radius = R.
Solution
Concept: Average speed is total distance travelled divided by total time A
interval. Fig. 1.162
pR pR
tAB = tBC = 2p R/(3 ¥ 2v) =
2v 3v
5 2p R 5p R
tCA = ¥ =
12 3v 18v
2p R 2
<v> = =
tAB + tBC + tCA 1
+
1
+
5
2v 3v 18v

2 2 ¥ 18v 9v
= = = = 1.8 v
(9 + 6 + 5)/18v 20 5

Problem 1.105 Two body of masses m1 = 10 kg and m2 = 5 kg, connected to each other by
a massless inextensible cord of length 0.3 m are placed along a diameter of a turntable. The
coefficient of friction between the table and m1 is 0.5 while there is no friction between m2
and the table. The table is rotating with an angular velocity of 10 rad/s about a vertical axis
passing through its centre O. The masses are placed along the diameter of the table on either
side of the centre O such that the mass m1 is at a distance of 0.124 m from O. The masses are
observed to be at rest with respect to an observer on the turntable:
82പWŚLJƐŝĐƐĨŽƌ//dͲ:͗DĞĐŚĂŶŝĐƐ//

(i) Calculate the frictional force on m1.


(ii) What should be the minimum angular speed of the turntable so that the masses slip
from this position?
(iii) How should the masses be placed with the string remaining taut, so that there is no
frictional force acting on the mass m1?
Solution w
(i) As for the circular motion of mass m2, centripetal force
is provided by tension alone m1 m2
T O T
T = m2r 2w 2 f
r1 r2
= 5 ¥ 0.176 ¥ (10)2 = 88 N
And as for circular motion of mass m1 centripetal force
is provided by both tension and friction, i.e., Fig. 1.163
T + f = m 1r 1w 2
So, f = 10 ¥ 0.124 ¥ 102 – 88 = 124 – 88 = 36 N
(ii) the masses will slip if
T + f L < m 1r 2 w 2 i.e., (m1r2 – m2r2)w2 > f L
[as T = m2r2w2]
49
i.e., (10 ¥ 0.124 – 5 ¥ 0.176) w2 > 49 or w2 >
0.36
i.e., w > 11.67 rad/s
So, for slipping wmin = 11.67 rad/s
(iii) In the absence of friction, the masses will not slip if
m1r1w2 = m2r2 w2 = T (∫ CPF) centripetal force
i.e., m 1r 1 = m 2 r 2 with r1 + r2 = 0.3 m
i.e., 10r1 = 5(0.3 – r1) or r1 = 0.1 m
And so, r2 = 0.3 – 0.1 = 0.2 m
i.e., if the centre of mass of m1 and m2 coincides with the centre of the table, the string,
will remain taught and masses will not slip whatever be w.
Problem 1.106 If a point moves along a circle with constant speed, prove that its angular
speed about any point on the circle is half of that about the centre.
Solution Let O be a point on a circle and P be the position of P
the particle at any time t, such that q

–POA = q. Then, –PCA = 2q


q 2q
O A
Here, C is the centre of the circle. Angular velocity of P about C
O is
dq
w0 =
dt
Fig. 1.164
ŝƌĐƵůĂƌDŽƟŽŶപ83

and angular velocity of P about C is,


d dq
wC = (2q ) = 2
dt dt
or wC = 2w0 Ans.
Problem 1.107 A car is travelling along a circular curve that has a radius of 50 m. If its
speed is 16 m/s and is increasing uniformly at 8 m/s2, determine the magnitude of its
acceleration at this instant.
2
Ê dv ˆ Ê v ˆ
2 2 2
Ê 256 ˆ
Solution a= at2 + an2 + Á ˜ + Á ˜ = (8)2 + Á = 9.5 m/s2
Ë dt ¯ Ë R ¯ Ë 50 ˜¯

Problem 1.108 A car goes on a horizontal circular road of radius R and the angular speed
dw
of the car is increasing at a constant rate = a. If the coefficient of friction between the
dt
road and the tyre is m, then find the speed at which the car will skid.
Solution For skidding a = mg
Now,
v2
ac = and at = aR
R
But,
a2 = ac2 + at2
2
Ê v2 ˆ 2
(mg) = ÁË R ˜¯ + (a R)
2

or v = [{(mg)2 – (aR)2}R2]1/4
Problem 1.109 A cyclist moves in a circular path of radius r. The friction coefficient
depends on r according m = m0 (1 – r/R) where m0 is a constant. Find the radius of a concentric
circle along which the cyclist can ride with the maximum velocity. What is that velocity?
Solution For motion on a horizontal circular path

Ê r2 ˆ
v2 = m r g = m0 Á r - ˜ g
Ë R¯
for v to be maximum
dv 2
m0 ÊÁ 1 - ˆ˜ g = 0 or r =
2r R
=0 fi
dr Ë R¯ 2
Also,
Ê R Rˆ Rg
vmax = m0 Á - ˜ g = m0
Ë 2 4¯ 4
84പWŚLJƐŝĐƐĨŽƌ//dͲ:͗DĞĐŚĂŶŝĐƐ//

Problem 1.110 A simple 2 kg pendulum is released from rest 800


mm
in the horizontal position. As it reaches the bottom position, the
A
cord wraps around the smooth fixed pin at B and continues in the
90°
smaller arc in the vertical plane. Calculate the magnitude of the 400
force R supported by the pin at B when the pendulum passes the mm
B
position q = 30°. (g = 9.8 m/s2)
q
Solution Speed of bob in the given position,
v = 2gh
2 kg
Here, h = (400 + 400 cos 30°) mm
= 746 mm = 0.764 m Fig. 1.165

\ v = 2 ¥ 9.8 ¥ 0.746 = 3.82 m/s

mv 2
Now, T – mg cos q =
r
2 ¥ (3.82)2
T = 2 ¥ 9.8 ¥ cos 30° +
(0.4)
or T = 90 N
\ R = T sin 30° = 45 N
T¢ = T cos 30° Ans.
Problem 1.111 A cord of length 1 m is fixed at one end and carries a mass of 100 g at other
end. The cord makes (2/p) revolutions per second around the vertical axis through the fixed
end. Calculate: (a) the tension in the string; (b) the angle of inclination of the string with the
vertical; and (c) the linear velocity of the mass. (g = 10 m/s2).
Solution As shown in Fig. 1.166, for vertical equilibrium of S
mass m
T cos q = mg ...(i) q

While for circular motion


T sin q = (mv2/r) = mrw2 ...(ii) h L T cos q
2 2 T
(a) So, from Eq. (ii), T = [mrw /sin q] = mLw
[as r = L sin q] q
\ T = 10 –1
¥ 1 ¥ 4 = 1.6 N
2 r
O T sin q
[as w = 2pf = 2p ¥ 2/p) = 4]
mg
mg 10 -1 ¥ 10 5
(b) From Eq. (i) cos q = = = Fig. 1.166
T 1.6 8
i.e., q = cos–1 (5/8)
(c) v = rw = L sin q ¥ w [as r = L sin q]
or v = 1 ¥ 0.78 ¥ 4 = 3.12 m/s [as sin q = 0.78]
ŝƌĐƵůĂƌDŽƟŽŶപ85

Problem 1.112 A hemispherical bowl of radius R = 0.1 m is rotating about its own axis
(which is vertical) with an angular velocity w. A particle of mass 10–2 kg on the frictionless
inner surface of the bowl is also rotating with the same w. The particle is at a height h from
the bottom of the bowl. (IIT-JEE, 1993)
(a) Obtain the relation between h and w. What is the minimum value of w needed, in order
to have a non-zero value of h?
(b) It is desired to measure g using this setup, by measuring h accurately. Assuming
that R and w are known precisely and that the least-count in the measurement of
h is 10–4 m, what is the minimum possible error Dg in the measured value of g?
(g = 9.8 m/s2)
w
N
Solution The forces acting on the particle are reaction and weight, so R
for vertical equilibrium of the particle
q q R–h
N cos q = mg …(i) r
And for circular motion in horizontal plane, h

mg
N sin q = mrw2 ...(ii)
(a) Dividing Eq. (ii) by Eq. (i) Fig. 1.167
rw 2 R (sin q ) w 2
tan q = = [as r = R sin q]
g g
g R-h g È R - h˘
or cos q = 2
, i.e., = ÍÎas cos q = R ˙˚
Rw R Rw 2
or h = R – (g/w2) ...(iii)
2
So for h > 0, R – (g/w ) > 0
i.e., w > ( g/R) so wmin = (9.8/0.1) = 7 2 rad/s

(b) From Eq. (iii), g = w2 (R – h). So for a given w and R


dg = – w2 dh, i.e., (Dg)min = – (w2)min (Dh)min
So substituting the value of wmin and (Dh)min
(Dg)min = – 98 ¥ 10–4 = – 9.8 ¥ 10–3 m/s2
w
Problem 1.113 A liquid is kept in a cylindrical vessel which is rotated
along its axis. The liquid rises at the sides. If the radius of the vessel
is 0.05 m and the speed of rotation is 2 rev per sec, find the difference
Y
in the height of the liquid at the centre of the vessel and at its sides.
(g = 9.8 m/s2) (IIT Roorkee, 1987)
R q
Solution Consider a particle at position (x, y) as shown in Fig. 1.168.
For its equilibrium along y-axis q
y
X
R cos q = mg ...(i) O x
mg
And for its circular motion in horizontal plane
R sin q = mxw2 ...(ii)
Fig. 1.168
86പWŚLJƐŝĐƐĨŽƌ//dͲ:͗DĞĐŚĂŶŝĐƐ//

So, dividing Eq. (ii) by Eq. (i)


tan q = (w2/g)x ...(iii)
But from differential calculus we know that
dy
Slope = = tan q ...(iv)
dx
So, from Eqs. (iii) and (iv)
dy w2 w2
= x, i.e., dy = xdx
dx g g
which on integration yields
w 2 x2
y= +C
g 2
But as at x = 0, y = 0, C = 0
w2 2
\ y = x ...(v)
2g
Which is the equation of a parabola. So, the surface of revolution is a paraboloid and for
a point on the rim y = h and x = r. So,
h = (wr)2/2g ...(vi)
Here, w = (2p ¥ 2/l) = 4p rad/s
and r = 0.05 m
( 4p )2 ¥ (0.05)2
So, h = = 2 ¥ 10–2 m = 2 cm
2 ¥ 9.8
Alternative solution: According to Bernoulli’s principle
1 2
P+ rv = constant
2
At sides the velocity is higher [as v = rw], so the pressure is lower. Since the pressure at
a given horizontal level must be equal, the liquid rises at the sides to compensate for this
drop in pressure, i.e.,
1 2 v2 w 2r 2
rv = rgh or h = = [as v = rw]
2 2g 2g
which is the desired result, i.e., Eq. (vi)
Problem 1.114 Three particles, each of mass m are situated at the vertices of an equilateral
triangle of side a. The only forces acting on the particles are their mutual gravitational
forces. It is desired that each particle moves in a circle while maintaining the original mutual
separation a. Find the initial velocity that should be given to each particle and also the time
period of the circular motion. (IIT-JEE, 1988)
Solution As the gravitational force between any two particles is F = G mm/a2, the resultant
force on each particle due to the other two
ŝƌĐƵůĂƌDŽƟŽŶപ87

FR = F 2 + F 2 + 2 F 2 cos 60∞

i.e., FR = 3 F = 3 Gm2/a2 ...(i)


So, if the particles were at rest they will move under the action B
of FR acting on each and will meet at the centre O. Now if each m
particle is given a tangential velocity v so that FR acts as centripetal
force, they will move in a circle of radius r = (2/3) a sin 60° = F FR
O F
(a/ 3) maintaining the original mutual separation a. FR
60°
i.e., FR = (mv2/r) A
FR
m
Which in the light of Eq. (i) and above value of r yields F
m C
2 2
Gm mv Gm
3 2
= 3, i.e., v=
a a a Fig. 1.169
So, that time of one revolution
2p r 2p a a a3
T= = = 2p
v 3 Gm 3Gm

Problem 1.115 A mass m is released from the top of a vertical circular track of radius r
with a horizontal speed v0. Calculate the angle q with respect to the vertical where it leaves
contact with the track.
H
Solution The forces acting on the body are its weight mg and v0
R
reaction R as shown in Fig. 1.170. y
So, for circular motion of the body at any position q P
q
mv 2 h
= mg cos q – R q r
mg
r
mv 2 O
or R = mg cos q –
r
Fig. 1.170
The body will leave contact where R Æ 0
mv 2
i.e., mg cos q – =0
r
v2
i.e., cos q = ...(i)
rg
Now, applying conservation of mechanical energy between H and P, we get
1 1
mv 2 = mv02 + mgr (1 – cos q) [as y = r (1 – cos q)]
2 2
or v2 = v02 + 2gr (1 – cos q) ...(ii)
88പWŚLJƐŝĐƐĨŽƌ//dͲ:͗DĞĐŚĂŶŝĐƐ//

Substituting the value of v2 from Eq. (ii) in Eq. (i)


v02
cos q = + 2 (1 – cos q)
rg

È v2 2 ˘
i.e., cos q = Í 0 + ˙
Î 3rg 3 ˚

È v2 2 ˘
or q = cos–1 Í 0 + ˙
Î 3rg 3 ˚

Problem 1.116 A heavy particle hanging from a fixed point by a light inextensible string
of length l is projected horizontally with speed gl . Find the speed of the particle and the
inclination of the string to the vertical at the instant of the motion when the tension in the
string is equal to the weight of the particle.
Solution Let T = mg at angle q as shown in Fig. 1.171.
h = l(1 – cos q) ...(i)
Applying conservation of mechanical energy between T
q B
points A and B, we get
mg cos q
1 h mg sin q
m(u2 – v2) = mgh
2
A u = gl
Here, u2 = gl ...(ii)
Fig. 1.171
and v = speed of particle in position B
\ v2 = u2 – 2 gh ...(iii)
2
mv
Further, T – mg cos q =
l
mv 2
or mg – mg cos q = (T = mg)
l
or v2 = gl (1 – cos q) ...(iv)
2 2
Substituting values of v , u and h from Eqs. (iv), (ii) and (i) Eq. (iii) we get
gl (1 – cos q) = gl – 2gl (1 – cos q)
2
or cos q =
3
Ê 2ˆ
or q = cos–1 Á ˜
Ë 3¯
2
Substituting cos q = in Eq. (iv), we get
3
gl
v=
3
ŝƌĐƵůĂƌDŽƟŽŶപ89

Problem 1.117 A stone tied to a string of length L is whirled in a vertical circle with the
other end of the string at the centre. At a certain instant of time the stone is at it lowest
position and has a speed u. Find the magnitude of the change is its v
velocity as it reaches a position, where the string is horizontal.
(IIT-JEE, 1998)

Solution v = u2 - 2 gh = u2 - 2 gL
h=L
  
|Dv| = |v f - v i|= v 2 + u2 - 2v ◊ u cos 90∞
u
= (u2 - 2 gL) + u2
Fig. 1.172
= 2(u2 - gL) Ans.

Problem 1.118 A small object slides without friction from the height H = 50 cm and then
loops the vertical loop of radius R = 20 cm from which a symmetrical section of angle 2a
has been removed. Find angle a such that after losing contact at A and flying through the
air, the object will reach point B.
Solution Applying the law of conservation of mechanical energy between point C and A:
1
mgH = mvA2 + mgR(1 + cos a)
2
As here H = 0.5 m and R = 0.2 m
So, vA2 = 2g (0.3 = 0.2 cos a) ...(i)
C

B A

2a v
H a
B A
O H aa
(R + R cos a )

(a) (b)

Fig. 1.173

Now, as at A(vA π 0) the path of object from A to B will be that of a projectile, with angle
of projection a, so that
Range = Distance AB
or (vA2 sin 2a)/g = 2R sin a
i.e., vA2 cos a = gR ...(ii)
So, substituting the value of vA2 from Eqs. (i) in (ii)
2g (0.3 – 0.2cos a) cos a = 0.2 g
i.e., 2cos2 a – 3cos a + 1 = 0
or (cos a – 1) (2cos a – 1) = 0
90പWŚLJƐŝĐƐĨŽƌ//dͲ:͗DĞĐŚĂŶŝĐƐ//

i.e., cos a = 1 or cos a = 1/2


or a = 0° or a = 60°
As according to problem a π 0°, so a = 60°.
Problem 1.119 A mass of 2.9 kg suspended from a cord of length 50 cm is at rest. Another
bullet of mass 100 g which is moving horizontally with a velocity of 150 m/s strikes and
sticks to it. (a) What is the tension in the string when it makes an angle of 60° with the
vertical? (b) Will it complete a vertical circle?
H
Solution
(a) Applying the law of conservation of linear momentum to
the collision, the velocity V of the composite mass after
collision will be given by O
T
0.1 ¥ 150 = (2.9 + 0.1) V q P
or V = 5 m/s L=r

Now, to calculate the velocity v of the composite body h


Mg
when it makes an angle of 60° with the vertical we apply L V
Bullet
law of conservation of mechanical energy between L and
P, i.e., Fig. 1.174
1 1
MV 2 = Mv2 + Mgh
2 2
i.e., v2 = V2 – 2gL(1 – cos q) [as h = L(1 – cos q)]
1 È 1˘
or v2 = 52 – 2 ¥ 9.8 ¥ 1-
2 ÍÎ 2 ˙˚
or v2 = 20.1 (m/s)2
Now, as for circular motion of P
(Mv2/L) = T – Mg cos q [as r = L]
3 ¥ (20.1) 1
So, T = + 3 ¥ 9.8 ¥ = 120.6 + 14.7 = 135.3 N
(1/2) 2
(b) For looping the loop
vL ≥ 5gr , i.e., vL ≥ 5 ¥ 9.8 ¥ (1/2)
or vL ≥ 4.93 m/s
But as here the velocity at lowest point V is 5 m/s, the body will be looping the loop.
Problem 1.120 A nail is located at a certain distance vertically below the point of suspension
of a simple pendulum. The pendulum bob is released from a position where the string makes
an angle of 60° with the vertical. Calculate the distance of nail from the point of suspension
such that bob will just perform revolution with the nail as the centre. Assume the length of
the pendulum to be one metre.
Solution If r is the radius of the loop about the nail, for just completing the revolution
vL = 5gr ...(i)
ŝƌĐƵůĂƌDŽƟŽŶപ91

Now, this vL is provided by conversion of potential energy S


of the bob into kinetic energy in moving from A to L, so by
conservation of mechanical energy between A and L L 60°

1 y
mvL2 = mgL (1 – cos q) ...(ii)
2 A
[as h = L(1 – cos q)]
h N
Now, substituting the value of vL from Eqs. (i) in (ii)
r
5gr = 2gL (1 – cos q)
L
2 È 1˘ 1 Fig. 1.175
or r = ¥ 1 Í1 - ˙ = = 0.2 m
5 Î 2˚ 5
So, the distance between point of suspension S and nail N,
y = L – r = 1 – 0.2 – 0.8 m
Problem 1.121 Two different masses are con-
nected to two light and inextensible strings as
shown in the figure. Both masses rotate about a
central fixed point with constant angular speed
of 10 rad s–1 on a smooth horizontal plane. Find R1 T2 M2
T
the ratio of tensions 1 in the strings. T1 M1 M1 = 0.25 kg
T2
M2 = 1.0 kg
Solution Drawing the FBDs for masses M1 R1 = 5 cm
and M2 R2 = 10 cm

T1 T2 R2

T 1 – T 2 = M 1R 1 w 2 T 2 = M 2R 2w 2 T2

T1 - T2 M R 1 1 Fig. 1.176
\ = 1◊ 1 = ◊
T2 M2 R2 4 2
T1 1 9
\ = 1+ =
T2 8 8

Problem 1.122 A small ball of mass 2 ¥ 10–3 kg having a charge of 1mC is suspended by a
string of length 0.8 m. Another identical ball having the same charge is kept at the point of
suspension. Determine the minimum horizontal velocity which should be imparted to the
lower ball so that it can make complete revolution. (IIT-JEE, 2001)
Solution If the ball has to just complete the circle then the tension must vanish at the
topmost point, i.e., T2 = 0.
From Newton’s second law,
q2 mv 2
T2 + mg – = ...(i)
4pe 0 l 2 l
92പWŚLJƐŝĐƐĨŽƌ//dͲ:͗DĞĐŚĂŶŝĐƐ//

At the topmost point T2 = 0, F


v
T2
q2 mv 2 mg
mg – = ...(ii)
4pe 0 l 2 l
q
From energy conservation,
Energy at lowest point = Energy at topmost point T1

1 1 u
mu2 = mv 2 + mg 2l ...(iii) q
2 2 mg
F
v2 = u2 – 4gl ...(iv)
q2 Fig. 1.177
From Eq. (ii), v2 = gl – ...(v)
4pe 0 ml
From Eqs. (iv) and (v),
q2
u = 5 gl -
4pe 0 ml
1/2
Ê 275 ˆ

Ë 8 ˜¯
= 5.86 m/s
Problem 1.123 A block is placed inside a horizontal hollow tube. The tube starts rotating
with one revolution per second about its axis. The angular position of the block at which it
begins to slide is 30° below the horizontal level passing through the centre. Find the radius
of the tube if the coefficient of friction is 0.6. What should be the minimum angular speed
of the cylinder so that the block can reach the highest point of the cylinder?
Solution Forces acting on the block are shown in Fig. 1.178.
The block begins to slip when
w
mN = mg cos 30° ...(i) A
30° mN
From Newton’s second law, N
N – mg sin 30° = mrw2 ...(ii) mg sin 30°
or N = mrw2 + mg sin 30° ...(iii) 30°
mg
or m [mrw2 + mg sin 30°] = mg cos 30° ...(iv) mg cos 30°

mg cos 30∞ - m mg sin 30∞ g cos 30∞ - m g sin 30∞ Fig. 1.178
or r = =
m mw 2 mw 2
On substituting numerical values, we get
r ª 0.25 m
Force acting on block at topmost position are shown in Fig. 1.179. For Newton’s second
law,
N + mg = mrw2
Block will just complete a circle when N = 0; mg = mrw2
ŝƌĐƵůĂƌDŽƟŽŶപ93

i.e., w = g/r

10 N
= = 6.32 rad/sec
0.25 mg

The block can reach the highest point only if able to just cross point A.
For point A, for Eq. (iv),
g
w = Fig. 1.179
mr
10
=
0.6 ¥ 0.25
= 8.16 rad/s
v 0 = 2ag
So, minimum velocity = 8.16 rad/s
Problem 1.124 A heavy particle slides under gravity h = a – a cos q
down the inside of a smooth vertical tube held in vertical q = a (1 – cos q)
a
plane. It starts from the highest point with velocity 2ag
where a is the radius of the circle. Find the angular
position q (as shown in Fig. 1.180) at which the vertical
acceleration of the particle is maximum.
At position q,
Fig. 1.180
v2 = v02 + 2gh
where h = a(1 – cos q)
\ v2 = ( 2 ag )2 + 2ag (1 – cos q)
or v2 = 2ag(2 – cos q) ...(i)
2
mv
N + mg cos q =
a
or N + mg cos q = 2mg(2 – cos q)
or N = mg(4 – 3 cos q)
Net vertical force,
F = N cos q + mg
= mg(4cos q – 3 cos2 q + 1)
dF
This force (or acceleration) will be maximum when =0
dq
or – 4sin q + 6 sin q cos q = 0
So, either
sin q = 0, q = 0°
2 Ê 2ˆ
or cos q = q = cos -1 Á ˜
3 Ë 3¯
q = 0° is unacceptable
Therefore, the desired position is at
Ê 2ˆ
q = cos–1 Á ˜ Ans.
Ë 3¯
94പWŚLJƐŝĐƐĨŽƌ//dͲ:͗DĞĐŚĂŶŝĐƐ//

Problem 1.125 A particle of mass m is moving inside a smooth H


vertical circle of radius r. If it is projected horizontally with velocity P
v0 from its lowest position, find the angle q (as shown in Fig. 1.181 R
at which it will lose contact with the circle.
mg
Solution If the body is at any position f, then for its circular q
motion: O f
(mv2/r) = R – mg cos f ...(i)
Here f = 90° + q [as for leaving the circle 90° < f < 180°]
So, R = (mv2/r) – mg sin q V0

The body will leave contact where R Æ 0 L


2
i.e., v = gr sin q ...(ii) Fig. 1.181
Now, applying conservation of energy between lowest position L and this position P with
h = r(1 + sin q)
1 1
mv02 = mv 2 + mgr(1 + sin q) ...(iii)
2 2
Substituting the value of v2 from Eqs. (ii) in (iii)
v02 = gr sin q + 2gr (1 + sin q)
È v2 2 ˘
or sin q = Í 0 - ˙ ...(iv)
Î 3 gr 3 ˚
Problem 1.126 A particle of mass m moves along a circle of constant radius with radial
acceleration changing with time as ar = ktn where k is constant and n > 1. The power
developed by the net force on the particle varies with time as.
Solution
Concept: Work done by centripetal force is always zero. Only tangential force perform work.
Power is developed by tangential force only.
Given radial acceleration
ar = ktn
v2 = Rktn fi v= Rkt n/2
Ên ˆ
dv n Á - 1˜
= RktË 2 ¯
dt 2
Ên ˆ
dv n Á - 1˜
\ Tangential force = m =m RktË 2 ¯
dt 2
 
\ Power developed = F ◊ v = Ft ¥ v
Ên ˆ
n Á - 1˜
= m RktË 2 ¯ ◊ Rkt n/2
2
mnRk n-1
= t
2
ŝƌĐƵůĂƌDŽƟŽŶപ95

Problem 1.127 A small block with mass m is placed inside an inverted cone that is rotating
about a vertical axis such that the time for one revolution of the cone is T. The walls of the
cone make an angle b with the vertical. The coefficient of static friction between the block
and the cone is m. If the block is to remain at a constant height h above the apex of the cone,
what are the max. and min. values of T?
Solution N b
mN
Concept: Solve problem in reference frame of the block. A b
Ps = mw 2r
h r
centrifugal force acts in this frame. b
When w is minimum the block has tendency to slip down the mg
plane therefore.
Fig. 1.182
Friction acts up the plane
r = h tan b ...(i)
N cos b = mN sin b + mw2r ...(ii)
(ÂFHorizontal = 0)
mN cos b + N sin b = mg
(ÂFvertical = 0)
g (1 - m tan b )
Solving we get w2min =
h tan b ( m + tan b )
2p
For wmin i.e., Tmax (T = )
w min
For wmax friction acts down the plane
g (1 + m tan b )
so replace m by – m w2max =
h tan b (tan b - m )

Problem 1.128 A 50 kg woman is on a large swing (generally seen in


fairs) of radius 9 m that rotates in a vertical circle at 6 rev./min. What is N1
the magnitude of her weight when she has moved halfway up?
Solution The woman experiences three forces: mg, her weight acting N2
mg
vertically downwards; N1, reaction due to her weight; N2, horizontal
N1
reaction which provides the centripetal acceleration. Y
From Newton’s second law, N2
mv 2 mg X
SFx = N2 =
r Fig. 1.183
SFy = N1 – mg = 0
v = (2pr)v (where v is frequency)
= (2p ¥ 9) (6)
= 1.8p m/s
Therefore,
(50)(1.8p )2
N2 = = 178 N
9
N1 = mg = 490 N
96പWŚLJƐŝĐƐĨŽƌ//dͲ:͗DĞĐŚĂŶŝĐƐ//

The magnitude of her weight is the magnitude of the resultant force exerted on her by
the chair.
N = N12 + N 22

= 490 2 + 178 2
= 521 N
Problem 1.129 A uniform electric field of strength 106 V/m is directed vertically downwards.
A particle of mass 0.01 kg and charge 10–6 coulomb is suspended by an inextensible thread
of length 1 m. The particle is displaced slightly from its mean position and released.
Calculate the time period of its oscillation. What minimum velocity should be given to the
particle at rest so that it completes a full circle in a vertical plane without the thread getting
slack? Calculate the maximum and the minimum tensions in the thread in this situation.
(g = 9.8m/s2)
Solution Here the body in addition to force of gravity (= mg) will also experience an electric
force (= qE) and as both are directed vertically downwards so,
F = mg + qE
i.e., mg¢ fi mg + qE
qE 10 - 6 ¥ 106
or g¢ fi g + = 9.8 + = 109.8 m/s2
m 0.01
(a) Now, as for a simple pendulum
l 1
T = 2p = 2p = 0.6s
g¢ 109.8
(b) And for looping the loop
vL ≥ 5 g ¢r
So, (vL)min = 5 ¥ 109.8 ¥ 1 = 23.43 m/s

(c) For looping the loop T > 0


So, Tmin = 0
And as in circular motion
mv 2
+ mg¢ cos q
T=
r
So. T will be maximum when cos q = max = 1
i.e., q = 0° (at lowest position)
mvL2
So, Tmax = + mg ¢ [as at lowest point v = vL]
r
or Tmax = 6mg¢ [as force that just looping the loop vL = 5 g ¢r ]
So, Tmax = 6 ¥ 0.01 ¥ 109.8 = 6.59 N
ŝƌĐƵůĂƌDŽƟŽŶപ97

Problem 1.130 A small body is placed on the top of a smooth sphere of radius R. Then
the sphere is imparted a constant acceleration a0 in the horizontal direction and the body
begins sliding down. Find: (a) the velocity of the body relative to the sphere at the moment
of break-off; (b) the angle q between the vertical and the radius vector drawn from the centre
of the sphere to the break-off point, calculate q for a0 = g.
Solution The situation is shown in Fig. 1.184. m N

(a) Let the body break off at angle q with the vertical. Here, we
have a0
mv 2
mg cos q + ma0 sin q – N = q
R
mv 2 mg
\ N = – mg cos q – ma0 sin q ...(1)
R
When the body breaks-off, N = 0 a0
2
mv
\ = mg cos q + ma0 sin q
R Fig. 1.184
v2 = gR cos q + a0 R sin q ...(2)
From work energy theorem
1
mv2 – mg (R – R cos q) = – ma0R sin q
2
1
mv2 = mgR (1 – cos q) – ma0R sin q
2
or v2 = 2gR (1 – cos q) – 2 a0R sin q ...(3)
Equating Eqs. (2) and (3), we get
gR cos q + a0 R sin q = 2gR – 2gR cos q – 2a0R sin q
2
\ a0 R sin q = g – g cos q ...(4)
3
Substituting the value of a0 sin q from Eq. (4) in Eq. (2), we get

È2 ˘ 2
v2 = gR cos q + R Í g - g cos q ˙ = gR
Î3 ˚ 3
\ v = (2/3) gR ...(5)
This gives the velocity at break off.
(b) From Eq. (4)
a0 2
sin q + cos q =
g 3
2 Ê a0 ˆ
or h sin q + cos q =
3 ÁË where g = h˜¯

or 3h sin q = (2 – 3 cos q)
or (3h sin q)2 = (2 – 3cos q)2
98പWŚLJƐŝĐƐĨŽƌ//dͲ:͗DĞĐŚĂŶŝĐƐ//

Solving we get
9(1 + h2) cos2 q – 12 cos q + (4 – 9h2) = 0

12 ± 144 - 4 ¥ 9(1 + h 2 )( 4 - h 2 )
\ cos q =
2 ¥ 9 ¥ (1 + h 2 )

2 + h (5 + 9h 2 )
or cos q =
3(1 + h 2 )
When a0 = g, then q0 ª 17°.
Problem 1.131 Water of density r flows with a linear speed through a horizontal rubber
tube having the form of a ring of radius R if the diameter of the tube is d << R find the
tension in the rubber tube. (IIT Roorkee, 1984)
Solution Consider a small position of the tube as shown in Fig. 1.185. Centripetal force is
provided by radial component of tension in the tube. So, if T is the tension in the tube

T F T
q

O
(a) (b)

Fig. 1.185

F = 2T sin q
2
v dm v 2 dm
So that = 2T sin q, i.e., T= …(i)
R 2R sin q
But dm = p(d/2)2 (2Rq)r [as m = pr2/r] ...(ii)
Substituting the value of dm from Eqs. (ii) in (i), we get

prv 2q d 2 1 È sin q ˘
T= = prv 2 d 2 Ías lim = 1˙
4 sin q 4 Î q Æ0 q ˚
Problem 1.132 A particle is projected with velocity 20 2 m/s at 45° with the horizontal.
After 1 s find tangential and normal accelerations of the particle. Also, find radius of
curvature of the trajectory at that point. (Take g = 10 m/s2)
  
Solution After 1 s v = u + at = 20 i + 10 j ,
v = 500 m/s = 10 5 m/s

a = -10 j  
a ◊ v - 100
at = a cos q = = = - 2 5 m/s 2 Ans.
v 10 5
ŝƌĐƵůĂƌDŽƟŽŶപ99

an = a 2 - at2 = (10)2 - (2 5 )3

= 80 m/s 2 = 4 5 m/s 2 Ans.


2 2
v (10 5 )
R = = = 25 5 m Ans.
an 4 5

Problem 1.133 A rifle was aimed at the vertical line on the target located precisely in the
northern direction, and then fired. Assuming the air drag to be negligible, find how much
off the line, and in what direction, will the bullet hit the target. The shot was fired in the
horizontal direction at the latitude f = 60°, the bullet velocity v = 900 ms–1, and the distance
from the target equals s = 1.0 km. (JEE ADVANCE LEVEL)
Solution
w 
Concept: In this problem, the earth is taken as non- v
w
inertial reference frame. It is essential to remember that
N
the reference frame fixed to the earth’s surface can be
regarded in many cases as practically inertial. However,
there are some phenomena whose interpretation in inertial
reference is impossible unless its non-inertial nature is
taken into account. This problem is also example of such f
phenomenon. The phenomenon of this kind is associated W E
with the motion of bodies relative to the earth’s surface
and can be explained by the coriolis force.
  
Coriolis force is given by Fcor = 2m[w ¥ v]
S
The position of rifle is shown in Fig. 1.186.
  Fig. 1.186
\ Coriolis force is Fc = – 2m 2m[w ¥ v]

 F
\ Coriolis acceleration is denoted by w c = c
m
 
= - 2(w ¥ v )
\ wc = 2w v sin f
The direction of wc is outward perpendicular to the plane of the paper. So, the deflection
in the direction of wc in the time in which it covers a horizontal distance s is
Ê 1ˆ
s0 = Á ˜ w c t 2
Ë 2¯
\ s0 = (wv sin f)t2
s
But t =
v
Ê ws2 ˆ
\ w0 = Á ˜ sin f = 7 cm where w is the angular velocity of the earth’s rotation.
Ë v ¯
100പWŚLJƐŝĐƐĨŽƌ//dͲ:͗DĞĐŚĂŶŝĐƐ//

Problem 1.134 A horizontal disc rotates with a constant angular velocity w = 6.0 rads–1
about a vertical axis passing through its centre. A small body of mass m = 0.50 kg moves
along a diameter of the disc with a velocity v¢ = 50 cm/s which is constant relative to the
disc. Find the force that the disc exerts on the body at the moment when it is located at the
distance r = 30 cm from the rotation axis.
Solution
Concept: The forces acted by disc on the body are:
(a) Normal reaction, N = mg in upward direction.
(b) Centrifugal force, Fc = mrw2 radially outward
(c) Coriolis force, Fcr = 2mwv¢
These three forces are mutually perpendicular to each other.
\ Resultant force acted by disc on the body is

F = N 2 + Fc2 + Fcr2

= m g 2 + r 2w 4 + 4v ¢ 2w 2 = 8 N

Problem 1.135 A small body of mass m = 0.30 kg starts sliding down from the top of a
smooth sphere of radius R = 1.00 m. The sphere rotates with a constant angular velocity
w = 6.0 rads–1 about a vertical axis passing through its centre. Find the centrifugal force of
inertia and the Coriolis force at the moment when the body breaks off the surface of the
sphere in the reference frame fixed to the sphere. (JEE ADVANCE LEVEL)
Solution The situation is shown in Fig. 1.187.
Ê 2ˆ
According to problem 1.185, at the time of break-off, q0 = cos–1 Á ˜
Ë 3¯
The centrifugal force is given by
r
Fc = mrw2 = mRw2 sin q0 w N=0

= mw2R 1 - cos 2 q 0 r mrw 2


mg cos q
4 q0 R
= mRw2 1 - = mRw 2 5/9 mg sin q
9
After putting the values,
Fc = 8 N
mg
The Coriolis force is
2 Ê 8g ˆ
Fcr = mw 2 R 5 + Á ˜ Rw 2 = 17 N
3 Ë 3¯ Fig. 1.187

Problem 1.136 A train of mass m = 2000 tonnes moves in the latitude f = 60° North. Find:
(a) the magnitude and direction of the lateral force that the train exerts on the rails if it
moves along a meridian with a velocity v = 54 km per hour.
ŝƌĐƵůĂƌDŽƟŽŶപ101

(b) in what direction and with what velocity the train should move for the resultant for
the inertial forces acting on the train in the reference frame fixed to the earth to be
equal to zero. (JEE ADVANCE LEVEL)
Solution N
w r
Concept: The situation of problem is shown in Fig. 1.188. r w
v
(a) The Coriolis force on train is q
 
Fcr = |- 2m(w ¥ v )| = 3.8 kN r
(b) ∵ the resultant force on the train in rotatory frame is q
       W E
Fin = Fre - 2m(w ¥ v ) - m w ¥ (w ¥ r )
 
According to problem, Fre = 0, Fin = 0
  
\ 2w ¥ v = – w2 r
or 2w v sin 90° = w2 R cos f (in scalar) S

\ v = 1/2 wR cos f ª 420 km/hour Fig. 1.188

Problem 1.137 A horizontal smooth rod AB rotates with a constant angular velocity
w = 2.00 rads–1 about a vertical axis passing through its end A. A freely sliding sleeve of mass
m = 0.50 kg moves along the rod from the point A with the initial velocity v0 = 1.00 m/s.
Find the Coriolis force acting on the sleeve (in the reference frame fixed to the rotating rod)
at the moment when the sleeve is located at the distance r = 50 cm from the rotation axis.
r
Solution w

Concept: The situation is shown in Fig. 1.189. rw


w
The resultant velocity of sleeve is v = v02
+ (rw ) 2

90°
\ The Coriolis force is r
  r
Fcr = 2m|- (w ¥ v )|= 2m w v = 2m w (rw )2 + v02 v0

2
Êv ˆ
= 2m w2r 1+ Á 0 ˜
Ë rw ¯
Putting the values, Fcr = 2.8 N Fig. 1.189
Problem 1.138 A horizontal disc of radius R rotates with a constant angular velocity w
about a stationary vertical axis passing through its edge. Along the circumference of the
disc a particle of mass m moves with a velocity that is constant relative to the disc. At the
moment when the particle is at the maximum distance from the rotation axis, the resultant
of the inertial forces Fin acting on the particle in the reference frame fixed to the disc turns
into zero. Find:
(a) the acceleration w of the particle relative to the disc; and
(b) the dependence of Fin on the distance from the rotation axis.
Solution
Concept:
(a) Since the body is rotating about the centre of disc with the velocity v0 = Rw
\ The acceleration of the particle relative to the disc is w¢ = Rw2
102പWŚLJƐŝĐƐĨŽƌ//dͲ:͗DĞĐŚĂŶŝĐƐ//

(b) ∵ (mrw2)2 + F2in = 2mw (wR)2


2
Ê 2R ˆ
\ Fin = mrw2 Á ˜ - 1
Ë r ¯

Problem 1.139 In Fig. 1.190, ABCDE is a channel in the A


vertical plane, part BCDE being circular with radius r. A ball
is released from A and slides without friction and without D
E
rolling. Show that it will complete the loop path if h is greater h r
r
than 5 r/2. C
r
Solution Let m be the mass of the ball. When the ball comes
down to B, it loses its potential energy mgh which is converted B
into kinetic energy. Let vB, be the velocity of the ball at B. Fig. 1.190
1
Then, mgh =mvB2 ...(1)
2
The ball now rises to a point D, where its potential energy is mg(h – 2 r). If vD is the
velocity of the ball at D, then
1 2
mg(h – 2r) = mvD ...(2)
2
Now to complete the circular path, it is necessary that the centrifugal force acting upwards
at point D should be equal or greater than the force mg acting downwards. Therefore,
2
mvD
≥ mg
r
or vD2 ≥ rg ...(3)
From Eqs. (2) vD2 = 2g (h – 2 r)
\ 2g(h – 2r) ≥ rg
5
h ≥ r
2
Problem 1.140 Find the maximum speed at which a car can turn round a curve of 30 metre-
radius on a level road if the coefficient of friction between the tyres and the road is 0.4.

Solution Let w = Mg be the weight of the car


Friction force = 0.4 w
Mv 2 wv 2
Centripetal force = =
r gr
wv 2
0.4 w =
gr
v = 0.4 ¥ g ¥ r = 0.4 ¥ 9.8 ¥ 30 = 117.6
2

v = 10.84 m/sec.
ŝƌĐƵůĂƌDŽƟŽŶപ103

Problem 1.141 At the equator a stationary (relative to the y



earth) body falls down from the height h = 500 m. Assuming
the air drag to be negligible, find how much off the vertical,
and in what direction, the body will deviate when it hits the î
ground. (ADVANCE LEVEL)
Solution Let v = velocity of body after time t. x¢
q
x
h = vertical fall of body in time t.
 
v = - vk (downward direction)

Here w = w cos q i + w sin q j
where q is the angle of latitude.
\ Coriolis acceleration y¢
  
w cr = - 2w ¥ v = 2w v cos q i Fig. 1.191
v = gt \ wcr = 2wgt cos q
dx x t
vx = wt = wgt2 cos q or
dt
= wgt2 cos q or Ú0 dx = Ú0 w gt 2 cos q dt
w gt 3 cos q 1 2 2h
\ x = But h = gt \ t =
3 2 g
1/2
Ê 8ˆ
After solving, x = Á ˜ h 3/2 cosq
Ë 9g ¯
1/2
Ê 8ˆ 2 Ê 2h ˆ
For equator, q = 0, \ x = Á ˜ w h 3/2 = w h Á ˜
Ë 9g ¯ 3 Ë g¯

Problem 1.142 Determine the speed with which the earth would have to rotate on it axis so
that a person on the equator would weight (3/5)th as much as present. Take the equatorial
radius as 6400 km.
Solution Let v be the speed of earth’s rotation.
We know that W = mg
3 mv 2
Hence, W = mg –
5 r
3 mv 2
or mg = mg -
5 r
2 mv 2 2 gr
\ mg = or v2 =
3 r 5
2 ¥ 9.8 ¥ (6400 ¥ 10 3 )
Now, v2 =
5
Solving, we get v = 5 ¥ 109 m/sec

Ê 2g ˆ 4
w= ÁË ˜¯ = 7.8 ¥ 10 radian/sec.
5r
104പWŚLJƐŝĐƐĨŽƌ//dͲ:͗DĞĐŚĂŶŝĐƐ//

DAILY PRACTICE PROBLEMS FOR JEE MAIN AND ADVANCE

‡˜‡Žͳ౨(Only one option is correct) 6. If a cyclist moving with a speed of


4.9 m/s on a level road take a sharp
1. A car is moving on a circular path
circular turn of radius 4 m, then
and takes a turn. If R1 and R2 be the
coefficient of friction between the cycle
reaction on the inner and outer wheels
tyres and road is
respectively, then (VITEEE, 2011)
(a) 0.41 (b) 0.51
(a) R1 = R2 (b) R1 < R2
(c) 0.61 (d) 0.71
(c) R1 > R2 (d) R1 ≥ R2
7. What is the value of linear velocity, if
2. A car sometimes over turns while   and r = 5i - 6j + 6 k
w = 3i - 4j + k 
taking a turn. When it overturns, it is
(a) the inner wheel leaves the ground 
(a) 6i + 2j - 3k
first 
(b) - 18i - 13j + 2k
(b) the outer wheel leaves the ground
first

(c) 4i - 13j + 6 k
(c) both the wheels leave the ground 
(d) 6i - 2j + 8 k
simultaneously 8. A small block is shot into each of the
(d) either wheel leaves the ground four tracks as shown below. Each of
first the track rises to the same height. The
3. A particle P is moving in a circle of speed with which the block enters the
radius a with a uniform speed v. C track is the same in all cases. At the
is the centre of the circle and AB is a highest point of the track, the normal
diameter. When passing through B the reaction is maximum in (IIT, 2001 S)
angular velocity of P about A and C are
in the ratio (WBJEE, 2013) (a) (b)
v v
(a) 1:1 (b) 1:2
(c) 2:1 (d) 4:1
4. A stone of mass of 16 kg is attached (c) (d)
to a string 144 m long and is whirled v v
in a horizontal circle. The maximum
tension the string can withstand is 9. A wheel is subjected to uniform
16 newtons. The maximum velocity angular acceleration about its axis.
of revolution that can be given to the Initially, its angular velocity is zero. In
stone without breaking it, will be the first 2s, it rotates through an angle
(a) 200 m/s (b) 16 m/s q1. In the next 2s, it rotates through an
(c) 14 m/s (d) 12 m/s q2
5. The length of second’s hand in a watch additional angle q2. The ratio of is
q1
is 1 cm. The change in velocity of its tip (a) 1 (b) 2
in 15 seconds is (c) 3 (d) 5
p 10. If the equation for the displacement of
(a) zero (b) cm/s
30 2 a particle moving on a circular path
is given by (q) = 2t3 + 0.5, where q
p p 2 is in radian and t in second, then the
(c) cm/s (d) cm/s
30 30 angular velocity of the particle after 2 s
from its start is
ŝƌĐƵůĂƌDŽƟŽŶപ105

(a) 8 rad/s (b) 12 rad/s (c) the centripetal force disappears


(c) 24 rad/s (d) 36 rad/s (d) centripetal force will be doubled
11. A mass is supported on a frictionless 15. A simple pendulum is oscillating with-
horizontal surface. It is attached to a out damping. When the displacement
string and rotates about a fixed centre of the bob is less than maximum its
at an angular velocity w0. If the length acceleration vector a is correctly shown
of the string and angular velocity are in (IIT-JEE, 2002)
double, the tension in the string which
was initially T0 is now (a) (b)
T
(a) T0 (b) 0
2
(c) 4 T0 (d) 8 T0
12. Which of the following statements is
false for a particle moving in a circle (c) (d)
with a constant angular speed?
a
(a) the velocity vector is tangent to
the circle 16. A rod of length 1 m rotates about the
(b) the acceleration vector is tangent z-axis passing through the point O in
to the circle the xy-plane with an angular velocity
(c) the acceleration vector points to of w = 10 (rad/s) + 5 (rad/s2)t, in the
the centre of the circle counterclockwise direction, and O is
(d) the velocity and acceleration at rest the velocity and acceleration of
vectors are perpendicular to each point A, at t = 0, are:
other y
13. A particle of mass m is moving in a Z
horizontal circle of radius R with
uniform speed v. When it moves from A O X
one point to a diametrically opposite

point, its
(a) momentum does not change (a) – 10 j m/s and (5 j – 100 i ) m/s2
(b) momentum changes by 2 mv (b) 10 j m/s and (5 j – 100 i ) m/s2
1 (c) – 10 k m/s and (5 k  – 100 j) m/s2
(c) kinetic energy changes by mv2
2 (d) – 10 j m/s and (– 5 j + 100 i ) m/s2
(d) kinetic energy changes by mv2 17. A vehicle is moving with a velocity v
14. A body is revolving with a constant on a curved road of width b and radius
speed along a circular path. If the of curvature R. For counteracting the
direction of its velocity is reversed, centrifugal force on the vehicle, the
keeping speed unchanged, then difference in elevation required in
(a) the centripetal force does not between the outer and inner edge of
suffer any change in magnitude the road is
and direction both
v 2b vb
(b) the centripetal force does not (a) (b)
suffer any change in magnitude Rg Rg
but its direction is reversed vb 2 vb
(c) (d)
Rg R2 g
106പWŚLJƐŝĐƐĨŽƌ//dͲ:͗DĞĐŚĂŶŝĐƐ//

18. One end of a string of length l is 23. A cyclist riding the bicycle at a speed
connected to a particle of mass m and of 14 3 m/s–1 takes a turn around a
the other to a small peg on a smooth circular road of radius 20 3 m without
horizontal table. If the particle moves skidding. Given g = 9.8 ms–2, what is
in a circle with speed v, the net force his inclination to the vertical
on the particle (directed towards the (a) 30° (b) 90°
centre) is (BITSAT, 2012) (c) 45° (d) 60°
mv 2 24. A car of mass 800 kg moves on a circular
(a) T (b) T –
l track of radius 40 m. If the coefficient of
mv 2 friction is 0.5, then maximum velocity
(c) T + (d) zero
l with which the car can move is
19. An object moves at a constant speed (a) 7 m/s (b) 14 m/s
along a circular path in a horizontal (c) 8 m/s (d) 12 m/s
xy plane, with the centre at the origin. 25. For a particle in a non-uniform accel-
What the object is at x = – 2 m, its veloc- eration circular motion
ity is – (4 m/s) j . What is the object’s (a) velocity is radial and acceleration
acceleration when it is y = 2 m is transverse only
(BITSAT, 2013) (b) velocity is transverse and accel-
2 
eration is radial only
(a) – (8 m/s )j (b) –(8 m/s2)i (c) velocity is radial and acceleration
(c) –(4 m/s2)j (d) (4 m/s2)i has both radial and transverse
components
20. A motor cyclist moving with a velocity
(d) velocity is transverse and accel-
of 72 km/h on a flat road takes a turn
eration has both radial and trans-
on the road at a point where the radius
verse components
of curvature of the road is 20 m. The
26. A point mass m is suspended from
acceleration due to gravity is 10 m/
a light thread of length l, fixed at O,
sec2. In order to avoid skidding, he
is whirled in a horizontal circle at a
must not bend with respect to the
constant speed as shown. From your
vertical plane by an angle greater than
point of view, stationary with respect
(a) q = tan–1 6 (b) q = tan–1 2
to the mass, the forces on the mass are
(c) q = tan–1 25.92 (d) q = tan–1 4
21. A particle describes a horizontal circle O
in a conical funnel whose inner surface
is smooth with speed of 0.5 m/s. What
is the height of the plane of circle from l
vertex of the funnel
(a) 0.25 cm (b) 2 cm m
(c) 4 cm (d) 2.5 cm
22. The angular speed of second’s needle
T T
in a mechanical watch is
p (a) F (b)
(a) rad/s (b) 2p rad/s
30
60 W W
(c) p rad/s (d) rad/s
p
ŝƌĐƵůĂƌDŽƟŽŶപ107

T T (a) N1 < N2 (b) N1 > N2


(c) N1 = N2 (d) none
(c) F (d) F F
32. A particle is moving in a vertical circle.
The tensions in the string when passing
W W
through two positions at angles 30°
27. A particle moves in a circular path and 60° from vertical (lowest position)
with decreasing speed. Choose the are T1 and T2 respectively.
correct statement? (BITSAT, 2004)
(a) T1 = T2
(a) angular momentum remains con-
(b) T2 > T1
stant
 (c) T1 > T2
(b) acceleration (a ) is towards the
centre (d) tension in the string always re-
(c) particle moves in a spring path mains the same
with decreasing radius
(d) the direction of angular momen- ‡˜‡Žʹ౨(Only one option is correct)
tum remains constant 1. A toy cart is tied to the end of an
28. A 2 kg stone at the end of a string 1 m unstreched string of length a. When
long is whirled in a vertical circle at a revolved, the toy cart moves in a
constant speed. The speed of the stone horizontal circle of radius 2a with time
is 4 m/s. The tension in the string will period T. Now the toy cart is speeded
be 52 N, when the stone is up until in a horizontal circle of radius
(a) at the top of the circle 3a with period of T. If Hooke’s law is
(b) at the bottom of the circle obeyed throughout then
(c) halfway down 3
(a) T¢ = T (b) T¢ = T
(d) none of the above 2
29. A can filled with water is revolved in
2 3
a vertical circle of radius 4 m and the (c) T¢ = T (d) T¢ = T
water just does not fall down. The time 3 2
period of revolution will be 2. A pendulum consisting of a sphere of
(a) 1 sec (b) 10 sec mass m, suspended by an inextensible
(c) 8 sec (d) 4 sec and massless string of length l, is made
30. A wheel is subjected to uniform angular to swing in a vertical plane. If the
acceleration about its axis. Initially, its breaking strength of the string is 2 mg
angular velocity is zero. In the first 2 then the maximum angular amplitude
sec, it rotates through an angle q1. In of the displacement from the vertical
the next 2 sec, it rotates through an can be
additional angle q2. The ratio of q2/q1 (a) 0° (b) 30°
is (c) 60° (d) 90°
(a) 1 (b) 2 3. Two masse M and m hang at the
(c) 3 (d) 5 ends of a string that passes through
31. A car is moving on a curved road with a smooth tube as shown. The mass m
constant speed. If N1 and N2 are the moves in a circular path which lies in
reactions at A and B then: a horizontal plane. The length of the
string from m to the top of the tube is
l and q is the angle this length makes
with the vertical. What should be the
A B
108പWŚLJƐŝĐƐĨŽƌ//dͲ:͗DĞĐŚĂŶŝĐƐ//

frequency of revolution of m so that M


remains stationary? q q
1 Mg
(a) A B
2p ml
q
1 Mg
(b) l
2p ml cos q (a) sin2q (b) cos2q
(c) tan2q (d) 1:1
1 ml cos q M
(c) 6. For the arrangement in the figure, the
2p Mg
particle M1 attached to one end of a
1 ml string which moves on a horizontal
(d) M
2p Mg l
table in a circle of radius = (where
4. Two identical particles are attached at 2
the ends of a light string which passes l is the length of the string) with
through a hole at the centre of a table. constant angular speed w. The other
One of the particles is made to move end of the string attached to mass M2
in a circle on the table with angular which rests on a vertical rod. When the
velocity w1 and the other is made to rod collapses, the acceleration of mass
move in a horizontal circle as a contact at that instant is
pendulum with angular velocity w2. If (a) g
l1 and l2 are the lengths of the string
w 2l
over and under the table, then in order (b)
that particle under the table neither 2
moves down nor moves up the ratio 2 M2 g - M1lw 2
l1 (c)
is 2( M1 + M2 )
l2
w1 w2 M2 g + M1lw 2
(a) (b) (d)
w2 w1 M1 + M2
w 12 w 22
(c) (d) M1
w 22 w 12
l1
m
M2
l2
Rod

m
7. A car enters a curved road in the form
of a quarter of a circle, the path length
5. A ball is held at rest in position A being 200 m. Its speed at the entrance is
in figure by two light cords. The 18 km/h but when it leaves, it increases
horizontal cord is cut and the ball to 54 km/h. If the car is travelling with
starts swinging as a pendulum. The constant acceleration along the curve,
ratio of the tension in the supporting the acceleration when the car leaves
cord in position B, to that in position the curved road is
A is
ŝƌĐƵůĂƌDŽƟŽŶപ109

(a) 50 i – 1.5 j m/s2


54 km/h

(b) – 50 i – 1.5 j m/s2


(c) 10 i – 1.5 j m/s2
(d) 1.5 i – 50 j m/s2
11. A car moving around a circular track
18 km/h
of 200 m radius is increasing its speed
of 2 m/s every second. The speed of
2
(a) 0.92 m/s (b) 0.54 m/s2 the car when the acceleration of the car
2
(c) 1.84 m/s (d) 2.76 m/s2 is 4 m/s2 is
8. A rotating system as shown in figure (a) 10 m/s (b) 16.25 m/s
has a mass of 0.2 kg hanging from (c) 24.64 m/s (d) 26.32 m/s
a cord of length 0.2 m. The length 12. Two particles of equal mass are
of the horizontal arm of the rotating connected to a rope AB of negligible
apparatus is 0.1 m. For the cord to mass such that one is at end A and
make an angle 45° with the vertical, other dividing the length of rope in
the number of revolutions per second the ratio 1:2 from B. The rope is rotated
that the system should make will be about end B in a horizontal plane.
Ratio of tensions in the smaller part to
0.1m
the other is (ignore effect of gravity)
0.2 m (a) 4:3 (b) 1:4
(c) 1:2 (d) 1:3
13. A roller coaster is designed such that
0.2 kg
riders experience “weightlessness” as
they go round the top of a hill whose
radius of curvature is 20 m. The speed
(a) 2.6 (b) 4
of the car at the top of the hill is
(c) 6.68 (d) 1.03
between
9. A wheel has constant angular accelera-
(a) 16 m/s and 17 m/s
tion of 3.0 rad/s2. During a certain 4.0
(b) 13 m/s and 14 m/s
interval, it turns through an angle of
(c) 14 m/s and 15 m/s
120 rad. Assuming that at t = 0, angu-
(d) 15 m/s and 16 m/s
lar speed w0 = 3 rad/s, how long is its
14. In the given figure, y
motion at the start of this 4.0 s interval
a smooth parabolic
(a) 7 s (b) 9 s w
wire track lies in the
(c) 4 s (d) 10 s
xy-plane (vertical).
10. A particle is mov- y
The shape of track is
ing along a circu- x
defined by the equa- O (0, 0)
lar path in the xy
tion y = x2. A ring of
plane (see figure).
2m x mass m, which can slide freely on the
When it crosses
wire track, is placed at the position A
the x-axis, it has an
10 m/s (1, 1). The track is rotated with con-
acceleration along
2 stant angular speed w such there is no
the path of 1.5 m/s , and is moving
relative slipping between the ring and
with a speed of 10 m/s in the negative
the track. The value of w is
y-direction. The total acceleration of
the particle is
110പWŚLJƐŝĐƐĨŽƌ//dͲ:͗DĞĐŚĂŶŝĐƐ//

(a) g/2 (b) g 2. A simple pendulum is vibrating with


an angular amplitude of 90° as shown
(c) 2g (d) 2 g in figure, then
15. A thin but rigid semicircular wire
frame of radius r is hinged at O and q
can rotate in its own vertical plane. A
smooth peg P starts from O and moves
horizontally with constant speed v0,
lifting the frame upward as shown in
figure. (a) q = 0, acceleration directed down-
ward
P (b) q = 0, acceleration directed up-
O v0
60°
ward
(c) q = 90° acceleration directed
Find the angular velocity w of the downward
frame when its diameter makes an Ê 1 ˆ
(d) q = cos–1 Á , acceleration di-
angle of 60° with the vertical Ë 3 ˜¯
(a) v0/r (b) v0/2 r rected horizontal
(c) 2 v0/r (d) none 3. A body of mass 1 kg is attached to
16. In the figure shown a smooth ring is an inextensible string of length 1 m, is
connected to rod AB, while rod CD made to rotate in vertical circle about
passes through the ring. At the given the free end. When the body is at its
instant angular velocity of rod AB highest position, the tension in the
about hinge A is rad/s and AC = CB. string is 10 N. Then
Instantaneous angular velocity of rod (a) the tension in the string remains
CD about hinge C is same for any position of the body
D (b) its velocity at highest position is
B
20 m/s
w = 1 rad/s (c) its velocity at its lowest position
is 50 m/s
(d) tension in the string at the lowest
30° A
C position of the body is 70 N
(a) 1 rad/s (b) 1/2 rad/s 4. A simple pendulum of length L and
mass (bob) m is oscillating in a verti-
(c) 3 /2 rad/s (d) 3/2 rad/s
cal plane about a vertical line between
angular limits – f and + f. For dis-
Multiple Correct Options placement q{[q] < f}, the tension in the
1. A body is moving with a constant string and velocity of bob are T and
speed v in a circle of radius r. v respectively. The following relations
(a) its tangential acceleration is zero hold good under the above condition
(b) its normal acceleration is zero
(a) T cos q = mg
v2 mv 2
(c) its normal acceleration is (b) T – mg cos q =
r L
v2
(d) its total acceleration is
r
ŝƌĐƵůĂƌDŽƟŽŶപ111

(c) tangential acceleration = g sin q 8. Particle A moves with 4 m/s along


(d) T = mg cos q positive y-axis and particle B in a
5. A ball tied to a string is swung in a circle x2 + y2 = 4 (anticlockwise) with
vertical circle. The physical quantities constant angular velocity w = 2 rad/s.
which do not remain constant At time t = 0 particle is at (2 m, 0). Then
(a) speed of ball (a) magnitude of relative velocity
(b) centripetal force between them at time t is (8 sin t)
(c) tension in string (b) magnitude of relative velocity
(d) earth’s pull on ball between them is maximum at t =
6. A small sphere of mass m suspended p
by a thread is first taken aside so that s
4
the thread forms the right angle with
the vertical and then released, then (c) magnitude of relative velocity
(a) total acceleration of sphere as a between them is maximum at t =
p
function of e is g 1 + 3 cos 2 q s
2
(b) thread tension as a function of e is
(d) magnitude of relative velocity
T = 3 mg cos q
between them at time t is (8 sin 2t)
(c) the angle q between the thread 9. Speed of a particle moving in a circle
and the vertical at the moment varies with time as, v = 2t. Then
when the total acceleration vector (a) angle between velocity vector and
of the sphere is directed horizon- acceleration vector is increasing
tally is cos–1 1/ 3 with time
(d) the thread tension at the moment (b) at is constant while ar is increasing
when the vertical component of with time
the sphere’s velocity is maximum (c) at is decreasing but ar is increasing
will be mg (d) both (a) and (b) are wrong
7. A car of mass m is moving on a As ar is increasing, q will also increase.
horizontal circular path of radius 10. A uniform rod of mass m and length 2l
r. At an instant its speed is v and is lies on a smooth horizontal surface. A
increasing at a rate a particle of same mass m is connected
(a) the acceleration of the car is to- to string of length l, whose other end
wards the centre of the path is connected to rod. Initially, the string
(b) the magnitude of the frictional is taut and both rod and string lies in
force on the car is greater than same horizontal plane with 90° angle
between them. If the particle is given
mv 2
initially velocity v0 perpendicular to
r string, then just after giving velocity
(c) the friction coefficient between v0 to particle
the ground and the car is not less
A B
than a/g
(d) the friction coefficient between
the ground and the car is m = tan–1 l

Êv 2ˆ v0
m
Á rg ˜
Ë ¯
112പWŚLJƐŝĐƐĨŽƌ//dͲ:͗DĞĐŚĂŶŝĐƐ//

(a) linear acceleration of the centre of one end and the block is at a distance L from
mass of rod will be v02/4l the fixed end. The ruler is rotated about the
(b) angular acceleration of rod will be fixed end in the horizontal plane through
6v02/5l the fixed end.
(c) tension in string will be mv02/5l 1. The maximum angular speed be for
(d) angular velocity of rod will be which the block does not slip
v0/2l (IIT-JEE, 2009)
11. A particle of mass m is moving in hori-
zontal circle inside a smooth inverted (a) gL (b) mg/L
fixed vertical cone above height h from (c) mgL/2 (d) none
apex. Angle of cone is q then
2. If the angular speed of the ruler is uni-
(a) normal force on particle by surface
formly increased from zero at an angu-
of cone is mg cos q
lar acceleration a, the angular speed,
(b) normal force on particle by surface
at which the block will slip is
of cone is mg cosec q
(c) time period of revolution of (a) mgl
particle increase if q increase 1/4
keeping h constant ÈÊ m g ˆ 2 ˘
(b) ÍÁ ˜ -a2˙
(d) time period of revolution increase ÍÎË L ¯ ˙˚
if h increase keeping q fixed
1/2
ÈÊ m g ˆ 2 ˘
(c) ÍÁ ˜ -a2˙
ÍÎË L ¯ ˙˚
q 1/2
h ÈÊ m g ˆ 2 ˘
(d) ÍÁ ˜ +a2˙
ÍÎË L ¯ ˙˚
12. The coordinates of a particle moving
in a plane are given by x(r) = a cos (pt) Passage for Q: 3–5
and y(t) = b sin (pt) where a, b(< a) and
The bob of mass 1 kg of a pendulum of length
p are positive constants of appropriate
1 m at rest is given a sharp hit to impart a
dimensions. Then
horizontal velocity 10 m/s. (g = 10 m/s2)
(a) the path of the particle is an ellipse
(b) the velocity and acceleration of
the particle are normal to each
other at t = p/(2p)
l=1m
(c) the acceleration of the particle is
always directed towards a focus u = 10 m/s
(d) the distance travelled by the
particle in time interval t = 0 to t 3. The tension in the string when, it is
= p/(2p) is a horizontal
(a) 20 N (b) 40 N
Passages and Matrix Matching
(c) 60 N (d) 80 N
Passage for Q: 1–2 4. The tension in the string, when the bob
A block of mass m is kept on a horizontal is its highest position
ruler. The coefficient of friction between the (a) 20 N (b) 30 N
ruler and the block is m. The ruler is fixed at (c) 40 N (d) 50 N
ŝƌĐƵůĂƌDŽƟŽŶപ113

5. The tension in the string, when string (a) 9.8 ¥ 10–3 m/s2
makes 120° from the vertical is (b) – 9.8 ¥ 10–3 m/s2
(c) 4.9 ¥ 10–3 m/s2
(d) none of these

120° Passage for Q: 9–11


A pendulum bob can swing along a circular
path on a smooth inclined plane, as shown in
figure, where m = 1.2 kg, l = 0.75 m, q = 37°.
At the lowest point of the circle the tension
(a) 40 N (b) 55 N in the string T = 110 N. Determine
(c) 65 N (d) 75 N

Passage for Q: 6–8


l
A hemispherical bowl of radius R = 0.1 m is v0
rotating about its own axis (which is vertical) m
q
with an angular velocity w. A particle of mass
10–2 kg on the frictionless inner surface of
9. The speed of the bob at the lowest
the bowl is also rotating with same w. The
point is
particle is at a height h from the bottom of
(a) 4.04 m/s (b) 8.02 m/s
the bowl.
(c) 10.20 m/s (d) 12.02 m/s
10. The speed of the bob at the height
R point on the circle is
(a) 6.83 m/s (b) 10.20 m/s
h
(c) 12.02 m/s (d) zero
6. The relation between h and w is 11. The tension in the string when bob is
at the highest point is
w2 (a) 40.80 N (b) 60.50 N
(a) h = (b) h = R/2
g (c) 64.32 N (d) 67.56 N
g
(c) h = R - (d) none of Passage for Q: 12–14
w2 these A weightless rod of length l with a small
7. The minimum value of w which is load of mass m at the end is hinged at point
needed in order to have a non-zero A and occupies a strictly vertical position,
value of h touching a body of mass M. A slight jerk sets
the system in motion. Friction is neglected
(a) g/R (b) g/2R
and l = 0.2 m.
(c) g/3R (d) none
m
8. It is desired to measure g using this
setup, by measuring h accurately. l M
Assuming that r and w are known
precisely, and that the least count in
the measurement of h is 10–4 m. The 12. At any instant before separation,
minimum possible error Dg in the the angle made by the rod with
measured value of g is (g = 9.8 m/s2) horizontal is q. If acceleration of M is
114പWŚLJƐŝĐƐĨŽƌ//dͲ:͗DĞĐŚĂŶŝĐƐ//

a and its velocity is v, then tangential M


acceleration of the small mass m is 13. The ratio for the two bodies (m and
m p
M) to separate at the instant q = is
(a) zero 6
1 Ê v2 ˆ (a) 2 (b) 4
(b) at = Á a + cos q ˜
sin q Ë l ¯ 1
(c) 1 (d)
1 Ê v ˆ
2 2
(c) at = Á a+
sin q Ë l cos q ˜¯ 14. Velocity of the mass m at the moment
of separation is
1 Ê v 2 cos q ˆ (a) 2 m/s (b) 1 m/s
(d) at = a +
sin q ÁË ˜
l sin 2 q ¯ (c) 0.25 m/s (d) 0.5 m/s

Matrix Matching
15. A particle moves in a circle of radius 1 m at a speed of, v = 2t, where v is in m/s and
t in second. Then match the columns:
Column I Column II
(a) radial acceleration of the particle at t = 1 s (p) 1 m/s2
(b) tangential acceleration of the particle at t = 1 s (q) 2 m/s2
(c) tangential acceleration of the particle at t = 2 s (r) 3 m/s2
(d) total acceleration at t = 1 s (s) 4 m/s2
(t) 2 5 m/s2
16. A small block of mass 1 kg is connected to an inextensible string of length 1 m and
made to rotate in a vertical circle about the free end of the string. The tension at the
highest point of the block is 10 N. Match the two columns (g = 10 m/s2)
A

1m
C

Column I Column II
(a) speed of block at A (p) 60 m/s
(b) speed of block at B (q) 20 m/s
(c) tension in string at B (r) 40 N
(d) tension in string at C (s) 70 N
17. A particle of 500 gm mass moves along a horizontal circle of radius 16 m such that
normal acceleration of the particle varies with time as an = 9t2
ŝƌĐƵůĂƌDŽƟŽŶപ115

Column I Column II
(a) Tangential force on particle at t = 1 second (in newton) (p) 72
(b) Total force on particle at t = 1 second (in newton) (q) 36
(c) Power delivered by total force at t = 1 sec (in watt) (r) 7.5
(d) Average power developed by total force over first one
second (in watt) (s) 6
18. Two light rids of length 1 m each are hinged together as shown in figure. Rod AB
makes an angle q with vertical while rod BC makes an angle f with horizontal. End C
of rod BC remains in contact with horizontal. Rod AB is rotated with constant angular
velocity w = 1 rad/s in clockwise direction. At the instant when q = 30° and f = 30°.
Match the variables in Column I with values in Column II
A

q w
B

f C

Column I Column II

(3 3 + 1)
(a) Angular velocity of rod BC in rad/s (p)
3 3

( 3 - 1)
(b) Velocity of block D in m/s (q)
6
1
(c) Magnitude of angular acceleration of rod BC in rad/s2 (r)
3
(d) Acceleration of point B in m/s2 (s) 1
(t) 0

Answers Key
Level 1
1. (b) 2. (a) 3. (b) 4. (d) 5. (d) 6. (c) 7. (b) 8. (a)
9. (c) 10. (c) 11. (d) 12. (b) 13. (b) 14. (a) 15. (c) 16. (d)
17. (a) 18. (a) 19. (a) 20. (b) 21. (d) 22. (a) 23. (d) 24. (b)
25. (d) 26. (c) 27. (d) 28. (b) 29. (d) 30. (c) 31. (a) 32. (c)
Level 2
1. (d) 2. (c) 3. (a) 4. (d) 5. (b) 6. (c) 7. (c) 8. (d)
9. (a) 10. (b) 11. (d) 12. (a) 13. (c) 14. (c) 15. (a) 16. (d)
116പWŚLJƐŝĐƐĨŽƌ//dͲ:͗DĞĐŚĂŶŝĐƐ//

Multiple Correct Options


1. (a, c, d) 2. (b, c, d) 3. (b, d) 4. (b, c)
5. (a, b, c) 6. (a, b, c) 7. (b, c) 8. (a, c)
9. (a, b) 10. (c) 11. (b, c, d) 12. (a, b)
Passages and Matrix Matching
1. (b) 2. (b) 3. (d) 4. (d) 5. (c) 6. (c) 7. (a) 8. (b)
9. (b) 10. (a) 11. (d) 12. (d) 13. (b) 14. (b)
15. AÆs; BÆq; CÆq; DÆt 16. AÆq; BÆp; CÆs; DÆr
17. AÆs; BÆr; CÆp; DÆq 18. AÆr; BÆr; CÆp; DÆs

SOLUTION AND HINTS

Level 1
1. (b) It has been calculated in the theory of the chapter, i.e.,

mg Ê v2 h ˆ
R1 = 1 -
2 ÁË rga ˜¯

mg Ê v2 h ˆ
and R2 = 1 + , so R1 < R2
2 ÁË rga ˜¯

2. (a) In overturning, inner wheels leave the ground first (see figure).

3. (b)
A

q
c
2q

From the geometry of the figure, the angle traverses about A and C are 2q
respectively. So
q 2q
wA = and wC = = 2 wA.
t t
ŝƌĐƵůĂƌDŽƟŽŶപ117

mv 2
4. (d) T =
r
16v 2
or 16 = fi v = 12 m/s.
144

5. (d) Dv = 2v = 2 w r

Ê 2p ˆ p 2
= 2Á ˜ ¥1= cm/s
Ë 60 ¯ 30
6. (c) v = mrg ,
v2 4.92
\ m = = = 0.61
rg 4 ¥ 9.8

  
7. (b) v = w ¥ r = (3i - 4j + k
 ) ¥ (5i - 6j + 6k
)

i j k

= 3 - 4 1 = i [(– 4) ¥ 6 – 1 ¥ (– 6)] + j [5 ¥ 1 – 3 ¥ 6] + k
 [3 ¥ (– 6) – (– 4) ¥ 5]
5 -6 6

= - 18i - 13j + 2k

mv 2
8. (a) N + mg =
R N mg
2
mv
\ N = – mg.
R
As R is least in (a), so N is greatest in this case.
1
9. (c) q1 = a(2)2
2
1
and q1 + q2 = a(4)2 = 4q1,
2
\ q2 = 3q1
dq d(2t 3 + 0.5)
10. (c) w = =
dt dt
= 3 ¥ 2t2 = 6(2)2 = 24 rad/s
11. (d) T0 = mw2r,
and T = m(2w)2 ¥ 2r = 8T0.
12. (b) For a particle with constant speed, the acceleration vector tends towards the centre
of the path.
118പWŚLJƐŝĐƐĨŽƌ//dͲ:͗DĞĐŚĂŶŝĐƐ//

    
13. (b) DP = P f - Pi = - mv - mv
\ DP = 2 mv
r
v

r
-v

mv 2
14. (a) Centripetal force, Fx = . So Fx remains as such, v is either positive or negative.
r
15. (c) At this stage, it has two accelerations, an and at, so its resultant will be like as shown
in figure.

an

at

16. (d) At t = 0, w = 10(– j ) = – 10 j rad/s
dw d(10 + 5t)
a = = = 5 rad/s2
dt dt
\ at = ar = 5 ¥ 1 = 5 m/s2
an = w2r = 102 ¥ 1 = 100 m/s2

\ a = (- 5j + 100i) m/s2
h v2
17. (a) tan q = =
b Rg
v 2b
\ h =
Rg
18. (a) It has the centripetal force, which is equal to the tension in the string (T).
v 2 ( 4 )2
19. (a) an = = = 8 m/s2, v
r 2 an
along negative z-axis.
O
5
20. (b) v = 72 ¥ = 20 m/s
18
v2 20 2
tan q = = =2
rg 20 ¥ 10
ŝƌĐƵůĂƌDŽƟŽŶപ119

mv 2
21. (d) N sin q =
r N q
and N cos q = mg r
h
mg
v2
\ tan q =
rg
h v2
or =
r rg
v 2 0.52
\ h = = = 2.5 cm
g 10

q 2p p
22. (a) w = = = rad/s
t 60 30
v2 (14 3 )2
23. (d) tan q = = = 3
rg 20 3 ¥ 9.8
or q = 60°
24. (b) vmax = m s rg = 0.5 ¥ 40 ¥ 9.8
= 14 m/s
25. (d) Explained in the theory.
26. (c) In a frame attached to the mass, there are three forces on it, these are tensions of
the string in the string and centrifugal force.
27. (d) For a particle moving with decreasing speed, it must have a torque along axis of
the rotation. Direction of momentum will not change.
28. (b) At the bottom of the circle
mv 2
T = mg +
r
2 ¥ 42
= 2 ¥ 10 +
1
= 52 N.
29. (d) mw2r = mg
g
or w=
r
r 4
\ T = 2p = 2p = 4s
g 9.8
1
30. (c) q1 = a(2)2
2
1
and q1 + q2 = a(4)2 = 4q1
2
q2 3
\ =
q1 1
120പWŚLJƐŝĐƐĨŽƌ//dͲ:͗DĞĐŚĂŶŝĐƐ//

mv 2
31. (a) mg – N =
R
mv 2
or N = mg -
R
As RB > RA, and so NB > NA.
q
mv 2
32. (c) T = mg cos q +
R T
T will be greater when cos q is greater. cos q will be greater
when q is small. q
mg
Level 2
2
Ê 2p ˆ
1. (d) ka = mw2(2a) = m Á ˜ ¥ 2 a
Ë T¯
2
Ê 2p ˆ
and k(2a) = mw2(3a) = m Á ˜ ¥ 3 a
Ë T¢ ¯
3
\ T¢ = T
2
2. (c) v2 = 2g(l – l cos q)

mv 2
Now T – mg =
l
mv 2 q
or T = mg +
l
T
2 g(l - l cos q )
or 2mg = mg +
l mg
\ q = 60°

3. (a)

q T
q
r
m

mg

Mg

T sin q = mw2r
= mw2 ¥ l sin q
ŝƌĐƵůĂƌDŽƟŽŶപ121

or T = mw2l ...(i)
Also T = Mg ...(ii)

Mg
\ w =
ml

4. (d) For the particle on the table

T = mw12l1
T

q
T
r

For the particle under the table


T sin q = mw22 r
= m w22 l2 sin q

or T = mw22 l2
\ mw12l1 = mw22l2
l1 w2
or = 22
l2 w1
5. (b)

q q
TA TB

mg mg

At position A,
TA cos q = mg,
and at position B,
TB = mg cos q
TB
\ = cos2 q.
TA
122പWŚLJƐŝĐƐĨŽƌ//dͲ:͗DĞĐŚĂŶŝĐƐ//

6. (c) From Newton’s second law

T M1w 2l/2

M2g

l
T – M 1w 2 = M 1a
2
and M 2g – T = M 2a
After solving the above equations, we get
2 M2 g - M1lw 2
a=
2( M1 + M2 )

pr 400
7. (c) = 200, \ r= m
2 p
v2 152
an = = = 1.76 m/s2
r 400/p

v 2 - u2 152 - 52
at = = = 0.5 m/s2
2s 2 ¥ 200

\ a = an2 + at2 = 1.84 m/s2


8. (d)

°
45 0.2 m
T

0.1 m 0.2
m
2

T sin 45° = mw2r


and T cos 45° = mg
w 2 r (2p n)2 r
\ tan 45° = =
g g
0.2
where r = 0.1 + = 0.24 m
2
After solving, we get n = 1.03.
ŝƌĐƵůĂƌDŽƟŽŶപ123

1
9. (a) q = w 0t + a t 2
2
t=0
1
or q = 3t + ¥ 3 ¥ t 2 ...(i)
2 t=t
1 q
and 120 + q = 3(4 + t) + ¥ 3 ¥ (4 + t)2 ...(ii) 120 rad
2
After solving the above equations, we get
t =7s
2 2
v 10
10. (b) an = = = 50 m/s2, along negative x-axis.
r 2

so a = - 50i - 1.5j m/s2
v2
11. (d) at = 2 m/s2, an = m/s2
200
\ 42 = an2 + at2
2
Ê v2 ˆ 2
or 16 = Á ˜ +2
Ë 200 ¯
\ v = 26.32 m/s
B T2 T1 T1
12. (a)
l 2l
T1 = mw2(3l)
and T2 – T1 = mw2l
or T2 = T1 + mw2l = mw2(3l) + mw2l
= 4 mw2l
T2 4
\ =
T1 3
13. (c) For weightlessness, N £ 0
Ê mv 2 ˆ mv 2
or Á mg - £0
r ˜¯
N r
Ë
or v ≥ gr mg

≥ 14 m/s.
14. (c)
N
q

mg
q
124പWŚLJƐŝĐƐĨŽƌ//dͲ:͗DĞĐŚĂŶŝĐƐ//

N cos q = mg and N sin q = mw2r


w 2r
\ tan q = ...(i)
g
Given y = x2
dy
\ = 2x
dx
or tan q = 2 ¥ 1 = 2 ...(ii)
From the above equations, we get
w = 2g (r = 1 m)

15. (a)

90 – 0 x 90 – 0
v0
r
r
q = 60°
2q

x r
=
sin 2q sin (90 - q )

fi x = 2r sin q

dx dq
\ = 2r cos q ¥
dt dt
dq dx/dt v0 v
= = = 0
dt 2r cos q 2r cos 60∞ r
16. (d) Velocity of contact point on rod CD and ring should have same component of
velocity along normal to rod CD.
\ BC(w¢) = 2(BC cos q ) w cos q
fi w¢ = 2w cos2 q = 3/2 rad/s

Multiple Correct Options


1. (a, c, d)
Body moving with constant speed,
v2
at = 0 and an =
r
ŝƌĐƵůĂƌDŽƟŽŶപ125

2
Ê v2 ˆ
\ a = an2 + at2 = Á ˜ + 02
Ë r¯
v2
=
r

2. (b, c, d)
v2
At q = 90°, v = 0, \ an = =0
r
and at = g sin 90° = g.

q
l cos q
an
a

at

At any angular position q, h = l cos q


\ v2 = 2gl cos q

v 2 2 gl cos q
an = = = 2g cos q
r l
and at = g sin q
ax
Thus, tan q =
at
sin q 2g cos q
or =
cos q g sin q

or 2cos2 q = sin2 q
= 1 – cos2 q
1
\ cos q =
3
3. (b, d)
At the highest position, we have
mv12
T1 + mg = v1
r
1 ¥ v12
or 10 + 1 ¥ 10 =
1 v2

\ v1 = 20 m/s
126പWŚLJƐŝĐƐĨŽƌ//dͲ:͗DĞĐŚĂŶŝĐƐ//

At the lowest position,


v22 = v12 + 2gh
= 20 + 2 ¥ 10 ¥ 2 = 60
mv22
\ T2 = mg +
r
1 ¥ 60
= 1 ¥ 10 + = 70 N
1
4. (b, c)
For q < f, v π 0
mv 2 T
T – mg cos q =
r
mv 2
or T = mg cos q + q
L mg
Also, at = g sin q
5. (a, b, c)
Because of gravitational force, speed of the ball continuously changes and so the
centripetal force and tension in the string.
6. (a, b, c)
1 A
Between A and B, mgL cos q = mvB2
2 q
ar
\ vB2 = 2gL cos q L
vB2
Now, ar = = 2g cos q B
L
and at = g sin q C
ar
\ a = at2 + ar2 = g 1 + 3 cos 2 q
mvB2
Now, at B: TB – mg cos q =
L
Put vB fi TB = 3 mg cos q
When total acceleration vector is directed horizontally
at g sin q 1
tan(90° – q) = = = tan
ar 2 g cos q 2
1
On solving, q = cos–1
3
7. (b, c)
mv 2
Ft = ma and Fn =
r
mv 2
\ F = frictional force = Fn2 + Ft2 >
r
Ft ma a
Also, m = tan q = = =
mg mg g
ŝƌĐƵůĂƌDŽƟŽŶപ127

8. (a, c) y
vB = Rw = 4 m/s
4 m/s
q = wt = 2t

At time t: v A = 4j

v B = (- 4 sin 2t i + 4 cos 2t j) x

\ v AB = ( 4 sin 2t)i + 4(1 - cos 2t)j
\ vAB = 4 1 + 1 - 2 cos 2t (8 sin t)
p B¢
vAB is maximum at t =
2
9. (a, b)
dv v
at = = 2 = constant at
q
dt
2
v
ar = a ar
R
As v is increasing, therefore ar is increasing.
ar
tan q =
at

10. (c) T = Mac ...(i)


2
M( 4 l )
Tl = a ...(ii)
12
Mv02
T + M(ac + la) = ...(iii)
l
A a B

T ac
T m(ac + la)

(FBD of particle in frame of end A or rod) From (i), (ii) and (iii)
v02 3v 2
ac = ,a = 0
5l 5l
mv02
Tc =
5l
11. (b, c, d), consult theory.
12. (a, b)
x
x = a cos pt, \ cos pt =
a
y
and y = b sin pt, \ sin pt =
b
128പWŚLJƐŝĐƐĨŽƌ//dͲ:͗DĞĐŚĂŶŝĐƐ//

x2 y2
Now + = cos2 pt + sin2 pt = 1,
a2 b 2
it represents an ellips.
 
After calculation, we find a ◊ v = 0.

Passages and Matrix Matching


Passage (Q. 1–2)
1. (b) mmg = mw2L
mg L
\ w =
L
2. (b) at = aL, an = w2L
\ F = mmg = Fn2 + Ft2
or (mmg)2 = (mw2L)2 + (maL)2
1/4
Èmg ˘
\ w = Í -a2˙
Î L ˚
v
Passage (Q. 3–5)

3. (d) v2 = 102 – 2g ¥ 1 = 80
mv 2
Now, tension, T =
r
1 ¥ 80 10 m/s
= = 80 N
1
4. (d) At the highest position
v2 = 102 – 2g ¥ 2 = 60
mv 2 1 ¥ 60
T = - mg = - 1 ¥ 10 = 50 N
r 1
v
5. (c) v2 = 102 – 2g(1 + 1 sin 30°) = 70
mv 2 T
T= – mg cos 60° mg
r 120°
1 ¥ 70 1
= - 1 ¥ 10 ¥
1 2
= 65 N

Passage (Q. 6–8)


6. (c) N sin q = mw2r
qN
= mw2R sin q
or N = mw2R
and N cos q = mg mg
ŝƌĐƵůĂƌDŽƟŽŶപ129

\ mw2 R cos q = mg
g
or cos q =
w 2R
R-h g
or = 2
R w R
g
\ h =R–
w2

g
7. (a) We have, h = R-
w2
for h ≥ 0, we have w ≥ g/R .
Passage (Q. 9–11)
9. (b) If TA is the tension in the string at lowest point, then
mv02
TA – mg sin q =
l
TAl
or v02 = – gl sin q
m
(110)(0.75)
= – (9.8) (0.75) (sin 37°)
(1.2)
= 64.34
or v0 = 8.02 m/s
10. (a) At the highest point,
mv 2
TB + mg sin q =
l
From energy conservation between position A and position B,

N
T
m
q

g
s
co

si
n
g

q
m

mg

KA + UA = KB + UB
1 1
mv02 + 0 = mv2 + 2mgl sin q
2 2
or 2
v = v02 – 4gl sin q
130പWŚLJƐŝĐƐĨŽƌ//dͲ:͗DĞĐŚĂŶŝĐƐ//

= (64.34) – 4 ¥ (9.8)(0.75)(sin 37°)


= 46.7
or v = 6.83 m/s
11. (d) From Eq. (2)

B
v TB q
sin ce
mg en
fer
TA Re el
v0 lev
2/
h = 2/sin q
A q
sin
mg
q

mv 2
TB = – mg sin q
l
1.2 ¥ (6.83)2
= – 1.2 ¥ 9.8 ¥ sin 37°
0.75
= 67.56 N
Passage (Q. 12–14)
12. (d); 13. (b); 14. (b)
As long as the load touches the body, the velocity of the latter is equal to the horizontal
component of the velocity of the load, and the acceleration of the body is equal to the
horizontal component of the acceleration of the load.
Let a be to acceleration of the load. Then we can write
  
a = a n + at
Horizontal component of the acceleration
v2
an = at sin a – cos a
l
The body also has the same acceleration. We can write for the motion of the body
v2
N = Man = Mat sin a – M sin a, where N is the normal force between load and
l
body. At the moment of separation, N = 0, and so
v2
at sin a = cos a.
l

an
an M
ar
mg
ŝƌĐƵůĂƌDŽƟŽŶപ131

The acceleration component at at the moment of separation of the load is only due to
the force of gravity at = g cos a.
Thus, the velocity of the load at the moment of separation is,
v= gl sin a
and the velocity of the body at the same moment is
u = v sin a = sin a gl sin a
According to the energy conservation, we have
1 1
mgl = mgl sin a + mv2 + Mv2 sin2 a
2 2
p
Here, a = . After simplifying, we get
6
M 1 gl
= 4 and u = v sin a =
m 2 2
Matrix Matching
Passage (Q. 15–18)
15. A Æ s; B Æ q; C Æ q; D Æ t
At, t = 1s = v = 2t = 2 m/s
v 2 22
\ an = = = 4 m/s2
r 1
dv d(2t)
at = = 2 m/s2
dt dt

Now, a = an2 + at2 = 4 2 + 22 = 2 5 m/s2


16. A Æ q; B Æ p; C Æ s; D Æ r
mvA2
At A: mg + T =
r
1 ¥ vA2
or 1 ¥ 10 + 10 =
1
\ vA = 20 m/s
Now, vc2 = vA2 + 2g ¥ 1
= 20 + 2 ¥ 10
\ vc = 40 m/s
mvc2 1 ¥ 40
Tc = = = 40 N
r 1
At B:
vB2 = vA2 + 2g ¥ 2
= 20 + 2 ¥ 10 ¥ 2
132പWŚLJƐŝĐƐĨŽƌ//dͲ:͗DĞĐŚĂŶŝĐƐ//

\ vB = 60 m/s
mvB2
= TB = mg +
r
1 ¥ 60
= 1 ¥ 10 + = 70 N
1
17. A Æ s; B Æ r; C Æ p; D Æ q
v2 dv
an = = 9t2 fi v = 12t and = 12
16 dt
dv 3
Tangential force, m ◊ = 16 = 6 N
dt 2
2
Ê mv 2 ˆ 2 Ê 9ˆ
2
Total force = 62 + Á ˜ = 6 + ÁË ˜¯ = 7.5 N.
Ë R ¯ 2

Power = FT ◊ v = 6 ¥ 3 16 = 72 watt
72
Average power = = 36 watt
2

18. A Æ r; B Æ r; C Æ p; D Æ s
df
Angular velocity of rod BC w¢ = –
dt
vc lw A
w¢ = =
y x q
x y 1 B
= = 1w 90 – (q +f)
cos f cos(q + f ) sin q
f C
sin q vc
w¢ = w x
cos f q y D

Angular acceleration of rod (IAOR)


2 2
dw (cos f cos q - sin f sin q )
BC = a = = - w2
dt cos 3 f
2 CHAPTER

Collisions

Striking of one body with another is commonly known as collision or impact.


Is it necessary to have at least two bodies for a collision to occur?
No. (We will elaborate it a little later).
Is physical contact necessary for a collision to take place between two bodies?
No.

WHAT IS A COLLISION?
“A collision is an isolated event in which a relatively strong force acts on each of two or
more colliding bodies for a relatively short time. Moreover, it must be possible to make a
clear distinction between times that are before, during and after the collision... . In an ideal
collision, only internal forces (between the bodies) play a role.”
[Halliday, Resnick, Walker; 4th edn.]
Notes:
(i) It is not necessary that a physical contact takes place in a collision, e.g., when an alpha
particle (2He4) ‘collides’ with a nucleus of gold (Au197), the force acting between them
may be repulsive — the particles may not touch, even then it may be called a ‘collision’.
(ii) When a space probe approaches a large planet, swings around it, and then continues
its course with increased speed (a slingshot encounter), that too is a collision. The
probe and planet do not actually “touch”, but, a collision does not require contact,
and a collision force does not have to be a force of contact; it can just as easily be a
gravitational force, as in this case.
[Halliday, Resnick, Walker; 4th edn.]

COLLISIONS
Define the terms: elastic collision, inelastic collision, perfectly inelastic collision, super-
elastic collision, head-on collision and oblique collision. Give important characteristics and
examples of different types of collisions.
The collisions between particles are of the following types:
1. Elastic collision. If there is no loss of kinetic energy during a collision, it is called an elastic
collision.
134പWŚLJƐŝĐƐĨŽƌ//dͲ:͗DĞĐŚĂŶŝĐƐ//

Characteristics of elastic collisions:


(i) The momentum is conserved.
(ii) Total energy is conserved.
(iii) The kinetic energy is conserved.
(iv) Forces involved during the collision are conservative.
(v) The mechanical energy is not converted into heat, light, sound, etc.
Examples. Collision between subatomic particles, collision between glass balls, etc.
2. Inelastic collision. If there is a loss of kinetic energy during a collision, it is called an inelastic
collision.
Characteristics of inelastic collisions:
(i) The momentum is conserved.
(ii) Total energy is conserved.
(ii) The kinetic energy is not conserved.
(iv) Some or all of the forces involved are non-conservative.
(v) A part of the mechanical energy is converted into heat, light, sound, etc.
Examples. Collision between two vehicles, collision between a ball and floor.
3. Perfectly inelastic collision. If two bodies stick together after the collision and move as a
single body with a common velocity, then the collision is said to be perfectly inelastic collision.
In such collisions, momentum is conserved, but the loss of kinetic energy is maximum.
Examples. Mud thrown on a wall and sticking to it, a man jumping into a moving
trolley, a bullet fired into a wooden block and remaining embedded in it, etc.
4. Superelastic or explosive collision. In such a collision, there is an increase in kinetic
energy. This occurs if there is a release of potential energy on an impact.
Examples. Bursting of a cracker when it hits the floor forcefully, the collision of a
trolley with another may release a compressed spring and thereby releasing the energy
stored in the spring.
5. Head-on or one-dimensional collision. It is the collision in which the colliding bodies
move along the same straight line path before and after the collision.
Example. Collision between two railway compartments.
6. Oblique or two-dimensional collision. If two bodies do not move along the same straight
line path but lie in the same plane before and after the collision, the collision is said to be oblique
or two-dimensional collision.
Example. Collision between two carrom coins.
Show that total linear momentum is conserved in all collisions.
Conservation of linear momentum in a collision. Suppose two bodies 1 and 2 collide
against each other. They exert mutual impulsive forces on each other during the collision
time Dt. The changes produced in the momenta of the two bodies will be
   
Dp1 = F12 Dt and Dp2 = F21 Dt

where F12 = the force exerted on body 1 by body 2

and F21 = the force exerted on body 2 by body 1
ŽůůŝƐŝŽŶƐപ135

According to Newton’s third law,


 
F12 = F21
 
\ F12 Dt = - F21 Dt
 
or F12 Dt + F21 Dt = 0
 
or Dp1 + Dp2 = 0
 
or D(p1 + p2 ) = 0
 
or p1 + p2 = constant
Thus, linear momentum is conserved during a collision even though the forces vary in a
complex manner during a collision.

For Your Knowledge


s 4OTAL LINEAR MOMENTUM IS CONSERVED AT EACH INSTANT OF EVERY COLLISION
s 4OTAL ENERGY IS CONSERVED IN ALL COLLISIONS
s 4HE TOTAL KINETIC ENERGY MAY OR MAY NOT BE CONSERVED DURING A COLLISION
s %VEN FOR AN ELASTIC COLLISION THE KINETIC ENERGY CONSERVATION HOLDS AFTER THE COLLISION
is over and does not hold at every instant of the collision.
s 7HEN TWO BODIES COLLIDE THEY GET DEFORMED AND MAY BE MOMENTARILY AT REST WITH
respect to each other.
s 4HE IMPACT AND DEFORMATION DURING A COLLISION MAY CONVERT PART OF THE INITIAL KINETIC
energy into heat and sound.

ELASTIC COLLISION IN ONE DIMENSION


Prove that in an elastic one-dimensional collision between two bodies, the relative velocity
of approach before collision is equal to the relative velocity of separation after the collision.
Hence, derive expressions for the velocities before collision. Discuss the special cases also.
Elastic collision in one dimension. As shown in Fig. 2.1, consider two perfectly elastic
bodies A and B of masses m1 and m2 moving along the same straight line with velocities u1
and u2 respectively. Let u1 > u2. After some time, the two bodies collide head-on and continue
moving in the same direction with velocities v1 and v2 respectively. The two bodies will
separate after the collision if v2 > v1.
m1 m1 m1
m2 m2 m2
u1 u2 v1 v2

A B A B
A B
Before During After
collision collision collision
Fig. 2.1 Elastic collision in one dimension

As linear momentum is conserved in any collision, so


m1u 1 + m 2u 2 = m 1v 1 + m 2v 2 ...(1)
or m1u 1 – m 1v 1 = m 2v 2 – m 2u 2
or m1(u1 – v1) = m2 (v2 – u2) ...(2)
136പWŚLJƐŝĐƐĨŽƌ//dͲ:͗DĞĐŚĂŶŝĐƐ//

3INCE +% IS ALSO CONSERVED IN AN ELASTIC COLLISION SO


1 1 1 1
m1u12 + m2 u22 = m1v12 + m2 v22
2 2 2 2
or m1u12 – m1v12 = m2v22 – m1u22
or m1 (u1 + v1) (u1 – v1) = m2 (v2 + u2) (v2 – u2) ...(3)
$IVIDING %Q  BY %Q  WE GET
u 1 + v1 = v2 + u2
or u1 – u2 = v2 – v1 ...(4)
or relative velocity of A w.r.t. B before collision
= Relative velocity of B w.r.t. A after collision
or relative velocity of approach
= Relative velocity of separation
Thus, in an elastic one-dimensional collision, the relative velocity of approach before collision is
equal to the relative velocity of separation after the collision.
Velocities of the bodies after the collision. &ROM %Q  WE GET
v 2 = u1 – u2 + v1
Putting this value of v2 IN %Q  WE GET
m1u1 + m2u2 = m1v1 + m2(u1 – u2 + v1)
= m1v 1 + m 2u 1 – m 2u 2 + m 2v 1
or (m1 – m2) u1 + 2m2u2 = (m1 + m2) v1

Ê m - m2 ˆ Ê 2m2 ˆ
or v1 = Á 1 ˜ u1 + Á u2 ...(5)
Ë 1
m + m2¯ Ë m1 + m2 ˜¯
Interchanging the subscripts 1 and 2 in the above equation, we get

Ê m - m1 ˆ Ê 2m1 ˆ
v2 = Á 2 ˜ u2 + Á u1 ...(6)
Ë m1 + m2 ¯ Ë m1 + m2 ˜¯
%QUATIONS  AND  GIVE THE lNAL VELOCITIES OF THE COLLIDING BODIES IN TERMS OF THEIR
initial velocities.

Special Cases
(i) When two bodies of equal masses collide. Let
m1 = m2 = m (say).
&ROM %Q 
2mu2
v1 = = u2
2m
= velocity of body of mass m2 before collision
ŽůůŝƐŝŽŶƐപ137

&ROM %Q 
2mu1
v2 = = u1
2m
= velocity of body of mass m1 before collision
Hence, when two bodies of equal masses suffer one-dimensional elastic collision, their velocities
get exchanged after the collision.
(ii) When a body collides against a stationary body of equal mass. Here m1 = m2 = m
and u2 = 0.
&ROM %Q  v1 = 0
&ROM %Q  v 2 = u1
Hence, when an elastic body collides against another elastic body of equal mass, initially at
rest, after the collision the first body comes to rest while the second body moves with the initial
velocity of the first.
(iii) When a light body collides against a massive stationary body. Here m1 << m2 and
u2 = 0. Neglecting m1 IN %Q  WE GET
m2 u1
v1 = – = – u1
m2
&ROM %Q  v2  0.
Hence, when a light body collides against a massive body at rest, the light body rebounds after
the collision with an equal and opposite velocity while the massive body practically remains at
rest. A light ball on striking a wall rebounds almost with the same speed and the wall
remains at rest.
(iv) When a massive body or a projectile collides against a light stationary body. Here
m1 >> m2 and u2 = 0.
Neglecting m2 IN %Q  WE GET
mu
v1 = 1 1 = u1
m1
2m1u1
and v2 = = 2u1
m1
Hence, when a massive body collides against a light body at rest, the velocity of the massive
body remains almost unchanged while the light body starts moving with twice the velocity of
the massive body.

COEFFICIENT OF RESTITUTION
What is the coefficient of restitution? What is its significance?
%QGHſEKGPVQHTGUVKVWVKQPQTEQGHſEKGPVQHTGUKNKGPEG. Most of the real collisions are neither
perfectly elastic nor perfectly plastic. They are partially elastic collisions IN WHICH THE +%
reduces and so the speed of separation is less than the speed of approach.
The coefficient of restitution gives a measure of the degree of restitution of a collision and is defined
as the ratio of the magnitude of relative velocity of separation after collision to the magnitude of
relative velocity of approach before collision. It is given by
138പWŚLJƐŝĐƐĨŽƌ//dͲ:͗DĞĐŚĂŶŝĐƐ//

|v1 - v2| v - v2
e= =- 1
|u1 - u2| u1 - u2
The value of e depends on the materials of the colliding bodies. For two glass balls,
e = 0.95 and for the lead balls, e = 0.20.
The coefficient of restitution can be used to distinguish between the different types of
collisions as follows:
(i) For a perfectly elastic collision, e = 1, i.e., relative velocity of separation is equal to the
relative velocity of approach.
(ii) For an inelastic collision, 0 < e < 1, i.e., the relative velocity of separation is less than the
relative velocity of approach.
(iii) For a perfectly inelastic collision, e = 0, i.e., the relative velocity of separation is zero. The
two bodies move together with a common velocity.
(iv) For a superelastic collision, e > 1, i.e., the kinetic energy increases.

Table 2.1 Different types of collisions


Kinetic %QGHſEKGPV Main
Collision
energy of restitution domain
%LASTIC Conserved e=1 Between atomic particles
Inelastic Not conserved 0<e<1 Between ordinary objects
Perfectly inelastic -AX LOSS OF +% e=0 During shooting
Super elastic +% INCREASES e>1 In explosions

Maximum energy transfer in a head on elastic collision: Consider a ball of mass m1


moving with velocity u1 collides with a ball of mass m2 at rest. Let the velocity of the first
ball after collision be v1. Now
Ê 1ˆ
Initial kinetic energy of first ball Ki = Á ˜ m1u12
Ë 2¯
Ê 1ˆ
Final kinetic energy of first ball Kf = Á ˜ m1v12
Ë 2¯
The fractional decrease in kinetic energy is
Ki - K f u12 - v12 v12
= =1– ...(7)
Ki u12 u12
!CCORDING TO %Q 
Ê m - m2 ˆ
v1 = Á 1 u1
Ë m1 + m2 ˜¯
2
v12 Ê m - m2 ˆ
or =Á 1 ...(8)
2
u1 Ë m1 + m2 ˜¯
Substituting the value of (v12/u12 FROM %Q  IN %Q  WE HAVE
Ki - K f Ê m - m2 ˆ
2
4 m1 m2
= 1-Á 1 ˜ =
Ki Ë m1 + m2 ¯ (m1 + m2 )2
ŽůůŝƐŝŽŶƐപ139

If m1 = m and m2 = n m, then
Ki - K f 4n
=
Ki (1 + n)2
The transfer of energy will be maximum when Ki = 0. For
4n
=1
(1 + n)2
or n = 1.
Thus, when the mass ratio is unity, the whole of the kinetic energy of the moving ball is
transferred to the ball initially at rest.
Perfectly inelastic collision. The collision is known as perfectly inelastic when there is a loss
of kinetic energy during collision and colliding bodies stick together and move as a single
unit. For example, the collision between a bullet and a target is perfectly inelastic when the
bullet remains embedded in the target. In this case kinetic energy is not conserved. Between
the two limits of perfectly elastic and perfectly inelastic collisions, all other collisions are
imperfectly elastic.
Now we shall calculate the change of kinetic energy in an imperfectly elastic collision. Let
us consider the case of two bodies of masses m1 and m2 moving along the line joining their
centres with velocities u1 and u2 respectively. Let them move after collision with velocity v1
and v2. As total momentum remains constant.
Loss of Kinetic Energy in an Inelastic Collision
m1u 1 + m 2u 2 = m 1v 1 + m 2v 2 ...(1)
Here we introduce a term e known as coefficient of restitution as
(v1 - v2 )
e =–
(u1 - u2 )
or v1 – v2 = – e (u1 – u2) ...(2)
Initial kinetic energy of the bodies
1 1
= m1u12 + m2 u22
2 2
Final kinetic energy of the bodies
1 1
= m1v12 + m2 v22
2 2
Decrease in kinetic energy

Ê1 1 ˆ Ê1 1 ˆ
DE = Á m1u12 + m2 v22 ˜ - Á m1v12 - m2 v22 ˜
Ë2 2 ¯ Ë 2 2 ¯
or 2 DE = m1u1 + m2u2 – m1v1 – m2v2
2 2 2 2
...(3)
-ULTIPLYING %Q  BY m2 AND ADDING TO %Q  AND SIMPLIFYING WE GET

1
v1 = [(m1u1 + m2u2) – m2 e(u1 – u2)] ...(4)
m1 + m2
140പWŚLJƐŝĐƐĨŽƌ//dͲ:͗DĞĐŚĂŶŝĐƐ//

3IMILARLY MULTIPLYING %Q  BY m1 AND SUBTRACTING FROM %Q  AND SIMPLIFYING WE GET

1
v2 = [(m1u1 + m2u2) + m1e (u1 – u2)] ...(5)
m1 + m2

Substituting the values of v1 and v2 FROM %QS  AND  IN %Q  WE GET

2DE = m1u12 + m2u22


m1
- 2
[(m1u1 + m2u2) – m2 e(u1 – u2)]2
(m1 + m2 )

m2
- 2
[(m1u1 + m2u2) + m1 e(u1 – u2)]2
(m1 + m2 )
After simplification, we have

Ê mm ˆ
2DE = Á 1 2 ˜ (e2 – 1) (u1 – u2)2
Ë m1 + m2 ¯

1 Ê m1 m2 ˆ 2
or DE = (e – 1) (u1 – u2)2 ...(6)
2 ÁË m1 + m2 ˜¯

When the collision is elastic


For an elastic collision, e   )N THE %Q  SETTING e = 1, we have
DE = 0
As expected, there is no loss of kinetic energy in an elastic collision.
+ORQTVCPV0QVG9TQPIWUCIGQHEQGHſEKGPVQHTGUVKVWVKQP.
Many students use the coefficient of restitution wrongly. The two fundamental mistakes
that students make are:
(a) Not considering the signs of velocities of approach and separation.
(b) Not considering the correct directions of velocities.
(a) Not considering the signs of velocities of approach and separation
All of us know that the equation relating to restitution and velocities is:
(v2 – v1) = – e ¥ (u2 – u1) or e = v2 – v1/u1 – u2
If the approach velocities are pointing towards each other, u1 u2
or the separation velocity is away from each other than what
equation should we use?
v1 v2
This equation is valid only for the following situation
of approach and separation (Fig. 2.2).
Fig. 2.2 Velocities for correct usage
of equation
ŽůůŝƐŝŽŶƐപ141

If the direction is as follows (Fig. 2.3). u1 u2


Then use the equation
(v2 – v1) = – e ¥ (– u2 – u1) v1 v2

If the direction is as follows (Fig. 2.4).


Fig. 2.3 Different directions

u1 u2

v1 v2

Fig. 2.4 Different directions

Then use the equation


(v2 – v1) = – e ¥ (– u2 + u1)
If the direction is as follows (Fig. 2.5).
u1 u2

v1 v2

Fig. 2.5 Different directions

Then use the equation


v2 + v1 = – e ¥ (– u2 + u1)
If the direction is as follows (Fig. 2.6).
u1 u2

v1 v2

Fig. 2.6 Different directions

Then use the equation


(– v2 + v1) = – e ¥ (– u2 + u1)
(b) Not considering the correct directions of
velocities. This mistake happens when the velocities u1
are not in the same line. If the bodies have velocities u2
in different directions, then you should resolve the
velocities of approach and the velocities of separation v2
along the line joining the bodies. These components
v1
are what you should use in the coefficient of
restitution equation.
Fig. 2.7 Different directions of approach
As an example consider Fig. 2.7. Assume two
and separation
bodies are approaching each other with velocities
142പWŚLJƐŝĐƐĨŽƌ//dͲ:͗DĞĐŚĂŶŝĐƐ//

u1 and u2 and separating with velocities of v1 and v2 u2 cos q u2


respectively.
In such a case, you should first resolve the velocities v2 cos a v2
along the line joining the bodies. The line joining the bodies
in this example is horizontal, but this need not be the
case everywhere. In such a case, the velocities have to be Fig. 2.8 Resolve the velocities
along the direction of
resolved along the directions of the line joining the bodies.
line of the bodies
This is shown in Fig. 2.8.
The correct equation is therefore:
(v2 – v1 cos a) = – e ¥ (u2 + u1 cos q)
You should note that the coefficient of restitution formula is valid only along the line
joining the centres of the two bodies.

WORKED PROBLEMS
Problem 2.1 Two particles of masses m and 2 m moving in m
2v v
m
opposite directions collide elastically with velocity 2v and v,
respectively. Find their velocities after collision. Fig. 2.9
Solution Let the final velocities of m and 2m be v1 and v2, v1 v2
respectively, as shown in Fig. 2.10. m 2m
By conservation of momentum, we get
Fig. 2.10
m(2v) + 2m(– v) = m(v1) + 2m(v2)
or 0 = mv1 + 2mv2
or v1 + 2v2 =0 …(i)
and since the collision is elastic
v2 – v1 = 2v – (– v)
or v 2 – v1 = 3v …(ii)
Solving the above two equations, we get
v2 = v and v1 = – 2v 2v
m
v
2m
That is, mass 2m returns with velocity v while mass m
returns with velocity 2v in the direction shown in Fig. 2.11. Fig. 2.11
Problem 2.2 Two ball bearings of mass m each moving in opposite directions with equal
speeds v collide head on with each other. Predict the outcome of the collision, assuming it
to be perfectly elastic.
Solution Here m1 = m2 = m (say), u1 = v, u2 = – v
As the collision is perfectly elastic, velocities after the collision will be
m1 - m2 2m2
v1 = ◊ u1 + ◊ u2
m1 + m2 m1 + m2
m-m 2m
= ◊v + ( - v)
m+m m+m
= 0 – v = – v.
ŽůůŝƐŝŽŶƐപ143

2m1 m - m1
v2 = ◊ u1 + 2 ◊ u2
m1 + m2 m1 + m2
2m m-m
= ◊v + ◊ ( - v)
m+m m+m
= v + 0 = v.
Thus, the two balls bounce back with equal speeds after the collision.
Problem 2.3 A railway carriage of mass 9000 kg moving with a speed of 36 kmh–1 collides
with a stationary carriage of the same mass. After the collision, the carriages get coupled and
move together. What is their common speed after collision? What type of collision is this?
(NCERT)
Solution Here
m1 = 9000 kg, u1 = 36 kmh–1 = 10 ms–1
m2 = 9000 kg, u2 = 0, v1 = v2 = v = ?
By conservation of momentum,
m1u1 + m2u2 = (m1 + m1)v
9000 ¥ 10 + 0 = (9000 + 9000)v
90000
or v = = 5 ms–1.
18000
4OTAL +% BEFORE COLLISION
1 1
= m1u12 + m2 u22
2 2
1
= ¥ 9000 ¥ 10 ¥ 10 + 0 = 450000 J
2
1
4OTAL +% AFTER COLLISION  (m1 + m2 )v 2
2
1
= ¥ 2 ¥ 9000 ¥ 52 = 225000 J.
2
4HUS TOTAL +% AFTER COLLISION  4OTAL +% BEFORE COLLISION
Hence the collision is inelastic.
Problem 2.4 What percentage of kinetic energy of a moving particle is transferred to a
stationary particle, when moving particle strikes with a stationary particle of mass (i) 9 times
in mass, (ii) equal in mass, and (iii) 1/19th of its mass?
Solution For the moving particle, m1 = m(say),
Initial velocity = u1
For the stationary particle, m2 = xm (say),
Initial velocity = u2 = 0
144പWŚLJƐŝĐƐĨŽƌ//dͲ:͗DĞĐŚĂŶŝĐƐ//

2m1 m - m1
As v2 = ◊ u1 + 2 ◊ u2
m1 + m2 m1 + m2
2m 2u1
\ v2 = ◊ u1 + 0 =
m + xm 1+ x
+% OF THE MOVING PARTICLE BEFORE COLLISION
1 1
K1 = m1u12 = mu12
2 2
+% OF THE STATIONARY PARTICLE AFTER COLLISION
1 1 4u12
K2 = m2 v12 = ◊ mx ◊
2 2 (1 + x )2
4x 1 4x
= 2
◊ mu12 = ◊ K1
(1 + x ) 2 (1 + x)2
 OF +% TRANSFERRED
K2 4 xK1 100
= ¥ 100 = 2
¥
K1 (1 + x ) K1
4x
= ¥ 100%
(1 + x )2
(i) When moving particle strikes a stationary particle 9 times in mass, x = 9.
4¥9
 OF +% TRANSFERRED  ¥ 100 = 36%.
(1 + 9)2
(ii) When moving particle strikes a stationary particle of equal mass, x = 1.
4¥1
 OF +% TRANSFERRED  ¥ 100 = 100%
(1 + 1)2
1 1
(iii) When moving particle strikes a stationary particle th of its mass, x = .
19 19
4 ¥ (1/ 19)
 OF +% TRANSFERRED  ¥ 100 = 19%.
(1 + 1/ 19)2

Problem 2.5 A ball is dropped to the ground from a height of 2 m. The coefficient of
restitution is 0.6. To what height will the ball rebound?
Solution As the ball falls to the ground, its potential energy mgh1 changes into kinetic
1
energy mv12.
2
1
\ mgh1 = mv12 ...(i)
2
ŽůůŝƐŝŽŶƐപ145

1
After rebounding, its kinetic energy mv22 changes into potential energy mgh2.
2
1
\ mgh2 = mv22 ...(ii)
2
2
h2 Ê v2 ˆ
$IVIDING %Q II BY %Q I WE GET =
h1 ÁË v1 ˜¯
v2
But h1 = 2 m, e = = 0.6
v1
h2
\ = (0.6)2 = 0.36 or h2 = 0.72 m.
2m

Problem 2.6 A ball is dropped vertically from a height of 3.6 m. It rebounds from a
horizontal surface to a height of 1.6 m. Find the coefficient of restitution of the material of
the ball. (OJEE, 2013)
Solution Here h1 = 3.6 m, h2 = 1.6 m
Velocity of the ball with which it reaches the horizontal surface,

u = 2 gh1 = 2 ¥ 9.8 ¥ 3.6 = 8.4 ms–1


Velocity of the ball with which it rebounds,

v= 2 gh2 = 2 ¥ 9.8 ¥ 1.6 = 5.6 ms–1


v 5.6
Coefficient of restitution, e = = = 0.667.
u 8.4

Problem 2.7 A ball is dropped from a height h. It rebounds from the ground a number
of times. Given that the coefficient of restitution is e, to what height does it go after the nth
rebounding?
Solution Let v0 be the velocity with which the ball strikes the ground first time and vn the
velocity after nth rebounding. Then the coefficient of restitution will be
v1 v2 v3 v
e = = = = ºº = n
v0 v1 v2 vn - 1
v1 v2 v3 vn v
\ en = ¥ ¥ ¥ ºº ¥ = n
v0 v1 v2 vn - 1 v0

But v0 = 2gh and vn = 2gH


where H is the height to which the ball rises after nth rebounding.

vn 2 gH h
Hence, en = = = or H = he2n.
v0 2 gh h
146പWŚLJƐŝĐƐĨŽƌ//dͲ:͗DĞĐŚĂŶŝĐƐ//

Problem 2.8 Slowing down of neutrons. In a unclear reactor a neutron of high speed
(typically 107 ms–1) must be slowed to 103 ms–1 so that it can have a high probability of
interacting with isotope 235
92U and causing it to fission. Show that a neutron can lose most of
its kinetic energy in an elastic collision with a light nucleus like deuterium or carbon which
has a mass of only a few times the neutron mass. The material making up the light nuclei,
usually heavy water (D2O) or graphite is called a moderator.
or
A body of mass M at rest is struck by a moving body of mass m. Prove that fraction of
THE INITIAL +% OF THE MASS m transferred to the struck body is 4 m M/(m + M)2 in an elastic
collision.
Solution Here m1 = mass of neutron = m
m2 = mass of target nucleus = M
u1 = u and u2 = 0
2m1 m - m1
Now, v2 = ◊ u1 + 2 ◊ u2
m1 + m2 m1 + m2
2m 2mu
= ◊u+ 0 =
m+ M m+ M
)NITIAL +% OF MASS m,
1 1
K1 = m1u12 = mu2
2 2
&INAL +% OF MASS M,
2
1 1 Ê 2mu ˆ 2 Mm2 u2
K2 = m2 v22 = M Á ˜ =
2 2 Ë m + M¯ ( m + M )2
&RACTION OF THE INITIAL +% TRANSFERRED
K 2 2 Mm2 u2 2 4 mM
f= = 2
¥ 2
=
K1 ( m + M ) mu ( m + M )2
(i) For deuterium, M = 2m, therefore,
4 m ¥ 12m 8
f= 2
=  0.9
( m + 2 m) 9
About 90% of the neutron’s energy is transferred to deuterium.
(ii) For carbon, M = 12 m, therefore
4 m ¥ 12m
fraction = = 0.284
(m + 12m)2
About 28.4% of the neutron’s energy is transferred to carbon.
Problem 2.9 Consider the collision depicted in Fig. 2.12 to be between two billiard balls
with equal masses m1 = m2. The first ball is called the cue while the second ball is called the
target. The billiard player wants to “sink” the target ball in a corner pocket, which is at an
angle q2 = 37°. Assume that the collision is elastic and that friction and rotational motion are
not important. Obtain q1.
ŽůůŝƐŝŽŶƐപ147

Solution By conservation of momentum, we have


  
m u1 + 0 = m v 1 + m v 2
  
or u1 = v 1 + v 2 ...(i)
By conservation of energy, we have
1 1 1
mu12 = mv12 + mv22
2 2 2
or u12 = v12 + v22 ...(ii)
&ROM %Q I WE HAVE
     
u1 ◊ u 1 = ( v 1 + v 2 ) ◊ ( v 1 + v 2 )
       
= v1 ◊ v1 + v1 ◊ v 2 + v 2 ◊ v1 + v 2 ◊ v 2
 
or u12 = v12 + v22 + 2v1 ◊ v 2
 
or u12 = u12 + 2v1 ◊ v 2 ;5SING %Q II =
 
or v1 ◊ v 2 = 0
 
Thus, the angle between v1 and v 2 is 90°
or q1 + q2 = 90°
\ q1 = 90° – q2 = 90 – 37° = 53°.
Problem 2.10 A nucleus of radius (226 222 4
88 Ra) decays to 86Rn by the emission of a-particle ( 2He)
of energy 4.8 MeV. If mass of 222 4
86Rn = 222.0 amu and mass of 2He = 4.003 amu, then calculate
222
the recoil energy of the daughter nucleus 86Rn.

Solution The nuclear decay may be represented as follows:

Ra226 Rn222 a-particle


Decay
+
At rest KE = ? KE = 4.8 MeV

Fig. 2.12

The kinetic energy of a particle is given by

p2
K= \ p= 2mK
2m
As momentum is conserved in the absence of an external force, so
mK = constant
or mRnKRn = maKa
ma Ka
or KRn =
mRn

4.003 ¥ 4.8
= = 0.0866 MeV.
222
148പWŚLJƐŝĐƐĨŽƌ//dͲ:͗DĞĐŚĂŶŝĐƐ//

Problem 2.11 The nucleus Fe57 emits a g -ray of energy 14.4 KeV. If the mass of the nucleus
is 56.935 amu, calculate the recoil energy of the nucleus.
Take 1 amu = 1.66 ¥ 10–27 kg.
Solution The nuclear decay may be represented as follows:

Fe57 Æ Fe57 + hv (g -ray photon)


%XCITED STATE 'ROUND STATE

According to de-Broglie hypothesis, momentum of a photon of energy E is

E 14.4 keV 14.4 ¥ 1.6 ¥ 10 -16 J


p = = =
c c 3 ¥ 108 ms -1
= 7.68 ¥ 10–24 kg ms–1.
By conservation of momentum, the momentum of daughter nucleus,
p = momentum of g-ray photon
= 7.68 ¥ 10–24 kg ms–1
The recoil energy of the nucleus will be
p2 (7.68 ¥ 10 -24 )2
K = =
2m 2 ¥ 56.935 ¥ 1.66 ¥ 10 -27

0.312 ¥ 10 -21
= 0.312 ¥ 10–21 J = KeV
1.6 ¥ 10 -16
= 1.95 ¥ 10–6 KeV.
Problem 2.12 A ball of 0.1 kg makes an elastic head on collision with a ball of unknown
mass that is initially at rest. If the 0.1 kg ball rebounds at one-third of its original speed,
what is the mass of the other ball?
Solution Here m1 = 0.1 kg, m2 = ?
u
u1 = u(say), v1 = – , u2 = 0
3
m1 - m2 2m2
From formula v1 = ◊ u1 + ◊ u2
m1 + m2 m1 + m2
u 0.1 - m2
\ – = ◊u+ 0
3 0.1 + m2
1 0.1 - m2
or - =
3 0.1 + m2
or – 0.1 – m2 = 0.3 – 3m2 or 2m2 = 0.4
\ m2 = 0.2 kg.
ŽůůŝƐŝŽŶƐപ149

Problem 2.13 Show that when a moving body collides with a stationary body of mass m
4m
or 1/m times its mass, then the moving body transfers part of its kinetic energy to
(1 + m)2
the stationary body.
Solution Case 1. Mass of moving body = m1
Initial velocity = u1
Mass of stationary body, m2 = mm1
Initial velocity, u2 = 0;
Velocity of the stationary body after collision will be
2m m - m1
v2 = ◊ u1 + 2 ◊ u2
m1 + m2 m1 + m2
2m1 2
= ◊ u1 + 0 = ◊ u1
m1 + mm1 1+ m
&RACTION OF THE +% TRANSFERRED TO THE STATIONARY BODY
2
Ê 2m ˆ
1
m2 v22 mm1 Á ◊ u1 ˜
2 Ë 1 + m ¯ 4m
= 2
=
1 m u (1 + m )2
m1u12 1 1
2
m1
Case 2. Here m2 =
m
2m1 2m1
v2 = ◊ u1 + 0 = ◊u
m Ê 1ˆ 2
m1 + 1 m1 Á 1 + ˜
m Ë m¯
2m
= ◊ u1
1+ m
&RACTION OF THE +% TRANSFERRED TO THE STATIONARY BODY
2
1 m1 Ê 2m ˆ
m2 u22 m ÁË 1 + m u1 ˜¯ 4m
2 = =
2
1
m1u12 m u
1 1 (1 + m )2
2
\ )N EACH CASE +% OF THE STATIONARY BODY AFTER COLLISION
4m
= ¥ +% OF THE MOVING BODY BEFORE COLLISION
(1 + m)2

Problem 2.14 A ball is dropped from a height h on to a floor. If the coefficient of restitution
is e, calculate the height to which the ball first rebounds?
150പWŚLJƐŝĐƐĨŽƌ//dͲ:͗DĞĐŚĂŶŝĐƐ//

Solution The velocity with which the ball strikes the floor is
u= 2gh
If the ball rebounds with velocity v, then
v
e= or v = eu = 2gh
u
If the ball rebounds to height h’, then
02 – v2 = 2(– g) h’
v2 1
or h’ = = ¥ e 2 ◊ 2 gh = he2.
2g 2g

Problem 2.15 Two identical balls are approaching m 2 m/s 4 m/s m


towards each other on a straight line with velocity 2
m/s and 4 m/s respectively. Find the final velocities,
after elastic collision between them. Fig. 2.13

Solution The two velocities will be exchanged and the final motion is reverse of initial
motion for both.

4 m/s 2 m/s

Fig. 2.14

Problem 2.16 Three balls A, B and C of same mass m are placed on m


u
m m
a frictionless horizontal plane in a straight line as shown. Ball A is A B C
moved with velocity u towards the middle ball B. If all the collisions
are elastic then, find the final velocities of all the balls. Fig. 2.15

Solution A collides elastically with B and comes to rest but B starts moving with velocity u
m m m
u
A B C

Fig. 2.16

After a while B collides elastically with C and comes to rest but C starts moving with
velocity u
m m m
u
A B C

Fig. 2.17

\ Final velocities VA = 0; VB = 0 and VC = u


Note:
u u
m m m m m m m m m m m m

as ki = kf
ŽůůŝƐŝŽŶƐപ151

Problem 2.17 Four identical balls A, B, C and D are placed in A


u
B C
u
D
a line on a frictionless horizontal surface. A and D are moved
with same speed u towards the middle as shown in Fig. 2.18. Fig. 2.18
Assuming elastic collisions, find the final velocities.
Solution A and D collide elastically with B and C respectively and come to rest but B and
C start moving with velocity u towards each other as shown in Fig. 2.19.
u u
A B C D

Fig. 2.19

B and C collide elastically and exchange their velocities to move in opposite directions.
u u
A B C D

Fig. 2.20

Now B and C collide elastically with A and D respectively and come to rest but A and D
start moving with velocity u away from each other as shown in Fig. 2.21.
u u
A B C D

Fig. 2.21

\ Final velocities: VA = u (¨); VB = 0; VC = 0 and VD = u (Æ)


Problem 2.18 If A is moved with velocity u and D is moved 2u
u
with 2u as shown in Fig. 2.22. What will be the final velocities A B C D
now?
Fig. 2.22
A B C C
Solution 2u u

Fig. 2.23

Problem 2.19 A 1 kg ball, moving at 12 ms–1, collides head on with a 2 kg ball moving in
2
the opposite direction at 24 ms–1. If the coefficient of restitution is , then the energy lost in
the collision is: 3

(a) 60 J (b) 120 J (c) 240 J (d) 480 J


Solution Initial momentum = Final momentum
m 1u 1 + m 2u 2 = m 1v 1 + m 2v 2
(1) (12) + (2) (–24) = 1v1 + 2v2
fi v1 + 2v2 = – 36 ...(i)
152പWŚLJƐŝĐƐĨŽƌ//dͲ:͗DĞĐŚĂŶŝĐƐ//

Ê v - v1 ˆ 2 Ê v - v1 ˆ
Further, e = –Á 2 fi = -Á 2
Ë u2 - u1 ˜¯ 3 Ë -24 - 12 ˜¯
2 v2 - v1
fi = fi v2 – v1 = 24 ...(ii)
3 36
!DD %Q I AND %Q II
3v2 = – 12 fi v2 = – 4 ms–1 fi v1 = – 28 ms–1
Ê 1 ˆ
fi ,OSS  4OTAL INITIAL +% n 4OTAL lNAL +% Á +% = mv 2 ˜
Ë 2 ¯
1 1
3INCE TOTAL INITIAL +%  (1)(144) + (2)(576) fi E1 = 72 + 576 = 648 J
2 2
3IMILARLY TOTAL lNAL +%
1 1
E2 = (1)(784) + (2)(16) fi E2 = 392 + 16
2 2
fi E2 = 408 J fi Loss = 648 – 408 fi Loss = 240 J
Problem 2.20 A ball of mass m moving at a speed v makes a head on collision with an
identical ball at rest. The kinetic energy of the balls after the collision is 3/4ths of the original.
Find the coefficient of restitution.
m m m v¢2 m v¢1
v

Before collision After collision


Fig. 2.24

Solution As we have seen in the above discussion, that under the given conditions
Ê 1 + eˆ Ê 1 - eˆ
v 1¢ = Á v 2¢ = Á
Ë 2 ˜¯ Ë 2 ˜¯
v and v
3
Given that Kf = Ki
4
1 1 3Ê1 ˆ
or mv1 ¢ 2 + mv2 ¢ 2 = Á mv 2 ˜
2 2 4Ë2 ¯
Substituting the value, we get
2 2
Ê 1 + eˆ Ê 1 - eˆ 3
ÁË ˜¯ + ÁË ˜¯ =
2 2 4
2 2
or (1 + e) + (1 – e) = 3
or 2 + 2e2 = 3
1
or e2 =
2
1
or e =
2
ŽůůŝƐŝŽŶƐപ153

Problem 2.21 Two bodies of masses 5 kg and 3 kg moving in the same direction along
the same straight line with velocities 5 ms–1 and 3 ms–1 respectively suffer one-dimensional
elastic collision. Find their velocities after the collision.
Solution Here m1 = 5 kg, u1 = 5 ms–1,
m2 = 3 kg, u2 = 3 ms–1
m1 - m2 2m2
\ v1 = ◊ u1 + ◊ u2
m1 + m2 m1 + m2
5-3 2¥3
= ¥5+ ¥3
5+3 5+3
2 6 5 9 14
= ¥5+ ¥3= + = = 3.5 ms–1.
5 8 4 4 4
2m1 m - m1
v2 = ◊ u1 + 2 ◊ u2
m1 + m2 m1 + m2

2¥5 3-5
= ¥5+ ¥3
5+3 5+3
50 6 44
= - = = 5.5 ms–1.
8 8 8

Problem 2.22 A 10 kg ball and 20 kg ball approach each other with velocities 20 ms–1 and
10 ms–1 respectively. What are their velocities after collision if the collision is perfectly elastic?
Solution Here m1 = 10 kg, m2 = 20 kg,
u1 = 20 ms–1, u2 = – 10 ms–1
m1 - m2 2m2
v1 = ◊ u1 + ◊ u2
m1 + m2 m1 + m2
10 - 20 2 ¥ 20
= ¥ 20 + ¥ ( -10)
10 + 20 10 + 20
20 40 60
=- - =- = –20 ms–1.
3 3 3
2m1 m - m1
v2 = ◊ u1 + 2 ◊ u2
m1 + m2 m1 + m2
2 ¥ 10 20 - 10
= ¥ 20 + ¥ ( -10)
10 + 20 10 + 20
40 10 30
= - = = 10 ms–1.
3 3 3
154പWŚLJƐŝĐƐĨŽƌ//dͲ:͗DĞĐŚĂŶŝĐƐ//

Problem 2.23 Consider a ball dropping from some height h. Let e be the coefficient of
restitution between the ball and the ground and ball rebounds again and again, then find:
(i) Velocity after nth collision
(ii) Height attained after nth collision
(iii) Total distance travelled by ball before stop
(iv) Total time of motion.
(v) Change in momentum and the average force
Solution Velocity of the ball just before collision with the ground u = 2gh.
(a) After first collision
v1 - v2
e=–
u1 - u2 h
h1
h2
here suffix 1 is for the ball and 2 is for fi•
the ground (earth).
u v1 v2
%ARTH BEING VERY MASSIVE IN COMPARI-
son to ball, u2 = 0 and v2 = 0.
Ground
v1 - 0
\ e=– fi v1 = – eu = – e 2gh. Fig. 2.25
u-0
Minus sign indicates that after collision the direction of velocity becomes opposite.
Now height attained after first collision
2
1 v 2 (e 2 gh )
mgh1 = mv12 fi h1 = 1 = = e2 h .
2 2g 2g
(b) Before the second collision, velocity of the ball is v1 = e 2 gh
\ Velocity after the second collision,
v2 = – ev1
= - e ( - e 2 gh ) = + e 2 2 gh .
Height attained after the second collision,
1 1
mgh2 = mv22 = m(e 2 2 gh )2
2 2
fi h2 = e4h.
After nth collision:
(i) vn = en 2gh
(ii) hn = e2nh.
(iii) Theoretically, the ball will make infinite number of collisions with the ground.
Thus, the total distance travelled till last
S = h + 2[h1 + h2 + h3 + ... •]
= h + 2[e2h + e4h + e6h + ... •]
= h + 2e2h[1 + e2 + e4 + ... •]
ŽůůŝƐŝŽŶƐപ155

È 1 ˘ È 2e 2 ˘
= h + 2 he 2 Í 2˙
= h Í 2˙
Î1 - e ˚ Î1 - e ˚
Ê 1 + e2 ˆ
= hÁ 2˜
Ë1- e ¯
Ê 1 + e2 ˆ
or S =Á 2˜
h
Ë1- e ¯
(iv) Total time of motion T = t + 2(t1 + t2 + ... •)

2h È 2 h1 2 h2 2 h3 ˘
= + 2Í + + + ºº • ˙
g Î g g g ˚
2h 2
= +2 È h1 + h2 + h3 + ºº • ˘˚
g gÎ

2h 2È 2
= +2 e h + e 4 h + e 6 h + ºº • ˘˚
g gÎ

2h 2h
= +2 È e + e 2 + e 3 + ºº • ˘˚
g g Î

Ê 2h ˆ Ê 2h ˆ È 1 ˘
= ÁË g ˜¯ + 2 ÁË g ˜¯ e Í 1 - e ˙
Î ˚

Ê 2h ˆ È 2e ˘ Ê 2h ˆ Ê 1 + e ˆ
= ÁË g ˜¯ Í1 + 1 - e ˙ = ÁË g ˜¯ ÁË 1 - e ˜¯
Î ˚

Ê 1 + eˆ 2h
or T =Á
Ë 1 - e ˜¯ g
(v) Change in momentum after the first impact
= mv1 – (–mv0) = m(v1 + v0)
Change in momentum after the second impact = m(v2 + v1).
Change in momentum after the nth impact = m(vn + vn–1).
Total change in momentum
= m(v1 + v0 + v2 + v1 + ... vn + vn –1)
= m(v0 + 2v1 + 2v2 + ... 2vn)
= mv0 [1 + 2e + 2e2 + ...]

È Ê 1 ˆ˘ È1 + e ˘
= mv0 Í1 + 2e Á ˜¯ ˙ = m 2 gh Í 1 - e ˙
Î Ë 1 - e ˚ Î ˚
156പWŚLJƐŝĐƐĨŽƌ//dͲ:͗DĞĐŚĂŶŝĐƐ//

 dp 
Dp
(c) Now as F = so, Fav =
dt DT
Substituting the value of DT and Dp FROM %QS I AND II
È1 + e ˘ È1 - e ˘
g
Fav = m 2 gh Í ˙¥ Í 1 + e ˙ = mg
Î1 - e ˚ Î 2h˚

Problem 2.24 A particle of mass 1.0 g moving with velocity v1 = 3.0 i – 2.0 j experiences

a perfectly inelastic collision with another particle of mass 2.0 g and velocity v 2 = 4.0 j –

6.0 k. Find the velocity of the formed particle (both the vector v and its modulus), if the
components of the vectors v1 and v2 are given in the SI units.
Solution By conservation of momentum, we have
  
m1 v1 + m2 v 2 = (m1 + m2) v
 
 È m v + m2 v 2 ˘
\ v = Í 1 1 ˙
Î m1 + m2 ˚
)
1 ¥ (3.0i - 2.0j) + 2( 4.0j - 6.0 k
=
1+ 2
   ) m/s
= (i + 2.0 j - 4.0 k
Problem 2.25 A block of mass 2.0 kg is moving on a frictionless horizontal table with a
velocity of 1.0 m/s towards another block of equal mass kept at rest (see Fig. 2.26). The
spring constant of the spring fixed at one end is 100 N/m. Find the maximum compression
of the spring.
1.0 m/s Rest v v

2.0 kg 2.0 kg 2.0 kg 2.0 kg

Fig. 2.26

Solution Maximum compression will occur when their velocities become equal (i.e.,
approach velocity becomes zero). Let the velocity of each block at this instant be v. Then by
conservation of momentum, we have m1u1 + m2u2 = m1v1 + m2v2
As u2 = 0 and v1 = v2 = v
2 ¥ 1 + 0 = 2v + 2v
1
\ v =
m/s.
2
Now by conservation of mechanical energy, we have
2 2
1 1 Ê 1ˆ 1 Ê 1ˆ 1
¥ 2 ¥ 12 = ¥ 2 ¥ Á ˜ + ¥ 2 ¥ Á ˜ + ¥ 100 ¥ xmax
2
2 2 Ë 2¯ 2 Ë 2¯ 2
where xmax is the maximum compression of the spring.
ŽůůŝƐŝŽŶƐപ157

1
As v= m/s
2
2 2
1 1 Ê 1ˆ 1 Ê 1ˆ 1
\ ¥ 2 ¥ 12 = ¥ 2 ¥ Á ˜ + ¥ 2 ¥ Á ˜ + ¥ 100 ¥ xmax
2
2 2 Ë 2 ¯ 2 Ë 2 ¯ 2
After solving, we get
xmax = 0.1 m
Problem 2.26 Two blocks of mass 3 kg and 6 kg 2.0 m/s
1.0 m/s
respectively are placed on a smooth horizontal surface.
They are connected by a light spring of force constant 3kg 6kg
k = 200 N/m. Initially, the spring is unstretched. The
indicated velocities are imparted to the blocks. Find the
maximum extension of the spring. Fig. 2.27
Solution At maximum extension their velocities are same.
This common velocity is given by
Total momentum
v =
Total mass
2¥ 6 -1¥ 3
=
3+6
= 1 m/s
Now, Ei = Ef
1 1 1 1
\ ¥ 6 ¥ (2)2 + ¥ 3 ¥ (1)2 = ¥ 9(1)2 + ¥ 200 ¥ X m2
2 2 2 2
Solving we get, Xm = 0.3 m
= 30 cm
Problem 2.27 Two blocks of masses m1 = 2 kg and k
u1 m2 u2
m2 = 5 kg are moving in the same direction along a m1
frictionless surface with speeds 10 m/s and 3 m/s
respectively, m2 being ahead of m1. An ideal spring
Fig. 2.28
with k = 1120 N/m is attached to block side of m2.
Find the maximum compression of the spring when the blocks collide.
Solution If v is the speed of the blocks at the instant of maximum compression, then
m1u1 + m2u2 = (m1 + m2)v
1 1 1 1 2
and m1u12 + m2 u22 = (m1 + m2 )v 2 + kxmax
2 2 2 2
After substituting the given values and simplifying, we get
xmax = 0.25 m
158പWŚLJƐŝĐƐĨŽƌ//dͲ:͗DĞĐŚĂŶŝĐƐ//

ELASTIC OBLIQUE COLLISION OR ELASTIC COLLISION IN TWO DIMENSIONS


When line joining the geometric centres of the colliding bodies is different from the line of
motion; this results in oblique collision.
Consider a body of mass m1 moving along x-axis with a velocity u, makes elastic oblique
collision with another body of mass m2, initially at rest. Let after collision their velocities
become v1 and v2, making angles q1 and q2 with the x-axis.
v1 sin q1 v1

q1
v1 cos q1
u
x
m1
v2 cos q2
q2

v2
v2 sin q2

Fig. 2.29

Using principle of conservation of momentum;


(i) along x-axis m 1u 1 + m 2u 2 = m 1v 1 + m 2v 2
m1u + 0 = m1v1 cos q1 + m2v2 cos q2 ...(i)
(ii) along y-axis
0 + 0 = m1v1 sin q1 – m2v2 sin q2 ...(ii)
As the collision is elastic, so we have
1 1 1
m1u2 + 0 = m1v12 + m2 v22 ...(iii)
2 2 2
Here we have three equations, but four unknowns. To get all unknowns, some
experimental data is to be provided. Assuming q1 + q2 = q, called deflection angle,
and solving above equations, we get
m1
sin q = . Here m1 £ m2 ...(iv)
m2
Important Cases:
Method A: u
(i) Glancing collision: For such colli-
sions, u

Here q1 = 90° and q2 = 0° m1


&ROM %QS I AND II WE GET m2
v1 = u and v2 = 0,
Fig. 2.30
1
\ +% OF THE BODY m2 = m2 v22 = 0
2
ŽůůŝƐŝŽŶƐപ159

(ii) If m1 = m2 = m THEN FROM %Q IV WE GET


sin q = 1,
or q = q1 + q2 = 90°
Method B: By the vector method we can also prove
  
mu = mv 1 + mv 2
  
or u = v1 + v 2
     
or u ◊ u = ( v1 + v 2 ) ◊ ( v1 + v 2 )
 
or u2 = v12 + v22 + 2v1 ◊ v 2
As v12 + v22 = u2, thus we have
 
v1 ◊ v 2 = 0
That is, q = q1 + q2 = 90°
Hence, if two identical bodies make elastic oblique collision, they will move
perpendicular to each other after collision.

WORKED PROBLEMS
Problem 2.28 Two identical balls marked B and C, in contact with each other and at rest
on a horizontal smooth surface, are hit head-on by another identical ball marked A moving
initially with speed v as shown in Fig. 2.31. If collision is elastic. Then which of the case(s)
are possible?
Solution
Before collision After collision
v B C v/2
A B C (i) A
v=0

(ii) v
Momentum = mv A B C
KE = 1/2 mv 2 v=0

(iii) v/3
A B C

Fig. 2.31

Since collision is elastic, the kinetic energy and momentum before and after collision
remain constant.
v
Case (i) momentum = 2m ¥ = mv
2
2
1 Ê vˆ 1
+%  (2m) Á ˜ = mv 2
2 Ë 2 ¯ 4
Case (ii) momentum = mv
1
+%  mv 2
2
160പWŚLJƐŝĐƐĨŽƌ//dͲ:͗DĞĐŚĂŶŝĐƐ//

Case (iii) momentum = 3m ¥ v/3 = mv


2
1 Ê vˆ 1
+%  (3m) Á ˜ = mv 2
2 Ë 3¯ 6
Therefore, only case (ii) is the possible one.
Problem 2.29 A ball moving with a velocity v strikes a wall moving towards the ball with
a velocity u. An elastic impact occurs. Determine the velocity of the ball after the impact.
What is the cause of change in kinetic energy of the ball? Consider the mass of the wall to
be infinitely great.
Solution Velocity of ball w.r.t. wall before collision = v – (–u) = v + u.
After collision the velocity of ball w.r.t. wall will be = – (v + u).
Velocity of ball relative to ground = – (v + u) – u = – (v + 2u).
The change in kinetic energy
1 1 v u
m(v + 2u)2 - mv 2 = 2mu (u + v)
2 2
Reaction force from wall
DP m[-(v + 2u) - v] Fig. 2.32
F = =
Dt Dt
2m(v + u)
or F =
Dt
where Dt is the duration of collision.
Work done by this force
Work = Force ¥ Displacement
2m(v + u)
W = Fs = ¥ uDt = 2m(v + u)u.
Dt
4HUS WE CAN SAY THAT +% OF THE BALL CHANGES DUE TO THE WORK DONE BY THE WALL
Problem 2.30 A gun is mounted on a railroad car. The mass of the car, the gun, the shell
and the operator is 50 m where m is the mass of one shell. If the muzzle velocity of the shell
is 200 m/s, what is the recoil speed of the car after the second shot? Neglect friction.
Solution Let v1 be the velocity of car after firing 1st shot. By conservation of momentum,
we have
m
0 = (49m)v1 + m ¥ 200
200
or v1 = m/s
49 v1
49m
After firing 1st shot, let velocity of shell w.r.t. the
ground is vshell’ then
vshell – vgun = 200
Fig. 2.33
\ vshell = 200 + vgun = 200 – v1
200 48
= 200 - = 200 ¥
49 49
ŽůůŝƐŝŽŶƐപ161

Now by conservation of momentum, we have


200 Ê 48 ˆ
- ( 49m) ¥ = 48 mv2 + m ¥ Á 200 ¥ ˜
49 Ë 49 ¯
Ê 1 1ˆ
After solving, we get v2 = - 200 Á +
Ë 48 49 ˜¯
m/s

Problem 2.31 A small particle travelling with a velocity v collides elastically with a spherical
body of equal mass and radius r initially kept at rest. The centre of the spherical body is
located at a distance (r < r) away from the direction of motion of the particle. Find the final
velocity of the particle.
Solution During collision:
r
sin q =
r
r2
cos q = 1 -
r2
v
r r

x
q
sin
v
r v v cosq
q
y

Fig. 2.34

Using conservation of momentum along y-axis, we get


m(v cos q) + 0 = mv1 + mv2 ...(i)
1 1 1
and m(v cos q )2 + 0 = mv12 + mv22 ...(ii)
2 2 2
After solving above equations we get
v1 = 0 and v2 = v cos q

v r2 - r2
=
r
The resultant velocity of the particle will be v sin q along x-axis (see Fig. 2.34).
Note: The collision will take place along normal direction. In the tangential direction no force
acts and the velocity does not change in this direction.
162പWŚLJƐŝĐƐĨŽƌ//dͲ:͗DĞĐŚĂŶŝĐƐ//

Problem 2.32 Two identical balls in contact on a floor are in


equilibrium. A third ball collides them simultaneously symmetri-
cally and remains at rest after the impact. Calculate the coefficient
of restitution between the balls.
Solution
Fig. 2.35
Line of collision
v
v

°
30
s
co
u 30°

u
30°
u
30°
v
(a) (b)
Fig. 2.36

Applying conservation of momentum in the direction of motion of the colliding ball


mu = mv cos 30° + mv cos 30°
or u = 3v
Coefficient of restitution
Ê v - v2 ˆ
e=Á 1
Ë u1 - u2 ˜¯
where velocities of balls are along the line of collision.
(0 - v ) v v 2
\ e = = = = Ans.
(u cos 30∞ - 0) u cos 30∞ 3 3
3v ¥
2

Problem 2.33 A smooth ball collides at a velocity v on a group of three smooth identical
blocks resting on a smooth horizontal surface as shown in Fig. 2.37. The mass of each block
is equal to the mass of the ball. The diameter of the ball and its height are equal to the edge
of the block. Determine the velocity of the blocks after the impact.
Solution

v 4
2

Fig. 2.37
ŽůůŝƐŝŽŶƐപ163

The ball collides with the blocks 3 and 1 symmetrically, therefore both will move with
the same speed. After collision the situations of blocks are as shown in Fig. 2.38. Let each
of them have speed v¢.

45° v
2


Fig. 2.38

Using conservation of momentum along x-axis, we have


mv = mv¢ cos 45° + mv¢ cos 45°
v
\ v¢ =
2

Problem 2.34 A grenade explodes in air when it has a horizontal speed of v. It breaks into
two identical pieces of equal mass. If one goes vertically up at a speed of 4v, find the velocity
of the other immediately after the explosion.
Solution Momentum of grenade before explosion
 2v
P = mv î
Using conservation of momentum m/2
   m v
P = P1 + P2 m/2
  
\ P2 = P - P1
v2
 m
where P1 = ¥ 4v ĵ Fig. 2.39
2

\ P = mv ˆi - 2mvˆj
2
m
or v 2 = mv iˆ - 2mvˆj
2

or v 2 = 2v ˆi - 4vˆj
Problem 2.35 Two identical buggies A and B with one man in each move without friction
due to inertia along the parallel rails towards each other. When the buggies get opposite to
each other, the men exchange their places by jumping in the direction perpendicular to the
motion direction. As a consequence, buggy A stops and buggy B keeps moving in the same
direction, with its velocity becoming equal to v. Find the initial velocities of the buggies v1
and v2 if the mass of each buggy (without a man) equals M and the mass of each man m.
164പWŚLJƐŝĐƐĨŽƌ//dͲ:͗DĞĐŚĂŶŝĐƐ//

Solution Let v1 and v2 be the initial velocities of the v1 ¨ +ve


buggies. The direction of v1 is taken as positive and
that of v2 as negative.
The momentum of the man in the buggy B will
be mv2 in the direction of the motion of the buggy –ve ¨ v2
B. When he jumps into buggy A perpendicular to its
motion, as the buggy stops, its momentum becomes Fig. 2.40
zero
Mv1 – mv2 = 0 ...(i)
Man in buggy 1 jumps into buggy 2 perpendicular to its direction, then
Mv2 – mv1 = (M + m)v ...(ii)
3OLVING %QS I AND II WE GET
mv
v1 = -
( M - m)
mv
and v2 = -
( M - m)

Problem 2.36 A spherical ball 1 of mass m is released from rest on a frictionless bowl 0.2
high. The sphere slides down and collides elastically with another sphere 2 of mass m/4
placed on the bottom of the bowl. If ball 2 has to just reach the top and escape the bowl,
calculate from where 1 should be released?
Solution Let ball 1 be released from height h.
1
Let v be the velocity of ball 2 just after collision, then m
0.2 m
1 h m/4
(m / 4)v 2 = (m/4)g ¥ 0.2 2
2
which gives v = (0.2 g ¥ 2) = 2 m/s Fig. 2.41

Now elastic collision between ball 1 and ball 2


mv1 + 0 = mv¢1 + (m/4) ¥ 2 ...(i)
where v1 and v¢1 are the velocities of ball 1 before and after collision.
(v¢1 - 2)
Also, 1 = ...(ii)
(v1 - 0)
Solving the above equations, we get v1 = 1.25 m/s
1
Now, mv12 = mgh
2
which after solving gives h = 0.078 m.
Problem 2.37 A rubber ball strikes a wall at an angle a with the horizontal. It rebounds
rubber at an angle q with the horizontal. Calculate the coefficient of restitution between the
rubber ball and the wall.
ŽůůŝƐŝŽŶƐപ165

Solution Since the line of collision is AB, therefore the velocity u sin a
of the ball along a line parallel (normal to AB) to wall does not u
change.
\ u sin a = v sin q ...(i) u cos a
a
A B
( - v cos q - 0) v cos q
and e = ...(ii) q Wall
(u cos a - 0)
where u and v are the velocities of the ball before and after v sin q
collision. The velocity of the wall before and after collision is v
practically zero.
Fig. 2.42
Solving the above equations, we get
e = tan a/tan q.
Problem 2.38 A ball moving straight collides elastically with another stationary ball of the
same mass. At the moment of impact the angle between the straight line passing through
the centres of the balls and the direction of the initial motion of the striking ball is equal
to a = 45°. Assuming the balls to be smooth, find the fraction h of the kinetic energy of the
striking ball that turned into potential energy at the moment of the maximum deformation.
Solution Let the velocity of the colliding ball be y
u. Applying the law of conservation of momentum u y
along x-axis, we have
m 1u 1 + m 2u 2 = m 1v 1 + m 2v 2 a x
x
m m
mu cos a + 0 = mv1x + mv2x
At the maximum deformation Fig. 2.43
v1x = v2x
\ u cos a + 0 = v1x + v2x = 2v1x
u cos a
or v1x =
2
1 1
)NITIAL +% OF THE SYSTEM  mu2 + 0 = mu2
2 2
1 1 1 mu2 cos 2 a
&INAL +% OF THE SYSTEM  mv12x + mv22x = 2 ◊ mv12x =
2 2 2 4

È mu2 cos 2 a ˘
Final K% ÍÎ 4
˙
˚
Now, h = =
Initial K% 1 2
mu
2
cos 2 45∞
= = 0.25
2
166പWŚLJƐŝĐƐĨŽƌ//dͲ:͗DĞĐŚĂŶŝĐƐ//

Problem 2.39 A train of mass M is moving on a circular track of radius R with constant
speed V. The length of the train is half of the perimeter of the track. The linear momentum
of the train will be
2MV
(a) zero 5 (b) (c) MVR (d) MV
p
2R
Solution If we treat the train as a ring of mass M then its COM will be at a distance
p
from the centre of the circle. Velocity of centre of mass is
2R 2R Ê V ˆ Ê Vˆ
VCM = RCM ◊ w = ◊w = ◊Á ˜ ÁË∵ w = ˜¯
p p Ë R¯ R
2V 2MV
fi VCM = fi MVCM =
p p
As the linear momentum of any system = MVCM
So = 2 MV/p
Problem 2.40 Three identical discs B, C and D as shown in Fig. 2.44 rest on a smooth
horizontal plane. The disc B is set in motion with velocity v after which it experiences an
elastic collision simultaneously with the discs C and D. The distance between the centres of
the latter discs prior to the collision is h times greater than the diameter of each disc. Find
the velocity of disc B after the collision. At what value of h will the disc B recoil after the
collision, stop, move on?
hd / 2
Solution From Fig. 2.44 cos q = = h/ 2
d
v2
v2
q
C y B
v d q
B v1 hd v nd/2 v1
D x
q

v2
v2

Fig. 2.44

Applying the law of conservation of momentum along x-axis, we have


mv + 0 = mv1 + 2 mv2 sin q ...(i)
where v1 and v2 are the velocities of the discs after collision.
From Newton’s law of collision,
- (v1 sin q - v2 )
e=
v sin q
For perfectly elastic collision e = 1
- (v1 sin q - v2 )
\ 1 = ...(ii)
v sin q
ŽůůŝƐŝŽŶƐപ167

Solving the above equations, we get


v(1 - 2 sin 2 q )
v1 = ...(iii)
(1 + 2 sin 2 q )
After substituting the value of cos q IN %Q III WE GET

v(h 2 - 2)
v1 =
(6 - h 2 )
Disc B will stop after collision, v1 = 0

v(h 2 - 2)
or =0
(6 - h 2 )
or h = 2
For recoil, v1 should be negative or less than 0
or v1 < 0
\ h < 2
For move is its direction of initial motion v1 > 0

or h > 2
Problem 2.41 A chain BC of length l is located in a
smooth horizontal pipe so that its fraction of length h
hangs freely and touches the surface of the table with
its end C shown in Fig. 2.45 At a certain moment the
B
end B of the chain is set free. With what velocity will
this end of the chain slip out of the tube?
x h
Solution Suppose l is the mass per unit length of
the chain.
Let x be the length of the chain within the tube at C
any instant, then weight of hanging portion outside
the tube = lhg and weight of the portion inside the lx T
tube = lxg.
Let a be the acceleration of the chain at any instant.
Acceleration is variable because the length of the
chain inside the tube decreases as the chain slips out lhg
from the tube but force on it remains constant (T). By
Fig. 2.45
Newton’s second law
lhg – T = (lh)a ...(i)
and T = (lx)a ...(ii)
&ROM %QS I AND II WE HAVE
l(x + h)a = lhg
dv
or ( x + h) = hg ...(iii)
dt
168പWŚLJƐŝĐƐĨŽƌ//dͲ:͗DĞĐŚĂŶŝĐƐ//

vdv
or ( x + h) = – gh
dx
Negative sign is due to fact that as x decreases v increases
v 0
dx
or Ú vdv = - gh Ú ( x + h)
...(iv)
0 (l - h)

or v = 2gh ln (l / h)
Problem 2.42 A boat of mass M with a man of mass m abroad stays motionless on the
surface of a lake. The man moves a distance l¢ relative to the boat with velocity v¢ and then
stops. Assuming the water resistance to be negligible, find:
(i) the displacement of the raft l relative to the shore; and
(ii) the horizontal component of the force with which the man acted on the boat during
the motion.
Solution
(a) Since no net external force acts on
the system, the centre of mass re-
mains at rest.
m(l¢ / 2) + ml¢ m(l + l¢ / 2) + ml
\ = Shore
( M + m) ( M + m)
l
where l is the displacement of the
raft w.r.t. shore.
Solving the above equation, we get Fig. 2.46

ml¢
l = ...(i)
M+m
B $IFFERENTIATING %Q I WRT TIME TWICE WE HAVE

Ê dv¢ ˆ

d2l Ë dt ˜¯
a = 2 =
dt ( M + m)

The force with which the man acted on the raft is


mM Ê dv¢ ˆ
F = Ma = Á ˜
M + m Ë dt ¯

Problem 2.43 If L and L¢ be the forces of compression and restitution, then show that
L¢ = eL where e is the coefficient of restitution.
Solution Consider the head-on collision between two bodies of masses m1 and m2 are
moving in the same direction with velocities u1 and u2 (u1 > u2). If v is the common velocity
at the instant of collision, then loss of momentum of the first ball during compression.
ŽůůŝƐŝŽŶƐപ169

= m1(u – v)
and the gain in momentum of the second ball
= m2(v – u2)
By impulse-momentum theorem, we can write
L = m1(u1 – v) = m2(v – u2)
L L
or + = u1 – u 2 ...(i)
m1 m2

u1 u2 v v1 v2

Before collision During collision After collision

Fig. 2.47

If v1 and v2 are their velocities after collision, then loss of momentum of the first ball
= m1(v – v1),
and gain in momentum by the second ball
= m2(v2 – v).
Again by impulse-momentum theorem, we have
L¢ = m1(v – v1) = m2(v – v2)
L¢ L¢
or + = – (v1 – v2) ...(ii)
m1 m2
.OW FROM %QS I AND II WE GET
L¢ v - v2
=- 1
L u1 - u2
È v - v2 ˘
As -Í 1 ˙ = e,
Î u1 - u2 ˚
\ L¢ = eL
Problem 2.44 A block A of mass 2m is placed on another block B of mass 4m which, in
turn, is placed on a fixed table. The two blocks have the same length 4d and they are placed
as shown in Fig. 2.48. The coefficient of friction (both static and kinetic) between the block
B and the table is m. There is no friction between the two blocks. A small object of mass m
moving horizontally along a line passing through the centre of mass (cm see Fig. 2.48) of
block B and perpendicular to its face with a speed v collides elastically with block B at a
height d above the table. (IIT-JEE, 1991)
(a) What is the minimum value of v (call it v0) required to make block A topple?
(b) If v = 2v0, find the distance (from the point P in Fig. 2.48) at which the mass m falls on
the table after collision (ignore the role of friction during the collision).
170പWŚLJƐŝĐƐĨŽƌ//dͲ:͗DĞĐŚĂŶŝĐƐ//

A
2m 2m Block A is about to topple

B cm v1 v2
v 2mg
m 2d 2mg
d 4m

4d 2d
x

Fig. 2.48

Solution
(a) When the object collides with block B, the block moves forward. As there is no friction
between the two blocks, due to inertia block A remains at its position. For toppling of
the block A, its c.g. must be out of the block B. For this minimum distance traversed
by block B must be 2d.
Friction between block B and the table is m(6mg). Block B moves when it experiences a
force little greater than the frictional force. The work done in moving block B through
a resistance 2d will be
W = F ◊ s = m(6mg) ¥ 2d = 12 mmgd ...(i)
Now consider the collision between the moving mass m and the block B. From the law
of conservation of momentum, we have
mv + 0 = mv1 + (4m)v2 ...(ii)
where v1 and v2 are the velocities after collision of the moving mass m and the block
after collision.
The collision is completely elastic
(v1 - v2 )
\ 1 =- ...(iii)
v-0
Solving the above equations, we get
v1 = – 3/5 v and v2 = 2/5 v.
1 1
When v = v0 THE +% ACQUIRED BY THE BLOCK B = ( 4 m)v22 = ( 4 m)(2 / 5v0 )2 .
2 2
4HE +% ACQUIRED BY THE BLOCK B is just sufficient to move the block a distance 2d against
the frictional force, i.e., 12 mmgd.
1
or ( 4 m)(2 / 5v0 )2 = 12 mmgd
2
5
or v0 = 6m gd
2
(b) The velocity of the mass after collision is – 3/5 v. Its value corresponding to v = 2v0 is
equal to – 6/5 v0.
The mass m being at a height d above the table. The time taken by it to fall through a
height d will be
1
d = gt 2 fi t = (2d / g )
2
ŽůůŝƐŝŽŶƐപ171

The horizontal distance travelled by the mass in this time with horizontal velocity
(–6/5 v0).
6 5
x = v ◊ t = (– 6/5 v0) . (2d / g ) = - ¥ 6 m gd ¥ (2d / g )
5 2
or x = - 6d 3m
Problem 2.45 B and C are two identical blocks of same mass 2m and same physical
dimensions. B is placed over C which is attached to one end of the spring of natural length l
and spring constant k. The other end of the spring is attached to a wall. The system is resting
on a smooth horizontal surface with the spring in the relaxed state. A small object of mass
m moving horizontally with speed v at height d above the horizontal surface hits the block
C along the line of their centre of mass in the perfectly elastic collision. There is no friction
between B and C.
(a) Find the minimum value of V (say V0) such that block B will topple over block C
(b) If V0 = V/2, find the period of oscillation of the block spring system and amplitude.
(c) What is the energy stored in the spring when the block C returns to its initial position
as before collision?

B
B 2d

k 2mg C
m V C
d
d
2d
Fig. 2.49

Solution
(a) The upper block will topple down if the lower block displaces through a minimum
distance d as shown in Fig. 2.49. When the lower block displaces distance d, the
spring also compresses by d. The energy required to compress the spring will be =
1 2 1 2
kx = kd .
2 2
Consider the elastic collision between the moving mass m and the lower block B.
Applying conservation of momentum, we have
mV + 0 = mv1 + (2m)v2 ...(i)
(v1 - v2 )
and 1 =- ...(ii)
V -0
where v1 and v2 are the velocities of the mass m and the block C after collision.
Solving the above equations, we get
v1 = – V/3 and v2 = 2V/3.
1 4 mV 2
4HE +% OF THE BLOCK C after collision = (2m)(2V / 3)2 =
2 9
172പWŚLJƐŝĐƐĨŽƌ//dͲ:͗DĞĐŚĂŶŝĐƐ//

This energy will be equal to the energy required to compress the spring (1/2) kd2.
1 2 4 mV 2
\ kd =
2 9
k
or V = V0 = 3 d
8m
1 2
(b) Given that V = V0 4HE MAXIMUM +% OF THE OSCILLATING PARTICLE  MAXIMUM 0%  kA
2
1 2 4 mV 2
\ kA =
2 9
or A = (V0 / 3) (8 m / k )
2m
The time period of motion = 2p
k
(c) When block B returns to its initial position, the spring will be in its normal state.
Hence, energy stored in it is zero.
Problem 2.46 Three balls A, B and C of masses 2 kg, 4 kg and
8 kg, respectively, move along the same straight line and in the A B C
same direction, with velocities 4 m/s, 1 m/s and 3/4 m/s. If
A collides with B and subsequently B collides with C, find the
Fig. 2.50
velocity of ball A and ball B after collision, taking the coefficient
of restitution as unity.
(a) VA = 3, VB = 9/4 (b) VA = 0, VB = 3
(c) VA = 3, VB = 0 (d) VA = 0, VB = 0
Solution First consider the collision of balls A and B. Let the velocities of these two balls
after their collision be v and v¢.
Momentum after impact = Momentum before impact
\ 2v + 4v¢ = 2 ¥ 4 + 4 ¥ 1 4 m/s 1 m/s v v¢

\ 2v + 4v¢ = 12
A 2 kg B 4 kg A 2 kg B 4 kg
fi v + 2v¢ = 6 …(i)
Relative velocity after impact = – e ¥ relative
velocity before impact Fig. 2.51
v – v¢ = – 1 (4 – 1)
v – v¢ = (–3) …(ii)
3UBTRACTING %Q II FROM %Q I WE GET
3v¢ = 9
v¢ = 3 m/s
3UBSTITUTING IN %Q II WE GET
v – 3 = –3 fi v=0
Hence, after the collision ball A is brought to rest, 3 m/s 3/4 m/s V V¢
while ball B will move with a velocity of 3 m/s.
Now consider the collision of balls B and C. Let the B 4 kg C 8 kg B 4 kg C 8 kg
velocities of these balls after collision be V and V¢,
respectively.
Fig. 2.52
ŽůůŝƐŝŽŶƐപ173

Total momentum after impact = Total momentum before impact


3
4V + 8V¢ = 4 ¥ 3 + 8 ¥ = 18
4
9
V + 2V¢ = …(iii)
2
Relative velocity after impact = – e ¥ relative velocity before impact
V – V¢ = (– 1) (3 – 3/4)
V – V¢ = – 9/4 …(iv)
3UBTRACTING %Q IV FROM %Q III WE GET
3V¢ = 27/4 fi V¢ = 9/4 m/s
V =0
VA = 0, VB = 0
Problem 2.47 A big block of mass m1 = 150 kg is at rest on a very long frictionless floor, one
end which terminates in a wall. Another block of mass m2 is placed between the first block
and the wall, and set in motion towards m1 with constant speed u2. Assume that all collisions
are perfectly elastic, find the value of m2 for which both the blocks move with the same
velocity after m2 once with m1 and once with the wall. The wall has effectively infinite mass.

m1 Wall
u2 m2

Fig. 2.53

Solution Let v1 and v2 be the velocities of the blocks towards left just after collision.
Applying momentum conservation, we have m1u1 + m2u2 = m1v1 + m2v2 as u1 = 0
0 + m2u 2 = m 1v 1 + m 2v 2 ...(i)
(v1 - v2 )
and 1 =– ...(ii)
(0 - u2 )
Solving the above equations, we have
2m2 u2
v1 =
m1 + m2
(m1 - m2 )u2
and v2 = -
m1 + m2
The –ve sign with v2 indicates that block of mass m2 moves towards right (or wall). It
rebounds elastically from the wall with same velocity (v2). According to the given condition
both the blocks start moving with the same velocity.
174പWŚLJƐŝĐƐĨŽƌ//dͲ:͗DĞĐŚĂŶŝĐƐ//

i.e., v1 = v2
2m2 u2 (m - m2 )u2
or = 1
m1 + m2 m1 + m2
which after solving gives m2 = m1/3 = 150/3 = 50 kg.
Problem 2.48 A ball of mass m moving with velocity v0 experiences a head-on elastic
collision with one of the spheres of a stationary rigid dumbbell as shown in Fig. 2.54. The
mass of each sphere equal to m/2, and the distance between them is l, find the proper
angular momentum L of the dumbbell after the collision, i.e., the angular momentum in the
reference frame moving translationally and fixed to the dumbbell’s centre of inertia. (we do
not require size of sphere.)
Solution Let v1 and v2 be the velocities of m and m/2 after collision. m/2
Applying the law of conservation of momentum, we have m v0

m1u1 + m2u2 = m1v1 + m2v2 fi mv0 + 0 = mv1 + (m/2)v2 ...(i)


u2 = 0 l
Since the collision is perfectly elastic and hence
- (v1 - v2 ) m/2
1= ...(ii)
(v0 - 0) Fig. 2.54
Solving the above equations, we get
v1 = v0/3 and v2 = 4v0/3 towards right. Due to inertia the velocity of the lower mass m/2
will be zero.
Ê m ˆ 4v0 Ê m ˆ
ÁË ˜¯ +Á ˜ ¥0
Ë 2¯ 2v
The velocity of centre of mass, vCM = 2 3 = 0
m m 3
+
2 2
The angular momentum of the dumbbell with respect to the centre of mass is
or LCM = r1 ¥ (mv1/2) + r2 ¥ (mv2/2)
Ê l ˆ Ê m 2v ˆ Ê l ˆ Ê m 2 v ˆ
or LCM = Á ˜ Á ¥ 0 ˜ + Á ˜ Á ¥ 0 ˜
Ë 2¯ Ë 2 3 ¯ Ë 2¯ Ë 2 3 ¯
 mv0 l
or L CM = k̂
3
Problem 2.49 A small sphere of mass 10 g is attached to a point of a smooth vertical wall
by a light string of length 1 m. The sphere is pulled out in a vertical plane perpendicular
to the wall so that the string makes an angle of 60° with the wall and is then released. It is
found that after the first rebound the string makes a maximum angle of 30° with the wall.
#ALCULATE THE COEFlCIENT OF RESTITUTION AND THE LOSS OF +% DUE TO IMPACT )F ALL THE ENERGY IS
converted into heat, find the heat produced by the impact.
Solution Let v1 be the velocity of the sphere just before the collision with the wall, then
0%  +%
1 1
mgh = m12 , where h = (1 – 1 cos 60°) =
2 2
\ v1 = g.
ŽůůŝƐŝŽŶƐപ175

If v2 is the velocity of the sphere after collision with the wall, then
1
mv22 = mg(1 – cos 30°)
2
or v2 = g(2 - 3 ) 30°
1m 60°
Now according to the Newton’s experimental law
( v2 - 0 )
e =-
(v1 - 0)

g( 2 - 3 )
= = 0.518
g
Fig. 2.55
1 1 1 1
,OSS OF +%  mv12 - mv22 = ¥ 0.01 ¥ g - ¥ 0.01 ¥ g(2 - 3 ) = 0.036 J
2 2 2 2

CENTRE OF MASS
The centre of mass is a representatory point at which the whole mass of the system is
supposed to be concentrated for describing its translatory motion.
For example, consider a hammer placed on a plane surface. If a force F is applied on it
in such a way that the line of action passes through the centre of mass of the hammer, then
the hammer will have only translatory motion. But when force is applied in such a way that
the line of force does not pass through the centre of mass, the hammer will have rotatory
motion in addition to translatory motion (see Fig. 2.56).

F F

F
F

Centre of mass Centre of mass

(a) Translatory motion (b) Translatory together with rotatory motion

Fig. 2.56

NEWTON’S SECOND LAW FOR SYSTEM OF PARTICLES


Consider a system consisting of two particles of masses m1 and m2. They are acted upon
 
by external forces F1external and F2external respectively in addition to internal forces between
 
them as shown in Fig. 2.57. Let at any instant their positions be r1 and r2 respectively. Their
accelerations are defined as:
 
 d 2 r1  d 2 r2
a1 = 2 and a 2 = 2
dt dt
176പWŚLJƐŝĐƐĨŽƌ//dͲ:͗DĞĐŚĂŶŝĐƐ//

According to Newton’s second law of motion, we have r


y F 2external
  
F1 2 + F1external = m1a1 ...(i) m2 r external
  external r
 F21
F net
and F21 + F2 = m2 a 2 ...(ii)
r r
!DDING %QS I AND II WE GET r rcm M r F1external
r2 F12
     
(F1 2 + F21 ) + (F1external + F2external ) = m1a1 + m2 a 2 r m1
r1
 
By Newton’s third law, we know that F12 = - F21
    O x
\ F1external + F2external = m1a1 + m2 a 2
Fig. 2.57
 external  
or Fnet = m1a1 + m2 a 2 ...(iii)

Suppose the total mass of the system is M. Then


m1 + m2 = M.
Thus, we can write the Newton’s second law for total mass of the system as
 external 
Fnet = Ma ...(iv)
.OW FROM %QS III AND IV WE GET
 
 m a + m2 a 2 IMP
a = 1 1
M
which defines the acceleration of total mass of the system, called acceleration of centre of
mass of system. Thus, we can write  
 m a + m2 a 2
a CM = 1 1 ...(1)
m1 + m2
%QUATION  CAN BE WRITTEN AS  
d 2 r1 d 2 r2
 +
m1m
d 2 rCM
dt 2
2
dt 2
= 2
dt m1 + m2
 
d 2 È m r + m2 r2 ˘
= 2Í 11
dt Î m1 + m2 ˙˚
 
 m1 r1 + m2 r2
or r CM = ...(2)
m1 + m2
%QUATION  DElNES THE POSITION OF CENTRE OF MASS AT WHICH TOTAL MASS m1 + m2) is
supposed to be concentrated.
Note: Internal forces contribute to the acceleration of any particle of the system. But they are
not considered for acceleration of the system as a whole.
Similarly, velocity of centre of mass can be defined as;
  
 d rCM m1 v 1 + m2 v 2
v CM = IMP ...(3)
dt m1 + m2
 
 m S + m2 S2
and SCM = 1 1 IMP
m1 + m2
ŽůůŝƐŝŽŶƐപ177

CENTRE OF MASS OF nǧ 


We take a system of n particles of masses m1, m2 ..., mn y
   mn
and position vectors r1, r 2 ,... r n relative to origin A as
shown in Fig. 2.58. The centre of mass of the system
can be defined as  M
rn
    m2
 m r + m2 r2 + º + m2 rn
r CM = 1 1 rCM

m1 + m2 + º + mn r2 m1

n r1
 1 
or r CM =
M
 mi ri ; M = m1 + m2 + ... + mn A
x
i=1

Cartesian Coordinates of Centre of Mass


z
If xCM, yCM and zCM are the Cartesian coordinates of
Fig. 2.58
the centre of mass of the system, then
È m x + m2 x2 + º + mn xn ˘
xCM = Í 1 1
 mx
m1 + m2 + º + mn ˙= M
Î ˚

È m y + m2 y2 + º + mn xn ˘
yCM = Í 1 1
 my
m1 + m2 + º + mn ˙= M
Î ˚

È m z + m2 z2 + º + mn zn ˘
yCM = Í 1 1
 mz
and ˙=
Î m1 + m2 + º + mn ˚ M
Similarly, for n-particle system, we can write
   
 È m1 v1 + m2 v 2 + º + mn v n ˘ Pnet
v CM = Í
m1 + m2 + º + mn ˙= M
Î ˚

CENTRE OF MASS OF A RIGID BODY



Centre of mass rcm of a rigid body can be defined as y

r CM = Ú
 dmr 1 
= Ú dmr
Ú dm M dm

where M is the total mass of the body.


In Cartesian coordinate system we have r
r

xCM =
Ú dmx , y = Ú dmy and zCM =
Ú dmz
CM x
Ú dm Ú dm Ú dm Q

 Fig. 2.59
Also r CM = xCM ˆi + yCM ˆj + zCM kˆ
178പWŚLJƐŝĐƐĨŽƌ//dͲ:͗DĞĐŚĂŶŝĐƐ//

WORKED PROBLEMS
Problem 2.50 Two particles of masses m1 and m2 are placed at a distance r. Find centre of
mass of the system of two particles.
Solution Let CM is at a distance xCM from m1, we have CM
m1 m2
m x + m2 x2
xCM = 1 1
m1 + m2 x
Let x1 = 0, then x2 = r r
m1 0 + m2 r È m2 r ˘
\ xCM = =Í ˙
Fig. 2.60
m1 + m2 Î m1 + m2 ˚

È m2 r ˘ È m1 r ˘
Thus, the CM of the system is at a distance Í ˙ from m1 and r – xCM = Í ˙
from m2. Î m1 + m2 ˚ Î m1 + m2 ˚

ÊM ˆ
Problem 2.51 The mass per unit length of a bar varies as m = Á 0 ˜ x, where M0 is a constant
Ë L ¯
and x is the distance of any point on the bar measured from one end. If L is the length of
the bar, then find its centre of mass.
Solution Choose an element of rod of length dx at a distance x from one of its ends. The
mass of the element
ÈM ˘ x
dm = Í 0 x ˙ dx
Î L ˚ x=0 dx

Centre of mass of the rod can be defined as Fig. 2.61 Bar or rod

L L
È M0 ˘
Ú dmx Ú ÍÎL
xdx ˙ x
˚
xCM = 0
L
= 0
L
È M0 ˘
Ú dm Ú ÍÎL
xdx ˙
˚
0 0

Úx
2
dx
|x 3 / 3|0L 2L
= 0
L
= 2
=
|x / 2|0L 3
Ú xdx
0

How to Calculate Centre of Mass of Semicircular Wire


Let M be the mass and r be the radius of semicircular wire. Take the origin at the centre of
the wire. Choose a small element of angular width dq at an angular position q as shown in
Fig. 2.62. The mass of the element
ŽůůŝƐŝŽŶƐപ179

y-axis
Ê mˆ
dm = Á ˜ dq
Ëp¯
dm
Its centre of mass from origin is at x = R cos q and
y = R sin q. The centre of mass of whole wire can be dq y
defined as:
p p q
Êm ˆ x-axis
Ú dmx Ú ÁË p dq ˜¯ R cos q
O
R x

xCM = 0
p
= 0
p
Êm ˆ Fig. 2.62
Ú dm Ú ÁË p dq ˜¯
0 0

|sin q|p0
= =0
|q|p0
p p
Êm ˆ
Ú dmy Ú ÁË p dq ˜¯ R sin q
and yCM = 0
p
= 0
p
Êm ˆ
Ú dm Ú ÁË p dq ˜¯
0 0

R|- cos q|p0 2R


= =
|q|p0 p
Ê 2R ˆ
Thus, the centre of mass coordinates of a semicircular wire are Á 0 ,
Ë p ˜¯
.

How to Calculate Centre of Mass of a Semicircular Plate


Let M be the mass and R be the radius of the plate. y-axis
Choose an element of thickness dr at a distance all over
the semicircular plate.
The mass of the element
dr
M 2M
dm = ¥ (2p rdr ) = (2p rdr )
Ê p R2 ˆ p R2 r
ÁË ˜
2 ¯
x-axis
Q
R
Because of symmetric distribution of mass of plate
about y-axis, its x coordinate of CM will be zero. The y Fig. 2.63
coordinate of CM of plate is given by
R

Ú dmy 2r
0
yCM = R
; where y=
p
Ú dm
0
180പWŚLJƐŝĐƐĨŽƌ//dͲ:͗DĞĐŚĂŶŝĐƐ//

R R
Ê 2M ˆ 2r
ÁË
pR 2˜
¯ Ú (2p rdr )
p
2Ú r 2 dr
2|r 3 /3
3|0R 4R
=
0
R
= 0
R
= 2
=
2M |r / 2|0R 3p
Ú 2p rdr
p R2 0
Ú rdr
0
Ê 4R ˆ
Thus, the centre of mass coordinates of a semicircular plate are Á 0 ,
Ë 3p ˜¯
.

WORKED PROBLEM
Problem 2.52 Find the centre of mass of an annular half disc shown in R2
Fig. 2.64.
Solution The area of the element, dA = (2prdr)
R1
This behaves like a wire of radius r, whose centre of mass is at a height
2r Fig. 2.64
of from base.
p
y-axis
m
Mass of the element, dm = ¥ ( 2p rdr ) dr
p (R22 - R12 )
R R
1 2 1 2 Ê 2r ˆ
Now, yCM =
mRÚ dmy =
mRÚ (dm) Á ˜
Ëp¯ r
1 1
x-axis
After substituting the values and simplifying, we get
Fig. 2.65
4(R12 + R1 R2 + R22 )
yCM =
3p (R1 + R2 )

Centre of Mass of a Hemisphere y-axis


Let M be the mass and R be the radius of the hemisphere.
Choose an element in the form of disc of thickness dy
at a distance y from the origin. The radius of the disc dy
x2 = (R2 – y2)
È M ˘ 2 y
The mass of the element = Í
2 3
˙ ¥ (p x ) dy R
x-axis
Í pR ˙ O
Î3 ˚ R

Fig. 2.66
È 3M ˘
=Í ¥ p (R2 - y 2 ) dy
Î 2p R3 ˙˚
R
1
M Ú0
The centre of mass of the hemisphere can be defined as yCM = dmy
R
1 Ê 3M ˆ
M Ú0 Ë 2p R3 ¯
2 2
= Á ˜ p (R - y ) dy ¥ y
ŽůůŝƐŝŽŶƐപ181

R
3
Ú (R
2
= y - y 3 ) dy
2R 3 0
R
3 È R2 y 2 y 4 ˘
R
3R
3 ÚÍ
= - ˙ = .
2R 0 Î 2 4 ˚0 8
Ê 3R ˆ
Thus, the centre of mass coordinates of a hemisphere are Á 0 , ˜.
Ë 8 ¯

CENTRE OF GRAVITY
Centre of gravity is the point at which the whole y
weight of the system is supposed to act. Consider a W2
system of two particles of weight W1 and W2. Their
centre of gravity can be defined as W
    r W2
 W r + W1 r2 m1 g1 r1 + m2 g 2 r 2 r2
r
r Centre of Gravity = 1 1 = rCG W1
W1 + W2 m1 g1 + m2 g 2
r
If value of gravity is same every where, then r1 W1
g1 = g2, we have
  O x
 m1 r1 + m2 r2 
r Centre of Gravity = = r CM
m1 + m2
Thus, if g is same everywhere, then centre of
z
gravity and centre of mass lie at the same point.
Therefore, for practical objects, centre of mass and Fig. 2.67
centre of gravity are not differentiable.

WORKED PROBLEM
Problem 2.53 Two small identical bodies are at a separation equal to the radius of the earth
(R) are situated in such a way, that one of them is at the earth’s surface. Find:
m
(i) Centre of mass of the system of bodies CM
CG R
(ii) Centre of gravity of the system of bodies
m
Solution
(i) If m is the mass of each body, then centre of mass (CM) R

m ¥ 0 + mR Earth
yCM =
m+m
R
=
2 Fig. 2.68
(ii) If g is the value of gravity at earth’s surface, then its value at a height R,
g g g
g¢ = 2
= 2
=
Ê hˆ Ê Rˆ 4
ÁË 1 + ˜¯ ÁË 1 + ˜¯
R R
182പWŚLJƐŝĐƐĨŽƌ//dͲ:͗DĞĐŚĂŶŝĐƐ//

The centre of gravity


mg
mg ¥ 0 + ¥R
4 R
ycg = =
mg 5
mg +
4 y
A2
CENTROID
It is the point at which the whole area is supposed to r A
concentrated. It can be defined as: r2 A1
r
  r
 A r + A2 r 2 r
r centroid = 1 1 r1
A1 + A2
x
O
How to Calculate Centroid of a Triangle?
Consider a triangle of width b and height h as shown in z
Fig. 2.70. Let us locate its centroid from the base. Choose
an element of width b1 and thickness dy at a distance y Fig. 2.69
from the base. B
Since DABC and A1BC1 are similar triangles, we can
write,
dy
b1 h - y Ê h - yˆ
= b1 = Á
Ë h ˜¯
or b A1 C1
h
b h
b1
y
Ê h - yˆ
\ Area of the element dA = b1dy = Á
Ë h ˜¯
bdy A C

1 b
Area of the triangle A = bh.
2 Fig. 2.70
The centroid of the triangle can be defined as,
h h
È (h - y) ˘

Ú (dA)y Ú ÎÍ h
bdy ˙ y
˚
y= 0
= 0
A Ê bh ˆ
ÁË ˜¯
2
h h
2 2 hy 2 y 3 h
Ú ÈÎ hy - y ˘˚ dy =
2
= 2
- =
h 0 h2 2 3 0 3
Ê 1ˆ
Thus, the centroid of triangle of any shape from its base will be Á ˜ times the height of
Ë 3¯
the triangle.

How to Calculate Centre of Mass of Right Circular Cone?


Consider a cone of height h. Let M be the mass and R be the radius of its base. Choose an
elemental disc of radius x and thickness dy at a height y from the base.
ŽůůŝƐŝŽŶƐപ183

In similar triangles A‘O’C and AOC, we have y

x R Ê h - yˆ C
= fi x=Á
Ë h ˜¯
R
h-y h

Ê M ˆ
The mass of the element dm = ¥ p x 2 dy dy A¢ x
Áp 2 ˜ O h
ÁË R h ˜¯
3
y
The centre of mass of the cone can be defined as
O x
h h A
1 1 Ê M ˆ B
yCM =
M0 Ú (dm)y =
M Ú0 Á p R2 h ˜
p x 2 dy ¥ y R
ÁË ˜¯
3 Fig. 2.71
h 2
3 Ê h - yˆ
= ÚÁ ˜ y dy
h Ë h ¯
0
h
3
Ú (h
2
= 3
+ y 2 - 2 yh) y dy
h 0

h
3
Ú (h
2
= y + y 3 - 2 y 2 h) dy
h3 0
h
3È h2 y 2 y 4 2y 3 ˘
h
h
= 3 ÚÍ + - h˙ =
h 0Î 2 4 3 ˚
0
4
h
Thus, the centre of mass of cone is at a height from the base.
4

WORKED PROBLEM
y
Problem 2.54 Find the centre of mass of a uniform
L-shaped lamina (a thin flat plate) with dimensions as 1m
shown in Fig. 2.72.
Solution The plate is of uniform density and same A2
thickness everywhere. So its CM will coincide with the L2
centroid. 2m 1m
Divide the given plate into two parts of area A1 and
A2 as shown in figure. We have A1 1m
L1
A1 = 2 ¥ 1 = 2 m2 with its centroid L1 (1, 1/2) and
A2 = 1 ¥ 1 = 1 m2 with its centroid L2 (1/2, 3/2) x
O (0,0)
The centroid of whole plate can be defined as; Fig. 2.72
 
 A1 r1 + A2 r 2
r centroid =
A1 + A2
184പWŚLJƐŝĐƐĨŽƌ//dͲ:͗DĞĐŚĂŶŝĐƐ//

1
 A1 x1 + A2 x2 2 ¥ 1 + 1 ¥ 2 5
x = = = m
A1 + A2 2+1 6
1 3
 A y + A2 y2 2 ¥ 2 + 1 ¥ 2 5
y = 1 1 = = m
A1 + A2 2+1 6

Remember
Centre of Mass of Some Bodies
h
yCM
h
1. yCM = Triangular plate
3
Fig. 2.73

h
h
2. yCM = yCM
4 O
Right circular cone

Fig. 2.74

4R R
3. yCM = yCM
3p
Semicircular plate

Fig. 2.75

2R
4. yCM = R
p yCM

Semicircular wire

Fig. 2.76

3R
5. yCM = R
8
yCM

Hemisphere

Fig. 2.77
ŽůůŝƐŝŽŶƐപ185

Centre of Mass of Residual Body y



Consider a body of mass M, let its CM be R
with respect to the origin of the coordinate M
system. Now mass m is cut off from the body m

at the position r, then the CM of the residual
r Cut-off
body can be defined as; R
  r
 MR - mr r
r CM =
M-m O x
If mass m is added into the body then CM
of resulting body can be defined as;
 
 MR + mr
r CM =
M+m z
Fig. 2.78

WORKED PROBLEMS
y
Problem 2.55 From a uniform disc of radius R, a circular
hole of radius R/2 is cut. The centre of the hole is at R/2
from the centre of the original disc. Locate the centre of
gravity of the resulting flat body. R
Solution The mass of the cut-out hole (–R/2, 0)
2 x
M Ê Rˆ M O
m= ¥pÁ ˜ =
pR 2 Ë 2¯ 4
Let the centre of disc be at the origin of coordinates.
Then CM coordinates of cut-out hole are (– R/2, 0).
Thus, by the definition of CM, we have
Fig. 2.79
 M Ê - Rˆ
 M¥0- ¥Á ˜
xCM =
MR - mr
= 4 Ë 2 ¯ = R and y = 0.
CM
M-m M-
M 6
4
R
The centre of mass of remaining disc is at a distance of from the origin.
6
y
Problem 2.56 Figure 2.80 shows a uniform disc of radius R,
from which a hole of radius R/2 has been cut out from the
left of the centre and is placed on right of the centre of disc.
Find the CM of the resulting disc.
Solution Mass of the cut-out disc
2 (–R/2, 0) (R/2, 0) x
M Ê Rˆ M
m= ¥pÁ ˜ =
pR 2 Ë 2 ¯ 4 O

Let centre of the disc be at the origin of coordinates, then R


we can write the CM of the system as,
Fig. 2.80
186പWŚLJƐŝĐƐĨŽƌ//dͲ:͗DĞĐŚĂŶŝĐƐ//

  
 MR - mr + mr ¢
x CM =
M-m+m
M Ê -R ˆ M Ê R ˆ
M¥0- Á ˜+ Á ˜
= 4 Ë 2 ¯ 4 Ë 2¯ = R
M M 4
M- +
and yCM = 0. 4 4

Problem 2.57 A solid cone of radius R is joined to a uniform solid hemisphere of radius
R. Both are made of same material. The centre of mass of the composite solid lies at the
common base. Find the height of cone.
Solution Let the height of the cone be h. Then with respect to its
base, the CM is at a height of h/4. The CM of the hemisphere is at
3R/8 below the base.
h
If r be the density of the material, then mass of the cone,
p R2 h pr R2 h h/4
m1 = r ¥ = R
3 3 O
3R/3
3 3
2p R 2pr R
and mass of hemisphere m2 = r ¥ =
3 3
As the CM of the whole system lies at the base, thus we have
Fig. 2.81
m y + m2 y2
yCM =0= 1 1
m1 + m2

Ê pr R2 h ˆ Ê h ˆ Ê 2pr R3 ˆ Ê - 3R ˆ
ÁË ˜ ¥Á ˜ +Á ˜Á ˜
3 ¯ Ë 4¯ Ë 3 ¯ Ë 8 ¯
0 =
m1 + m2
or h = 3R

Problem 2.58 The structure of water molecule is shown in Fig. 2.82. Locate CM of H2O
molecule.
Solution The CM will lie on line of symmetry shown as vertical line. Also the mass of H
atoms can be assumed concentrated at point P of figure.
Now, O
m1 = mass of oxygen molecule = 16 u
0.96 Å
m2 = mass assumed at P = 2u
104°
OP = distance between m1 and m2 1m
= 0.96 cos 52° = 0.59 Å
H P H
\ r1 = distance of centre of mass from oxygen molecule
m2 r 2 ¥ (0.59) Fig. 2.82
= = = 0.066 Å
m1 + m2 2 + 16
ŽůůŝƐŝŽŶƐപ187

Problem 2.59 A body of mass 2.5 kg is subjected to the forces Y 4N


shown in Fig. 2.83. Find the acceleration of the centre of mass. 6N
60°
Solution Take the X and Y axes as shown in Fig. 2.83. The
x-component of the resultant force is
X
Fx = – 6 N + (5 N) cos 37° + (6 N) cos 53° + (4 N) cos 60° 53°
= – 6 N + (5 N) ◊ (4/5) + (6 N) ◊ (3/5) + (4 N) ◊ (1/2 = 3.6 N.
5N 6N
Similarly, the y-component of the resultant force is
37°
Fy = 5 N sin 37° – (6 N) sin 53° + 4 N sin 60°
= (5 N) ◊ (3/5) – (6 N) ◊ (4/5) + (4 N) ◊ ( 3 / 2) = 1.7 N. Fig. 2.83
The magnitude of the resultant force is

F= Fx2 + Fy2 = (3.6 N)2 + (1.7 N)2  4.0 N

The direction of the resultant force makes an angle q with the x-axis where
Fy 1.7
tan q = = = 0.47.
Fx 3.6
The acceleration of the centre of mass is
F 4.0 N
aCM = = = 1.6 m/s2
M 2.5 kg

Problem 2.60 Consider a two-particle system with the particles having masses m1 and m2. If
the first particle is pushed towards the centre of mass through a distance d, by what distance
should the second particle be moved so as to keep the centre of mass at the same position?
Solution Consider Fig. 2.84. Suppose the distance of d d¢
m1 from the centre of mass C is x1 and that of m2 from C C
m1 m2
is x2. Suppose the mass m2 is moved through a distance x1 x2
d¢ towards C so as to keep the centre of mass at C.
Fig. 2.84
Then,
m 1x 1 = m 2 x 2 ...(i)
and m1(x1 – d) = m2 (x2 – d¢). ...(ii)
3UBTRACTING %Q II FROM %Q I
m1d = m2 d¢
m1
or, d¢ = d
m2
Problem 2.61 A cubical block of ice of mass m and edge L is placed in a large tray of mass
M. If the ice melts, how far does the centre of mass of the system “ice plus tray” come down ?
Solution Consider Fig. 2.85. Suppose the centre of mass of the tray is a distance x1 above
the origin and that of the ice is a distance x2 above the origin. The height of the centre of
mass of the ice-tray system is
188പWŚLJƐŝĐƐĨŽƌ//dͲ:͗DĞĐŚĂŶŝĐƐ//

mx2 + Mx1
x=
m+ M
When the ice melts, the water of mass m spreads on the x2
surface of the tray. As the tray is large, the height of water x1
is negligible. The centre of mass of the water is then on the 0
surface of the tray and is at a distance x2 – L/2 above the Fig. 2.85
origin. The new centre of mass of the ice-tray system will be
at the height,
È L˘
m Í x2 - ˙ + Mx1
x¢ = Î 2˚
m+M
mL
The shift in the centre of mass = x – x¢ = .
2(m + M )
2m

Problem 2.62 Find CM of (equilateral triangle) having side a a a


a
m ¥ 0 + ma + 2m m m
Solution xCM = 2 = 2a = a a
4m 4 2 Fig. 2.86
y
3a 2m 2
3 B Ê aˆ
m ¥ 0 + 2m ¥ a 2 AB = a 2 - Á ˜
Ë 2¯
2 =a 3
yCM = a a
4m 4 a = 3
2

ÊÊ aˆ a 3 ˆ m A m
CM Á Á ˜ , m(a, 0)
Ë Ë 2 ¯ 4 ˜¯ (0, 0) a
2

Fig. 2.87
Problem 2.63 Regular hexagon Æ
Masses at A and E can be placed at centre of AE, similarly masses at B and D can be placed
at centre of BD.
2m m
E D

G 2m 4m G 2m
m fi m
F H C (reducing method)
F 4 2m C
B
A m
2m

Fig. 2.88

a a
Solution Hence, yCM = 3 = AH xCM from A =
2 2
ŽůůŝƐŝŽŶƐപ189

4m 2m C
m 2m
a/2 a a/2

5a/4
m 4m
a/2 4m
9a/8
m 8m
a
F 9m
A
a/2 a/2

Fig. 2.89

Problem 2.64 A mass m after falling freely through a distance h, begins to raise a mass
M (M > m) connected to it by means of an inextensible string passing over a fixed pulley.
Calculate the time in which M returned to its original position.
Solution The velocity acquired by mass after falling h, u = gh . If after the jerk the velocity
of mass m is v, then
mu = (m + M) v
Ê mu ˆ m gh
or v =Á ˜ =
Ë m + M ¯ (m + M )
m
After this moment the retardation of mass m
h
Ê M - mˆ
a =Á g.
Ë M + m ˜¯
M

Fig. 2.90
If T is the required time, then
1 2
s = ut - at
2
1 Ê M - mˆ 2v ( M + m)
or 0 = vT - Á ˜ gT 2 or T =
2 Ë M + m¯ ( M - m) g

m gh M + m Ê 2m ˆ 2h
=2 ¥ = .
m+m M - m ÁË M - m ˜¯ g

Problem 2.65 From a solid cylinder of height H, a cone whose base coincides with the base
of the cylinder is scooped out so that the centre of mass of the remaining solid coincides
with the vertex of the cone. Find the height of the cone.
Solution Suppose h, R are the height and radius of the cone.
If r is the density, then mass of the cone,
Ê p R2 h ˆ
m = rÁ
Ë 3 ˜¯
,
190പWŚLJƐŝĐƐĨŽƌ//dͲ:͗DĞĐŚĂŶŝĐƐ//

H
h
CM of the cone is at a distance fromsss A.
4 CM of cone
Mass of the cylinder, M = r(pR2H), and its CM is at B A
G2 G G1
a distance H/2 from A.
The CM of the residual is at a distance of h from A.
We know that CM of the residual
CM of cylinder
Mr - mr ¢
x = Fig. 2.91
M-m
H Ê p R2 h ˆ h
r (p R2 H ) - rÁ ¥
2 Ë 3 ˜¯ 4
or h =
Ê p R2 h ˆ
r (p R2 H ) - r Á
Ë 3 ˜¯

After simplifying, we get h = (2 - 2 )H Ans.


x
Problem 2.66 The centre of mass of two masses m and m¢ move by distance when mass
5

m is moved by distance x and m¢ is kept fixed. The ratio is (JEE, 2015)
m
1
(a) 2 (b) 4 (c) (d) None of these
4
Ê xˆ m¢
Solution (m + m¢ ) Á ˜ = mx + m¢(0) fi =4
Ë 5¯ m
\ (b) is correct answer.
Problem 2.67 A man of mass m climbs a rope of length L suspended below a balloon of
mass M. The balloon is stationary with respect to the ground. If the man begins to climb up
the rope at a speed v (relative to rope) in upward direction then with what speed (relative
to ground) will the balloon move?
Solution Balloon is stationary
fi No net external force acts on it. Balloon
fi The conservation of linear momentum of the system (balloon +
man) is valid
     M
fi Mv b + mv m = 0, where v m = v mb + v b
  
fi Mv b = m[v mb + v b ] = 0
vb
where vmb = velocity of man relative to the balloon (rope)
m vm
 Man
 m v mb
fi vb =
M+m
mv Fig. 2.92
where vmb = v fi vb = and directed opposite to that of the motion
M+m
of the man.
ŽůůŝƐŝŽŶƐപ191

Problem 2.68 Two identical buggies move one after the other due to inertia (without
friction) with the same velocity v0. A man of mass m rides the rear buggy. At a certain
moment the man jumps into the front buggy with a velocity u relative to his buggy. The
mass of each buggy is M. Find the velocities with which the buggies will move afterwards.
Solution Initial momentum of rear buggy = (M + m)v0. The momentum of man when he
jumps = m(v1 + u), where v1 is the velocity of buggy as he jumps.
By the conservation of linear momentum
(M + m)v0 = Mv1 + m(v1 + u) Before jumping
fi v1(M + m) = (M + m)v0 – mu
m v0 v0
v1 = v0 – u M M
M+m
Initial momentum of front buggy = Mv0
Mv0 + m(v1 + u) = (M + m)v2 After jumping

Ê mu ˆ
Mv0 + m Á v0 - u + u˜ = (M + m)v2
Ë M+m ¯ M v1 v2
M
Ê mu ˆ
fi Mv0 + m Á v0 - u + u˜ = (M + m)v2
Ë M + m ¯
mMu Fig. 2.93
fi (M + m)v0 + = (M + m)v2
M+m
mMu
fi v2 = v0 +
( M + m )2
y
Problem 2.69 A toy is constructed as shown in Fig. 2.94. If the
2R O2 m2 O2
density of the material of the sphere is 12 times that of cone,
compute the position of the centre of mass of the toy. CM

Solution If the density of cone is r, then its mass will be m1 =


1 16 4R
p (2R)2 ( 4 R)r = p R3 r and its centre of mass O1 will be at a
3 3 O1 m1 O1
height (h/4) = (4R/4) = R from O on the line of symmetry, i.e., O
O
y1 = R. 2R
4 3 3
Similarly, the mass of the sphere m2 = p R (12 r ) = 16pR r =
3
Fig. 2.94
3m1 and its centre of mass will be at its centre O2, i.e., y2 = 5R
Now treating sphere and cone as point masses with their masses concentrated at their
centre of masses respectively and taking the line of symmetry as y-axis with origin at O, for
the centre of mass of the toy
m y + m2 y2
Ycentre of mass = 1 1
m1 + m2
m1 ¥ R + 3m1 ¥ 5R
=
m1 + 3m1
= 4R
192പWŚLJƐŝĐƐĨŽƌ//dͲ:͗DĞĐŚĂŶŝĐƐ//

i.e., centre of mass of the toy is at a distance 4R from O on the line of symmetry, i.e., at the
apex of the cone.
Problem 2.70 If the linear density of a rod of length L varies as l = A + Bx, compute its
centre of mass.
Solution Let the x-axis be along the length of the y
rod and origin at one of its ends as shown in Fig.
2.95. As rod is along x-axis for all points on it y and
x
z will be zero so,
x
YCM = 0 and ZCM = 0 O
dx
i.e., centre of mass will be on the rod. Now consider L
z
an element of rod of length dx at a distance x from
the origin, then dm = ldx = (A + Bx)dx Fig. 2.95
L L

So,
Ú xdm = Ú0 x( A + Bx) dx
XCM = 0 L L
Ú0 dm Ú0 ( A + Bx) dx
AL2 BL2
+
or XCM = 2 3
BL2
AL +
2
L(3 A + 2BL)
=
3(2 A + BL)
Note:
(i) If the rod is of uniform density.
i.e., l = constant = A, and B = 0
XCM = L/2 (which is expected)
(ii) If the density of rod varies linearly with x, i.e., l = Bx and A = 0
XCM = 2L/3
Problem 2.71 The position vector of three particles of mass m1 = 1 kg, m2 = 2 kg and m3 = 3
  
kg are r1 + (i + 4 j + k) m, r 2 + (i + j + k ) m and r3 + (2i - j - k) m respectively. Find the position
vector of their centre of mass.
Solution The position vector of centre of mass of the three particles will be given by
  
 m r + m2 r2 + m3 r3
r CM = 1 1
m1 + m2 + m3
Substituting the values, we get
           
 (1)( i + 4 j + k) + (2)( i + j + k) + (3)(2 i - j - 2k) 9 i + 3 j - 3k
r CM = =
1+ 2+ 3 6
 1   
r CM = (3 i + j - k )m
2
ŽůůŝƐŝŽŶƐപ193

Problem 2.72 A square lamina of side a and a circular lamina


of diameter a are placed touching each other as shown in Fig.
2.96. Find distance of their centre of mass from point O, the O
centre of square. (IIT-JEE Main (Online), 2016)

A1 d1 + A2 d2
Solution d=
A1 + A2
Fig. 2.96
Here d = distance of centre of mass from O.
Êp ˆ
( a 2 )(0) + Á a 2 ˜ ( a)
Ë4 ¯
\ d =
a 2 + (p / 4) a 2

Ê p ˆ
=Á a
Ë p + 4 ˜¯

y
Problem 2.73 Consider a rectangular plate of dimensions a ¥
b. If this plate is considered to be made up of four rectangles
a b
of dimensions ¥ and we now remove one out of four
2 2
b x
rectangles. Find the position where the centre of mass of the O
remaining system will be.

A1 X1 - A2 X 2
Solution XCM = . Here we are cutting so (–) a
A1 - A2
Fig. 2.97
Ê ab ˆ Ê a ˆ
( ab)(0) - Á ˜ Á ˜
Ë 4 ¯ Ë 4¯ a
= =-
ab 12
ab -
4
A1 y1 - A2 y2
yCM =
A1 - A2

Ê ab ˆ Ê b ˆ
( ab)(0) - Á ˜ Á ˜
Ë 4 ¯ Ë 4¯
=
ab
ab -
4
b
=-
12

Problem 2.74 Find the centre of mass of a uniform disc of radius a from which a circular
section of radius b has been removed. The centre of the hole is at a distance c from the centre
of the disc.
194പWŚLJƐŝĐƐĨŽƌ//dͲ:͗DĞĐŚĂŶŝĐƐ//

Solution Let the circular disc of radius a is made up of the


circular section of radius b and remainder. Further, let the
a
line of symmetry joining the centres O and O1 be the x-axis
O b
with O as origin. The centre of mass of the disc of radius a O1 x-axis
O2
will be given by
m x + m2 x2 x2 C
XCM = 1 1 ...(i)
m1 + m2
while YCM and ZCM will be zero (as for all points on x-axis, Fig. 2.98
y and z = 0).
If r is the density of the material of disc:
m1 = pb2r and x1 = c
m2 = p(a2 – b2)r and x2 = ?
M = m 1 + m 2 = p a 2r and XCM = 0
3O SUBSTITUTING THESE VALUES IN %Q I

r b 2 r c + p ( a 2 - b 2 )r x 2
0 =
p a2 r
-cb 2
x2 = ...(ii)
( a2 - b 2 )
i.e., centre of mass of the remainder (say O2) is at a distance cb2/(a2 – b2) to the left of O on
the line joining the centres O and O1.
Problem 2.75 If the density of the material of a square
plate and a circular plate shown in Fig. 2.99 is same. The
centre of mass of the composite system will be
(a) inside the square plate
(b) inside the circular plate
(c) at the point of contact
(d) outside the system l l

Solution Mass of square plate = l r 2


Fig. 2.99
Ê p l2 ˆ
Mass of circular plate = Á
Ë 4 ˜¯
r

where r is mass per unit area. As mass of square plate is larger than that of circular plate,
the CM will lie inside the square plate.
Problem 2.76 Two bodies of masses m1 and m2 attached to its ends of a string on a pulley
which is fixed to the ceiling the masses of the pulley and thread are negligible friction is
absent. Find the acceleration of the centre of mass of this system.
Solution The magnitude of acceleration of masses is
(m1 - m2 ) g
=
m1 + m2
ŽůůŝƐŝŽŶƐപ195

Ê m - m2 ˆ
Let acceleration of mass m1 be a1 = – Á 1 g
Ë m1 + m2 ˜¯

Ê m - m2 ˆ
Then acceleration of mass m2, a2 = + Á 1 g
Ë m1 + m2 ˜¯

The acceleration of centre of inertia (centre of mass) can be defined as, m2

  cm
 m1a1 + m2 a 2 m2
a CM = m1
m1 + m2

È m - m2 ˘ È m - m2 ˘ acm m1
m1 Í - 1 ˙ g + m2 Í 1 ˙g
= Î m1 + m2 ˚ Î m1 + m2 ˚ Fig. 2.100
m1 + m2
2
Ê m - m2 ˆ
= -Á 1 g
Ë m1 + m2 ˜¯
2
Ê m - m2 ˆ
That is, the acceleration of CM is Á 1 g downwards.
Ë m1 + m2 ˜¯

Motion of Centre of Mass of a Bomb Exploding in Air


Suppose a bomb is projected at some angle y
with the ground, it is moving on a parabolic
path. Suddenly it explodes in flight and splits
into number of fragments. Since the explosion
is caused due to internal forces only, so the
centre of mass of the fragments all together will
continue to move on the same parabolic path as
before explosion.
x
Shift in Position of Centre of Mass
Consider a system of two particles, their CM is
defined as,
  z
 m r + m2 r 2
r centre of mass = 1 1 ...(i) Fig. 2.101
m1 + m2
 
If Dr1 and Dr 2 be the shift in positions of the particles, then shift in position of CM of the
system is,
 
 m Dr + m2 Dr 2
Dr CM = 1 1 ...(ii)
m1 + m2
196പWŚLJƐŝĐƐĨŽƌ//dͲ:͗DĞĐŚĂŶŝĐƐ//


Thus, if Fexternal = 0, then  
 m Dr + m2 Dr 2
Dr cm = 1 1 =0
m1 + m2
 
or m1 Dr1 + m2 Dr 2 = 0
Important: If there is no net external force acting on the system, and the particles move due
   
to internal forces, then Fexternal = 0, so Dr CM = 0 and P is constant, so v CM is constant and

a CM is zero.

TILTING OF A BODY
Consider a rectangular block of weight W placed on a rough N
horizontal surface (m). It is acted by a pushing force F. If line of
action of force F is above the centre of gravity, then the block F
experiences a force and a torque. Accordingly, the block may slide
or lift about the edge. The block has: h
(i) Force F in addition to weight W, normal reaction N and mN
frictional force mN. r

(ii) If F acts at a height h from the base, then its torque about P, W
t = Fh. Fig. 2.102
(iii) Restoring torque due to W about P
a
trest = W ¥
;
2
where a is the length of the block.
For the equilibrium of the block, we have
F = mN and N = W
\ F = mW ...(i)
and Fh = W a/2 ...(ii)
&ROM %QS I AND II WE HAVE
a
m = .
Thus, 2 h
a
(i) If m < , block will slide.
2h
a
(ii) If m > , block will tilt about the edge.
2h

WORKED PROBLEMS
Problem 2.77 A solid cone of height h and base radius R is placed with its base on a rough
inclined plane whose coefficient of friction is m. The inclination of the plane is increased
4R 4R
gradually. Show that the cone slips if m < and topple if m > .
h h
ŽůůŝƐŝŽŶƐപ197

Solution The free-body diagram is shown in Fig. 2.103. N


For translational equilibrium along the inclined plane, we h/4
have F = 0. mN
or W sin q = mN, where N = W cos q q s q
sin co
\ m = tan q ...(i) W
W
W
q
For rotational equilibrium, St = 0.
Taking moment of the forces acting on the cone about P, Fig. 2.103
we have
h
W sin q ¥ - W cos q ¥ R = 0
4
4R
or tan q = ...(ii)
h
From Eqs. (i) and (ii), we have
4R
m=
h
4R 4R
Thus, if m < , the cone will slip and if m > , the cone will topple.
h h

Problem 2.78 A boy of mass m moves on a plank


of mass M with a constant velocity u with respect CM
to the plank, as shown in Fig. 2.104.
(i) If the plank rests on a smooth horizontal L
surface, then determine the velocity of the
plank.
v1
(ii) If the boy travels a distance L with respect v2
to the plank, then find the distance travelled
by the plank with respect to the ground. Dx1
Dx2
Solution As no external force is acting on
the system in horizontal direction, so its linear Fig. 2.104
momentum remains constant in that direction. Also there will be no shift in the position of
centre of mass i.e.,
DxCM = 0.
(i) Let v1 and v2 be the velocities of the man and the plank w.r.t. the ground. Then we
have by relative theory
  
[v boy ]plank = [v boy ]ground - [v plank ]ground
then,
  
[v boy ]ground = [v boy ]plank + [v plank ]ground
or v1 = u – v 2
Initially, the system is at rest, therefore
0 = mv1 – mv2
or 0 = m(u – v2) – Mv2
È mu ˘
or v2 = Í
Î m + M ˙˚
198പWŚLJƐŝĐƐĨŽƌ//dͲ:͗DĞĐŚĂŶŝĐƐ//

(ii) Let Dx1 and Dx2 be the distances moved by the boy and the plank relative to the ground,
then we have Dx1 = (L – Dx2)
and shift in the position of centre of mass
 
 mDx1 + M Dx 2
Dx CM = 0 =
m+ M
m(L - Dx2 ) + M(- Dx2 )
or 0 =
m+ M

fi Dx2 = ÊÁ mL ˆ˜ Ans.
Ë m + M¯

Problem 2.79 The balloon, the light string and the cat
shown in Fig. 2.105 are at rest in air. If the cat reaches
the top, by what distance does the balloon descend? Dy2
Mass of balloon is M, mass of the cat is m and length
of the rope ascended by the cat is L.
Solution Let the CM of cat ascend by Dy1 while CM of
balloon descends by Dy2, where Dy1 = L – Dy2. 
As the whole system (balloon + cat) is at rest, so Fnet Dy1 = L – Dy2
= 0. Therefore, the position of CM will not change, i.e.,
DyCM = 0. Thus, we have
Dy2
m Dy + m2 Dy2
DyCM = 1 1
m1 + m2 Fig. 2.105
m(L - Dy2 ) + M(- Dy2 )
=
m+ M
Ê mL ˆ
After solving, Dy2 = Á
Ë m + M ˜¯

Problem 2.80 A ball of mass M and radius R is placed inside


a spherical shell of same mass M and the inner radius 2R. The 2R
combination is at rest on a table top in the position shown in R
Fig. 2.106. The ball is released, rolls back and forth inside, and
finally comes to rest at the bottom of the shell. Find the maximum
displacement of the shell during the process. Dx2 Smooth surface
Solution Let Dx1 be the shift in position of CM of the ball and Dx2 Dx1 = R – Dx2
Dx1
be the shift in position of CM of the shell. As there is no external
force acting on the system in horizontal direction, so there is no
shift in the position of CM of the system (ball + shell). Thus, we
have,
m1Dx1 + m2 Dx2
DxCM = 0 = Fig. 2.106
m1 + m2
ŽůůŝƐŝŽŶƐപ199

From the figure Dx1 = R – Dx2


M(R - Dx2 ) + M(- Dx2 )
\ 0 =
M+m
R
or Dx2 =
2
This is the displacement of the shell when the ball moves down to its lowest position.
When the ball moves to the right of the centre of the shell, the shell is further displaced by
R/2.
Therefore, the total displacement of the shell
R R
= + =R Ans.
2 2

Problem 2.81 A square hole is cut out from a circular lamina, the diagonal of the square
being the radius of the circle. Show that the centre of mass of the remaining is at a distance
R/(4p – 2) from the centre of the circle, where R is the radius of the circular lamina.
Solution Let the mass of the circular lamina having area pR2 is m.
The side of the square hole can be obtained as
R
a
a2 + a 2 = R2 fi a = R/ 2 O
\ Area of the hole = a2 R2/2
Mass of the hole (m¢) = m/2p
Let O be the origin, then the co-ordinates of the CM of the circular lamina Fig. 2.107
is (0, 0) and that of square hole will be = (– R/2, 0)
Centre of mass of the remaining portion
 mx - m¢ x ¢
x CM =
m - m¢
m ◊ 0 - (m/2p ) ◊ (- R/2)
or xCM = m - m/2p
which after solving gives x = R/(4p – 2),
m ◊ 0 - (m/2p ) ◊ 0
and yCM = = 0.
m - m/2p
As the removed portion is symmetrical about x-axis, the y co-ordinate of the CM of the
remaining portion remains at y = 0.
Problem 2.82 A projectile is fired at a speed of 100 m/s at an angle of 37° above the
horizontal. At the highest point, the projectile breaks into two parts of mass ratio 1 : 3, the
smaller coming to rest. Find the distance from the
launching point to the point where the heavier
piece lands.
37°
Solution See Fig. 2.108. At the highest point,
the projectile has horizontal velocity. The lighter Fig. 2.108
200പWŚLJƐŝĐƐĨŽƌ//dͲ:͗DĞĐŚĂŶŝĐƐ//

part comes to rest. Hence, the heavier part will move with increased horizontal velocity.
In vertical direction, both parts have zero velocity and undergo same acceleration, hence
they will cover equal vertical displacements in a given time. Thus, both will hit the ground
together. As internal forces do not affect the motion of the centre of mass, the centre of mass
hits the ground at the position where the original projectile would have landed. The range
of the original projectile is
4 3 4
2u2 sin q cos q 2 ¥ 10 ¥ 5 ¥ 5
xCM = = m
g 10
= 960 m.
The centre of mass will hit the ground at this position. As the smaller block comes to
rest after breaking, it falls down vertically and hits the ground at half of the range i.e., at
x = 480 m. If the heavier block hits the ground at x2, then
m1x1 + m2 x2
xCM =
m1 + m2
M 3M
¥ 480 m + ¥ x2
or, 960 m = 4 4
M
or, x2 = 1120 m.
Y
Problem 2.83 An object of mass 10 kg is launched from the ground at
t = 0, at an angle of 37° above the horizontal with a speed of 30 m/s. 30 m/s
At some time after its launch, an explosion splits the projectile into two
pieces. One piece of mass 4 kg is observed at (105 m, 43 m) at t = 2 s.
Find the location of the second piece at t = 2 s?
37° X
Solution As only the gravity force is acting on the system, the centre
of mass of the system follows a parabolic path. Fig. 2.109
At t = 2 s,
xCM = 30 cos 37° ¥ 2 = 48 m.
1
yCM = 30 sin 37∞ ¥ 2 - ¥ 10 ¥ 22 = 16 m
2
Let coordinates of the second piece, i.e., 6 kg piece be (x, y).
Then
6 x + 4 ¥ 105
xCM = 48 = fi x = 10 m
10
6 y + 4 ¥ 43
yCM = 16 = fi y = –2 m
10
Negative value of y shows that the second piece collides with the ground before t = 2 s.
ŽůůŝƐŝŽŶƐപ201

Problem 2.84 A projectile of mass 3 m is projected from the ground with velocity 20 2
m/s at 45°. At highest point it explodes into two pieces. One of mass 2 m and the other of
mass m. Both the pieces fly off horizontally in opposite directions. Mass 2 m falls at a distance
of 100 m from point of projection. Find the distance of the second mass from the point of
projection where it strikes the ground. (g = 10 m/s2).
y
Solution Range of the projectile in the absence of explosion
u2 sin 2q
R =
g
(20 2 )2 sin 90∞
=
10
= 80 m x
O A
The path of centre of mass of projectile will not change, i.e., xCM
R = XCM
is still 80 m. Now, from the definition of centre of mass
m x + m2 x2 Fig. 2.110
as R = xCM xCM = 1 1
m1 + m2
(m)( x1 ) + (2m)(100)
or 80 =
m + 2m
Solving this equation, we get
x1 = 40 m
Therefore, the mass m will fall at a distance x1 = 40 cm from point of projection.
Problem 2.85 A bomb is projected with velocity
v0 at some point it explodes in three parts in mass
ratio 1 : 1 : 2 our lighter parts falls at origin. Other
lighter part falls at half range what is the distance v0
of heavier part from origin.
Solution Mass ratio is 1 : 1 : 2 so they will be
m m 2m
: :
4 4 4
Fig. 2.111
Let the heavier part fall at distance x. The centre
of mass wall be at range point
R = xCM
m m R 2m
( o) + ◊ + x
R = xCM = 4 4 2 4
m
R x R x
R = + fi R- =
8 2 8 2
7R
x =
2
202പWŚLJƐŝĐƐĨŽƌ//dͲ:͗DĞĐŚĂŶŝĐƐ//

Problem 2.86 Find the centre of mass of letter E, relative to origin A4


O, whose dimensions are given in Fig. 2.112. Take width of the
letter 2 cm everywhere.
A3
A B 20 cm
Solution Centre of mass of the given letter E can be obtained
A2
by area of the different parts of the geometry because thickness is 4 cm 7 cm

same everywhere and density is uniform throughout. A1


Let the given geometry be divided into 4 segments, having O
10 cm
A1 = 10 ¥ 2 = 20 cm , and CM = (5, 1)
2

A2 = (20 – 4) ¥ 2 = 32 cm2, [1, (2 + 8)] = (1, 10) Fig. 2.112


A3 = 4 ¥ 2 = 8 cm2, (2 + 2, 10) = (4, 10)
A4 = 10 ¥ 2 = 20 cm2, (5, 20 – 1) = (5, 19)
Position of CM can be obtained as:
 A x + A2 x2 + A3 x3 + A4 x 4
x = 1 1
A1 + A2 + A3 + A4
20 ¥ 5 + 32 ¥ 1 + 8 ¥ 4 + 20 ¥ 5
= = 3.3 cm
20 + 32 + 8 + 20
 A y + A2 y2 + A3 y3 + A4 y 4
and y = 1 1
A1 + A2 + A3 + A4
20 ¥ 1 + 32 ¥ 10 + 8 ¥ 10 + 20 ¥ 19
= = 10 cm
20 + 32 + 8 + 20

There is no need of calculation of y, because the geometry is symmetrical about the dotted
line AB which is at a height of 10 cm.
Problem 2.87 A block of mass M with a semi-
circular track of radius R, rests on a horizontal A
frictionless surface. A uniform cylinder of ra- R
dius r and mass m is released from rest at the x0
top point A as shown in Fig. 2.113. The cylinder B
M
slips on a semicircular frictionless track. How far (x0 + R)
has the block moved when the cylinder reaches O
the bottom (point B) of the track? How fast is
Smooth surface
the block moving when the cylinder reaches
A
the bottom of the track? (IIT-JEE, 1983) R
Solution Since there is no force acting in the
horizontal direction, the centre of mass of the M
whole system remains fixed throughout.
(x0 + R – x)
Let x be the displacement of larger block O
towards left when the mass m reaches at B. x Smooth surface
Taking O as the origin; the initial position of
the CM Fig. 2.113
ŽůůŝƐŝŽŶƐപ203

M( x0 + R) + m( x0 + r )
xCM = ...(i)
M+m
Final position of the CM
( M + m) ◊ ( x 0 + R - x )
xCM = ...(ii)
M+m
Since there should be no change in the position of CM. Therefore,
M( x0 + R) + m( x0 + r ) ( M + m) ◊ ( x 0 + R - x )
=
M+m M+m
m(R - r )
After solving, we get, x =
M+m
Now according to the principle of conservation of momentum, we have
0 = MV + mv ...(iii)
where V and v are velocities of M and m w.r.t. the ground.
Also by conservation of mechanical energy, we have
1 1
mg(R – r) = MV 2 + mv 2 ...(iv)
2 2
After solving Eqs. (iii) and (iv), we get
2 g( R - r )
V = m
M ( M + m)

STABILITY OF EQUILIBRIUM
A body in equilibrium, is disturbed slightly from its mean position. If the body tends to
regain its initial position of equilibrium, then the equilibrium is said to be stable; otherwise,
it is said to be unstable equilibrium. If the body remains in equilibrium, in the new position
also, then the equilibrium is said to be neutral.

Stability of Spherical Bodies


Consider a body P of radius R (which is the radius of the curved
surface in contact) is in equilibrium on another fixed body of
radius r. Assuming that friction is enough to prevent sliding f C¢
P C
between them. Suppose the body P is disturbed slightly from its q
G
mean position (see Fig. 2.114). M¢
From the geometry of the Fig. 2.114, G¢
A L
rq = Rf B
r
r
\ f = q ...(i) q
R
Let G and G¢ be the initial and final positions of the c.g. then for
stable equilibrium G¢ must lie left of the vertical line through B.
When this happens, the body will return towards mean position.
Thus, Fig. 2.114
204പWŚLJƐŝĐƐĨŽƌ//dͲ:͗DĞĐŚĂŶŝĐƐ//

For stable equilibrium,


G¢M > BL
or G¢C¢sin(q + f) > BC¢ sin q
For small value of f; q is also small and so
sin q ª q and sin (q + f) = q + f,
\ G¢C¢(q + f) > (BC¢)q ...(ii)
If h is the height of c.g. of P from A, then G¢C¢ = R – h. Equation (i), now becomes
(R – h)(q + f) > Rq
or Rf – hq – hf >0
rq
or rq – hq – h >0
R
or rR – hR – hr >0
1 Ê1 1ˆ
\ >Á + ˜ ...(1)
h Ë r R¯
1 1 1 1 Ê1 1ˆ
Hence, for stable equilibrium > + . For unstable equilibrium, < Á + ˜ .
h r R h Ë r R¯

Problem 2.88 A rough solid hemisphere rests on a fixed rough sphere of equal radius.
Show that the equilibrium is:
(i) stable if the flat surface of the hemisphere rests on the sphere; and
(ii) unstable if the curved surface of the hemisphere rests on the
G
sphere. h
R
Solution
(i) Suppose R is the radius of the each sphere, then height of c.g. R
3R
h=
8
In this case, radius of the surface in contact is R = •.
1 1 8
The value = = ...(i) Fig. 2.115
h 3R/8 3R
1 1 1 1 1 R
and + = + = ...(ii)
R r • R R G
h
1 Ê 1 1ˆ
Clearly, > Á + ˜ , therefore the equilibrium is stable.
h Ë R r¯
R
(ii) In this case
3R 5R
h = R- =
8 8
1 8
\ =
h 5R
Fig. 2.116
ŽůůŝƐŝŽŶƐപ205

Ê 1 1ˆ 1 1 2
and ÁË + ˜¯ = + =
R r R R R
1 Ê 1 1ˆ
Clearly < Á + ˜ , therefore the equilibrium is unstable.
h Ë R r¯

Problem 2.89 If two like parallel forces of magnitudes P and Q (P > Q), acting on a rigid
rod of length L at its ends, are interchanged in position, show that the line of action of the
resultant is displaced through a distance,
L(P - Q)
Dx = .
( P + Q)

Solution Suppose, initially the resultant force (P + Q) passes through A; a distance x from
line of action of P, then A
Px = Q(L – x¢) x L–x
P (P + Q) Q
QL
\ x = ...(i) Fig. 2.117
P +Q
Now when forces are interchanged, let the line of action of resultant force (P + Q), pass
through B, at a distance x¢ from line of action of Q, then
Qx¢ = P(L – x¢) B
LP x¢ L – x¢
\ x¢ = ..(ii) Q (P + Q) P
P +Q
Fig. 2.118
Now from Eqs. (i) and (ii), we have
Dx = x¢ – x
L(P - Q)
or Dx = .
( P + Q)

Problem 2.90 A pack of cards is laid on a table, and each card is projected in the direction of
the length of the pack, beyond the one below it, if each projects as far as possible, show that
the distances between the extremities of successive cards will form a harmonical progression.
Solution Let A, B, C, D, so on are the maximum projecting end of the cards, and let m be
the mass and l be the length of each card. The top card can project at most a distance l/2
beyond the one below it and hence its centre of gravity will be just above B. Again, the centre
of gravity of the top two cards is similarly just above C.
Consider the equilibrium of cards 1 and 2 over 3 Fig. 2.119. Let c.g. of combined weight
of 1 and 2 is at a distance x1 from B, then
A
1 A 1
2 B 1 x1 2 x2
3 C 2 B 3 2W
4 D 3 C W 4
W 2W D 3W
W
(a)
(b) (c)

Fig. 2.119
206പWŚLJƐŝĐƐĨŽƌ//dͲ:͗DĞĐŚĂŶŝĐƐ//

Taking moment of weight about C, we get


W ◊ x1 = W(l/2 – x1)
or x1 = l/4.
Now consider the equilibrium of card 1, 2 and 3 over 4 Fig. 2.119(c). Let c.g. of combined
weight of 1, 2 and 3 is at a distance x2 from C, then taking moment of weights about D, we get
W(l/2 – x2) = 2W ◊ x2
or x2 = 1/6.
In general, for n cards, we have
W(l/2 – x) = (n – 1)Wx or x = l/2n
Hence, the distance between the ends of the successive cards are,
l/2, l/4, l/8, l/8, ….
If maximum overhang is asked, then
x = l/2 + l/4 + l/6 + l/8 + ….
Problem 2.91 A circular disk 0.5 m in diameter, pivoted about a horizontal axis through its
centre, has a cord wrapped around its rim. The cord passes over a frictionless pulley P and
it attached to a body of weight 240 N. A uniform rod 2 m long is fastened to the disk, with
one end at the centre of the disk. The apparatus is in equilibrium, with the cord horizontal,
as shown in Fig. 2.120.
(a) What is the weight of the rod?
(b) What is the new equilibrium direction of the rod when a second body weight 20 N is
suspended from the outer end of the rod, as shown by the broken line?
Solution
(a) Let W be the weight of the disk (see Fig. 2.120(b)). Taking moment of the forces acting
on the disk about its centre, we have
240 ¥ 0.25 = 1 ¥ W
W = 60 N.
When 20 N weight is suspended at the end B of the rod, let it bend by an angle q with
the horizontal to acquire new equilibrium position.

A
(a)

P
2m
0.5 m

240 N
ŽůůŝƐŝŽŶƐപ207

240 N
0.25 m
(b)
1m w
240 N
(c)
q

60 N
20 N

Fig. 2.120

(b) Again taking the moment of forces about the centre of the disk (Fig. 2.120(c)).
240 ¥ 0.25 = 60 ¥ 1 cos q + 20 ¥ 2 cos q
After solving the above equation, we get
cos q = 3/5 or q = 53°
Problem 2.92 A rectangular block 0.25 m high is dragged to the right along level surface
at constant speed by a horizontal force P, as shown in Fig. 2.121. The coefficient of sliding
friction is 0.40, the block weighs 25 N, and its centre of gravity is at its centre.
(a) Find the magnitude of the force P.
(b) Find the position of line of action of the normal force N exerted on the block by the
surface, if the height h = 0.125 m.
(c) Find the value of h at which the block just starts to tip.
Solution
(a) Force required to move the block with constant speed
= frictional force = mN = 0.40 ¥ 25 = 10 N
(b) Let the line of action of N is at a distance d from the centre of the block. Forces acting
on the block are shown in Fig. 2.121.
The value of N = weight of the block = 25 N
Taking moment of the forces about point O, we have
10 ¥ 0.125 + 25 ¥ (0.125 – d) = 25 ¥ 0.125
Solving the above equation, we get d = 0.05 m.

0.25 0.25

CG CG
0.5 m P = 10 N
P 0.5 m P = 10 N
25 N 25 N
h h = 0.125 m h
N
10 N d O O
(a) (b) (c)

Fig. 2.121
208പWŚLJƐŝĐƐĨŽƌ//dͲ:͗DĞĐŚĂŶŝĐƐ//

(c) See Fig. 2.121(c). To just tip the block the deflecting torque must be equal to the
resisting torque
or torque of P about O = torque of weight about O
or 10 ¥ h = 25 ¥ 0.125
or h = 0.3125 m.
Problem 2.93 A light spring of spring constant k is kept compressed k
between two blocks of masses m and M on a smooth horizontal m M
surface (Fig. 2.122). When released, the blocks acquire velocities in
opposite directions. The spring loses contact with the blocks when it
acquires natural length. If the spring was initially compressed through Fig. 2.122
a distance x, find the final speeds of the two blocks.
Solution Consider the two blocks plus the spring to be the system. No external force acts
on this system in horizontal direction. Hence, the linear momentum will remain constant.
As the spring is light, it has no linear momentum. Suppose the block of mass M moves with
a speed V and the other block with a speed v after losing contact with the spring. As the
blocks are released from rest, the initial momentum is zero. The final momentum is MV – mv
towards right. Thus,
m
MV – mv = 0 or V = v. ...(i)
M
1
Initially, the energy of the system = kx 2
2
1 1
Finally, the energy of the system = mv 2 + MV 2
2 2
As there is no friction,
1 1 1
mv 2 + MV 2 = kx 2 ...(ii)
2 2 2
Using Eqs. (i) and (ii),
Ê mˆ
mv 2 Á 1 + ˜ = kx2
Ë M¯
kM
or, v = x
m( M + m)

km
and V = x
M ( M + m)

Problem 2.94 Two blocks of masses m1 and m2 are connected by a spring of force constant
k. Block of mass m1 is pulled by a constant force F1 and other block is pulled by a constant
force F2. Find the maximum elongation that the spring will suffer.
Solution Let us take two blocks plus the spring as the k
F2 F1
system. The centre of mass of system moves with an m2 m1

( F1 - F2 )
acceleration, ac = .
m1 + m2 Fig. 2.123
ŽůůŝƐŝŽŶƐപ209

Supposing F1 > F2. As this frame is acceler- m2 a c m1 a c


ated with respect to ground, we have to apply m2 m1 F1
F2
pseudo force on the blocks.
Therefore, net external force on m1
Fig. 2.124
Ê F -F ˆ
F¢1 = F1 – m1 ac = F1 - m1 Á 1 2 ˜
Ë m1 + m2 ¯
ÊFm +F m ˆ
= Á 1 2 2 1 ˜ towards right
Ë m1 + m2 ¯
Ê F -F ˆ
and on m2, F¢2 = F2 + m2 ac = F2 + m2 Á 1 2 ˜
Ë m1 + m2 ¯
ÊFm +F m ˆ
= Á 1 2 2 1 ˜ towards left.
Ë m1 + m2 ¯

As the centre mass is at rest in this frame, the blocks move in opposite directions and
come to instantaneous rest at some instant. The spring will have maximum extension at this
instant. Suppose the right block displaces distance x1 and left displaces a distance x2 from
their initial positions.
Therefore, work done by external force = Increase in PE of the spring
1
i.e., F¢1 x1 + F¢2 x2 = k( x1 + x2 )2 F2
m2 m1 F1
2
1
or F¢1(x1 + x2) = k( x1 + x2 )2 Fig. 2.125
2
2 F1¢
fi (x1 + x2) =
k
2 Ê F1m2 + F2 m1 ˆ
or xmax =
k ÁË m1 + m2 ˜¯

Problem 2.95 Two masses m1 and m2 are connected by a spring of force constant k and are
placed on a frictionless horizontal surface. Initially, the spring is stretched through a distance
x0, when the system is released from rest. Find the distance moved by two masses before
they again come to rest.
Solution Blocks again come to rest when the spring is compressed by x0. Since no external
force is acting on the system, so there is no change in the position of CM of the system i.e.,
Dxcentre of mass = 0
Let mass m1 displaced by Dx and m2 displaced by Dx2, then
Dx1 + Dx2 = 2x0 ...(i)
m1Dx1 + m2 Dx2
and DxCM =
m1 + m2
210പWŚLJƐŝĐƐĨŽƌ//dͲ:͗DĞĐŚĂŶŝĐƐ//

As DxCM = 0
m1Dx1 + m2 Dx2
\ =0 ...(ii)
m1 + m2

k f0
m1 m2

l0 + x0
m1 m2

l0 – x0
m1 m2

Dx1 Dx2

Fig. 2.126

After solving Eqs. (i) and (ii), we get


2m2 x0 2m1x0
Dx1 = , Dx2 =
m1 + m2 m1 + m2

Problem 2.96 Two blocks A and B of masses 2m and 3m K


v 2m 3m v
placed on smooth horizontal surface are connected with a
light spring. The two blocks are given velocities as shown A B
in Fig. 2.127 when spring is at natural length. Fig. 2.127

Column-I Column-II
(a) Minimum magnitude of velocity of A (vAmin) during motion (P) v
(b) Maximum magnitude of velocity of A (vAmax) during motion (Q) v
5

(c) Maximum magnitude of velocity of B (vBmax) during motion (R) 0

(d) Velocity of centre of mass (vCM) of the system comprises blocks (S) 7v
A, B and spring
5
(3mv) - 2mv v
Solution Step I: vCM = =
5m 5
ŽůůŝƐŝŽŶƐപ211

Step II: In CoM frame


Ê vˆ 6v
Initial velocity of A = Á - v - ˜ = - to left
Ë 5¯ 5
v 4 4
Initial velocity of B = v -
= v = v to right
5 5 5
Blocks are executing SHM in CM frame with initial position as equilibrium position
Step III: Velocity variation of B in ground frame, considering right as + ve
Ê 4v v ˆ
from ÁË + ˜ = v to 4v + v = - 3v
5 5¯ 5 5 5
So |vBmax| = v and |vBmin|= 0
Velocity variation A in ground frame
Ê 6v v ˆ 7 v - 6v v
from ÁË + ˜¯ = to + = -v
5 5 5 5 5
-v
Thus, minimum velocity of A is when spring is at maximum extension.
5
The spring shown has initial extension x0. The blocks are released.

NUMERICAL QUESTIONS
The spring shown has initial extension x0. The blocks are released.
m k 2m

Fig. 2.128

Q.1. Find the velocity of m at mean position (when spring attains natural length)
2k k
(a) x0 (b) x0
3m 3m

3k 3k
(c) x0 (d) x0
2m 4m
Ans. (a) Momentum of system will remain conserved as net force on system is zero. Initial
momentum of system is zero as masses are released.
0 = mV1 + 2mV2

kx02 mV12 2mV22


= +
2 2 2
2k
On solving the equations we get V1 = x0
3m
212പWŚLJƐŝĐƐĨŽƌ//dͲ:͗DĞĐŚĂŶŝĐƐ//

Q.2. Find the velocity of 2m at mean position

1 k 1 3k
(a) x0 (b) x0
2 3m 2 2m

1 3k 1 2k
(c) x0 (d) x0
2 4m 2 3m
1 2k
Ans. (d) On solving the equations we get V2 = x0
2 3m
Q.3. The ratio of forces on m and 2m at the time of release is
(a) 1:1 (b) –1:1
(c) 1:2 (d) –1:2
Ans. (b) The forces are always equal and opposite.
\ Ratio is –1:1
Q.4. The ratio of acceleration of m and 2m at the time of release is
(a) 1:1 (b) –1:1
(c) – 2:1 (d) 1:2
Ans. (c)
In the Fig. 2.129 shown, collision between A and B is elastic.

2m V0 m m
A B C

Fig. 2.129

Q.5. Velocity of A just after collision (elastic) is


V0 2V0
(a) (b)
3 3
3V0
(c) (d) none
4
Ans. (a) Collision between A and B
(2m - m)V0 V0
Velocity of A = 0 + =
3m 3
Q.6. Velocity of B just after collision (elastic) is
V0 2V0
(a) (b)
3 3
4V0
(c) (d) none
3
2(2m)V0 4V0
Ans. (c) Velocity of B = =
3m 3
ŽůůŝƐŝŽŶƐപ213

Q.7. Velocity of B and C at the time of maximum compression is


V0 2V0
(a) (b)
3 3
3V0
(c) (d) none
4
Ans. (b) Velocity of C is 0 just after collision between A and B. Now conservation of
momentum of spring system.
At maximum compression the velocity of blocks will become same.
Ê 4V ˆ 2V0
m Á 0 ˜ + 0 = 2mV fi V=
Ë 3 ¯ 3

WORKED PROBLEMS
Problem 2.97 The masses are given initial velocity as shown in Fig. 2.130. What is the
velocity of each mass at maximum extension? Also find the extension.

2V0 k V0
m 2m

Fig. 2.130

Solution The net force is zero. Hence, momentum is conserved.


2V0 k V0
At maximum extension their velocity would become same. m m
The forces are spring force (conservative) hence, mechanical
energy is conserved. –
V
2mV0 – mV0 = 2mV fi V = 0 Fig. 2.131
2
(left is taking positive)
KEi + PEsi = PEsf + KEf
2
m m kx 2 2m Ê V0 ˆ
or, (2V0 )2 + V02 = + Á ˜
2 2 2 2 Ë 2¯

9mV02
\ x =
2k

Problem 2.98 The spring shown in Fig. 2.132 is initially


relaxed. Find the maximum extension when 2m block is k V0
m 2m
given speed V0?
Solution The momentum of system will remain conserved.
Fig. 2.132
The spring force on the two blocks would be equal and
opposite. When the spring has maximum extension, the velocity would be same.
214പWŚLJƐŝĐƐĨŽƌ//dͲ:͗DĞĐŚĂŶŝĐƐ//

2V0
2mV0 = (m + 2m)V \ V=
3
For maximum extension (energy conservation is used).
2
2m 2 3m Ê 2V0 ˆ kx 2 2m
V0 = ÁË ˜¯ + fix= V0
2 2 3 2 3k

Problem 2.99 Two blocks with spring having initial velocity 3V0
V0
of 3v0 and v0 given, calculate VCM and SCM of the block. m 2m

Solution For finding VCM.


Fig. 2.133
m ¥ 3V0 + 2m ¥ V0 5V0
VCM = =
3m 3
Now S = VCM t
5V0 5V t
SCM = ¥t = 0
3 3

Problem 2.100 Two blocks with spring having initial +ve


velocity is shown in Fig. 2.134. Calculate VCM.
2V0 m 2m V0
m 2V0 + 2mV0
Solution VCM = – =0 Fig. 2.134
3m

Problem 2.101 Two blocks of equal mass m are connected by an unstretched spring and
the system is kept at rest on a frictionless horizontal surface. A constant force F is applied
on one of the blocks pulling it away from the other as shown Fig. 2.135.
(a) Find the position of CM at time t.
(b) If the extension of the spring is x0 at time t, find the displacement of the blocks at that
instant.
Solution
y
CM
m m F

Dx1 Dx2
CM
F
m1 m2

O
Dxc

Fig. 2.135
ŽůůŝƐŝŽŶƐപ215

(a) The acceleration of centre of mass



 F F F Net force
ac = = =
M m + m 2m Net mass
The position of CM at time t
1 2 1 F 2 Ft 2
Dxc = act = t =
2 2 2m 4m
(b) We have,
m1Dx1 + m2 Dx2
Dxc =
m1 + m2

Ft 2 mDx1 + mDx2
or =
4m m+m
Ft 2
or Dx1 + Dx2 = ...(i)
2m
The extension of spring is,
\ Dx2 – Dx1 = x0 ...(ii)
After solving Eqs. (i) and (ii), we get

1 È Ft 2 ˘ 1 È Ft 2 ˘
Dx1 = Í - x0 ˙ and Dx2 = Í + x0 ˙ Ans.
2 Î 2m ˚ 2 Î 2m ˚
Problem 2.102 Figure 2.136 shown a small body of mass m placed over a larger mass M
whose surface is horizontal near the smaller mass and gradually curved to become vertical.
The smaller mass is pushed on the longer one at a speed v and the system is left to itself.
Assume that all the surfaces are frictionless.

vy
vx m

vx y
M M m

Fig. 2.136

(a) Find the speed on the larger block when smaller block is sliding on the vertical part.
(b) Find the speed of the smaller mass when it breaks off the larger mass at height h.
(c) Find the maximum height (from the ground) that smaller mass ascends.
216പWŚLJƐŝĐƐĨŽƌ//dͲ:͗DĞĐŚĂŶŝĐƐ//

Solution
(a) Since there is no external force acting on the system in horizontal direction, its
momentum remains constant in this direction or we can also say that velocity of CM
remains constant. The velocity of CM initially was
mv + M ¥ 0 mv
vc = = = vx
m+ M m+ M
This will be the velocity of larger mass when smaller mass is sliding on vertical part.
(b) Since there acts only gravitational force on the system, therefore its mechanical energy
remains constant, i.e.,
1 1 1
mv 2 = m(vx2 + vy2 ) + Mvx2 + mgh
2 2 2
(vy is the vertical component of velocity of smaller block at height h)
Ê mv ˆ
We have vx = Á
Ë m + M ˜¯
2
1 1 1 Ê mv ˆ
\ mv 2 = m(vx2 + vy2 ) + M Á ˜ + mgh
2 2 2 Ë m + M¯
After solving, we get
È ( M 2 + Mm + m2 ) 2 ˘
vx2 + vy2 = Í 2
v - 2 gh ˙
Î ( M + m) ˚
1/2
È ( M 2 + Mm + m2 )v 2 ˘
and v¢ = vx2 + vy2 = Í 2
- 2 gh ˙
Î ( M + m) ˚
(c) Let hmax be the maximum height, vy = 0
1 1 1
\ mv 2 = mvx2 + Mvx2 + mghmax
2 2 2
mv
where vx =
m+ M
Mv 2
After solving, we get hmax =
2 g( m + M )

Problem 2.103 A small sphere of mass m = 1 kg moving with a velocity ( 4i - j) m/s. It hits
a fixed smooth floor and rebounds with velocity (i - 3j) m/s. The coefficient of restitution
9
between the sphere and the floor is e = . Find the value of x. (BITSAT, 2004)
x
Solution The impulse
  
J = m(v f - v i )
= [(i + 3j) - ( 4i - j)]
= – 3i + 4j
ŽůůŝƐŝŽŶƐപ217

r
The component of ( 4i - j) along (- 3i + 4j) is J
r
= [( 4i - j) ◊ (- 3i + 4j)] (- 3i + 4j) ( 32 + 4 2 )2 vi
r
vf
-16
= (- 3j + 4j)
25
16 16
or speed, u1 = ¥ 5 = m/s Fig. 2.137
25 5
Component of (i + 3j) along (- 3i + 4j) is
(- 3i + 4j)
= [(i + 3j) ◊ (- 3i + 4j)]
( ( 3 2 + 4 2 )2
9
= (- 3i + 4j)
25
9¥5 9
or speed, v1 = = m/s.
25 5
If u2 and v2 are the speed of floor before and after collision, then
Èv -v ˘
e = – Í 2 1˙
Î u2 - u1 ˚

È 0 - (- 9/5) ˘
=–Í ˙
Î 0 - 16/5 ˚
9
= .
16
Thus, x = 16
Problem 2.104 A shell of mass (m1 + m2) is fined with a given velocity u2 u1
in a given direction. At the highest point of its path, the shell explodes
into two fragments of mass m1, and m2. The explosion produces an
additional kinetic energy E and the fragments separate in a horizontal x

direction. Find the horizontal distance on the ground at which they hit
the ground, if vertical component of velocity is v0. Fig. 2.138
v
Solution Time taken by the shell to reach the highest point, T = 0 .
g
The fragments m1 and m2 take the same time T to reach the ground. In this duration the
horizontal component of relative speed of the fragments is (v1 + v2), so
v
x¢ = (v1 + v2) T = (u1 + u2) 0 ...(i)
g
If u is the speed of shell before explosion, then
(m1 + m2)u = m1u1 – m2u2 ...(ii)
1 1 1
and (m1 + m2 )u2 + E = m1u12 + m2u22 ...(iii)
2 2 2
218പWŚLJƐŝĐƐĨŽƌ//dͲ:͗DĞĐŚĂŶŝĐƐ//

After solving above equations, we get

v0 Ê 1 1 ˆ
x¢ = 2E Á + .
g Ë m1 m2 ˜¯

Problem 2.105 The masses of five balls at rest in a straight line are in geometrical progression
with ratio 2 and their coefficients of restitution are each 2/3. If the first ball is started towards
the second with velocity u, then the velocity communicated to 5th ball is
(a) (5/9)u (b) (5/9)2u
(c) (5/9)3u (d) (5/9)4u

Solution Let the masses of five balls be m, 2m, 4m, 8m and 16m.
For collision between Ist and IInd ball:
m 2m m 2m
u v1 v2
mv1 + 2mv2 = mv + 0
v1 + 2v2 = u ...(i)
Fig. 2.139
v -v
Also, e = 2 1
u
2 v -v
= 2 1
3 u
2
v2 – v1 = u ...(ii)
3
Adding Eqs. (i) and (ii),
5 5
3v2 = u fi v2 = u
3 9
Proceeding in the same way, the velocity of the fifth ball after collision will be
4
Ê 5ˆ
v5 = Á ˜ u
Ë 9¯

Problem 2.106 In Fig. 2.140(a) shown the wedge of mass M has a A B


R
M Smooth
semicircular groove. A particle of mass m = is released from A. M
2
It slides on the smooth circular track and starts climbing on the right
Fig. 2.140
face.
(i) Find the maximum value of q which it can subtend with vertical and also find the
distance displaced by the wedge at this position.
(ii) Find the maximum velocity of the wedge during process of motion.
ŽůůŝƐŝŽŶƐപ219

Solution
(i)
q

Fig. 2.141

Initially, no momentum along x-axis. So, final momentum will be zero also and relative
velocity is also zero. So, no velocity of any object.
By energy conservation,
Initial potential energy = final potential energy
Hence, q = 90°
DxCM = 0
m(2R – x) = Mx
m(2R) = (M + m)x

2mR 2( M/2)R
x = =
M + m M + ( M/2)

Fig. 2.142

(ii) Maximum velocity of the wedge will be when the ball is at the lowest point in the
wedge as till this point the horizontal component of normal on the wedge will be
speeding the wedge.
pi = 0
pf = – Mv + mu
pi = pf

Mv
u = = 2v
m
Ui + Ki = Uf + Kf
1 1
mgR + 0 = 0 + mu2 + Mv 2
2 2
2mgR = m(2v)2 + Mv2

M
2¥ ¥ gR = 4mv2 + Mv2
2
220പWŚLJƐŝĐƐĨŽƌ//dͲ:͗DĞĐŚĂŶŝĐƐ//

M
MgR = 4 ¥ ¥ v 2 + Mv 2
2
MgR = 2Mv2 + Mv2
MgR = 3Mv2

gR
v =
3

Problem 2.107 A ball of mass 100 g is dropped from a height h = 10 m


on the floor. It rebounds losing 75 per cent of its total mechanical energy.
If it remains in contact with the floor for Dt = 0.01 sec. Find the impulse
of the impact force.
Solution We use impulse and momentum equation. The change in Fig. 2.143

momentum DP of the ball during Dt = 0.01 sec is the impulse (FDt) of
the force of impact.
 
F Dt = DP
  
DP = mv 2 - mv1 = m(v1 + v2 )
 
Since m v1 and m v 2 are vectors at 180°

Hence, F Dt = m(v1 + v2) ...(i)

Since the ball falls through a height h, v1 = 2gh . It is given in the problem that the ball
loses 75 per cent of its total mechanical energy, i.e., final kinetic energy,
1 1 1
mv22 = mv12 ¥
2 2 4
v1 2 gh gh
Hence, v2 = = =
2 2 2
Substitution of the values of v1 and v2 in Eq. (i) yields
gh (9.8)(10)
FDt = 3m = 3 ¥ 0.10 = 2.10 N sec.
2 2
Problem 2.108 A flat car of mass M is at rest on a frictionless floor with a child of mass
m standing at its edge. If child jumps off from the car towards right with an initial velocity
urel, with respect to the car, find the velocity of the car after its jump.
Solution Velocity of man and trolley are assumed with respect to ground as v1 and v2
respectively.
Pi = 0 Pf = mv1 – Mv2
ŽůůŝƐŝŽŶƐപ221

m
vrel v1
v2
M M

(a) (b)

Fig. 2.144

v1 + v2 = urel
Applying momentum conservation
Pf = Pi
mv1 – Mv2 = 0
m(vrel – v2) – Mv2 = 0
murel
or v2 =
m+ M

Problem 2.109 A flat car of mass M with a child of mass m is moving with a velocity v1.
The child jumps in the direction of motion of car with a velocity u with respect to the car.
Find the final velocities of the child and that of the car after jump.
Solution This case is similar to the previous example, except now the car is moving before
jump. Here, also no external force is acting on the system in horizontal direction, hence the
momentum remains conserved in this direction. After the jump, car attains. a velocity v2 in
the same direction, which is less than v1, due to backward push of the child for jumping.
After the jumps, child attains a velocity u + v2 in the direction of motion of car with respect
to ground.
m

M v1

v2 u
M

Fig. 2.145

According to momentum conservation,


(M + m)v1 = Mv2 + m(u + v2)
222പWŚLJƐŝĐƐĨŽƌ//dͲ:͗DĞĐŚĂŶŝĐƐ//

( M + m)v1 - mu
Velocity of car after jump is, v2 =
M+m
( M + m)v1 + Mu
Velocity of child after jump is u + v2 =
M+m

Problem 2.110 A man of mass m is standing on a platform of mass M kept on smooth ice.
If the man starts moving on the platform with a speed vrel relative to the platform, with what
velocity relative to the ice does the platform recoil?
Solution Consider the situation shown in Fig. 2.146. Suppose
v1
the man moves at a speed v1 towards right and the platform
recoil at a speed v2 towards left, both relative to the ice. Hence, v2
the speed of the man relative to the platform is v1 + v2. By the
question,
v1 + v2 = vrel, or v1 = vrel – v2 ...(i) Fig. 2.146

Taking the platform and the man to be the system, there is no external horizontal force
on the system. The linear momentum of the system remains constant. Initially, both the man
and the platform were at rest. Thus,
Pf = Pi fi mv1 – Mv2 = 0
fi m(vrel – v2) – Mv2 = 0
mvrel = Mv2 + mv2
mvrel
v2 =
m+ M

NUMERICAL QUESTIONS
Two persons, A of mass 60 kg and B of mass 40 kg, are standing on a horizontal platform of
mass 50 kg. The platform is supported on wheels on a horizontal frictionless surface and is
initially at rest. Consider the following situations:
A B

Fig. 2.147

(i) Both A and B jump from the platform simultaneously and in the same horizontal
direction.
(ii) A jumps first in a horizontal direction and after a few seconds B also jumps in the
same direction.
ŽůůŝƐŝŽŶƐപ223

In both the situations above, just after the jump, the person (A or B) moves away from the
platform with a speed of 3 m/s relative to the platform and along the horizontal.
Answer these questions.
Q.8. In situation (i), just after both A and B jump from the platform, velocity of centre of
mass of the system (A, B and the platform) is
(a) 2 m/s (b) 6 m/s
(c) 5 m/s (d) none of these
Ans. (d) Centre of mass will remain at rest because there is no external force on the system
of A, B and platform.
Q.9. Final speed of the platform in situation (i), i.e., just after both A and B have jumped
will be
(a) 2 m/s (b) 6 m/s
(c) 5 m/s (d) 3 m/s
Ans. (a) When both jump simultaneously.

A B

(3 – v)
v 50 kg 60 kg 40 kg

Fig. 2.148

Let speed of the platform be v, then speed of A and B will be 3 – v as shown because
relative velocity is 3 m/s. Conserving momentum is
50v = (60 + 40) (3 – v) fi v = 2 m/s
Q.10. Final speed of the platform in situation (ii), i.e., just after B has jumped, will be nearly
(a) 7.5 m/s (b) 5.5 m/s
(c) 4.5 m/s (d) 2.5 m/s
Ans. (d) After A jumps:

B A

3 – v1
40 kg
v1 50 kg
60 kg

Fig. 2.149

(50 + 40)v1 = 60 (3 – v) fi v1 = 1.2 m/s


224പWŚLJƐŝĐƐĨŽƌ//dͲ:͗DĞĐŚĂŶŝĐƐ//

After B jumps:
B

3 – v2
v2 50 kg
40 kg

Fig. 2.150

(50 + 40)v1 = 50v2 – 40(3 – v2)


228
fi 90 ¥ 1.2 = 50v2 – 120 + 40v2 fi v2 = = 2.53 m/s
90

WORKED PROBLEMS
Problem 2.111 vrel

(a) A rail road flat car of mass M can roll without friction along v0
a straight horizontal track (Fig. 2.151). Initially, a man of M
mass m is standing on the car which is moving to the right
with speed v0. What is the change in velocity of the car if
the man runs to the left so that his speed relative to the car Fig. 2.151
is vrel just before he jumps off at the left end?
(b) If there are n men each of mass m on the car, should they all run and jump off together
or should they run and jump one by one in order to give a greater velocity to the car?
Solution
(a) Let v be the velocity of the car relative to the earth, when the man runs to the left, then
conservation of linear momentum gives
(M + m)v0 = Mv + m(v – vrel)
m
\ v = v0 + vrel …(i)
M+m
mvrel
Change in velocity, Dv = v – v0 =
M+m

(b) When there are n men on the car and they jump off together, then velocity of the car
after jumping off, from Eq. (i) will be
mn
v1 = v0 + vrel …(i)
M + mn

If the men jump one by one, the mass of the system will go on changing till the last
man jumps.
ŽůůŝƐŝŽŶƐപ225

Let v¢ be the velocity when one men jumps off the car, then from Eq. (i)
m
v¢ = v0 + vrel
M + (n)m
When the second man also jumps off, then
m
v≤ = v ¢ + vrel
M + (n - 1)m
mvrel mvrel
= v0 + +
M + (n)m M + (n - 1)m

When the last man jumps, then final velocity


mvrel mvrel mvrel
v2 = v0 + + +… + …(ii)
M + (n)m M + (n - 1)m M+m
Comparing Eqs. (i) and (ii), we note that
v2 > v 1
i.e., the men would impart greater velocity to the car by running and jumping off one
by one.

Problem 2.112 A man of mass 80 kg is riding on a trolley of mass 40 kg which is rolling


along a level surface at a speed of 2 m/s. He jumps off the back of the trolley so that
his speed relative to the ground is 1 m/s in the direction opposite to the motion of the
trolley.
(a) What is the speed of the centre of mass of the man-trolley system before and after his
jumps?
(b) What is the speed of the trolley after the man jumps?
(c) What is the speed of the centre of mass of the system after the man hits the ground
and comes to rest?
(d) What force is responsible for the change in vCM?
(e) How much energy did the man expend in jumping?

Solution
m1 m2 m1
v¢1
v v¢2

Initial state Final state


Fig. 2.152
226പWŚLJƐŝĐƐĨŽƌ//dͲ:͗DĞĐŚĂŶŝĐƐ//

(a) Velocity of CM of man-trolley system before the man jumps,


m1v1 + m2v2
vCM = =v [as v1 = v2 = v]
m1 + m2
Before the man touches the ground, there is no external force, hence v¢CM = v
(b) Determine the final velocities of man and trolley by v¢1 and v¢2, we have, from
conservation of momentum,
–m1v¢1 + m2v¢2 = (m1 + m2)v
m1 80
or v¢2 = v + (v1 + v1¢ ) = 2 + (2 + 1) = 8 m/s
m2 40
(c) The man comes to rest after hitting the ground, so the speed of the centre of mass of
the system is
m ¥ 0 + m2v2¢ 40 ¥ 8
v≤CM = 1 = = 2.67 m/s
m1 + m2 80 ¥ 40
(d) Due to the force of friction exerted by the ground, on the man, the velocity of CM is
changed.
(e) While jumping, the force between the man and the trolley is internal to the system.
It has no influence on the motion of CM. However, it changes the total energy of the
system by the amount
DE = KEf – KEi
Ê1 1 ˆ Ê1 ˆ
= Á m1v1¢ 2 + m2v2¢ 2 ˜ - Á (m1 + m2 )v 2 ˜
Ë2 2 ¯ Ë2 ¯
1 1 1
= ¥ 80 ¥ 12 + ¥ 40 ¥ 8 2 - (80 + 40) ¥ 22
2 2 2
= 1080 J
This is equal to the energy expended by the man in jumping.
Problem 2.113
(i) A rail road car of mass M is moving without friction on a straight horizontal track
with a velocity u. A boy of mass m lands on it normally from a helicopter. What will
be the new velocity of the car?
(ii) If now the boy begins to run on it with speed vrel with respect to car in a direction
opposite to motion of the car, what will be the new velocity of the car?

vrel

Fig. 2.153
ŽůůŝƐŝŽŶƐപ227

Solution
(a) Considering the car and boy as system, as there is no external force in the horizontal
direction, linear momentum along the horizontal is conserved. Initially, the horizontal
velocity of boy is zero while that of car is u so, pi = m ¥ 0 + Mu. Finally, when boy
lands on the car, both moving with same common velocity (say V) so, that pf = (m +
M)V. Hence, from conservation of linear momentum along the horizontal
Mu = (m + M)V
Ê M ˆ
i.e., V = Á u
Ë m + M ˜¯
i.e., the velocity of the car decreases as [M/(m + M)] < 1
(b) Here initial momentum of the (car + boy) system will be
pi = (m + M)V ...(i)
If the velocity of car when boy starts running on the car is V1, the velocity of boy
relative to ground will be V1 – vrel. So, the final momentum of the system (car + boy)
relative to ground will be
Pf = MV1 + m(V1 – vrel) = (M + m)V1 = mvrel
Now as force of running is internal so linear momentum of the system must be
conserved
i.e., (m + M)V1 – mvrel = (m + M)V
mvrel
or V1 = V +
(m + M )
i.e., the velocity of car will increase and will be > V.
Note:
(i) The velocity of mass relative to ground will be
Mvrel
VM = V1 – vrel = V - (< V)
m+ M
(ii) If the boy runs in the direction of train then vrel will be replaced by – vrel in all the
above results
mvrel mvrel
V1 = V - ; (< V ) and VM = V + (> V )
(m + M ) m+ M

Problem 2.114 Two men, each of mass m, stand on the edge of a stationary buggy of mass
M. Assuming the friction to be negligible, find the velocity of the buggy after both men jump
off with the same horizontal velocity u relative to the buggy:
1. simultaneously, and
2. one after the other.
In what case will the velocity of the buggy be greater and how many times?
228പWŚLJƐŝĐƐĨŽƌ//dͲ:͗DĞĐŚĂŶŝĐƐ//

Solution
1. Let v be the velocity of the buggy just after both the men jump off
0 = 2m(u + v) + Mv
2mu
fi v =–
M + 2m
2. Let v1 be the velocity of the buggy just after the first man jumps off and v2 be the
velocity the when the second man jumps off
0 = m(u + v1) + (M + m)v1 and
(M + m) v1 = m(u + v2) + Mv2
Solving the above equations, we get
mu(2 M + 3m)
v2 = –
( M + m)( M + 2m)

Problem 2.115 Radiations of wavelength 200 nm. Propagating in the form of parallel beam,
fall normally on a plane metallic surface. The intensity of the beam is 5 mW and its cross-
sectional area is 1.0 mm2. Find the pressure exerted by the radiation on the metallic surface
if the radiation is completely reflected.

hc
Solution Energy of each photon, E =
l
If P is the power of source, the number of photons per second incident on the metallic
surface
P lP
= =
E hc
h
Momentum of each photon =
l
2h
\ Change in momentum due to reflection =
l
Total momentum imparted to the surface per second
2 h l P 2P
= ◊ =
l hc c
Force 2P
\ Pressure = =
Area cA

2 ¥ 5 ¥ 10 - 3
=
3 ¥ 108 ¥ 10 - 6
= 3.33 ¥ 10– 5 N/m2
ŽůůŝƐŝŽŶƐപ229

Problem 2.116 A boy of mass m climbs a rope of length L suspended below a balloon of
mass M. The balloon is stationary with respect to the ground (a) If the boy begins to climb up
the rope at a speed vrel (relative to the rope) in what direction and with what speed (relative
to the ground) will the balloon move? (b) How much has the balloon descended when the
boy reached the balloon by climbing the rope? (c) What is the state of motion after the boy
stops climbing?
Solution

(a) Given that initially the system is at rest, i.e., v CM = 0
Now as force of climbing is an internal force

So, v CM constant = 0
  M
mv + MV
i.e., =0 v
m+ M
 
or mv + MV = 0 [as (m + M) = finite] vrel
  L
i.e., MV = – mv ...(i)
m
Furthermore, here it is given that
  
v rel = v - V ...(ii) v

Substituting the value of v from Eq. (ii) in Eq. (i), we get
   Fig. 2.154
MV = – m(v rel + V )
 
mv rel
or V = ...(iii)
(m + M )
This is the desired result and from this it is clear that the direction of motion of the

balloon is opposite to that of climbing (v rel ) , i.e., vertically down.
(b) From Eq. (i), we have 

mv + M V =0
  
Dr1 Dr2 È  Dr ˘
m +M =0 ÍÎas v = Dt ˙˚
Dt Dt
 
or mDr1 + M Dr2 =0 [as Dt π •]
   
or md1 + Md 2 =0 [Dr = d]
 
or md1 – Md2 = 0 [as d 2 is opposite to d1 ]
or md1 = Md2 ...(iv)
Now as the man climbs up L towards the balloon (relative to the balloon), the
balloon will descend a distance d2 downwards relative to the ground, so that upward
displacement of man relative to the ground will be
d1 = L – d 2 (i.e., d1 + d2 = L) ...(v)
230പWŚLJƐŝĐƐĨŽƌ//dͲ:͗DĞĐŚĂŶŝĐƐ//

Substituting the value of d1 from Eq. (v) in Eq. (iv)


m(L – d2) = Md2
mL
i.e., d2 = ...(vi)
m+ M

i.e., the balloon will descend by mL/(m + M) relative to the ground when the man
climbs up a distance L (relative to balloon).

(c) When man stops climbing v rel = 0

So that from Eq. (iii) V = 0, i.e., balloon will also stop descending and will become
stationary relative to the ground.
Problem 2.117 A tank and a supply of tank bomb are inside a sealed rail road car. The
tank fires to the right, the tank recoils to the left. The bomb remains in the car after hitting
the far wall. Show that no matter how the bomb balls are fired, the tank cannot travel more
than L, assuming it starts from rest.

Solution Given that initially the system is at rest so, v CM = 0
Now as in firing the balls no external force is applied to the system

So, v CM = constant = 0
 
mv1 + Mv 2
i.e., =0
m+ M
 
or mv1 + Mv 2 = 0 [as m + M = finite]
  
Dr Dr È  Dr ˘
or m 1 +M 2 = 0 ÍÎ as v =
Dt Dt Dt ˙˚
 
or mDr1 + M Dr2 = 0 [as Dt π •]
   
or md1 + Md 2 ª 0 [as Dr ª d ª displacement]
 
or md1 ~ Md2 ª 0 ...(i) [as d 2 is opposite to d1 ]
Now as tank balls cannot leave the car, so the maximum
L
displacement of balls relative to the car is L. And as in doing
so the car will shift a distance d2 relative to the ground M
opposite to the displacement of balls, the displacement of m
balls relative to the ground will be
d1 = L – d 2 [i.e., d1 + d2 = drel = L] ...(ii)
Substituting the value of d1 from Eqs. (ii) in (i), we get Fig. 2.155
m(L – d2) = Md2

Ê mL ˆ L
i.e., d2 = Á =
Ë m + M ˜¯ (1 + M/m)
ŽůůŝƐŝŽŶƐപ231

Now as M and m are both > 0 and finite


So, d2 < L
i.e., no matter how the tank balls are fired, the rail road car cannot travel more than L.

Problem 2.118 A 20 gm bullet pierces through a plate of mass M1 = 1 M1 M2


kg and then comes to rest inside a second plate of mass M2 = 2.98 kg as
shown in Fig. 2.156. It is found that the two plates, initially at rest, now
move with equal velocities. Find the percentage loss in the initial velocity
of the bullet when it is between M1 and M2. Neglect any loss of material of
the plates, due to action of bullet. (IIT-JEE, 1979)
M1 M2
Solution The velocities of bullet and plates are shown in Fig. 2.160. If u
is the initial velocity of the bullet, then Fig. 2.156
mu + 0 = M1v1 + mv ...(i)
and mv = (m + M2)v1 ...(ii) M1 M2

After solving, above equations, we get


m
u(m + M2 ) u v m
v =
(m + M1 + M2 )

u(0.02 + 2.98) 3u
= = v1 v1
(0.02 + 1 + 2.98) 4
Fig. 2.157
Èu - v˘
Percentage loss in velocity = Í ¥ 100
Î u ˙˚

È 3u ˘
Íu - 4 ˙
= Í ˙ ¥ 100 = 25%
Í u ˙
Î ˚

Problem 2.119 Suppose a moving oxygen atom makes a head on inelastic collision with a
stationary H2 atom. Before collision both atoms are in the ground state and after collision
they move together. What is the minimum velocity of the moving H2 atom if one of the
atoms is to be given the minimum excitation energy after the collision? (m0 = 1.0078u,
lu = 1.66 ¥ 10–27 kg and ionisation energy of hydrogen is 13.6 eV).

Solution As collision is inelastic, momentum is conserved,


i.e., mu + 0 = (m + m)V or V = (u/2) ...(i)
i.e., the velocity of hydrogen molecule after collision will be half that of hydrogen atom.
232പWŚLJƐŝĐƐĨŽƌ//dͲ:͗DĞĐŚĂŶŝĐƐ//

1
Now as KI = mu2
2
2
1 1 Ê1 ˆ 1
and KF = (2m)V 2 = (2m) Á u˜ = mu2
2 2 Ë2 ¯ 4
So, in this collision there is loss of KE.
1 1 1
DKE = KI – KF = mu2 - mu2 = mu2 ...(ii)
2 4 4
Now according to Bohr’s theory, energy of an electron in a hydrogen atom in nth state
is given by
13.6
En = – eV
n2
As atom will be excited from ground state which for hydrogen atom is n = 1 so,
EG + Eext = En, i.e., Eext = En – E1

È 1˘
i.e., Eext = 13.6 Í1 - 2 ˙ eV (with n > 1)
Î n ˚
And it will be minimum when
n =2
È 1˘
i.e., (Eext)min = 13.6 Í1 - 2 ˙ eV = 10.2 eV ...(iii)
Î 2 ˚
According to given problem
DKE = (Eext)min for one H-atom
Substituting the values of DKE and (Eext)min from Eqs. (ii) and (iii) in the above

1
mu2 = 10.2 eV
4
4 ¥ 10.2 ¥ 1.6 ¥ 10 - 19
or u2 = = 39.02 ¥ 108
1.0078 ¥ 1.66 ¥ 10 - 27

or u = 6.25 ¥ 104 m/s

Problem 2.120 A particle of mass 2 m is projected at an angle of 45° with horizontal with
a velocity of 20 2 m/s. After 1 s explosion takes place and the particle is broken into two
equal pieces. As a result of explosion one part comes to rest. Find the maximum height
attained by the other part. Take g = 10 m/s2.
Solution At 1 second particle is at point P as shown in Fig. 2.158.
ŽůůŝƐŝŽŶƐപ233

 y
Let velocity of the second part be v . Then applying 10 m/s
momentum conservation, just before and just after
explosion we have, 1s

2m (20i + 10j) = m(0) + m v P 20 m
20 m/s
15 m

\ v = ( 40i + 20j)
20 m/s x
Its vertical component of velocity is still 20 m/s. So
the total height, 20 m
u2 (20)2
h = hi + = 15 + = 35 Fig. 2.158
m 2g 2 ¥ 10

Problem 2.121 Two balls shown in Fig. 2.159 are identical.


v
Ball A is moving towards right with a speed v and the
second ball is at rest. Assume all collisions to be elastic.
A B
Show that the speeds of the balls remain unchanged after
all the collisions have taken place.
Solution Fig. 2.159

v
A B

First collision

v
A B

Second collision

v
A B

Third collision

Fig. 2.160

Problem 2.122 A boy of mass 60 kg is standing over a platform of mass 40 kg placed over
a smooth horizontal surface. He throws a stone of mass 1 kg with velocity v = 10 m/s at an
angle of 45° with respect to the ground. Find the displacement of the platform (with boy)
on the horizontal surface when the stone lands on the ground. Take g = 10 m/s2.
Solution Let v be the horizontal velocity of platform in opposite direction. Then from
momentum conservation in opposite direction we have,
(60 + 40)v = (1) (10 cos 45°)
10
\ v = cos 45° m/s
100
10
= cm/s
2
234പWŚLJƐŝĐƐĨŽƌ//dͲ:͗DĞĐŚĂŶŝĐƐ//

\ Displacement of platform = vt
1
2 ¥ 10 ¥
(v)(2u sin 45∞) Ê 10 ˆ 2 = 10 cm
= = Á =
g Ë 2 ˜¯ 10

Problem 2.123 Three balls A, B and C are placed on


a smooth horizontal surface. Given that mA = mC = A B v C
4mB. Ball B collides with ball C with an initial velocity
v as shown in Fig. 2.161. Find the total number of
collisions between the balls. All collisions are elastic. Fig. 2.161
Solution #HVGTVJGſTUVEQNNKUKQPYKVJB + C

Ê m - 4m ˆ v
v B¢ = Á v
Ë m + 4 m ˜¯ B C

3 3 2
v
=– v v B C
5
5 5

Ê 2¥m ˆ 2 Fig. 2.162


v C¢ = Á v= v
Ë m + 4 m ˜¯ 5
#HVGTVJGUGEQPFEQNNKUKQPYKVJB + A
A B
9 2
Now since v < v or vC 2 Ê 3ˆ 6 3Ê3 ˆ 9
25 5 Á ˜v = v Á v˜ = v
5 Ë 5¯ 10 5 Ë 5 ¯ 25
So, B will not collide with C further.
Hence, total collisions are only 2. Fig. 2.163

Problem 2.124 In one-dimensional elastic collision of equal masses, the velocities are
interchanged. Can velocities in a one-dimensional collision be interchanged if the masses
are not equal.
Ê m - m2 ˆ Ê 2m2 ˆ
Solution v¢1 = Á 1 v1 + Á v2
Ë m1 + m2 ˜¯ Ë m1 + m2 ˜¯
We can see that v¢1 = v2 only if m1 = m2
Similarly, v¢2 = v1 if m1 = m2.
Problem 2.125 A particle of mass 0.1 kg moving at an initial speed v collides with another
particle of same mass kept initially at rest. If the total energy becomes 0.2 J after the collision,
what would be the minimum and maximum values of v?
Solution In elastic collision, velocities are interchanged. So, v is minimum
1
\ ¥ 0.1 ¥ v 2 = 0.2
2
\ v = 2 m/s = minimum value
In perfectly inelastic collision, speed of combined mass will remain half.
2
1 Ê vˆ
\ ¥ 0.1 ¥ Á ˜ = 0.2
2 Ë 2¯
\ v = 2 2 m/s = maximum value
ŽůůŝƐŝŽŶƐപ235

Problem 2.126 A ball is projected from the ground at 1


some angle with horizontal. Coefficient of restitution 2
between the ball and the ground is e. Let a, b and c be the
ratio of times of flight, horizontal range and maximum
height in two successive paths. Find a, b and c in terms
of e? Fig. 2.164

Solution Just after collision x-component of velocity remain unchanged but y-component
of velocity becomes e times.
2uy uy2 2ux uy
Now, T= ,H= and R = euy
g 2g g

or T μ u y, H μ u2y and R μ uy
ux fi ux
T1 uy 1
\ =a= =
T2 euy e
uy
R1 uy 1
=b= = Just before Just after
R2 euy e collision collision

H1 ( u y )2 1 Fig. 2.165
and =c= =
H2 (euy )2 e 2

Problem 2.127 Two blocks A and B of equal masses are attached


B
to a string passing over a smooth pulley fixed to a wedge as A
shown in Fig. 2.166. Find the magnitude of acceleration of centre
of mass of the two blocks when they are released from rest. 60° 30°
Neglect friction.
Fig. 2.166
Net pulling force 1  
Solution a = = (a 1 + a 2 ) (as m1 = m2)
Total mass 2
r
a2

a 1 90°
mg 90°
2 co
°

s3
60


sin

60° 30°
mg

r
a1

Fig. 2.167
236പWŚLJƐŝĐƐĨŽƌ//dͲ:͗DĞĐŚĂŶŝĐƐ//
mg sin 60∞ - mg sin 30∞
=
2m
 
Ê 3 - 1ˆ  |a + a 2|
a =Á ˜g \ |a CM| = 1
Ë 4 ¯ 2
 
|a1| = |a 2| = a
  2a
\ |a1 + a 2| = 2 a at 90 =
2
 
 m1a1 + m2a 2 a Ê 3 - 1ˆ
a CM = = = g
m1 + m2 2 ÁË 4 2 ˜¯

Problem 2.128 A projectile is fired from a gun at an angle of 45° with the horizontal and
with a speed of 20 m/s relative to ground. At the highest point in its flight the projectile
explodes into two fragments of equal mass. One fragment, whose initial speed is zero falls
vertically. How far from the gun does the other fragment land, assuming a level terrain?
Take g = 10 m/s2 ? (VITEEE, 2010)
Solution Path of CM will remain unchanged
R R R 3R y-axis
+ + =
2 2 2 2
3R 3 Ê u2 sin 2q ˆ
X2 = = Á ˜
2 2Ë g ¯
R/2
3 (20)2 sin 90∞ R/2 R/2 x-axis
= 1 CM 2
2 10
Fig. 2.168
= 60 m
Problem 2.129 A ball B is suspended from a string of length l attached to
Cart A mA
a cart A, which may roll on a frictionless surface. Initially, the cart is at rest
and the ball is given a horizontal velocity v0 [see Fig. 2.169]. Determine:
(a) the velocity of B as it reaches the maximum height; and l
(b) the maximum height reached by the ball.
B
Solution We choose ball and cart as our system. No external force acts on mB v0
the system in x-direction; therefore momentum along x-axis is conserved.
Fig. 2.169
The ball will continue to move upwards until its velocity relative to the
cart is zero.
  
i.e., v BA = v B - v A = 0
 
or v B = v A,
When the ball reaches maximum height, the cart and ball move horizontally with same
velocity at the extreme position.
ŽůůŝƐŝŽŶƐപ237

(vA)i = 0 (vA)f = u
A A

(vBA)i = 0

Reference level (vB)i = u0 h B (vB)f = (vA)f = u


B
Initial position Final position

Fig. 2.170

P i = m Bv B = m Bv 0
Pf = mB(vB)f + mA(vA)f
= (mB + mA)v
From conservation of momentum,
Pi = Pf
mBv0 = (mA + mB)v
mBv0
or v =
mA + mB
In order to find maximum height reached by the ball we will apply law of conservation
of energy.
1
Ei = mA gl + mBv02
2
1
Ef = mA gl + mB gh + (mA + mB )v 2
2
Ei = Ef
1 1
mAgl + mBv02 = mAgl + mBgh + (mA + mB)2
2 2
2
1 1 Ê mBv0 ˆ
or mBv02 = mB gh + (mA + mB ) ¥ Á
2 2 Ë mA + mB ˜¯
which on solving for h yields
È mBv0 ˘ v02
h =Í ˙
Î (mA + mB ) ˚ 2 g
%QPEGRV Solve problem in CM frame

1 Ê mA mB ˆ 2
v0 = mAgh
2 ÁË mA + mB ˜¯
Wgravity = – mAgh
1 mA mB 2
DKE = 0 - v0
2 mA + mB
238പWŚLJƐŝĐƐĨŽƌ//dͲ:͗DĞĐŚĂŶŝĐƐ//

Note that for mA >> mB,


(vB)f = (vA)f = 0
v02
and h =
2g

Ball B oscillates as a pendulum with A fixed for mA << mB.


(vB)f = (vA)f = v0
and h =0
A and B move with the same constant velocity v0.
Problem 2.130 A boy of mass m is standing in a stationary lift of mass M which is
counterpoised by a mass (M + m). If the boy jumps suddenly generating a velocity 2gh,
find the velocity of the man relative to lift immediately after jumping. Show that the height
2 h( M + m)
to which the man rises relative to the floor is .
2M + m

Solution Given velocity of boy is his absolute velocity. When man jumps
upwards lift is jerked downwards. Consider (man + lift + counterpoise) as
system. For this system momentum is conserved
m 2 gh = (m + 2M)V
M
m 2 gh m
V =
(m + 2 M )
Vman, lift = Vlift = Vman + Vlift
M+ m
m 2 gh
= 2 gh +
(m + 2 M ) Fig. 2.171

2( M + m)
or Vrel = 2 gh
2M + m
After man jumps in air, the acceleration of lift and counterpoise
[( M + m) - M]g mg Net force
a= = =
( M + m) + M ( 2 M + m) Net mass
Downward distance moved by lift
V 2 m2 ◊ 2 gh(2 M + m) mh
h2 = = =
2 a (m + 2 M )2 ¥ 2mg (m + 2 M )

( 2 gh )2
Upward distance moved by man = =h
2g
2( M + m)
hrel = h1 + h2 = h
2M + m
ŽůůŝƐŝŽŶƐപ239

Problem 2.131 A simple pendulum is suspended from a point on a vertical wall. The
pendulum is pulled away from the wall to a horizontal position and released. The ball
hits the wall, the coefficient of restitution, being (2/ 5 ). What is the minimum number of
collisions after which the amplitude of oscillation becomes less than 60°? (IIT-JEE, 1987)
Solution When simple pendulum released from position A strikes L
the wall with velocity v then by conservation of mechanical energy P A

1
mgL + 0 = mv 2 + 0 q
2
i.e., v = 2gL L
Now as coefficient of restitution is e, so speed of pendulum after
first collision will be
h
v1 = ev = e 2 gL
B
Now after completing oscillation in accordance with conservation
Fig. 2.172
of mechanical energy it will strike the wall with same velocity and
so its velocity after second collision will be
v2 = ev1 = e(e 2 gL ) = e 2 2 gL
So the velocity of the pendulum after n collisions will be

v n = e nv = e n 2gL
Now if it rises to a height h, by conservation of mechanical energy
1 1 2n
m(vn )2 = mgh, i.e., e 2gL ª gh
2 2
h L(1 - cos q )
or e2n ª ª
L L
2n
Ê 2 ˆ È 2 ˘
or ÁË ˜ ª 1 – cos q Ías e ª 5 ˙
5¯ Î ˚
n
Ê 4ˆ
or ÁË ˜¯ ª 1 – cos q
5
1
Now for q to be lesser than 60°, cos q > (1/2), i.e., 1 ~ cos q <
2
n n
Ê 4ˆ 1 Ê 5ˆ
So, ÁË ˜¯ < or ÁË ˜¯ > 2
5 2 4
or n(log 10 ~ 3 log2) > log2
0.301
or n> [as log 10 ª 1 and log 2 ª 0.3010]
0.097
or n > 3.1
As n (no. of collisions) must be integer so for q < 60°, n = 4.
240പWŚLJƐŝĐƐĨŽƌ//dͲ:͗DĞĐŚĂŶŝĐƐ//

Problem 2.132 A bullet of mass m is fired into a large block of mass M suspend by a wire
of length L. The bullet gets embedded in the block. What is the maximum angle made by
the string after impact. (IIT Roorkee, 1988)
Solution If V is the velocity of the block-bullet system just after collision, then by
conservation of linear momentum.
(i) Collision between bullet and block
mv = (M + m)V

È M + m˘
or v = Í V ...(i)
Î m ˙˚
1
(m + M )V 2 (which is less than
So the KE of the block-bullet system just after collision is
2
1
mv 2 as collision is inelastic). Now due to this remaining KE if the system rises up to a
2
height h, conservation of ME for this part of problem yields,
1
(m + M)V2 = m(m + M)gh, i.e., V= 2gh ...(ii)
2
Substituting this value of V from Eq. (ii) in Eq. (i), we get
q
Ê M + mˆ
v = Á 2 gh ...(iii)
Ë m ˜¯ L
L– h
2
v2 Ê m ˆ
or h = Á ˜ ...(iv)
2g Ë M + m¯
Now from Fig. 2.173,
h
m
L-h Ê hˆ v
cos q* = = Á1- ˜ ...(v)
L Ë L¯
M
Substituting the value of h from Eq. (iv) in Eq. (v)
Fig. 2.173
2
1 Ê mv ˆ
cos q = 1 - Á ˜
2 gL Ë m + M ¯

È 1 Ê mv ˆ ˘
2
or q = cos - 1 Í1 - Á ˜ ˙
ÍÎ 2gL Ë m + M ¯ ˙˚
Problem 2.133 A ball falls vertically for 2s and hits a plane inclined at 30° to the horizontal.
If the coefficient of restitution be 5'8, prove that the time that elapses before it again hits the
plane is 2.5 sec. Find also the distance along the plane between the first and second impact
(g 10 m/s2).
Solution Let the ball strike the plane at A. We assign a co-ordinate system with the origin
at A. The velocity of the ball while striking the 30° plane = gt = 2g. The components of u
along and perpendicular to the plane are u sin 30° and u cos 30° respectively. After impact,
ŽůůŝƐŝŽŶƐപ241

the velocity component u sin 30° along the plane remains unaffected while the component
u cos 30° perpendicular to the plane becomes e(u cos 30°) on rebound.
as v = u – gt
0 = u – gt
u = gt
3
Thus, velocity of the rebound = e ◊ 2 g ◊ = 3 eg
2
y

e
+v
A +v
e x
B –g cos q +g sin q
g
30°

Fig. 2.174

The acceleration perpendicular to the plane


= – g cos 30° = - g 3/2
If t be the time of striking the plane again, then using
1 2
s = ut + gt
2
1 3 2
0 = 3 egt - gt
2 2
5
fi t = 4e = 4 ¥ = 2.5 sec
8
1
The distance AB = u sin 30°t + g sin 30° t2
2
2
1 1 1 Ê 5ˆ
= (2 g ) ¥ 2.5 ¥ + ¥ 10 ¥ Á ˜
2 2 2 Ë 2¯
250
= 25 + = 40.63 m
16

Problem 2.134 A particle of mass m moving with a speed v hits


elastically another stationary particle of mass 2m on a smooth v
horizontal circular tube or radius r. Find the time when the next m
collision will take place?
2m
Solution Since collision is elastic. It means relative velocity of
separation will remain v and relative velocity means one mass is
assumed at rest.
2p r
\ t = Fig. 2.175
v
242പWŚLJƐŝĐƐĨŽƌ//dͲ:͗DĞĐŚĂŶŝĐƐ//

Problem 2.135 A cat weighing 5 kg is standing on a flat boat that he is 10 m from the shore.
He walks 4 m on the boat toward shore and then halts the boat weighs 20 kg and one can
assume that there is no friction between it and the water. How far is the cat from the shore
at the end of this time.
Solution Given that initially the system is at rest so

v CM = 0.
Now as in motion of cat no external force is applied to the system

So, v CM = constant = 0
 
mv1 + Mv 2
i.e., =0
m+ M
 
or mv1 + Mv 2 = 0 [as (m + M) = Finite]
  
Dr Dr È  dr ˘
or m 1 +M 2 = 0 ÍÎ as v =
dt dt dt ˙˚
   
or mDr1 + M Dr2 = 0 [as Dr = d = displacement]
 
or md1 – Md2 = 0 [as d 2 is opposite to d1]
i.e., md1 = Md2 ...(i)
Now when cat moves 4 m towards shore relative Shore
to boat, the boat will shift a distance d2 relative to Boat
Cat
the shore opposite to the displacement of cat so, the
displacement of cat relative to shore (towards shore)
will be 10 m

d1 = 4 – d2 (i.e., d1 + d2 = drel = 4) ...(ii) Fig. 2.176


Substituting the value of d2 from Eq. (ii) in Eq. (i)
md1 = M(4 – d1)
M¥4 20 ¥ 4
or d1 = = = 3.2 m
(m + M ) 5 + 20
As initially the cat was 10 m from the shore, so now he will be 10 – 3.2 = 6.8 m from the
shore.
Problem 2.136 A plank of mass 5 kg is placed on a
1 kg
frictionless horizontal plane. Further, a block of mass
1 kg is placed over the plank. A massless spring of 5 kg
natural length 2 m is fixed to the plank by its one
end. The other end of the spring is compressed by the 4m
block by half of spring’s natural length. The system
is now released from the rest. What is the velocity of Fig. 2.177
the plank when the block leaves the plank? (Stiffness constant of spring is 100 N/m).
Solution Let the velocity of the block and the plank, when the block leaves the spring, be
u and v respectively.
ŽůůŝƐŝŽŶƐപ243

1 2 1 1
By conservation of energy kx = mu2 + Mv 2
2 2 2
[M = mass of the plank, m = mass of the block]
\ 100 = u2 + 5v2 ...(i)
By conservation of momentum
mu + Mv = 0
fi u = – 5v ...(ii)
Solving Eqs. (i) and (ii)
30v2 = 100
10
\ v = m/s
3
From this moment until the block falls, both plank and block keep their velocity constant.
Thus, when the block falls velocity of plank

10
= m/s
3

Problem 2.137 To test the manufactured properties B

of 10 N steel balls, each ball is released from rest


1.5 m
as shown and strikes a 45° inclined surface. If the
coefficient of restitution is to be e = 0.8, determine the
distance s to where the ball must strike the horizontal
1.0 m
plane at A. At what speed does the ball strike at A? 45°
(g = 9.8 m/s2). A
s
Solution v0 = 2 gh = 2 ¥ 9.8 ¥ 1.5 = 5.42 m/s
Fig. 2.178
Component of velocity parallel and perpendicular
to plane at the time of collision.
v0 C
v1 = v2 = = 3.83 m/s v2 v1
2
Component parallel to plane (v1) remains un- v0
45°
changed, while component perpendicular to plane
becomes ev2,
where, ev2 = 0.8 ¥ 3.83 = 3.0 m/s ev2 = 0.8v2

\ Component of velocity in horizontal direction


C
after collision v1
1.0 m
(v1 + ev2 ) 3.83 + 3.0 45°
vx = = = 4.83 m/s x
2 2 D E A
s
While component of velocity in vertical direction x y
after collision.
Fig. 2.179
244പWŚLJƐŝĐƐĨŽƌ//dͲ:͗DĞĐŚĂŶŝĐƐ//

v1 - ev2 3.83 - 3.0


vy = = = 0.59 m/s
2 2
Let t be the time, the particle takes from point C to A, then
1
1.0 = 0.59t + ¥ 9.8 ¥ t 2
2
Solving this we get,
t = 0.4 s (Positive value)
\ DA = vxt = (4.83)(0.4) = 1.93 m
\ S = DA – DE
= 1.93 – 1.0
S = 0.93 m
vyA = vyc + gt
= (0.59) + (9.8)(0.4) = 4.51 m/s
vxA = vxC = 4.83 m/s

\ vA = (vxA )2 + (vyA )2 = 6.6 m/s

Problem 2.138 Two particles A and B of equal masses lie close together on a horizontal table
and are connected by a light inextensible string of length l. A is projected vertically upwards
with a velocity 10gl . Find the velocity with which it reaches the table again.

Solution String becomes tight when A moves upwards by a distance l. Let v1 be the velocity
of A at this moment, then

A v1 A v2

l l

v2
B B

A v2

B v2

Fig. 2.180
ŽůůŝƐŝŽŶƐപ245

v12 = ( 10 gl )2 - 2 gl = 8 gl

or v1 = 8gl
Let v2 be the common velocities of both A and B just after string becomes tight. Then from
conservation of linear momentum.
v 8 gl
v2 = 1 =
2 2
Both particles return to their original height with same speed v2. String becomes lose after
B strikes the ground and the speed v with which A strikes the ground is,
8 gl
v2 = v22 + 2gl = + 2 gl
4
or v2 = 4gl
or v = 2 gl

Problem 2.139 A thin hoop of mass M and radius r is placed on a


horizontal plane. At the initial instant, the hoop is at rest. A small
washer of mass m with zero initial velocity slides from the upper
point of the hoop along a smooth groove in the inner surface of
the hoop. Determine the velocity u of the centre of the hoop at the
f
moment when the washer is at a certain point A of the hoop, whose
radius vector forms an angle f with the vertical (Fig. 2.181). The A
friction between the hoop and the plane should be neglected.
Solution Let vr be the velocity of the washer relative to centre of Fig. 2.181
hoop and v the velocity of centre of hoop. Applying conservation of
linear momentum and mechanical energy we have,
m(vr cos f – v) = Mv ...(i)
1 1
mgr(1 + cos f) = Mv 2 + m(vr2 + v 2 - 2vvr cos f ) ...(ii)
2 2
Solving Eqs. (i) and (ii), we have,
2 gr(1 + cos f )
v = m cos f
( M + m)( M + m sin 2 f )
R
Problem 2.140 A small cube of mass m slides down a circular m
path of radius R cut into a large block of mass M, as shown in
Fig. 2.182. M rests on a table, and both blocks move without
friction. The blocks are initially at rest, and m starts from the
top of the path. Find the horizontal distance from the bottom
of block when cube hits the table.
M
Solution mv1 = Mv2 ...(i) R/2

Fig. 2.182
246പWŚLJƐŝĐƐĨŽƌ//dͲ:͗DĞĐŚĂŶŝĐƐ//

1 1
mgR = mv12 + Mv22 ...(ii)
2 2

2(R/2) R
t = = ...(iii)
g g
The desired distance is
S = (v1 + v2)t …(iv)
Solving Eqs. (i) and(ii) for v1 and v2 and substitut- v1
ing in Eq. (iv), we get v2

S = R 2( M + m)
M Fig. 2.183

Problem 2.141 A shell of mass 1 kg is projected with


velocity 20 m/s at an angle 60° with the horizontal. It 1m
collides inelastically with a ball of mass 1 kg which is
suspended through a thread of length 1 m. The other
end of the thread is attached to the ceiling of a trolley of 16 m
4 20 m/s
mass kg as shown in Fig. 2.184. Initially, the trolley
3 60°
is stationary and it is free to move along horizontal rails
without any friction. What is the maximum deflection
Fig. 2.184
of the thread with vertical? String does not slack. Take
g = 10 m/s2.
2 3
u2 sin 2 q (20) ¥ 4
Solution H = = = 15 m
2g 2 ¥ 10
i.e., the shell strikes the ball at the highest point of its trajectory. Velocity of (ball + shell)
just after collision,
u cos 60∞
v=
2
(from conservation of linear momentum)
20
= = 5 m/s
2¥2
At highest point combined mass is at rest relative to the
trolley. Let v be the velocity of trolley at this instant. From
q
conservation of linear momentum we have, v
Ê 4ˆ
2 ¥ 5 = Á 2 + ˜ v or v = 3 m/s
Ë 3¯ vr = 0
From conservation of energy, we have
1 1Ê 4ˆ Fig. 2.185
¥ 2 ¥ (5)2 - Á 2 + ˜ (3)2 = 2 ¥ 10(1 – cos q)
2 2Ë 3¯
ŽůůŝƐŝŽŶƐപ247
1
Solving we get, cos q =
2
\ q = 60°
Problem 2.142 Two large rigid vertical walls A and B are
parallel to each other and separated by 10 m. A particle
of mass 10 g is projected with an initial velocity of 20
m/s at 45° to the horizontal from point P on the ground,
such that AP = 5 m. The plane of motion of the particle 45°
is vertical and perpendicular to the walls. Assuming that
all the collisions are perfectly elastic, find the maximum B
A P
height attained by the particle and the total number of
collisions suffered by the particle with the walls before it Fig. 2.186
hits ground. Take g = 10 m/s2.
Solution As the collisions are perfectly elastic, collision of the ball will not affect the vertical
component of its velocity while the horizontal component will be simply reversed.
vy2 [20 ¥ sin 45∞]2
Hence, Hmax = = = 10 m
2g 2 ¥ 10
Total time of flight
Ê 1 ˆ
2 ¥ 20 ¥ Á
2vy Ë 2 ˜¯
T = = =2 2s
g 10
Total horizontal distance travelled before striking the ground x = vxT = 40 m
PB + BA + AB + BA + AB = 45 m
Hence, total number of collision suffered by the particle with the walls before it hits
ground = 4.
Problem 2.143 A sphere of mass m moving with a velocity u hits another stationary sphere
of same mass. If e is the coefficient of restitution, what is the ratio of the velocities of two
spheres after the collision?
Solution Here u1 = u, u2 = 0
v2 - v1 v2 - v1
\ e = =
u1 - u2 u-0
or v2 – v1 = eu ...(i)
By the law of conservation of momentum,
mu + m ¥ 0 = mv1 + mv2
or v1 + v2 = u ...(ii)
Adding Eqs. (i) and (ii),
2v2 = u + eu = u(1 + e)
u(1 + e )
or v2 =
2
248പWŚLJƐŝĐƐĨŽƌ//dͲ:͗DĞĐŚĂŶŝĐƐ//

Again, from Eq. (ii),


u(1 + e ) u(1 - e )
v1 = u – v 2 = u - =
2 2
v2 1+ e
Hence, = .
v1 1- e

Problem 2.144 A small block of mass m is placed on top of a


smooth convex hemisphere also of mass m which is placed on a
smooth horizontal surface. If the block begins to slide down due to
a negligible small impulse, show that it will loose contact with the
convex hemisphere when the radial line through vertical makes
an angle q given by the equation.
Fig. 2.187
Solution Let vr be the relative velocity of block as it leaves contact
with the sphere (N = 0) and v the horizontal velocity of sphere at this instant.
Applying conservation of linear momentum in horizontal direction, we get
mv = m(vr cos q – v)
or 2v = vr cos q ...(i)
Conservation of mechanical energy gives,
1
mgr(1 – cos q) = mv 2
2 1
vr
+ m(vr2 + v 2 - 2vvr cos q )
2 v q

vr2
or gr(1 – cos q) = v 2 + - vvr cos q ...(ii)
2
Equation of law of motion gives, vr cos q – v
mvr2
mg cos q =
r
vr sin q
vr2
or gr = ...(iii)
cos q Absolute components of
velocity of block
Solving Eqs. (i), (ii) and (iii), we get
Fig. 2.188
cos3 q = 6 cos q – 4 = 0
Note: We have not considered pseudo force while writing the equation of motion. Think
why?
M
Problem 2.145 Two blocks of mass 2 kg and M are at rest
on an inclined plane and are separated by a distance of 6.0 m
m as shown. The coefficient of friction between each block 2 kg 6.0
and the inclined plane is 0.25. The 2 kg block is given a
velocity of 10.0 m/s up the inclined plane. It collides with
M, comes back and has a velocity of 1.0 m/s when it reaches q
its initial position. The other block M after the collision
moves 0.5 m up and comes to rest. Calculate the coefficient Fig. 2.189
ŽůůŝƐŝŽŶƐപ249

of restitution between the blocks and the mass of the block M. [Take sin q ª tan q = 0.05 and
g = 10 m/s2] (JEE, 1999)
Solution Let v1 = velocity of block 2 kg just before collision
v2 = velocity of block 2 kg just after collision
and v3 = velocity of block M just after collision.
Applying work energy theorem
(change in kinetic energy = work done by all the forces) at different stages as shown in
Fig. 2.190.
v1

2kg
/s
m
10 m h1
6
q
2 kg

Fig. 2.190

DKE = Wfriction + Wgravity


È1 2 2 ˘
ÍÎ 2 m{v1 - (10) }˙˚ = – 6m mg cos q – mgh1
(m = 2 kg)
or v12 – 100 = 2 [6 mg cos q + gh1]

cos q = 1 - sin 2 q = 1 - (0.05)2 ª 0.99


\ v12 = 100 – 2 [(6)(0.25)(10)(0.99) + (10)(0.3)]
fi v1 ª 8 m/s

2 kg
v2

/s
m
1
2 kg

Fig. 2.191
250പWŚLJƐŝĐƐĨŽƌ//dͲ:͗DĞĐŚĂŶŝĐƐ//

DKE = Wfriction + Wgravity


1
m[(1) - (v22 )] = – 6m mg cos q + mgh1
2
2
or 1 – v12 = 2 [– 6mg cos q + gh1]
= 2 [(– 6)(0.25)(10)(0.99) + (10)(0.3)]
= – 23.7
\ v22 = 24.7 or v2 = 5 m/s

5
0.
h2
v3
q
M
q

Fig. 2.192

DKE = Wfriction + Wgravity


1
M[0 - v32 ] = – (0.5)(m)(M)g cos q – Mgh2
2
or –v23 = – mg cos q – 2gh2
or v23 = (0.25)(10)(0.99) + 2(10)(0.25)
or v23 = 2.975
\ v3 ª 1.72 m/s
Now
(i) Coefficient of restitution
Relative velocity of separation
=
Relative velocity of approacch
v2 + v3
=
v1
5 + 1.72 6.72
= =
8 8
or e ª 0.84
(ii) Applying conservation of linear momentum before and after collision
2v1 = Mv3 – 2v2
2(v1 + v2 ) 2(8 + 5) 26
\ M = = =
v3 1.72 1.72
M ª 15.12 kg
ŽůůŝƐŝŽŶƐപ251

Problem 2.146 A ball of mass m = 1 kg falling vertically


with a velocity v0 = 2 m/s strikes a wedge of mass M = 2
kg kept on a smooth, horizontal surface as shown in Fig. m
2.193. The coefficient of restitution between the ball and
v0
1 M
the wedge is e = . Find the velocity of the wedge and the
2 30°
ball immediately after collision. (Solve by impulse method)
Solution Given M = 2 kg and m = 1 kg Fig. 2.193

v3
J
v1
m v2 J sin 30°
M m
J sin 30°

30° J 30°

J cos 30°

Fig. 2.194

Let, J be the impulse between the ball and the wedge during collision and v1, v2 and v3 be
the components of velocity of the wedge and the ball in horizontal and vertical directions
respectively.
Applying impulse = change in momentum
we get J sin 30° = Mv1 = mv2
J
or = 2v1 = v2 ...(i)
2
J cos 30° = m(v3 + v0)

3
or J = (v3 + 2) ...(ii)
2
Applying, relative speed of separation = e (relative
speed of approach) in common normal direction,
Common normal
we get direction
1
(v1 + v2) sin 30° + v3 cos 30° = (v0 cos 30∞)
2
30°
or v1 + v2 + 3 v3 = 3 ...(iii)
Fig. 2.195
252പWŚLJƐŝĐƐĨŽƌ//dͲ:͗DĞĐŚĂŶŝĐƐ//

Solving Eqs. (i), (ii) and (iii), we get


1 1
v1 = m/s v1 = m/s 2
3 3 v2 = m/s
3
2
v2 = m/s and v3 = 0
3 30°
Thus, velocities of wedge and ball are
1 2 Fig. 2.196
v1 = m/s and v2 = m/s in horizontal
3 3
direction as shown in Fig. 2.199.
Note: If a particle (or a body) can move in a straight line and we want to find its velocity
from the given conditions we take only one unknown v. If the particle can move in a plane
we take two unknowns vx and vy (with x ^ y). Similarly, if it can move in space we take
three unknowns vx, vy and vz.
For instance in the above problem, the wedge can move only in horizontal line, so we
took only one unknown v1. The ball can move in a plane, so we took two unknowns v2 and
v3. Further, note that x- and y-axis should be perpendicular to each other. They may be along
horizontal and vertical or along common tangent (along the plane in this case) and common
normal (perpendicular to plane).
Problem 2.147 A smooth sphere of mass m is moving on a horizontal plane with a velocity
3i + j when it collides with a vertical wall which is parallel to the vector j. If the coefficient
1
of restitution between the sphere and the wall is , find
2
(a) the velocity of the sphere after impact,
(b) the loss in kinetic energy caused by the impact,

(c) The impulse J that acts on the sphere.

Solution Let v be the velocity of the sphere after impact. r 1
 v
To find v we must separate the velocity components parallel and
3
perpendicular to the wall. e(3) =
2
Using the law of restitution the component of velocity parallel to the
1 r
wall remains unchanged while component perpendicular to the wall u
becomes e times in opposite direction.
3
 3
Thus, v = - i + j Fig. 2.197
2
3
(a) Therefore, the velocity of the sphere after impact = - i + j
2

1 ÊÏ3¸ ˆ 27
2
1 1 1
(b) The loss in KE = mv 2 - mu2 = m(32 + 12 ) - m Á Ì ˝ + 11 ˜ = m
2 2 2 2 ËÓ2˛ ¯ 8
      Ê 3 ˆ 9
(c) J = DP = P f - Pi = m(v ) - m(u) = m Á - i + j˜ - m(3i + j) = - mi
Ë 2 ¯ 2
ŽůůŝƐŝŽŶƐപ253

Problem 2.148 A space shuttle, while travelling at a speed of 4000 km/h with respect to the
earth, disconnects and ejects a module backward, weighing one-fifth of the residual part. If
the shuttle ejects the disconnected module at a speed 100 km/h with respect to the state of
the shuttle before the ejection, find the final velocity of the shuttle.
Solution Suppose the mass of the shuttle including the module is M. The mass of the
module will be M/6. The total linear momentum before disconnection
= M (4000 km/h)
The velocity of the ejected module with respect to the earth = its velocity with respect to
the shuttle + the velocity of the shuttle with respect to the earth
= – 100 km/h + 4000 km/h = 3900 km/h.
If the final velocity of the shuttle is V then the total final linear momentum
5M M
= V+ ¥ 3900 km/h.
6 6
By the principle of conservation of linear momentum,
5M M
M(4000 km/h) = V+ ¥ 3900 km/h
6 6
or, V = 4020 km/h.
Problem 2.149 The hero of a stunt film fires 50 g bullets from a machine gun, each at a speed
of 1.0 km/s. If he fires 20 bullets in 4 seconds, what average force does he exert against the
machine gun during this period?
Solution The momentum of each bullet
= (0.050 kg) (1000 m/s) = 50 kg – m/s
The gun is imparted this much of momentum by each bullet fired. Thus, the rate of change
of momentum of the gun
(50 kg - m/s) ¥ 20
= = 250 N.
4s
In order to hold the gun, the hero must exert a force of 250 N against the gun.
Problem 2.150 A block moving horizontally on a smooth surface with a speed of 20 m/s
bursts into two equal parts continuing in the same direction. If one of the parts moves at 30
m/s with what speed does the second part move and what is the fractional change in the
kinetic energy?
Solution There in no external force on the block. Internal forces break the block in two
parts. The linear momentum of the block before the break should therefore be equal to the
linear momentum of the two parts after the break. As all the velocities are in same direction,
we get,
M M
M(20 m/s) = (30 m/s)+ v
2 2
254പWŚLJƐŝĐƐĨŽƌ//dͲ:͗DĞĐŚĂŶŝĐƐ//

where v is the speed of the other part. From this equation v = 10 m/s. The change in kinetic
energy is
1M 1M 1
(30 m/s)2 + (10 m/s)2 - M(20 m/s)2
2 2 2 2 2
M m2 Ê m2 ˆ
= ( 450 + 50 - 400) 2 = Á 50 2 ˜ M
2 s Ë s ¯
Hence, the fractional change in the kinetic energy

Ê m2 ˆ
Á 50 s2 ˜
Ë ¯ 1
= = .
1 4
M(20 m/s)2
2

Problem 2.151 A car of mass M is moving with a uniform velocity v on a horizontal road
when the hero of a Hindi film drops himself on it from above. Taking the mass of the hero
to be m, what will be the velocity of the car after the event?
Solution Consider the car plus the hero as the system in the horizontal direction, there is
no external force. Since the hero has fallen vertically, so his initial horizontal momentum = 0
Initial horizontal momentum of the system = Mv towards right.
Finally, the hero sticks to the roof of the car, so they move with equal horizontal velocity
say V. Final horizontal momentum of the system
= (M + m)V
Hence, Mv = (M + m)V
Mv
or V = .
M+m

Problem 2.152 Two blocks A and B of masses m and 2m, respectively are connected by
a spring of force constant k. The masses are moving to the right with uniform velocity v
each, the heavier mass leading the lighter one. m
The spring is in the natural length during this m 2m
motion. Block B collides head on with a third k
B
v C
A
block C of mass m, at rest, the collision being
completely inelastic. Calculate the maximum
compression of the spring. Fig. 2.198

1 3mv 2 mv 2
(a) (b)
2 k 12k

3mv 2 mv 2
(c) (d)
4k 4k
ŽůůŝƐŝŽŶƐപ255

Solution
A B C
k
v
m 2m m

Fig. 2.199

Collision between blocks B and C


2mv = (2m + m)v¢
2v
fi v¢ =
3
After the collision the blocks move as shown in Fig. 2.200.
A
k
v 2v/3
m 3m

Fig. 2.200

When both the blocks get equal velocity the spring will have maximum compression. Let
the velocity be v0.
Using principle of conservation of momentum, we have
2v
mv + 3m ¥ = mv0 + 3mv0
3
3v
After solving, we get v0 =
4
Using conservation of energy just after collision and at the time of maximum compression.
1 1 1 1
mvA2 + 3mv(2B + C ) = 4 mvCM
2
+ kx 2
2 2 2 2
1 1 1 1
mv 2 + mv ¢ 2 = mv02 + kx 2
2 3 4 2
1 4v 2 1 9v 2 1 2
mv 2 + 3m = 4m + kx
2 9 2 16 2
Ê 1 2 9ˆ 2 1 2
ÁË + - ˜¯ mv = 4 x
2 3 8 2
1 1
mv 2 = 4 x 2
24 2
mv 2
x =
12k
256പWŚLJƐŝĐƐĨŽƌ//dͲ:͗DĞĐŚĂŶŝĐƐ//

Problem 2.153 Two blocks A and B, each of mass m, v L


are connected by a massless spring of natural length L C A B
and spring constant k. The blocks are initially resting on
a smooth horizontal floor with the spring at its natural
Fig. 2.201
length, as shown in Fig. 2.201. A third identical block C,
also of mass m, moves on the floor with a speed v along the line joining A and B, and collides
elastically with A. Then (JEE, 1984)
(a) the kinetic energy of the A–B system, at maximum compression of the spring, is zero.
(b) the kinetic energy of the A–B system, at maximum compression of the spring, is mv2/4.
(c) the maximum compression of the spring is v (m/k ) .
(d) the maximum compression of the spring is v (m/2k ) .
Solution As a result of head-on collision between C and A, C stops and A begins to move
with speed v. It compresses the spring L which pushes the block B towards right. At maximum
compression, A and B both have same speed v/2 (from conservation of momentum).
v v=0 v/2 v/2
L
A B A B

Fig. 2.202

Let x be the maximum compression of the spring. Then the KE of A–B system at maximum
compression,
2 2
1 Ê vˆ 1 Ê vˆ mv 2
K = mÁ ˜ + mÁ ˜ =
2 Ë 2¯ 2 Ë 2¯ 4
Again by conservation of mechanical energy,
1 1 1
mv 2 = mv 4 + kx 2
2 4 2
1 2 1
or kx = mv 2
2 4
m
or x = v
2k

Problem 2.154 Two blocks A and B of masses m and B


2m respectively are placed on a smooth floor. They C v0 A
2m
are connected by a spring. A third block C of mass m m
m moves with a velocity v0 along the line joining A
and B and collides elastically with A, as shown in
Fig. 2.203. At a certain instant of time t0 after collision,
Fig. 2.203
it is found that the instantaneous velocities of A and
B are the same. Further, at this instant the compression of the spring is found to be x0.
Determine: (i) the common velocity of A and B at time t0, and (ii) the spring constant.
(JEE, 1993)
ŽůůŝƐŝŽŶƐപ257

Solution Initially, the blocks A and B are at rest and C is moving with velocity v0 to the
right.
As masses of C and A are same and the collision is elastic the body C transfers its whole
momentum mv0 to body A and as a result the body C stops and A starts moving with velocity
v0 to the right. At this instant the spring is uncompressed and the body B is still at rest.
The momentum of the system at this instant = mv0
Now, the spring is compressed and the body B comes in motion. After time t0, the
compression of the spring is x0 and common velocity of A and B is v (say).
As external force on the system is zero, the law of conservation of linear momentum gives
mv0 = mv + (2m)v
v0
or v = …(i)
3
The law of conservation of energy gives
1 1 1 1
mv02 = mv 2 + (2m)v 2 + kx02
2 2 2 2
1 3 1
or mv02 = mv 2 + kx02 ...(ii)
2 2 2
1 3 Êv ˆ 1
mv02 = m Á 0 ˜ + kx02
2 2 Ë 3¯ 2
1 2 1 1
\ kx0 = mv02 - mv02
2 2 6
1 2 1
or kx0 = mv02
2 3
2 mv02
\ k = .
3 x02

Problem 2.155 Two identical blocks each of mass M = 9 kg are


placed on a rough horizontal surface of frictional coefficient m v0 A B
= 0.1. The two blocks are joined by a light spring and block B M M
is in contact with a vertical fixed wall as shown in Fig. 2.204. A m
bullet of mass m = 1 kg and v0 = 10 m/s hits block A and gets
Fig. 2.204
embedded in it. (BITSAT, 2001)
Find the maximum compression of spring. (Spring constant = 240 N/m, g = 10 m/s2)
Solution For the collision
1 ¥ 10 = 10 ¥ v fi v = 1 m/s
If x be the maximum compression
1 1
¥ 10 ¥ 12 = m(m + M ) gx + kx 2
2 2
1
5 = 10x + 120x2 fi x= m
6
258പWŚLJƐŝĐƐĨŽƌ//dͲ:͗DĞĐŚĂŶŝĐƐ//

Problem 2.156 Two blocks A and B of masses m and 2m, v


respectively, are connected with the help of a spring having m 2m m
spring constant k as shown in Fig. 2.205. Initially, both the A B C
blocks are moving with same velocity v on a smooth horizontal
plane with the spring in its natural length. During their course Fig. 2.205
of motion, block B makes an inelastic collision with block C
of mass m which is initially at rest. The coefficient of restitution for the collision is 1/2. The
maximum compression in the spring is

2m
(a) (b) will never be attained
k

m m
(c) v (d) v
12k 6k

Solution For collision of B and C v At rest v1 v2


2mv = 2mv1 + mv2 B C B C
1 v -v
= 2 1 fi 2v2 - 2v1 = v Before collision After collision
2 v
v Fig. 2.206
Solving the above equations, v1 = and v2 = v
2
Now for blocks A and B plus spring system, using reduced mass concept, and applying
work–energy theorem, let maximum compression in the spring be x0 and at the time of
maximum compression relative velocity of blocks be zero. Reduced mass m is given by
m ¥ 2m 2m
m = =
3m 3
m ¥ (v - v/2)2 kx 2 Ê mˆ
0– = – 0 fi x0 = Á ˜v
2 2 Ë 6k ¯
y
Problem 2.157 A small ball of mass m is attached with the upper end
of a uniform straight rod of equal mass m and length l. The rod is held
vertical over a smooth horizontal surface as shown in Fig. 2.207. When m
the system is released, the lower end slips freely and the system falls m, l
down. Assume the initial position of the lower end to be the origin l
P
4
and initially rod to be along y-axis as shown in Fig. 2.207. What is the x
equation of trajectory of point P of the rod (P is at distance l/4 from O
the lower end)? Fig. 2.207
Solution Since the floor is smooth, no horizontal force is acting on the system. Only three
forces are acting on it, weight mg of the ball, weight mg of the rod and vertically upward
reaction of the floor.
Since the rod is released from rest and no horizontal force is acting on it, therefore centre of
mass of the system will not displace horizontally; it means it will fall vertically downwards.
Since masses of the rod and ball are equal, therefore centre of mass c if at mid-point of
centre of the rod and the ball; it means at distance l/4 from the ball or 3l/4 from the lower end.
ŽůůŝƐŝŽŶƐപ259

If at an instant the rod makes angle q with the vertical, then point y
P and the system will be as shown in Fig. 2.208. l
Co-ordinates of the point P are 4
m
l
l
x = sin q 2
2 c l
q P 4
l
y = cos q
4 x
O
2x
\ sin q = Fig. 2.208
l
4y
cos q =
l
Using sin2 q + cos2 q = 1, we have
4x2 + 16y2 = l2

Problem 2.158 A point mass of 1 kg collides with a stationary point mass of 5 kg. After
their collision, the 1 kg mass reverses its direction and moves with a speed of 2 ms–1. Which
of the following statement(s) is/are correct for the system of these two masses.
(IIT-JEE, 2010)
(a) Total momentum of the system is 3 kg ms–1
(b) Momentum of 5 kg mass after collision is 4 kg ms–1
(c) Kinetic energy of the centre of mass is 0.75 J
(d) Total kinetic energy of the system is 4 J
Solution
u
1 5
Before collision
2 m/s v
1 5
After collision

Fig. 2.209

Since collision is elastic, so e = 1


Velocity of approach = Velocity of separation
So, u =v+2 ...(i)
By momentum conservation
1 ¥ u = 5v – 1 ¥ 2
u = 5v – 2 fi v + 2 = 5v – 2
So, v = 1 m/s and u = 3 m/s
Momentum of system = 1 ¥ 3 = 3 kgm/s
Momentum of 5 kg after collision = 5 ¥ 1 = 5 kgm/s
260പWŚLJƐŝĐƐĨŽƌ//dͲ:͗DĞĐŚĂŶŝĐƐ//

So, kinetic energy of the centre of mass


2
1 Ê m1u ˆ
= (m1 + m2 ) Á
2 Ë m1 + m2 ˜¯
2
1 Ê 1 ¥ 3ˆ
= (1 + 5) Á = 0.75 J
2 Ë 6 ˜¯
1
Total kinetic energy = ¥ 1 ¥ 32 = 4.5 J.
2

Problem 2.159 A rifle bullet loses 1/20th of its velocity in passing through a plank. The
least number of such planks required just to stop the bullet is (BITSAT, 2009)
(a) 5 (b) 10
(c) 11 (d) 20
Solution Let the thickness of one plank be s if bullet enters with velocity u then it leaves
with velocity

Ê u ˆ 19 19
v = ÁË u - ˜¯ = u u
v=
20
u
20 20
From v2 = u2 – 2as
s
2
Ê 19 ˆ 2 400 u2
fi ÁË u˜ = u - 2 as fi =
20 ¯ 39 2 as Fig. 2.210
Now if the n planks are arranged just to stop the bullet then again
from v2 = u2 – 2as
u v =0
0 = u2 – 2ans
u2 400
fi n = =
2 as 39 ns
fi n = 10.25 Fig. 2.211
As the planks are more than 10 so we can consider n = 11.
Hence, (c) is the correct answer.
Problem 2.160 A stationary light, smooth pulley can rotate without friction
about a fixed horizontal axis. A light rope passes over the pulley. One end of the
rope supports a ladder with man and the other end supports a counterweight of
mass M. Mass of the man is m. Initially, the centre of mass of the counterweight
is at a height h from that of man as shown in Fig. 2.212.
If the man starts to climb up the ladder slowly, calculate the work done by
him to reach his centre of mass in level with that of the counterweight. M

Solution Mass of ladder = M – m, because net mass on both sides of string is h


same. Let in the given process, block M rise by height y, then ladder will come m
down by distance y and when man comes in the level of M, then height raised
by man w.r.t. ground will be h + y. Fig. 2.212
ŽůůŝƐŝŽŶƐപ261

Now centre of mass (man + ladder) system will be raised by y, then


m( h + y ) + ( M - m)(- y ) mh
y= fi y=
M 2( M - m)
Now work done by man:
y
W = total change in potential energy of system h +y
M
fi W = mgy + mg(h + y) – (M – m)g y. y
h
Mmgh m
Put the values of y and solve to get W = .
M-m
Fig. 2.213

Problem 2.161 A ball is projected from the ground with speed u at an angle a with the
horizontal. It collides with a wall at a distance a from the point of projection and returns to
its original position. Find the coefficient of restitution between the ball and the wall.
(BITSAT, 2007)
Solution As we have discussed in the theory, the horizontal
component of the velocity of the ball during the path OAB is u cos C
a while in its return journey BCO it is eu cos a. The time of flight
T also remains unchanged. Hence, B

T = tOAB + tBCO A

2u sin a a a
or = + O
g u cos a eu cos a
a
a 2u sin a a
or = -
eu cos a g u cos a Fig. 2.214

a 2u2 sin a cos a - ag


or =
eu cos a gu cos a
ag 1
\ e = or e =
2
2u sin a cos a - ag Ê u2 sin 2a ˆ
Á - 1˜
Ë ag ¯
Note: The concept which we have discussed in the oblique collision can also be applied
when a ball collides with a wedge.
Problem 2.162 Two particles A and B of mass 1 kg and 2 kg respectively B
are projected in the directions shown in Fig. 2.215 with speeds uA =
200 m/s and attained by the centre of mass of the particles. Assume uB
acceleration due to gravity to be constant. (g = 10 m/s2)
90 m
Solution Using m Ar A = m Br B uA
or (1)(rA) = (2)(rB) ...(i)
and rA + rB = 90 m ...(ii) A

Solving these two equations, we get


rA = 60 m and rB = 30 m Fig. 2.215
262പWŚLJƐŝĐƐĨŽƌ//dͲ:͗DĞĐŚĂŶŝĐƐ//

i.e., CM is at height 60 m from the ground at time t = 0


 
 m a + mB a B
Further, a CM = A A
mA + mB
= g = 10 m/s2 (downwards)
 
as a A = aB = g (downwards)
 
 mA u A + mB uB
u CM =
mA + mB
(1)(200) - (2)(50) 100
= = m/s (upwards)
1+ 2 3
Let h be the height attained by CM beyond 60 m. Using
v2CM = u2CM + 2aCMh
2
Ê 100 ˆ
or 0= Á - (2)(10)h
Ë 3 ˜¯
(100)2
or h= = 55.55 m
180
Therefore, maximum height attained by the centre of mass is
H = 60 + 55.55 = 115.55 m
Problem 2.163 A wooden plank of mass 20 kg is resting on a
smooth horizontal floor. A man of mass 60 kg starts moving from
one end of the plank to the other end. The length of the plank
is 10 m. Find the displacement of the plank over the floor when
the man reaches the other end of the plank.
Option: (a) 8 m (b) 7.5 m
10 m
(c) 7 m (d) 11 m
Fig. 2.216
Solution Here, the system is man + plank. Net
force on this system in horizontal direction is zero
and initially the centre of mass of the system is at
rest. Therefore, the centre of mass does not move in
horizontal direction. x
Let x be the displacement of the plank. Assuming x=0 10 m Initial position
the origin, i.e., x = 0 at the position shown in Fig.
2.217.
As we said earlier also, the centre of mass will
not move in horizontal direction (x-axis). Therefore,
for centre of mass to remain stationary,
x i = xf x 10 – x Final position
Ê 10 ˆ Ê 10 ˆ
(60)(0) + 20 Á ˜ (60)(10 - x ) + 20 Á - x˜ Fig. 2.217
Ë 2¯ Ë 2 ¯
=
60 + 20 60 + 20
ŽůůŝƐŝŽŶƐപ263

Ê 10 ˆ
6(10 - x) + 2 Á - x˜
5 Ë 2 ¯ 60 - 6 x + 10 - 2 x
or = =
4 8 8
or 5 = 30 – 3x + 5 – x
or 4x = 30
30
or x = m
4
or x = 7.5 m
Note: The centre of mass of the plank lies at its centre.
Problem 2.164 A child of mass 4 kg jumps from cart B to cart
A and then immediately back to cart B. The mass of each cart
A B
is 20 kg and they are initially at rest. In both the cases the child
jumps at 6 m/s relative to the cart. If the cart moves along the
same line with negligible friction with the final velocities of VB
and VA, respectively, find the ratio of 6vB and 5vA. Fig. 2.218

Solution All the velocities shown in diagrams are w.r.t. the ground. After first jump:
v2
4 kg
v1
A 20 kg B 20 kg

Fig. 2.219

20v1 = 4v2 and v 1 + v2 = 6 (given)


Solve to get v1 = 1 m/s, v2 = 5 m/s
When child arrives on A:

v3 A B v1

Fig. 2.220
5
(20 + 4) v3 = 4v2 fi v3 = m/s
After the second jump: 6
v4
vA A B v1

Fig. 2.221

v4 + vA = 6, 24v3 = 20vA – 4vA


11 25
Solve to get vA = m/s, v4 = m/s
6 6
264പWŚLJƐŝĐƐĨŽƌ//dͲ:͗DĞĐŚĂŶŝĐƐ//

when child arrives on B:

vA A B vB

Fig. 2.222

24vB = 4v4 + 20v1


Ê 25 ˆ 55
fi 24vB = 4 Á ˜ + 20 ¥ 1 fi vB = m/s
Ë 6¯ 36

6vB 6 ¥ 55 ¥ 6
Now, = = 1.
5vA 36 ¥ 5 ¥ 11

Problem 2.165 Two identical balls A and B each of mass 2 kg


and radius R are suspended vertically from inextensible strings as
1 kg g
shown. Third ball C of mass 1 kg and radius r = ( 2 - 1) R falls
and hits A and B symmetrically with 10 m/s. Speed of both A and c m
r r
B just after the collision is 3 m/s.
Speed of C just after collision is R R
(a) 2 m/s (b) 2 2 m/s
2 kg 2 kg
(c) 5 m/s (d) ( 2 - 1) m/s
A B
Solution As the balls A and B are constrained to move horizon-
tally, if I be the impulse imparted by ball C to each of A and B, the Fig. 2.223
impulse received by ball C then would be 2I cos q.
Now, each of ball B and C received impulse I as q
I 90 I–
shown, but moves horizontally as its vertical as its q
vertical comp. gets balanced by impulse imparted C
to ball B and C by the respective strings and hence.
A q B
I cos q = MAVA = MBVB 3 m/s 3 m/s
q
M AVA
fi I = I
cos q 10 m/s I
(I = magnitude of impulse) Fig. 2.224
Now, for ball C, if its final velocity is Vc¢ downwards.
we have McVc¢ = McVc – 2I cos q
M
fi Vc¢ = Vc - 2 A VA
MC

= – 2 m/s (– ve sign indicates that Vc¢ is directed upwards)]
\ (a) is the correct option.
Problem 2.166 Impulse provided by each string during collision is
(a) 6 2 N sec (b) 12 N sec (c) 3 2 N sec (d) 6 N sec
Solution Impulse provided by each string I cos q = 6 N sec.
ŽůůŝƐŝŽŶƐപ265

Problem 2.167 The value of coefficient of restitution is


1 1 2
(a) (b) 45° 2 sin 45°
4 2
1
(c) ( 2 - 1) (d) 3
2 45°

2 sin 45∞ + 3 cos 45∞ 1 3 cos 45°


Solution e= =
10 sin 45∞ 2 Fig. 2.225
\ (d) is correct.
Problem 2.168 Two pendulum bobs of mass m and 2m collide elastically at the lowest point
in their motion. If both the balls are released from a height H above the lowest point, to what
heights do they rise for the first time after collision?
Solution Given, m1 = m, m2 = 2m, v1 = – 2gH and v2 = 2gH
Since, the collision is elastic. Using Eqs. (iii) and (iv) discussed in the theory the velocities
after collision are
Ê m - 2m ˆ Ê 4m ˆ
v 1¢ = Á (- 2 gH ) + Á 2 gH
Ë m + 2m ˜¯ Ë m + 2m ˜¯

2 gH 4 2 gH 5
= + = 2 gH
3 3 3
2 1 +ve
Ê 2m - m ˆ Ê 2m ˆ
and v 2¢ = Á ( 2 gH ) + Á (- 2 gH )
Ë m + 2m ˜¯ Ë m + 2m ˜¯
v2 v1

2 gH 2 2 gH 2 gH Fig. 2.226
= - =-
3 3 3
i.e., the velocities of the balls after the collision are as shown Fig. 2.227.
Therefore, the heights to which the balls rise after the collision are:
(v1¢ )2
h1 = (using v2 = u2 – 2gh)
2g
2
Ê5 ˆ
ÁË 2 gH ˜
3 ¯ 25
or h1 = or h1 = H
2g 9
2
Ê 2 gH ˆ
Á 3 ˜ 2gH v 1¢ =
5
( v ¢ )2 Ë ¯ v 2¢ = 3
2gH
and h2 = 2 or h2 = 3
2g 2g
Fig. 2.227
H
or h2 =
9
266പWŚLJƐŝĐƐĨŽƌ//dͲ:͗DĞĐŚĂŶŝĐƐ//

Note: Since the collision is elastic, mechanical energy of both the balls will remain conserved,
or
Ei = Ef
fi (m + 2m)gH = mgh1 + 2mgh2
Ê 25 ˆ Ê Hˆ
fi 3mgH = (mg ) Á H ˜ + (2mg ) Á ˜
Ë 9 ¯ Ë 9¯
fi 3mgH = 3mgH.
Problem 2.169 Figure 2.228 shows a smooth spherical ball of
mass m striking two identical equilateral triangular wedges of
mass M. At the instant of impact velocity of the ball is v0. Taking m
coefficient of restitution e, determine the velocities of the sphere
and the wedges just after collision. v0
M M
Solution (by impulse method) Let J be the impulse between
the ball and wedges and v1 and v2 be the velocities of the ball 60° 60°
and the wedge.
From impulse-momentum equation on the ball, Fig. 2.228

2J sin 30° = mv1 – (mv0)


J = mv1 + mv0 ...(1)
From the wedge, J cos 30° = Mv2 ...(2)
On eliminating J from Eqs. (1) and (2), we have
2
Mv2 = mv1 + mv0 ...(3)
3
From the definition of coefficient of restitution, v1
v1 cos 60∞ + v2 cos 30∞ v1 + 3 v2
e = =
v0 cos 60∞ v0

or ev0 = v1 + 3 v2 ...(4) J J

v2 J v2
On solving Eqs. (3) and (4) for v1 and v2, we get
M
J
(2eM - 3m)v0 60° 60°
v1 =
2 M + 3m
Fig. 2.229
3 (1 + e )mv0
v2 = .
2 M + 3m

Problem 2.170 Two spheres are moving towards each other. Both have same radius but
their masses are 2 kg and 4 kg. If the velocities are 4 m/s and 2 m/s respectively and
coefficient of restitution is e = 1/3, find:
(a) The common velocity along the line of impact.
(b) Find velocities along line of impact.
(c) Impulse of deformation.
ŽůůŝƐŝŽŶƐപ267

(d) Impulse of reformation.


(e) Maximum potential energy of deformation.
(f) Loss in kinetic energy due to collision.

2 kg
4 m/s
A
R R
2 m/s

B
4 kg

A 4 m/s C Line of motion


2 kg q
R R
R 4 kg
Line of motion 2 m/s B
Line of impact

Fig. 2.230

BC R 1
Solution In DABC sin q = = =
AB 2R 2
or q = 30°
(a) By conservation of momentum along the line of impact.
Line of impact
4 sin 30°

2 kg 4 m/s
30°
4 cos 30° 4 sin 30°
2 cos 30°
30°
B 4 kg
v
2 sin 30° 2 sin 30°

Just before collision along the line of impact Maximum deformed state

Fig. 2.231

2(4 cos 30°) – 4(2 cos 30°) = (2 + 4)v


or v = 0 (common velocity along the line of impact)
268പWŚLJƐŝĐƐĨŽƌ//dͲ:͗DĞĐŚĂŶŝĐƐ//

(b) Let v1 and v2 be the final velocity of A and B respectively then, by conservation of
momentum along the line of impact.
4 sin 30°

A v
2 kg 1
4 kg B
v2
2 sin 30°

Just after collision along the line of impact

Fig. 2.232

2(4 cos 30°) – 4(2 cos 30°) = 2(v1) + 4(v2)


or 0 = v1 + 2v2 ...(1)
By coefficient of restitution,
Velocity of separation along the line of impact
e =
Velocity of approach along the line of impact
1 v2 - v1
or =
3 4 cos 30∞ + 2 cos 30∞

or v2 – v1 = 3 ...(2)
From the above two equations,
2
v1 = - m/s
3
1
and v2 = m/s
3
(c) JD = m1(v – v1)
= 2 (0 – 4 cos 30°) = - 4 3 N-s
1 4
(d) JR = e JD = (- 4 3 ) = - N-s
3 3
(e) Maximum potential energy of deformation is equal to loss in kinetic energy during
deformation up to maximum deformed state.
1 1 1
U = m1 (u1 cos q )2 + m2 (u2 cos q )2 - (m1 + m2 )v 2
2 2 2
1 1 1
= 2( 4 cos 30∞)2 + 4(- 2 cos 30∞)2 - (2 + 4)(0)2
2 2 2
or U = 18 joules.
ŽůůŝƐŝŽŶƐപ269

(f) Loss in kinetic energy


1 1 Ê1 1 ˆ
DKE = m1 (u1 cos q )2 + m2 (u2 cos q )2 – Á m1v12 + m2v22 ˜
2 2 Ë2 2 ¯

1 1 Ê 1 Ê 2 ˆ 2 1 Ê 1 ˆ 2ˆ
= 2( 4 cos 30∞)2 + 4(- 2 cos 30∞)2 – Á 2 Á ˜ + 2 4 ÁË ˜ ˜
2 2 Ë 2 Ë 3¯ 3¯ ¯
DKE = 16 joules
Problem 2.171 Two equal discs initially at rest are in contact on a
table. A third disc of same mass but of double radius strikes them
symmetrically and itself comes to rest after the impact. What is the
coefficient of restitution?

Solution (By Impulse Method)


Fig. 2.233
J
J

J 3R

a
a 2R
2J cos a
2 2R
J
3R

J
J

Fig. 2.234

2J cos a = mv ...(1)
2 2p 2 2
cos a = = m
3R 3

mv 3mv
J = =
2 cos a 4 2
3mv
J = mv ¢ = m
4 2
v
Ê 3v ˆ
v¢ = Á
Ë 4 2 ˜¯ v¢
m
Èv -v ˘
e = -Í 2 1 ˙ Fig. 2.235
Î u2 - u1 ˚
2 2v
u1 = v cos a =
3
v1 = 0, u2 = 0
270പWŚLJƐŝĐƐĨŽƌ//dͲ:͗DĞĐŚĂŶŝĐƐ//

3v
v2 =
4 2

È 3v ˘
Í 4 2 -0 ˙ 9
e =– Í ˙=
Í 2 2v ˙ 16
ÍÎ 0 -
3 ˙˚
#NVGTPCVKXG
2 2
cos q =
3 v
Assume initial velocity of big block = u R
and final velocity of small ball is v
2R R
Conserving momentum u q
fi mu = 2mv cos q ...(1)
v2 - v1 v
fi –e =
u2 - u1

v-0 Fig. 2.236


fi –e =
0 - u cos q
v
fi e = ...(2)
u cos q
From Eqs. (1) and (2)
1 9
e = 2
=
2 cos q 16
y
Problem 2.172 Two smooth balls A and B, each of mass m
and radius R, have their centres at (0, 0, R) and at (5R, – R, R)
x(m)
respectively, in a coordinate system as shown in Fig. 2.237. Ball
A, moving along positive x-axis, collides with ball B. Just before A
the collision, speed of ball A is 4 m/s and ball B is stationary. The B
collision between the balls is elastic.
Fig. 2.237
1. What is the velocity of the ball A just after collision?
2. What is the impulse of the force exerted by A on B during the collision?
3. If coefficient of restitution during the collision is changed to 1/2 keeping all other
parameters unchanged what is the velocity of the ball after the collision?

Solution
1. vA = 4 sin 30° [cos 60i + sin 60j]
vA = i + 3 j m/s
ŽůůŝƐŝŽŶƐപ271

A
4 m/s
30°
R
R

Before collision

4 sin 30°

60°
30°

B 4 cos 30°

After collision

Fig. 2.238
  
2. JA on B = mv Bf - v Br
= m[4 cos 30∞(cos 30∞i - sin 30∞j) - 0]
= (3mi - 3mj) kg-m/s

3.
4 sin 30°

A
60°
0
30°

4sin 30°
B
0
Before collision

4 sin 30°

v1 30°

v2
After collision

Fig. 2.239
272പWŚLJƐŝĐƐĨŽƌ//dͲ:͗DĞĐŚĂŶŝĐƐ//

1 - (v2 - v1 )
1. =
2 (0 - 4 cos 30∞)
v2 – v 1 = 3
4 3
2. m = mv1 + mv2
2
v2 + v1 = 2 3
3 3
v2 = m/s
2
3 3
v2 = m/s[cos 30 i + sin 30∞(- j)]
2
Ê9 3 3 ˆ
= Á i - j .
Ë4 4 ˜¯
Problem 2.173 A cart of mass M rests on a frictionless
horizontal surface and a pendulum bob of mass m hangs
from the roof of the cart (see Fig. 2.240). The string breaks,
L
the bob falls on the floor, makes several collisions on the
floor and finally lands up in a small slot made in the floor.
The horizontal distance between the string and the slot is
L. Find the displacement of the cart during this process. Fig. 2.240
(BITSAT, 2007)
Solution As no external force is acting in horizontal direction, so
there is no shift in position of CM in this direction. If Dx1 and Dx2 are Dx 2
the displacements of bob and cart respectively, then
 
mDx1 + M Dx 2 = 0
Dx1
Here, Dx1 = (L – Dx2) = L - Dx 2
\ m(L – Dx2) + M(– Dx2) = 0
mL
or Dx2 = Fig. 2.241
m+ M
Problem 2.174 A car P is moving with a uniform speed of 5 3 m/s towards a carriage of
mass 9 kg at rest on the rails at a point B are shown in Fig. 2.242. The height AC is 120 m.
Cannon balls of 1 kg are fired from the car with an initial velocity of 100 m/s at an angle
30° with the horizontal. The first cannon ball hits the stationary carriage after a line to and
sticks to it. Determine t0. (IIT-JEE, 2001)
At t0 the second cannon ball is fired.
Assume that the resistive force between P C
the rails and the carriage is constant and
ignore the vertical motion of the carriage
throughout. If the second ball also hits
and sticks to the carriage, what will be the A
B
horizontal velocity of the carriage just after
Fig. 2.242
the second impact?
ŽůůŝƐŝŽŶƐപ273

Solution For vertical motion of ball, we have u sin 30°


u
1 2
120 = – u sin 30∞t0 + gt0
2 u cos 30°
1
120 = – 100 sin 30∞t0 + gt02 P C
120 m
2
After solving, t0 = 12 s. A B
The velocity of the ball in horizontal direction, w.r.t. car
Fig. 2.243
vx = u cos 30∞ + 5 3
= 100 cos 30° + 5 3 = 55 3 m/s
By conservation of momentum, we have
mvx = (m + M)v
or 1 ¥ 55 3 = (1 + 9)v
11 3
\ v = m/s
2
If a is the retardation of the carriage, then in the next t0
= 12 s, the distance travelled
1
x = vt0 - at02
2
The distance travelled by this time
= 5 3 t0 = 60 3 m
According to the given condition, we have
1
vt0 - at02 = 60 3
2
After substituting the known values, we have
1
\ a = m/s2
4 3
The velocity of car at the instant of impact of the second ball

11 3 1
v¢ = v – at0 = - ¥ 12
2 4 3
9
3 m/s=
2
Now again by conservation of momentum
9
1 ¥ 55 3 + 10 ¥ 3 = (10 + 1)v2
2
\ v2 = 15.7 m/s
274പWŚLJƐŝĐƐĨŽƌ//dͲ:͗DĞĐŚĂŶŝĐƐ//

Problem 2.175 A particle of mass m having collided with a stationary particle of mass M
p
deviated by an angle whereas the particle M recoiled at an angle q = 30° to the direction
2
of the initial motion of the particle m. How much (percent) and in what way has the kinetic
M
energy of this system changed after the collision, if = 5.0?
m
Solution The situation is shown is Fig. 2.244.
By conservation of momentum, we have v1
m
mv = Mv2 cos q ...(i)
m v
mv1 = Mv2 sin q ...(ii) 90°
M
q = 30°
Percentage change in KE
M
v2
Ê1 2 1 2ˆ 1 2
DK ÁË mv1 + Mv2 ˜¯ - mv
2 2 2 Fig. 2.244
¥ 100 = ¥ 100
K 1
mv 2
2
After solving the above equations, we get
DK
¥ 100 = – 40%
K

Problem 2.176 A horizontally flying bullet of mass m gets stuck


in a body of mass M suspended by two identical threads of
length l shown in Fig. 2.245. As a result, the threads swerve l l
through an angle q. M

Assuming m << M, find


m
(a) the velocity of the bullet before striking the body; and Fig. 2.245
(b) the fraction of the bullet’s initial kinetic energy that turned
into heat.
Solution
(a) If v be the velocity of the bullet, then q

mv = (M + m)V

È mv ˘
\ V = Í ...(i)
Î M + m ˙˚
h
1
Now, ( M + m)V 2 = (M + m)gh v
v
2 m
where h = (l – l cos q) Fig. 2.246
1
\ ( M + m)V 2 = (M + m)g (l – l cos q) ...(ii)
2
ŽůůŝƐŝŽŶƐപ275

From Eqs. (i) and (ii), we get


M+m
V = 2 gl(1 - cos q )
m

M+m q
= ¥ 2 gl ¥ 2 sin 2
m 2
As m << M, \ M + m = M
2M q
and v  gl sin
m 2

Ê1 1 ˆ
DK ( M + m)V 2 - mv 2
Á2 2 ˜
(b) = Á ˜
K 1 2
ÁË mv ˜¯
2
m
 1-
M

Problem 2.177 A particle of mass m1 experienced a perfectly elastic collision with a stationary
particle of mass m2. What fraction of the kinetic energy does the striking particle lose, if:
(a) it recoils at right angles to its original motion direction; and
(b) the collision is a head-on one?
Solution
v1
(a) The situation is shown in Fig. 2.247. m1
By conservation of momentum, we have
m1 u
m1u = m2v2 cos q ...(i) m2
q
and m1v1 = m2v2 sin q ...(ii) m2
v2
Fractional loss in KE of the striking particle
Fig. 2.247
1 2 1 2
Ki - K f 2 m1u - 2 m1v1
h = =
Ki 1
m1u2
2
u2 - v12
= ...(iii)
u2
After solving the above equations, we get

È 2m1 ˘
h = Í ˙
Î m1 + m2 ˚
(b) m 1u = m 1v 1 + m 2v 2
1 1 1
and m1u2 = m1v12 + m2v22
2 2 2
276പWŚLJƐŝĐƐĨŽƌ//dͲ:͗DĞĐŚĂŶŝĐƐ//

Ki - K f u2 - v12
h = =
Ki u2
4 m1m2
After solving, we get h =
(m1 + m2 )2

Problem 2.178 From a point in a smooth horizontal plane a plane a particle is projected
with velocity u at an angle q to the horizon; if the coefficient of restitution between the
particle and the plane be e, then that the distance described along the plane before the particle
u2 sin 2q
ceases to rebound is .
g (1 - e )

Solution The initial velocity in vertical direction is u sin q. The velocities after first, second,
... rebounds are eu sin q, e2u sin q, ... . Hence, the time between the first and second rebounds
2eu sin q 2e 2u sin q
are, , ,… .
g g
The total time that elapses the particle ceases to rebound
2u sin q 2eu sin q 2e 2u sin q
T = + + +….
g g g
2u sin q
= [1 + e + e 2 + … .]
g
2u sin q
=
g(1 - e )
During this time the horizontal velocity remains constant which is u cos q. Hence, the
horizontal distance described.
2u sin q
R = ¥ u cos q
g(1 - e )

u2 sin 2q
= .
g(1 - e )

Problem 2.179 A body of weight P slides without P


friction down an inclined board into a cart standing
at rest. What velocity v will be imparted to the cart
when the body drops on it? The weight of the cart h Q
is Q, the initial height of the body above the level a
of the cart is h and the angle while the board is
inclined to the horizontal is a (Fig. 2.248). The cart
moves without friction.
Fig. 2.248
ŽůůŝƐŝŽŶƐപ277

Solution The velocity of the body after slide from height


h, v0 = 2gh . h
Its component along horizontal direction will be v0 cos a.
By conservation of momentum, we can write a

P ¥ v0 cos a = (P + Q)v
Pv0 cos a P 2 gh cos a Fig. 2.249
\ v = =
( P + Q) ( P + Q)

Problem 2.180 A heavy ball of mass 2M moving with 2M


u0 1 2 3
a velocity vo collides elastically head-on with a cradle of
M M M
three identical balls each of mass M as shown in Fig. 2.250.
Determine the velocity of each ball after collision. Fig. 2.250 A heavy ball collides a
Solution When the heavy ball of mass 2M collides with cradle of three identi-
the first stationary ball of mass m, their velocities can be cal balls.
obtained by using Eqs. 5 and 6 page no. 136 of this book.
Ê 2M - M ˆ u
Heavy ball: v = Á u = o
Ë 2 M + M ˜¯ o 3

Ê 2(2 M ) ˆ 4
Ball 1: v1 = Á u = u
Ë 2 M + M ˜¯ o 3 o

As Ball 1 gains momentum, immediately it transfers it u0/3 1 2 4u0/3


to Ball 2, which in turn, transfers to Ball 3. Thus, Ball 3
4
comes out with a velocity v3 = vo leaving the balls 1 and Fig. 2.251
2 stationary. 3
Now, the heavy ball collides with Ball 1 with a velocity uo/3 and their final velocities are:
u
v¢ = o
9
4
v1¢ = uo
9
Ball 1 transfers its momentum to Ball 2, which starts u0 1 2
4u 0
4 9
3
moving with a velocity v2 = uo , leaving Ball 1 stationary. 4u 0
9 9
Now, the heavy ball collides with Ball 1 with a velocity
Fig. 2.252
uo
and their final velocities are:
9
u 4
v≤ = o ; v1¢¢ = uo
27 27
278പWŚLJƐŝĐƐĨŽƌ//dͲ:͗DĞĐŚĂŶŝĐƐ//

The final velocity of each ball after collision may be summarized as


2M
1 2 3 2M
u0 u0/27 1 4u0/27 2 4u0/9 3 4u0/3
M M M M M M

Initial Final

Fig. 2.553

Êu ˆ Ê 4u ˆ Ê 4u ˆ Ê 4u ˆ
Pi = 2Muo Pf = 2M Á o ˜ + M Á o ˜ + M Á o ˜ + M Á o ˜
Ë 27 ¯ Ë 27 ¯ Ë 9 ¯ Ë 3 ¯

È1 2 2 2˘
Pf = 2Muo Í + + +
Î 27 27 9 3 ˙˚
Pf = 2Muo
Note: The collision does not occur actually in the sequence described in the illustration. In
fact, all the balls attain their final velocities simultaneously.
Problem 2.181 A ball is released from rest relative to
the elevator at a distance h1 above the floor. The speed
of the elevator at the time of ball release is v0. Determine
the bounce height h2 relative to elevator of the ball (a) if
v0 is constant, and (b) if an upward elevator acceleration
a = g/4 begins at the instant the ball is released. The
coefficient of restitution for the impact is e. a=
g
h1 4
Solution h2
(a) ur = 0, ar = g v0
\ vr = 2 gh1
After collision relative velocity vr¢ = e 2 gh1 and Fig. 2.254
relative retardation is still g (downwards). Hence,

(vr¢ )2
h2 = = e 2 h1
2g
g 5g
(b) ur = 0, ar = g + =
4 4
Ê 5g ˆ
\ Just before collision vr = 2 Á ˜ h1
Ë 4¯
Just after collision ur¢ = evr
5g
Relative retardation is still .
4
(vr¢ )2
Hence, h2 = = e 2 h1
Ê 5g ˆ
2Á ˜
Ë 4¯
ŽůůŝƐŝŽŶƐപ279

Problem 2.182 A gun of mass M (including the carriage) fires a shot of mass m. The gun
along with the carriage is kept on a smooth horizontal surface. The muzzle speed of the
bullet vr is constant. Find:
(a) The elevation of the gun with the horizontal at which maximum range of bullet with
respect to the ground is obtained. vr sin q
(b) The maximum range of the bullet. vr

Solution Muzzle velocity vr is given to be constant. q


vr cos q
From conservation of linear momentum in horizontal m
direction we have,
Mv = m(vr cos q – v) v M

mvr cos q
or v = ...(i)
M+m
Components of velocity
Further, range of bullet on horizontal ground of bullet with respect
to gun
2vr sin q
R = (vr cos q - v) vr sin q
g
2vr sin q Ê mvr cos q ˆ
= ÁË vr cos q - ˜ vr cos q – v
g M+m ¯

2 Mvr2 sin q cos q v M


=
( M + m) g
2
Ê M ˆ vr sin 2q Components of velocity
or R = Á ˜ ...(ii) of bullet with respect
Ë M + m¯ g to ground

(a) From Eq. (ii) we see that maximum range is at Fig. 2.255
q = 45°
(b) At q = 45°,
2
Ê M ˆ vr
Rmax = Á ˜
Ë M + m¯ g

Problem 2.183 A small elastic ball is dropped from a height of 5 m onto a rigid cylindrical
body (see Fig. 2.256) having a radius of 1.5 m. At what position on the x-axis does the ball land?
y

0.6 m

5m

Fig. 2.256
280പWŚLJƐŝĐƐĨŽƌ//dͲ:͗DĞĐŚĂŶŝĐƐ//

Solution The situation is shown in Fig. 2.257.


0.6 2
sin q = = = 0.40 0.6 m
1.5 5
u q u
cos q = 1 - sin 2 q = 0.92
y = R cos q = 1.5 ¥ (0.92)
q y
= 1.38 m
The height fall by the ball before collision,
Fig. 2.257
h = 5 – 1.38 = 3.62 m
The velocity of the ball just before collision

u = 2 gh = 2 ¥ 9.8 ¥ 3.62 = 8.42 m/s


The velocity components after collision are:
ux = u sin 2q
= 8.42 ¥ 2 sin q cos q = 6.20 m/s
and uy = u cos 2q = 8.42(cos2q – sin2q)
= 5.78 m/s
If t is the time taken by the ball to hit the ground,
1 2
y = - uy t + gt
2
9.8 2
1.38 = - 5.78 t + t
2
\ t = 1.38 s
The horizontal distance from the point of collision at which ball lands
= uxt = 6.20 ¥ 1.38 = 8.5 m
The x coordinate = – (8.5 + 0.6) = – 9.1 m
Problem 2.184 A cart is moving along positive z
x-direction with a velocity of 4 m/s. A person on the
cart throws a stone with a velocity of 6 m/s relative u L
to himself. In the frame of reference of the cart the y
stone is thrown in yz-plane making an angle of 30° 30°
with z-axis. At the highest point of the trajectory, the
stone hits an object of equal mass hung vertically 4 m/s
from branch of a tree by means of a string of length
L. A completely inelastic collision occurs, in which
the stone gets embedded in the object. Determine: x

(IIT-JEE, 1997) Fig. 2.258

(i) the speed of combined mass immediately after the collision with respect to an observer
on the ground.
(ii) the length L of the string such that the tension in the string becomes zero when the
string becomes horizontal during the subsequent motion of the combined mass.
ŽůůŝƐŝŽŶƐപ281

Solution The velocity component of stone w.r.t. the ground are


ux = 4 m/s,
uy = 6 sin 30° = 3 m/s,
and uz = 6 cos 30° = 3 3 m/s.

(i) Thus, velocity vector of the stone


  ) m/s
u = ( 4i + 3j + 3 3 k
At the highest point of the trajectory, uz = 0,

\ v = ( 4i + 3j) m/s
By conservation of momentum, we have
 
mv = (m +m) v ¢

 v 4i + 3j
v¢ = = = (2i + 1.5j) m/s
2 2

or v¢ = 22 + 1.52 = 2.5 m/s


(ii) The KE of the combined mass will convert into PE and so
1
(2m)v¢ 2 = (2m)g ¥ L
2
v¢ 2 2.52
or L = = = 0.319 m
2 g 2 ¥ 9.8

Problem 2.185 A ball of mass m is projected with speed u


m
into a barrel of spring gun of mass M initially at rest on a M
frictionless surface. The mass m sticks in the barrel at the
point of maximum compression of the spring. What fraction
of the initial energy of the ball is stored in the spring? Neglect Fig. 2.259
friction.
Solution If u and v are the velocities of the ball before and u
v
after collision, then
mu = (m + M)v …(i)
È mu ˘
\ v = Í Fig. 2.260
Î m + M ˙˚
1 1 1
The initial KE mu2 changes into elastic PE = kx2 and KE of whole system = (m + M) v2
2 2 2
1 2 1 2 1 2
mu = kx + (m + M )v …(ii)
2 2 2
1
Dividing Eq. (ii) by mu2
2 1 2 1
kx + (m + M )v 2
1 = 2 2 …(iii)
1
mu + mu2
2
2
282പWŚLJƐŝĐƐĨŽƌ//dͲ:͗DĞĐŚĂŶŝĐƐ//

kx 2 (m + M )v 2
1 = + …(iv)
v m mu2 mu2
From Eq. (i) =
u M+m
Substituting this value in Eq. (iv)
kx 2 m+ M m2 kx 2 m
1 = + ◊ = +
mu2 m (m + M ) 2
mu 2
m+ M
kx 2 m M
fi =1- =
mu 2
m + M m +M
kx 2
Here, represent the fraction of initial energy.
mu2
Problem 2.186 The magnitude and direction of the two iden- Line of impact
30°
tical smooth balls before central oblique collision are as shown 60°
9 m/s
in Fig. 2.261. Assuming coefficient of restitution e = 0.9, deter- 12 m/s
mine the magnitude and direction of the velocity of each ball
Fig. 2.261
after the collision.
Solution The situation is shown in Fig. 2.262.
u x1 = 9 cos 30∞ u x 2 = 12 cos 60∞

u y 1 = 9 sin 30∞ u y 2 = 12 sin 60∞


= 4.5 m/s = 10.40 m/s

Fig. 2.262

By conservation of momentum along x-axis, we can write

vA uy2
vB

uy2

qA
qB
v x1
v x2

Fig. 2.263

mux1 – mux2 = mvx1 + mvx2


= 9 cos 30° – 12 cos 60° = 7.8 – 6
= 1.8 ...(i)
È v x - v x2 ˘
and e =– Í 1 ˙
ÍÎ ux1 - ux2 ˙˚
È v x - v x2 ˘
or 0.9 = – Í 1 ˙
Î 7.8 - (- 6) ˚
\ – vx1 + vx2 = 12.42 ...(ii)
ŽůůŝƐŝŽŶƐപ283

After solving Eq. (i), (ii), we get


vx1 = – 5.3 m/s, and
vx2 = 7.1 m/s

Thus, vA = vx21 + uy21 = (- 5.3)2 + 4.52


= 6.95 m/s.
Ê 4.5 ˆ
tan qA = Á ; or qA  40°
Ë 5.3 ˜¯

Similarly, vB = 7.12 + 10.40 2 = 12.6 m/s

Ê 10.40 ˆ
and qB = tan - 1 Á = 56°
Ë 7.1 ˜¯

Problem 2.187 Central impact takes place between a 4 kg ball and a 8 m/s
stationary cart of mass 6 kg as shown in Fig. 2.264. The cart rests on rollers 60°
and can move freely on horizontal surface. Find the velocity of the cart
and the ball after impact. Take e = 0.8.
Fig. 2.264
Solution The component of velocity of the ball along the axes y
shown are: x
ux = – 8 cos 30° = - 4 3 m/s
30° 8 m/s
and uy = 8 sin 30° = 4 m/s
60°
The impact will take place along the line of x-axis.
Thus, by conservation of momentum
Fig. 2.265
4 ¥ ux = 4v1x + 6v2x

or 4(- 4 3 ) = 4vx1 + 6v2x


3
or - 4 3 = v x 1 + v2 x ...(i)
2
Èv -v ˘
Also e = -Í 1 2 ˙
Î u1 - u2 ˚
È v - v2 x ˘
or 0.8 = - Í 1x ˙
Î- 4 3 - 0˚
or - 3.2 3 = – v1x + v2x ...(ii)
After solving, we get vx1 = 5.72 m/s v x1
and vx2 = – 5 m/s 30°
v x2 u y1
Velocity of the ball, Vball = vx21 + uy2

= 5.722 + 4 2 = 6.98 m/s Fig. 2.266


284പWŚLJƐŝĐƐĨŽƌ//dͲ:͗DĞĐŚĂŶŝĐƐ//

Horizontal velocity of the block will be


-5¥ 3
Vblock = vx2 cos 30° =
2
= – 4.3 m/s
Problem 2.188 A ball dropped on to a smooth horizontal
floor and bounces as shown in Fig. 2.267. Drive expression
for coefficient of restitution in terms of: h0
(a) two successive heights, and
h1
(b) two successive ranges. Also determine the time of h2
h3
flight in nth bounce. R1 R2 R3
Solution
Fig. 2.267
(a) After falling from height h, the velocity of the ball will
be
u = 2gh h0 h1 hn–1
Before collision, h = hn –1, u1 = 2 ghn - 1 hn

Rn–1 Rn
After collision, h = hn, v1 = - 2ghn
Fig. 2.268
Èv -v ˘
Thus, e = -Í 1 2 ˙
Î u1 - u2 ˚
È - 2 gh - 0 ˘ hn
=–Í n
˙=
ÍÎ 2 ghn - 1 - 0 ˙˚ hn - 1

2ux uy
(b) We have R =
g
Rg
\ uy = [ux constant]
2u x
Rn -1 g
Before collision, u1 =
2ux
- Rn g
After collision, v1 =
2ux
È v -v ˘
e = -Í 1 2 ˙
Î (u1 - u2 ) ˚
È - Rn g/2ux - 0 ˘
= -Í ˙
ÍÎ - Rn - 1 g/2ux - 0 ˙˚
Rn
=
Rn -1
ŽůůŝƐŝŽŶƐപ285

For nth bounce, the time of flight


2hn
Tn = 2tn = 2
g

Problem 2.189 Two small beads A and B of masses 0.003 kg and 0.006 kg respectively are
threaded on a smooth circular wire of radius 0.5 m which is maintained in a horizontal
plane. Initially, the beads A and B are at rest at points P and Q where P and Q are opposite
ends of the diameter of a wire. The coefficient of restitution between beads is 1/4. Bead A
is projected towards B with a speed u m/s. (IIT Roorkee, 1974)
(a) Find the speeds of the beads immediately after collision. Given u = 6 m/s, the time
that elapsed between the first and the second collision.
(b) If the wire is rough and A is projected from P with the same speed 6 m/s so as to
collide with B, which is at rest, the velocity of B after collision is 1.25 m/s. Find the
work done by friction as A goes from P to collide with B and the magnitude of the
impulse acting on A during collision.
Solution
(a) By conservation of momentum, we have
m1u + 0 = m1v1 + m2v2 u
or 0.003 u = 0.003v1 + 0.006v2 R
\ u = v1 + 2v2 ...(i) A B

Èv -v ˘ Èv - v ˘
Also, e = -Í 1 2 ˙ = -Í 1 2 ˙
Î u1 - u2 ˚ Î u-0 ˚
Fig. 2.269
1 Èv - v ˘
or = -Í 1 2 ˙
4 Î u ˚
u
\ = – v1 + v2 ...(ii)
4
After solving the above equations, we get
u 6
v1 = = = 1 m/s
6 6
5u 5
and v2 = = m/s
12 2
Time taken between the first and the second collision

2p R 2p ¥ 0.5 2p
t = = = s
v2 - v1 5
-1 3
2
286പWŚLJƐŝĐƐĨŽƌ//dͲ:͗DĞĐŚĂŶŝĐƐ//

(b) If u¢ is the speed of the bead A before collision, then


m1u¢ = m1v1¢ + m2v2¢
or 0.003u¢ = 0.003v1¢ + 0.006 ¥ 1.25
\ u¢ = v1¢ + 2.50 ...(iii)
v1¢ - v2¢
and e =
u¢ - 0
1 v ¢ - 1.25
or = - 1
4 u¢ - 0

or = – v1¢ + 1.25 ...(iv)
4
From Eqs. (iii) and (iv), we get
u¢ = 3 m/s, v1¢ = 0.5 m/s.
Work done by friction on A
1
= ¥ m1 ¥ [u¢ 2 - u2 ]
2
1
= ¥ 0.003 ¥ (32 - 0.52 )
2
= 0.0405 J
Impulse on A, J = m(v1¢ +u¢)
= 0.003 (0.5 – 3)
= 0.0075 N-s (magnitude)
Problem 2.190 A 3000 kg space probe is moving in a gravity-free space at a constant velocity
of 300 m/s. To change the direction of space probe, rockets have been fired in a direction
perpendicular to the direction of initial motion of the space probe, the rocket firing exerts
a thrust of 4000 N for 225 s. The space probe will turn by an angle of (neglect the mass of
the rockets fired)
(a) 30° (b) 60°
(c) 45° (d) 37°
Solution From impulse-momentum theorem, r
      Pi
J = p f - pi fi p f = J + pi
Here pi = 3000 ¥ 300 = 9 ¥ 105 kg m/s
r
J = 4000 ¥ 225 = 9 ¥ 105 kg m/s (Impulse method) J r
Pf
p q
tan q = i = 1 fi q = 45° a
J r
Pi
So, the angle by which the space probe rotates is
Fig. 2.270
p
a = - q = 45°
2
ŽůůŝƐŝŽŶƐപ287

Problem 2.191 In Fig. 2.271, there are n identicals spheres of mass


m suspended with wires of equal length. The spheres are almost
in contact with each other. Sphere 1 is pulled aside and released.
If sphere 1 strikes sphere 2 with velocity u, find the expression for
velocity vn of the nth sphere immediately after being struck by the
one adjacent to it. The coefficient of restitution for all the impacts is e.
r
u r
Solution Let velocities of sphere 1 and 2 after collision be v1 and vn
v2, respectively, then
Fig. 2.271
v2 - v1
= e and mv2 + mv1 = mu
u
(1 + e )u
From the above equations v2 =
2

Now sphere 2 will collide with sphere 3 and after collision velocity of 3 can be formed as
2
v3 = Ê 1 + e ˆ u
Á Ë 2 ¯˜

Hence, similarly velocity of nth sphere can be formed as


n -1
Ê 1+ eˆ
vn = Á u
Ë 2 ˜¯

Problem 2.192 A ball of mass 100 g is projected vertically upwards from the ground with a
velocity of 49 m/s. At the same time, another identical ball is dropped from a height of 98
m to fall freely along the same path as that followed by the first ball. After some time, the
two balls collide and stick together and finally fall to the ground. Find the time of flight of
the masses. (IIT-JEE, 1985)

Solution Let the balls collide at a height h after time t1, then for ball 100g
1: u2 = 0

1 2 98–h
v
h = 49t1 - gt1 …(i) v1
2 v2
t1
1 2
For ball 2: 98 – h = gt1 …(ii) h
2
100g u1 = 49 m/s
98 = 49 t1 fi t1 = 2 s
Put the value of t1 in Eq. (i) to get h = 78.4 m Fig. 2.272
Velocity of ball 1 after time t1: v1 = 49 – 9.8 ¥ 2 = 29.4 m/s
Velocity of ball 2 after time t1: v2 = gt1 = 9.8 ¥ 2 = 19.6 m/s
After collision, balls collide and stick together. Now onwards balls will move together.
Let v be their common velocity just after collision, then apply conservation of momentum,
288പWŚLJƐŝĐƐĨŽƌ//dͲ:͗DĞĐŚĂŶŝĐƐ//

(m + m)v = mv1 – mv2


v1 - v2 29.4 - 19.6
fi v = = = 4.9 m/s
2 2
Let the balls take time t2 after collision to come to the ground, then
1 2
– h = vt2 - gt2
2
– 78.4 = 4.9t2 – 4.9t22
fi t22 – t2 – 16 = 0
1 + 1 + 64
fi t2 = = 4.53 s
2
So the time of flight: T = t1 + t2 = 2 + 4.53 = 6.53 s
Problem 2.193 An object of mass 5 kg is projected with a velocity of 20 m/s at an angle of
60° to the horizontal. At the highest point of its path the projectile explodes and breaks up
into two fragments of masses 1 kg and 4 kg. The fragments separate horizontally after the
explosion releases internal energy such that the kinetic energy of the system at the highest
point is doubled. Calculate the separation between the two fragments when they reach the
ground. (IIT-JEE, 1990)

Solution

5 kg 4 kg
u cos 60° 1 kg
u = 20 m/s = 10 m/s v2 v1 t
H H

60° A P Q
s
Before explosion After explosion

Fig. 2.273

The maximum height reached by mass 5 kg:

u2 sin 2 60∞ 20 2 ¥ 3/4


H = = = 15 m
2g 2 ¥ 10

Velocity of 5 kg mass at the highest point before explosion is


1
u cos 60° = 20 ¥ = 10 m/s
2
Now here 5 kg explodes into two fragments of masses 4 kg and 1 kg. Let their velocities
be v1 and v2 as shown, v1 and v2 will be horizontal because it is given that masses separate
horizontally. Apply conservation of momentum at the highest point:
ŽůůŝƐŝŽŶƐപ289

4v1 + 1 ¥ v2 = 5 ¥ 10 …(i)
It is given that KE of the system doubles due to explosion, so
1 2 1 2 È1 ˘
4v1 + 1v2 = 2 Í ¥ 5 ¥ 10 2 ˙
2 2 Î2 ˚
fi 4v12 + v22 = 1000 …(ii)
Put the value of v2 from Eq. (i) into Eq. (ii), 4v12 + (50 – 4v1)2 = 1000
fi v12 – 20v1 + 75 = 0 fi (v1 – 15)(v1 – 5) = 0
First solution: v1 = 15 m/s, v2 = – 10 m/s
Second solution: v1 = 5 m/s, v2 = 30 m/s
The first solution is possible if 4 kg is ahead of 1 kg and second solution is possible if 1 kg
is ahead of 4 kg. Since initial velocities of masses after explosion are in horizontal direction,
so they will take same time to return to the ground:

2H 2 ¥ 15
t= = = 3s
g 10
Now AQ = v1t, AP = v2t
Separation between them on reaching the ground:
S = AQ – AP = (v1 – v2)t = [15 – (– 10)] 3 = 25 3 m
For the second solution:
S = (5 - 30) 3 = - 25 3 m
–ve sign indicates that 1 kg will be ahead of 4 kg.
J
Problem 2.194 Three particles A, B and C respective masses m1
m1, m2 and m3 lie on a smooth horizontal surface, and are A
fastened to two light inextensible strings as shown in Fig.

2.274. The particle A is imparted an impulse J along BA . Find
45° m2 m3
the initial speed of each particle.
B C
Solution (Impulsed method) J – J 1 = m 1v 1 …(i)
Fig. 2.274
J1 – J2 (cos 45°) = m2v2y …(ii)
J
J2 sin 45° = m2v2x …(iii) v1
m1
J2 = m 3v 3 …(iv) J1
v 2x
Now, v2y = v1 and v2y cos 45° – v2x cos 45° = v3 J1
v3
v 2y
45° J2 J2
2m2 J m2 m2
Solve to get: v3 =
m1m3 + 2m2 (m1 + m2 + m3 )

Fig. 2.275
290പWŚLJƐŝĐƐĨŽƌ//dͲ:͗DĞĐŚĂŶŝĐƐ//

(m3 + 2m2 ) J
v1 =
m1m3 + 2m2 (m1 + m2 + m3 )

m3v3 cos 45∞ m3 J


v2x = =
m2 m1m3 + 2m2 (m1 + m2 + m3 )

Net velocity of m2.

Problem 2.195 A smooth ball is dropped from a height h on a smooth


incline, as shown in Fig. 2.276. After collision the velocity of the ball h
is directed horizontally. v

(a) Find the coefficient of restitution.


(b) If the collision is elastic, what is the impulse on the ball? q

%QPEGRV Fig. 2.276


u
n-axis

h

P
v
t-axis
q
q

n-axis
v sin q

v
q
q
v cos q î
u cos q u u sin q
q
t-a
xis

Fig. 2.277

Solution (a) Normal axis and tangential axis are shown in the Fig. 2.277. Reaction of the
incline is along n-axis and in the absence of friction there is no force along t-axis; therefore
velocity along t-axis remains unchanged,
i.e., v cos q = u sin q ...(1)
From the definition of coefficient of restitution,
ŽůůŝƐŝŽŶƐപ291

v sin q
e =– ...(2)
- u cos q
or v = eu cot q ...(3)
From Eqs. (1) and (3),
(eu cot q) cos q = u sin q
or e = tan2 q
(b) When the collision is elastic, the component of velocity along n-axis is reversed in
direction. Therefore, the change in velocity
Dv = 2v cos q n-axis
There is no change in velocity along the t-axis, therefore no q
impulse along t-axis
q
Change in momentum of the ball
v
= m Dv
= 2mv cos q
u q
Velocity of the ball when it strikes the plane = 2gh
Thus, impulse = 2m 2 gh cos q. Fig. 2.278

Problem 2.196 An inelastic ball is projected with velocity v0 = gh , at an angle a to the


horizontal, towards a wall distant d from the point of projection. After collision the ball
returns to the point of projection. After collision the ball returns to the point of projection.
What is the coefficient of restitution?
Solution Time taken to reach the wall vy
d
= v0 cos a = vx
v0 cosa
v0 Velocity before impact
Time taken to return to the point of projection after vy

d a ev0 cos a = vx
the impact =
ev0 cos a O d Velocity after impact

Note that x-component of velocity after impact is ev0 Fig. 2.279


cos a.
d d
Total time of flight = +
v0 cos a ev0 cos a

d È1 + e ˘
=
gh cos a ÍÎ e ˙˚

There is no change in the vertical component of the velocity after impact, therefore total
time of flight remains unchanged.
292പWŚLJƐŝĐƐĨŽƌ//dͲ:͗DĞĐŚĂŶŝĐƐ//

2v0 sin a 2 gh sin a


T = =
g g

d Ê 1+ eˆ 2 gh sin a
ÁË ˜¯ =
gh cos a e g

Ê 1+ eˆ h sin 2a
or ÁË ˜¯ =
e d
d
or e =
( h sin 2a - d)

Problem 2.197 A tennis ball with (small) mass m2 rests on the top of a basketball of mass
m1 which is at a height h above the ground, and the bottom of the tennis ball is at the height
h + d above the ground. The balls are dropped. To what height does the tennis ball bounce
with respect to ground? (Assume all collisions to be elastic and m1 >> m2).

Tennis ball

Basket ball

Fig. 2.280

Solution Just before collision, speed of any ball, v = 2gh m2 v m2 v¢


Just after collision m1 moves up with v while m2 moves down with v.
m1 v m1 v
v¢ - v
After their collision, e = =1 fi v¢ = 3v
v - ( - v) Fig. 2.281
2 2
0 = (3v) – 2gh¢

9v 2 dv 2
fi h¢ = fi H = d+ = d + 9h
2g 2g

Problem 2.198 Two identical balls A and B lie on a smooth C


horizontal surface, which gradually merges into a curve to a
height 3.2 m. Ball A is given a velocity of 10 m/s, to collide
head-on with ball B, which then takes up the curved path. What 3.2 m
is the minimum coefficient of restitution, e, for the collision A B
between A and B so that ball B reaches the highest point C of
the curve. Fig. 2.282
ŽůůŝƐŝŽŶƐപ293

Solution Velocity of B after collision:


(1 + e )m1u1 (1 + e )m ¥ 10
v2 = = fi v2 = 5 (1 + e)
m1 + m2 m+m
v22
Height raised by B =
2g

v22 (5(1 + e ))2 3


Now, ≥ 3.2 fi ≥ 3.2 fi e ≥
2g 2 ¥ 10 5

Problem 2.199 A ball of mass m moving with constant horizontal m


velocity u strikes a stationary wedge of mass M on its inclined
surface as shown in Fig. 2.283. After collision, the ball starts moving M
up the inclined plane. The wedge is placed on frictionless horizontal a
surface.
(i) Calculate the velocity of the wedge immediately after collision. Fig. 2.283
(ii) Calculate the maximum height the ball can ascend on the
wedge.
Solution
(i) Let after collision velocity of wedge be V and that
w.r.t. wedge CT
of ball be v0 as shown. v0 is w.r.t. the wedge. Apply
conservation of linear momentum along x-axis, we v0
get a
V
mu = Mv + m(V + v0 cos a) ...(i) m u
M
Velocity along CT remains, constant a
u cos a = v0 + V cos a ... (ii) CN
Net velocity of ball just after collision is

v b = (v0 cos a + V) i + (v0 sin a) j Fig. 2.284

From Eqs. (i) and (ii),


mu = MV + mV + m cos a (u cos a – V cos a)
= (M + m)V + mu cos2 a – mV cos2 a
mu sin 2 a
fi V =
M + m sin 2 a
(ii) When the ball ascends the maximum height, then the ball will be at rest w.r.t. the wedge
and their velocity will be same. Let their common velocity be v¢. From conservation of
momentum we can find this common velocity as v¢ mu/(M + m).
From Eq. (ii), v0 = (u – V) cos a, put the value of V in this to get
Mu cos a
v0 =
M + m sin 2 a
294പWŚLJƐŝĐƐĨŽƌ//dͲ:͗DĞĐŚĂŶŝĐƐ//

È M cos 2 a + m sin 2 a ˘
Also, v0 cos a + V = Í ˙u
ÍÎ M + m sin 2 a ˙˚
To find maximum height:
From energy conservation:
1 1 1
mgh + (m + M )v ¢ 2 = MV 2 + m(v0 cos a + V )2 + m(v0 sin a )2
2 2 2
m2u2
2mgh + = MV2 + m(v0 cos a + V)2 + m(v0 sin a)2
M+m
m2u2 Mm2u2 sin 4 a + m( M cos 2 a + m sin 2 a )2 u2 + mM 2u2 cos 2 a sin 2 a
2mgh + =
M+m ( M + m sin 2 a )2

u2 È M 2 cos 2 a ˘
Solve to get: h = Í 2 ˙.
2 g Î ( M + m)( M + m sin a ) ˚
m
Problem 2.200 A ball of mass m collides with a stationary wedge
of mass M, perpendicular to its inclined face, inclined at an angle
as shown in Fig. 2.286. If the coefficient of restitution between the M
wedge and ball is e, calculate the ratio of modulus of velocity of a
the ball immediately after and before collision. Also calculate the
velocity of the wedge just after collision. Fig. 2.285

Solution Let velocity of the wedge after collision be v2 and that


v1
of the ball be v1. We can show that v1 < u, because otherwise final
KE would be greater than initial KE which is not possible. v1 will M
u
be perpendicular to wedge surface. v2
a a
From Newton’s experimental law:
v2 sin a - (- v1 ) eu - v1
e = fi v2 = ...(i) Fig. 2.286
u sin a
Conservation of linear momentum along x-axis
mu sin a = mv1 sin a + Mv2 …(ii)
2
v1 Me - m sin a
From Eqs. (i) and (ii), =
u M + m sin 2 a
Put the value of v1 in Eq. (i) to get v2

1 È ( Me - m sin 2 a )u ˘ mu sin a (e + 1)
v2 = Íeu - ˙=
sin a Î M + m sin 2 a ˚ M + m sin 2 a

Problem 2.201 A ball of mass m hits a wedge of mass M vertically with speed u, which is
placed, on a smooth horizontal surface. Find the maximum compression in the spring, if the
collision is perfectly elastic and no friction anywhere. Spring constant of spring is K.
ŽůůŝƐŝŽŶƐപ295

u
M
45°

Fig. 2.287

Solution Let after collision, the ball has velocity v1 and wedge
m
v2 as shown in Fig. 2.288. u
Apply conservation of momentum in horizontal direction: 45° M
q v2
mv1 cos q = Mv2 ...(i) v1 common
45° normal

From Newton’s experimental law:


Fig. 2.288
v2 sin 45∞ - [-v1 cos( 45 + q )]
e =1=
u cos 45∞
fi u = v2 + v1 cos q – v1 sin q ...(ii)
Velocity component of the ball along the surface of wedge will remain the same as during
collision there is no force on the ball in this direction. So
u sin 45° = v1 cos (45 – q) fi u = v1 cos q + v1 sin q ...(iii)
From Eqs. (i), (ii) and (iii)
2mu
v2 =
m + 2M
To find maximum compression in the spring, after collision apply conservation of energy
for the system of wedge and spring:

1 1 M Ê 2mu ˆ M
Mv22 = Kx 2 fi x= v2 fi x = Á .
2 2 K Ë m + 2 M ˜¯ K

Problem 2.202 A small steel ball A is suspended by an inextensible thread of O


length l = 1.5 m from O. Another identical ball is thrown vertically downwards
such that its surface remains just in contact with the thread during downward B
motion and collides elastically with the suspended ball, if the suspended ball
just completes vertical circle.

Solution (Impulse Method) v2 = 5 gl = 5 ¥ 10 ¥ 1.5 = 5 3 m/s


A
For A: J sin q = mv2 = 5 3 m
1 Fig. 2.289
fi J¥ = 5 3 m fi J = 10 3 m
2
296പWŚLJƐŝĐƐĨŽƌ//dͲ:͗DĞĐŚĂŶŝĐƐ//

v1
q
B J
q v2
B
J A
u A u sin q

Fig. 2.290

For B: J = m(v1 + u cos q)

Ê u 3ˆ
fi 10 3m = m Á v1 +
Ë 2 ˜¯

fi 20 3 = 2v1 + u 3 ...(i)
By Newton’s experimental law:
v2 sin q + v1 u 3 v2
e =1= fi = + v1
u cos q 2 2
fi u 3 = 5 3 + 2v1 ...(ii)
From Eq. (i) and (ii), u = 12.5 m/s = 1250 cm/s.
Problem 2.203 A particle of mass 1 kg is attached to a string 3m
O M
of length 5 m. The string is attached to a fixed point O. It is
released from the position as shown in Fig. 2.291. Calculate: P

(i) the impulse developed in the string when it becomes taut; Q


(ii) the velocity of the particle just after the string becomes taut;
and
(iii) the impulse developed in this string PQ at this instant.
Fig. 2.291
Solution (Impulse method) From M to N the ball will fall
a distance 4 m, then impulse will be developed in the 3m
string ON. O M
Velocity of the ball just before the string becomes taut J
q 4m
q
u0 = 2 ¥ 10 ¥ 4 = 80 m/s J¢ J
N
Velocity of the ball in the direction perpendicular to the
v u0
impulse will remain same (just after and just before the
impulse developed).
3 12
v = u0 sin q = 80 ¥ = m/s J¢¢
5 5
Fig. 2.292
4 16
J = mu0 cos q = 1 80 ¥ = kg m/s
5 5
48
J¢ = J sin q = kg m/s
5 5
and impulse given by earth,
64
J≤ = J cos q = kg m/s
5 5
ŽůůŝƐŝŽŶƐപ297

Problem 2.204 Two small discs, A of mass 2m and B of mass m,


are connected by a light, inextensible string of length 13l, placed
on a smooth horizontal plane, separated by a distance of 5l. Disc
B is given a velocity 26v in a direction normal to AB. Find the A B
velocity of A when string just becomes taut. (2m) (m)
5l (26v)
Solution Apply conservation of momentum along AC:
Fig. 2.293
13l

2m m
5l
A B
v1
26v

12l
13l
q

m C
q v1
26v sin q

Fig. 2.294

m 26 v cos q = (2m + m)v1


v1 = 8v
Problem 2.205 A smooth ball of mass m is suspended from a light
string of length 1 m. Another ball of mass 2 m strikes the ball of
q = 30°
mass m horizontally with a speed of u = 35 m/s. The coefficient O
of restitution for the collision is e. The string becomes lose, when it
makes an angle of 30° with the horizontal; find the value of e. 1m
(BITSAT, 2014) 2m u m
Solution v1 and v2 are after the collision.
Fig. 2.295
(1 + e )2mu 2u(1 + e )
v2 = = v
2m + m 3
B
Since tension becomes zero at B, so
mv 2 gr 30° mg
mg sin 30° = fiv=
r 2
Apply conservation of energy between A and B
1 1 2m
mv 2 + mg(r + r sin 30∞) = mv22 m
2 2 v2
A
v1
fi e = 0.5
Fig. 2.296
298പWŚLJƐŝĐƐĨŽƌ//dͲ:͗DĞĐŚĂŶŝĐƐ//

Problem 2.206 After a totally inelastic collision, two objects of the same mass and same
initial speeds are found to move together at half of their initial speeds. The angle between
the initial velocities of the objects is (BITSAT, 2009)
(a) 120° (b) 60°
(c) 150° (d) 45°
Solution Let angle between initial velocities be p – q v/2
Y
and the situation is as shown in Fig. 2.297. Calculate
the momentum along X- and Y-directions. a
X
v m v 2m q v
mv – mv cos q = 2m cos a, for x-axis
2 q m
v
mc sin q = 2m sin a , for y-axis
2 Fig. 2.297
Solving above equations, q = 60°
Required angle = p – 60° = 120°
\ (a) is correct.
Problem 2.207 A ball of mass m is distributed from the top of a fixed smooth circular tube
in a vertical plane and falls impinging on a ball of mass 2 m at the bottom. The coefficient
of restitution is 1/2. Find the heights to which the balls rise after the second impact.
Solution The situation is shown in Fig. 2.298. If u is the velocity of ball A just before the
impact. Then m
u = 2 g ( 2 a)
Consider the first impact between A and B. If velocities of the balls
become v1 and v2, respectively according to linear momentum conservation
mu + 0 = mv1 + 2mv2 ...(i) m 2m
B
Coefficient of restitution is given as A v2
v1
v2 - v1 1
e = = ...(ii)
u 2 Fig. 2.298
On solving Eqs. (i) and (ii), we get
u
v1 = 0 and v2 =
2
Ball A comes to rest, so the second impact between A and B occurs, when ball B returns
to its initial position with the same speed u/2. If after the second impact, the velocities of
the two balls become v3 and v4, again from linear momentum conservation and coefficient
of restitution.
Ê uˆ
2m Á ˜ + 0 = mv3 + 2mv4 …(iii)
Ë 2¯
v - v4 1
and e = 3 = …(iv)
u 2
2
On solving Eqs. (iii) and (iv), we get
u u
v3 = and v4 =
2 4
ŽůůŝƐŝŽŶƐപ299

Both the velocities are positive, it implies that both masses will move in the same direction
after the second impact. If h1 and h2 are the heights to which the masses m and 2m will rise
after the second impact, according to energy conservation, we have
For ball of mass m, 2
1 Ê uˆ u2 a
m Á ˜ = mgh1 or h1 = =
2 Ë 2¯ 8g 2
For ball of mass 2m,
2
1 Ê uˆ u2 a
(2m) Á ˜ = 2mgh2 or h2 = =
2 Ë 4¯ 8g 8

Problem 2.208 A cylindrical solid of mass 10–2 kg and cross-sectional area 10–4 m2 is moving
parallel to its axis (x-axis) with a uniform speed of 103 m/s in the positive direction. At t
= 0 its front face passes the plane x = 0. The region to the right of this plane is filled with
stationary dust particles of uniform density 10–3 kg/m3. When a dust particle collides with
the face of the cylinder, it sticks to its surface. Assuming that the dimensions of the cylinder
remain practically unchanged and that the dust sticks only to the front face of the cylinder,
find the x-co-ordinate of the front of the cylinder at t = 150 s.
Solution Considering cylinder and dust particles as
the system let AB be the front face of cylinder at t = 0 C C¢ A A¢
(and x = 0). So, the initial momentum of the system Pi
= MV0 + 0 (as dust particles are at rest). Now in time
x-axis
t the front face has shifted a distance x so that its new
D D¢ B B¢
position is A¢ B¢ and so, the mass of the particles stick
x=0
to the cylinder will be Axr and if v is the velocity of t=0
cylinder at this instant
Fig. 2.299
Pf = (M + Arx)v + 0
Now as in this process no external force is acting on the system, the linear momentum of
the system must be conserved, i.e.,
MV0 = (M + Arx)v
Ê dx ˆ È dx ˘
i.e., MV0 = ( M + r Ax )v Á ˜ ÍÎas v = dt ˙˚
Ë dt ¯
150 x
or MV0 Ú dt = Ú0 ( M + r Ax)dx
0

or 150 MV0 = Mx + (1/2)rAx2


Now substituting the given data in this equation we have
1
150 ¥ 10–2 ¥ 103 ª 10 - 2 x + ¥ 10 - 3 ¥ 10 - 4 x 2
2
or x2 + 2 ¥ 105 x – 3 ¥ 1010 = 0
[- 2 ¥ 10 5 ± (2 ¥ 10 5 )2 - 4 ¥ 1 ¥ (- 3 ¥ 1010 )]
or x =
2
Here negative sign is inadmissible as x cannot be negative
So, x ª (1/2)[– 2 ¥ 105 + 4 ¥ 105] ª 1 ¥ 105 m
300പWŚLJƐŝĐƐĨŽƌ//dͲ:͗DĞĐŚĂŶŝĐƐ//

DAILY PRACTICE PROBLEMS FOR JEE MAIN AND ADVANCE

‡˜‡Žͳ౨ 1PN[QPGQRVKQPKUEQTTGEV (a) (i) (b) (ii)


1. Consider the following statements (i) (c) (iii) (d) all
and (ii) and identify the correct answer: 4. Consider the following statement:
(i) Coefficient of restitution varies When jumping from some height, you
between 0 and 1 should bend your knees as you come
(ii) In inelastic collision, the law of to rest, instead of keeping your legs
conservation of energy is satisfied stiff. Which of the following relations
(a) (i) and (ii) are true can be useful in explaining the state-
(b) (i) and (ii) are false ment:
(c) (i) is true but (ii) is false where symbols have their usual mean-
(d) (i) is false but (ii) is true ing.  
2. Four identical balls are in contact and (a) DP1 = - DP2
at rest on a horizontal smooth surface (b) DE = – D (PE + KE) = 0
 
are hit by another identical ball moving (c) FDt = mDV
initially as shown in the figure. Ball 1  
(d) DX μ DF
comes to rest after collision. Which of
5. The centre of mass is defined as R =
the following can possibly represent
1 n
the motion of balls after collision  Miri. Suppose we define “centre
M i =1
1 n
of charge” as Rc = Âqi ri where qi
Q i =1
represents the ith charge placed at ri
1 (i) (ii) and Q is the total charge of the system.
The centre of charge of two-charge
system:
(a) centre of charge always lies be-
tween the charges
(iii) (iv) (b) centre of charge always out of the
charges
(a) (i) (b) (ii)
(c) centre of charge can lie out of
(c) (iii) (d) all
charges
3. Choose the correct statement(s) from
(d) none of the above.
the following
6. I. Two particles be in equilibrium
(i) In an elastic collision of two
under the action of their mutual
bodies, the momentum and
gravitation force.
energy of each body is conserved.
II. Three particles be in equilibrium
(ii) The work done by a force on a
under the action of their mutual
body in nature, over a closed loop
gravitational force.
is always zero.
III. One of the particles be in
(iii) In an inelastic collision of two
equilibrium in three-particle
bodies, the final kinetic energy is
system.
less than the initial kinetic energy
(a) I is correct (b) II is correct
of the system.
(c) III is correct (d) II and III
are correct
ŽůůŝƐŝŽŶƐപ301

7. Consider the following two statements: (a) 2 (b) 4


(A) Linear momentum of a system (c) 6 (d) 8
of particles is zero 12. A body of mass 1 kg initially at
(B) Kinetic energy of a system of rest, explodes and breaks into three
particles is zero, then fragments of masses in the ratio 1:1:3.
(a) A implies B and B implies A The two pieces of equal mass fly off
(b) A does not imply B and B does not perpendicular to each other with a
imply A speed of 15 m/s each. The speed of the
(c) A implies B but B does not imply heavier fragment is (EAMCET, 2010)
A (a) 5 2 m/s (b) 45 m/s
(d) A does not imply B but B implies (c) 5 m/s (d) 15 m/s
A 13. A ball of mass 0.2 kg is thrown
8. Two bodies A and B have masses M vertically upwards by applying a force
and m respectively, where M > m and and the ball goes up to 2 m if the hand
they are at a distance d apart. Equal moves 0.2 m which applying the force
force is applied to them so that they height further, find the magnitude of
approach each other. The position the force (take g = 10 m/s2)
where they hit each other is (a) 22 N (b) 4 N
(a) nearer to B (c) 16 N (d) 20 N
(b) nearer to A 14. A player caught a cricket ball of mass
(c) at equal distance from A and B 150 g moving at a rate of 20 m/s. If the
(d) cannot be decided catching process is completed in 0.1 s,
9. A ball is dropped from height 10 m. the force of the blow exerted by the
Ball is embedded in sand 1 m and ball on the hand of the player is equal
stops, then (BITSAT, 2011) to (MHCET, 2004)
(a) only momentum remains con- (a) 300 N (b) 150 N
served (c) 3 N (d) 30 N
(b) only kinetic energy remains con- 15. Ball A of mass 1 kg, moving with
served speed of 12 m/s, collides obliquely
(c) both momentum and KE are con- with another Ball B which was initially
served at rest. Ball A then moves off at right
(d) neither KE nor momentum is angles to its direction with a speed of
conserved 5 m/s. The momentum of Ball B after
10. Which of the following is not a perfect- collision is
ly inelastic collision? (MHCET, 2007) (a) 5 kg m/s (b) 11 kg m/s
(a) Striking of two glass balls (c) 13 kg m/s (d) 17 kg m/s
(b) A bullet striking a bag of sand 16. A ball is dropped from a height of 1 m,
(c) An electron captured by a proton if the coefficient of restitution between
(d) A man jumping onto a moving the surface and the ball is 0.6. The ball
cart rebounds to a height of (VITEEE, 2007)
11. Two particles having position vectors (a) 0.6 m (b) 0.4 m
     
r1 = (3i + 5j) metre and r2 = (- 5i - 3j) (c) 1 m (d) 0.36 m
metre are moving with velocities 17. A particle of mass 4 m which is at rest
     
v1 = ( 4i + 3j) m/s and v 2 = ( ai + 7 j) explodes into three fragments. Two of
m/s. If they collide after 2 seconds, fragments, each of mass m are found to
the value of a move with a speed v each in mutually
302പWŚLJƐŝĐƐĨŽƌ//dͲ:͗DĞĐŚĂŶŝĐƐ//

perpendicular directions. The total U U


energy released in the process is
(c) (d)
1 r r
(a) mv2 (b) mv2 2R 2R
2
3 5 22. A neutron of mass m collides elastically
(c) mv2 (d) mv2 with a uncleus of mass M, which is
2 2
initially at rest. If the Neutron’s initial
18. A child is sitting at one end of a long kinetic energy is k0, the maximum
trolley moving with a uniform speed kinetic energy that it can lose during
v on a smooth horizontal track. If the the collision is
child starts running towards the other k
(a) k0 (b) 0
end of the trolley with a speed u, the 2
speed of the centre of mass of the
m M k0 4 m M k0
system will be (c) (d)
(a) u + v (b) v – u ( M + m)2 ( M + m)2
(c) v (d) none 23. Three forces start acting simultaneously
19. A bomb of mass 12 kg explodes into on a particle moving with velocity
two pieces of masses 4 kg and 8 kg. v. These forces are represented in
The velocity of 8 kg mass is 6 m/s. The magnitude and direction by the three
kinetic energy of the other mass is sides of a triangle ABC (as shown). The
(a) 48 J (b) 32 J particle will now move with velocity:
(c) 24 J (d) 288 J
20. A massive ball moving with speed C
v collides head-on with a tiny ball
having mass much smaller than the
mass of the first ball. If the collision
is completely elastic, then the speed
of the second ball after the collision
A B
approximately equal to (BITSAT, 2003)
(a) v (b) 2 v (a) v1, remaining unchanged
(b) less than v
v (c) greater than v
(c) (d) •
2 (d) |v| in the direction of the largest
21. Which of the following potential force BC
energy curve can describe the elastic 24. A smooth sphere of mass M moving
collision of two billiards balls? Here r with velocity u directly collides elasti-
is the distance between centres of balls. cally with another sphere of mass m at
(VITEEE, 2014) rest. After collision their final veloci-
ties are V and v respectively. The value
U U of v is

(a) (b) 2uM 2um


(a) (b)
m M
r r
2R 2R
ŽůůŝƐŝŽŶƐപ303

2u 2u 28. Two blocks of masses 10 kg and 4 kg


(c) (d) are connected by a spring of negligible
m M
1+ 1+ mass and are placed on a frictionless
M m
horizontal surface. An impulse gives a
25. A 238U nucleus decays by emitting an speed of 14 m/s to the heavier block in
alpha particle of speed v ms–1. The the direction of the lighter block. Then,
recoil speed of the residual nucleus is the velocity of the centre of mass is
(in ms–1) (a) 30 ms–1 (b) 20 ms–1
–1
(a) – 4v/234 (b) v/4 (c) 10 ms (d) 5 ms–1
(c) – 4v/238 (d) 4v/238 29. A body A of mass M while falling
26. A particle of mass m moves on the vertically downward under gravity
x-axis under the influence of a force of breaks into two parts; a body B of
attraction towards the origin O given M 2M¢
mass and a body C of mass .
 Ê 1ˆ 3 3
by F = - k Á 2 ˜ i . If the particle starts
Ëx ¯ The centre of mass of bodies B and C
from rest at x = a, the speed it will taken together shifts compared to that
attain to reach at a distance x from the of body A towards
origin will be (a) body C
1/2
(b) body B
2k Ê a - x ˆ (c) depends on height of breaking
(a) Á ˜
m Ë ax ¯ (d) does not shift
30. The mass per unit length of a non-
1/2
2k Ê a + x ˆ uniform rod of length L varies as m =
(b) Á ˜
m Ë ax ¯ lx where l is constant. The centre of
mass of the rod will be at
k Ê ax ˆ 2 3
(c) Á ˜ (a) L (b) L
m Ë a - x¯ 3 2
1 4
m Ê a - xˆ
1/2 (c) L (d) L
(d) 2 3
Á ˜
2k Ë ax ¯
31. Three identical spheres each of radius
27. Consider a two-particle system with R are placed touching each other on
particles having masses m1 and m2. If a horizontal table as shown in figure.
the first particle is pushed towards the The co-ordinates of centre of mass are
centre of mass through a distance d, y
by what distance should the second
particle be moved, so as to keep the
centre of mass at the same position? x
m2
(a) d (b) d
m1 (a) (R, R) (b) (0, 0)

(c)
m1
d (d)
m1
d Ê R Rˆ Ê Rˆ
(c) Á , ˜ (d) Á R, ˜
m1 + m2 m2 Ë 2 2¯ Ë 3¯
304പWŚLJƐŝĐƐĨŽƌ//dͲ:͗DĞĐŚĂŶŝĐƐ//

(a) - 2i + 2k  (b) - 2i - j + k


32. Two bodies of masses 1 kg and 2 kg
are moving in two perpendicular
directions with velocities 1 m/s and 2

(c) 2i - j + k 
(d) - i + j + k
m/s as shown in figure. The velocity 36. A cricket bat is cut at the location of its
of the centre of mass (in magnitude) of centre of mass as shown. Then
the system will be

1 kg 1 m/s
(a) the bottom piece will have the
2 m/s same mass
(b) the bottom piece will have larger
2 kg
mass
(c) the handle piece will have larger
(a) 3 m/s (b) 1.67 m/s mass
(c) 1.5 m/s (d) 1.37 m/s (d) mass of handle piece is double the
33. Two particles A and B initially at mass of bottom piece
rest. Move towards each other under 37. Two astronauts each of mass 150 kg are
a mutual force of attraction. At the travelling in a closed skylab moving at
instant, when the speed of A is v and a speed of 5 km/s in the outer space
the speed of B is 2 v, the velocity of far removed from all other material
centre of the system is objects. The total mass of the skylab is
(a) 0 (b) v 800 kg. If the astronauts do slimming
(c) 1.5 v (d) 3 v exercise and thereby reduce their
34. Body A is released from a condition of masses to 100 kg each, the velocity
rest on a frictionless circular surface. It now with which the skylab is moving.
then moves on a horizontal surface BD (a) 5.5 km/s (b) 11 km/s
whose coefficient of friction with the (c) 5 km/s (d) 3 km/s
body is 0.1. A spring having a spring 38. Three thin rods, each of length L, are
constant k = 2 ¥ 104 N/m is positioned arranged in an inverted U, as shown
at D as shown in the diagram. The mass in figure. The two rods on the arms of
of the body is 10 kg. The compression the U each have mass M; the third rod
of the spring approximately is (Take g has mass 3M. The centre of mass of the
= 10 m/s2) system from upper rod is
A 3M
10 m L

D B
10 m L L
M M

(a) 0.1 m (b) 0.2 m


(c) 0.3 m (d) 1 m
35. Two bodies of mass 1 kg and 3 kg
L L
have position vectors i + 2j + k
 and (a)
3
(b)
2
-3i - 2j + k
 respectively. The centre
L 2L
of mass of this system has a position (c) (d)
vector 5 3
ŽůůŝƐŝŽŶƐപ305

39. A body of mass 2 kg moving with a (a) 250 3 N to right


velocity of 3 m/s collides head on
(b) 250 N to right
with a body of mass 1 kg moving in
opposite direction with a velocity of 4 (c) 250 3 N to left
m/s. After collision, two bodies stick (d) 250 N to left
together and move with a common 43. A body falling from a height of 10 m
velocity which in m/s is equal to rebounds from hard floor. If it loses 20%
(a) 1/4 (b) 1/3 energy in the impact, then coefficient
(c) 2/3 (d) 3/4 restitution is (BCECE, 2003)
40. Two identical masses M moving with (a) 0.89 (b) 0.56
velocity u1 and u2 collide perfectly (c) 0.23 (d) 0.18
inelastically. The loss in energy is 44. A ball hits a vertical wall horizontally
(VITEEE, 2011) at 10 m/s bounces back at 10 m/s
M (a) there is no acceleration because 10
(a) (u2 - u1 )2 m/s – 10 m/s = 0
2
(b) there may be an acceleration
M because its initial direction is
(b) (u1 - u2 )2
2 horizontal
M (c) there is an acceleration because
(c) (u1 - u2 )2 there is a momentum change
4
(d) even though there is no change
M in momentum there is a change
(d) (u2 - u1 )2
8 in direction. Hence, it has an
41. A particle of mass m moving eastward acceleration
with a speed v collides with another 45. A ball is dropped from a height h. If
particle of the same mass moving the coefficient of restitution is e, then to
northwards with the same speed v. what height will it rise after jumping
The two particles coalesce on collision. twice from the ground (WBJEE, 2011)
The new particle of mass 2 m will move (a) eh/2 (b) 3eh
in the north-eastern direction with a (c) eh (d) e4 h
velocity 46. A particle of mass m moving with
(a) v/2 (b) 2v horizontal speed 6 m/sec as shown
in figure. If m << M then for one
(c) v/ 2 (d) v dimensional elastic collision, the speed
42. A mass of 100 g strikes the wall with of lighter particle after collision will be
speed 5 m/s at an angle as shown in
u1 = 6 m/s u2 = 4 m/s
figure and it rebounds with the same m M
speed. If the contact time is 2 ¥ 10–3 sec,
what is the force applied on the mass (a) 2 m/s in original direction
by the wall (b) 2 m/s opposite to the original
direction
(c) 2 m/s opposite to the original
60°
direction
(d) 4 m/s in original direction
60°
100 g
306പWŚLJƐŝĐƐĨŽƌ//dͲ:͗DĞĐŚĂŶŝĐƐ//

47. In an elastic collision between spheres (a) g


A and B of equal mass but unequal g
radii, A moves along the x-axis and B (b)
2
is stationary before impact. Which of 3m
the following is possible after impact? g
(c) m
(a) A comes to rest 4
(b) The velocity of B relative to A (d) none of these
remains the same in magnitude 51. A trolley of mass 500 kg carrying a
but reverses in direction sand bag of 50 kg is moving uniformly
(c) A and B move with equal speeds, with a speed of 20 m/s on a frictionless
making an angle of 45° each with track. After a while, sand starts leaking
the x-axis out of a hole on the floor of the trolley
(d) A and B move with unequal at the rate of 0.05 kg/s. The speed of
speeds, making angles 30° and the trolley after the entire sand bag is
60° with the x-axis respectively. empty is
48. A mass m moves with a velocity v
and collides inelastically with another Sand 20 m/s
identical mass. After collision the 1st
v (a) 20 m/s (b) 22 m/s
mass moves with velocity in a
3 (c) 27 m/s (d) none of
direction perpendicular to the initial these
direction of motion. Find the speed of 52. If the masses of two balls be as 3 : 2
the 2nd mass after collision and their respective velocities before
(IIT-JEE, 2005) impact be as 2 : 3 and in opposite
2 v directions, and the coefficient of
(a) v
3 3 restitution is 2/3. With what velocity
v will each ball move back with respect
(b) v At rest to the original velocity?
3 m m (a) 1/3 times the original velocity
(c) v Before collision After collision
(b) 2/3 times the original velocity
(d) 3v (c) 3/1 times the original velocity
49. An object of mass 3 m splits into three (d) 3/2 times the original velocity
equal fragments. Two fragments have
velocities v j and v i . The velocity of the ‡˜‡Žʹ౨ 1PN[QPGQRVKQPKUEQTTGEV
third fragment is 1. A block of mass m is pushed towards
a movable wedge of mass nm and
(a) v (j - i) (b) v (i - j) height h with velocity u. All surfaces
are smooth. The minimum value of u
v(i + j)
(c) – v (i + j) (d) so that the block would reach the top
2 of the wedge is
50. Two masses 3 m and m are suspended
(a) 2 gh
from a light frictionless pulley with the
help of a massless string. If the system (b) n 2 gh
is set free the acceleration of centre of
mass will be
ŽůůŝƐŝŽŶƐപ307

stop system. If the spring is non-linear


Ê 1ˆ
(c) 2 gh Á 1 - ˜ such that it develops 300 x2 N force
Ë n¯
for a deflection of x m. The maximum
deceleration that the car A undergoes
Ê 1ˆ
(d) 2 gh Á 1 + ˜
Ë n¯ A k
2. In the following diagram, the small
prism of mass M slides down on the
bigger prism of mass 5 M from position (a) 1 m/s2 (b) 1.5 m/s2
2
shown at the top of the bigger prism to (c) 2 m/s (d) 2.5 m/s2
the position at the bottom of the bigger 5. A shell is fired from a cannon with
prism as shown in the figure. By what velocity v m/s, at an angle q with the
distance does the combination move horizontal direction. At the highest
to the left if the bigger prism initially point in its path it explodes into two
rests on a frictionless floor? pieces of equal mass. One of the pieces
retraces its path to the cannon. The
L
speed of the other piece immediately
M L after explosion is
5L L (a) 3v cos q
5M
M L (b) 2v cos q
5L 3
(c) v cos q
2
L 4L
(a) (b) 3
5 5 (d) v cos q
2
2L L
(c) (d) 6. The rear block moves with a speed of
3 6 2 m/s towards the front block kept at
3. Three identical bricks of length l are rest on a smooth surface. The spring
placed on the top of each other as is massless and has force constant 50
shown in figure, so that the part of N/m. The maximum compression of
each overhangs the one below. Find in the spring if each block is of 1 kg mass?
terms of l, the maximum value of the k
total overhang x 1 kg 1 kg

l
(a) 0.15 m (b) 0.26 m
(c) 0.30 m (d) 0.40 m
x 7. Two equal balls, in contact on a
table, are in equilibrium. A third ball
l l collides with them simultaneously
(a) (b)
4 2 symmetrically
3l 11l
(c) (d)
4 12
4. A car having a mass of 200 kg is rolling
at a speed of 1 m/s towards a spring- and remains at rest after impact. The
coefficient of restitution is
308പWŚLJƐŝĐƐĨŽƌ//dͲ:͗DĞĐŚĂŶŝĐƐ//

2 1 mu 2mu
(a) (b) (a) (b)
3 3 T T
1 3 4 mu 3mu
(c) (d) (c) (d)
4 5 3T 4T
8. A military tank whose mass together
with an artillery gun is M moves at a 11. Two small particles of equal masses
speed of v. The gun barrel makes an start moving in opposite directions
angle a with the horizontal. A shell of from a point A in a horizontal circular
mass m leaves the barrel at a speed v orbit. Their tangential velocities are v
relative to the barrel in the direction and 2v respectively, as shown in the
of the tank’s motion. The speed of the figure. Between collisions, the particles
tank in order that it may stop after the move with constant speeds. After
firing is making how many elastic collisions,
other than that at 4, these two particles
mv cos a mv
(a) (b) will again reach the point A?
M+m M+m (IIT-JEE, 2009)
mu cos a
(c) (M + m) v cos a (d) v A
M+m 2v
9. In a gamma decay process, the inter-
nal energy of a nucleus of mass M de-
creases, a gamma photon of energy E
and linear momentum E/c is emitted
(a) 4 (b) 3
and the nucleus recoils. The decrease
in internal energy is (VITEEE, 2011) (c) 2 (d) 1
E2 12. Three objects A, B and C are kept
(a) E (b) E + in a straight line on a frictionless
2 Mc 2
E2 horizontal surface. These have masses
(c) (d) zero
2 Mc 2 m, 2m and m, respectively. The object A
moves towards B with a speed 9 m/s
10. A particle of mass m moving with a
and makes an elastic collision with
velocity u makes an elastic one-di-
it. Thereafter, B makes completely
mensional collision with a stationary
inelastic collision with C. All motions
particle of mass m establishing a con-
occur on the same straight line. Find
tact with it for extremely small time T.
the final speed (in m/s) of the object C.
Their force of contact increases from
zero to F0 linearly in time T/4, remains
m 2m m
constant for a further time T/2 and
decreases linearly from F0 to zero in
(a) 3 m/s (b) 4 m/s
further time T/4 as shown. The mag-
nitude possessed by F0 is (c) 5 m/s (d) 1 m/s
13. A ball of mass 1 kg bounces against
F
the ground as shown in the figure. The
F¢ approaching velocity is 25 m/s and the
velocity after hitting the ground is 20
m/s. The impulse exerted on the ball
t
O T/4 3T/4 T is
ŽůůŝƐŝŽŶƐപ309

y
3L 4L
(a) (b)
1 kg
20 m/s
4 5
25 m/s
L L
45° 30° x (c) (d)
4 3
(a) 7.8 N-s (b) 27.68 N-s 18. A ball of mass M and radius R is
(c) 31 N-s (d) 62 N-s placed inside a spherical shell of same
mass M and the inner radius 2R. The
14. An intense stream of water of cross-
combination is at rest on a table top
sectional area A strikes a wall at an
in the position shown in figure. The
angle q with the normal to the wall and
ball is released, rolls back and forth
returns back elastically. If the density
inside and finally comes to rest at the
of water is r and its velocity is v, then
bottom of the shell. The maximum
the force exerted in the wall will be
displacement of the shell during this
(a) 2Av2r cos q (b) 2Av2r cos q process is
2
(c) 2Av r (d) 2Avr R
15. After a totally inelastic collision, two (a)
2
objects of the same mass and initial
(b) R 2R
speed are found to move away to-
gether at half of their initial speed. The 3
(c) R
angle between the initial velocities of 2
the object is (d) 2R
(a) 30° (b) 60° 19. A ball of mass 1 kg is attached to an
(c) 120° (d) 45° inextensible string. The ball x released
16. Two balls collide and bounce off each from the position shown in figure. The
other as shown in the figure. A 1 kg ball impulse imparted by the string to the
has a speed of 10 cm/s after collision. ball just after string becomes taut is
The velocity of the 0.5 kg ball will be (g = 10 m/s2).
30 cm/s
1 kg q 0.5 kg
20 cm/s 30°
1m
(a) 36 cm/s (b) 24 cm/s
(c) 12 cm/s (d) 18 cm/s
17. A man of mass M stands at one end of (a) 20 N-s (b) 40 N-s
a plank of length L which lies at rest on (c) 10 N-s (d) none of
a frictionless surface. The man walks these
to the other end of plank. If the mass 20. A block of mass m and a pan of equal
M mass are connected by a string going
of the plank is , the distance that the
3 over a smooth light pulley. Initially,
man moves relative to the ground is the system is at rest. A particle of mass
m falls on the pan and sticks to it. If
310പWŚLJƐŝĐƐĨŽƌ//dͲ:͗DĞĐŚĂŶŝĐƐ//

the speed of the particle just before the sm


strike is v, then the speed of the system (a) s (b)
M
after the collision is
sM Ê Ms ˆ
(c) (d) Á
m Ë m + M ˜¯

24. A trolley of mass 200 kg moves with


m a uniform speed of 10 m/s on a
m m v frictionless track. A child of mass 20 kg
runs on the trolley from one end to the
(a) v (b) v/2 other with a speed of 4 m/s relative to
(c) v/3 (d) v/4 the trolley in a direction opposite to its
21. Block A of mass m is hanging from motion, and jumps out of the trolley.
a vertical spring of force constant k. The final speed of the trolley is
Another identical block B strikes the
block A with velocity v and sticks to
it. The value of v for which the spring
just attains natural length is 10 m/s

6m 8m
(a) g (b) g
k k
(a) 10.4 m/s (b) 12.4 m/s
24m 12m (c) 16 m/s (d) none of
(c) g (d) g
k k these
22. A body of mass m1 + m2 is split into 25. Two particles, each of mass 2 kg are
two parts of masses m1 and m2 by an put at (2 m, 0) and (0, 2 m), as shown
internal explosion which generates a in figure. Now 1 kg mass of particle A
kinetic energy E. If after explosion the is put on to the particle B. The change
parts move in the same line as before, in x-coordinate of centre of mass of the
then their relative speed is system is
y
E(m1 + m2 )
(a)
m1 + m2 B (0, 2)

2E(m1 + m2 )
(b) A
m1m2 O
x
(2, 0)

E(m1 + m2 )
(c)
m1 + m2 (a) 0.5 m (b) 1 m
(c) 1.5 m (d) none of
E(m1 + m2 ) these
(d)
2m1 ¥ m2 26. From what minimum height h must the
system be released with unstretched
23. A shot of mass m penetrates a thickness sring of force contant k so that after
s of a fixed uniform wooden block of perfectly inelastic collision with the
mass M. If the block is free to move, ground, the sphere B my be lifted off
then the thickness penetrated will be the ground? (IIT-JEE, 2013)
ŽůůŝƐŝŽŶƐപ311

m the centre of mass of particles at the


original position

l0
m1 m1
k (a) d (b) d
m1 + m2 m2
m2
2m
B
(c) d (d) d
m1
h
31. Two spheres of masses 2M and M are
initially at rest at a distance R apart.
(a) mg/k (b) mg/2k Due to mutual force of attraction, they
(c) 3mg/k (d) 4mg/k approach each other. When they are at
27. Centre of mass of 3 particles 10 kg, separation R/2, the acceleration of the
20 kg and 30 kg is at (0, 0, 0). Where centre of mass of spheres would be
should a particle of mass 40 kg be (a) 0 m/s2 (b) g m/s2
placed so that the combination centre (c) 3g m/s2 (d) 12g m/s2
of mass will be at (3, 3, 3)
32. Look at the drawing given in the
(a) (0, 0, 0)
figure which has been drawn with ink
(b) (7.5, 7.5, 7.5)
of uniform line-thickness. The mass
(c) (1, 2, 3)
of ink used to draw each of the two
(d) (4, 4, 4)
inner circles, and each of the two line
28. An isolated particle of mass m is segments is m. The mass of the ink
moving in a horizontal plane xy along used to draw the outer circle is 6 m.
the x-axis at a certain height above The coordinates of the centres of the
the ground. It suddenly explodes into different parts are: outer circle (0, 0) left
m 3m inner circle (– a, a), right inner circle (a,
two fragments of masses and .
4 4 a) vertical line (0, 0) and horizontal line
An instant later, the smaller fragment (0, – a). They co-ordinate of the centre
is at y = 15 cm. The larger fragment at of mass of the ink in this drawing is
this instant is at (IIT-JEE, 2009)
y
(a) y = – 5 cm (b) y = + 20 cm
(c) y = + 5 cm (d) y = – 20 cm
x
29. If linear density of a rod of length 3 m
varies as l = 2 + x, then the position of
the centre of gravity of the rod is a a
(a) (b)
7 12 10 8
(a) m (b) m a a
3 7 (c) (d)
12 3
10 9
(c) m (d) m 33. Two spherical bodies of mass M and
7 7
5M and radi R and 2R respectively
30. Consider a particle of two particles
are released in free space with initial
having masses m1 and m2. If the
separation between their centres equal
particle of mass m1 is pushed towards
to 12R. If they attract each other due
the centre of mass of particles through
to gravitational force only, then the
a distance d by what distance would
distance covered by the smaller body
be particle of mass m2 move so as keep
just before collision is
312പWŚLJƐŝĐƐĨŽƌ//dͲ:͗DĞĐŚĂŶŝĐƐ//

(a) 1.5 R (b) 2.5 R (a) u (d) u/s


(c) 4.5 R (d) 7.5 R 7 3u
34. Figure shows a smooth spherical ball (c) u (d)
4 4
of mass m striking two identical equi-
lateral triangular wedges, each of mass 37. In the following figure, if the coeffi-
m. The velocity of ball at the instant cient of restitution is 0.8, what is the
of impact is v0. If e is the coefficient maximum angle from the vertical that
of restitution, then velocity of either body B will reach after the first impact
wedge after impact is

1m
60°
v0 A
m m 1 kg
B
5 kg
3 5
(a) (1 + e)v0 (b) (1 + e)v0 (a) 30° (b) cos–1 (0.45)
5 3 –1
(c) cos (0.9995) (d) none
ev0
(c) 3 (1 + e)v0 (d) 38. Two identical spheres A and B, lie on a
3
smooth, horizontal groove at opposite
35. A ball is projected from a point on a ends of a diameter. A is projected
smooth horizontal plane with a veloc- along the groove and after time t it
ity u at an angle q with the horizontal impinges upon B. If e is the coefficient
and continuous to rebound. If the coef- of restitution, then the second impact
ficient of restitution is e, then range af- will occur after a time
ter the first rebound is (WBJEE, 2016)
(a) 3 t/2 (b) 2 t/e
u2 sin 2q eu2 sin 2q (c) e t/2 (d) none of
(a) (b)
g g these
39. A ball is suspended from the top of a
eu2 sin 2q cart by a string of length 1.0 m. The
(c) (d) none of
2g these cart and the ball are initially moving
to the right at constant speed v, as
36. Two particles P and Q of equal mass
shown in the following figure (a).
m are attached by a string of length
The cart comes to rest after colliding
2 l and initially placed on a smooth
and sticking to a fixed bumper, as in
horizontal table in the position shown
figure (b). The suspended ball swings
in figure. Particle Q is projected across
through a maximum angle 60°. The
the table with speed u perpendicular
initial speed v is (Take g = 10 m/s2)
to the line joining them. The velocities
of particle Q after the string becomes v
taut is L

l
n Bumper
Q (a)
ŽůůŝƐŝŽŶƐപ313

(a) g sin q
60°
g sin q - m g cos q
(b)
3
g sin q
(c)
Bumper 3
(b) 2 g sin q - m g cos q
(d)
3
(a) 2 5 m/s (b) 5 2 m/s
42. Two particles of equal mass have ve-
(c) 10 m/s (d) 4 m/s  
locities v1 = 2i m/s and v 2 = 2j m/s.
40. Two small bodies of masses m and 2m First particle has an acceleration
are placed in a fixed smooth horizontal 
circular hollow tube of mean radius r a = (3i + 3j) m/s2, while the accelera-
1

as shown. The mass m is moving with tion of the other particle is zero. The
speed u and the mass 2m is stationary. centre of mass of the two particles
After their first collision, the time moves in a
elapsed for next collision is [coefficient (a) circle (b) parabola
of restitution e = 1/2] (VITEEE, 2009) (c) straight line (d) ellipse
43. A particle of mass 2m is projected at
an angle of 45° with horizontal with a
r velocity of 20 2 m/s. After 1 second
2m explosion takes place and the particle
u is broken into two equal pieces. As a
u
result of explosion one part comes to
m rest. The maximum height from the
ground attained by the other part is
2p r 4p r
(a) (b) (g = 10 m/s2)
u u (a) 50 m (b) 25 m
3p r 12p r (c) 40 m (d) 35 m
(c) (d)
u u 44. Particles of masses m, 2m, 3m, ..., nm
41. A block A slides over an another grams are placed on the same line at
block B which is placed over a smooth distances l, 2l, 3l, ..., nl cm from a fixed
inclined plane as shown in figure. The point. The distance of centre of mass
coefficient of friction between the two of the particles from the fixed point in
blocks A and B is m. Mass of block B centimeters is
is two times the mass of block A. The (2n + 1)l 1
acceleration of the centre of mass of (a) (b)
3 n+1
two blocks is
n(n2 + 1)l 2l
(c) (d)
n(n2 + 1)
A

2
B

Multiple Correct Options


1. A set of n identical cubical blocks lies at
rest parallel to each other along a line
q on a smooth horizontal surface. The
314പWŚLJƐŝĐƐĨŽƌ//dͲ:͗DĞĐŚĂŶŝĐƐ//

separation between the near surfaces 1


of any two adjacent blocks is L. The (b) is equal mv2, if he walks normal
2
block of one end is given a speed v to rails
towards the next one at time t = 0. 1
(c) can never be less than mv2
All collisions are completely inelastic, 2
then 1
(d) is greater than mv2, if he walks
(a) the last block starts moving at t = 2
along the rails
(n - 1)L 4. A sphere A moving with a speed u and
v rotating with an angular velocity w,
makes a head-on elastic collision with
(b) the last block starts moving at t =
an identical stationary sphere B. There
n(n - 1)L is no friction between the surfaces of A
2v and B. Disregard gravity.
(c) the centre of mass of the system (a) A will stop moving but continue
will have a final speed v to rotate with an angular velocity
(d) the centre of mass of the system w.
v (b) A will come to rest and stop
will have a final speed rotating
n
(c) B will move with a speed u
2. Following figure shows four square without rotating
metal plates that have a section re- (d) B will move with a speed u and
moved. The origin of the x- and y-axis rotate with an angular velocity w.
is at the centre of the original plate. In 5. In head-on collision of two identical
which case is the centre of mass of the bodies
remaining plate is below the origin? (a) the momenta are interchanged
y y (b) the velocities are interchanged
(c) the kinetic energies are inter-
(a) (b) changed
x x
(d) none of the above
6. A body moves towards another sta-
y y tionary body and collides. Then
(a) both the bodies move after colli-
(c) (d)
x x sion
(b) the moving body may come to
rest and other starts moving
3. A man of mass m is stationary on a
(c) both come to rest
stationary flat car. The car can move
(d) stationary body may move with
without friction along horizontal rails.
a velocity helping the velocity of
The man starts walking with v relative
colling body
to the car. Work done by him
7. A bomb at rest explodes into large
1 number of tiny fragments. Then
(a) is less than mv2, if he walks
2 (a) the momentum of all the fragments
along the rails is zero
ŽůůŝƐŝŽŶƐപ315

(b) the momentum of all the fragments (d) When the spring is in the state
increases of maximum compression the
(c) the KE of all the fragment remains kinetic energy in the centre of
zero mass frame is zero.
(d) the KE of all the fragment is more 11. A ball of mass 1 kg strikes a wedge of
than zero mass 4 kg horizontally with a velocity
8. In head-on elastic collision of two of 10 m/s. Just after collision velocity
bodies of equal masses of wedge becomes 4 m/s. Friction is
(a) the velocities are interchanged absent everywhere and collision is
(b) the speeds are interchanged elastic. Select the correct alternative(s)
(c) the momenta are interchanged
(d) the faster body slows down and 10 m/s
the slower body speeds up
9. A ball hits the floor and rebounds
after an inelastic collision. In this case, 30°
choose the correct alternative
(a) the momentum of the ball just (a) Speed of ball after collision is 6
after the collision is same as that m/s
just before the collision (b) Speed of ball after collision is 8
(b) the total energy of the ball and the m/s
earth is conserved (c) Impulse between ball and wedge
(c) the mechanical energy of the ball during collision is 16 N-s
remains the same in the collision (d) Impulse between ball and wedge
(d) the total momentum of the ball during collision is 32 N-s
and the earth is conserved 12. A horizontal block A is at rest on a
10. A block of mass m moving with a smooth horizontal surface. A small
velocity v0 collides with a stationary block B, whose mass is half of A, is
block of mass M at the back of which a placed on A at one end and project
spring of spring constant k is attached, along other end with some velocity
as shown in the figure. Select the u. The coefficient of friction between
correct alternative(s) blocks is m. Then
v0 k
Smooth m M B u

A
(a) The velocity of centre of mass is
Ê m ˆ
ÁË ˜ v0
m + M¯ (a) the blocks will reach a final
(b) The initial kinetic energy of the common velocity u/3
system in the centre of mass frame (b) the work done against friction is
two-thirds of the initial kinetic
1 Ê mM ˆ 2 energy of B
is Á ˜ v0
4 Ë M + m¯ (c) before the block reach a common
(c) The maximum compression in the velocity, the acceleration of A
relative to B is (2/3) mg
mM 1
spring is v0
( M + m) k
316പWŚLJƐŝĐƐĨŽƌ//dͲ:͗DĞĐŚĂŶŝĐƐ//

(d) before the block reach a common


u
velocity, the acceleration of A
relative to B is (3/2) mg a
13. Two balls, having linear momenta q
 
P1 = Pi and P2 = - Pi , undergo a colli-
sion in free space. There is no external 
force acting on the balls. Let P1¢ and P2¢ ,
be their final momenta. The following (a) the speed of the ball after collision
option(s) is not allowed for any non- will be less than u
zero value of P, a1, a2, b1, b2, c1 and c2. (b) the angle q will be greater than
(IIT-JEE, 2008) angle a
 
  
(a) P1¢ = a1 i + b1 j + c1 k ; P2¢ = a2 i + b2 j (c) the coefficient of restitution be-
  tween ball and wall is 1
(b) P1¢ = c1 k  ; P¢ = c k 
2 2 (d) the coefficient of restitution be-
 
 ; P ¢ = a i + b j - c k
(c) P1¢ = a1 i + b1 j + c1 k 2 2 2 1

tan a
  tween ball and wall is e =
(d) P ¢ = a i + b j; P ¢ = a i + b j
1 1 1 2 2 1 tan q
14. A body of mass 3 kg, moving with 17. In a one-dimensional collision between
a speed of 4 m/s, collides head-on two identical particles A and B, B is
with a stationary body of mass 2 kg. stationary and A has momentum P
Their velocity of separation after the before impact. During impact, B gives
collision is 2 m/s. Then impulse J to A
(a) the coefficient of restitution is 0.5 (a) the total momentum of the ‘A
(b) the impulse of collision is 7.2 Ns plus B’ system is P before and af-
(c) the loss of kinetic energy due to ter the impact, and (P – J) during
collision is 3.6 J the impact
(d) the loss of kinetic energy due to (b) during the impact, A gives im-
collision is 7.2 J pulse J to B
15. Two balls A and B having masses m
(c) the coefficient of restitution is
kg and 2m kg, moving with speeds
21 m/s and 4 m/s respectively in 2J
-1
opposite directions, collide head-on. P
After collision, A moves with a speed (d) the coefficient of restitution is
of 1 m/s in the same direction. Then
(a) the velocity of B after collision is J
+1
6 m/s opposite to the direction P
before collision 18. A projectile body moving in the posi-
(b) the coefficient of restitution is 0.2 tive direction of an x-axis on a friction-
(c) the loss of kinetic energy due to less surface into an initially stationary
collision is 200 m J. target body in a one-dimensional col-
(d) the impulse of the force between lision. Four choices of momenta of the
the two balls is 20 m Ns. bodies versus time (before and after
16. A ball strikes a wall at an angle a with the collision) are given. Select the pos-
the normal of the wall. It rebounds at sible situation
an angle q
ŽůůŝƐŝŽŶƐപ317

(b) The acceleration of centre of mass


Projectile Target
g sin 2 q
is
(1 + sin 2 q )
P P
(c) The acceleration of centre of mass
(a) (b)
g cos q
t t is
1 + sin 2 q
P P (d) The acceleration of block vertically
(c) (d) 2 g sin 2 q
downwards is
t t (1 + sin 2 q )
21. A block of mass 1 kg is pushed towards
19. A smooth sphere of mass m is moving
another block of mass 2 kg from 6 m
on a horizontal plane with a velocity
distance as shown in figure. Just after
(3i + j) , when it collides with a vertical collision velocity of 2 kg block becomes
wall which is parallel to vector j. If the 4 m/s.
coefficient of restitution between the
1 1 kg 2 kg
sphere and the wall is , then 6 m/s
2
(a) velocity of the sphere after impact 6m
-3  (a) coefficient of restitution between
i+j
2 two blocks is 1
(b) the loss in kinetic energy caused (b) coefficient of restitution between
27 m  two blocks is 1/2
by the impact j (c) velocity of centre of mass after 2 s
8
 is 2 m/s
(c) the impulse I that acts on the
(d) velocity of centre of mass after 2 s
-9  is 1 m/s
sphere = mi
2
(d) none of these. Passages and Matrix Matching
20. A block of mass m slides down an in- Passage for Q: 1–2
clined wedge of same mass m shown in Block A in figure has a mass of 1 kg, and
figure. Friction is absent everywhere. block B has a mass of 2 kg. The blocks are
forced together, compressing a spring S
m between them, and the system is released
from rest on a level frictionless surface. The
m spring is not fastened to either block and
drops to the surface after it has expended.
q
Block B acquires a speed of 0.5 m/s.

(a) The acceleration of block vertically S


A B
g cos 2 q
downwards is
(1 + sin 2 q )
318പWŚLJƐŝĐƐĨŽƌ//dͲ:͗DĞĐŚĂŶŝĐƐ//

1. The kinetic energy of the block A is (a) 9 N-s (b) 10 N-s


(a) 0.5 J (b) 1 J (c) 15 N-s (d) 20 N-s
(c) 2.0 J (d) 2.5 J 6. The average force on the ball from
2. The potential energy stored in the the player’s foot during the period of
spring is contact is
(a) 0.5 J (b) 0.75 J (a) 1000 N (b) 2000 N
(c) 1.0 J (d) 2.5 J (c) 3000 N (d) 4500 N
7. The maximum force on the ball from
Passage for Q: 3–4
the player’s foot during the period of
A jet of liquid of cross-sectional area A and contact is
density r moves with speed v in the positive (a) 200 N (b) 3000 N
x-direction and impinges against a perfectly (c) 4500 N (d) 5000 N
smooth blade B, which deflects the stream
at right angle but does not slow it down, Passage for Q: 8–9
as shown in figure. The blade moves to the A ball is dropped on an inclined plane and
right with a speed vB. is observed to move horizontally after the
impact. The coefficient of restitution between
y
plane and ball is e.
v

v
x
u 0
B v

q
3. The force exerted by the jet, Fx on the 8. The inclination q is
blade is
(a) r A (v – vB) (b) rA (v – vB)2 (a) 45° (b) tan–1 e
(c) r A vB (v – vB) (d) rAv2 e
4. The power delivered on the blade (c) tan–1 e (d) tan–1
2
(a) r A (v – vB)2
9. The velocity of ball after impact is
(b) r A vB2 (v – vB)
(a) v = u/2 (b) v = eu
(c) r A vB (v – vB)2
(d) r A v2 vB (c) v = u e (d) v = e2 u

Passage for Q: 5–7 Passage for Q: 10–12


A soccer player kicks a soccer ball of mass A small block of mass M moves on a friction-
0.45 kg that is initially at rest. The player’s less surface of an inclined plane as shown
foot is in contact with the ball for 3.0 ¥ 10–3 s, in figure. The angle of the incline suddenly
and the force of the kick is given by changes from 60° to 30° at point B. The block
is initially at rest at A. Assume that collision
F(t) = [(6.0 ¥ 10)6 t – (2.0 ¥ 10–9)t2] N,
between the block and the incline are totally
for 0 £ t 3.0 x–3 s, when t is in seconds. inelastic (g = 10 m/s2).
5. The impulse on the ball due to the kick 10. The speed of the block at B immediately
is after it strikes the second incline is
(IIT-JEE, 2008)
ŽůůŝƐŝŽŶƐപ319

A M 15
v (a) 8 m/s (b) m/s
3
60° B 25 25
(c) m/s (d) m/s
3 7
30° C
14. Impulse produced by string on block B
3m 3 3m (a) 50 N-s (b) 25 N-s

(a) 60 m/s (b) 45 m/s 50 50


(c) N-s (d) N-s
3 4
(c) 30 m/s (d) 15 m/s
15. Maximum displacement of block A in
11. The speed of the block at point C im-
left direction is approximately (g = 10
mediately before it leaves the second
m/s2)
incline is
(a) 2.2 m (b) 3.2 m
(a) 120 m/s (b) 105 m/s
(c) 4.2 m (d) 5.2 m
(c) 90 m/s (d) 15 m/s
Passage for Q: 16–18
12. If collision between the block and
A soccer player kicks a soccer ball of mass
the incline is completely elastic then
0.45 kg that is initially at rest. The player’s
the vertical (upward) component of
foot is in contact with the ball for 3.0 ¥ 10–3 s,
the velocity of the block at point B,
and the force of the kick is given by
immediately after it strikes the second
incline is F(t) = [(6.0 ¥ 10)6 t – (2.0 ¥ 10–9) t2] N,

(a) 30 m/s (b) 15 m/s for 0 £ t £ 3.0 ¥ 10–3 s, when t is in seconds.


(c) 0 (d) – 15 m/s 16. The time of journey of sphere is
Passage for Q: 13–15
A
Block A(1 kg) is placed on smooth horizontal t2
surface and connected with a block B (2 kg), t1
as shown in the figure, by an inextensible
string. A bullet of mass 250 gm, strikes the
u
block A horizontally with speed 200 m/s. a
The bullet penetrates through the block A 0 B
d
and comes out with velocity 100 m/s.
A m = 250 gm
(a) u sin a/g (b) 2u cos a/g
Speed = 200 m/s
1 kg u tan a
(c) 2u cos a/g (d)
2g
17. The horizontal distance d from the
wall is
u2 sin 2a
B 2 kg (a)
g

13. Velocity of 2 kg block just after bullet u2 cos 2d (1 + e )


(b) =
comes out of block A g e
320പWŚLJƐŝĐƐĨŽƌ//dͲ:͗DĞĐŚĂŶŝĐƐ//

u2 sin 2a (1 + e ) rests on a smooth horizontal surface with the


(c) = spring in the relaxed state. A small object of
g e
mass m moving horizontally with speed v
(d) none of these at height d above the horizontal surface hits
18. If the line joining the point of projection the block B along the line of their centre of
and the point of impact makes an angle mass in perfectly elastic collision. There is no
q with the horizontal, then tan q is friction between A and B.
tan a
(a) e tan a (b) A
(1 + e ) A 2d
k 2mg B
tan a m V B
(c) (1 + e) cot a (d) d d
(1 + e )
2d

Passage for Q: 19–21 22. The minimum value of V (say V0) such
A rectangular block 0.25 m high is dragged that block A will topple over block B
to the right along level surface at constant
speed by a horizon- k
0.25 (a) v0 = 3d
tal force P, as shown 8m
in the adjacent figure. c.g.
0.5 m P k
The coefficient of slid- (b) v0 = d
ing friction is 0.40, the
h 8m
block weighs 25 N, and d k
its centre of gravity is at its centre (c) v0 =
2 8m
19. The magnitude of the force P is
(a) 5 N (b) 10 N d k
(d) v0 =
(c) 15 N (d) 20 N 3 m
20. The position of line of action of the 23. If V0 = V/2, find the period of oscil-
normal force N exerted on the block by lation of the block spring system and
the surface, if the height h = 0.125 m. amplitude is
(a) 0.5 m (b) 0.25 m
(c) 0.20 m (d) 0.05 m m 2m
(a) 2p (b) 2p
21. The value of h at which the block just k k
starts to tip is
(a) 0.31 m (b) 0.62 m k m
(c) 2p (d) p
(c) 0.93 m (d) none of 2m k
these
24. The energy stored in the spring when
Passage for Q: 22–24 the block B return to its initial position
A and B are two identical blocks of same as before collision is
mass 2 m and same physical dimensions. A
1 2
is placed over the block B which is attached (a) kd (b) 2 kd2
to one end of the spring of natural length l 2
and spring constant k. The other end of the 3 2
spring is attached to a wall. The system is (c) kd (d) zero
2
ŽůůŝƐŝŽŶƐപ321

Passage for Q: 25–27 26. The speed of the smaller mass when it
Figure shows a small body of mass m placed breaks off the larger mass at height h is
over a larger mass M whose surface is hori- Ê mv ˆ Ê Mv ˆ
(a) Á (b) Á
zontal near the smaller mass and gradually Ë m + M ˜¯ Ë m + M ˜¯
curved to become vertical. The smaller mass
is pushed on the longer one at a speed v and 1/2
È ( M 2 + Mm + m2 )v 2 ˘
the system is left to itself. Assume that all the (c) Í 2
- 2 gh ˙
surfaces are frictionless. Î ( M + m) ˚
vy (d) none of these
vx m
27. The maximum height (from the
h ground) that smaller mass ascends is
vx v
M M m v2
(a) h =
2g
25. The speed of the larger block when È Mv 2 ˘
smaller block is sliding on the vertical (b) Í ˙
part is Î 2 g ( M + m) ˚

Ê mv ˆ Ê Mv ˆ mv 2
(a) Á (b) Á (c)
Ë m + M ˜¯ Ë m + M ˜¯ 2 gM
v (d) none of these
(c) (d) v
2

Matrix Matching
28. A particle of mass m, kinetic energy K and momentum p collides head-on elastically
with another particle of mass 2 m at rest. Match the following (after collision):
Column I Column II
4
(a) Momentum of the first particle (p) p
3
(b) Momentum of the second particle (q) K/9
(c) Kinetic energy of the first particle (r) –p/3
(d) Kinetic energy of the second particle (s) 8K/9
29. A body initially moving towards the right explodes into two pieces 1 and 2. The
magnitudes of v1 and v2 (the final velocities) are completely arbitrary. Directions of
motion of the pieces are shown in Column I and possible mass ratios are shown in
Column II.
Column I Column II
v1

1
(a) (p) m1 > m2
2

v2
322പWŚLJƐŝĐƐĨŽƌ//dͲ:͗DĞĐŚĂŶŝĐƐ//

1 v1

(b) 2 (q) m1 = m2

v2

v1

1
(c) (r) m1 < m2
2

v2

v1

1
(d) (s) impossible for any masses
2

v2

30. Match the columns:


Column I Column II
(a) Collision of two light nuclei to form a (p) Elastic collision
heavier nucleus
(b) Speeding bullet getting embedded in (q) Perfectly inelastic collision
a wooden plank
(c) Collision of neutron with heavy unstable (r) Nuclear fission
nucleus
(d) Collision in which there is no loss of (s) Nuclear fusion
external kinetic energy

Answers Key
.GXGN
1. (a) 2. (c) 3. (c) 4. (c) 5. (c) 6. (c) 7. (d) 8. (b)
9. (a) 10. (a) 11. (d) 12. (a) 13. (a) 14. (d) 15. (c) 16. (d)
17. (c) 18. (c) 19. (d) 20. (b) 21. (c) 22. (d) 23. (a) 24. (c)
25. (a) 26. (a) 27. (d) 28. (c) 29. (d) 30. (a) 31. (d) 32. (d)
33. (a) 34. (c) 35. (b) 36. (b) 37. (c) 38. (c) 39. (c) 40. (c)
41. (c) 42. (c) 43. (a) 44. (c) 45. (d) 46. (a) 47. (a) 48. (a)
49. (c) 50. (c) 51. (a) 52. (b)
.GXGN
1. (d) 2. (c) 3. (c) 4. (b) 5. (a) 6. (b) 7. (a) 8. (a)
9. (b) 10. (c) 11. (c) 12. (b) 13. (b) 14. (a) 15. (c) 16. (c)
17. (c) 18. (b) 19. (b) 20. (c) 21. (a) 22. (b) 23. (d) 24. (a)
25. (a) 26. (d) 27. (b) 28. (a) 29. (b) 30. (b) 31. (a) 32. (a)
33. (d) 34. (a) 35. (b) 36. (c) 37. (c) 38. (b) 39. (c) 40. (b)
41. (a) 42. (b) 43. (b) 44. (a)
ŽůůŝƐŝŽŶƐപ323

/WNVKRNG%QTTGEV1RVKQPU
1. (b, d) 2. (a, c) 3. (a, b) 4. (a, c)
5. (a, b, c) 6. (a, b, d) 7. (a, d) 8. (a, c, d)
9. (b, d) 10. (a, c, d) 11. (a, d) 12. (a, b, d)
13. (a, d) 14. (a, b, d) 15. (a, b, d) 16. (a, b, d)
17. (a, b, c) 18. (a, b, c) 19. (a, b, c) 20. (b, d)
21. (a, c)
2CUUCIGUCPF/CVTKZ/CVEJKPI
1. (a) 2. (b) 3. (b) 4. (c) 5. (a) 6. (c) 7. (c) 8. (c)
9. (c) 10. (b) 11. (b) 12. (c) 13. (c) 14. (c) 15. (d) 16. (b)
17. (c) 18. (d) 19. (b) 20. (d) 21. (a) 22. (a) 23. (b) 24. (d)
25. (a) 26. (c) 27. (b) 28. AÆr; BÆp; CÆq; DÆs
29. AÆs; BÆs; CÆp, q, r; DÆp, q, r 30. AÆq, s; BÆq; CÆr; DÆp, r, s

SOLUTION AND HINTS

Level 1
1. (a) 0 £ e £ 1 and total energy of the system remains constant.
2. (c) Collision between ball 1 and the first two balls is oblique collision, while between
these and the next two is head-on collision, so option (iii) is right.
3. (c) In an inelastic collision, some part of KE will convert into heat energy.
 
4. (c) F Dt = mDv
  
mDv
\ F = , as Dt becomes longer so F becomes smaller.
Dt
5. (c) Charge can be negative and so, centre of charge can lie out of charges.
6. (c) All the three particles of the system cannot be in equilibrium.
7. (d) If linear momentum of the system is zero, then KE of the system need not be zero.
But if KE of the system is zero, then momentum of the system must be zero.
8. (b) As Q. 35.
9. (a) In inelastic collision, non-conservative forces evolve and so only momentum
remains constant.
10. (a) In perfectly inelastic collision, the striking bodies will stick together.
11. (d) For collision,
   
r1 + v1t = r2 + v 2t
or (3i + 5j) + ( 4i + 3j) ¥ 2 = (- 5i - 3j) + ( ai + 7j) ¥ 2
\ a =8
1 1 3
12. (a) Mass of pieces are: kg , kg , kg
5 5 5
m3v3 = (m1v1)2 + (m2v2)2
324പWŚLJƐŝĐƐĨŽƌ//dͲ:͗DĞĐŚĂŶŝĐƐ//

2 2
3 Ê1 ˆ Ê1 ˆ
or v3 = ÁË ¥ 15˜¯ + ÁË ¥ 15˜¯
5 5 5
\ v3 = 3 2 m/s
13. (a) F ¥ 0.2 = mg (2 + 0.2)
F ¥ 0.2 = 0.2 ¥ 10 ¥ 2.2
\ F = 22 N.
DP 0.15 ¥ 20
14. (d) F = = = 30 N
Dt 0.1
  
15. (c) P = P1 + P2
  
\ P2 = P - P1
or P2 = P 2 + P12 = 122 + 52
= 13 kgm/s.
16. (d) h1 = e2h = 0.62 ¥ 1 = 0.36 m
     
17. (c) P1 + P2 + P3 = 0, and so P3 = -(P1 + P2 )

or (2m) v3 = (mv)2 + (mv)2


v
\ v3 =
2
Total energy released in the process
2
1 1 1 Ê v ˆ
= mv 2 + mv 2 + (2m) Á
2 2 2 Ë 2 ˜¯
3
= mv2
2
18. (c) Child is the internal part of the system, so velocity of centre of mass will not change
due to his movement.
19. (d) 4v1 + 8 ¥ 6 = 0
\ v1 = – 12 m/s.
1
Now K1 =¥ 4 ¥ (12)2 = 288 J.
2
20. (b) Mv + 0 = mv1 + mv2
v

1 1 1
Mv2 + 0 = Mv12 + mv22
2 2 2
Simplifying for m << M, we get vz = 2v.
ŽůůŝƐŝŽŶƐപ325

1
21. (c) Collision occurs, when r £ 2R, and U μ , and so option (c) is the correct.
r
1
22. (d) K0 =mu2
2
mu + 0 = mv1 + mv2
1 1
and k0 + 0 = mv12 + Mv22
2 2
After simplifying the above equations, we get
1 4 mMk0
Mv22 =
2 ( M + m )2
23. (a) The net of the forces is zero, and so momentum of the particle will not change.
24. (c) Mu + 0 = Mv + mv
1 1 1
and Mu2 + 0 = Mv 2 + mv 2
2 2 2
After solving the above equations, we get
2u
v =
Ê mˆ
ÁË 1 + ˜¯
M
25. (a) 0 = 4v + (234) v2
- 4v
\ v2 =
234
26. (a) W = Dk
0 1
or Úa F dx =
2
m(v 2f - 0)

0 -k 1
Úa x 2 dx =
2
mv 2f

0
k 1
= mv 2f
xa 2

2k Ê a - x ˆ
\ vf = Á ˜
m Ë ax ¯

27. (d) m1Dx1 + m2Dx2 = 0


m1Dx1
\ Dx2 = –
m2
m1d
=–
m2
326പWŚLJƐŝĐƐĨŽƌ//dͲ:͗DĞĐŚĂŶŝĐƐ//

10 ¥ 14 + 4 ¥ 0
28. (c) vCM = = 10 m/s
10 + 4
29. (d) As body breaks into pieces due to internal forces and so there will no shift in centre
of mass of the body.
L
L (dm)x Ú0 (l xdx)x = 2L
30. (a) XCM = Ú0 L
= L
Ú0 (dm) Ú0 (l xdx) 3
31. (d) Coordinates of spheres are: (0, 0), (2R, 0) and (R, 3 R)

m ¥ 0 + m ¥ 2R + m ¥ R
XCM = =R
m+m+m

m ¥ 0 + m ¥ 0 + m 3R R
and YCM = =
m+m+m 3

P12 + P22 (1 ¥ 1)2 + (2 ¥ 2)2


32. (d) v = =
m1 + m2 1+ 2
= 1.37 m/s
m1 ¥ 0 + m2 ¥ 0
33. (a) Initially, vCM = =0
m1 + m2
As no external force acts on the system, and so vCM remains constant.
34. (c) The kinetic energy of the body at B
K = mgh = 10 ¥ 10 ¥ 10 = 1000 J
Work done by friction on the body in moving from
B to C = – fr ¥ s
= – m mg ¥ s = – 0.1 ¥ 10 ¥ 10 ¥ 10
= – 100 J
Thus, the kinetic energy of the body at C
= 1000 – 100 = 900 J
Let spring get compressed by x, then we have
1 2
kx = 900
2
1
or (2 ¥ 104) x2 = 900
2
or x = 0.3 m
ŽůůŝƐŝŽŶƐപ327

 
 m r + m2 r2
35. (b) r cm = 1 1
m1 + m2
 ) + 3(- 3i - 2j + k
1 ¥ (i + 2j + k )
=
1+ 3

= – 2i - j + k
m1r1 - m2 r2
36. (b) 0 =
m1 + m2
m1r1
or m2 = , as r2 < r1, so m2 > m1.
r2
37. (c) In isolated system, its total mass remains constant and so its velocity remains
constant ( P = const).
L L
M¥ + 3M ¥ 0 + M ¥
38. (c) yCM = 2 2 =L
M + 3M + M 5

39. (c) 2 ¥ 3 – 1 ¥ 4 = (2 + 1) v
2
\ v =
m/s
3
40. (c) Loss in KE in inelastic collision is given by
1 Ê m1m2 ˆ
DK = (u1 – u2)2
2 ÁË m1 + m2 ˜¯

1 Ê MM ˆ 2
= Á ˜ (u2 – u1)
2 Ë M + M¯
M
= (u2 – u1)2
4

41. (c) 2m(v¢) = (mv)2 + (mv)2


v
\ v¢ =
2
42. (c) DP = 2mv sin 60°
DP 2mv ¥ 3/2
F = =
Dt 2 ¥ 10 - 3
2 ¥ 0.1 ¥ 5 ¥ 3/2
=
2 ¥ 10 - 3

= 250 3 N
328പWŚLJƐŝĐƐĨŽƌ//dͲ:͗DĞĐŚĂŶŝĐƐ//

43. (a) h = 10 m,
\ u1 = 2 g ¥ 10
h1 = 0.8 h = 8 m,
\ v1 = 2g ¥ 8
Èv -v ˘
Now, e = – Í 2 1˙
Î u2 - u1 ˚
È 0 - (- 2 g ¥ 8 ) ˘
=–Í ˙ = 0.89
ÍÎ 0 - 2 g ¥ 10 ˙˚
 Dv 2 ¥ 10
44. (c) a = =
Dt Dt
45. (d) hn = e4 h
46. (a) m ¥ 6 + M ¥ 4 = mv1 + Mv2
1 1 1 1
and m ¥ 6 2 + M ¥ 4 2 = mv12 + Mv22
2 2 2 2
After simplifying for, m << M, we get
v1 = 2 m/s
47. (a) Their velocities get exchanged.
   
48. (a) P1 + P2 = P1¢ + P2¢
v  
mv i + 0 = m j + mv 2
3
2
Ê v ˆ 2v
\ v2 = v2 + Á =
Ë 3 ˜¯ 3
  
49. (c) 0 = mv 1 + mv 2 + mv 3

or 0 = m(vi + vj) + mv 3

\ v 3 = - v(i + j)
( 3 m - m) g g
50. (c) a = =
3m + m 2
3m ¥ g/2 + m(- g/2) g
aCM = = m/s2
3m + m 4
51. (a) (500 + 50) ¥ 20 = 500 v + 50 ¥ 20
\ v = 20 m/s.
52. (b) 3 ¥ 2 – 2 ¥ 3 = 3v1 + 2v2
2 [v - v ]
and = 2 1
3 [3 - (-2)]
After simplifying, we get
v1 = – 4/3, v2 = 2
ŽůůŝƐŝŽŶƐപ329

Level 2
1. (d) mu = (mu + m) v
È u ˘
or v = Í ...(i)
Î n + 1 ˙˚

h
u

1 1
Now, mu2 = (nm + m)v2 + mgh ...(ii)
2 2
After solving the above equations, we get

u = 2 gh(1 + 1/n)
2. (c) 5M (– Dx1) + M(4L – Dx1) = 0
2L
\ Dx1 =
3

5M
Dx1
Dx2

5M

Êl ˆ
3. (c) Wy = W Á - 2˜
Ë2 ¯
l l/2
\ y= y
4
w w
l l 3l 2w
Now, x= + =
2 4 4
4. (b) For the deflection of spring by x, the energy stored in the spring,
x
U= Ú Fdx
0
x
x3
Ú 300 x dx = 300
2
= = 100x3
0
3
330പWŚLJƐŝĐƐĨŽƌ//dͲ:͗DĞĐŚĂŶŝĐƐ//

Thus, by conservation of mechanical energy, we have


1
100x3 = mv 2
2
1
or 100x3 = ¥ 200 ¥ (1.0)2
2
or x =1m
m m
5. (a) mv cos q = (- v cos q ) + v ¢
2 2
\ v¢ = 3 v cos q
6. (b) 1 ¥ 2 + 0 = (1 + 1) v,
1
\ v = m/s
2
2
1 1 Ê 1ˆ 1
Now, ¥ 1 ¥ 22 + 0 = (1 + 1) ¥ Á ˜ + ¥ 50 ¥ x 2
2 2 Ë 2¯ 2
\ x = 0.26 m
7. (a) mu = mv cos 30° + mv cos 30° 8

Ê 0-v ˆ
and e =– Á ˜¯
Ë u cos 30∞ - 0
8. (a) Suppose velocity of the tank is v¢ after the shell is fired. The velocity of shell in the
horizontal direction relative to ground is, u = v cos a + v¢.

a
v

Thus, by conservation of linear momentum, we have


(M + m)v = Mv¢ + m (v cos a + v¢)
mv cos a
=v–
M+m
The tank will stop after firing, if v¢ = 0

mv cos a
or v =
M+m
ŽůůŝƐŝŽŶƐപ331

 
9. (b) 0 = Pgamma + Pnucleus
 
\ Pnucleus = Pgamma
or Pnucleus = E/c
2
Pnucleus
\ Energy released = +E
2M

E2
= +E
2 Mc 2
Ê Tˆ
F0 Á T + ˜
Ë 2¯
10. (c) = m(0 – u)
2
4 mu
\ F0 =
3T
11. (c) The time taken in the first collision
III

120°
I II

2p r 120∞
= =
3v w

240∞ 360∞
The second collision will be at and the third . So there are two collisions,
excluding at A. w w

12. (b) m ¥ 9 + 0 = mv1 + 2mv2


-(v2 - v1 )
and 1=
0-9
\ v2 = 6 m/s
Now, 2m ¥ 6 + 0 = (2m + m)v
or v = 4 m/s
13. (b) J = m(vf + vi)y = 1 (25 sin 45° + 20 sin 30°)
= 27.67 N-s
332പWŚLJƐŝĐƐĨŽƌ//dͲ:͗DĞĐŚĂŶŝĐƐ//

14. (a) F = rAQ = rAV2

Q
q

\ Fwall = 2F cos q = 2rAv2 cos q


15. (c) 2mu cos q/2 = 2m u/2
m

q u/2
2m

1
\ cos q/2 =
2
or q/2 = 60° and q = 120°
16. (c) 1 ¥ 20 – 0.5 ¥ 30 = 1 ¥ 10 cos 30° – 0.5 v cos q ...(i)
1 ¥ 10 sin 30° = 0.5 v sin q ...(ii)
After solving the above equations, we get
v = 12 cm/s
M
17. (c) M(L + Dx1) + (-Dx1 ) = 0
3
3L
\ Dx1 =
4
L
Thus, L – Dx1 =
4
18. (b) Solution is given in the example.
19. (b) J = (mv – 0)
= 1 ¥ 2 gh = 1 ¥ 2 ¥ 10 ¥ 2

= 40 N-s
20. (c) From conservation of momentum, we have
mv = (2m + m) v¢
\ v¢ = v/3
21. (a) The initial extension of the spring
mg
y0 = ...(i)
k
ŽůůŝƐŝŽŶƐപ333

The velocity of combined block after collision


mv = (m + m)v
v
\ v¢ =
2
Now from conservation of mechanical energy, we have
2
1 Ê vˆ 1
(2m) Á ˜ + ky02 = 2m gy0 ...(ii)
2 Ë 2¯ 2
After solving the above equations, we get
6m
v = g
k
22. (b) 0 = m 1v 1 – m 2v 2
1 1
and m1v12 + m2v22 = E
2 2
After simplifying the above equations, we get

2E(m1 + m2 )
v1 + v2 =
m1m2

23. (d) If F is the resistance of the block, then by work energy theorem, we have

Ê 1 ˆ
–Fs = Á 0 - mu2 ˜ (m + M)
Ë 2 ¯
v
mu2 s
\ F = ...(i)
2s
In the second case, by conservation of momentum, we have mu = (m + M)v
È mu ˘
\ v = Í
Î m + M ˙˚

1 1
Now, –Fs¢ = 2
(m + M )v 2 - mu2 ...(ii)
2mu 2
After simplifying the above equations, we get

È Ms ˘
s¢ = Í
Î m + M ˙˚
24. (a) The speed of the child u = – 4 + 10 = 6 m/s.
Now, 220 ¥ 10 = 20 ¥ 6 + 200 v
\ v = 10.4 m/s.
334പWŚLJƐŝĐƐĨŽƌ//dͲ:͗DĞĐŚĂŶŝĐƐ//

2¥2+2¥0
25. (a) xi = =1m
2+2
1¥ 2 + 3 ¥ 0
and xf = = 0.5 m
1+ 3
\ Dx = 0.5 m
2mg
26. (d) For the sphere B to be lifted of N = 0 and so x = m
x
k
From conservation of energy, we have
2 (l0 + x)
1 2mg 1 Ê 2mg ˆ
mx 2 + 0 = mg + + kÁ ˜ kx
2 k 2 Ë k ¯ N
4mg 2m
\ h =
k
2mg

   (10 + 20 + 30) ¥ 0 + 40 ¥ r2
27. (b) 3i + 3j + 3k =
(10 + 20 + 30 + 40)

\ r2 = 7.5i + 7.5j + 7.5k


m 3m
28. (a) y1 + y2 = 0
4 4
y - 15
\ y2 = – 1 = = – 5 cm
3 3
3 3

29. (b)
Ú (l dx)x = Ú0 (2 + x)xdx
xCM = 0 3 3
Ú0 l dx Ú0 (2 + x)dx
3
2x 2 x 3
+
2 3 12
= 0
= m
2 3 7
x
2x +
2 0

30. (b) m 1d – m 2x 2 = 0
md
\ x2 = 1
m2

31. (a) Initially, a1 = a2 = 0 and so am = 0, which remains as such.

6 m ¥ 0 + m ¥ a + m ¥ a + m ¥ 0 + m(- a)
32. (a) yCM =
6m + m + m + m + m
a
=
10
ŽůůŝƐŝŽŶƐപ335

33. (d) Total distance moved by the bodies,


x1 + x2 = 12R – 3R = 9R ...(i)
Also, Mx1 = 5Mx2 ...(ii)
After solving the above equations, we get
x1 = 7.5R
x2 = 1.5R
34. (a) If v1 and v2 are the velocities of the ball and the wedge after collision, then
v1

J 60° 60° J
v2 v2

60° 60°

For the ball:


2 J cos 60° = mv1 – (–mv0)
or J = Mv1 + Mv2
From the above equations, we get
2v2
= v1 + v0 ...(i)
3
[v cos 60∞ - (-v2 cos 30∞)]
Also, e = 1
(v0 cos 60∞ - 0)

v1 + 3v2
= ...(ii)
v0
After simplifying the above equations, we get
3
v2 = (1 + e )v0
5
2ux uy 2(u cos q )(eu sin q )
35. (b) Range, R = =
g 2

eu2 sin 2q
=
g
36. (c) The taut string is shown in figure. If vx and vy are the velocities along PQ and
perpendicular to it, then
P
J
J
l 2l vy
q u
Q vx
336പWŚLJƐŝĐƐĨŽƌ//dͲ:͗DĞĐŚĂŶŝĐƐ//

1
for system as a whole sin q
2
mu cos q + 0 = 2mvx
37. (c) The velocity of body A, before collision
u= 2 gy = 2 g(1 - 1 cos 60∞) = 3.13 m/s
Now, 1 ¥ 3.13 + 5 ¥ 0 = v1 + 5v2 ...(i)
È v -v ˘
and 0.8 = – Í 2 1 ˙ ...(ii)
Î 0 - 3.13 ˚
After solving, we get h = 0.0005 m

h
v2

h Ê 1 - 0.0005 ˆ
Now, cos q = =Á ˜¯ = 0.995 m
L Ë 1
38. (b) Let the velocity of projection of A be u, then A has to travel a u A
distance equal to half the perimeter to hit B, so
ut = pa
If vA and vB are the velocities of A and B after first impact, then pa
vA – vB = – e (u – 0) = – eu
For the second impact, the relative distance is to be travelled
= 2 pa. Thus, time taken
B
2p a 2ut 2t
t¢ = = =
vA - vB eu e
39. (c) As string does no work on the ball, energy conservation can be applied.
1
mv 2 = mg (L – L cos q)
2
fi v = 2mgL(1 - cos q )
On putting values, v = 10 m/s
40. (b) If just after collision, relative velocity v, then
v 1 v u
= \ wrel = =
u 2 r 2r
2p 4p r
\ time between 1st and 2nd collision, t = =
w rel u
ŽůůŝƐŝŽŶƐപ337


Fexert Mg sin q
41. (a) aCM = = = g sin q
M M
 1
42. (b) r1 = (2i)t + (3i + 3j)t 2
2

and r2 = 2j t
   
 m r + m2 r2 r1 + r2
Thus, rcm = 1 1 =
m1 + m2 2
3
= (i + j)t + (i + j)t 2
4
It represents a parabola.
43. (b) After 1 second,
vy = uy – gt = 20 2 45° – 10 ¥ 1 = 10 m/s
Now, 2m ¥ 10 = m ¥ 0 + mv
\ v = 20 m/s
Height attained in addition to previous height
10 2
h = =5m
2g
Previous height,
U y2 (20 2 sin 45∞)2
h0 = =
2g 2g
= 20 m.
Thus, h0 + h = 20 + 5 = 25 m.

44. (a) xCM = m ¥ l + 2m ¥ 2l + 3m ¥ 3l + … + nm ¥ nl


m + 2m + 3m + … + nm

(1 + 4 + 9 + … + n2 )l
=
1 + 2 +… + n

ln(n + 1)(2n + 1)/6


=
n(n + 1)/2

= (2n + 1)l
3
Multiple Correct Options
1. (b, d)
v
v v/2 v/3 n -1 v/n

m l 2m l 3m (n – 1)m
338പWŚLJƐŝĐƐĨŽƌ//dͲ:͗DĞĐŚĂŶŝĐƐ//

l l 2l
t1 = , t2 = =
v v/2 v

3l (n - 1)l
t3 = , … , tn - 1 =
v v

l 2l 3 l (n - 1)l
So T = + + +… +
v v v v

l
= [1 + 2 + 3 + … +(n – 1)]
v
n(n - 1)l
=
2v

mv + 0 + 0 + … + 0 v
vCM = =
nm n

2. (a, c)
Explanation in the theory.
3. (a, b)
If the man walks along the rails, some velocity say V is imparted to the car also. Let
M be the mass of car. Then from conservation of linear momentum.
M ◊ V = m (v = V)
mv
\ V =
m+ M
\ Work done by man
1 1
= m(v - V )2 + mV 2
2 2
1 Ê mM ˆ 2 1 2
= Á ˜ v < mv
2 Ë m + M¯ 2

Hence, option (a) is correct. If the man moves normal to the rails then the car will
1
not move. Hence, work done by him in this case will be mv2 and option (b) is also
correct. 2

4. (a, c)
Linear momentum of A will transfer to B, but not angular momentum.
5. (a, b, c)
6. (a, b, d)
System has non-zero initial momentum, so it must be after collision. Therefore, both
the bodies cannot be at rest after collision.
ŽůůŝƐŝŽŶƐപ339

7. (a, d)
The initial momentum of the bomb is zero, and so after explosion, it must be zero.
Some internal energy will convert into external KE.
8. (a, c, d)
In head-on elastic collision, momentum of bodies gets exchanged. If m1 = m2, then
speed gets exchanged.
9. (b, d)
The total momentum and total energy of (earth + ball) system remains conserved.
10. (a, c, d)
Ê m ˆ
Clearly, the velocity of centre of mass = Á v
Ë m + M ˜¯ 0
Initial KE in the centre of mass frame (KCM)
1
= (m + M)v2CM
2
2
1 ÈÊ m ˆ ˘
= (m + M ) ÍÁ ˜ v0 ˙
2 ÎË m + M ¯ ˚
1 m2v02
=
2 m+ M
The maximum compression (xm) in the spring is given by
1 2
kxm = E – KCM
2
1 2 1 1 m2v02
or kxm = mv02 -
2 2 2 m+ M

Ê m ˆ
or kx2m = mv02 Á 1 - ˜
Ë m + M¯
Ê M ˆ
= mv02 Á
Ë m + M ˜¯

Ê mM ˆ 2
= Á v
Ë m + M ˜¯ 0

Ê mM ˆ 1
fi xm = v0 Á
Ë m + M ˜¯ k

11. (a, d)
Collision is elastic. Therefore, kinetic energy will be conserved. Let v be the speed of
the ball after collision.
340പWŚLJƐŝĐƐĨŽƌ//dͲ:͗DĞĐŚĂŶŝĐƐ//

Then,

j sin 30°

j
j cos 30°
30°

1 1 1
(1)(10)2 = ( 4)( 4)2 + (1)v 2
2 2 2
or v = 6 m/s
Let J be the impulse between the two during collision.
Applying impulse = Change in linear momentum
J sin 30° = (4) (4)
or J = 32 N-S
12. (a, b, d)
From conservation of momentum,
Ê mˆ Ê mˆ
ÁË ˜¯ u = ÁË m + ˜¯ ◊ v
2 2
u
\ v =
3
Work done against friction = Ei – Ef
2
1 Ê m ˆ 2 1 Ê 3m ˆ Ê u ˆ
= Á ˜ u - ÁË ˜Á ˜
2Ë 2 ¯ 2 2 ¯ Ë 3¯

1 2Ê 1 ˆ
= mu2 = Á mu2 ˜
6 Ë
3 4 ¯
Force of friction on the two blocks before the blocks reach a common velocity is as
shown below,
a1
m/2 m
f =m g
2

m m
f = mg a2
2

m 3
a1 = mg and a2 = g \ ar = mg
2 2
ŽůůŝƐŝŽŶƐപ341

13. (a, d)
Momentum before collision,
  
P = P1 + P2 = Pi - Pi = 0
Option (a) and (b), added together cannot give zero momentum.
14. (a, b, d)
velocity of separation 2
e = =
velocity of approach 4 - 0
= 0.5.
Now, 3 ¥ 4 – 2 ¥ 0 = 3v1 + 2v2
Èv - v ˘
and 0.5 = – Í 2 1 ˙
Î 0-4 ˚
After simplifying the above equations, we get
v1 = 1.5 m/s.
Thus, J = m1(v1 – v1) = 3 (4 – 1.6)
= 7.2 N-s
15. (a, b, d)
m ¥ 21 – 2m ¥ 4 = m ¥ 1 + 2m ¥ v2
\ v2 = 6 m/s
Èv -v ˘ È 6-1 ˘
e = Í 2 1 ˙ = -Í ˙ = 0.2
Î u2 - u1 ˚ Î - 4 - 21 ˚
J = m(v1 – u1) = m [21 – 1]
= 20 m N-s
16. (a, b, d)
See examples.
17. (a, b, c)
According to conservation of momentum (a, b) are obviously correct.
By the definition
Èv -v ˘
e = – Í 2 1˙
Î u2 - u1 ˚
If v is the velocity during collision, then
m È (v1 - v) - (v - v2 ) ˘
=– Í ˙
mÎ u2 - u1 ˚
m(v1 - v) - m(v - v2 )
=–
m(u2 - 0)
(P - J ) - J
=–
P
2J - P
=
P
342പWŚLJƐŝĐƐĨŽƌ//dͲ:͗DĞĐŚĂŶŝĐƐ//

18. (a, b, c)
The system must have no momentum in positive x-axis and so (a, b, c) are correct only,
and also target cannot be move in negative x-axis.
19. (a, b, c)
If we resolve the initial velocity along x- and y-axis, then it is clear that uy will
remain constant after collision but ux will change. Also, m2 >> m1, therefore m1 can be
neglected. Hence,
uy
Ê - em2 ˆ 1 3 r
v
(vx) 1 = Á (ux )1 = - e(ux )1 = - ¥ 3 = -
Ë m2 ˜¯ 2 2

\ Velocity after collision is


 3
v = – i + j ux
2
Similarly, we can find that
27
loss in KE = ĵ
8m
9m
and impulse = - î Wall
2
20. (b, d)
Let a be the acceleration of wedge leftwards and ar the relative acceleration of block
down the plane. Then absolute acceleration of block in horizontal direction will be
(ar cos q – a) towards right. Net force on the system in horizontal direction is zero.
Therefore, acceleration of COM in horizontal direction will be zero or acceleration of
wedge towards left is equal to the acceleration of block towards right.
\ ar cos q – a = a
or 2a = ar cos q ...(1)
Now let N be the normal reaction between the block and the wedge. Then free body
diagram of the wedge gives

N sin q

N cos q
q
a

N sin q = ma ...(2)
ŽůůŝƐŝŽŶƐപ343

Free-body diagram of the block with respect to wedge is:


N

ma (pseudo force)
q

mg
q

Net force on the block perpendicular to the plane is zero.


Hence, N + ma sin q = mg cos q ...(3)
Solving Eqs. (1), (2) and (3), we get,
2 g sin q
ar =
1 + sin 2 q
x
Acceleration of block vertically downwards q

ay = ar sin q
2 g sin 2 q ar
ay =
1 + sin 2 q y

\ Acceleration of CM is
ay g sin 2 q
acom = =
2 (1 + sin 2 q )
21. (a, c)
From conservation of linear momentum we can see that velocity of 1 kg block just
after collision is 2 m/s leftwards.

6 m/s
Before collision

2 m/s 4 m/s
After collision

Now relative velocity of approach = 6 m/s


and relative velocity of separation = 6 m/s
Relative velocity of separation
e = =1
Relative velocity of approacch

m1v1 + m2v2 1 ¥ 6 + 2 ¥ 0
Initially, vCM = = = 2 m/s
m1 + m2 1+ 2

During collision vCM will not change.


344പWŚLJƐŝĐƐĨŽƌ//dͲ:͗DĞĐŚĂŶŝĐƐ//

Passages and Matrix Matching


Passage (Q. 1–2)
1. (a) 0 = u1 + 2 ¥ 0.5
\ u = – 1 m/s
1 1
K = m1u12 = ¥ 1 ¥ (-1)2 = 0.5 J
2 2
1 1
2. (b) U = m1u12 + m2u22
2 2
1 1
= ¥ 1 ¥ (-1)2 + ¥ 2 ¥ (0.5)2
2 2
= 0.75 J
Passage (Q. 3–4)
3. (b), 4. (c)
Let vjb be the velocity of jet w.r.t. moving blade, then,
vjb = v – vB
The force exerted by jet on the blade,
Fx = rAv2jb = r A (v – vB)2
The power delivered to the blade
P = Fx vB
= rA vB (v – vB)2
Passage (Q. 8–9)
8. (c) Along the inclined plane
mu sin q = mv cos q
\ u sin q = v cos q
v
fi = tan q
u
Èv -v ˘
Now, e = – Í 2 1˙
Î u2 - u1 ˚
È 0 - (-v sin q ) ˘
=–Í ˙
Î 0 - u cos q ˚
v
e = tan q = (tan q) (tan q)
u
\ q = tan–1 e
9. (c) v = u tan q
= u e
ŽůůŝƐŝŽŶƐപ345

Passage (Q. 10–12)


10. (b) The vertical height of AB is 3 tan 60° = 3 m. The velocity of block before collision
v0 = 2 g ¥ 3 = 60 m/s. Its perpendicular component get absorbed by the plane
while component along the plane
vx = v0 cos 30°
3
= 60 ¥ = 45 m/s
2
1 1
11. (b) m( 45 )2 + mg ¥ 3 = mvc2
2 2
\ vc = 105 m/s
12. (c) The vertical component after collision with the second incline
v = 15 cos 30° – 45 cos 60°

45 45
= - =0
2 2

v0
30°
60° 15
v cos 30°

v sin 30° 15
30° C
3m 3 3m

Passage (Q. 13–15)


13. (c) Use impulse momentum equation
 
P2 - P1 = impulse = F ◊ DT
– 0.25 ¥ 200 + 0.25 ¥ 100 = F ◊ DT ...(i)
Block A
– 1 ¥ v = T ◊ Dt – F ◊ DT ...(ii)
Block B
2 ¥ v = T ◊ Dt ...(iii)
25
Solving we get, v = m/s
3
14. (c) From Eq. (iii)
50
T ◊ Dt = 2 ◊ v = N-s
3
346പWŚLJƐŝĐƐĨŽƌ//dͲ:͗DĞĐŚĂŶŝĐƐ//

15. (d) DKE = Work done by all the forces


1
– (1 + 2)v2 = –(2)(10)h
2
3 25 ¥ 25 625
h = ¥ = = 5.2 m.
4 ¥ 10 3¥3 12 ¥ 10

Passage (Q. 16–18)


16. (b) 17. (c) 18. (d)
The horizontal component of velocities before and after impact are: u cos a eu cos a
respectively. Thus,
d = u cos a t1 and d = eu cos a t2 ...(i)
2u sina
or t1 + t2 = ...(ii)
g
The coordinates of points of impact are (d, d tan q ) so,

gd 2
d tan q = d tan a - ...(iii)
2u2 cos 2 a
From Eqs. (ii) and (iii), we get
tan a
tan q =
(1 + e )
Passage (Q. 19–21)
19. (b) Force required to move the block with constant speed
= frictional force = mN = 0.40 ¥ 25 = 10 N.
20. (d) Let the line of action of N be at a distance d from the centre of the block. Forces
acting on the block are shown in the figure.
The value of N = Weight of the block = 25 N
Taking moment of the forces about point O, we have
10 ¥ 0.125 + 25 ¥ (0.125 – d) = 25 ¥ 0.125
Solving the above equation, we get d = 0.05 m.
0.25 0.25

cg cg
0.5 m P 0.5 m P = 10 N P = 10 N
h 25 N h = 0.125 m 25 N h
N
10 N d O O
(a) (b) (c)

21. (a) See figure (c). To just tip the block the deflecting torque must be equal to the
resisting torque
ŽůůŝƐŝŽŶƐപ347

or torque of P about O = torque of weight about O


or 10 ¥ h = 25 ¥ 0.125
or h = 0.3125 m
Passage (Q. 22–24)
22. (a) The upper block will topple down if lower block displaces through a minimum
distance d as shown in figure. When lower block displaces distance d, the spring
also compresses by d. The energy required to compress the spring will be
1 2 1
kx = kd 2
2 2
Consider the elastic collision between the moving mass m and the lower block B.
Applying conservation of momentum, we have
mV + 0 = mv1 + (2m)v2) ...(i)
(v1 - v2 )
and 1 =– ...(ii)
V -0
where v1 and v2 are the velocities of the mass m and the block B after collision.
Solving the above equations, we get
v1 = V/3 and v2 = 2V/3
The KE of the block B after collision
1 4 mV 2
= (2m)(2V/3)2 =
2 9
This energy will be equal to the energy required to compress the spring (1/2)kd2.
1 2 4 mV 2
\ kd =
2 9
k
or V = V0 = 3 d
8m
23. (b) Given that V = V0/2. The maximum KE of the oscillating particle
1 2
= Max. PE = kA
2
1 2 4 mV 2
\ kA =
2 9

or A = (V0/3) (8m/k )

2m
The time period of motion = 2p
k
24. (d) When block B returns to its initial position, the spring will be in its normal state.
Hence, energy stored in it is zero.
348പWŚLJƐŝĐƐĨŽƌ//dͲ:͗DĞĐŚĂŶŝĐƐ//

Passage (Q. 25–27)


25. (a) Since there is no external force acting on the system in horizontal direction, its
momentum remains constant in this direction or we can also say that velocity of
c.m. remains constant. The velocity of c.m. initially was
mv + M ¥ 0 mv
vc = = = vx
m+ M m+ M
This will be the velocity of larger mass when smaller mass is sliding on vertical
part.
26. (c) Since only gravitational force acts on the system, therefore its mechanical energy
remains constant, i.e.,
1 1 1
mv 2 = m(vx2 + vy2 ) + Mvx2 + mgh
2 2 2
(vy is the vertical component of velocity of smaller block at height h)
Ê mv ˆ
We have vx = Á
Ë m + M ˜¯
2
1 1 1 Ê mv ˆ
\ mv 2 = m(vx2 + vy2 ) + M Á ˜ + mgh
2 2 2 Ë m + M¯
After solving, we get
È ( M 2 + Mm + m2 ) 2 ˘
vx2 + vy2 = Í 2
v - 2 gh ˙
Î ( M + m) ˚
1/2
È ( M 2 + Mm + m2 )v 2 ˘
and vx2 + vy2 = Í
v¢ = 2
- 2 gh ˙
Î ( M + m) ˚
27. (b) Let hmax is the maximum height vy = 0
1 1 1
\ mv 2 = mvx2 + Mvx2 + mghmax.
2 2 2
mv
where vx =
m+ M
Mv 2
After solving, we get hmax =
2 g( m + M )
Matrix Matching
28. A Æ (r); B Æ (p); C Æ (q); D Æ (s)
29. A Æ s; B Æ s; C Æ p, q, r; D Æ p, q, r
(a) not possible, Px cannot be conserved
(b) not possible, Py cannot be conserved
(c) possible, for any mass ratio – p, q, r
(d) possible, for any mass ratio – p, q, r
30. A Æ q, s; B Æ q; C Æ r; D Æ p, r, s
In perfectly inelastic collision bodies combine together after the collision. It is similar
to nuclear fusion process.
3 CHAPTER

Rotation

PARTICLE
A particle can be defined as an object whose mass is finite but the size is negligible small.

WHAT IS A RIGID BODY?


A rigid body is a body within which the distance between any two points does not change
irrespective of the motion of the body. We assume that a real object behaves as a rigid body
even though no perfectly rigid body can exist. In this chapter we will study (i) fixed axis
rotation, for example, hands of a clock, blades of a fan, hinged door, and (ii) combination of
translation and rotation, for example, a rolling wheel, a spinning top.

Translatory Motion: When different particles of a body undergo same displacement, the
motion of the body is called translatory motion.

A A¢

Here AA’ = BB’ = CC’

B B¢

C C¢

Fig. 3.1 Translatory motion

Rotatory Motion: When different particles of a body undergo same angular displacement,
the motion of the body is called rotatory motion. But different particles have different linear
displacements.
If angular displacement traversed by the particles A, B and C on the body in time t is q,
then
350പWŚLJƐŝĐƐĨŽƌ//dͲ:͗DĞĐŚĂŶŝĐƐ//

q
wA = wB = wC = A
t
while linear velocity q
AA¢ 0
vA = = =0
t t B’
B
BB¢ q rB C’
vB = = = w rB
t t
C
CC ¢ q rC
and vC = = = w rC
t t
It is clear that, rA < rB < rC Fig. 3.2 Rotatory motion
so vA < vB < vC

Combined Translation and Rotation: The combination of the above two motions results
in combined translation and rotation.

A
A
q

B’
B
C’

Fig. 3.3

EQUATIONS OF MOTION OF A ROTATING BODY


First Equation of Motion of Rotating Bodies
Let us consider a rigid body rotating about a fixed axis with constant angular acceleration
a. By definition
dw
=a
dt
or dw = adt
If at t = 0, w = w0, then the angular velocity of the body at any time t is given by using
integration
w t

Ú dw = Ú a dt
w0 0

w t
w w0
=a t 0
ZŽƚĂƟŽŶപ351

or w – w0 = a(t – 0)
or w = w0 + at ...(1)

Second Equation of Motion of Rotating Body


By definition, we have
dq
=w
dt
or dq = wdt
If at t = 0, q = 0, then angular displacement in time t is given by
q t t

Ú dq = Ú w dt= Ú (w 0 + a t) dt
0 0 0
t
q 1
or q 0
= w0t + a t2
2 0
1
or q = w0t + a t2 ...(2)
2

Third Equation of Motion


The angular acceleration can be expressed as by definition
dw dw dq SI units:
a = = ◊
dt dq dt SI unit of angular displacement is radian.
dw SI unit of angular velocity is rad/s.
or a = w SI unit of angular acceleration is rad/s2.
dq
Note:
or wdw = adq p
1 rpm = rad/s
Integrating the above equation, we get 30
w q

Ú w dw = Ú a dq
w0 0

w
w2 q
or =a q 0
2 w0

or 2
w – w02 = 2aq
or w2 = w02 + 2aq ...(3)
For uniformly retarded motion, these equations become
w = w0 – at
1
q = w0t – at 2
2
2 2
and w = w0 – 2aq.
352പWŚLJƐŝĐƐĨŽƌ//dͲ:͗DĞĐŚĂŶŝĐƐ//

WHAT HAPPENS WHEN WHEEL


IN CONTACT ROTATES? r2 w2
w1
Consider two wheels of radii r1 and r2 are rotating in r1
contact without slipping. If v1 and v2 are the linear
velocities of their points of contact, then v1 = v2
v1
or w1r 1 = w 2r 2
v2
If r1 = r and r2 = 2r,
then w1 = 2w and w2 = w. Fig. 3.4

WORKED PROBLEMS
Problem 3.1 The angular speed of a truck wheel is increased from 1200 rpm to 3120 rpm
in 16 seconds.
(i) What is its angular acceleration, assuming the acceleration to be uniform?
(ii) How many revolutions does the wheel make during this time?

1200
Solution Given, w0 = 2pn0 = 2p ¥
60
= 40p rad/s
3120
w = 2pn = 2p ¥
60
= 104 p rad/s
(i) Angular acceleration
w - w 0 104p - 40p
a = =
t 16
= 4p rad/s2 Ans.
(ii) Angular displacement can be obtained as
1 2
q = w 0t + at
2
1
= 40p ¥ 16 + ¥ 4p ¥ (16)2
2
= 1152 p rad
Number of revolutions in 16 s
q 1152p
= =
2p 2p
= 576 Ans.
Problem 3.2 The angular acceleration of a heavy wheel is given by a = 12 – t where a is
in rad/s2 and t in second. If the angular velocity of the wheel is 60 rad/s at the end of 4
seconds, determine the angular velocity at the end of 6 seconds. How many revolutions take
place in these 6 seconds?
ZŽƚĂƟŽŶപ353

Solution Given that a = 12 – t


dw
i.e., = 12 – t
dt
or dw = (12 – t)dt
Integrating the above equation, we get

Ú dw = Ú (12 – t)dt
t2
or w = 12t – +C
2
At t = 4 s, w = 60 rad/s
42
\ 60 = 12 ¥ 4 – +C
2
or C = 20
t2
\ w = 12t – + 20 ...(i)
2
62
At t = 6 s, w = 12 ¥ 6 - + 20
2
= 74 rad/s Ans.
Now we can write 2
dq t
= 12t - + 20
dt 2
Integrating the above equation, we get
t2
Ú dq = Ú (12t - 2
+ 20)dt

12t 2 t 3
or q = - + 20t + C ¢
2 6
Let t = 0, q = q0, \ C¢ = q0
Thus, we have
t3
q = 6t2 – + 20t + q 0
6
t3
or q – q0 = 6t2 – + 20t ...(ii)
6
Angular displacement during 6 s
63
q6 – q 0 = 6 ¥ 6 2 – + 20 ¥ 6
6
= 300 rad
300
\ Number of revolutions = = 47.8 Ans.
2p
354പWŚLJƐŝĐƐĨŽƌ//dͲ:͗DĞĐŚĂŶŝĐƐ//

Problem 3.3 The pulley block system shown in Fig. 3.5 starts from rest and accelerates at
2 rad/s2. What time is required for block A to move 20 m? Find also the velocity of A and
B at that time.
Solution When A moves 20 m, its angular displacement q is given 1m
by 0.75 m
l = rq
l 20
or q == = 20 rad
r 1
Given, a = 2 rad/s2 and w0 = 0.
By the second equation of motion, we have
1
Earlier proved q = w0t + = at 2
2 A
1
20 = 0 + ¥ 2 ¥ t 2
2
or t = 4.47 s Ans. B
Angular velocity of pulley at this time
w = w0 + at Fig. 3.5
= 0 + 2 ¥ 4.47 = 8.94 rad/s Ans.
Now velocity of A, vA = wrA = 8.94 ¥ 1
= 8.94 m/s
and vB = wrB = 8.94 ¥ 0.75
= 6.70 m/s Ans.
Problem 3.4 The angular rotation in radians of an accelerated flywheel is given by equation
q = 9t3/32. Find the linear velocity and acceleration of a point at a distance of 0.75 m from
the axis of rotation at the instant when its tangential acceleration and normal acceleration
are equal.
Solution We have q = 9t3/32
dq Ê 9 ˆ 2
Formula of angular velocity w = = Á ˜ 3t
dt Ë 32 ¯
= 27t2/32
dw
and angular acceleration a =
dt
= 27t/16
Tangential acceleration of a point at a radius r is given by
at = ar
and the normal acceleration is given by an = w2r
Since the tangential and normal accelerations at a distance r = 0.75 m are equal, so
ar = w2r
ZŽƚĂƟŽŶപ355

or a = w2
2
27 t Ê 27 t 2 ˆ
or =
16 ÁË 32 ˜¯
which gives t = 4/3 s
Ê 27 t 2 ˆ 2
Ê 27 ˆ Ê 4 ˆ
Linear velocity = w r = Á ˜ r = Á ˜ Á ˜ ¥ 0.75
Ë 32 ¯ Ë 32 ¯ Ë 3 ¯
or = 9/8 m/s Ans.
2 2 ˘2
È 27 Ê 4 ˆ
Ê 27 t 2 ˆ
Tangential acceleration = w2r = Á ˜ r = Í Á ˜ ˙ ¥ 0.75
Ë 32 ¯ ÍÎ 32 Ë 3 ¯ ˙˚
= 7/16 m/s2 Ans.
Problem 3.5 In a mechanism of gears load A rotates a pulley of radius r and a gear wheel
of radius r1. This gear wheel is geared with a second gear wheel of radius r2. If the load
starts from rest and moves down with a constant acceleration, find the equation of the
second gear wheel.
Solution Since load A starts moving with constant
acceleration a, its velocity at time t, r1

v = at as u=0 r2
w1
Let w1 be the angular velocity of the pulley and the w2
r
first gear wheel.
\ Peripheral velocity of the pulley = w1r = at
w1 = at/r
Peripheral speed of the first gear wheel w1r1.
Since the two gear wheels have the same peripheral
velocities, so
w 2r 2 = w 1r 1 A
at r1 atr1
or w2 = w1r1/r2 fi ◊ = Fig. 3.6
r r2 rr2
= at r1/(rr2)
Let q be the angular displacement of the second gear at time t.
\ w2 = dq/dt = at r1/(rr2)
or dq = at r1/(rr2) dt
Integrating the above equation, we get
q = at2 r1/(2rr2) Ans.
Problem 3.6 The composite pulley shown in Fig. 3.7 weighs 800 N and has a radius of
gyration of 0.6 m. The 2000 N and 4000 N blocks are attached to the pulley by inextensible
strings as shown in figure. Determine the angular acceleration of the pulley and the tension
in the strings. Neglect the weight of the string.
356പWŚLJƐŝĐƐĨŽƌ//dͲ:͗DĞĐŚĂŶŝĐƐ//

Solution
0.75 m

0.5 m

TA TB

2000 N
TA B
4000 N A 2000 N

4000 N
(a) (b) FBD

Fig. 3.7

Since the moment of 4000 N is more than that of 2000 N block about the axis of rotation
of pulley, therefore the pulley rotates in anticlockwise direction. Let aA be the acceleration
of block A, aB that of B and a is the angular acceleration of the pulley, then
aA = a ¥ 0.5, aB = a ¥ 0.75
Equations of motion are
4000 – TA = (4000/9.81)aA ...(i)
TB – 2000 = (2000/9.81 )aB ...(ii)
and for pulley
4000 ¥ 0.5 – 2000 ¥ 0.75 = la ...(iii)
where I = mpulley ¥ k = (800/9.81) ¥ 0.6
2 2

= 29.36 kg-m2
Solving the above equations, we get
a = 2.03 rad/s2, TA = 3585.58 N, TB = 2310.82 N Ans.
Problem 3.7 In Fig. 3.8 the mirror of weight W is initially at the same
level as a monkey of equal weight. The fixed pulley is massless and has
a radius R. If the cat starts running up the rope, can he get away from
the mirror?
Solution Both sides of the pulley, the weight suspended are equal, so
net torque acting on the centre of the pulley will be zero.
Hence, the angular momentum about this point is constant. Since W
the initial angular momentum of the system in equilibrium is zero, so it W
must remain same throughout. When the monkey starts running up the
rope, let its velocity at any instant be v. This gives an angular momentum
(W/g) vR about O. Thus, for the total angular momentum to be zero, the Mirror
mirror must also have the same instantaneous velocity. In other words, Fig. 3.8
the monkey cannot get away from his mirror image.
Problem 3.8 A man of mass m stands on a horizontal platform in the shape of a disk of
mass M and radius R, pivoted on a vertical axis through its centre about which it can freely
rotate (Fig. 3.9). The man starts to move around the centre of the disk in a circle of radius r
with a velocity v relative to the disk. Calculate the angular velocity of the disk.
ZŽƚĂƟŽŶപ357

Solution Since there is no torque acting about the axis of rotation


of the disk, so the angular momentum of the system (disk + man)
remain constant. Initially, it is zero. Suppose w is the angular v
r
velocity of the disk (take positive in the sense of motion of the R
man). The velocity of the man with respect to the ground observer
will be wing relative motion theory as earlier explained
  
[v man ]disk = [v man ]ground - [v disk ]ground
  
\ [v man ]ground = [v man ]disk + [v disk ]ground

or [v man ]ground = v + w r Fig. 3.9

Thus, angular momentum of the man = m(v + wr)r.


MR2
And angular momentum of the disk = Idisk w = w
2
By conservation of angular momentum, we have

MR2
0 = m(v + wr)r + w
2
After solving, we get
mvr
w =– Ans.
Ê 2 MR2 ˆ
Á mr + 2 ˜
Ë ¯

Problem 3.9 Three links are hinged together to form a triangle B


ABC as shown in Fig. 3.10. At a certain instance, point A is
moving towards the mid-point of BC with a velocity of 5 m/s
and B is moving at a perpendicular direction to AC. Find the
velocity of C. A
45° 60°
C
Solution Given : Velocity of A (vA) = 5 m/s Fig. 3.10
First of all, let us locate the position of instantaneous centre of
the point A and B graphically as shown in Fig. 3.11 and as discussed below:
1. Draw the triangle ABC with the given vB
data.
2. Now draw the lines indicating the B
O
directions of motion of points A (towards
mid-point of BC) and B (at right angles
to AC).
3. Now draw perpendiculars at A and B on vB
directions of motion of vA and vB. vA
60°
4. Let these perpendicular meet at O, which 45°
is the instantaneous centre of the link AB A C
and BC. Fig. 3.11
5. Now join OC and draw a line at right
angle to OC indicating the direction of motion of the point C.
358പWŚLJƐŝĐƐĨŽƌ//dͲ:͗DĞĐŚĂŶŝĐƐ//

Measuring the diagram to some scale, we find that OA = 2.6 cm and OC = 5.4 cm
vC OC 5.4
We know that = = = 2.08
vA OA 2.6
\ Velocity of C,
vC = vA ¥ 2.08 = 5 ¥ 2.08 = 10.4 m/s Ans.
Problem 3.10 Wheel A has radius 10 cm is coupled by
a belt B to wheel C of radius 30 cm as shown in Fig. 3.12.
Wheel A increases its angular speed from rest at a uniform A C
rate of 1.57 rad/s2. Determine the time for wheel C to reach
a rotational speed of 100 rev/min assuming that the belt
does not slip.
B
Solution As the belt does not slip vA = vC
Fig. 3.12
i.e., r Aw A = r Cw C [as v = rw]
Here, rA = 10 cm, rC = 30 cm
and wC = (2p ¥ 100/60) rad/s
2p ¥ 100
So, 10wA = 30 ¥
60
i.e., wA = 10p rad/s
Now, from the 1st equation of rotational motion, i.e.,
w = w0 + at
w
t = [as w0 = 0]
a
10 ¥ p
i.e., t = = 20 sec.
1.57

TORQUE OR MOMENT OF FORCE


Figure 3.13 shows a bolt being opened with the help of a wrench. The amount of turn the
bolt gets depends on the magnitude of force, direction of force and point of application of
force. No turning effect is produced when force F passes through the centre of bolt (Fig.
3.13). Turning effect is increased by increasing q and r and decreased by decreasing q and r.
Turning effect of force is described by a vector torque. Position vector from point O on the
axis of rotation to the point of application of force is called moment arm. The moment of the
force or torque about O is defined as the product of F and the moment arm.
t = Fr^ = Fr sin q
Alternatively resolve F into its components (F||, parallel to r and F^, perpendicular to r).
Since F|| passes through O, its moment arm is zero, hence its torque is zero.
r = rF^ = rF sin q
rF sin q is magnitude of torque. Torque is a vector and expressed as
  
t=r ¥F
ZŽƚĂƟŽŶപ359

To find the direction
 of the torque, curl the fingers of the right hand from r through the
  
smallest angle to F. The upwardly extended thumb then points in the direction of t = r ¥ F .
F^ = F cos q O
r^ = r sin q
q O
q r

q r F^ = F sin q
Rotation
axis F
F F
(a) (b)
r r r
t = r ¥F

r
F
q Dimension of torque is = ML2 T–2
r
r

(c)

Fig. 3.13
Here anticlock-
WORKED PROBLEMS wise torque is F3^ F3
taken +ve f
Problem 3.11 Figure 3.14 shows three forces F1, F2 and F3 acting F2^
F2
on a rod pivoted at its end. Find the torque of each force about
pivot.
+
Solution In order to determine the sign of each torque, imagine
which way the rod would rotate if the given force were the only q
force acting. Generally, anticlockwise direction is considered to be r1^
F1
positive, but this choice is arbitrary.
t = rF sin q = F ¥ r^ = r ¥ F^ Fig. 3.14
In order to determine magnitude of torque we may choose the expression F ¥ r^ or r ¥
F^ according to convenience, where r^ is component of moment arm perpendicular to force
and F^ is component of force perpendicular to moment arm.
t1 = – r1F1 sin (90° + q) = – r1 F1 cos q
t2 = + r2F2 sin (180° – a) = + r2 F2 sin a
t3 = + r3F3 sin (90° – f) = + r3 F3 cos f
Important:
Torque can be taken clockwise or anticlockwise as + ve depends upon you.
Concept
1. Figures, 3.15 (a), (b) and (c) show a pivoted rigid body. Tangential component Ft
produces torque, normal component Fn produces zero torque as it passes through the
pivot.
360പWŚLJƐŝĐƐĨŽƌ//dͲ:͗DĞĐŚĂŶŝĐƐ//

y 
F
Ft = F sin f
f

r Fn

Pivot
(a)

y  Always measure angle


 F f from radial line in
F exerts a torque f anticlockwise direction
and causes
rotation Point where
force is applied
Fn

x
Rigid body
Pivot point

(b)

 F2
F5 exerts zero torque
exerts maximum 
F3
torque in exerts a clockwise
anticlockwise torque
sense 
F4
exerts maximum torque
in clockwise sense


F1 Rigid body
A force applied at pivot
exerts zero torque
(c)

Fig. 3.15
axis of rotation
   A
2. The quantity r ¥ F is torque of F about O. When we say torque about
 
axis of rotation, it means axial component of vector r ¥ F along OA.
Torque of a force about axis OA is independent of the choice of origin
r P r
O, but it must lie on the axis. F
 O1
r1
Torque of F about O1 is r
     r2
r
O1P ¥ F = (r2 + r ) ¥ F O r
   
= r2 ¥ F + r ¥ F
  
Vector r2 ¥ F^ + r2 ; therefore it will not have any component along B
   
AB. Thus, component of O1P ¥ F and O1P ¥ F along axis is same. Fig. 3.16
ZŽƚĂƟŽŶപ361

  
3. If force F is parallel to axis of rotation, then r ¥ F is perpendicular to axis. Component

of r ¥ F along the axis is zero.
axis of rotation axis of rotation
A A
r
F

r r P
r t t
r r
r r
r r
F r
F O

B B
(a) (b)

Fig. 3.17

4. If line of action of F intersects the axis, the point of intersection is our origin. Torque

about O is r ¥ F = 0, therefore its component along the axis will be zero.
axis of rotation
A

r
r^
P
r
O rr F

Fig. 3.18

5. If force F is skew with the axis (line
 of action of F is perpendicular but non-intersecting
with the axis), then torque of F has magnitude
 
|t |=|F|.(r^)
where r^ is moment arm.

Moment Arm
It is the perpendicular distance from axis of rotation to the line of action of the force. This
is r sin q.
  
We know that t = |t |=|r ¥ F|
362പWŚLJƐŝĐƐĨŽƌ//dͲ:͗DĞĐŚĂŶŝĐƐ//

Fr sinq
Moment arm, r sin q =
F
 
|r ¥ F|
or Moment arm = 
|F|

Torque in Terms of Rectangular Components


   
If F and r are given as: F = Fx iˆ + Fy ˆj + Fz kˆ and r = x ˆi + yˆj + zkˆ
  
then torque can be defined as: t =r ¥F
= (x î + y ĵ + z k̂ ) ¥ (Fx î + Fy ĵ + Fz k̂ )
ˆi ˆj kˆ
= x y z
Fx Fy Fz

or t = t x ˆi + t y ˆj + t z kˆ = ˆi( yFz - zFy ) + ˆj( zFx - xFz ) + kˆ (xFy - yFx )
Thus, we have,
tx = yFz – zFy; ty = zFx – xFz; tz = xFy – yFx.

COUPLE
F
Two equal and opposite forces whose lines of action are different
constitutes a couple. The moment of couple can be found by A
taking moments of both the forces about any axis perpendicular O
B
to plane of forces and adding them algebraically.
Thus, moment of couple F
t = F ¥ AO + F ¥ OB d

= F (AO + OB) Fig. 3.19


= Fd
or t = Force ¥ perpendicular distance between the lines of action of forces.

NUMERICAL QUESTIONS
Q.1. Evaluate torque about pivot A provided by the forces shown in Fig. 3.20.
Ans. Torque of 90 N and 50 N force is zero, because their lines of action pass through the
point about which torque is required.
90 N 70 N
80 N
30°

60° 37°
50 N
A L/2 L/2

60 N
Fig. 3.20
ZŽƚĂƟŽŶപ363

Now, torque of remaining forces


L L
= ¥ 80 ¥ sin 37 + 70 ¥ L ¥ sin 120 – 60 ¥
2 2
L 3 3 60 L
= ¥ 80 ¥ + 70 ¥ L ¥ - = 54.6 L
2 5 2 2
Q.2. In Fig. 3.21 four forces are shown. Calculate the torque of forces about point O.
27 N
10 N
4m 4m
2m
O
2m

2N 20 N

Fig. 3.21

Ans. t = 2 ¥ 4 + 20 ¥ 2 ¥ sin 45 = 36.3 N


Q.3. Two parallel forces of magnitude 6 N and 10 N are 5 m apart. Calculate their resultant.
Ans. R = 6 + 10 = 16 N 5m
Also, if distance of resultant from 6 N is x x 5–x
Then, 6x = 10 (5 – x) 6N 10 N
R
50
fi x = m
16 Fig. 3.22
Q.4. A uniform stick of length 10 m carries two loads of 10 B
kg and 40 kg at its two ends. In order to keep the stick 10 – x x
horizontal, where it should be supported? 10 kg 40 kg
Ans. The force must pass through the balance point B.
S
10 (10 – x) = 40(x)
Fig. 3.23
100 = 50 x
or x = 2 m from 40 kg
Q.5. A uniform rod of length 2 m carries a load of 20 kg and 60 kg at its two ends. This
rod is to be suspended by a string to keep it horizontal. Where the string should be
attached?
Ans. 1.5 m from 20 kg load.
Q.6. A child weighing 400 N sits on the end of a see-saw that is 3 m long and is pivoted
1.4 m from the child. If another child sitting at another end just balances, what is his
weight?
Ans. F 1r 1 = F 2r 2
400 ¥ 1.4 = w(3 – 1.4) or w = 350 N
Q.7. A heavy electric motor is to be carried by two men by placing it on a light board 2 m
long. To lift the board, one man lifts one end with a force of 600 N and the other at
the opposite end with a force of 400 N. What is the weight of the motor, and where is
it located?
364പWŚLJƐŝĐƐĨŽƌ//dͲ:͗DĞĐŚĂŶŝĐƐ//

Ans. For equilibrium w and resultant R of forces 600 N and 400 N must have same magnitude,
opposite directions and should pass through the same point.
R = 600 + 400 = 1000 N = w
Now, let R be at distance x from 600 N force.
Then, 600 ¥ x = 400(2 – x) or x = 0.8 m
Q.8. A uniform rod acted upon by various forces is in equilibrium. Evaluate the unknown
forces.
Ans. For equilibrium, Â t = 0 about any arbitrary point.

20 N F2

4d 2d d d

10 N F1 30 N

Fig. 3.24

Taking torque about the another end we have


20 ¥ 8d + 10 ¥ 4d + F1 ¥ 2d + 30 ¥ d = 0
or – 160d + 40d + 2 F1d + 30d = 0
or F1 = 45 N
Also, ÂF ≠ = ÂF Ø
\ 20 + F2 = 10 + F1 + 30
or 20 + F2 = 10 + 45 + 30
\ F2 = 65 N
Q.9. A uniform metre stick of mass 200 g is suspended from the ceiling through two vertical
strings of equal lengths fixed at the ends. A small object of mass 20 g is placed on the
stick at a distance 70 cm from the left end. Find the tension in the two strings.
Ans. The situation with forces is shown in Fig. 3.25.

T1 T2

50 cm 20 cm 30 cm
200 g 20 g

Fig. 3.25

For equilibrium, Ât = 0.
Calculating torque about the left end,
200 ¥ 50 + 20 ¥ 70 + T2 ¥ 100 – 10000 – 1400 + 100 T2 = 0
or T2 = 114 g.
ZŽƚĂƟŽŶപ365

Also,
ÂF ≠ = Â F Ø
T1 + T2 = 200 + 20
T1 = 200 + 20 – T2 = 200 + 20 – 114 = 106 g
Q.10. A 60 kg person is walking along a level bridge and stops three-fourths of the way from
the end. The bridge is uniform and weighs 600 kg. What are the values of vertical
forces exerted on each end of the bridge by its supports?
Ans. Taking moment about A

2d d d
A B
n1 n2
600 kg 60 kg

Fig. 3.26

600 ¥ 2d + 60 ¥ 3d + n2 ¥ 4d = 0
or – 1200 – 180 + 4n2 = 0
or n2 = 345 kg
Again,
n1 + n2 = 600 + 60
n1 + 345 = 660
\ n1 = 315 kg
Q.11. A diver of weight 600 N stands at the end of a uniform 6 m diving board of weight 150
N. The board is attached to two pedestals 1.5 m apart. Find tension (or compression)
in each pedestals.
Ans. Calculating moment about A.
T2 ¥ 1.5 + 150 ¥ 3 + 600 ¥ 6 = 0
or T2 = 2700 N
T1 T2
1.5 m 1.5 m 3m

A B

150 N 600 N

Fig. 3.27

Now,
T1 + T2 = 150 + 600
or T1 = 150 + 600 – T2 = 150 + 600 – 2700 = – 1950 N
The (– ve) sign shows that this force is tension.
366പWŚLJƐŝĐƐĨŽƌ//dͲ:͗DĞĐŚĂŶŝĐƐ//

Q.12. A uniform plank 15 m long, weighing 400 N rests symmetrically on two supports 8 m
apart. A boy weighing 640 N starts at point A and walks towards the right. Calculate
the upward forces FA and FB acting on the plank at points A and B, when boy is at
distance x from the support A.
Ans. Taking moment about A.
400 ¥ 4 + 640 ¥ x + FB ¥ 8 = 0
fi FB = 200 + 80x

FA FB
x

640 N
400 N
A

3.5 m 4m 4m 3.5 m

Fig. 3.28

Again,
FB + FA = 400 + 640
FA = 400 + 640 – (200 + 8x) = 840 – 80x
Q.13. In the above question plot the graph of FA vs x and FB vs x.

Ans.

840 840

200 200
O 8 10.5 x O 8 x

(a) (b)

Fig. 3.29

WORKED PROBLEMS
Problem 3.12 Find the tension in the two threads as shown in Fig. 3.30. Rod of mass M.

L/4

L/8
Rod of mass M M

Fig. 3.30
ZŽƚĂƟŽŶപ367

Solution The rod is in equilibrium hence, net force will be zero


T1 + T2 = 2Mg ...(1)
Torque about point P is taken and equated to zero M; L

L Ê 7Lˆ Ê 3L ˆ
Mg + Mg Á ˜ = T2 Á ˜ ...(2) T1 T2 L/4
2 Ë 8¯ Ë 4¯
P T L/8
11 Mg
fi T2 = Mg T = Mg
6 M
And from Eq. (1)
11
T1 = 2Mg – Mg Fig. 3.31
6

Problem 3.13 A rod of mass M and length l is supported by two threads. Find the tension
in each thread as shown in Fig. 3.32. (BITSAT, 2011)

M; l
l
4

Fig. 3.32

Solution Rod is in equilibrium


\ T1 + T2 = Mg
T1 T2
Since the rod is in rotational equilibrium torque about any
point is zero. l
mg
Taking torque about A. Due to T1 torque about this point is 4
zero.
Due to mg and T2 torques are opposite. Fig. 3.33

l Ê 3l ˆ 2 Mg
\ Mg = T2 Á ˜ Mg fi T2 =
2 Ë 4¯ 3
Mg
From the above Eq. (1) T1 =
3
Problem 3.14 A system of masses is shown in Fig. 3.34. Find tensions T1 and T2.

T1 l l T
2
4 4

Mg
2Mg M

Fig. 3.34
368പWŚLJƐŝĐƐĨŽƌ//dͲ:͗DĞĐŚĂŶŝĐƐ//

Solution Due to linear equilibrium


T1 + T2 = 4Mg ...(1)
The net torque about any point is zero.
Taking torque about point A. The torque due to T1 will become zero.
Torque due to 2M, M and M will add and would be equal to torque due to T2.
2 Mgl Mgl Ê 3l ˆ
+ + Mg Á ˜ = T2 l
4 2 Ë 4¯
7 Mg
\ T2 =
9 Mg 4
\ T1 from Eq. (1) =
4
Problem 3.15 Figure 3.35 shows two blocks, each of mass m, suspended from the ends of
a rigid weightless rod of length l1 + l2, with l1 = 20 cm and l2 = 80 cm. The rod is held in
horizontal position shown in figure and then released. Calculate the accelerations of the two
blocks as they start to move.

l1 l2
+
Here clockwise torque
is taken +ve

m m

Fig. 3.35

Solution Net torque, tnet = mgl2 – mgl1


If a is the angular accelerations, then
t net
a =
I
mg(l2 - l1)
=
ml12 + ml22
g(l2 - l1)
=
l12 + l22
9.8(0.8 - 0.2)
=
0.22 + 0.8 2
= 8.65 rad/s2
\ a1 = a l1
= 8.65 ¥ 0.2
= 1.73 m/s2
a2 = a l2
= 8.65 ¥ 0.8
= 6.92 m/s2
ZŽƚĂƟŽŶപ369

Problem 3.16 Find moment of force about the axis passing through the origin and

perpendicular to the plane of F and r.

10 N

60°
30°
10 N
1m 1m

30° 30°
x
z–axis z–axis
(a) (b)

Fig. 3.36

Solution Method I:

(a) t = Fr sin q k̂
= 10 ¥ 1 ¥ sin 60° k̂ N-m
= 5 3 k̂ N-m

(b) t = Fr sin q (– k̂)
= 10 ¥ 1 ¥ sin 30°(– k̂) N-m
= – 5 k̂ N-m
Method II:
 Ê 3 ˆ 1 ˆˆ
r = 1cos 30° î + 1sin 30° ĵ = Á i + j˜ m
  Ë 2 2 ¯
F = 10 j N
   Ê 3 ˆ 1 ˆˆ
t = r¥F ¥ Á i + j˜ ¥ 10 ˆj = 5 3kˆ N-m
Ë 2 2 ¯

Hint: Always use vector method for solving these type of problems.
Problem 3.17 Figure 3.37 shows a compound wheel A
vA
which rolls without slipping. If the velocity of the centre
O is 1.25 m/s. Find the velocities of the points A, B and C. 0.3 m

Solution Since the wheel rolls without slipping on the v0


B
supporting surface MN, the point of contact D is the O v0
0.15 m
instantaneous centre of rotation. Let w be the angular
M N
velocity about the instantaneous centre D. D
0.15 m
Velocity of O = v0 = w ◊ DO vC C
Velocity of A = vA = w ◊ DA
Velocity of B = vB = w ◊ DB Fig. 3.37

Velocity of C = vC = w ◊ DC
370പWŚLJƐŝĐƐĨŽƌ//dͲ:͗DĞĐŚĂŶŝĐƐ//

We have DO = 0.15 m, DA = 0.45 m


DB = (0.32 + 0.152 ) = 0.335 m
and DC = 0.15 m
\ V0 = 1.25 m/s
1.25 0.15
1.25: vA : vB : vC = (0.15) : (0.45) : (0.335) : (0.15) =
va 0.45
vA = 1.25 ¥ 0.45/0.15 = 3.75 m/s
vB = 1.25 ¥ 0.335/0.15 = 2.80 m/s
vC = 1.25 ¥ 0.15/0.15 = 1.25 m/s Ans.

WHAT IS INSTANTANEOUS AXIS OF ROTATION?


At any instant it is possible to locate a point in the plane which has zero velocity and hence
plane motion of other points may be looked as pure rotation about this axis. Such point is
called instantaneous centre and the axis passing through this point and right angles to the
plane of motion is called instantaneous axis of rotation.

More about Instantaneous Axis of Rotation


Motion of such an object may be looked as pure rotation about a point has zero velocity to
simplify the study. Such a point is called instantaneous centre and the axis passing through
this point and perpendicular to the plane of motion is called Instantaneous Axis of Rotation
(IAOR).
In Fig. 3.38 I1 and I2 are the instantaneous centres of rotation y
of the rod at two different instants. I1
To understand instantaneous centre consider a rigid body
which has plane motion. Let A be a point having velocity vA at I2
the instant considered. Now locate a point I on perpendicular A
to the direction of vA at A at a distance rA. The motion of A
can be split into translation of I and rotation about I. Thus,
B
we can write x
v A = v I + w r A. Fig. 3.38
vA vA
if we take rA = , then vA = vI + w ¥ I
w w
\ vI = 0
v
Thus, point I is selected at a distance A along the
w rB
perpendicular to the direction of A, the plane motion of point rA
A can be reduced to pure rotation about I. Hence, I is the
instantaneous centre.
If B is any other point on the rigid body then its velocity will
B
be given by vB
vB = vI + wrB vA
A
or vB = wrB (vI = 0)
Fig. 3.39
ZŽƚĂƟŽŶപ371

There are two methods of locating instantaneous centre.


Instantaneous centre can be located by any of the following two methods:
(i) If the angular velocity w and linear velocity vA are known, instantaneous centre can
be located at a distance vA/w along the perpendicular to the direction of vA at A, as
discussed earlier.
(ii) If the linear velocities of two points of rigid body are known, say vA and vB, drop
perpendiculars to them at A and B. The intersection point is the instantaneous centre
(see Fig 3.38).

WORKED PROBLEM
Problem 3.18
(a) A wheel of radius 1 m is acted upon by the forces as shown in Fig. 3.40. Find the
resultant moment of force about an axis passing through the centre of the wheel.

y 10 2 N y 10 2 cos 45∞
45°
10 2 sin 45∞
10 N
1m 10 N

1m 1m
O O

20 N 20 N
x x
z–axis z–axis

(a) (b) Equivalent system

Fig. 3.40

(b) Calculate moment of system of forces about an axis passing through B and perpen-
dicular to the plane of the forces.
Solution 10 2 N
40 N
(a) Moment of force of 20 N and 10 cos 45° N is zero about 1m
O because their moment arm is zero. D C
The net moment of force

t = 10 ¥ 1 ¥ k̂ + (10 2 sin 45°) ¥ 1 ¥ (– k̂) 50 N
1m
= 10 k̂ – 10 k̂ = 0 Ans.
0.4 m
(b) Moment of forces about B, 20 N
tB = 20 ¥ 0 + 40 ¥ 0 + 10 2 ¥ 0 + 50 ¥ 0 + 10 ¥ 1 B
10 N
= 10 N-m clockwise.
Fig. 3.41
Hint: Always use vector method to solve these types of
problems.
372പWŚLJƐŝĐƐĨŽƌ//dͲ:͗DĞĐŚĂŶŝĐƐ//

WHAT IS MOMENT OF INERTIA OR ROTATIONAL INERTIA?


Moment of inertia of a body is a measure of its ability to resist change in state
of rotation. It plays the same role in rotational motion what inertia plays in
m
translational motion. Mathematically, moment of inertia of a particle of mass r
m about an axis is defined as
I = mr2
Fig. 3.42
where r is the distance of particle from axis of rotation.
It is called a tensor quantity. Its SI unit is kg-m2.

Moment of Inertia of System of Particles


Consider a system of n particles as shown in Fig. 3.43. Its
m3 r3
moment of inertia is given by m2
r2
rn
I = m1r12 + m2r22 + .... + mnrn2 r1 m1
mn
n
or I = I = Â mi ri 2
i =1 Fig. 3.43

z
Moment of Inertia of a Rigid Body
Consider a body rotating about an axis as shown
in Fig. 3.44. Choose a small element of mass
dm at a distance r from the axis, its moment of y
inertia about the axis of rotation (here z-axis)
r dm
dI = dm r2
The moment of inertia of the whole body can
be obtained by
x
I = Ú dmr 2 Fig. 3.44
Limits of integration depends on the shape of the body.
The moment of inertia of a body depends on the following factors:
(i) Mass of the body.
(ii) Size and shape of the body.
(iii) Position and orientation of axis of rotation.
Important:
m
(a) Moment of inertia of a particle about an axis passing through the particle
itself will be zero.
(b) Any body can have infinite numbers of moment of inertia. Fig. 3.45
ZŽƚĂƟŽŶപ373

WHAT IS RADIUS OF GYRATION?


Consider a system of particles or a rigid body rotating about the axis as shown in Fig. 3.46.
Its moment of inertia about the axis is given by
I = m1r12 + m2r22 + .....+ mnrn2
Now suppose the whole mass of the system M is concentrated m3 r3
at a point and placed at a distance k from the axis, then the r2 m2
moment of inertia of the equivalent system can be defined as, rn
I = MK2 r1 m 1
mn
If Mk2 = m1r12 + m2r22 + ....+ mnrn2 = I,
(a) Actual system
then k is called radius of gyration and can be written as

I
k= or I = Mk2
M
M
Thus, radius of gyration of any system about its axis of rotation k
may be defined as the distance from the axis of rotation, if square
of it is multiplied by total mass of the system it gives moment of
inertia of the system about the axis. Radius of gyration depends (b) Equivalent system
on position and direction of axis of rotation and distribution of
Fig. 3.46
mass about axis of rotation.

IMPORTANT THEOREMS OF MOMENT OF INERTIA A CM


ͳǤ౧ƒ”ƒŽŽ‡Žš‹•Ї‘”‡
x m
Consider a body, whose moment of inertia about an axis passing d
through CM is ICM. Let m be the mass of the particle at a distance
x from the axis of rotation, then the moment of inertia of whole
body about CM axis
ICM = Â mx2 ...(i) Fig. 3.47

Choose an axis A parallel to the CM axis, and the separation between the axes is d. The
moment of inertia of the body about axis A,
IA = Â m(d + x)2
=Â m(d2 + x2 + 2xd)
= Â md2 + Â mx2 + 2 Â mxd ...(ii)
Here  md2 = Md2,  mx2 = ICM and  mx is the moment of total mass about the CM

axis which will be zero, i.e., Â mx = 0.


\ IA = ICM + Md2 ...(1)
Thus, the moment of inertia of a body about any axis is equal to its moment of inertia
about a parallel axis through its centre of mass plus the product of the mass of the body and
the square of the perpendicular distance between the two axes.
Important:
The parallel axis may lie inside or outside the body.
374പWŚLJƐŝĐƐĨŽƌ//dͲ:͗DĞĐŚĂŶŝĐƐ//

z
ʹǤ౧‡”’‡†‹ —Žƒ”š‹•Ї‘”‡
Consider a body lying in the xy-plane. It can be
assumed to be made of a large number of particles.
Consider one such particle of mass m at a distance y
r from the origin of axis. In terms of Cartesian r
coordinate, y x

r2 = x2 + y 2 ...(i)
The moment of inertia of the particle about x-axis is x
Ix = my2 Fig. 3.48
The moment of inertia of the body about x-axis is
Ix = Â my2 ...(ii)
Moment of inertia of the body about y-axis is
Iy = Â mx2 ...(iii)
Moment of inertia of the body about z-axis is
Iz =Â mr2
= Â m(x2 + y2)

= Â mx2 + Â my2
From Eqs. (ii) and (iii), we get
Iz = Ix + Iy ...(2)
Thus, the moment of inertia of a body lying in a plane about an axis perpendicular
to its plane is equal to the sum of the moment of inertia of the body about any two
mutually perpendicular axes in its plane and intersecting each other at the point where the
perpendicular axis passes through the body.
Important:
Intersection of axes need not be the centre of mass of the body. Perpendicular axis theorem
can be used for planar body but not for cone, etc.

Moment of Inertia of a Thin Uniform Rod


Consider a thin uniform rod of mass
m and length L, which is rotating
about an axis passing through its
CM and perpendicular to its length. I
CM
For the thin rod its moment of inertia
x
about an axis passing through its dx
length will be zero.
L/2 L/2
Consider a small element of
length dx at a distance x from its
CM. The mass of the element Fig. 3.49
ZŽƚĂƟŽŶപ375

M
dm = dx
L
Moment of inertia of the element dI = dm x2
Moment of inertia of whole rod
L/ 2
I = 2 Ú dm x 2
I cm =
ML2
0
12
L/ 2
ÊM ˆ 2
= 2 Ú ÁË dx˜¯ x
L L/2 L/2
0

L/ 2 Fig. 3.50
2M x3 ML2
= =
L 3 0
12

Radius of gyration of thin rod about CM axis,

I ML2 L
k = = =
M 12 M 12
Moment of inertia about any other axis can be obtained by using parallel axis theorem.
Moment of inertia about the end of the rod can be obtained as
Iend = ICM + Md2
2 M
ML2 Ê Lˆ
= + MÁ ˜ k
12 Ë 2¯
ML2
= Fig. 3.51
3
Radius of gyration about the end of the rod,

I ML2/3 L
k = = =
M M 3
Some Important Cases
(i) M.I. of thin rod about an axis passing through the end of
the rod, and it is inclined at an angle q with the axis.
r
The mass of the element
dx
M
dm = dx
L q x
The distance of the element from the axis
r = x sin q.
The M.I. of the element about the axis
Fig. 3.52
dI = (dm)r2
376പWŚLJƐŝĐƐĨŽƌ//dͲ:͗DĞĐŚĂŶŝĐƐ//

The M.I. of the whole rod


L
I = Ú dm r2
0

L
ÊM ˆ
Ú ÁË L dx˜¯ (x sin q )
2
=
0
L
M ML2
= sin 2 q Ú x 2 dx = sin 2 q
L 0
3
(ii) The rod of mass M and length L is bent L-shape. Its moment of inertia
about the end L/2
2
( M/2)(L/2) M
Iend = + (L/ 2)2
3 2 L/2
ML2
=
6
ML2 Fig. 3.53
(iii) Iz = + Md 2
12
z

y
d

Fig. 3.54

ML2
(iv) I = 0 + + ML2
3 M, L A
4
= ML2
3
I = IA + I B + I C
B
IA = Â m ¥ 02 = 0 M, L

ML2
IB = C
3
IC = Â m L2 = ML2 M, L

ML2 4 Fig. 3.55


=0+ + ML2 = ML2
3 3
ZŽƚĂƟŽŶപ377

WORKED PROBLEM
Problem 3.19 There is a system of two particles of masses m1 and m2. The separation
between them is r. Find M.I. of the system about an axis passing through their CM and
perpendicular to the line joining them.
m1 ¥ 0 + m2 r m2 r m1 r
Solution Here, r1 = = and r2 = r - r1
m1 + m2 m1 + m2 m1 + m2

CM
m1 m2 m
r
r1 r2 fi
r
(a) (b) Equivalent single particle
system of mass m

Fig. 3.56

M.I. of the system about CM axis


ICM = m1r12 + m2r22
2 2
Ê m2 r ˆ Ê m1 r ˆ
= m1 Á ˜ + m2 Á
Ë m1 + m2 ¯ Ë m1 + m2 ˜¯

Ê m m ˆ
= Á 1 2 ˜ r2
Ë m1 + m2 ¯

= mr2
m1 m2
where = m, is called reduced or effective mass of the system.
m1 + m2

Now we will Find Radius of Gyration

I CM
k =
M m1 m2 k
r
(m1 + m2)
Ê m1 m2 ˆ 2
ÁË m + m ˜¯ r (a) (b) Equivalent single particle
1 2 system for MI
=
(m1 + m2 )
Fig. 3.57

m1 m2
= r
(m1 + m2 )
378പWŚLJƐŝĐƐĨŽƌ//dͲ:͗DĞĐŚĂŶŝĐƐ//

Moment of Inertia of Thin Rectangular Lamina z


Consider a rectangular lamina of mass m and length a dy
and width b. Choose a small element of width dy and
length b at a distance y from the origin O. The mass of O y
the element y b
M M
dm = ¥ bdy = ◊ dy
ab a a
x
(i) Moment of inertia of the element about x-axis Fig. 3.58
dIx = (dm)y2
Moment of inertia of the whole lamina about x-axis and by using the integration
method
a/2
Ix = 2 Ú (dm) y 2
0

a/ 2
ÊM ˆ 2
= 2 Ú ÁË dy˜¯ y
a
0

a/ 2
M y3 Ma 2
= 2 =
a 3 12
0
2
Ma
or Ix =
12
(ii) Similarly, moment of inertia of lamina about y-axis
Mb 2
Iy =
12
(iii) Moment of inertia of the lamina about z-axis: By using perpendicular axis theorem
Iz = Ix + Iy
M 2
or Iz = (a + b2)
12
Some Special Cases
I2
(a) For square lamina, a = b I3
Ma 2 Ma 2
Ix = , Iy =
12 12 90° 90°
a I1
M 2 Ma 2
\ Iz = Ix + I y = ( a + a2 ) = O
12 6
(b) From Fig. 3.59, I4
a
I0 = I1 + I2 ...(i)
Fig. 3.59
ZŽƚĂƟŽŶപ379

here I1 = I2 I2
I5
Also, I 0 = I3 + I4 ...(ii)
90°
here I3 = I4 q
From Eqs. (i) and (ii), we get O

I1 + I2 = I3 + I4
or 2I1 = 2I3 a
I6
Ma 2
Thus, we have I1 = I2 = I3 = I4 = Fig. 3.60
12
(c) Now consider two perpendicular axes as shown in Fig. 3.60. a I

I0 = I5 + I 6 ...(iii)
As I5 = I6
Ma 2 a M
\ I1 = I2 = I3 = I4 = I5 = I6 =
12
(d) If we take isosceles triangle lamina of mass M, its M.I. about its
diagonal
Ma 2 Fig. 3.61
I=
12
(e) M.I. of a rectangular lamina about its diagonal S R
Let the rectangular lamina PQRS have a mass
M and dimensions L ¥ B.
h
By symmetry, the M.I. of the rectangular lamina B
about the diagonal QS = 2 (M.I. of the triangular
lamina about base QS). M
Q
Let h be the altitude of the triangle QRS. Then P L
from area of rectangle PQRS = 2(area of DQRS),
Fig. 3.62
È1 ˘
L ¥ B = 2 Í ¥ L2 + B2 ¥ h ˙
Î2 ˚
LB
or h =
L + B2
2

È 2 ˘
Í M Ê LB ˆ 1˙
Required M.I., IQS = 2Í ¥Á ˜ ¥
2 ÁË L2 + B2 ˜¯ 6˙
ÍÎ ˙˚

M L2 B2
IQS =
6 (L2 + B2 )
380പWŚLJƐŝĐƐĨŽƌ//dͲ:͗DĞĐŚĂŶŝĐƐ//

Moment of Inertia of a Ring or Hoop


Consider a ring mass M and radius R, and take a small element,
M
its mass per unit length of ring = R dl
2p R

M
The mass of element dm = dl
2p R Fig. 3.63
Moment of inertia of this element about the axis (called
geometrical axis) shown in Fig. 3.63.
dI = (dm)R2
Moment of inertia of the whole ring
2p R
By using integration method I = Ú (dm)R2
0
2p R
MR2
=
2p R Ú dl
0

MR2 2p R MR2
= l0 =
|| (2p R - 0)
2p R 2p R
= MR2 MR2
IT
Important Cases for Ring or Hoop Ê MR 2 2
ˆ
R Á 2 + MR ˜
Ë ¯
(i) M.I. about the tangent parallel to the
ID
geometrical axis: By parallel axis theorem ID

IT = I + Md2
Fig. 3.64
= MR2 + MR2
= 2MR2
I
(ii) M.I. about diameter: By perpendicular axis theorem
ID + ID = MR2 R
2 M
MR
\ ID =
2
(iii) M.I. about tangent parallel to the diameter. By parallel axis theorem, Fig. 3.65

MR2 3
I = + MR2 = MR2
2 2
(iv) A part of ring of mass M and radius R a
I = MR2 b
M 2
(v) M.I. of elliptical hoop: I = (a + b2 )
2 Fig. 3.66
ZŽƚĂƟŽŶപ381

Moment of Inertia of a Uniform Circular Disc


Consider a disc of mass M and radius R rotating about its
geometrical axis. Choose an element of radius r and width dr,
its mass
M r dr
M R
dm = ¥ (2p rdr)
p R2
Moment of inertia of the ring element about the axis shown
dI = (dm)r2 Fig. 3.67
Moment of inertia of whole disc
R

Ú (dm)r
2
I=
0
R
M
Ú p R2 (2p rdr)r
2
=
0
R
2M r 4
= 2
R 4 0
2
MR
=
2

Important Cases for Disc


(a) M.I. about the tangent parallel to geo- MR 2
metrical axis. By parallel axis theorem 2
IT = I + Md2 IT Ê MR 2 2
ˆ
Á I = 4 + MR ˜
R Ë ¯
MR2
= + MR2 ID
2 ID
3
= MR2
2 Fig. 3.68
(b) M.I. about diameter of the disc. By per-
pendicular axis theorem
MR2
ID + ID =
2
MR2
\ ID =
4
(c) M.I. about the tangent parallel to diameter. By parallel axis theorem
I = ID + Md2
MR2 5
= + MR2 = MR2
4 4
382പWŚLJƐŝĐƐĨŽƌ//dͲ:͗DĞĐŚĂŶŝĐƐ//

(d) M.I. of a small part of disc of mass M about the axis shown in Fig. 3.69. I
2 R
MR
I= M
2
(e) M.I. of annular disc (Important) Fig. 3.69
Choose an elemental ring of radius r and thickness dr,
mass of element
M M
dm = ¥ (2p rdr ) R1 dr
p (R22 - R12 ) R2 r

M.I. of the elemental ring


dl = (dm)r2
M.I. of the annular disc about the axis shown Fig. 3.70
R2

Ú (dm)r
2
I =
R1

R2
M
Ú p (R2 - R2 ) (2p rdr )r
2
=
R1 2 1

R2
2M
Úr
3
= dr
(R22 - R12 ) R
1

R2
2M r4
=
(R22 - R12 ) 4 R1

M
= [R24 - R14 ]
2(R22 - R12 )
M
= (R22 + R12 )(R22 - R12 )
2(R22 - R12 )

M(R22 - R12 )
or I =
2

Moment of Inertia of a Solid Cylinder


Consider a solid circular cylinder of mass M and cross-sectional radius R. The length of the
cylinder is L.
(i) M.I. of the cylinder about geometrical axis. Choose an element of cylinder in the form
of a pipe of length L and radius r and thickness dr.
The mass of the element
M M
dm = ¥ (2p rdr )L = ¥ (2p rdr )
p R2 L p R2
ZŽƚĂƟŽŶപ383

M.I. of the elemental pipe about the axis shown dr


in Fig. 3.71. r
dI = (dm)r2
M.I. of the whole cylinder
R L
Ú (dm)r
2
I =
Fig. 3.71 Cylinder or thick rod
0
R
Ê M ˆ
Ú ÁË p R2 ˜¯ (2p rdr )r
2
=
0
R
2M
Úr
3
= dr
R2 0

2M R 4 MR2
= = ¥
R2 4 2
(ii) M.I. of the cylinder about equitorial axis:
Choose an element disc of thickness dx at a distance
x from axis of rotation.
The mass of the element
M
dm = dx
L
The M.I. of the elemental disc about the axis of rotation
x dx
dI = MI of disc about diameter + (dm)x2
L/2 L/2
È (dm)R2 ˘
= Í + (dm)x 2 ˙ Fig. 3.72
Î 4 ˚
M.I. of the cylinder by using the integration method
L/ 2
È (dm)R2 ˘
I = 2 Ú Í + (dm)x 2 ˙
0 Î 4 ˚

L/ 2 È Ê Mdx ˆ 2 ˘
Í ÁË L ˜¯ R Ê M ˆ 2˙
= 2 Ú Í + Á dx˜ x ˙
0 ÍÎ 4 Ë L ¯ ˙˚
L/ 2 L/ 2
MR2 2M
= 2¥
4L Ú dx +
L Ú x 2 dx
0 0

3 L/ 2
MR2 L/ 2 2M x
= |x|0 +
2L L 3 0

Ê MR ML 2 2ˆ
= Á +
Ë 4 12 ˜¯
384പWŚLJƐŝĐƐĨŽƌ//dͲ:͗DĞĐŚĂŶŝĐƐ//

Special Cases of Cylinder or a Thick Rod


(i) If M.I. of the cylinder about both the axes calculated are equal, then

MR2 MR2 ML2


= +
2 4 12
fi L = 3R
(ii) M.I. of cylinder about the tangent: MR 2
2
By parallel axis theorem, IT = I + Md2 R

MR2
= + MR2 Fig. 3.73
2
3 I2
= MR2
2
I1 R
M.I. of thin circular pipe M
2
I1 = MR

ML2 MR2 L/2


I2 = +
12 2 L

Fig. 3.74
Moment of Inertia of a Solid Sphere
Consider a sphere of mass M and radius R rotating about
any of its diameter. Choose an element in the form of a r 2 = R2 - x2
R r
disc of radius r and thickness dx. The mass of the element
M 3M 2
dm = ¥ p r 2 dx = 3
r dx
4 3 4 R
pR
3
Here r2 = (R2 – x2) x
(dm)r 2 dx
M.I. of the element about the axis shown dI =
2 Fig. 3.75

Ê 3M 2 ˆ 2
R RÁ r dx˜ r
(dm)r 2 Ë 4R3 ¯
M.I. of the whole sphere I = 2Ú = 2Ú
2
2 0
2
R
3M
Úr
4
I = 3
dx
4R 0
R
3M
Ú (R
2
= - x 2 )2 dx
4R3 0
R
3M
Ú (R
4
= + x 4 - 2R2 x 2 )dx
4R3 0
ZŽƚĂƟŽŶപ385

R
3M x 5 2R 2 x 3
= R4 x + -
4R3 5 3 0

3M Ê 5 R 2R5 5ˆ
=
3 Á
R x+ -
4R Ë 5 3 ˜¯
2
= MR2
5
It is also called M.I. of a solid sphere about a diameter.

Important Cases of Solid Sphere Icm

(i) M.I. about any tangent of the sphere. By parallel axis theorem IT
2
IT = ICM + Md
2 R
= MR2 + MR2
5
7
= MR2
5
Fig. 3.76
(ii) Radius of gyration of the sphere about any tangent

7
Mk2 = MR2
5
7
fi k = R
5

Moment of Inertia of Thin Circular Shell


Consider a shell of mass M and radius R rotating about one of its diameters. Choose an
element in the form of a ring of radius r and angular width dq.
Here r = R sin q.
The mass of the element
M
dm = ¥ (2p r )(Rdq )
4p R2
M.I. of the element about the axis shown in Fig. 3.77. Rdq
R
dI = (dm)r2 dq r
M.I. of the whole shell q
R x
I = 2Ú (dm)r 2
0
R
È M ˘
Ú ÍÎ 4p R2 (2p r )Rdq )˙˚ r
2
= Fig. 3.77
-R
386പWŚLJƐŝĐƐĨŽƌ//dͲ:͗DĞĐŚĂŶŝĐƐ//

p
M 3
2R Ú0
= r dq

p
M
2R Ú0
= (R sin q)3 dq r = R sin q

p
MR2
=
2 0Ú sin 3 q dq

p
MR2
2 Ú0
= sin 2 q (sin q dq )

p
MR2
2 Ú0
= sin 2 q ( - d cos q )

p
MR2
2 Ú0
= 1 - cos 2 q )(d cos q ) sin2 q = 1 – cos2 q.

p
MR2 cos 3 q
=– cos q -
2 3
0

MR2 ÈÊ cos 3 p ˆ Ê cos 3 0 ˆ ˘ 2 2


I =– ÍÁ cos p - ˜ - Á cos 0 - ˜ ˙ = 3 MR
2 ÍÎË 3 ¯ Ë 3 ¯ ˙˚
5
About a tangent I = MR2
3
Moment of Inertia of a Triangular Lamina about its Base
Consider a triangular lamina of base b, altitude h and mass M.
M 2M
Mass per unit area of the lamina = =
Ê1 ˆ bh
ÁË bh˜¯
2
Consider a rectangular differential strip parallel to the base of width dy, at a distance y.
from the axis of rotation (base). A
From similar triangles DADE and DABC, we have
DE h - y
= dy
BC h
b h D E
DE = (h – y)
h
y
b
Area of strip DE = (h – y) dy
h
M.I. of the strip DE about the axis of rotation BC B b C
is given by
Fig. 3.78
ZŽƚĂƟŽŶപ387

dIBC = (Mass of strip DE) (distance from axis)2


Ê 2M b ˆ
= Á ( h - y ) dy˜ y 2
Ë bh h ¯
2M
= 2
y 2 ( h - y )dy
h
Integrating the above expression between the limits y = 0 to y = h,
h 2M
IBC = Ú0 h 2
( hy 2 - y 2 )dy

h
2 M ÈÊ hy 3 ˆ Ê y 4 ˆ ˘
= ÍÁ ˜ - Á ˜˙
h 2 ÍÎË 3 ¯ Ë 4 ¯ ˙˚
0

2M È h 3 1 ˘
= ( h - 0 3 ) - ( h 4 - 0 4 )˙
h 2 ÍÎ 3 4 ˚

2M È h4 h4 ˘ 2M h4 Mh 2
= Í - ˙ = =
h2 Î 3 4 ˚ h 2 (12) 6

Mh 2
\ IBC =
6

Moment of Inertia of a Hollow Sphere


Consider a hollow sphere of mass M and inner and outer radii R1 and
R2 respectively. We can imagine the sphere to be made up of a number
of thin, concentric spherical shells. Take one such shell of radius r and R1
R2
thickness dr. Its mass
dM = r(4pr2dr)
r
Moment of inertia of the shell about its diameter
2 dr
dI = (dM)r2
3
Fig. 3.79
2
= (4pr2drr)r2
3
8
= prr4dr
3
R2
Moment of inertia of the sphere I = Ú dI
R1

2 R
8
= pr Ú r 4 dr
3 R
1
388പWŚLJƐŝĐƐĨŽƌ//dͲ:͗DĞĐŚĂŶŝĐƐ//

R
8 È r5 ˘
2

= pr Í ˙
3 Î 5 ˚R
1
8
= pr (R25 - R15 )
15
Mass of the hollow sphere M = r ¥ volume
4
= r ¥ p (R23 - R13 )
3
4 2 (R25 - R15 )
\ I = p (R23 - R13 )r ¥
3 5 (R23 - R13 )
5 5
2 (R2 - R1 )
or I = M
5 (R23 - R13 )

Moment of Inertia of a Solid Right Circular


Cone about Its Vertical Axis y

Consider a cone of mass M and base radius R. Suppose the height


x
of cone is h. Choose an element in the form of a disc of radius x and
thickness dy as shown in Fig. 3.80. From the similar triangles, we have y
x R Ry h
= fix= dy
x
y h h
The mass of the element
M 3M
dm = 2
¥ (p x 2 dy ) =
x 2 dy
pR h R2 h R
3
M.I. of the element about the axis shown Fig. 3.80
(dm)x 2
dI =
h
(dm)x 2 2
M.I. of the whole cone = Ú
0
2
Ê 3M 2 ˆ 2
ÁË 2 x dy˜¯ x
h
R h
= Ú
0
2
h 4
3 M Ê Ry ˆ
= Ú 2R2 h ÁË h ˜¯ dy
0
h
3 MR2
Úy
4
= dy
2h5 0
h
3 MR2 y 5 3
= 5
= MR2
2h 5 10
0
3
About an axis passing through the vertex and parallel to the base I = MR2 + Mh2
20
ZŽƚĂƟŽŶപ389

Moment of Inertia of a Right Circular Hollow Cone y

We take a cone of mass M, radius R and altitude H about its axis.


M x
The mass per unit area of the cone, r = , where L is the slant
p RL L
height. dx
Consider a slice of the cone with its plane normal to the axis of r
the cone, of width dx, at a distance x from the vertex, measured
along the slant height.
x R
The radius of the ring r = R. y¢
L
Ê xR ˆ Fig. 3.81
Its area = 2p Á ˜ dx.
Ë L¯
Mass of the ring,
M 2p xR 2M
dm = dx = 2 x dx
p RL L L
M.I. of the elemental ring, 2
Ê 2M ˆ Ê xR ˆ
dI = Á 2 x dx˜ Á ˜
Ë L ¯Ë L ¯

L 2 MR2
Iyy¢ = Ú0 L4
x 3 dx

MR2
=
2
Note: M.I. of a hollow cone is independent of the altitude or slant height or vertex angle.

WORKED PROBLEMS
Problem 3.20 Four particles of masses 1, 2, 3 and 4 kg are placed at the vertices A, B, C and
D of a rectangle ABCD whose sides AB and CD are 3 m and 4 m respectively. What is the
moment of inertia of these particles about AB, BC and AC?
Solution

1 kg 4 kg D
A
F

3m

E
B C
2 kg 4m 3 kg

Fig. 3.82
390പWŚLJƐŝĐƐĨŽƌ//dͲ:͗DĞĐŚĂŶŝĐƐ//

4
(i) IAB = Â mi ri2
i= 1

= 1 ¥ 02 + 2 ¥ 02 + 3 ¥ 42 + 4 (4)2
= 36 + 64 = 100 kgm2 Ans.
(ii) IBC = 1 ¥ (3)2 + 2 ¥ (0)2 + 3 ¥ (0)2 + 4 ¥ (3)2
= 9 + 36 = 45 kgm2 Ans.
(iii) IAC = 1 ¥ (0)2 + 2 ¥ (BF)2 + 3 ¥ (0)2 + 4 ¥ (ED)2
= 2 (BF)2 + 4 (ED)2
Now, by symmetry BF = ED
\ IAC = 6 (BF)2 ...(i)
Now, to find out the value of BF:
Area of rectangle ABCD = 2 ¥ Area of DABC
È1 ˘
AB ¥ BC = 2 ¥ Í base ¥ perpendicular ˙
Î2 ˚
È1 ˘
3 ¥ 4 = 2 ¥ Í ¥ AC ¥ BF ˙
Î2 ˚
È1 ˘
12 = 2 ¥ Í ¥ AB2 + BC 2 ¥ BF ˙
Î2 ˚
= 32 + 4 2 ¥ BF = 5 BF
12
\ BF = m
5
Putting it in Eq. (i), we get
2
Ê 12 ˆ
IAC = 6 ¥ Á ˜ = 34.56 kg m 2 Ans.
Ë 5¯

Problem 3.21 A circular hole of radius r/2 is cut from a circular disc of radius r. The disc
lies in the xy-plane. Determine the moment of inertia about an axis passing through the
centre and perpendicular to the plane of the disc.
O O O

r
r
r/2 = – r/2

O¢ O¢ O¢

Fig. 3.83
ZŽƚĂƟŽŶപ391

Solution
Concept: Subtract the contribution of M.I. of a disc of size that of hole from the M.I. of
complete disc.
Let m be the mass of remaining disc.
Area of original disc = pr2,
Area of removed part = p(r/2)2,
Therefore, area of the remaining disc = pr2 – p(r/2)2 = (3/4)pr2
m 4 m
Mass per unit area, r = = ◊ 2,
(3/4)p r 2 3 pr
4
Mass of the original disc, pr2r = m,
3
1 1
Mass of the removed part, p r 2 r = m ,
4 3
1 4 2 2 2
Moment of inertia of the complete disc, Icom = ¥ mr = mr
2 3 3
2 2
1 m Ê rˆ mÊ rˆ
Moment of inertia of the removed part, Iremoved = ÁË ˜¯ + = ÁË ˜¯
2 3 2 3 2
1 2
= mr
8
Therefore, the moment of inertia of the remaining disc

Ê 2ˆ Ê 1ˆ
Iremaining = Á ˜ mr 2 - Á ˜ mr 2
Ë 3¯ Ë 8¯

Ê 13 ˆ
Iremaining = Á ˜ mr 2
Ë 24 ¯

Problem 3.22 Three identical thin rods, each of mass m and length l are jointed to form an
equilateral triangle. Find moment of inertia of the triangle about one of its sides.

60°

l l
fi l sin 60° l sin 60°

60° 60°
l l
(a) (b)

Fig. 3.84
392പWŚLJƐŝĐƐĨŽƌ//dͲ:͗DĞĐŚĂŶŝĐƐ//

Solution The given system of rods for its moment of inertia is equivalent to the system
shown in Fig. 3.84. Thus, the moment of inertia about the axis given
m (l sin 60∞)2 m (l sin 60∞)2
I =0+ +
3 3
ml 2
=2 ¥ sin 2 60∞
3
2 2 3 ml 2
= ml ¥ = Ans.
3 4 2 A

Problem 3.23 Three rods each of mass m and length L are joined
together to form an equilateral triangle as shown in Fig. 3.85. Find
the moment of inertia of the system about an axis passing through its
centre of mass and perpendicular to the plane of the triangle. CM

Solution Moment of inertia of rod BC about an axis perpendicular


to plane of triangle ABC and passing through the mid-point of rod
BC (i.e., D) is B C
2 Fig. 3.85
ml
I1 =
12 A

From the theorem of parallel axes, moment of inertia of this rod


about the asked axis is
I2 = I1 + mr2
2 CM
m l2 Ê l ˆ ml 2
fi + mÁ ˜ =
12 Ë 2 3¯ 6
\ Moment of inertia of all three rods is 30°
B D C
Ê ml 2 ˆ ml 2
I = 3I2 = 3 Á ˜= 2 Fig. 3.86
Ë 6 ¯
Problem 3.24 A circular plate of uniform thickness has a diameter of 56 cm. A circular
portion of diameter 42 cm is removed from one edge of the plate. Find CM of the remaining
portion. (IIT-JEE, 1980)
Solution The situation is shown in Fig. 3.87. Let O be CM
of the original circular plate, O1 that of the circular portion
28 cm
removed and O2 that of the remaining shaded portion. Let m
21 cm
be the mass per unit area of the plate.
A
Mass of original plate, O2 O O1

M = p (28)2 m
Mass of the circular portion removed,
m1 = p (21)2 m
Fig. 3.87
ZŽƚĂƟŽŶപ393

Mass of the shaded part,


m2 = M – m1 = p [(28)2 – (21)2] m = 343 p m
Masses m1 and m2 may be assumed to be concentrated at O1 and O2 respectively and O
is their CM.
\ Moment of m1 about O = Moment of m2 about O
or m 1 ¥ O 1O = m 2 ¥ O 2O
m1
or O 2O = ¥ O1O
m2
441 p m
= ¥ (28 - 21) = 9 cm.
343 p m
Problem 3.25 From a circular disc of radius R and mass 9M, R/3
a small disc of radius R/3 is removed from the disc, as shown
in Fig. 3.88. Find the moment of inertia of the remaining disc R 2R/3
about an axis perpendicular to the plane of the disc and passing
through the point O. (JEE, 2005, 2018) O

Solution Given, total mass of the disc = 9M


M.I. of the disc about a perpendicular axis through O is
1
I = (mass) ¥ R2
2 Fig. 3.88
1 9
= ¥ 9 M ¥ R2 = MR2
2 2
Mass of the small disc removed
9M
m = ¥ p (R/3)2 = M
p R2
The M.I. of the small disc about an axis perpendicular to O,
I¢ = ICM + Md2
2
1 Ê 2R ˆ
= M(R/3)2 + M Á ˜
2 Ë 3 ¯
MR2
=
2
M.I. of the remaining disc about an axis perpendicular to O
Inet = I – I¢
9 MR2
= MR2 -
2 2
= 4MR2 Ans.
394പWŚLJƐŝĐƐĨŽƌ//dͲ:͗DĞĐŚĂŶŝĐƐ//

Problem 3.26  A T-shaped object, with dimensions shown in Fig. 3.89, is lying on a smooth
floor. A force F is applied at the point P parallel to AB, such that the object has only the
translational motion without rotation. Find the location of P with respect to C.
(IIT-JEE, 2005)
Option: (a) 4/3l (b) 5/3l (c) 3/4l (d) l/3
Solution Point P must be the centre of mass of the system. Let l be the mass per unit length,
then mass of AB, m1 = l l and mass of CD, m2 = 2ll
CM of AB
l
A B
D
r
F
P
CM of CD, l from C
2l

C
Fig. 3.89
Suppose y is the distance of CM from C,
m1 y1 + m2 y2
y =
m1 + m2
( l l ) ¥ 2l + ( 2 l l ) ¥ l 4
= = l Ans.
( l l) + ( 2l l) 3 Axis

Problem 3.27 Two thin wires of length L are touching each other at
an angle of 30°. Find the moment of inertia of this system about the line
passing through the point where the wires meet and the perpendicular 30°
to the bisector of the 30° angle.
The M.I. of the wire about an axis perpendicular to the wire (ZZ) is
2
1 ÈL˘ 1
IZZ = mL2 + m Í ˙ = mL2 parallel axis theorem. Fig. 3.90
12 Î4˚ 3
X
Solution If the distance of an element of the wire from (ZZ) is D then its
distance from the axis XX which makes and angle q with ZZ is d cos q. 15°
The M.I. is directly proportional to the square of the mass from the
axis. M.I. about XX is
2
Ê d cos q ˆ 1 X
IXX = IZZ Á = mL2 cos2 q. Here, q = 15°.
Ë d ˜¯ 3 Fig. 3.91
And there are two wires. Multiply the result by two.
È2˘
I = Í ˙ mL2 cos 2 (15∞)
Î3˚
ZŽƚĂƟŽŶപ395

Problem 3.28 Two thin discs each of mass 4.0 kg and radius 0.4 m are attached as shown
in Fig. 3.92 to form a rigid body. What is the rotational inertia of this body about an axis
perpendicular to the plane of disc B and passing through its centre?
Option: (a) 3.4 kg-m2 (b) 3.2 kg-m2 (c) 3.6 kg-m2 (d) 3.0 kg-m2
Solution Moment of inertia of each disc A and B about the
axis through their centre of mass and perpendicular to the
plane will be
r r A
B
IAA = IBB = (1/2) Mr2
Now moment of inertia of disc A about an axis through B
by ‘theorem of parallel axes’ will be
IAB = IAA + M (2r)2 = (9/2) Mr2
Fig. 3.92
1 9
So, I = IBB + IAB = MR2 + MR2
2 2
i.e, I = 5Mr2 = 5 ¥ 4 ¥ (0.4)2 = 3.2 kg-m2

IMPORTANT FORMULAE
ͳǤ౧‘‡–‘ˆ ‡”–‹ƒ
ȋ‹Ȍ౯Š‹‘† 3 2
I1 = 0 5 4
ml 2 q q 1
I2 =
12
ml 2 x
I3 =
3 6
ml 2
I4 = sin 2 q Fig. 3.93
12
ml 2
I5 = sin 2 q
3
I6 = mx2
4 2
ȋ‹‹Ȍ౯‹” —Žƒ”‹• 
mR2
I1 = I2 =
4
I3 = I1 + I 2
1
mR2 5
3
=
2
5
I4 = I2 + mR2 = mR2
4
3
I5 = I3 + mR2 = mR2
2 Fig. 3.94
396പWŚLJƐŝĐƐĨŽƌ//dͲ:͗DĞĐŚĂŶŝĐƐ//

ȋ‹‹‹Ȍ౯‹” —Žƒ”‹‰ 4 2
mR2
I1 = I2 =
2
Ie = I1 + I2 = mR2
1
3
I4 = I2 + mR = mR2 2
5
3
2
I5 = I3 + mR2 = 2mR2

ȋ‹˜Ȍ౯‡ –ƒ‰—Žƒ”Žƒ„
mb 2 Fig. 3.95
I1 =
12
2
ma 2
I2 =
12
1
I3 = I1 + I 2 3
b
m 2
= (a + b2 ) a
12
ȋ˜Ȍ౯“—ƒ”‡Žƒ„
Fig. 3.96
3
2

a q
1
4

Fig. 3.97

ma 2
I1 = I2 = I 3 =
12

ma 2
I4 = I1 + I 3 = 2 1
6

ȋ˜‹Ȍ౯‘Ž‹†’Ї”‡
2
I1 = mR2
5
7
I2 = I1 + mR2 = mR2
5
m = mass of sphere
Fig. 3.98
ZŽƚĂƟŽŶപ397

ȋ˜‹‹Ȍ౯ ‘ŽŽ‘™’Ї”‡
2 1

Fig. 3.99

2 5
I1 = mR2, I2 = I1 + mR2 = mR2
3 3

Moments of inertia of uniform bodies of various shapes

R
+ 1
+ mR 2
l 2

1 2 1 1
ml ml 2 mR 2
3 12 2

Slender rod Slender rod Disc Cylinder


(a) (b) (c) (d)

2 1
mR 2 mR 2 mR 2
5 2

MR 2

Thin–walled clinder Hoop


about its axis Solid sphere
(g)
(e) (f)

Contd.
398പWŚLJƐŝĐƐĨŽƌ//dͲ:͗DĞĐŚĂŶŝĐƐ//

1
m(R12 + R 2 2 )
2

3
R mR 2
2 L 10
mR 2 R R1
3
1 1
MR 2 + ML2
4 12

Cone R2
A solid cylinder about
Spherical shell a perpendicular (j)
axis through its centre
(h) Hoop
(I) (k)

L
L W

R2 R1

1
m(a 2 + b 2 )
12 1
1 2 M (L2 + W 2 )
ML 12
3 2 R 2 5 - R15
M
Rectangular 5 R 2 3 - R13
parallelepiped A rectangle about a
A rectangle perpendicular axis A thick-walled hollow sphere
(l) about one edge through its centre about a diameter

(m) (o)
(n)

NUMERICAL QUESTIONS
Q.14. Two particles of masses 1 kg and 2 kg are separated by a distance 3 m. Evaluate
moment of inertia of the system about:
(a) a line joining the particles;
(b) a line passing through particle of mass 1 kg and perpendicular to the line joining
the particles; and
(c) a line passing through the mid-point of the 2 3
line joining the particles and making an angle
of 30° with it. r2
30°
1
Ans. M.I. about 1 kg 3m 2 kg
axis 1: I1 = 0 r1

axis 2: I2 = (2 kg) (3 m)2 = 18 kgm2


axis 3: = I3 = 1 (r1)2 + 2 (r2)2 = 1 (1.5 sin 30)2 + 2 (1.5 Fig. 3.100
sin 30)2 = 1.69 kgm2
Q.15. Find the M.I. of a square plate about a diagonal and show that it is equal to rotational
inertia about a median line.
ZŽƚĂƟŽŶപ399

M2
ma 2 ma 2 ma 2 D2 D1
Ans. Iz = + =
12 12 6
Also, Iz = ID1 + ID2 = 2ID M1
O
2
1 ma
\ ID = Iz = a
2 12
By symmetry, Fig. 3.101
1 ma 2
Iz = IM1 + IM2 = 2IM \ IM = I z = I D =
2 12
Q.16. A non-uniform rod has its linear density varying according the expression l = ax +
bx2, where x is the distance of the point from the end of the rod. Find the moment of
inertia of the rod about an axis passing through this end and perpendicular to the rod.
Ans. We know,
L L aL4 bL5
I = Ú dm r2 = Ú ldx x2 = Ú (ax + bx2) ◊ x2 dx = +
0 0 4 5
Q.17. The M.I. of a thin uniform rod of mass M and length L about an axis passing through
its centre is I1. This rod is bent in the form of a ring, and if I2 is the M.I. of the ring
formed about its centre then compare I1 and I2.

ML2
Ans. I1 =
12
L
Radius of ring r =
2p
ML2 I1 p 2
\ I2 = , so, =
4p 2 I2 3

WORKED PROBLEM
Problem 3.29 Calculate the moment of inertia of a system of (2N + 1) particles, separated
by a lying along a straight line about an axis passing through the centre.
L
Solution The total length, L = 2Na fi a=
2N
If m is the mass of each particle and M is the total mass of the system, then
M
M = (2N + 1)m fi m=
(2N + 1)
The moment of inertia of the system,
N N
I= Â 2 ma2 n2 = 2 ma2 Â n2
n =1 n =1

N
N (N + 1)(2N + 1)
 n2 = 6
n =1
400പWŚLJƐŝĐƐĨŽƌ//dͲ:͗DĞĐŚĂŶŝĐƐ//

N (N + 1)(2N + 1)
\ I = 2ma2
6
After substituting values of a and m, we get

ML2 È 1˘
I =
12 ÍÎ1 + N ˙˚ Ans.

ML2
For N Æ •, I =
12

NUMERICAL QUESTIONS
Q.18. Four sphere each of mass M and radius r are kept with their M A
L B M
centres on the corners of a square of side L. Calculate M.I. of the
system about an axis passing through one side of the square. L L
(IIT-JEE, 2006)
2 M C D M
Ans. I of sphere A = Mr 2 L
5
Fig. 3.102
2
I of sphere B = Mr 2
5 A B

2
I of sphere C = Mr 2 + ML2 [Parallel axis theorem]
5
2 2
I of sphere D = Mr + ML2 C D
5
8 Fig. 3.103
\ I = ÂI = Mr2 + 2 ML2
5
Q.19. A uniform thin rod has mass M and length L. Find the moment of inertia of the rod
about an axis passing through the centre of the rod and making an angle q with the
rod.
Ans. Consider an elementary particle
M
dm = ldx = dx
L
at distance x from centre, r for this particle
r = x sin q
r
q
L/ 2
Ú- L/2 dm r
2
\ I = x
dx

L/ 2 m ML2 Fig. 3.104


Ú- L / 2 L
2
= dx ( x sin q ) = sin 2 q
12
ZŽƚĂƟŽŶപ401

ANGULAR MOMENTUM
The angular momentum of a particle about an axis is defined as the moment of the linear
momentum of the particle about that axis. It is a vector quantity. Its SI unit is kg-m2/s.
(i) Angular momentum of a particle due to translational motion
Consider a particle D of mass m, moving with z
 
a velocity v. Its line of motion is at a distance r
  
from the z-axis. Its linear momentum p = m v . L
Angular momentum of the particle about z-axis
is given by
O y
L = (mv sin q) ¥ r 
r  q
mv sin
v
and direction can be given by right-hand screw
rule. D, m q

In the case given, the direction of L is along x mv cos q
positive z-axis. Fig. 3.105
Thus,   z
L = mvr sin q k
    
or L = m( r ¥ v ) = r ¥ p
Here we have resolved momentum perpendic-
 O
ular to r . The same thing can also be obtained y
  r
r r
by resolving r perpendicular to momentum p. v
r sin q
Thus, we can write 90° q
L = r sin q ¥ mv
x
= mvr sin q
Fig. 3.106
(ii) Angular momentum due to rotation of the particle
Consider a particle of mass m is rotating about z
an axis with constant angular velocity w. Its r
linear velocity L
r
w
v = wr
The angular momentum of the particle about
z-axis; y
L = mv ¥ r sin 90° r
v
= m (wr)r
= (mr2)w x
= Iw Fig. 3.107
 
The direction of L is along the direction of w,
so we can write
 
L = Iw
402പWŚLJƐŝĐƐĨŽƌ//dͲ:͗DĞĐŚĂŶŝĐƐ//

WORKED PROBLEMS
Problem 3.30 Find the value of IP – IQ, where IP and IQ are the M.I. of the uniform disc of
mass m and radius R about the z-axis.

m
P
y
2R/3
C
R x
R/2
Q

Fig. 3.108

Solution According to parallel axis theorem,


IP = IC + mPC2 P
PQ = IC + mQC2
or IP – PQ = m (PC2 – QC2) C

ÔÏÊ 2R ˆ
2 2¸
Ê Rˆ Ô
= m ÌÁ ˜ - Á ˜ ˝
Ë
ÔÓ 3 ¯ Ë 2¯ Ô˛
Q

7 mR2
=+ Ans. Fig. 3.109
36
If you write IP = IQ + mQP2, it will be wrong because IP = IC + mr2, where r = CP.
Problem 3.31 Find the M.I. of a uniform disc of mass m
and radius R about the line x1x1¢.
Solution According to perpendicular axis theorem,
(IO)z = IQx + I Oy ...(i)
1 R/2
where IOz = mR2 ...(ii) x1 x¢1
2
Since the disc is uniform,
Fig. 3.110
Ix = Iy ...(iii) y
Using Eqs. (i), (ii) and (iii),

mR2
IOx = Ans.
4
x
Applying parallel axis theorem, O
R/2
2
mR2 Ê Rˆ mR2 O¢
Ix¢1 = IOx 2
+ m(OO¢) = + mÁ ˜ = Ans. x1 x¢1
4 Ë 2¯ 2
Fig. 3.111
ZŽƚĂƟŽŶപ403

(iii) Angular momentum of a rigid body due to rotation


Consider a body rotating about an axis shown z
in Fig. 3.112. Choose a small element whose M.I.
about the axis is dI. Its angular momentum about w
the axis shown in figure.
dL = (dI)w
Angular momentum of the whole body L = Ú (dI)w O r y

As w is same for each element of the body,


and Ú (dI) is the moment of inertia of the whole
x
body, say it is I, so
L = Iw Fig. 3.112

(iv) Angular momentum of a rigid body due to translation and rotation both
Consider a body of mass m is rotating with angular y
velocity w about CM axis and translating with a

linear velocity v as shown in Fig. 3.113.
w r
The angular momentum of the body v0
   O¢
L = L translation + L rotation y

or L = mv0y (– k̂ ) + Iw (– k̂ ) x
O
= – (mv0y + Iw) k̂ kg-m2/s
where I is the moment of inertia of the body about z
the perpendicular axis through O¢.
Fig. 3.113

WORKED PROBLEM
Problem 3.32 Find angular momentum of a disc about the axis shown in the following
situations.
z z
w w

M R R
M

(a) (b)

y y
M w M w
v v
R R
d d
O x O x

z (c) z (d)

Fig. 3.114
404പWŚLJƐŝĐƐĨŽƌ//dͲ:͗DĞĐŚĂŶŝĐƐ//

Solution
  Ê mR2 ˆ ˆ 2
(a) L = I w = Á ˜ w k kg-m /s
Ë 2 ¯
  2
(b) L = Iw = (I CM + Md )w k̂
Ê mR2 ˆ
= Á + Md 2 ˜ w kˆ kg-m 2/s
Ë 2 ¯

(c) L = mvd (– k̂) + Iw (– k̂)
Ê mR2 ˆ ˆ
= Á mvd + w ˜ (- k ) kg - m 2/s
Ë 2 ¯

(d) L = mvd (– k̂) + Iw( k̂)

Ê mR2 ˆ ˆ 2
= Á mvd - w ˜ (- k) kg - m /s
Ë 2 ¯
Hint: Always use the vector method for this type of problem.

Angular Momentum of Earth’s Motion about the Sun


Figure 3.115 shows the earth’s rotation on its axis as it orbits the sun.
Lorbital
w2 Lrotation

23

Earth
w1

Sun

Fig. 3.115

(i) Angular momentum due to rotation of earth about its axis

LRotation = I1w1 = ÊÁ MR2 ˆ˜ ÊÁ ˆ˜ [M = mass of the earth]


2 2p
Ë5 ¯Ë T ¯
2 24 6 2 2p
= ¥ (5.98 ¥ 10 )(6.37 ¥ 10 ) ¥
5 24 ¥ 60 ¥ 60
= 7.1 ¥ 1023 kg-m2/s.
(ii) Angular momentum is associated with the orbital motion of the earth about the sun
LOrbital = I2 w2
2p
= (Mr2) ¥
T
ZŽƚĂƟŽŶപ405

[Assuming the earth as point mass in comparison to the distance between the earth
and the sun]
Here M = 5.98 ¥ 1024 kg, r = mean earth-sun distance
= 1.50 ¥ 1011 m
and T = 1 year = 365 ¥ 24 ¥ 60 ¥ 60 s
2p
\ Lorbital = (5.98 ¥ 1024) (1.50 ¥ 1011)2 ¥
365 ¥ 24 ¥ 60 ¥ 60
= 2.7 ¥ 1040 kg-m2/s
The directions of Lrotation and Lorbit are shown in Fig. 3.115. The resultant angular
momentum of the earth will be the vector sum of these two angular momenta.

Thus, we can write |L net|= L2rotation + L2orbital + 2Lrotation Lorbit cos 23.5

Geometrical Meaning of Angular Momentum


 z
Consider a particle of mass m moving with velocity v
  
in x–y plane. Let r and ( r + Dr ) be the position vectors r
L
of the particle at instant t and (t + Dt) respectively. The

displacement of the particle in time Dt, PQ = D r = ADt
O r r y
 1   r
(r + D r )
The area vector of the triangle OPQ, DA = ( r ¥ Dr ) r
r r
Q
2 D r = vDt
 1 
P
or D A = ( r ¥ vDt)
2 x
 Fig. 3.116
DA 1 
or m = ( r ¥ mv )
Dt 2
 
Ê DA ˆ L
or mÁ =
Ë Dt ˜¯ 2

 Ê DA ˆ
or L = 2m Á
 Ë Dt ˜¯
DA 
The quantity is the area covered by the position vector r per unit time and is called
areal velocity. Dt
Angular momentum = 2 ¥ Mass ¥ Areal velocity
y
WORKED PROBLEMS mv

Problem 3.33 A particle of mass m is moving with velocity v
along a line y = x + 5. Find the angular momentum of the particle 45°
mv cos 45°
about a perpendicular axis passing through origin O.
Solution The linear momentum of the particle p = mv. Compare 5
the given equation of the line of motion with y = mx + c, we have
x
m = 1 or q = 45° and c = 5 unit. It represents a particle moving O
as shown in Fig. 3.117. The angular momentum about O
Fig. 3.117
406പWŚLJƐŝĐƐĨŽƌ//dͲ:͗DĞĐŚĂŶŝĐƐ//

LO = mv cos 45° ¥ 5
5
= mv unit
2
Its direction is along negative z-axis. Thus,
5
LO = – mvk̂ unit Ans.
2

Problem 3.34 Two particles, each of mass m and speed v, travel in opposite directions
along parallel lines separated by distance d. Show that the vector angular momentum of the
two-particle system is the same whatever be the point about which the angular momentum
is taken.
Solution Suppose the two particles are moving parallel to y
the y-axis as shown in Fig. 3.118. r
m1 v
Total angular momentum of the system
  
L = L1 + L 2
   
= r1 ¥ p1 + r2 ¥ p2 O x
r
v m2
= (x î) ¥ (–mv ĵ) + (x + d) î ¥ (mv ĵ ) x
x+d
= mvx( î ¥ ĵ) + (mvx + mvd)( î ¥ ĵ)
= – mvxk̂ + mvxk̂ + mvd k̂ Fig. 3.118

= mvdk̂
It is clear from the result that angular momentum does not depend on x and hence on
the origin. Thus, the angular momentum of the two-particle system is same whatever be the
point about which the angular momentum is taken.
Problem 3.35 A particle of mass m is released from point O x0
x
P at x = x0 on the x-axis from origin O and falls vertically
along the y-axis, as shown in Fig. 3.119. q
r
r
(i) Find the torque t acting on the particle at a time t
when it is at point Q with respect
 to O.
(ii) Find the angular momentum L of the particle about Q
O at this time t.
y F = mg
dL
(iii) Show that t = in this example.
dt Fig. 3.119
Solution

(i) The gravitational force F = mg produces the torque t. If r is the position vector of Q,
then the magnitude of the torque about an axis about O is given by
t = Fr sin q
= mgx0 (r sin q = x0) Ans.
The directionof the torque will be along the negative z-axis which is perpendicular to

the plane of F and r .
ZŽƚĂƟŽŶപ407

(ii) The velocity v of the particle at time t is, v = 0 + gt = gt.


The magnitude of the angular momentum is given by,
L = pr sin q
= mv(r sin q)
= mgt(x0)
= mgx0t Ans.
The direction of the angular momentum is also along the negative z-axis.
(iii) We have L = mgx0t
Differentiating the above equation w.r.t. time, we get
dL d
= (mgx0 t)
dt dt
= mgx0
=t Ans.
dL
Hence, the relation t = holds in this example.
dt
y
Problem 3.36 A uniform rod of mass M and length
L, with two particles m1 and m2 attached to its ends, m2
is pivoted at its centre. The system rotates in a vertical
plane with angular velocity w.
L
(a) Determine the expression for angular momen- O q
tum. x
(b) What is the angular acceleration of the
system when the rod makes angle q with the
m2g
horizontal?
(c) If m2 > m1, where does angular velocity reach m1
maximum?
Solution
m1g
(a) Moment of inertia of system is the sum of
Fig. 3.120
moment of inertia of rod and two particles.
2 2
ML2 Ê Lˆ Ê Lˆ
I = + m1 Á ˜ + m2 Á ˜
12 Ë 2 ¯ Ë 2¯

L2 Ê M ˆ
= ÁË + m1 + m2 ˜
4 3 ¯
Therefore, the angular momentum is
L2 ÊM ˆ
L = Iw = ÁË + m1 + m2 ˜ w
4 3 ¯
(b) Weight of the particles exerts torque about centre, taking anticlockwise torque as
positive.
408പWŚLJƐŝĐƐĨŽƌ//dͲ:͗DĞĐŚĂŶŝĐƐ//

L
Torque due to m1g, t1 = m 1g cos q
2
L
Torque due to m2 g, t2 = – m2g sin q
2
1
Net torque, Âtext = t1 + t2 = (m1 – m2)gL cos q
2
Âtext = Ia
St ext 2(m1 - m2 ) g cos q
a = =
I L( M/3 + m1 + m2 )
If the masses of particles are equal, m1 = m2, a = 0, the system has no angular
acceleration.
Angular acceleration a is zero when q = p/2 or –p/2, i.e., vertical position.
Angular acceleration a is maximum when q = 0 or p, i.e., horizontal position.
(c) Angular acceleration continues to be positive unit q = – p/2; the system gains angular
velocity at this angle. Therefore angular velocity is maximum when q = – p/2.

ROTATIONAL KINETIC ENERGY AND MOMENT OF INERTIA


Consider a body rotating about an axis as shown in Fig. 3.121. z
Choose a particle of mass m at a distance r from the axis of
1
rotation. Kinetic energy of the particle = mv2
2
As v = wr v
1
\ Kinetic energy of the particle = m(wr)2 r
m
2 y
O
1
(mr2)w2
=
2
x
1
Kinetic energy of whole body, K = Â (mr 2 )w 2 Fig. 3.121
2
w2
= (Smr 2 )
2
where Âmr2 is the moment of inertia of the body about axis y
of rotation, say it is I.
Thus, kinetic energy of rotating body
w v
1 2
KRot = Iw
2
O x
Kinetic Energy due to Translation and Rotation
both
Consider a rotating body, which is translating with a velocity z

v as shown in Fig. 3.122. Fig. 3.122
ZŽƚĂƟŽŶപ409

The total KE of the body K = KTranslation + KRotation


1 1
= mv 2 + Iw 2
2 2
ROTATIONAL WORK
z
Consider a body subjected to a force F. Suppose body
undergoes an angular displacement Dq due to the torque
w
of force F. The work done by the torque
DW = F ¥ displacement PQ F
r Q
= F ¥ Dl Dq y
O Dl
Here, Dl = r Dq P
\ DW = F ¥ rDq
or DW = tDq x
In case of variable torque, we can write Fig. 3.123
q2
WRotation = Ú t dq
q1
Power Delivered by a Torque
We know that DW = tDq
Dividing both sides by Dt, we get
DW Dq
=t
Dt Dt
DW
or Power P = = tw
Dt
Power = Torque ¥ Angular velocity

Newton’s Second Law for Rotating Rigid Body


  
We have, torque t =r ¥ F ...(i)
  
Angular momentum, L =r ¥ p ...(ii)
Differentiating both sides of Eq. (ii) w.r.t. time t, we get
  
dL d( r ¥ p)
=
dt dt
 
dr   dp
= ¥p+ r ¥
dt dt
   
= v ¥ p+ r ¥ F
  
= ( v ¥ mv ) + t
  
= 0 + t [∵ v ¥ v = 0]

 dL
\ t ext =
dt
Thus, the torque acting on a particle is equal to its rate of change of angular momentum.
410പWŚLJƐŝĐƐĨŽƌ//dͲ:͗DĞĐŚĂŶŝĐƐ//

This equation is the rotational analogue of Newton’s second law for linear motion,
 
dp
i.e., F ext =
dt
 
Also, we have L = Iw
 
 dL d  dw
\ t ext = = (I w ) = I
dt dt dt
 
or t ext = Ia

ANGULAR IMPULSE
y Hinge
Consider a rod hinged at its one end. It is acted upon by an

external force F for time Dt as shown in Fig. 3.124. The rod
starts rotating about the hinge due to the torque t = Fr.
The product of torque t with the duration of its exertion is
r
called angular impulse. Thus, angular impulse
 
J = tDt

Its direction is along the direction of t. Here in the case F

discussed the direction of J is along positive z-axis. Its SI unit
is kg-m2/s. By Newton’s second law, we have
 x
 dL O
t ext =
dt Fig. 3.124
 
or t ext dt = dL
 
The product t ext (dt) is angular impulse and d L is the change in angular momentum.
Thus, we can write
Angular impulse = Change in angular momentum

Conservation of Angular Momentum


So far we have studied two conservation laws, the conservation of linear momentum and
conservation of energy. Now we study a third law, the conservation of angular momentum.
We can start from Newton’s second law in angular form

 dL
t ext =
dt 
dL
If no net external torque acts on the system, this equation becomes =0
 dt
or L = constant (For isolated system)
This equation represents the law of conservation of angular momentum.
ZŽƚĂƟŽŶപ411

r
Examples based on Conservation of Angular L
Momentum
1. Planetary motion around sun: v1
In planetary motion, the gravitational force
(centripetal force) always passes through the r1
F
axis of rotation, so its moment of force zero. And
therefore angular momentum of the orbiting r2
Sun
planet remains constant. If v1 and v2 are the F
speeds of the planet when it is at distances r1 v2
and r2 respectively, then
mv1r1 = mv2r2 Fig. 3.125
or v1r 1 = v 2r 2 r r
L = Iiw i L = If w f
2. Figure 3.126 shows a girl sitting on a stool that
can rotate freely about a vertical axis.
The girl, who is rotating initially with angular
speed w1, holds his outstretched  hands. His
angular momentum vector L lies along the
vertical axis as shown in Fig. 3.126(a). The girl
now pulls his arms; this decreases his moment
of inertia from its initial value Ii to If (If < Ii). His
angular speed increases from wi, to wf. As no net
external torque acts on the system along the axis
of rotation, so the angular momentum of the
(a) (b)
system about axis remains constant. Thus, we
have Fig. 3.126
r
Iiwi = Ifwf. As If < Ii, \ wf < wi L

3. Consider a device shown in Fig. 1.127 in which a small rf


body attached with the string is rotating initially in a circle ri
of radius ri with angular speed wi. The free end of the string F
is pulled down and so the body moves on a circle of smaller
radius rf (rf < ri). As the torque of force F is zero about the
axis of rotation, so the angular momentum of the rotating
body remains constant.
Therefore,
Iiw i = I fw f.
The increase in KE of the body
F
= 1 I f w 2f - 1 I iw i2
2 2 Fig. 3.127

4. Consider another similar device in which the radius of path decreases by wrapping
the string over the pipe as shown in Fig. 1.128. Here the tension in the string T which
is acting upon at a distance R (radius
 of pipe), constitutes a torque t = TR along the
axis of rotation (along the line of L), so the angular momentum of the rotating body
will change. Kinetic energy of the body remains constant.
412പWŚLJƐŝĐƐĨŽƌ//dͲ:͗DĞĐŚĂŶŝĐƐ//

r
L

T
T

(a) (b)

Fig. 3.128

NUMERICAL QUESTIONS
Q.20. A grindstone 1.0 m in diameter, of mass 50 kg, is rotating at 900 rev/min–1. A tool is
pressed normally against the rim with a force of 200 N, and the grindstone comes to
rest in 10 s. Find the coefficient of friction between the tool and the grindstone. Neglect
friction in the bearings.
Diameter 1
Ans. Radius R = = = 0.5 m; Mass m = 50 kg
2 2

mR2 50 (0.5)2
\ I = = = 6.25
2 2
Also,
p
wi = 900 rpm = 900 ¥ = 30 p rad s -1
30
wf = 0; t = 10 s
w f - wi 0 - 30 p
a = = = - 3p
t 10
Torque = I a = (6.25) (–3 p)
Now,
Torque = r Ft = (0.5)f
\ (0.5)f = – 3 (6.25 p)
Friction, f = – 37.5 p
But f = mn and n = 200
37.5 p = m (200) or m = 0.1875 p = 0.58
Q.21. A block of mass 0.05 kg is attached to a cord passing through a hole in a horizontal
frictionless surface as in Fig. 3.129. The block is originally revolving at a distance of
0.2 m from the hole, with an angular velocity of 3 rad s–1. The cord is then pulled from
below, shortening the radius of the circle in which the block revolves to 0.1 m. The
block may be considered a point mass.
ZŽƚĂƟŽŶപ413

(a) What is the new angular velocity?


(b) Find the change in kinetic energy of the block.

Fig. 3.129

Ans.
(a) I1w 1 = I 2w 2
MR12 w1 = MR22w2
2 2
ÊR ˆ Ê 0.2 ˆ
or w2 = Á 1 ˜ w 1 = Á 3 = 12 rad s -2
Ë R2 ¯ Ë 0.1 ˜¯

1 1
(b) Change in KE = MR22w 22 - MR12 w 12
2 2
1
= M (R22w22 – R12w12)
2
0.05
= [(0.1)2 (12)2 - (0.2)2 (3)2 ] = 0.027
2
Q.22. Determine the constant torque that must be applied to a 50 kg flywheel, of radius of
gyration 40 cm, to give it an angular speed of 300 rpm in 10 s. (OJEE, 2013)
Ans. m = 50, d = 0.4, \ I = md2 = 50(0.4)2 = 8
p
wi = 0, wf = 300 ¥ = 10p , t = 10 s
30
w f - w i 10p
a = = =p
t 10
t = Ia = 8p Nm
Q.23. A 500 g wheel that has a moment of inertia of 0.015 kg m2 is initially turning at 30
rev/s. It comes to rest after 180 rev. How large is the torque that slowed it?
(EAMCET, 2010)
Ans. wi = 30 ¥ 2p rad s–1 = 60p rad s–1, wf = 0
q = 180 ¥ 2p = 360p rad
w 2f - w i2 0 2 - (60p )2
a = = = - 10 p
2q 360 p
414പWŚLJƐŝĐƐĨŽƌ//dͲ:͗DĞĐŚĂŶŝĐƐ//

I = 0.015 kgm2
\ t = I a = – (0.015) (10p) = – 0.15 p Nm
Q.24. A certain 8 kg wheel has a radius of gyration 25 cm. What is its moment of inertia?
How large a torque is required to give it an angular acceleration of 3 rad/s2?
Ans. M = 8 kg, d = 0.25, I = md2 = 8(0.25)2 = 0.5 kgm2
a =3
t = I a = (0.5) (3) = 1.5 Nm
Q.25. A rod of mass m and length L, lying horizontally, is free to rotate about a vertical
axis through its centre. A horizontal force of constant magnitude F acts on the rod at
a distance of L/4 from the centre. The force is always perpendicular to the rod. Find
the angle rotated by the rod during the time t after the motion starts.
FL
Ans. t =
4
ML2
I =
12
t FL/4 3F
a = = =
I ML2 /12 ML
1 2 1 3 F 2 3 Ft 2
q = at = t =
2 2 ML 2 ML
Q.26. A large wooden turntable of radius 2 m and total mass 120 kg is rotating about a
vertical axis through its centre, with angular velocity 3 rad s–1. A bag of sand of mass
100 kg is dropped vertically onto it, at a point near the outer edge.
(a) Find the angular velocity of the turntable after the sand bag is dropped.
(b) Compute the kinetic energies before and after the sand bag is dropped. Why are
they not equal?
MR2 120 ¥ 22
Ans. I of Disc = = = 240
2 2
w of disc = 3 rad s–1
L = Angular momentum of disc = 240 ¥ 3 = 720 kgm2 s–1
Now,
I¢ of Disc + Bag = Idisc + Ibag = 240 + 100 ¥ 22 = 640
If w¢ be final speed; Iw = I¢w¢
Iw 720
(a) w¢ = ; w= = 1.125 rad s -2
I¢ 640
1 1
(b) Ki = Iw 2 = ¥ 240 ¥ 32 = 1080 J
2 2
1 1
Kf = I¢w ¢ 2 = ¥ 640 ¥ (1.125)2 = 405 J
2 2
Some energy is lost during collision.
ZŽƚĂƟŽŶപ415

Q.27. A rope of negligible mass is wound round a hollow cylinder of mass 3 kg and radius
40 cm. What is the angular acceleration of the cylinder if the rope is pulled with a force
of 30 N? What is the linear acceleration of the rope? Assume that there is no slipping.
Ans. Torque on cylinder
F = 30 N
t = 30 ¥ (0.4) = 12 Nm
Moment of inertia M = 3 kg
Mr 2 3(0.4)2
I = = = 0.24 R = 0.4 km
2 2
t 12
Now, a = = = 50 rad s -2
I 0.24 Fig. 3.130
2
Thus, tangential acceleration, at = 50(0.4) = 20 ms .
Q.28. A wheel of radius 0.20 m is mounted on a frictionless horizontal P
axis. The rotational inertia of the wheel about the axis is 0.05
kgm2. A massless cord wrapped around the wheel is attached to
a 2.0 kg block that slides on a horizontal frictionless surface. If a
Fig. 3.131
horizontal force of magnitude P = 3.0 N is applied to the block as
shown in Fig. 3.131, what is the magnitude of the angular acceleration of the wheel?
Assume the string does not slip on the wheel.
Ans. FBD of pulley and block are shown in Fig. 3.132:
a
T
T P
a

Fig. 3.132

For motion of block


P – T = ma
or 3 – T = 2a ...(i)
For rotational motion of pulley
T ◊ r = Ia
or T(0.2) = 0.05a
or T = 0.25a ...(ii)
a a
But a = = = 5a
r 0.2
\ T = (0.25) (5a) = 1.25a
Carrying T in Eq. (i)
3 – 1.25a = 2 a or a = 0.92 ms–2 and a = 5 a = 4.6 rad s–2
Q.29. The angular momentum of a flywheel having a rotational inertia of 0.140 kgm2 about
its central axis decreases from 3 to 0.8 kgm2/s in 1.5 s. What is the magnitude of the
average torque acting on the flywheel about its central axis during this period?
(BCECE, 2009)
Ans. I = 0.140; Li = 3, Kf = 0.8
Dt = 1.5
L f - L f 0.8 - 3
tav = = = 1.47 Nm
Dt 1.5
416പWŚLJƐŝĐƐĨŽƌ//dͲ:͗DĞĐŚĂŶŝĐƐ//

Q.30. A standing disc with rotational inertia 1.2 ¥ 10–3 kgm2 is attached to an electric drill
whose motor delivers a torque of 16 Nm. Find the angular momentum of the disc
about its central axis (a) initially and, (b) at t = 10 s
Ans. ∵ Li = 0; Lf = J = t t = 16 ¥ 10 = 160
1
Q.31. The rotational inertia of a collapsing spinning star changes to its initial value. What
3
is the ratio of the new rotational kinetic energy to the initial rotational kinetic energy?
Ans. I1w2 = I2w2 = constant
L2 1
K= ; i.e., K μ
2I I
1
Since I reduces to rd of initial value, the energy will increase 3 times.
3

WORKED PROBLEMS
Problem 3.37 Figure 3.133 shows a person, sitting on a stool that can rotate freely about
a vertical axis. The person, initially at rest, is holding a fly wheel whose rim is loaded with
land and whose moment of inertia is I about its central axis. The wheel is rotating at an
angular speed wi from an overhead perspective, the rotation  is counterclockwise. The axis
of the wheel points vertical, and the angular momentum Li of the flywheel points vertically
upwards. The person now inverts the wheel; as a result, the person and stool rotate about
the stool axis. With what angular speed and direction does the student then rotate? (The
moment of inertia of the person + stool + wheel system about the stool axis is I0.)
Solution There is no torque acting on the system (person + stool + wheel), so the angular
 
momentum of the system about vertical axis remains constant. Let D L 1 and D L 2 be the
change in angular momentum of (person + stool) and wheel respectively, then we have
 
DL1 + DL2 = 0
 
\ DL 1 = – D L 2

–Li
Li
wi

w wi

(a) A person holds a flywheel (b) The person inverts the flywheel,
rotating around the vertical. setting himself into rotation

Fig. 3.133
ZŽƚĂƟŽŶപ417

  
As D L 2 = – L i – Li

= – 2Li
 
\ D L 1 = 2L i
As initial angular momentum of (person + stool) is zero, so
 
2 L i = I0w

 2Li
or w=
I0
The positive result tells that the angular velocity of rotation is counterclockwise.
Problem 3.38 Consider a disc of mass M and radius R is rotating with angular velocity w
about its geometrical axis as shown in Fig. 3.134. A small object of mass m falls gently on
the edge of the disc and sticks to it. Find the angular velocity of the disc.
Solution Weight of the object constitutes a torque which is perpendicular to the axis of
rotation (or angular momentum), so angular momentum of the system remains constant
along its initial direction. Thus, we have
r r
L L
wi = w wf
R M m R

mg

(a) (b)
Fig. 3.134
 
Li = L f
or Ii w i = If wf
Ê MR2 ˆ Ê MR2 2ˆ
or Á 2 ˜ w = Á 2 + mR ˜ w f
Ë ¯ Ë ¯
È M ˘
\ wf = Í ˙w Ans.
Î M + 2m ˚

ANALOGY BETWEEN LINEAR AND ROTATIONAL MOTIONS


Translation Rotational Motion
 
Linear displacement s Angular displacement q
 
 ds  dq
Linear velocity v = Angular velocity w =
dt dt 
  dw
 dv Angular acceleration a =
Linear acceleration a dt
dt
     
s = qr ; v = wr ; a = ar
418പWŚLJƐŝĐƐĨŽƌ//dͲ:͗DĞĐŚĂŶŝĐƐ//

Equations of Motion
v = u + at w = w0 + at
1 2 1 2
s = ut + at q = w 0t + at
2 2
v2 = u2 + 2as w2 = w02 + 2aq
Mass m Moment of inertia I
   
Force F = m a Torque t = I a
   
Linear momentum P = m v Angular momentum L = Iw
 
 dP dL
Newton's law F ext = Newton's law t ext =
dt dt
   
Linear impulse J = F Dt Angular impulse J = tDt
1 2 1
Translation KE = mv Rotational KE = Iw2
2 2
 
Work done = Ú F ◊ ds Work done = Ú t ◊ dq
   
Power = F ◊ v Power = t ◊w

WORKED PROBLEMS
Problem 3.39 A mass point is wound over a solid cylinder of mass M and radius R which
rotates having zero friction A mass m is tied from the free end and released. Calculate the
speed angular velocity, tension when mass m reaches the floor.
Solution Dynamical Method:

a RM

T
m

h mg

Floor

Fig. 3.135
ZŽƚĂƟŽŶപ419

Let acceleration of translation of mass m be a and angular acceleration of cylinder be a.


As there is no slipping of rope over the cylinder, so
a
a =
R
For the translational motion of the block;
mg – T = ma ...(i)
For rotation of cylinder;
TR = Ia
a
or TR = I
R
Ia
or T = …(ii)
R2
Solving Eqs. (i) and (ii), we get
mg g
a= =
Ê I ˆ Ê I ˆ
ÁË m + 2 ˜¯ ÁË 1 + ˜
R MR2 ¯
I Ê I ˆ
Here m is the inertia of translation and 2 is the inertia due to rotation. Á m + 2 ˜ is called
R Ë R ¯
total inertia of motion.
MR2
As I=
2
mg g
\ a= 2
=
MR M
m+ 1+
2R 2 2m
The velocity of block when it strikes the floor: By using third equation of motion,
v2 = 0 + 2ah

È g ˘
= 2Í h

Í1 + ˙
Î 2m ˚

2 gh
or v= Ans.
M
1+
2m
Angular velocity of cylinder

v 1 2 gh
w= =
R R 1+ M
2m
420പWŚLJƐŝĐƐĨŽƌ//dͲ:͗DĞĐŚĂŶŝĐƐ//

Energy Method: Only conservative forces are acting on the system, so


loss in PE = gain in KE
1 1
or mv 2 + Iw 2
mgh =
2 2
MR2 v
The MI of the cylinder I = , and w =
2 R
Using these relations, we have
1Ê 2ˆ v 2
1 Ê ˆ
mgh = mv 2 + Á MR ˜ Á ˜
2 2 Ë 2 ¯ Ë R¯
2 gh
\ v =
M
1+
2m
Note: In this type of problems when only velocity is asked, energy method is short and easy
for calculation.
Tension in the string. From equations we have
Ia Ig
T = =
R 2
Ê I ˆ
R2 Á 1 + ˜
Ë mR2 ¯
Ig
=
I Ê mR2 ˆ
R2◊ Á + 1˜
mR2 Ë I ¯
mg
or T =
mR2
1+
I
Clearly, T is less than the weight mg of the point mass.
Problem 3.40 A cord of negligible mass is wound around the rim
of a fly wheel of mass 20 kg and radius 20 cm. A steady pull of 25
N is applied on the cord as shown in Fig. 3.136. The flywheel is
R
mounted on a horizontal axle with frictionless bearings.
(a) Compute the angular acceleration of the wheel.
(b) Find the work done by the pull, when 2 m of the cord is M = 20 kg
unwound. R = 20 cm
(c) Also find the kinetic energy of the wheel at this point. Assume
F = 25 N
that the wheel starts from rest.
(d) Compare answers to parts (b) and (c).
Solution
Fig. 3.136
(a) Torque, t = FR = 25 N ¥ 0.20 m
= 5.0 Nm
ZŽƚĂƟŽŶപ421

Moment of inertia of the wheel about its axis,

MR2 20 ¥ (0.20)2
I = = = 0.4 kg m2
2 2
As t = Ia
\ Angular acceleration,
t 5.0 Nm
a = = = 12.5 rad s -2 .
I 0.4 kg m 2
(b) Work done by the pull unwinding 2 m of the cord,
= 25 N ¥ 2 m = 50 J.
(c) Angular displacement of the wheel,
Lenght of unwound string
q =
Radius of the wheel
2m
= = 10 rad
0.20 m
As the wheel starts from rest, w0 = 0
Final angular velocity w is given by
w2 = w02 + 2 aq = 0 + 2 ¥ 12.5 ¥ 10
= 250 (rad s–1)2
1 2 1
\ KE gained = Iw = ¥ 0.4 ¥ 250 = 50 J.
2 2
(d) The answers are the same, i.e., the kinetic energy gained by the wheel = work done
by the force. There is no loss of energy due to friction.
Problem 3.41 A uniform bar AB is pivoted at one end A. The bar is kept in the horizontal
position by a massless string tied to point B as shown in Fig. 3.137. Find the reaction of the
hinge on the end A of the bar at the instant when string is cut. (IIT Roorkee, 1998)

String
A B

A B RA mg
l/2

(a) (b)

Fig. 3.137
422പWŚLJƐŝĐƐĨŽƌ//dͲ:͗DĞĐŚĂŶŝĐƐ//

Solution When string is cut, the weight of the bar constitutes torque about the hinge, so
l
tA = mg ...(i)
2
According to Newton’s second law
tA = Ia ...(ii)
where a is the angular acceleration of the rod about the end A. From Eq. (i) and (ii)
l
Ia = mg
2
l
mg
or a = 2
I

ml 2
Here I =
3
mgl/2 3g
\ a = 2
=
ml 2 l
3
Acceleration of CM of the rod
3g l
acm = ar = ¥
2 l 2
3g
=
4
Again by Newton’s second law
mg – RA = macm
3g
or mg – RA = m ¥
4
mg
\ RA = Ans.
4
Problem 3.42 Two small blocks of masses m1 and m2 are attached to the ends of a string
which passes over the pulley of mass M and radius R. Calculate the acceleration of the blocks
and tensions in the string. (IIT-JEE, 2004)
Solution Due to inertia of the pulley, the tension in two sides of the cord are T1 and T2. Let
acceleration magnitude of each block be a.
For the blocks, by Newton’s second law
m1g – T 1 = m 1a ...(i)
and T 2 – m 2g = m 2a ...(ii)
For the rotation of pulley:
T1R – T2R = Ia
ZŽƚĂƟŽŶപ423

Important: In using the equation tnet = Ia, one should write


tgreater – tsmaller = Ia
Here a is along direction of greater torque. For no slipping
between pulley and cord
a
a = MR
R T2 T1
a
\ T 1R – T 2R = I
R
T2
Ia
or T1 – T2 = ...(iii) m2
R2 T1
+
Solving above equations, we get
m2g m1 Torque
(m1 - m2 ) g MR2
a = , here I =
I 2
m1 + m2 + 2 m1g
R
Ê I ˆ
m1Á 2m2 + 2 ˜ g R
Ë R ¯ a
and T1 = m1(g – a) = Ans.
Ê I ˆ T2 T1
ÁË m1 + m2 + 2 ˜¯
R Fig. 3.138
Ê I ˆ
m2 Á 2m1 + 2 ˜ g
Ë R ¯
T2 = m2 (g + a) =
Ê I ˆ
ÁË m1 + m2 + 2 ˜¯
R

Short Cut Method


Unblanced load
Acceleration, a =
Total inertia

È Unbalanced load ˘
= Í
n ˙˚
Î Inertia of translation + Inertia of rotation
For tension T = mdown(g – a)
or T = mup(g + a)
Problem 3.43 In the system shown in Fig. 3.139,
blocks A and B have masses m1 = 2 kg and m2 = (26/7)
kg respectively. Pulley having moment of inertia I =
0.11 kg-m2 can rotate without friction about a fixed
axis. Inner and outer radii of pulley are a = 10 cm and
b = 15 cm respectively. B is hanging with the thread A
wrapped around the pulley, while A lies on a rough m1
B
inclined plane. Coefficient of friction being m = 3 /10. 30°
m2
Calculate
Fig. 3.139
424പWŚLJƐŝĐƐĨŽƌ//dͲ:͗DĞĐŚĂŶŝĐƐ//

(i) tension in each thread, and


(ii) acceleration of each block.
Solution
(i) Given that m2 > m1. Here, the pulley has a tendency to rotate in clockwise direction.
The reason is that the weight m2 g tries to rotate the pulley in clockwise direction while
the component of weight m1 g, i.e., m1 g sin 30 tries to rotate in anticlockwise direction.
But the moment produced by m2 g is greater. Let the angular acceleration of the pulley
be a.
The free-body diagram is shown in Fig. 3.140.

N
T1 )
°
A (aa
0 m1
i n3 a
gs O
m1 T1 b

m1g cos 30°


m1g T2
+
T2
Torque

m2(ba)
B

m2g

Fig. 3.140

for block A, N = m1g cos 30


\ mN = m m1g cos 30
and T1 – m1g sin 30 – m m1g cos 30 = m1 (aa) ...(1)
for block B, m 2g – T 2 = m2 (ba) ...(2)
Taking moments of the forces acting on pulley about O, we get
T2 b – T1 a = Ia ...(3)
Substituting the given values in Eqs. (1), (2) and (3) we get
T1 – 13 = 0.2a ...(4)
T2 = 260 – 3.9a ...(5)
0.15T2 – 0.1T1 = 0.11a ...(6)
Solving Eqs. (4), (5) and (6), we have
T1 = 17 N, T2 = 26 N and a = 20 rad/sec2
Now, the acceleration of block A = a a = 2 m/s2 (upward)
and acceleration of block B = b a = 3 m/s2 (downward).
ZŽƚĂƟŽŶപ425

ROTATION ABOUT A MOVING AXIS


When there is a both translation and rotation
 of a body required two separate equations of
  
motion for the body, one for translation, F net = m a and other for rotation, t = Ia . In addition
to these equations it often requires the relation between these two motions, i.e., a = aR.

WORKED PROBLEMS
Problem 3.44 A cord is wrapped around a solid cylinder and then the end of the string is
held stationary while the cylinder is released from rest. Find the acceleration of the cylinder
and tension in the string.
Solution Let M and R be the mass and radius of the cylinder. a is the
acceleration of CM of the cylinder. T
For translation motion,
Mg – T = Ma ...(i) T
For rotation motion, R

TR = Ia
a Mg
As a =
R Fig. 3.141
a
\ TR = I
R
Ia
or T = ...(ii)
R2
Solving Eqs. (i) and (ii), we get
Mg
a =
Ê I ˆ
ÁË M + 2 ˜¯
R
Mg È MR2 ˘
= 2 ÍI = ˙
MR Î 2 ˚
M+
2R 2
2g
= Ans.
3
Ê MR 2ˆ 2g
Ia ÁË ˜¯ ¥ 3
and T = = 2
R2 R2
Mg
= Ans.
3

Problem 3.45 Find the acceleration of the device given in the figure.
Solution Acceleration magnitude of the block
426പWŚLJƐŝĐƐĨŽƌ//dͲ:͗DĞĐŚĂŶŝĐƐ//

È Unbalanced load ˘
a= Í
n ˙˚
Î Inertia of translation + Inertia of rotation
T3 T3
Here inertia of translation = (m1 + m2).
T2 T1
Ê I I ˆ I2, R2 I 1, R 1
and inertia of rotation = Á 12 + 12 ˜
Ë R1 R2 ¯ T2
m2 T1
(m1 - m2 ) g
\ a = m1
Ê I I ˆ m2g
(m1 + m2 ) + Á 12 + 22 ˜
Ë R1 R2 ¯ m1g
T1 = m1(g – a) Fig. 3.142
and T2 = m2 (g + a)
Ê aˆ
To get T1R1 – T3R1 = I1 Á ˜ Ans.
Ë R1 ¯

Problem 3.46 In Fig. 3.143 mass m1 slides without friction on the horizontal surface, the
frictionless pulley is in the form of a cylinder of mass M and radius R, and string turns the
pulley without slipping. Find the acceleration of each mass, and tension in each part of the
string.
a
N1
m1 M T1 T1 a

m1g T2
N2 +

T2 Torque
m2 a
m2

(a) (b)
m2g

Fig. 3.143

Solution Positive direction of angular acceleration a is taken along the corresponding


acceleration a1 which is taken positive in the direction of net force. Also due to inertia of the
pulley the tension in two parts of the string will be different. Let it is T1 and T2 as shown
in Fig. 3.144.
The equation of motion for masses m1 and m2 are
T 1 = m 1a ...(i)
and m 2g – T 2 = m 2a ...(ii)
The equation of motion of pulley is
T2R – T1R = Ia
(N2 constitutes no torque about the axis of rotation)
ZŽƚĂƟŽŶപ427

a
As there is no slipping of string over pulley, so a = .
R
a
\ T 2R – T 1R = I
R
Ia
or T2 – T1 = 2 …(iii)
R
Solving the above equations, we get
m2 g MR2
a = , here I=
Ê I ˆ 2
ÁË m1 + m2 + 2 ˜¯
R
m1m2 g
and T1 =
Ê I ˆ
ÁË m1 + m2 + 2 ˜¯
R
Ê Mˆ
ÁË m1 + ˜¯ m2 g
2
T2 = Ans.
Ê I ˆ
ÁË m1 + m2 + 2 ˜¯
R

Problem 3.47 A cord is wound around two discs on either side. The pulley
and the two discs have the same mass and radius. There is no slipping at
the pulley and no friction at the hinge. Find out the accelerations of the two
discs and the angular acceleration of the pulley.
Solution Let R be the radius of the discs and T1 and T2 be the tensions in
the left and right segments of the rope.

T1 1 2
T2

Fig. 3.144
a1 a2

a1 mg mg a2

(a) (b)

Fig. 3.145

Acceleration of disc 1,
mg - T1
a1 = ...(i)
m
428പWŚLJƐŝĐƐĨŽƌ//dͲ:͗DĞĐŚĂŶŝĐƐ//

Acceleration of disc 2, a
mg - T2
a2 = ...(ii)
m +

t TR 2T Torque
Angular acceleration of disc 1, = 1
a1 = = 1 ...(iii)
I 1
mR2 mR
2
T1 T2
2T2
Similarly, angular acceleration of disc 2, a2 = ...(iv)
mR Fig. 3.146
Both a1 and a2 are clockwise.
Angular acceleration of pulley,
(T - T )R 2(T2 - T1 )
a = 2 1 = ...(v)
1 mR
mR2
2
For no slipping, Ra1 – a1 = a2 – Ra2 = Ra ...(vi)
Solving these equations, we get
2g
a =0 and a 1 = a2 =
3
Problem 3.48 A uniform cylinder of radius R and mass M can rotate freely
about a stationary horizontal axis O shown in Fig. 3.147. A thin cord of length
l and mass m is wound on the cylinder in a single layer. Find the angular
acceleration of the cylinder as function of the length x of the hanging part of
O R
the cord. The wound part of the cord is supposed to have its centre of gravity M
of the cylinder axis.
Solution The mass of the hanging part of the cord x

mx
m¢ =
R
Thus, m¢g – T = m¢a ...(i) Fig. 3.147
and TR = Ia
la
or T = ...(ii)
R2
On solving Eq. (i) and (ii) we get
m¢ g R
a=
I T
m¢ +
R2
T
2
MR m
Here, I= + (l - x )R2
2 l
After simplifying, we get m¢g
2mgx
a = Ans. Fig. 3.148
R/( M + 2m)
ZŽƚĂƟŽŶപ429

Problem 3.49 A cylindrical rod of mass M, length L and radius


R has two cords wound around it whose ends are attached to
the ceiling, as shown in Fig. 3.149. The rod is held horizontally T T
with the two cords vertical. When the rod is released, the cords
unwind and the rod rotates. Find the tension in the cords as they T T
unwind and determine the linear acceleration of the cylinder as
it falls.
Solution The cylinder rotating under gravity has both trans- R
lational and rotational motions. Let v be the linear velocity of
its centre of mass and w its angular velocity about the axis of
Mg
rotation, then in descending a distance h it will lose PE = Mgh
while gain in KE Fig. 3.149
Ê1 1 ˆ
= Á /w 2 + Mv 2 ˜ ,
Ë2 2 ¯

1 2 1
So, Mgh = /w + Mv 2
2 2
1
Here, I =MR2 and v = Rw
2
˘Èv ˘
2
1 1 È1
So, Mgh = Mv 2 + Í MR2 ˙ Í 2 ˙
2 2 Î2 ˚ ÎR ˚
4
or v2 = gh
3
Now differentiating both sides with respect to time
dv 4 dh
2v = g
dt 3 dt
2 È dv dh ˘
or a = g as = a and = v˙
3 ÍÎ dt dt ˚
Now as the cylinder is accelerated down [W = M (g – a)]
1 È 2˘ 1
2T = M(g – a), i.e., T= Mg Í1 - ˙ = Mg
2 Î 3˚ 6
Alternative Solution: The equations of translational and rotational motion of the cylinder
will be
Mg – 2T = Ma (Translational motion) ...(i)
and 2TR = Ia (Rotational motion) ...(ii)
1 2
Now as I = MR and a = a/R, the equation of rotational motion, i.e., Eq. (ii) reduces to
2
1
2T = Ma ...(iii)
2
430പWŚLJƐŝĐƐĨŽƌ//dͲ:͗DĞĐŚĂŶŝĐƐ//

Solving Eqs. (i) and (iii) for a and T, we find


2 1
a= g and T = Mg .
3 6
Problem 3.50 The arrangement shown in Fig. 3.150 consists of two identical uniform solid
cylinders, each of mass m, on which two light threads are wound symmetrically. Find the
tension of each thread in the process of motion. The friction in the axle of the upper cylinder
is assumed to be absent.

a1 T T
T T

a1
mg

(a) (b)

Fig. 3.150

Solution Different forces are shown in Fig. 3.150.


Applying Newton’s law of motion, we get
mg – 2T = m ¥ 2a ...(1)
(∵ acceleration of lower cylinder will be the sum of acceleration of upper cylinder and
acceleration of lower cylinder)
Taking torque we have
a
2TR = Ia = I
R
1 2 a ma
or 2T = mR ¥ 2 = ...(2)
2 R 2
Substituting the value of 2T from Eq. (2) in Eq. (1) we have
ma
mg – = m ¥ 2a
2
Solving we get, a = (2/5 g)
m 2 mg
From Eq. (2) 2T = ¥ g=
2 5 5
mg
\ T =
10
ZŽƚĂƟŽŶപ431

Problem 3.51 In the arrangement shown in Fig. 3.151, a weight A possesses mass m, a
pulley B possesses mass M. Also known are the moment of inertia l of the pulley relative to
its axis and the radii of the pulley R and 2R. The mass of the threads is negligible. Find the
acceleration of the weight A after the system is set free.

T1 2T1 (T1 + T1)

2R T1

T2 T2
R Mg
a2 A a2
A a1
A
mg

(a) (b) (c)

Fig. 3.151

Solution See free-body diagram [Fig. (3.151].


For mass A, we have
mg – T2 = ma2 = 3a R m ...(1)
[∵ a2 = 3 a R, because linear acceleration of bigger mass is 2 a R and smaller mass is a R]
For drum
Mg – 2T1 + T2 = ma1 = Ma R ...(2)
From torque equation
(2T1)R + T2 (2R) = I a

ÊIˆ
or 2T1 + 2T2 = ÁË ˜¯ a ...(3)
R
Adding Eqs. (2) and (3), we get
I
Mg + 3T2 = a + MaR ...(4)
R
From Eq. (1) T2 = mg – 3aRm
Substituting this value in Eq. (4), we get
I
Mg + 3[mg – 3 a Rm] = a + MaR
R
I
Mg + 3 mg = a + MaR + 9a Rm
R

(M + 3 m) g = a È + MR + 9Rm˘
I
ÍÎ R ˙˚
432പWŚLJƐŝĐƐĨŽƌ//dͲ:͗DĞĐŚĂŶŝĐƐ//

( M + 3 m) g
\ a=
[ MR + 9Rm + (I/R)]

Now acceleration of block, a2 = 3aR


( M + 3 m) g
or a2 = 3R ¥
[ MR + 9Rm + (I/R)]

3( M + m) g
a2 =
[ M + 9m + (I/R2 )]

Problem 3.52 The cylinder of mass M and radius R is connected to a small block of
mass m with the help of inextensible cord. The cylinder is in pure rolling motion. Find the
acceleration of the block.
Solution
(a)
N

R a T T Massless pulley

f
T
Mg

T
m

mg

Fig. 3.152

The magnitude of accelerations of the block and the centre of mass of the cylinder are
equal. Let it be a.
For the motion of the cylinder:
T – f = Ma ...(i)
and fR = Ia ...(ii)
a
For pure rolling, a = ...(iii)
R
For the motion of the block:
mg – T = ma ...(iv)
Also solving above equations, we get
mg
a =
I
m+ M+
R2
ZŽƚĂƟŽŶപ433

(b)
N
T T
Massless pulley
a a
f
T
Mg

T
2a
m

mg

Fig. 3.153

If a is the acceleration of CM of the cylinder, then the acceleration of the topmost point
of the cylinder is 2a, and hence acceleration of the hanging block is 2a.
For the motion of the cylinder:
T + f = Ma ...(i)
and TR – fR = Ia ...(ii)
a
For pure rolling, a = ...(iii)
R
For the motion of the block
mg – T = m(2a) ...(iv)
After solving the above equations, we get
2mg
a =
I
4m + M +
R2

Important: The situation is shown in Fig. 3.153. As force F rotates the sphere, the point of
contact has a tendency to slip towards left so that the static friction on the sphere will act
towards right or the friction value will come +ve so assumed direction is correct.
Problem 3.53 A thin massless cord is wound on a reel of mass 3 kg and moment of inertia
0.6 kg-m2. The hub radius is R = 10 cm and peripheral radius is 2R = 20 cm. The reel is placed
on a rough table and the friction is enough to prevent slipping. Find the acceleration of the
centre of reel and of hanging mass of 1 kg.

2R
R

Fig. 3.154
434പWŚLJƐŝĐƐĨŽƌ//dͲ:͗DĞĐŚĂŶŝĐƐ//

Solution Let, a1 = acceleration of centre of mass of reel a


a2 = acceleration of 1 kg block
a = angular acceleration of reel (clockwise) a1
T = tension in the string T
f
and f = force of friction
Fig. 3.155
Free-body diagram of reel is as shown in Fig. 3.155: (only horizontal
forces are shown).
Equations of motion are:
T – f = 3a1 ...(i)
t f (2R) - T ◊ R 0.2 f - 0.1T f T
a = = = = - ...(ii)
I I 0.6 3 6
Free-body diagram of mass is,
Equation of motion is,
10 – T = a2 ...(iii)
T
For no slipping condition,
a1 = 2Ra or a1 = 0.2a ...(iv)
a2
and a2 = a1 – Ra or a2 = a1 – 0.1a ...(v)
Solving the above five equations, we get
a1 = 0.27 m/s2 10 N
and a2 = 0.135 m/s2 Fig. 3.156

Problem 3.54 Figure 3.157 shows a composite cylinder with forces F1 and F2 applied on it.
Determine the net torque on the cylinder, about the rotation axis, which is z-axis.
Solution We consider outward torque y
(that tends to rotate a body anticlock-
F1
wise) as positive.
y R1
t1 = – R1 F1
t2 = + R 2 F2 x
R2
Âtext = t1 + t2 = – R1 F1 + R2 F2
x
Note that if F1 = F2, net torque is z
O
negative because R1 > R2, i.e., the system
will rotate clockwise.
Net torque is positive, which implies F2
that the system will commence anti-
clockwise rotation with increasing ve- Fig. 3.157
locity.
Problem 3.55 A cylinder of mass 10 kg and radius 10 cm has a cord wrapped round it.
The pulley weighs 100 N and has a radius 5 cm. When the system is released, the 5 kg mass
comes down and the cylinder rolls without slipping. Calculate the acceleration and velocity
of the mass as a function of time.
ZŽƚĂƟŽŶപ435

10 cm
20 cm

Rough

5 kg

Fig. 3.158

Solution Writing equations of motion, we get


5g – T2 = 5a2 ...(i)
(T2 - T1 )R2
= a2 ...(ii)
1
M2 R22
2
T1 + f = M 1a 1 ...(iii)

(T1 - f )R1
= a1 ...(iv)
1
M1R12
2
a1 = R 1a 1 ...(v)
a1 + R 1a 1 = R 2a 2 ...(vi)
R2a 2 = a2 ...(vii)
a1 T2
T1 T1 a2

a1 R2
M2 a2
R1
f
M1 T2
5g

Fig. 3.159

We have seven unknowns, T1, T2, a1, a2, a1, a2 and f . Solving above equations, we get
4
a2 = g = 3.6 m/s2 Ans.
11
4 gt
v = a 2t = Ans.
11

Problem 3.56 An extensible cord is wound over a rough pulley of mass M1 and radius R
and a cylinder of mass M2 and radius R such that as the cylinder rolls down, [Fig. 3.160(a)]
the string unwounds over the pulley as well as the cylinder. Find the acceleration of cylinder
M 2.
436പWŚLJƐŝĐƐĨŽƌ//dͲ:͗DĞĐŚĂŶŝĐƐ//

P
R C
M1 P
M1 C a2
a1 = d1R
R T a2
a2
M2

T

C¢ a2
Q

M2g

(a) (b)

Fig. 3.160

Solution Through this illustration we will learn the application of torque equation and
constraint relation in more complex case here. We have changed the block with cylinder M2.
Pulley M1 will have only pure rotation while cylinder M2 will have rotation and translation
combined. Let us analyse step by step in the same way as the previous illustration.
Step I: Analyse the motions of the pulley and the cylinder. Pulley: One rotational acceleration
a1 (clockwise); cylinder: One rotational acceleration a2 (clockwise) and a linear acceleration
a2 (downwards).
Step II: Equation of motion for M2:
M 2g – T = M 2a 2 …(i)
Step III: Torque equation for pulley: tc = Ica
Ê M R2 ˆ 2T
TR = Á 1 ˜ a 1 fi a 1 = …(ii)
Ë 2 ¯ M1R

Torque equation for the cylinder (about centre of mass of cylinder): tc = Ica
Ê M R2 ˆ
fi TR = Á 2 ˜ a 2
Ë 2 ¯
2T
a2 = …(iii)
M2 R
Step IV: The acceleration of P and Q should be equal as both are connected with the same
inextensible string.
Acceleration of P, aM = a1R (downwards) …(iv)
Acceleration of Q: aN = a2 – a2R (downwards) …(v)
Hence, constraint relation: a1R = a2 – a2R …(vi)
ZŽƚĂƟŽŶപ437

Step V: Solving equations:


After solving Eqs. (i), (ii), (iii) and (iv), we get

È 2(M1 + M2 ˘
a2 = Í ˙g
Î 3 M1 + 2 M2 ˚

Problem 3.57 A block X of mass 0.5 kg is held by a long massless string on a frictionless
inclined plane of inclination 30° to the horizontal. The cord is wound on a uniform solid
cylindrical drum Y of mass 2 kg and of radius 0.2 m as shown in Fig. 3.161. The drum is
given an initial angular velocity such that block X starts moving up the plane.
(a) Find the tension in the string during the motion.
(b) At a certain instant of time, the magnitude of the angular velocity of Y is 10 rad/s.
Calculate the distance travelled by X from that instant of time until it comes to rest.
[g = 10 m/s2] (IIT-JEE, 1994)
a
w

Y
v0
a X
Smooth

30°

Fig. 3.161

Solution
(i) Suppose a is the acceleration of the block, then the tangential acceleration of the pulley
will be a. Thus, angular acceleration of the pulley
a
a = M
R T
Thus, mg sin 30° – T = ma ...(i)
T
and TR = I a
MR2 a
or TR = ¥ 30
°
2 R in
mgs
Ma 30°
\ T = ...(ii)
2 Fig. 3.162
From Eqs. (i) and (ii), we get
mg sin 30∞
a =
M
m+
2
mg
=
2m + M
438പWŚLJƐŝĐƐĨŽƌ//dͲ:͗DĞĐŚĂŶŝĐƐ//

0.5 ¥ 9.8
=
2 ¥ 0.5 + 2
= 1.63 m/s2
2 ¥ 1.63
and T =
2
= 1.63 N Ans.
(ii) In the process the KE of the system will convert into potential energy of the block and so
1 1
mv 2 + Iw 2 = mgh
2 2
1 1 Ê MR2 ˆ 2
or m(w R)2 + Á w = mg(s sin 30°)
2 2 Ë 2 ˜¯
After simplifying, we get
s = 1.224 m Ans.
Problem 3.58 Two steel balls 1 and 2 have a mass of 500 g each, and are rotating about the
vertical axis with an angular velocity of 4 rad/s at a distance of 15 cm from the axis. Collar
L is now forced down until the balls are at a distance of 5 cm from the axis. How much work
must be done to move the collar down?
Solution In this process angular momentum remains constant
I 1w 1 = I 2w 2 1 2
2
where I1 = 2 m (0.05) , w1 = 4 rad/s
I2 = 2 m (0.05)2
L
Substituting these values in the above equation, we get
I1w 1
w2 = = 36 rad/s
I2
Work done on the collar
Fig. 3.163
1 1
W = I 2w 22 - I1w 12
2 2
1 1
or W = 2 m (0.05)2 ¥ 36 2 - 2 m (0.15)2 ¥ 4 2
2 2
where m = 500/1000 = 0.5 kg
= 1.44 J Ans.
Problem 3.59 A small block of mass 4 kg is attached to a string passing through a hole in
a horizontal frictionless surface. The block is originally revolving in a circle of radius 0.5
m about the hole, with a tangential velocity of 4 m/s. The cord is then pulled slowly from
below, shortening the radius of the circle in which the block revolves. The breaking strength
of the cord is 600 N. What will be the radius of the circle when the cord breaks?
ZŽƚĂƟŽŶപ439

Solution Cord will break when centrifugal force exceeds break- T mv 2/r
ing strength. Let r2 be the radius of the circle at the instant
of breaking, then
mv22
600 = ...(i)
r
In the process of shorting of the string, angular momen- F
tum remains constant. L1 = L2 Fig. 3.164
mv1r1 = mv2r2 ...(ii)
or m ¥ 4 ¥ 0.5 = m ¥ v2 ¥ r
2
or v2 =
r
Substituting the value of v2 in Eq. (i), we get
r = 0.2988 m Ans.
Problem 3.60 A cord is passing through a hole at the centre of a frictionless table. At the
upper end a block of mass 0.5 kg is tied and a block of mass 8.0 kg is tied at the lower end
which is freely hanging. The smaller mass is rotated on the table with a constant angular
velocity about the axis passing through the hole so as to balance the heavier mass. If the
mass of the hanging block is changed from 8.0 kg to 1.0 kg, what is the frictional change
in the radius and the angular velocity of the smaller mass so that it balances the hanging
mass again?
Solution For circular motion of a body tied to a string on a horizontal plane
(mv2/r) = T
Here as tension is provided by the hanging mass M, i.e., O T
m
T = Mg r
v
So, (mv2/r) = Mg
According to the given problem T

(mv12/r1 ) M1 g 8
= =
(mv22/r2 ) M2 g 1 M

v12 r2 8
or 2
= ...(i) Fig. 3.165
v2 r1 1
Now as the force (T is central) so angular momentum is also conserved, i.e.,
mv1r1 = mv2r2 ...(ii)
So, substituting the value of (v1/v2) from Eqn. (ii) in (i)
2
È r2 ˘ r2 8 r2
Í ˙ ¥ = , i.e., =2 ...(iii)
r
Î 1˚ r1 1 r1
Dr r -r r
So that = 2 1= 2 –1=2–1=1
r r1 r1
440പWŚLJƐŝĐƐĨŽƌ//dͲ:͗DĞĐŚĂŶŝĐƐ//

Problem 3.61 Suppose a person of mass m stands at the edge of a circular platform of radius
R and moment of inertia I. The platform is at rest initially, but the platform begins to rotate
when the person begins to move with velocity v.
Axis
Determine the angular velocity of the platform.
Solution Problem Solving Strategy: If the per-
m
son starts to walk along the edge of the platform,
the platform starts rotating in the opposite direc-
tion. The person exerts a force (torque) on the plat-
CM v
form, the platform exerts an equal and opposite R
torque on the person. Therefore, the net torque on
the system of person plus platform is zero and the
total angular momentum is conserved. (a)
r
 Ê vˆ  L person + ve
L person = (mR2 ) Á ˜ ; L platform = – Iw
Ë R¯
When the person starts walking anticlockwise,
the person’s angular momentum points upwards r
(from right hand thumb rule). Initial angular L platform - ve
momentum is zero initially, so the platform must
(b)
rotate clockwise so that final angular momentum
is zero. Fig. 3.166
 
Li = L f
Ê vˆ
0 = mR2 Á ˜ - Iw
Ë R¯
mRv
w =
I

Problem 3.62 Consider the situation of the previous problem. If the platform is rotating
initially with angular velocity w and the person starts to move towards the centre. What is
the angular speed when he reaches a point at a distance r from the centre?
Solution Moment of inertia of system has decreased as there is redistribution of mass
without external torque. Therefore, angular momentum of the system is conserved.
 
Li = L f
(I + mR2)w = (I + mR2)w

(I + mR2 )
w = w0
I + mr 2
Problem 3.63 If a person runs at a speed v along a line tangent to the circumference of a
stationary platform, what is the rotational speed of the system of platform plus person when
the person lands on to the platform?
Solution Problem Solving Strategy: This situation is equivalent to an inelastic collision
(person sticks to the platform). In such cases energy is not conserved. Linear momentum
ZŽƚĂƟŽŶപ441

is not conserved because the system is not isolated. Large


horizontal forces from the earth are transmitted to the platform
at the support. However, we may apply conservation of
angular momentum because the horizontal forces at the posts
supporting the platform do not exert any torque, as they act
on the rotation axis. Here we ignore the small frictional torque m
on bearings.
From conservation of angular momentum,
   
L person + L platform = L person + L platform R

mvR + 0 = (I + mR2)w
mvR Fig. 3.167
w =
(I + mR2 )
Problem 3.64 An unknown rotating star has a period of 30 days about an axis passing
through its centre. The star undergoes an internal explosion and converts to a cyrus star.
Initial radius of the core is 1.0 ¥ 104 km, whereas final radius is 3.0 km. Determine the period
of rotation of the neutron star.
Solution During collapse of the star no external torque acts on it; it undergoes redistribution
of mass, resulting in change of moment of inertia. From conservation of angular momentum,
 
Li = L f
2 Ê 2p ˆ 2 Ê 2p ˆ
MRi2 Á ˜ = MR2f Á ˜
5 Ë Ti ¯ 5 Ë Tf ¯
2
Ê Rf ˆ
or Tf = Ti Á ˜ = 2.7 ¥ 10–6 days
Ë Ri ¯
Problem 3.65 A clutch consists of two discs A and B of moment of inertia 2I and I
respectively, one being the engine flywheel, the other one is the clutch plate. The discs are
initially rotating with angular velocities wA = w and wB = 2w as shown in Fig 3.168. When
the two discs are brought into contact, the discs rub against
each other and eventually reach a common angular velocity w.
wB
(a) Derive an expression for w. (b) What is the angular impulse
of friction on any one of the discs?
Solution There is no external torque on the system. The IB
frictional forces do exert torque on each disc but they are
internal forces for the system of two discs. Therefore, angular
momentum of the system is conserved.
wA
(a) From conservation of angular momentum, IA
 
Li = L f
IAwA + IBwB = (IA + IB)w
I Aw A + I Bw B 2Iw + I (2w ) 4
w = = = w Fig. 3.168
I A + IB 3I 3
442പWŚLJƐŝĐƐĨŽƌ//dͲ:͗DĞĐŚĂŶŝĐƐ//

*Note that we have taken angular velocity of the discs in the same direction; if they
have opposite rotational sense, the expression for w will be
I Aw A - I Bw B
w= =0
I A + IB
(b) Angular impulse is changed in angular momentum.
  Ê4 ˆ 2
|t A Dt| = |DL A|= (2I ) Á w ˜ - 2Iw = Iw
Ë3 ¯ 3

Problem 3.66 The device shown in Fig. 3.169 rotates


on the vertical axle as shown. The frame has negligible
mass as compared to the four masses each of mass m. m
Initial angular velocity of the system is w0. Due to an m
internal mechanism the spokes in the frame lengthen m
so that the radii of the masses become 2a. What will be m
the new angular velocity of the system?
(IIT-JEE Adv., 2014)
Solution There will be no torque on the system if
we ignore frictional torque at the axle. Therefore the
angular momentum of the system is conserved.
 
Linitial = Lfinal
Iinitialw0 = Ifinalw Fig. 3.169

4ma2w0 = 4m(2a)2w
w0
from which we have w =
4

Problem 3.67 A man of mass 100 kg stands at the rim of a turntable of radius 2 m, moment
of inertia 4000 kg-m2 mounted on a vertical frictionless shaft at its centre. The whole system
is initially at rest. The man now walks along the outer edge of the turntable with a velocity
of 1 m/s relative to the earth.
(a) With what angular velocity and in what direction does the turntable rotate?
(b) Through what angle will it rotate when the man reaches his initial position on the
turntable?
(c) Through what angle will it rotate when the man reaches his initial position relative to
wt
the earth? w m
Solution Let the man move anticlockwise.
(a) By conservation of angular momentum on the man-table
system,
 
Li = L f or 0 + 0 = Imwm + Itwt
I mw m v 1
wt = - where wm = = rad/s
It r 2 Fig. 3.170
ZŽƚĂƟŽŶപ443

1/2
= – 100(2)2 ¥
4000
1
=– rad/s
20
Thus, the table rotates clockwise (opposite to man) with angular velocity 0.05 rad/s.
(b) If the man completes one revolution relative to the table, then
qmt = 2p; 2p = qm – qt
2p = wmt – wtt (where t is the time taken)
2p 2p
t = =
(w m - w t ) 0.5 + 0.05
Angular displacement of the table is
2p
qt = wtt = – 0.05 ¥
0.55
2p
=– radian
11
The table rotates through 2p/11 radians clockwise.
(c) If the man completes one revolution relative to the earth,
then qm = 2p
2p 2p
Time = =
w m 0.5
During this time, angular displacement of the table,
2p
qt = wt (time) = – 0.05 ¥
0.5
p
qt = – radian
5
qt = 36° in clockwise direction.
Problem 3.68 A block of mass M rests on a rotating table that is rotating at constant angular
velocity w. A string runs from the block through a hole in the centre of the table down to a
hanging block of mass m. The coefficient of friction between the first block and the turntable
is m. (see Fig. 3.171). Find the largest and the smallest values of the radius r for which the
first block will remain at rest relative to the turntable.
N
N
T Mw 2r1
mN T
mN
Mw 2r2

T Mg T Mg

mg mg
(a) FBD in rotating frame (b) FBD in rotating frame

Fig. 3.171
444പWŚLJƐŝĐƐĨŽƌ//dͲ:͗DĞĐŚĂŶŝĐƐ//

Solution For minimum value of r = r1, the tendency of motion of the block M is towards
the centre, and therefore frictional force will act away from the centre.
For the equilibrium of M, we have
T = M w2r1 + mN ...(i)
where N = Mg
For the equilibrium of block m we have
T = mg ...(ii)
From (i) and (ii), we get
mg = Mw2r1 + mMg
or r1 = (mg – mMg)/(M w2)
For maximum value of r = r2, the tendency of motion of block M will be away from the
centre, therefore frictional force acts towards the centre.
Doing similar treatment for this case,
T + mMg = Mw2r2
and T = mg
After solving for r2, we get
r2 = (mg + mMg)/(Mw2) Ans.
Problem 3.69 Disks A and B are mounted on a shaft A B
MM and may be connected or disconnected by a clutch
L, as shown in Fig. 3.172. The moment of inertia of disk A
C
is one half that of disk B. With the clutch is disconnected, M MM
A is brought up to an angular velocity w0. The accelerat-
ing torque is then removed from A and it is coupled to
disk B by the clutch (bearing friction may be neglected).
It is found that 2000 J of heat are developed in the clutch
when the connection is made. What is the original kinetic Fig. 3.172
energy of disk A?
Solution Let IA = I and IB = 2I
In the connection of disk A with disk B, the angular momentum remains constant
or Iw0 + 0 = (I + 2I)w
w0
which gives w =
3
The change in KE evolved as heat, i.e.,
1 2 1
Iw 0 - (3I )w 2 = 2000
3 2
Substituting the value of w in the above equation, we get
2
1 2 1 Ê w0 ˆ
Iw 0 - 3I Á ˜ = 2000
3 2 Ë 3¯
ZŽƚĂƟŽŶപ445

1
or I0w2 = 2000
3
1
\ Iw02 = 3000 J
2
1
where Iw02 in the initial energy of the disk A.
2

ROLLING MOTION
Consider a wheel moving along a straight track, the centre of the wheel moves forward in
pure translation. A point on the rim of the wheel, however traces out a complex curve called
cycloid. We can analyse the motion of a rolling wheel as a combination of translation and
pure rotation.
Cycloid
P

vCM vCM
O O O O O
w P P

P P
s = 2pR

Fig. 3.173

In one complete rotation of the wheel, the CM of the wheel moves a distance s = 2pR. In
this motion there is no relative motion between the point of contact of the moving body and
the surface. If T is the time to complete a rotation, then we have
s 2p R
=
T T
or vCM = wR
Thus, in pure rolling, the velocity of the point of contact is zero and the velocity of centre
of mass is vCM = wR. If the wheel moves through a distance greater than 2pR in one full
rotation, then vCM > wR and the wheel slips forward. This type of motion occurs when you
apply sudden brakes to the car. The car stops after a long distance but the wheels rotate
only a little during the period.
When the wheel moves a distance shorter than 2pR in full rotation, vCM < wR and the
wheel slips backward. It happens when you drive the car on muddy rod, then wheels rotate
more than the forward motion of the car.

Equation of Cycloid
Suppose the motion of the wheel is along the positive x-direction in xy-plane with ox along
the horizontal and oy along the vertical, the origin O is at the centre of the wheel. Let R be
the radius of the wheel, – w, its angular velocity (being clockwise) and – q, the angle that a
446പWŚLJƐŝĐƐĨŽƌ//dͲ:͗DĞĐŚĂŶŝĐƐ//

point P on its axis makes with the x-axis at O at t = 0. y


In time t the point is given by q
O t=0
x
x = R cos (– wt – q) = R cos (w t + q). wt
y = R sin (– w t – q) = – R sin (w t + q).
For an observer on the ground, the velocity of CM Fig. 3.174
of the wheel is wR. Hence, after time t, the position of
the point is given by the coordinates
x = vCM t + R cos(wt + q),
= wRt + R cos(w t + q),
y = R – R sin(wt + q)

Velocity of a Point on the Rolling Body


Consider a point P on a rolling body, the velocity of point P is the vector sum of velocity
due to translation and due to rotation. Thus,
   P
v p = v translation + v rotation q
vCM
   q vCM
= v CM + w ¥ r vc = w R

2 2
or vP = vCM + vCM + 2vCM vCM cos q

q
or vP = 2vCM cos . Fig. 3.175
2

Geometrical Method of Finding Velocity of any Point on a Rolling Wheel


We know that rolling motion is the combination of translation and rotation. Figure 3.176(a)
shows purely translation motion: every point on the wheel moves with a speed vCM. Figure
3.176(b) shows purely rotational motion: every point on the wheel rotates about the centre
with angular speed w. We also have for vCM = wR.
vCM vCM 2vCM

vCM vCM vCM


O + O fi O
w
vCM
2v CM

vCM
–vCM
P
(a) Translation (b) Rotation (c) Rolling

Fig. 3.176

It is clear from Fig. 3.176(c) that the point of contact of the wheel (point P) is stationary,
i.e., there is no relative motion between the point of contact of the body and the surface.
And the topmost point is moving at speed 2vCM, faster than any other point on the wheel.
ZŽƚĂƟŽŶപ447

Note: w
vCM
wR
1. If the point of contact of the surface is moving with velocity u vCM
with respect to the ground, then u
Fig. 3.177
vCM – wR = u
2. For accelerated surface a
aCM
aCM – aR = a
a

Fig. 3.178
Kinetic Energy of Rolling Wheel
Let us calculate the kinetic energy of the rolling wheel as measured by the stationary
observer.
KE of rolling wheel = KE associated with translational motion + KE associated with the
rotation motion
1 2 1
or Krolling = MvCM + I CMw 2
2 2
For pure rolling, vCM = wR

1 1
\ Krolling = M(w R)2 + I CMw 2
2 2
1
= (MR2 + ICM)w2
2

Body ICM Rolling KE


2
Ring MR MR2 w 2

MR2 3
Disc MR2w 2
2 4

2 7
Sphere MR2 MR2w 2
5 10

As we know(ICM + MR2) is the moment of inertia of the wheel about the point of contact
P, say it is IP. Thus,
1
Krolling = I pw 2
2

IMPORTANT POINTS
Case I. If the body rolls on a stationary horizontal surface.
For pure rolling, point of contact P should be at rest; therefore,
vCM – wr = 0
i.e., vCM = wr [see Fig. 3.179(a)]
448പWŚLJƐŝĐƐĨŽƌ//dͲ:͗DĞĐŚĂŶŝĐƐ//

Case II. If the body rolls on a moving horizontal surface.


The point of contact P must be stationary with respect to surface.
vCM – wr = u [see Fig. 3.179(b)]
wr – vCM = u [see Fig. 3.179(c)]

r
w w
w vCM vCM vCM

r r
wr vCM
wr vCM wr vCM
P P P
u u

(a) (b) (c)

Fig. 3.179

Notes:
I: Similarly, for pure rolling motion on a stationary horizontal surface, the linear
acceleration a of the centre of mass should be related to the angular acceleration a as
a = a r.
II.
(i) vCM > wr, then the motion will be rolling with sliding.
(ii) If vCM < w r, then the motion will be rolling with slipping.

Direction of Friction in Various Cases


Case I. If vCM > w r then frictional force (FK) acts in w
negative direction.
(+ve direction)
If vCM < wr then frictional force (FK) acts in positive r
direction. wr vCM
If vCM = w r then FK and FS do not act.
Case II. If vCM > wr then frictional force (FK) acts in Fig. 3.180
negative direction.
If vCM < w r then frictional force (FK) acts in positive w
direction.
If vCM = wr then there is no relative motion between the +ve
vCM r
contact surface but tendency of relative motion is there due
to mg sin q so FS acts in negative direction.

(a)
ZŽƚĂƟŽŶപ449

Case III. If vCM > w r then frictional force (FK) acts in positive w vCM
direction.
+ve
If vCM < wr then frictional force (FK) acts in negative r
direction.
If vCM = w r then FS acts in negative direction.

Angular Momentum of Rolling Wheel q


(b)
Angular momentum of rolling wheel about an axis passing
through point of contact P and perpendicular to the plane Fig. 3.181
of wheel:
  
L = L translation + Lrotation
   w v
CM
= m (R ¥ v CM ) + I CM w O

or L = mwR2 + ICMw
P
or L = (ICM + mR2)w = IP w
Fig. 3.182
Rolling as Pure Rotation
Q
1
For a rolling wheel, KE = I pw2. Thus, rolling can be viewed as
2
vCM
pure rotation with angular velocity w, about an axis that always S
O
extends through point of contact P. It is clear from the figure that,
velocity of point P, vP = 0
vO = wR P w
vQ = w(2 R) = 2wR = 2vCM Rotational axis through P

vs = w ( 2R) = 2wR = 2vCM Fig. 3.183

Acceleration of a Point on a Rolling Wheel


Consider a rolling wheel; each point on the periphery of the wheel is rotating in a circle of
radius R, due to which centripetal acceleration of each point on the wheel is w2R. As the
wheel is moving with constant speed, so acceleration due to translation motion of each
point is zero.

w 2R w 2R
2 w 2
vCM w R w R w
O + O fi O
w 2R w 2R
2 2
w R w R

(a) Translation with (b) Rotation with


(c)
constant vCM constant w

Fig. 3.184
450പWŚLJƐŝĐƐĨŽƌ//dͲ:͗DĞĐŚĂŶŝĐƐ//

WORKED PROBLEMS
Problem 3.70 Consider a wheel rolls without slipping and its centre moves with constant
acceleration a. Find the acceleration of points O, P, Q and S when linear velocity of the
centre of wheel is v.
Solution
Q a 2a
a
Q Q
a 2
w R a 2
w R
y w v a w a 2
a w R a
R O S S + O a S + O
a a a 2
w R
2
w R a
a w 2R w 2R
x P a
a a a a
P (a) Acceleration due (b) Acceleration due (c) Centripetal (d) Acceleration in
to translation to rotation acceleration rolling motion

Fig. 3.185

aR = a
wR = v
From Fig. 3.185(d),
Resultant acceleration of point O,
aO = ai
aP = ai - ai + w 2 Rj
= w2Rj
aQ = ai + ai - w 2 Rj
= 2 ai - w 2 Rj
aS = ai - w 2 Ri - aj
= (a – w2R)i - aj
ar = (a + w2R)i + aj
Problem 3.71 Point on the periphery of the rolling body which has velocity equal to the
velocity of the centre of mass of body.
q
Solution We have vP = 2vCM cos
2
for vP = vCM;
120° 120°
q
vCM = 2vCM cos
2
q 1
or cos =
2 2
Fig. 3.186
or q = 120°
Problem 3.72 A disc of radius R spins with an angular speed w. Find the velocity of its CM
so that it rolls over the surface in each case as shown in Fig. 3.187.
ZŽƚĂƟŽŶപ451

w
w

v v
(a) (b)
R R

S vs = 0 S vs = 0

v
(c)
R

S v¢

Fig. 3.187

Solution
w
(a) vP = vQ
  v
or v PC + v CM = 0
or – Rw + v = 0 P
or v = Rw (to right) Ans.
  Q
(b) vP = vQ
  Fig. 3.188
or v PC + v CM =0
or Rw + v =0
or v = – Rw (from left) Ans.
 
(c) vP = vQ
  
or v PC + v CM = vQ
  
v CM = vQ - v PC
or vCM = v¢ – (– Rw)
or vCM = (v¢ + Rw) Ans.
Problem 3.73 A wheel is rolling without slipping on a horizontal surface
w
with angular speed w. What is the velocity of point of contact and the centre
of wheel. vc

Solution
P
(a) vP = 0
Fig. 3.189
(b) vP = vC – wR = 0
\ vC = wR
452പWŚLJƐŝĐƐĨŽƌ//dͲ:͗DĞĐŚĂŶŝĐƐ//

Problem 3.74 In the above question calculate the velocity at the topmost point and a point
on horizontal diameter.
vt = v
Solution At topmost point vr = v
velocity of translation vt = v
velocity of rotation vr = v
vt = v
Both vt and vr are in same direction.
So, vP = 2v
On horizontal diameter vr = v

Fig. 3.190
vP = v 2 + v 2 = 2 v [∵ vt ^ vr ]
Problem 3.75 An object is rolling without slipping on a horizontal
surface with speed v. Calculate velocity of a point on its circumference

v
at angular position q from lower vertical radius.

vr =

q

Solution

0
In this case the angle between vr and vt is 180 – q.

18
q
(180 - q ) P
So, vP = 2v cos vt = v
2
q Fig. 3.191
= 2v sin
2
Problem 3.76 Locate a point on the circumference of a rolling object which has the same
velocity as the velocity of centre.
Solution vP = vC
q q 1
2v sin = v or sin =
2 2 2
p
\ q = or 60°
3
Problem 3.77 Determine distance travelled by any point P on a rolling body in one
revolution.
vP P

w dl w
vCM
O O O
vCM P

P P

Fig. 3.192

Solution The speed of the point P at any angular position a, as shown in Fig. 3.192.
q
vP = 2vCM cos
2
ZŽƚĂƟŽŶപ453

Ê 180∞ - a ˆ P
= 2vCM cos Á ˜¯
Ë 2 q vP
O a
= 2vCM cos (90° – a/2)
a
= 2vCM sin
2
The distance travelled by point P in small time dt Fig. 3.193

dl = vp dt
a
= 2vCM sin (dt)
2
da da
We have w = fi dt =
dt w
a Ê da ˆ
\ dl = 2vCM sin Á ˜
2Ë w ¯
The distance travelled in one complete revolution
2p
a da
l = Ú 2vCM sin 2 w
0

2p
2vCM a
=
w Ú sin 2 da
0

2p
2w R È - cos a/2 ˘
= Í ˙
w Î 1/2 ˚ 0

= 4R[– cos p –(– cos 0)]


= 8R Ans.

Role of Friction in Rolling


Case 1. Take a wheel of radius R, and give it translational velocity vCM
and angular velocity w, so that vCM = wR and place the wheel on a w
rough horizontal surface. As vCM = wR, so there is no relative motion vCM
between points of contact of the wheel and the surface. The wheel will
continue its motion with constant velocity. P

Case 2. Consider a wheel acted upon by a force F at its centre chang- Rough surface
ing the velocity of the centre of the wheel or the angular speed about
Fig. 3.194
the centre, then there is a tendency for the wheel to slide at P, and a
static frictional force acts on the wheel, at the point of contact to oppose this tendency. At
subsequent motion the linear speed decreases due to force f and angular speed increases
due to its torque (fR).
454പWŚLJƐŝĐƐĨŽƌ//dͲ:͗DĞĐŚĂŶŝĐƐ//

For translation of wheel,


F – f = maCM ...(i)
For rotation of wheel
F
f ¥ R = Ia ...(ii)
aCM
For pure rolling of wheel, we have a = f P
R
Rough surface
Ia
\ f ¥ R = CM Fig. 3.195
R
IaCM
or f = ...(iii)
R2
Solving Eqs. (i) and (iii), we get
F
aCM = ...(iv)
Ê I ˆ
ÁË m + 2 ˜¯
R
Ê ˆ
Á 1 ˜
and f =F Á 2˜
…(v)
Á 1 + mR ˜
Ë I ¯
F- f fR
Note: From Eq. (i), we have aCM = and from Eq. (ii), a =
m I
As the maximum value of frictional force f can be f = flim = msN, so angular acceleration a

flim R
cannot be greater than a certain limit, i.e., a = amax = . In case, if F is large, then aCM
I
becomes larger than amax R. The wheel will have linear speed greater than the angular speed
and therefore the wheel will not have pure rolling motion.
The minimum value of coefficient of friction required for rolling of a body on horizontal
surface for given value of F
È ˘
Í 1 ˙
We have f =F Í 2
˙
Í mR ˙
ÍÎ 1 +
I ˙˚
The coefficient of friction (static)
È ˘
f F Í 1 ˙
mmin = = Í 2
˙
N Mg Í mR ˙
ÍÎ 1 + I ˙˚
ZŽƚĂƟŽŶപ455

Body MI mmin
F
Ring or hoop I = mR2
2mg

mR2 F
Disc or cylinder I=
2 3mg

2 2F
Sphere I= mR2
5 7 mg

Case 3. Let a wheel is thrown on a rough surface with initial velocity v0. The tendency of point
of contact P is to slide forward, so frictional force acts in backward direction of motion of
wheel. Due to the frictional force, the linear speed of centre of wheel decreases and angular speed
increases due to the torque exerted by frictional force. A condition is reached when v = wR, and then
the wheel will start in pure rolling motion. Thereafter friction stops acting.

f m N m mg
Retardation for translation motion, a = = = = mg ...(i)
m m m
t fR
Acceleration for rotational motion, a = = ...(ii)
I I

v0 v
w

f P v P
Rough surface
t=0 t=t
Translation Pure rolling
u = v0,w0 = 0

Fig. 3.196

Let the wheel start pure rolling after time t, then


v = v0 – at ...(iii)
and w = 0 + at ...(iv)

Important:
Why v = v0 – at? Because due to frictional force the linear speed of the centre of the wheel
decreases. Why w = w0 + a t as angular speed increases?

When the wheel starts rolling,


v = wR ...(v)
Work done by friction = – DKE
456പWŚLJƐŝĐƐĨŽƌ//dͲ:͗DĞĐŚĂŶŝĐƐ//

= Ki – Kf
1 Ê1 1 ˆ
= mv02 - Á mv 2 + Iw 2 ˜ ...(vi)
2 Ë2 2 ¯
we can solve the above equations to get unknowns.
Case 4. Let a rotating wheel, w = w0 is placed on a rough surface. The tendency of the point of contact
P is to slide backward, so the frictional force will act in forward direction. Because of this frictional
force, the centre of wheel starts accelerating, while torque of frictional force decreases the angular
speed. After some time the wheel starts rolling. Thereafter, friction stops acting.

w0 w v

wR P f P
Rough surface
t=0 t=t
Rotation Rolling

Fig. 3.197

f r m N m mg
Acceleration for translation motion, a = = = ...(i)
m m m
= mg
t fR
Retardation for rotational motion, a = = ...(ii)
I I
Let the wheel start pure rolling after time t, then we have
v = 0 + at ...(iii)
*w = w0 – at ...(iv)
When wheel starts rolling, we have
v = wR ...(v)
Work done by friction = – DKE
= Ki – Kf
1 2 Ê1 1 ˆ
Iw 0 - Á mv 2 + Iw 2 ˜
= ...(vi)
2 Ë2 2 ¯
We can solve the above equations to get unknowns.
Important:
Why v = u + at. The tendency of point of contact P is to slide backward so the frictional force
acts forward so the centre of wheel starts accelerating.
Why w = w0 – at as torque of frictional force decreases the angular speed?
By seeing these cases and analyzing the question you can make all types of problems.
ZŽƚĂƟŽŶപ457

WORKED PROBLEMS
Problem 3.78 A cylinder of mass M and radius R is resting on a horizontal platform (which
is parallel to the x–y plane) with its axis fixed along the y-axis and free to rotate about its
axis. The platform is given a motion in the x-direction given by x = A cos w t. If there is no
slipping, what is the maximum torque acting on the cylinder during its motion?
(IIT-JEE, 1988)
Solution We know that in rotational motion
t = Ia
1
But as tangential acceleration aT = Ra and Icylinder = MR2
2
Ê1 ˆÊa ˆ 1
So, t = Á MR2 ˜ Á T ˜ = MRaT ...(i)
Ë2 ¯Ë R ¯ 2
Now as the cylinder is moving without slipping with the platform, so, for it also
x = A cos wt
dx
or v = = - Aw sin w t y
y
dt x

dv x
or aT = = - Aw 2 cos w t ...(ii)
dt
Substituting aT from Eqs. (ii) in (i), we get Fig. 3.198

1
t = MRAw2 cos wt ...(iii)
2
As [cos w t]max = 1
1
So, |t|max = MRAw2
2

Problem 3.79 A tangential force F acts at top a thin spherical shell of mass m and radius
R. Find the acceleration of the shell if it rolls without slipping.
F

Fig. 3.199

Solution Let f be the force of friction between the shell and the horizontal surface.
For translational motion
F + f = ma ...(i)
458പWŚLJƐŝĐƐĨŽƌ//dͲ:͗DĞĐŚĂŶŝĐƐ//

For rotational motion, F


a
FR – fR = Ia = I [∵ a = Ra for pure rolling]
R
a
fi F–f =I 2 ...(ii)
R
Adding Eqs. (i) and (ii)
Ê a ˆ Ê 2 ˆ 5
2F = Á m + 2 ˜ a = Á m + m˜ a = ma
Ë R ¯ Ë 3 ¯ 3 f

5 È 2 ˘ Fig. 3.200
or F = ma Í\ I shell = mR2 ˙
6 Î 3 ˚
6F
fi a =
5m

Important Note:
So we assume that rolling occurs and that a friction f acts in the same direction as the applied force.
The choice of the direction of the assumed friction is arbitrary.
If the value of f obtained is + ve. Hence, the actual direction of the friction is the same as
the assumed direction if – ve then change the direction of friction.
Problem 3.80 A disc has mass M. A constant force acts at the top of disc. There is sufficient
friction for pure rolling. Find aC, a and friction force.
Solution In case of pure rolling motion, direction of friction can be assumed.
Let’s assume friction to be backward. F
For linear motion:
F – f = MaC ...(1)
For rotational motion: f
(Torque due to F and f are in the same direction, i.e., inwards)
Fig. 3.201
FR + fR = Ia
Ia
(F + f)R = C (pure rolling) ...(2)
R
From (1) and (2)
2F 2F 4F
aC = = =
I
M+ 2 MR2 3 M
M+
R 2R 2
aC 4F
a= =
R 3 MR
Friction force f:
Ê 4F ˆ
F–f=M Á
Ë 3 M ˜¯
F
\ f=–
3
Important: Since friction comes negative, it acts opposite to the assumed direction.
ZŽƚĂƟŽŶപ459

Problem 3.81 A horizontal force P is applied at a height h above the centre of a solid
cylinder of mass M, and radius R. Determine the force of friction in terms of h and show that
friction force is zero for h = R/2. Assume that the cylinder rolls without slipping.
Solution As you have learnt during rolling motion, friction
Y
force is static; its magnitude as well as direction are unknown.
Therefore, we assume friction to act towards left. If its value is
found to be positive, our assumption is correct, otherwise we P
h
rewrite the equations.
R x
The equations of motion are Mg
ÂFx = P – f = Ma ...(1) f
N
ÂFy = N – Mg = – 0 ...(2)
1
Ât = P ¥ h + f ¥ R = MR2a ...(3) Fig. 3.202
2
Here we have assumed clockwise torque to be positive. If the cylinder rolls without
slipping, a = Ra.
1
Now Eq. (3) becomes P ¥ h + f ¥ R = MRa ...(4)
2
From Eqs. (4) and (1), we eliminate a to obtain

Ê 2h ˆ
3f = P Á 1 - ˜
Ë R¯
Not that force of friction f is zero when (1 – 2h/R) = 0
i.e., h = R/2
Thus, rolling motion of the cylinder may take place even without friction.
*If P is applied at h = R, f = – P/3 which shows that friction force has reversed its direction; it
acts to the right.
For f – P/3, Eq. (3) yields
1 1
P ¥ R – P ¥ R = MR2a
3 2
4 P
a =
3 MR
a is positive, which indicates that cylinder rolls to the right.
*If force P is applied at the centre of mass, h = 0.
3f = P[1 – (2 ¥ 0)/R] = P
f = P/3
Friction force is positive, that means, assumed direction of friction force is correct. As P
is increased, the force of friction also increases; for large P it may exceed maximum possible
value of friction msN, which is not possible. Therefore, we will have to take a new assumption
that the cylinder slips. The equations of motion are now
460പWŚLJƐŝĐƐĨŽƌ//dͲ:͗DĞĐŚĂŶŝĐƐ//

ÂFx = P – mkNA = Ma
ÂFy = NA – Mg = 0
1
Ât = P ¥ h + (mkNA)R = MR2a
2
Problem 3.82 In the given Fig. 3.203 F = 10 N, R = 1 m, mass of F
the body is 2 kg and moment of inertia of the body about an axis F
passing through O and perpendicular to plane of body is 4 kg-m2. R O
O is the centre of mass of the body.
2R
1. If the ground is smooth, what is the total kinetic energy of
f IC
the body after 2 seconds?
(a) 100 J (b) 75 J Fig. 3.203
(c) 50 J (d) 25 J
2. If the ground is sufficiently rough to ensure rolling, what is kinetic energy of the body
now in the given time interval?
(a) 18.75 J (b) 25.67 J
(c) 16.67 J (d) None of these
Solution
1. When frictional force is absent, rigid body does not translate,
 
 Fexternal = 0. Therefore a CM = 0.
Taking torque about CM
F ¥ 2R – F ¥ R = ICMa
10 ¥ 1 = 4a
5
a = rad/sec2
2
w = at = 5 rad/sec
1 1
KEtotal = I CMw 2 = ¥ 4 ¥ 5 ¥ 5 = 50 J
2 2
2. Taking torque about IC
F ¥ 4R – F ¥ 3R = [ICM + M(2R)2]a
10 ¥ 1 = [4 + 2 ¥ (2 ¥ 1)2]a
10
a = rad/sec2
12
10 5
w = at = ¥ 2 = rad/sec
12 3
1
KEtotal = [I CM + M(2R)2 ]w 2
2
2
1 Ê 5ˆ
= [4 + 2 ¥ (2 ¥ 1)2 ] ¥ Á ˜
2 Ë 3¯
= 16.67 J
ZŽƚĂƟŽŶപ461

Problem 3.83 A sphere of mass M and radius r shown in Fig. w = v0/2r


3.204 slips on a rough horizontal plane. At some instant, it has
translational velocity v0 and rotational velocity about the centre r
(v0/2r). Find the translational velocity after the sphere starts pure v0
rolling.
Solution Given, velocity of centre = v0 fr

v0
Angular velocity about the centre = Fig. 3.204
2r
Here initial velocity of rotation w0 < v0/r, the sphere slips in forward direction. Frictional
force decelerates it to decrease its translational velocity v0 to a value v, which corresponds to
pure rolling. Frictional force increases angular velocity w 0 to a value w, which corresponds
to pure rolling and satisfies the relation v = w r.
fr
Deceleration of the centre of mass of the sphere a =
M
\ v = v0 – at
Ê f ˆ
or v = v0 – Á r ˜ t ...(i)
Ë M¯
t
and angular acceleration about centre a =
I
fr ◊ r 5 fr
or a = 2
=
(2 Mr /5) 2 Mr
w = w0 + at
5 fr
or w = w0 + t
(2 Mr )
v0 5 fr
or w = + t ...(ii)
2r (2 Mr )
fr
From Eq. (i) t = v0 – v
M
v
Put this value in Eq. (ii) also use w =
r
6 v0
We get v =
7

Alternative solution using conservation of angular momentum Let us consider the torque
about the initial point of contact A. The force of friction passes through this point and hence
its torque is zero. The normal force and the weight balance each other. The net torque about
A is zero. Hence, the angular momentum about A is conserved.
462പWŚLJƐŝĐƐĨŽƌ//dͲ:͗DĞĐŚĂŶŝĐƐ//

Initial angular momentum is,


L = LCM + Mrv0 = ICM w + Mrv0

Ê2 ˆÊv ˆ 6
= Á Mr 2 ˜ Á 0 ˜ + Mrv0 = Mrv0
Ë5 ¯ Ë 2r ¯ 5
Suppose the translational velocity of the sphere, after it starts rolling is v0. The angular
velocity is v/r. The angular momentum about A is,
L = LCM + Mrv

Ê2 ˆ Ê vˆ 7
= Á Mr 2 ˜ Á ˜ + Mrv = Mrv
Ë5 ¯Ë r¯ 5
6 7
Thus, Mrv0 = Mrv
5 5
6
or, v = v0
7
Problem 3.84 A solid sphere of radius r is gently placed on a
rough horizontal ground with an initial angular speed w 0 and
no linear velocity. If the coefficient of friction is m, find the time
t when the slipping stops. In addition, state the linear velocity v w0
and angular velocity w at the end of slipping.
Solution Let m be the mass of the sphere. Since it is a case of
Fig. 3.205
backward slipping, force of friction is in forward direction. Limiting
friction will act in this case.
f m mg
Linear acceleration, a = = = mg
m m v
t f ◊r 5 mg w0 fmax w
Angular retardation, a = = =
I 2 mr 2 2 r
5
Fig. 3.206
Slipping is ceased when v = rw
or (at) = r (w0 – at)
Ê 5 m gt ˆ
or mgt = r Á w 0 - ˜
Ë 2 r ¯
7
or mgt = rw0
2
2 rw 0
\ t =
7 mg
2
v = at = mgt = rw 0
7
v 2
and w = = w0
r 7
ZŽƚĂƟŽŶപ463

Alternative Solution using conservation angular momentum


Net torque on the sphere about the bottommost point is zero. Therefore, angular momentum
of the sphere will remain conserved about the bottommost point.
Li = Lf
\ Iw0 = Iw + mrv
2 2 2
or mr w 0 = mr 2w + mr(w r )
5 5
2 2
\ w = w 0 and v = rw = rw 0
7 7

Problem 3.85 A solid sphere is set into motion on a rough horizontal w


surface with a linear velocity v in the forward direction and an angular v
velocity v/R in the anticlockwise direction as shown in Fig. 3.207. Find the fr
linear speed of the sphere (a) when it stops rotating and (b) when slipping
finally ceases and pure rolling starts. Fig. 3.207

Solution Here friction provides retarding torque and therefore its angular velocity decreases with
time. Let it become zero after time t.
t f R m MgR
(a) a= = r =
I I 2
MR2
5
5 mg
=
2 R
Now using w = w0 – at
v Ê 5m g ˆ
or 0= -Á ˜t
R Ë 2R ¯
2v
\ t=
5m g
Let linear speed of the sphere become v in this time,
f r m mg
a= = = mg
m m
\ v¢ = v – at
2v
= v – (mg) ¥
5m g
3v
= Ans.
5
(b) Let the sphere have velocity v1 and angular 3v
w1
velocity w1 when it starts pure rolling 5
v1
w1 = 0 + at
fr fr
5 mg
w1 = 0 + t ...(i) t=0 t=t
2R
and v1 = v0 – at Fig. 3.208
464പWŚLJƐŝĐƐĨŽƌ//dͲ:͗DĞĐŚĂŶŝĐƐ//

3v
= - ( m g )t ...(ii)
5
After getting pure rolling
v1 = w 1R ...(iii)
3v Ê 5 mgt ˆ
\ - m gt = Á R
5 Ë 2R ˜¯
6v
or t =
35
Now from Eq. (i), we have
3v 6v
v1 = - mg ¥
5 35 m g
3v
= Ans.
7

ACCELERATED PURE ROLLING


ͳǤ౧‘ŽŽ‹‰‘–‹‘‘ˆ’‘‘Žƒ†Ї‡Ž
Take an example of a spool in which a tangential force F is acting on the axel of the spool.
In the first case, it is applied tangentially on the top of the axel and in the second case it is
applied tangentially at the bottom of the axel.
Case I. In this case sliding tendency of point of contact P is backward
F + f = maCM ...(i)
Fr – f R = Ia ...(ii)
a F
For pure rolling a = CM ...(iii)
R
r
After simplifying above equation we get,
Ê rˆ R
FÁ1+ ˜ v
Ë R¯ P
f
aCM =
I
m+ 2 Fig. 3.209
R
Case II. In this case sliding tendency of point of contact P is
forward
F – f = maCM ...(i)
fR – Fr = Ia ...(ii) r
aCM
For pure rolling motion, a = ...(iii) F
R R
f v
After simplifying the above equation we get, P

Ê rˆ Fig. 3.210
FÁ1- ˜
Ë R¯
aCM =
I
m+ 2
R
ZŽƚĂƟŽŶപ465

’‘”–ƒ–‹• —••‹‘ˆ‘”Ї‡Ž
w
2. Let a wheel vCM = wR land on a smooth inclined
vCM
plane. The centre of the wheel will accelerate due
to a force mg sin q, but the angular speed of wheel w
will remain constant as no torque is acting on the v¢CM mg
wheel about the axis of rotation. So, the motion
mg
of wheel will no remain pure rolling. Thus, the
body can have pure rolling only on rough inclined
Fig. 3.211
surface.
3. Let a wheel be placed on a rough inclined plane. The tendency of contact point P is to
slide down the inclined due to a net force down the plane. The friction will act up the
plane at the point of contact of the wheel. This frictional force constitutes a torque fR.
Due to which the wheel starts rotating in addition to translation. Here role of friction
is to transfer some part of translational KE into rotational KE. The mechanical energy
of pure rolling wheel remains constant.

‘ŽŽ‹‰‘‘—‰Š‹ Ž‹‡†Žƒ‡
Dynamical Method N
For translational motion of wheel
q f
mg sin q – f = maCM ...(i) sin R
mg
For rotational motion of wheel q
mg cos q
fR = Ia ...(ii) mg
q
aCM
For pure rolling motion, a =
R Fig. 3.212
IaCM
\ fR = ...(iii)
R

IaCM
or f = …(iv)
R2

Solving Eqs. (i) and (iv), we get


È ˘
mg sin q Í g sin q ˙
aCM = or aCM =Í ˙
Ê I ˆ Í1 + I ˙
ÁË m + 2 ˜¯ ÍÎ ˙˚
R mR2

Unbalanced load
Short-cut method: aCM =
{Inertia of translation + Inertia of rotation}
mg sin q
=
(m + I/R2 )
466പWŚLJƐŝĐƐĨŽƌ//dͲ:͗DĞĐŚĂŶŝĐƐ//

Velocity of CM after falling a height h


v2 = 0 + 2aCM(s)
È ˘
Í g sin q ˙ h
= 2Í
I ˙¥
Í1 + ˙ sin q
ÍÎ mR2 ˙˚

2 gh
or v =
I
1+
mR2
The minimum frictional force and coefficient of friction required to cause pure rolling of
IaCM
a body can be obtained from, fmin = .
R2

Body Moment Acceleration IaCM fmin


fmin = mmin=
of inertia Ê g sin q ˆ R2 N
aCM Á 2˜
Ë 1 + I/mR ¯ N = mg cos q

g mg sinq tan q
Ring MR2 sinq
2 2 2

MR2 2 mg sin q tan q


Disc/Cylinder g sin q
2 3 3 3

2 5 2 2 tan q
Sphere MR2 g sin q mg sin q
5 7 7 7

‹‡ƒ‡–‘‡ƒ Š–Ї‘––‘‘ˆƒ  Ž‹‡†Žƒ‡


1
Ê ˆ2 1 1
Á 2 gh ˜ g sin q t 1 Ê 2h ˆ 2 Ê I ˆ2
V = u + at fi Á = \ t= ÁË 1 + ˜
I ˜ I Á ˜
sin q Ë g ¯ MR2 ¯
Á 1+ ˜ 1+
Ë MR 2¯
MR2
1
1 Ê 2h ˆ 2
If the surface is frictionless then the time is .
sin q ÁË g ˜¯


Problem 3.86 A uniform disc of mass m and radius R is projected horizontally with velocity
v0 on a rough horizontal floor so that is starts off with a purely sliding motion at t = 0. After
t0 second, it acquires a purely rolling motion as shown in Fig. 3.213. (IIT-JEE, 1997)
ZŽƚĂƟŽŶപ467

(i) Calculate the velocity of the


centre of mass of the disc at t0.
(ii) Assuming the coefficient of fric- v0 v
tion to be m, calculate t0. Also
calculate the work done by the
frictional force as a function of
time and the total work done by t=0 t = t0
it over a time t much longer that (a)
t 0.
Solution
(i) During the time interval t = 0 to v0 v
w
t = t0 there is forward sliding;
so kinetic frictional force f acts
towards left. For time t = t0,
frictional force f will be zero, f = fmax = mmg f=0
because there is no relative t=0 t = t0
motion between the edge of the (b)

disc and the surface. Fig. 3.213


So, for time t < t0
f
Linear retardation, a = = m g (f = mmg)
m
t fR 2m g
and angular acceleration, a = = =
I 1 mR2 R a
2 a
Let v be the linear velocity and w the angular velocity of
the disc at time t = t0; then
v = v0 – at0 = v0 – mgt0 ...(1)
f = mmg
2 mgt0
and w = at0 = ...(2) Fig. 3.214
R
From pure rolling constraint, we have
v = Rw
i.e., v0 – mgt0 = 2mgt0 from Eqs. (1) and (2)
v0
fi t0 = 3m g
v
Substituting the value of t0 = 0 in Eq. (1), we have
3m g
Ê v ˆ
v = v0 – mg Á 0 ˜
Ë 3m g ¯
2
v = v0
3
468പWŚLJƐŝĐƐĨŽƌ//dͲ:͗DĞĐŚĂŶŝĐƐ//

(ii) Work done by friction: For t £ t0, linear velocity of disc at any time t is v = v0 – mgt and
2mgt
angular velocity is w = at = . From work-energy theorem, work done by friction
R
up to time t is equal to kinetic energy of the disc at time t minus kinetic energy of the
disc at time t = 0.
1 1 1
\ W = k F – KI = W = mv 2 + Iw 2 - mv02
2 2 2
2
1 1Ê 1 ˆ Ê 2 m gt ˆ 1
= m[v0 - m gt]2 + Á mR2 ˜ Á - mv02
2 2Ë 2 ¯ Ë R ˜¯ 2
1
= [mv02 + mm 2 g 2t 2 - 2mv0 m gt + 2mm 2 g 2t 2 - mv02 ]
2
mm gt
or W = [3 m gt - 2v0 ]
2
For t > t0, frictional force is zero, i.e., work done by friction is zero. Hence, the energy
will be conserved.
Therefore, total work done by friction over a time t much longer than t0 is total work
done up to time t0 (because beyond that work done by friction is zero), which is equal
to
mm gt0
W = [3 m gt0 - 2v0 ]
2
Substituting, t0 = v0/3mg, we get
mv0
W = [v0 - 2v0 ]
6
mv02
=–
6
Total work done by friction can be evaluated by this Method also
1 1 1
W = mv 2 + Iw 2 - mv02
2 2 2
2
1 1Ê 1 ˆv 1
= mv 2 + Á mr 2 ˜ 2 - mv02
2 2Ë 2 ¯r 2
1 1 1
= mv 2 + mv 2 - mv02
2 4 2
3 1
= mv 2 - mv02
4 2
2
3 Ê2 ˆ 1
= m Á v0 ˜ - mv02
4 Ë3 ¯ 2
1 2
=– mv02 ∵ V= v0
6 3
ZŽƚĂƟŽŶപ469

Problem 3.87 A ball (solid sphere) is thrown down an alley in such a way that it slides with
a speed v0 initially without rolling. Prove that it will roll without any sliding when its speed
falls to (5/7)v0. The transition from pure sliding to pure rolling is gradual, so that both sliding
and rolling take place during this interval also find distance S and number of revolution.
Solution Here the force of friction not only causes v to decrease but also produces a torque
which gives an angular acceleration a causing w to increase.
As the force of friction f = mR = m mg (as R = mg), the deceleration of the ball a = f/m =
mg and as for rotational motion of the ball t = Ia, i.e., fr = Ia [as t = fr]
fr 5 m g 2 2
or a = = [as f = mmg and I = mr ]
I 2 r 5
So, from equation of translatory motion v = u + at, we have R

v = v0 – mgt
[as u = v0 and a = – mg] ...(i)
and from equation of rotatory motion v
w = w0 + at, we have
f
w = 0 + (5mg/2r)t
[as w0 = 0 and a = 5mg/2r] ...(ii)
mg
Now as in the case of rolling without sliding v = rw, so
Fig. 3.215
(v0 – mgt) = r(5mg/2r)t, i.e., t = (2v0/7mg) ...(iii)
Substituting the value of (t) from Eq. (iii) in Eq. (i), we get
2v0 È 2˘ 5
v = v0 – mg ¥ = v0 Í1 - ˙ = v0
7mg Î 7˚ 7
Note: If during the period of transition from sliding to rolling the ball travels a distance S
then from the equation of motion
v2 = u2 + 2as we get
2
Ê5 ˆ
ÁË v0 ˜¯ = v02 - 2 m gs
7
12v02
S =
49 m g
If n is the number of revolutions made during the period of transition then from the
equation of motion.
q = w0t + 1/2 μ t2
q 5 mg 2v0
n = , w0 = 0, μ= , t=
2p 2 r 7mg
5v02
n =
98 m gr
470പWŚLJƐŝĐƐĨŽƌ//dͲ:͗DĞĐŚĂŶŝĐƐ//

Problem 3.88 A uniform solid cylinder of mass m and radius R is set in rotation about its
axis with an angular velocity w 0, then lowered with its lateral surface onto a horizontal plane
and released. The coefficient of friction between the cylinder and the plane is equal to m. Find:
(a) how long the cylinder will move with sliding; and
(b) the total work performed by the sliding friction force acting on the cylinder.

f N
Solution Acceleration for translation, a =
m
w0
mmg
=
m f
= mg mg

Fig. 3.216
t
Retardation for rotation a =
I
fR
=
I
mmgR
=
mR2
2
= 2mg/R
If t is the required time and v and w are linear and angular velocities after pure rolling,
then
w = w0 – at
and v = 0 + at
After getting pure rolling, v = wR
After solving equations, we get
w R
(a) t = 0
3m g
(b) Work done by friction W = Ki – Kf
1 2 È1 1 ˘
= Iw 0 - Í mv 2 + Iw 2 ˙
2 Î2 2 ˚

mw 02 R2
=– Ans.
6

Problem 3.89 A uniform disc of radius R is spinned to the angular velocity w and then
carefully placed on a horizontal surface. How long will the disc be rotating on the surface
if the friction coefficient is equal to m? The pressure exerted by the disc on the surface can
be regarded as uniform.
Solution To get frictional torque on the disc, take an element of width dr, and a distance r
from the centre of the disc. The frictional torque
ZŽƚĂƟŽŶപ471

dt = dFr
= m (dN)r dr

È mg ˘ r
= m Í 2 ¥ 2p rdr ˙ r
Îp R ˚

2 mmgr 2 dr dF
=
R2
R
2 mmg 2
R2 Ú0
\ t = r dr Fig. 3.217

2
= mmgR
3
t
The angular retardation, a =
I
2
mmgR
= 3
mR2
2
4 mg
=
3R
Now from w = w0 – a t, we have
w
t = 0
a
w
4m g
=
3R
3w R
= Ans.
4m g

Problem 3.90 A solid disc and a ring, both of radius 10 cm are placed on a horizontal table
simultaneously, with initial angular speed equal to 10p rad/s. Which of the two will start to
roll earlier? The coefficient of kinetic friction is mk = 0.2.
Solution Given, w0 = 10p rad/s.
f m mg
The acceleration for translation a = = = mg
m m
= 0.2 ¥ 9.8 w0
= 1.96 m/s2.
f
t fR m mg ¥ R
Angular retardation, a = = =
I I I Fig. 3.218
472പWŚLJƐŝĐƐĨŽƌ//dͲ:͗DĞĐŚĂŶŝĐƐ//

mR2
For disc I =
2
mmgR
\ a =
mR2
2
2mg
=
R
2 ¥ 0.2 ¥ 9.8
= = 39.2 rad/s2
0.1
For ring I = mR2,
m mgR m g
\ a = = = 1.96 rad/s2
mR2 R
If t is the time taken to start pure rolling, then
w = w0 – at, ...(i)
v = 0 + at ...(ii)
and v = wR ...(iii)
After solving the above equations, we get
w0
t =
Ê aˆ
ÁË a + ˜¯
R
10p
Thus, tdisc = = 0.53 s
Ê 1.96 ˆ
ÁË 39.2 + ˜
0.1 ¯
Similarly, tring = 0.80 s Ans.
Obviously disc begins to roll earlier than the ring.
Problem 3.91 A hollow sphere of radius 5.9 m is rotating about a horizontal axis at 10
rad/s. It is gently lowered on the ground (m = 0.34). How far does the sphere move before
it starts pure rolling?
Solution Acceleration for translation
f
a = w0
m
mmg f
=
m Fig. 3.219
= mg
t
Angular retardation a =
I
ZŽƚĂƟŽŶപ473

fR
=
I
mmgR
=
2
mR2
3
3 mg
=
2 R
For translational motion v2 = 0 + 2as
For rotational motion w2 = w02 + 2aq
where q = s/R
After getting pure rolling v = wR
After solving the above equations and substituting the values, we get
s = 0.69 m Ans.
Problem 3.92 If a solid sphere is projected along a rough horizontal floor with velocity v0
and having reverse spin with angular velocity w0, discuss all the possible situations.
N

w
w0
v0 v
r
r
mg = FK
P P¢

mg

(a) (b)

Fig. 3.220

LP¢ = LP
ICw – mvr = ICw0 – mv0r
2 2 2
mr w - mvr = mr 2w 0 - mv0 r
5 5
2 2
wr - v = w 0 r - v0
5 5
Condition (i): If v = 0 and w = 0, then body stops dead.
2 2
0= w 0 r - v0 , v0 = w0r
5 5
474പWŚLJƐŝĐƐĨŽƌ//dͲ:͗DĞĐŚĂŶŝĐƐ//

Condition (ii): If v = 0 and w > 0, then the body returns back.


2 2
w0r – v0 > 0, v0 < w0r
5 5
Condition (iii): If w = 0 and v > 0, then the body keeps on moving in forward direction.
2 2
v0 – w 0 r > 0 , v0 > w 0 r
5 5
Problem 3.93 A hoop of mass m is projected on a floor with linear velocity v0 and reverse
spin w0. The coefficient of friction between the hoop and the ground is m.
(a) Under what condition will the hoop return back?
(b) How far will it go?
(c) How long will it continue to slip when its centre of mass becomes stationary?
(d) What is the velocity of return?
Solution
(a) The acceleration of hoop, a = – mg.
mg
Angular acceleration of wheel = –
R
Linear velocity at time t, v = v 0 – m gt ...(1)
mg
Angular velocity at time t, w = w0 – t ...(2)
R
Let linear velocity be zero at t,
m0
So, 0 = v 0 – m gt or t1 =
mg
Angular velocity of the ring at t is
Ê mg ˆ Ê v ˆ
w = w0 – Á ˜ Á 0 ˜
Ë R ¯ Ë mg ¯
v0
= w0 – ...(3)
R
The hoop will return if it has reverse spin at the instant its linear velocity is zero. So
the condition for the hoop to return is
v v
w0 – 0 > 0 or w0 > 0
R R
v02
(b) Distance moved by hoop, S =
2m g
(c) Let t2 be the time for which the hoop slips on the floor and v be the return velocity.
Then v = m gt 2
mg
and w¢ = w – t2
R
ZŽƚĂƟŽŶപ475

for no slip condition is achieved when


mg mg
t2 = w - t2
R R
2m g v0
From Eq. (3), t 2 = w = w0 –
R R
R Ê v0 ˆ
or t2 = Á w 0 - ˜¯
2m g Ë R
w 0 R v0
(d) v = mg t 2 = -
2 2

Problem 3.94 A small mass particle is projected with an initial O


velocity v0 tangent to the horizontal rim of a smooth hemispherical v0
bowl at a radius r0 from the vertical centreline, as shown at point A
A. As the particle slides past point B, a distance h below A and a
r0
distance r from the vertical centreline, its velocity v makes an angle h
q with the horizontal tangent to the bowl through B. Determine q. r B
q
Solution Problem Solving Strategy. The forces on the particle v
are its weight and the normal reaction exerted by the smooth
surface of the bowl. Neither force exerts a moment about the axis O
O–O, so that angular momentum is conserved about that axis.
Fig. 3.221
Thus,
mv0r0 = mvr cos q
Also, energy is conserved so that E1 = E2. Thus,
1 1
mv02 + mgh = mv 2 + q
2 2

v = v02 + 2 gh

Eliminating v and substituting r2 = r02 – h2 give

v0r 0 = v02 + 2 gh r02 - h 2 cos q


1
q = cos–1
2 gh h2
1+ 1 -
v02 r02

Problem 3.95 A thin spherical shell of radius R lying on a rough horizontal surface is hit
sharply and horizontally by a cue. Where should it be hit so that the shell does not slip on
the surface?
Solution Let v be the velocity attained by its centre of mass and w be the angular velocity
about the centre of mass by the impact of the cue.
476പWŚLJƐŝĐƐĨŽƌ//dͲ:͗DĞĐŚĂŶŝĐƐ//

Suppose F is the force exerted by cue for small duration Dt, then
FDt = m(v – 0) ...(i)
and FhDt = I(w – 0) ...(ii)
From Eqs. (i) and (ii),
mv ¥ h = Iw
v h w
For pure rolling w = v
R R
2 v m
\ mv ¥ h = mR2 ¥
3 R Fig. 3.222
2R
fi h =
3

Problem 3.96 A circular hoop of mass m with a particle of mass N


m attached to its rim stands on a rough horizontal plane. Initially, Y
the particle is held at a point on the rim along the horizontal a
radius. Show that the hoop begins to roll or slide on the plane as X
CM
the coefficient of friction m > or < 2/7. O
Solution Moment of inertia of system about A R/2

a = (R/2) a mg
= Ihoop + Iparticle
= 2mR2 + m ( 2R)2 f = mN
mg
2
= 4mR
Fig. 3.223
The centre of mass of the system is at a distance R/2 from O.
R
Due to angular acceleration it has a downward acceleration a .
Equations of motion are: 2

ÂFx = mN = 2m(Ra) ...(1)


R
ÂFy = 2mg – N = 2m ◊ a ...(2)
2
Ât = mgR = 4mR2a ...(3)
Multiplying Eq. (2) by m and adding Eq. (1),
Ê mˆ
2mgm = 2mR Á 1 + ˜ a
Ë 2¯
g
On substituting the expression for a = from Eq. (3), we obtain
4R
Ê mˆ g
2mgm = 2mR Á 1 + ˜
Ë 2 ¯ 4R
1 + m/2
2m =
2
m = 2/7
Hence, if m > 2/7, the hoop will roll and if m < 2/7, it will slide.
ZŽƚĂƟŽŶപ477

Problem 3.97 A cue stick hits a cue ball horizontally at a distance x above the centre of the
ball. Find the value of x for which the cue ball will instantaneously roll without slipping.
Calculate the answer in terms of the radius R of the ball.
Solution Problem solving strategy: What would happen if the stick hits at the level of
the ball’s centre? Will the ball rotate? The answer is that the ball initially translates with
no rotation. If the stick hits below the centre, the ball initially has reverse spin. At a certain
value of x, the ball has just the right forward spin and forward acceleration to satisfy the
rolling constraint, nonslip condition. The torque (and hence its angular acceleration) exerted
on the ball depend on h. The linear r
acceleration a is F/m independent of F
x. For the ball to roll without slipping
from the start, set a = Ra (nonslip
x
condition) to find x. The weight and
normal force act through the centre of
mass and thus exert no torque about
it. The frictional force is much smaller
than the collision force of the stick Fig. 3.224
and can be neglected.
The torque about the centre of the ball,
t = Fx
Applying Newton’s second law for rotation and translation, we have
   
 F = ma and  t = Ia
F = ma and t = Fx = Ia ...(1)
a = Ra ...(2)
Expressing a and a in terms of the force F from step 2,
F Fx
= R
m I
I
Solving for h, x =
mR
2
For a sphere, I = mR2
5
2
mR2
5 2
x = = R
mR 5

ƒŽ›–‹ ƒŽ‹ŽŽ”‘„އ
Mark the correct choices:
(a) If the ball is struck at a height greater than 2R/5 it shows top spin.
(b) If the ball is struck at a height less than 2R/5 it shows back spin.
(c) Frictional force is much smaller than the impulse of the ball, therefore it is neglected.
(d) A ring will show pure rolling only if impulse acts tangentially at the topmost point.
478പWŚLJƐŝĐƐĨŽƌ//dͲ:͗DĞĐŚĂŶŝĐƐ//

Problem 3.98 A billiards ball, initially at rest, is given a sharp impulse by a cue. The cue
is held horizontally a distance h above the centreline as shown in Fig. 3.225. The ball leaves
the cue with a speed v0 and because of its forward energy
Ê 9v ˆ
eventually acquires a final speed of Á 0 ˜ , show that h = (4/5) R h
Ë 7 ¯
R where R is the radius of the ball.
Using Angular Momentum Method
fr
Solution The angular momentum acquired by the ball due
to the impulse given by a cue,
Fig. 3.225
(2mR2 )
L = mv0h = Iw0 = w0 ...(i)
5
v
Suppose initial angular velocity w0 < 0 , then the equations of motions are
R
v = v0 – at ...(ii)
f r m mg
where a = = = mg
m m
and w = w0 + at ...(iii)
t fr R 5m g
where a = = 2
=
I (2mR /5) 2R
When the ball starts pure rolling v = wR ...(iv)
from Eq. (ii)
v = v0 – at = v0 – mgt a = mg
from Eq. (iii)
w = w0 + at
5 mg 5 mg
w = w0 + t a=
2R 2R
5 mg
w – w0 = t
2R
2R(w - w 0 )
=t
5m g
Now from Eq. (i)
m g 2R(w - w 0 )
v = v0 –
5m g
2R(w - w 0 )
v = v0 –
5
(2Rw - 2Rw 0 )
v = v0 –
5
2Rw 2Rw 0
v = v0 – +
5 5
2Rw 2Rw 0
v+ = v0 +
5 5
ZŽƚĂƟŽŶപ479

5v + 2Rw 5v0 + 2Rw 0


=
5 5
5v + 2v = 5v0 + 2Rw0 v = Rw
7v = 5v0 + 2Rw0
5v0 + 2Rw 0
v =
7
5v0 + 2w 0 R
*v =
7
9v0
According to the given condition, v =
7
9v0 5v0 + 2w 0 R
\ =
7 7
2v0
or w0 =
R
Substituting this value in Eq. (i), we get
4
h = R
5

Problem 3.99 A billiards ball, initially at rest, is given a sharp F


impulse by a cue. The cue is held horizontally at a distance h
h above the centreline as shown in Fig. 3.226. The ball leaves
the cue with a speed v0 and because of its forward english
9
(backward slipping) eventually acquires a final speed v0.
Show that 7
Fig. 3.226
4
h= R
5
where R is the radius of the ball.
Using Impulse Method (an Easier Method)
Solution Let w0 be the angular speed of the ball just after it leaves the cue. The maximum
friction acts in forward direction till the slipping continues. Let v be the linear speed and w
the angular speed when slipping ceases.
v
\ v = Rw or w =
R

F
h
9
v= v0
w0 7
R w

fmax

Fig. 3.227
480പWŚLJƐŝĐƐĨŽƌ//dͲ:͗DĞĐŚĂŶŝĐƐ//

9
Given, v = v0 ...(i)
7
9 v0
\ w = …(ii)
7 R
Applying, Linear impulse = change in linear momentum
\ F dt = mv0 ...(iii)
Angular impulse = change in angular momentum
\ t dt = Iw0
2
or mR2w 0
Fh dt = ...(iv)
5
Angular momentum about the bottommost point will remain conserved.
i.e., Li = Lf
or Iw0 + mRv0 = Iw + mRv

2 2 Ê9v ˆ 9
\ mR2w 0 + mRv0 = mR2 Á 0 ˜ + mRv0
5 5 Ë7 R¯ 7
Solving Eqs. (iii), (iv) and (v), we get
4
h = R
5

Problem 3.100 A plank of mass M is placed on a smooth surface m


over which a cylinder of mass m (= M) and radius R = 1 m is
placed as shown in Fig. 3.228. Now the plank is pulled towards M
the right with an external force F (= 2Mg). If the cylinder does F
not slip over the surface of the plank, find the linear acceleration
of the plank and the cylinder and the angular acceleration of the Fig. 3.228
cylinder. (Take g = 10 m/s2) (WBJEE, 2011)
Solution Here we are given that cylinder does not slip over the plank surface, it is the
case of pure rolling, we can use friction on the cylinder in any direction. Here we choose
towards the right.
As friction is acting on the cylinder towards the right, it a2
must be towards the left on the plank as shown in the force m
diagram of Fig. 3.229. mai a
a1
Let the plank move towards the right with an acceleration f f
a1, the cylinder will experience a pseudo force ma1 in the M
F
left direction, due to which it will roll towards the left with
respect to the plank with an acceleration a2. As we have used
pseudo force, a2 must be with respect to the plank. Let its Fig. 3.229
angular acceleration during rolling be a, we have a2 = Ra.
For translational motion of the plank, we have
F – f = Ma1 …(i)
ZŽƚĂƟŽŶപ481

For translational motion of the cylinder with respect to the plank, we have
ma1 – f = ma2 …(ii)
For rotational motion of the cylinder with respect to the plank, we have
fR = Ia
Ê1 ˆÊa ˆ
fR = Á mR2 ˜ Á 2 ˜
Ë2 ¯Ë R¯
1
or f = ma2 …(iii)
2
From Eqs. (i) and (ii), we get
1
ma1 – ma2 = ma2
2
3
a1 = a2 …(iv)
2
Using Eqs. (i), (iii) and (iv), we get
1 3
F– ma2 = Ma2
2 2
2F
a2 = = 10 m/s2
3M + m
3F
From Eq. (iv), a1 = = 15 m/s2
3M + m
As we have already discussed that the value of a2 is relative to the plank, the net
acceleration of the cylinder will be given as a1 – a2.
Hence, the acceleration of the cylinder is
acylinder = a1 – a2 = 15 – 10 = 5 m/s2
The angular acceleration of the cylinder is
a = a2/R = 10 rad/s2
Problem 3.101 A rolling body is given a spin w0 and pushed
with a velocity v0 simultaneously on a horizontal surface (a v0
w0
plank, say). The horizontal surface is moved with a constant
R
velocity u. If the radius of gyration of the rolling body is k and
the coefficient of kinetic friction between the rolling body and u
horizontal surface is m, find the time after which the body starts
pure rolling on the horizontal surface. Assume the ground as Fig. 3.230
smooth.
Solution If we assume forward friction on the rolling body, we can use all equations of the
pervious example for the rolling body.
482പWŚLJƐŝĐƐĨŽƌ//dͲ:͗DĞĐŚĂŶŝĐƐ//

At the time of pure rolling


vP = vQ, where vP = – Rw + v and vQ = u v
C
This gives v – Rw = u …(i) f w
P
mgR vP
Directly substituting v = v0 + mgt and w = w0 – 2 t
k Q vQ
from the pervious example in Eq. (i), we have
Ê m gR ˆ
v0 + mgt – R Á w 0 - 2 t˜ = u Fig. 3.231
Ë k ¯
u + Rw 0 - v0
This gives t= Ans.
Ê R2 ˆ
mg Á1+ 2 ˜
Ë k ¯

–—†‡–ƒ•
s &IND THE FORCE REQUIRED TO MAINTAIN THE VELOCITY OF THE PLANK NEGLECTING THE FRICTION
between the plank and ground.
u + Rw 0 - v0
Ans. mmg for 0 £ t £ t0 and zero for, t > t0 where t0 =
Ê R2 ˆ
mg Á1+ 2 ˜
Ë k ¯

Problem 3.102 A rolling body of mass m, radius R and


radius of gyration k is spun with an angular velocity w0 and v0 w0
simultaneously projected with a linear velocity v0 on a plank of
mass M. If initially the plank is projected with a velocity u0 on
a smooth horizontal plane, find the time after which the body u0
starts pure rolling on the plank.
Solution In comparison to the previous example, the force Fig. 3.232
and kinematical equations for the rolling body are exactly the
same. a1
Law of friction: f = mN ...(i) C v w
mg
Force equation of m: f = ma1 ...(ii) N
P
N – mg = 0 ...(iii) f f
a2 Q u
Linear kinematics for m: v = v 0 + a 1t ...(iv)
Force equation for M: – f = – Ma2 ...(v) N
Kinematics for M: u¢ = u0 – a2t ...(vi)
Fig. 3.233
Torque equation for m: fR = mk2a ...(vii)
Rotational kinematics for m: – w = – w0 + at ...(viii)
Condition for rolling: vP = vQ, where vP = vP0 + v0 = – Rw + v and vQ = u, This gives
u = v – Rw ...(ix)
Substituting N = mg from Eq. (iii) in Eq. (i), we have f = mmg. Then substituting f = mmg
in Eq. (ii), we have a1 = mg. Now substituting a1 = mg in Eq. (iv), we have
v = v0 + mgt ...(a)
ZŽƚĂƟŽŶപ483

mmg
Substituting f = mmg in Eq. (v), we have a2 =
M
Then substituting a2 in Eq. (vi), we have
mmg
u = u0 – t ...(b)
M
mgR
Substituting f = mmg in Eq. (vii), we have a =
k2
mgR
Now substituting a = 2 in Eq. (viii), we have
k
mgR
w = w0 – t ...(c)
k2
Finally, substituting v from Eq. (a), u¢ from Eq. (b) and w from Eq. (c), in Eq. (ix), we have

m Ê m gR ˆ
u0 – m gt = (v0 + m gt) - R Á w 0 - 2 t˜
M Ë k ¯
u0 + Rw 0 - v0
This gives t = Ans.
Ê m R2 ˆ
mg Á1+ +
Ë M k 2 ˜¯

Problem 3.103 A solid sphere of mass M is R


Rough(m) Solid sphere (M)
placed on the top of a plank of the same mass, w0
after giving an angular velocity w0 at t = 0. (a) Find
Plank (M)
the velocity of the plank and the sphere when the
sphere starts rolling. (b) Find the displacement of
the plank till the sphere starts pure rolling. Smooth ground

Solution Fig. 3.234

(a) Step 1: Free-body diagram,

Mg

mMg

mMg N

Fig. 3.235
484പWŚLJƐŝĐƐĨŽƌ//dͲ:͗DĞĐŚĂŶŝĐƐ//

Step 2: Equation of motion,


Plank:
mMg = MA fi A = mg (backward)
(A will be positive)
Sphere:
mMg = MA fi a = mg (forward)
(a will be positive)
Step 3: Torque equation,
Ê2 ˆ
mMg R = Á MR2 ˜ a
Ë5 ¯
5 mg
μ = (anticlockwise)
2 R
(it will be negative)
Step 4: Constraint relation
VP = V = wR – v
v
Ê 5 mg ˆ
(0 + mgt) = Á w0 - t R - (0 + m gt)
2 R ˜¯
V P
Ë
Ê 5 ˆ 9 m gt
ÁË 2 m gt + m gt˜¯ = w0R fi = w0R Fig. 3.236
2 2
2w 0 R
fi t =
9m g
Velocity of the plank when rolling starts,
V = 0 + mgt
2w 0 R 2
Vplank = mg ¥ = w0R
9m g 9
Velocity of the sphere when rolling starts,
2
Vsphere = 0 + mgt = w 0 R
9
2
1 2 1 Ê 2w 0 r ˆ 2 w 02 r 2
(b) sP = a1t = m g Á =
2 2 Ë 9 m g ˜¯ 81 m g

Problem 3.104 A plank of mass M is moving on a smooth r


horizontal surface with speed v0. At t = 0 a sphere of mass m
m and radius r is gently paced on it and simultaneously F v0
M
a constant horizontal force F is applied on the plank in
the opposite direction of v0. Find the time at which the
sphere starts rolling on the plank. The coefficient of friction Fig. 3.237
between the plank and the sphere is m.
Solution Till rolling starts kinetic friction acts on the bodies. FBD of sphere for no vertical
acceleration N = mg ...(i)
ZŽƚĂƟŽŶപ485

fi fk = mmg (as fk = mN) N


fi a = mg
v = mgt
fk
m mgr m mgrt
Also, a = fiw =
I I mg
The velocity of the lowermost point of the sphere at time t is
Fig. 3.238
vs = v + wr
mmgr 2
vs = mgt + t
I
The FBD of the plank N
Ê F + fk ˆ Ê F + m mg ˆ fk
fi ap = – Á = -Á
Ë M ˜¯ Ë M ˜¯ F v

Ê F + m mg ˆ
vp = v0 – Á t N1
Ë M ˜¯
For no slipping v =v s p
Fig. 3.239
2
m mgr Ê F + m mg ˆ
fi mgt + t = v0 - Á t
I Ë M ˜¯
v0
fi t =
È7 Ê F + m mg ˆ ˘
Í 2 m g + ÁË M ˜¯ ˙
Î ˚
Problem 3.105 Find the acceleration of a system consisting m
of a cylinder of mass m and radius R and a plank of mass M
placed an a smooth surface if it is pulled with a force F as M F
shown in Fig. 3.240. Given that sufficient friction is present
between the cylinder and the plank surface to prevent sliding Fig. 3.240
of the cylinder.
Solution F – T – f = Ma ...(i)
T – f = Ma ...(ii)
fR 2f
a = = ...(iii)
1/2mR2 mR
Constraint relation aplank = ap
Ra – a = a
Ra = 2a ...(iv)
a
P
f
T F
T
a
P f

(a) (b)

Fig. 3.241
Solving these equations, we get
F
a=
3m + M
486പWŚLJƐŝĐƐĨŽƌ//dͲ:͗DĞĐŚĂŶŝĐƐ//

Problem 3.106 In Fig. 3.242, a sphere of radius 2 m


rolls on a plank. The accelerations of the sphere and
2m
the plank are indicated. The value of a is
(WBJEE, 2013) a2 = 4 m/s2
2 2 a1 = 2 m/s2
(a) 2 rad/s (b) 4 rad/s
(c) 3 rad/s2 (d) 1 rad/s2
Solution As sphere rolls, the lowest point of the Fig. 3.242
sphere should have the same acceleration as the
plank.
Hence, a1 = aR – a2
2 = 2a – 4 fi a = 3 rad/s2 Ans.
R Disc (m)
Problem 3.107 A plank of mass M,
whose top surface is rough with coeffi- Rough (m)
cient of friction m is placed on a smooth v0
ground (Fig. 3.243). Now a disc of mass
m = M/2 and radius r is placed on the Plank (M)
plank. The disc is now given a velocity
V0 in the horizontal direction, at t = 0.
Smooth ground
1. Find the time when the disc starts
rolling. Fig. 3.243
2. Find the velocity of the plank and the disc up to that time.
3. Find the distance travelled by the plank up to this instant.
4. Find the work done by the friction force up to this instant.
Solution Step 1: Free-body diagram,

mg Mg
N
mmg
O

mmg N

(a) (b)

Fig. 3.244

Step 2: Equation of motion


For the plank: mmg = MA
mmg
fi A = (towards right) …(i)
M
mg
fi A = (as m = M/2) (will be positive)
2
ZŽƚĂƟŽŶപ487

For the disc: mmg = ma


a = mg (backwards, towards left)
(will be negative)
Step 3: Torque equation
mmgR = Ia
mR2
mmgR = a
2
2mg
a = (clockwise)
R
(will be positive)

Step 4: Constraint relations w

At the time of rolling, point P should not slide with v


respect to the plank.
P V
V = v – wR
(0 + at) = (v0 – at) – (0 + at)R
Fig. 3.245
Now substituting the values of A and a, we can
calculate t
mmg Ê 2m g ˆ
t = (v0 - m gt) - Á t R
M Ë R ˜¯
Ê m mg ˆ
ÁË + m g + 2 m g ˜ t = v0
M ¯
v0 v
which gives t = = 0 (as m = M/2)
Ê m ˆ 5m g
m g Á + 3˜
ËM ¯
Velocity of the plank when rolling starts,
Ê m g ˆ Ê v0 ˆ v0
V = At = Á =
Ë 2 ˜¯ ÁË 5 m g ˜¯ 10
Velocity of the disc when rolling starts,
v = v0 + at
Ê v ˆ 4
= v0 + (– mg) Á 0 ˜ = v0
Ë 5m g ¯ 5
Angular velocity at the time of rolling,
Ê 2 m g ˆ Ê v0 ˆ 2 v0
w = at = Á =
Ë R ˜¯ ÁË 5 m g ˜¯ 5 R
Distance travelled by the plank,
2
1 2 1 Ê m g ˆ Ê v0 ˆ v02
Splank = At = Á ˜ =
2 2 Ë 2 ¯ ÁË 5 m g ˜¯ 100 m g
Work done by friction force,
Wfriction = DKtranslation + DKrotation
488പWŚLJƐŝĐƐĨŽƌ//dͲ:͗DĞĐŚĂŶŝĐƐ//

= [DKdisc + DKplank]translation + DKrotation

ÈÊ 1 1 ˆ Ê1 ˆ Ê1 ˆ˘
= ÍÁ mv 2 - mv02 ˜ + Á MV 2 - 0˜ + Á Iw 2 - 0˜ ˙
ÎË 2 2 ¯ Ë2 ¯ Ë2 ¯˚

Substituting the values of v, V and w, we get the work done by friction, i.e.,
13
Wfriction = – Mv02
200

Problem 3.108 A sheet (annular) is given velocity V0 and angular


w0R = v0
speed w0 on a plank. The plank is also given velocity V0. Find
velocity of the centre of mass when pure rolling starts on the plank m
V0
(see Fig. 3.246).
Solution Velocity of bottom point is V = V0 + w0R = 2V0. The V0
bottom point of sheet will slip forward. m
By this we decide the direction of friction.
Fig. 3.246

V0 Vc
f
V0 V
f

Fig. 3.247

ft
For plank V = V0 +
m
ft
For sheet V c = V0 –
m
fRt
w = – w0 +
mR2/2
When slipping stops
V = Vc – wR
3V0
From the above equations we can get Vc =
4
R
Problem 3.109 The ring and plank both given velocity V0 in
opposite directions. Find velocity of the centre of mass when pure
m V0
rolling motion starts.
Solution In this case we can easily decide the direction of friction. V0
m
Inward direction is taken positive.

Fig. 3.248
ZŽƚĂƟŽŶപ489

m; R

V0 Vc
(+)
f
V0 f
m V

Fig. 3.249

For ring
ft
Vc = V0 – ...(1)
m
fRt
w =0+ ...(2)
mR2
For plank
ft
V = – V0 + ...(3)
m
when slipping stops V = Vc – wR
V0
From the above equations we get Vc =
3

Problem 3.110 A plank of length 20 m and mass 1 kg is kept on a horizontal smooth surface.
A cylinder of mass 1 kg is kept near one end of the plank. The coefficient of friction between
the two surfaces is 0.5. Plank is suddenly given a velocity 20 m/s towards left.
1. Mark correct option(s)
(a) Initial acceleration of cylinder is 5 m/s2 towards left m
m
(b) Initial acceleration of cylinder is 5 m/s2 towards right
(c) Initial acceleration of plank is 5 m/s2 towards right l = 20 m
(d) Initial acceleration of plank is 5 m/s2 towards left
Fig. 3.250
2. Mark correct option(s)
(a) Pure rolling of cylinder takes place immediately
(b) Initially, cylinder slips and then pure rolling begins
(c) Pure rolling never begins
(d) Time of slipping is 1 sec
3. Velocity of plank when pure rolling begins is
(a) 10 m/s (b) 15 m/s (c) 20 m/s (d) 25 m/s
4. Time in which plank and cylinder separate
(a) 1 sec (b) 1.5 sec (c) 2.5 sec (d) 2 sec
MR2
Solution fR = a; f = 5 N; M = 1 kg a
2 a1
Ra = 10 m/s2
f a2 f
a1 = = 5 m/s2 f
M
Fig. 3.251
a2 = 5 m/s2
490പWŚLJƐŝĐƐĨŽƌ//dͲ:͗DĞĐŚĂŶŝĐƐ//

For cylinder
acontact = a1 + Ra
vcontact = (a1 + Ra) t = 15t ...(1)
For plank
vcontact = 20 – 5t ...(2)
From Eqs. (1) and (2), we get
t = 1 sec
Till the cylinder slips on the plank
1
Srel = urelt + arelt 2
2
1
= 20 ¥ 1 – ¥ 10 ¥ 12
2
= 15 m
Velocity of the plank when pure rolling begins
v = 20 – 5t = 15 m/s
Velocity of cylinder = a1t = 5 m/s
When pure rolling begins, friction force vanishes, velocity of the plank and the cylinder
is constant. After pure rolling begins
5 5 1
t = = = sec
vrel 10 2

Problem 3.111 A sphere of radius r and mass m has a linear velocity v0 directed to the
left and no angular velocity as it is placed on a belt moving to the right with a constant
velocity v1. If after sliding on the belt the
sphere is to have no linear velocity relative v0
to the ground as it starts rolling on the
belt without sliding, in terms of v1 and the v1
coefficient of kinetic friction mk between
the sphere and the belt, determine:
Fig. 3.252
(a) the required value of v0
(b) the time t1 at which the sphere will start rolling on the belt; and
(c) the distance the sphere will have moved relative to the ground at time t1.
Solution
(a) In ground reference frame the sphere has no relative velocity, but relative to belt it
performs pure rulling motion, i.e., it has only angular velocity w when pure rolling
starts. No slipping condition yields
v
wr = v1 or w = 1
r
Now we apply angular momentum conservation about the bottommost point.
Li = Lf (L = angular momentum)
2 v
or mrv0 = Iw = (mr 2 ) 1
5 r
ZŽƚĂƟŽŶപ491

2
fi v0 = v1
5
(b) Retardation of the sphere is due to friction force, till sliding occurs
m k mg
a = = mk g
m
Final linear velocity of the sphere has reduced to zero from the initial value v0. Hence,
0 = v0 – at1
v0 2v1
or t1 = =
a 5mk g
(c) Distance travelled relative to ground can be obtained from v2 = u2 – 2as.
We have 0 = v02 – 2(mkg)s
v02 2v12
or s = =
2 m k g 25 m k g

Problem 3.112 A cylindrical pipe of diameter


1 m is kept on a truck as shown in Fig. 3.253.
If the truck now starts moving with a constant
acceleration of 1 m/s2, the pipe rolls backwards a
without slipping on the floor of the truck and
finally falls on the road. If the pipe moves a total
length of 4 cm on the floor of the truck, what is
the velocity of the pipe relative to the truck and Fig. 3.253
relative to ground at the instant it leaves contact
with the truck? What is the angular velocity of the pipe at the same instant? (g = 10 m/s2).
Solution Let a be the acceleration of CM in reference frame of truck and a be its angular
acceleration about force, an axis through its CM as shown in Fig. 3.254.
Equations of motion in reference frame of truck are
ma0 – f = ma
f a
\ a = a0 – ...(1) ma0 a
m (Pseudo
force)
t fR 2f
a = = = ...(2) f
I 1 mR2 mR
2 Fig. 3.254
Rolling constraint a = Ra ...(3)
Solving Eqs. (1), (2) and (3), we get
2 2 2
a = a0 = (1) = m/s 2
3 3 3
492പWŚLJƐŝĐƐĨŽƌ//dͲ:͗DĞĐŚĂŶŝĐƐ//

Displacement of CM of cylinder in reference frame of truck is s = 4m in time t given by


1 1 Ê 2ˆ
s = at 2 = Á ˜ t 2
2 2 Ë 3¯
or t = 12 = 2 3s
The linear velocity of the cylinder in reference frame of truck is
Ê 2ˆ 4
vr = at = Á ˜ 2 3 = m/s (towards left)
Ë 3¯ 3
Velocity of truck after time t,
vT = a0t = (1)2 3 = 2 3 m/s (rightwards)
Hence, at the moment the cylinder leaves the truck, its absolute linear velocity [ground
reference frame],
4 2
v = v r – vT = -2 3 =- m/s w
3 3
2 v = 2 / 3 m/s
or v = m/s (rightwards)
3
and its angular velocity is
vr 4 8 fmax = mmg
w = = = rad/s (anticlockwise)
R 3 ¥ 0.5 3 Fig. 3.255

Problem 3.113 Two thin circular discs of mass 2 kg and radius 10 cm each are joined by
a rigid massless rod of length 20 cm. The axis of the rod is along the perpendicular to the
planes of the disc through their centres. An object is kept on a truck in such a way that the
axis of the object is horizontal and perpendicular to the direction of motion of the truck. Its
friction with the floor of the truck is large enough so that the object can roll on the truck
without slipping. Take X-axis as the direction of motion of the truck and Z-axis as the
vertically upwards direction. If the truck has an acceleration 9 m/s2. Calculate:
(i) the force of friction on each disc; and (IIT-JEE, 1997)
(ii) the magnitude and direction of the frictional torque acting on each disc about the
centre of mass O of the object express the torque in the vector form in terms of unit
vectors î, ĵ and k̂ in X, Y and Z directions.
Solution
(i) FBD of any one disc is shown in Fig. 3.256. a
Frictional force on the disc should be in
forward direction.
Let a0 be the linear acceleration of CM of
disc and a the angular acceleration about
Fig. 3.256
its CM. Then:
f f
a0 = = ...(1)
m 2
t fR
a = =
I 1
mR2
2
ZŽƚĂƟŽŶപ493

2f 2f
= = = 10 f …(2)
mR 2 ¥ 0.1
Since there is no slipping between the disc and truck, therefore a

a0 + Ra = a a0
Ê ˆ
f
or ÁË ˜¯ + (0.1)(10f) = a P
2 f
2 a 2 ¥ 9.0
or f = = N Q
3 3 f
\ f =6N Fig. 3.257
Since this force is acting in positive x-direction,
 z
f = (6i) N
   y
(ii) t = r¥f (0, 0)
 
rP = r1 = - 0.1j - 0.1k 
r
x
r
 
rQ = r2 = 0.1j - 0.1k  f f
r1 r2
  
r1 = r1 ¥ f Fig. 3.258
= (- 0.1j - 0.1k
 ) ¥ (6i) N-m
 - 0.6j
= 0.6 k
  
t 2 = r2 ¥ f
= (0.1j - 0.1k
 ) ¥ (6j) N-m
 ) N-m
= 0.6(- j - k
 
|t1| = |t 2|

Problem 3.114 A cylinder is sandwiched between two M


planks. Two constant horizontal forces F and 2F are ap- F
plied on the planks as shown. Determine the acceleration
of the centre of mass of the cylinder and the top plank,
if there is no slipping at the top and bottom of cylinder. M
R
Solution Equations of motion are, 2M
2F
F + f1 = Ma1 ...(i)
Smooth
f1 + f2 = Ma2 ...(ii)
Fig. 3.259
2F – f2 = 2Ma3 ...(iii)
( f1 - f 2 )R
a =
1
MR2
2
2( f1 - f 2 )
or a = ...(iv)
MR
494പWŚLJƐŝĐƐĨŽƌ//dͲ:͗DĞĐŚĂŶŝĐƐ//

For no slipping condition,


a2 + Ra = – a1 ...(v)
and a2 – Ra = a3 ...(vi)
a1
F
M
f1
f1
a
a2 M, R

f2

2F f2
2M

a3

Fig. 3.260

We have six unknowns, f1, f2, a1, a2, a3 and a.


Solving the above six equations, we get
21F F
a1 = and a2 = Ans.
26 M 26 M
t2
Problem 3.115 A bus is moving along a path in such a way that r = 2t2 and q = . Find
the acceleration of the bus at t = 1 s. 2
Option: (a) 10.4 m/s2 (b) 10.2 m/s2 (c) 8 m/s2 (d) 6 m/s2
d 2 r d 2 (2t 2 )
Solution a¢ = = = 4t = 4 m/s2
dt 2 dt 2
dq d(t 2/2)
w = = =t
dt dt
d 2q
and a = =1
Thus, dt 2
ar = (a¢ – w2r) = (4 – t2 ¥ 2t2)
= 4 – 2t4
and a¢t = 2vw + ar = 2 ¥ 4t ¥ t + 1 ¥ 2t2
= 8t2 + 2t2 = 10t2

\ a = ar2 + at2

= ( 4 - 2t 4 )2 + (10t 2 )2
At t = 1s
a = ( 4 - 2 ¥ 14 )2 + (10 ¥ 12 )2
= 10.2 m/s2 Ans.
ZŽƚĂƟŽŶപ495

Problem 3.116 Two solid bodies rotate about stationary mutually perpendicular intersect-
ing axes with constant angular velocities w1 = 3.0 rad/s and w2 = 4.0 rad/s. Find the angular
velocity and angular acceleration of one body relative to the other.
r
w2
Solution We know that
   r
w 12 = w 1 - w 2 w

\ w12 = w 12 + w 22
r
w1
2 2
= 3.0 + 4.0
= 5.0 rad/s
  Fig. 3.261
In rotating frame S¢ with angular velocity w the quantity A
can be written as
 
dA dA  
= +w¥A
dt S dt S¢
 
    dA dw 12
For the body 1 relative to 2, and for A = w 1 , we have w = w 1 and =
dt S¢ dt
 
dw 1 dw 12  
\ = + w1 ¥ w1
dt dt

 dw 1
According to the problem w 1 is constant and so =0
dt

dw 12  
Hence, = - w 2 ¥ w1
dt  
= w1 ¥ w 2

Ê dw 12 ˆ
and ÁË ˜ = w1w2 sin 90°
dt ¯
= w 1w 2
= 3.0 ¥ 4.0 = 12.0 rad/s2 Ans.
Problem 3.117 A particle A moves along a circle of radius R = 50 cm so that its radius

vector r relative to the point O rotates with the constant angular velocity w = 0.40 rad/s.
Find the modulus of the velocity of the particle, and the modulus and direction of its total
acceleration.
Solution In DOAB, by sine rule, we can write
y
A
R r
= q
sinq sin(180∞ - 2q ) r
r R
or r = 2R cos q ...(i) 2q x
 q
r can be written as; B
  O
r = r cosq i + r sinq j
= (2R cosq) cos q i + (2R cos q) sin q j
= 2R cos2q i + R sin 2q j ...(ii)
Fig. 3.262
496പWŚLJƐŝĐƐĨŽƌ//dͲ:͗DĞĐŚĂŶŝĐƐ//

The velocity of the particle



 dr d
v = = [2R cos 2 q i + R sin 2q j]
dt dt
È Ê dq ˆ Ê dq ˆ ˘
= Í- 2R sin 2q Á ˜ i + 2R cos 2q Á ˜ j˙
Î Ë dt ¯ Ë dt ¯ ˚
= – 2wR (sin 2q i – cos 2q j )

\ |v| = 2wR
= 2 ¥ 0.40 ¥ (0.5) = 0.4 m/s
Here w is constant, so the tangential acceleration of the particle is zero. Thus, the magnitude
of total acceleration
v 2 (2w R)2
a = = = 4w 2 R
R R
= 4 ¥ (0.40)2 ¥ 0.5 = 0.32 m/s2 Ans.
Problem 3.118 Consider a cubical block of side a moves down on a rough inclined plank
of inclination q with constant velocity. Discuss its motion.
Solution As the block moves down with constant N
velocity, the motion is zero, Thus, we have a
mg sin q – f = 0 x
q f
or f = mg sin q ...(i) sin
mg
mg cos q
Also, the block is not rotating, so the net torque on it mg
must be zero. The net torque can be zero only if normal
reaction on the block will pass some distance away from q
the CM (see Fig. 3.263). Thus, for rotational equilibrium
Fig. 3.263
of the block,
a
f ¥ - Nx = 0
2
fa
or x =
2N
From Eq. (i), we have f = mg sin q
mg sin q ¥ a
\ x =
2 ¥ mg cos q
a tan q
=
2
Thus, normal reaction exerted by inclined plane on the block will pass through a distance
a tan q
from CM to keep the block in rotational equilibrium.
2
ZŽƚĂƟŽŶപ497

Problem 3.119 A disc of mass M has a radius R can rotate freely about a horizontal shaft O
which is located at a distance r from the centre of mass of the disc C. Assume that the disc
is released from the position shown in Fig. 3.264. Find minimum value of r for which the
angular acceleration of the disc is maximum.
Solution The moment of inertia of the disc about the axis of rotation which passes through O;
I0 = Icm + Mr2

Ê MR2 ˆ R
= Á + Mr 2 ˜ r
Ë 2 ¯ O
C

The angular acceleration of the disc is given by Mg

t Mg ¥ r
a = =
I Ê MR2 2ˆ
Á 2 + Mr ˜ Fig. 3.264
Ë ¯
È 2 gr ˘
= Í 2 2˙
Î R + 2r ˚
da
The angular acceleration a to be maximum, =0
dr
d È 2 gr ˘
or =0
dr ÍÎ R2 + 2r 2 ˙˚
(R2 + 2r2) ¥ 2g – 2gr ¥ (0 + 2 ¥ 2r) = 0
R
fi r = Ans.
2

Problem 3.120 A bullet of mass m moving with velocity u just grazes the top of a solid
cylinder of mass M and radius R resting on a rough horizontal surface as shown. Assuming
that the cylinder rolls without slipping, find the angular velocity of the cylinder and the
final velocity of bullet.
Solution We can assume the rolling of cylinder as pure m
u v
rotation about an axis passing through O. M
Using conservation of angular momentum about O R
(mu) ¥ 2Ro = (mv) ¥ 2Ro + Iow
MRo2 w
here v = w ¥ 2Ro and Io = + MRo2
2 O

After substituting these values in the above equation, we get Fig. 3.265
4 mu Ê 8 mu ˆ
w = and v = Á Ans.
(8 m + 3 M )Ro Ë 8 m + 3 M ˜¯
498പWŚLJƐŝĐƐĨŽƌ//dͲ:͗DĞĐŚĂŶŝĐƐ//

Problem 3.121 A circular wooden hoop of mass m and radius Ro rests


Ro
flat on a frictionless surface. A bullet, also of mass m, and moving with a m
velocity v strikes the hoop and gets embedded in it. The thickness of hoop
m v
is much smaller than Ro. Find the angular velocity with which the system
rotates after the bullet strikes the hoop. Fig. 3.266

Solution Let the velocity of the CM of the system after strike is vc. By conservation of
linear momentum
mv = 2m ¥ vc Ro
v y w
fi vc = v
vc
2 m
m ¥ 0 + m ¥ Ro Ro
Position of CM y = = Fig. 3.267
m+m 2
Using conservation of angular momentum about CM of the system (hoop + bullet)
Ro
mv ¥ = ICw
2
mvRo = (I
or bullet + Ihoop)c w
2
mvRo = ÈÍ m Ê Ro ˆ + ÏÔmR2 + m Ê Ro ˆ Ô ˘
2 2¸
or Á ˜ Ì o ÁË ˜¯ ˝w ˙
2 ÍÎ Ë 2 ¯ ÓÔ 2 ˛Ô ˙˚
After solving, we get
v
w = Ans.
3Ro

Problem 3.122 A rigid body is made of three identical thin rods, each of length L fastened
together in the form of the letter H. The body is free to rotate about a horizontal axis that
runs along the length of one of the legs of the H. The body is allowed to fall from rest from
a position in which the plane of H is horizontal. What is the angular speed of the body when
the plane of H is vertical?
Solution Moment of inertia of the system about the
given axis A

I = I A + IB + IC
Now as rod is thin B C
IA = Âm ¥ 0 = 0 2

Rod B is rotating about one end


IB = ML2/3
And for rod C all points are always at distance L from Fig. 3.268
the axis of rotation, so
IC = ÂmL2 = ML2
\ I = 0 + (ML2/3) + ML2 = 4ML2/3
ZŽƚĂƟŽŶപ499

So, if w is the desired angular speed, gain in kinetic energy due to rotation of H from
horizontal to vertical position
1 2 1 È4 ˘
KR = Iw = Í ML2 ˙ w 2
2 2 Î3 ˚
And loss in potential energy of the system in doing so
L 3
= 0 + Mg + MgL = MgL
2 2
So, by conservation of mechanical energy
(2/3)ML2w2 = (3/2) MgL

3 g
or w =
2 L

Problem 3.123 A uniform spherical shell of mass M and


M, Ro
radius Ro rotates about a vertical axis on frictionless bearing
as shown in Fig. 3.269. A massless cord passes around the I, ro
equator of the shell, over a pulley of rotational inertia I and
radius ro and is attached to a small object of mass m that is
otherwise free to fall under the influence of gravity. There
is no friction of pulley’s axle; the cord does not slip on the
pulley. What is the speed of the object after it has fallen a
m
distance h from rest? Use work-energy considerations.
Solution Using a conservation of energy principle, we Fig. 3.269
have
Fall in PE of the block = Gain in KE of block + rotational KE of the pulley
+ rotational KE of the shell.
1 1 1 Ê 2 MRo2 ˆ 2
or mgh = mv 2 + Iw 2 + Á w¢
2 2 2 Ë 3 ˜¯
where w = v/ro and w¢ = v/Ro
After substituting these values in the above equation, and solving, we get

mgh
v2 = Ans.
Êm I Mˆ
Á2+ 2+ 3˜
Ë 2ro ¯

Problem 3.124 A constant horizontal force of 20 N is applied to a wheel of mass 10 kg and


radius 0.30 m as shown in Fig. 3.270. The wheel rolls without slipping on the horizontal
surface, and the acceleration of centre of mass is 0.60 m/s2.
(a) What are the magnitude and direction of the frictional force on the wheel?
(b) What is the rotational inertia of the wheel about an axis through its centre of mass and
perpendicular to the plank of the wheel?
500പWŚLJƐŝĐƐĨŽƌ//dͲ:͗DĞĐŚĂŶŝĐƐ//

Solution
(a) The direction of the frictional force will be in backward
direction of motion of the wheel. Equation of motion for
translation of wheel 20 N

20 – fr = ma
fr
Substituting m = 10 kg and a = 0.60 m/s2 in the above
equation, we get
fr = 4 N Ans. Fig. 3.270

(b) Equation of motion for rotation of wheel


fr ¥ R = Ia
R 4 ¥ 0.30
which gives I = fr ¥ =
a (0.60/0.30)
or I = 2.1 kg-m2

Problem 3.125 See Fig. 3.271. The masses of the


bodies are known to be m1 and m2, the coefficient of N
T1
friction between the body m1 and the horizontal plane mN m1
is equal to m, and a pulley of mass m is assumed to be a T2
uniform disc. The thread does not slip over the pulley. m1g
T2
At the moment t = 0 the body m2 starts descending.
m2
Assuming the mass of the thread and the friction in
the axle of the pulley to be negligible, find the work m2g
performed by the friction forces acting on the body m1
over the first t second after the beginning of motion. Fig. 3.271

Solution Let a be the acceleration of the blocks. Equation of motion of the blocks are
T1 – mN = m1a ...(i)
m 2g – T 2 = m 2a ...(ii)
and for the rotation of the pulley
T2R – T1R = I a = I(a/R) ...(iii)
2
mR
where I =
2
Solving the above equations, we get
(m2 - m m1 ) g
a =
Ê mˆ
ÁË m1 + m2 + ˜¯
2
The distance travelled by the block in t second
1 2
s = at
2
1 2
Work done by the friction W = fr ¥ s = mN ¥ at
2
ZŽƚĂƟŽŶപ501

1
( m m1 g ) ¥ (m2 - m m1 ) g
= 2 t2
Ê mˆ
ÁË (m1 + m2 + ˜¯
2
m m1 g 2t 2 (m2 - m m1 )
= Ans.
(2m1 + 2m2 + m)

Problem 3.126 A particle of mass m slides on a frictionless A OM


surface ABCD, starting from rest as shown in Fig. 3.272. The
part BCD is a circular arc. If it loses contact at point P, find the 2R P
R D
maximum height attained by the particle from point C. 45°
E B R R
È 1 ˘ È 1 ˘
(a) R Í2 + (b) R Í2 -
Î 2 2 ˙˚ Î 2 2 ˙˚ C

Fig. 3.272
(c) 3R (d) none of these
Solution Apply conservation of energy
A OM
R 1
2mgR – mg = mu2 v
2 2
2R 45° P
D
fi u = 4 gR - 2 gR 45°
45°
u2 sin 2 45∞ R 4 gR - 2 gR E B R R
Hmax = R + = R+ +
2g 2 2g ¥ 2
C
R R È 1 ˘
= 2R + - = R Í2 +
2 2 2 Î 2 2 ˙˚ Fig. 3.273
\ (a)
Problem 3.127 A uniform solid cylinder of mass M and radius R rolls on a rough inclined
plane with its axis perpendicular to the line of the greatest slope. As the cylinder rolls it
winds up a light cord which passes over a small mass m, the part of the string between the
pulley and the cylinder being parallel to the line of the greatest slope. If q is the inclination
of the plane with the horizontal, calculate the tension in the string.
Solution Method 1: Dynamic method
Let the acceleration of CM of the cylinder be T
a. The acceleration of a point on its tangent N
will be 2a, so the acceleration of the mass m T
will be 2a. T
Equation of motion for the translation of q
the cylinder sin fr
Mg
mg
Mg cos q
Mg sin q – T – fr = Ma ...(i) Mg
q
and for its rotation about the axis passing
through its CM. Fig. 3.274
502പWŚLJƐŝĐƐĨŽƌ//dͲ:͗DĞĐŚĂŶŝĐƐ//

fr R – TR = Ia
MR2
or fr R – TR = a ...(ii)
2
Equation of motion for mass m
T – mg = m(2a) ...(iii)
Solving the above equations, we get
2 Mg sin q - 4 mg
a =
8m + 3 M
(3 + 4 sin q )mMg
and T = Ans.
8m + 3 M

‡–Š‘† ǣ‡”‰›‡–Š‘†
Let v be the velocity of the CM of the cylinder when it travelled a distance y along the
plane. The velocity and the distance travelled by the mass m in this interval will be 2v and
2y respectively.
Decrease in PE of cylinder = Increase in (rolling KE of the cylinder + translational KE of
the mass m + PE of the mass)
1 1 1
Mg (y sin q) = Mv 2 + Iw 2 + m(2v)2 + mg(2 y )
2 2 2
v MR2
where w= and I =
R 2
Substituting these values in the above equation and solving for v, we get
4 g ¥ ( M sin q - 2m)
v2 =
8m + 3 M
Using the third equation of motion v2 = u2 + 2ay, where u = 0
v 2 (2 Mg sin q - 4 mg )
\ a= =
2y (8 m + 3 M )
Substituting this value in Eq. (iii), we can get T.
Problem 3.128 A uniform cylinder of radius R is spun about its
N1
axis to the angular velocity w 0 and then placed into a corner (Fig.
mN2
3.275). The coefficient of friction between the corner walls and the
cylinder is equal to m. How many turns the cylinder accomplish w
before it stops? N2

Solution For the vertical and horizontal equilibrium of the cyl-


inder, we have mN1 mg
N1 + mN2 = mg ...(i)
Fig. 3.275
and N2 = mN1 ...(ii)
ZŽƚĂƟŽŶപ503

Solving the above equations, we get


mg
N1 =
(1 + m 2 )
m mg
and N2 =
(1 + m 2 )

‡–Š‘†ͳǣ›ƒ‹ ƒŽ‡–Š‘†
The net retarding torque on the cylinder = (mN1 + mN2) ¥ R
therefore we have, (mN1 + mN2) ¥ R = Ia
mR2
Substituting the values of N1, N2 and I = , we get
2
2 m g( m + 1)
a =
R( m 2 + 1)
The time taken by the cylinder to come to rest can be obtained as
0 = w0 – at,
w 0 w 0 R( m 2 + 1)
\ t = =
a 2 m g( m + 1)
Now using the second equation of motion
1 2
q = w 0t – at
2
After substituting the values of a and t in the above equation, we get
w 02 R( m 2 + 1)
q =
[4 m g( m + 1)]

q (1 + m 2 )w 02 R
Number of turns n = = Ans.
2p [8p g m ( m + 1)]
Method II: Energy Method
Using work-energy theorem,
DK = Wf
1 2
or Iw0 = (mN1 + mN2) ¥ (2pRn)
2
Substituting all the known values in the above equation, we get
(1 + m 2 )w 02 R
n = Ans.
[8p g m ( m + 1)]

Problem 3.129 A uniform solid cylinder of radius Ro = 15 cm rolls over a horizontal plane
passing into an inclined plane forming an angle a = 30° with the horizontal shown in Fig.
3.276. Find the maximum value of the v0 which still permits the cylinder to roll onto the
inclined plane section without a jump. The sliding is assumed to be absent.
504പWŚLJƐŝĐƐĨŽƌ//dͲ:͗DĞĐŚĂŶŝĐƐ//

Solution
N
v0
h
a
h = R0(1 – cos a)
P a

mg

Fig. 3.276

Initial rolling kinetic energy of the cylinder


1 1
K1 = mv02 + Iw 2
2 2
2
1 1 Ê mRo2 ˆ Ê v0 ˆ
= mv02 + Á
2 2 Ë 2 ˜¯ ÁË Ro ˜¯
3
= mv02 ...(i)
4
When the cylinder passes on to the inclined plane its centre of mass descends through a
distance h = Ro (1 – cos a).
3
If v is the velocity of its centre of mass now, then rolling kinetic energy = mv2.
From energy conservation, we have 4

3 3 2
mv02 + mgRo (1 - cos a ) = mv ...(ii)
4 4
mv 2
At point P, mg cos a = N +
Ro
Cylinder passes the point P without jump, if N ≥ 0. For maximum value of v0, N should
have minimum + ve value i.e., N = 0
mv 2
\ mg cos a =
Ro
Solving Eqs. (i) and (ii), and substituting a = 30°, Ro = 0.15 m and g = 9.81 m/s2,

gRo
We get v0 = (7 cos a - 4) = 1.0 m/s Ans.
3

Problem 3.130 A uniform ball of radius ro rolls without slipping down from the top of a
sphere of radius Ro. Find the angular velocity of the ball at the moment it breaks off the
sphere. The initial velocity of the ball is negligible.
Solution Let v be the velocity and q the angle made by the radius vector with the vertical
at the instant when the ball break-off the sphere.
ZŽƚĂƟŽŶപ505

Let it happen at a vertical height h below the top.


Therefore, we have, N
h
1 1
mgh = mv 2 + Iw 2 R0 + r0
2 2
q mg
where h = (Ro + ro) (1 – cos q)
O
\ mg(Ro + ro) (1 – cos q) = 1 mv 2 + 1 Iw 2
2 2
1 1
mg (Ro + ro) (1 – cos q) = m(w ro )2 + Iw 2 ...(i)
2 2
At the break-off Fig. 3.277
2
mg cos q = N +
mv
(Ro + ro )
Substituting N = 0 and v = wr, we have
m(w r )2
mg cos q = ...(ii)
(R + r )
Solving Eqs. (i) and (ii), we get
10 g(Ro + ro )
w = Ans.
17 r 2

Problem 3.131 A uniform sphere of mass m and v


radius ro rolls without sliding over a horizontal plane,
L r
rotating about a horizontal axle LA as shown in Fig. m A
3.278. In the process, the centre of the sphere moves
with velocity v along a circle of radius Ro. Find the R
kinetic energy of the sphere.
Solution The kinetic energy of the sphere due to
its rotation about its own axis and its motion along a
circular path with velocity v. Fig. 3.278
1 1
\ KE = I1w 12 + I 2w 22
2 2
2 2 Ê2 ˆ
where I1 = mro and I 2 = Á mro2 + mRo2 ˜
5 Ë5 ¯
v v
w1 = and w 2 =
ro Ro
Substituting these values in the above equation, we get

7m 2 È 2r 2 ˘
KE = v Í1 + o2 ˙ Ans.
10 Î 7 Ro ˚
506പWŚLJƐŝĐƐĨŽƌ//dͲ:͗DĞĐŚĂŶŝĐƐ//

Problem 3.132 Two bodies with masses m1 = 3 kg and m2 = 2 kg which slide as a unit with a
common velocity of 2 m/s on a level frictionless surface, having between them a compressed
massless spring with k = 50 N/m. The spring originally compressed by 25 cm, is suddenly
released, sending the two masses, which are not connected with the spring, fly apart from
each other. (IIT Roorkee, 1973)
v¢2
m2 v2 = 2 m/s m2 v2 = 2 m/s

m1 a a = 30° m1 a a = 30°
v1 = 2 m/s v1 = 2 m/s
v¢1
(a) (b)

Fig. 3.279

The orientation of the spring with respect to the initial velocity is shown in Fig. 3.278.
(a) What is the relative velocity of separation after the particles lose contact?
(b) What is the velocity of CM after separation?
(c) What are the speeds of m1 and m2 with respect to the frictionless surface after separation?
Solution
(a) Since there is no force acting on the system, its momentum remains constant. Applying
conservation of momentum along the line of the spring, we have
0 = 3v¢1 + 2v¢2 ...(i)
Potential energy stored in the spring imparts KE to both masses.
1 1 1
Therefore, (50)(0.25)2 = ◊ 3 ◊ v1¢ 2 + ◊ 2 ◊ v2¢ 2 ...(ii)
2 2 2
Solving the above equations, we get
v¢1 = 0.65 m/s
and v¢2 = 0.97 m/s
Relative velocity of separation, which will be along the line of the spring
v¢1 + v¢2 = 0.65 + 0.97 = 1.62 m/s Ans.
( 3 ¥ 2 + 2 ¥ 2)
(b) Initial velocity of the centre of mass = = 2 m/s
( 3 + 2)
Since there is no external force acting on the system, its velocity of centre of mass
remains constant. Therefore, v = 2 m/s. Ans.
(c) The speeds of masses with respect to the surface
v1 = [22 + 0.6452 + 2(2)(0.645) cos 150°]1/2 = 1.48 m/s Ans.
and 2 2
v2 = [2 + 0.97 + 2(2)(0.97) cos 30°] 1/2
= 2.9 m/s Ans.
ZŽƚĂƟŽŶപ507

Problem 3.133 Two large bodies, with masses m1 = 0.5 kg and m2 = 1.0 kg are free to slide
on a horizontal frictionless surface. They are connected by an ideal inertia less spring with
a relaxed length I0 = 0.5 m and a spring constant k = 10 N/m. An initial observation at
t = 0 places the puck 2 m apart with puck 2 having a velocity with respect to the surface of
6 m/s directly away from the puck, which is momentarily at rest on the surface.
(a) What is the velocity of the cm?
(b) What is the relative velocity at t = 0?
(c) To what maximum length does the spring stretch?
Solution
m1v1 + m2v2
(a) The velocity of CM =
(m1 + m2 )
0.5 ¥ 0 + 1.0 ¥ 6
= = 4.0 m/s
(0.5 + 1.0)
(b) At t = 0, v1 = 0 and v2 = 6.0 m/s
The relative velocity vr = v1 – v2 = 0 – 6
or vr = 6 m/s
(c) The given system can be converted into a system, having a reduced mass
(0.5 ¥ 1.0) 1
m= = kg connected at one end of the spring and the other end of the spring
(0.5 + 1.0) 3
is fixed as shown in Fig. 3.279.
1.0 kg
0.5 kg k = 10 N/m k = 10 N/m
= 6 m/s

0 m/s 6.0 m/s 1/3 kg

Fig. 3.280

The kinetic energy imparted to the larger puck is stored in elastic potential energy of
the spring. Let the maximum extension in the spring be x.
Initial energy stored in the spring + KE imparted to the large puck = final energy stored
in the spring
1 1 1
or k( x0 - l0 )2 + m v 2 = k( x - l0 )2
2 2 2
where x0= 2.0 m, l0 = 0.5 m, k = 10 N/m and m = 1/3 kg
Substituting these values in the above equation, we get
x = 2.357 m Ans.
Problem 3.134 A rectangular rigid fixed block has a long horizontal edge. A solid
homogeneous cylinder of radius r is placed horizontally at rest with its length parallel to
the edge such that the axis of the cylinder and the edge of the block are in the same vertical
plane. There is sufficient friction present at the edge so that a very small displacement causes
508പWŚLJƐŝĐƐĨŽƌ//dͲ:͗DĞĐŚĂŶŝĐƐ//

the cylinder to roll off the edge without slipping. Determine: (a) the angle q through which
the cylinder rotates before it leaves contact with the edge, (b) the speed of the centre of
mass of the cylinder before leaving contact with the edge and (c) the ratio of translational to
rotational kinetic energies of the cylinder when its centre of mass is in horizontal line with
the edge. (IIT-JEE, 1995)
Solution When the cylinder rolls by an angle q about
the edge its centre of mass will go down by h = r (1 –
cos q). So by conservation of ME, O

1 1
mgr[1 – cos q] = mv 2 + Iw 2 r R mg
2 2 q
1 2
But here I = mr E
2
and v = rw
1 1 1 Fig. 3.281
So, mgr[1 – cos q] = mv 2 + ¥ mr 2 (v 2/r 2 )
2 2 2
i.e., gr[1 – cos q] = (3/4)v2 ...(i)
Now for circular motion of the cylinder about the edge,
mg cos q – R = (mv2/r)
The cylinder will leave contact when R Æ 0
i.e., g cos q = (v2/r) ...(ii)
2
(a) Eliminating v between Eqs. (i) and (ii)
gr[1 – cos q] = (3/4)gr cos q
i.e., q = cos–1 (4/7)
(b) Now substituting this value of cos q in Eq. (ii),

v = ( 4/7 ) gr
(c) At the time the cylinder leaves contact with the edge,
1 2 1 1
KR = Iw = mv 2 = mgr
2 4 7
1
and then as t = 0, a = 0 and so w = constant., i.e., after leaving the edge KR = mgr
= constant. 7

So when the centre of mass comes in horizontal line with the edge, by conservation
of ME,
1 6
mgr = KR + KT, i.e., KT = mgr – mgr = mgr
7 7
KT (6/7 )mgr
So, =
KR (1/7 )mgr
ZŽƚĂƟŽŶപ509

Problem 3.135 A uniform circular disc has radius R and mass A


m. A particle, also of mass m, is fixed at a point A on the edge of
the disc as shown in Fig. 3.282. The disc can rotate freely about a R
fixed horizontal cord PQ that is at a distance R/4 from the centre
C of the disc. The line AC is perpendicular to PQ. C
R/4
Initially, the disc is held vertical with the point A at its highest P Q
position. It is then allowed to fall so that it starts rotating about
PQ. Find the linear speed of the particle as it reaches its lowest
position. (IIT-JEE, 1998) Fig. 3.282

1Ê 1 2ˆ
Solution As the moment of inertia of a disc about a diameter is Á mR ˜¯ , the moment of
2Ë 2
inertia of the disc about the cord PQ by ‘theorem of parallel axes’ will be
2
1 Ê1 ˆ 5
(ID)PQ = mR2 + m Á R˜ = mR2
4 Ë4 ¯ 16
and as particle of mass m is at a distance [R + (R/4) = (5 /4)R] from PQ, the moment of
inertia of the system about PQ
2
5 Ê5 ˆ 5
I = (ID)PQ + (IP)PQ = mR2 + m Á R˜ = mR2
16 Ë4 ¯ 8
Now if w is the angular speed of the system when A reaches the lowest point A¢ on rotation
about the axis PQ, by ‘conservation of mechanical energy’,
1 2
Iw = mg(AA¢) + mgCC¢ = mg[2AD + 2CD]
2

Finally
m A
D
R P Q
R/4 C¢
C R/4
P Q R
D
A¢ m
Initially
(a) (b)

Fig. 3.283

1 15 ÈÊ 1 ˆ 1 ˘
i.e., ¥ mR2w 2 = 2mg ÍÁ R + R˜ + R˙ .
2 8 ÎË 4 ¯ 4 ˚

g
i.e., w =4
5R
Ê 1 ˆ g
So, (v)A¢ = rw = Á R + R˜ ¥ 4 = 5 gR
Ë 4 ¯ 5R
510പWŚLJƐŝĐƐĨŽƌ//dͲ:͗DĞĐŚĂŶŝĐƐ//

Problem 3.136 A block of mass m and a cylinder of mass 2 m are 2m


released on a rough inclined plane, inclined at an angle q = 45° with the
m
horizontal. The coefficient of friction for all the contact surfaces is 0.5. m = 0.5
Find the accelerations of the block and cylinder.
Solution If the block and cylinder move independently on the incline,
45°
their accelerations are
ablock = g sin q – mg cos q Fig. 3.284
g
=
2 2
g sin q
acylinder =
1 + I/mR2
g sin 45∞
=
1 + (mR2/2)/mR2
2g
=
3

N1 mN1
N2 N
y a = a/R

mN f
x
mN
y

N
x

mg 2 mg

(a) (b)

Fig. 3.285

As ablock < acylinder, both the bodies will move in contact with each other with common
acceleration a.
From the force diagram block we have
mg
ÂFx = + N – mN1 = ma ...(1)
2
mg
ÂFy = N1 – mN – =0 ...(2)
2
From Eqs. (1) and (2), we eliminate N1.
mg m mg
+ N - m 2N - = ma ...(3)
2 2
The equations of cylinder are
2mg
ÂFx = – N – f = 2ma ...(4)
2
ZŽƚĂƟŽŶപ511

1 Ê aˆ
and Ât = (f – mN)R = (2m)R2 Á ˜
2 Ë R¯
or f – mN = ma ...(5)
On solving Eqs. (2) and (5), we have

m( 2 g - 3 a)
N = ...(6)
1+ m
On substituting N in Eq. (3) and solving for a, we get
3(1 - m ) g 3g
a = =
2 ( 4 - 3m ) 5 2
2mg
From Eq. (7), we get N =
15
From Eq. (5), on substituting for a and N, we get
2mg
f =
3
From force diagram of the cylinder, we have
2mg Ê mˆ
N2 = - m N = 2mg Á 1 - ˜
2 Ë 15 ¯
2 g 2mg
=
30
As f < mN2 we can say that pure rolling of the cylinder takes place. Both the block and
3g
cylinder move with common acceleration .
5 2
Problem 3.137 Figure 3.286(a) shows a light ring with three rods each of mass M welded
on its rim. The rods form an equilateral triangle. The rigid assembly is released on a rough
inclined plane. Determine the minimum value of the coefficient of static friction that will
allow pure rolling of the assembly.
y
N

x
f
CM

3Mg sin q 3Mg cos q


q
3Mg
(a) (b)

Fig. 3.286
512പWŚLJƐŝĐƐĨŽƌ//dͲ:͗DĞĐŚĂŶŝĐƐ//

Solution From Newton’s second law, the equations are


ÂFx = 3Mg sin q – f = 3Ma ...(1)
ÂFy = N – mg cos q = 0 ...(2)
Ât = fR = Ia ...(3)
For pure rolling, a = Ra ...(4)
The moment of inertia of the assembly about the centre of mass from parallel axis theorem
is
= Triangle + Ring
È M(R cos 30∞ ¥ 2)2 Ê Rˆ ˘
2
I = Í + MÁ ˜ ˙ ¥ 3
ÍÎ 12 Ë 2¯ ˙
˚
3
= MR2
2
On solving Eqs. (1), (2), (3) and (4) simultaneously, we obtain
f = Mg sin q
If m is the coefficient of friction at the contact surface, then
f £ mN = 3mMg cos q
or Mg sin q £ 3mMg cos q
1
or m ≥ tan q
3
1
So the minimum value of m is tan q.
3

Problem 3.138 A solid cylinder rolls up an inclined plane of angle of inclination 30°. At
the bottom of the inclined plane the centre of mass of the cylinder has a speed of 5 m/s.
(a) How far will the cylinder go up the plane?
(b) How long will it take to return to the bottom?
Solution
(a) If h is the height reached by the cylinder, then
1 1
mv 2 + Iw 2 = mgh s
2 2 h

1 Ê mR2 ˆ Ê v ˆ
2
1 30°
or mv 2 + Á Á ˜ = mgh
2 2 Ë 2 ˜¯ Ë R ¯
Fig. 3.287
3
or mv 2 = mgh
4
3v 2 3 ¥ 5 2
\ h = = = 1.9 m
4 g 4 ¥ 9.8
ZŽƚĂƟŽŶപ513

h 1.9
The distance s = = = 3.8 m
sin 30∞ 1/2
2
(b) Acceleration of the cylinder = g sin 30°
3
2 1
or a = ¥ 9.8 ¥ = 3.27 m/s 2
3 2
Time taken to move the distance of 3.8 m
1 2
3.8 = at
2
\ t  1.5 s
Total time taken T = 2t = 3 s Ans.
Problem 3.139 A spinning cylinder of mass m and radius R is lowered w0
on a rough inclined plane of angle 30° with the horizontal and m = 1/
3 . The cylinder is released at a height of 3R from the horizontal. Find
the total time taken by the cylinder to reach the bottom of the incline. 3R

Solution We will check whether the cylinder slips down the incline 30°
or not. Fig. 3.288
As mg sin 30° = mmg cos 30°, initially the cylinder slips at its place till
the angular velocity becomes zero.
mR2 N
Ât = (mmg cos q)R = a t
2
2m cos q
a = g
R
0 = w0 – at1
mg sin q mg cos q
w0
or t1 = Fig. 3.289
a
Next the sphere will come down with the initial angular and linear velocity both zero.
mR2 mRa
fR = a, f =
2 2
and mg sin q – f = ma
If pure rolling takes place,
a = Ra
f – mg sin q = – 2f
1 mg
or, f = mg sin q =
3 6
1 3 mg
As fmax = mmg cos q = mg ¥ =
3 2 2
and fmax < f, so pure rolling is possible.
514പWŚLJƐŝĐƐĨŽƌ//dͲ:͗DĞĐŚĂŶŝĐƐ//

2 f 2(- mg/6) g
= a= =
m m 3
Let t2 be the time taken to reach the foot of the incline.
3R 1g 2
= t2
sin 30∞ 23
or, t2 = 6 R/g
Êw R Rˆ
Hence total time, t = t 1 + t2 = Á 0 + 6
Ë g g ˜¯

Problem 3.140 A sphere of mass m is released on the rough inclined surface of a wedge
having mass M. If the sphere rolls on the wedge [Fig. 3.290], calculate the acceleration of the
wedge and sphere. Neglect friction between the wedge and the ground.
y

N
x
m f

Rough
M
Smooth mg cos q mg sin q
q
mg

(a) (b)

Fig. 3.290

Solution Equations of sphere are:


ÂFx = mg sin q – f = m(a – A cos q) ...(1)
ÂFy = mg cos q – mA sin q – N = 0 ...(2)
2
Ât = f ¥ R = mR2a ...(3)
5
y
f
A cos q
x
N sin q
A q
N1 N
q
N cos q a

A sin q

Mg
(a) (b)

Fig. 3.291
ZŽƚĂƟŽŶപ515

a = Ra ...(4)
Equation of the wedge is, ÂFx = N sin q – f cos q = MA ...(5)
From Eqs. (1), (3) and (4), we get
7
mg sin g q = ma – mA cos q ...(6)
5
On multiplying Eq. (4) by sin q and adding to Eq. (5), we get
2
mg sin q cos q – ma cos q = (M + m sin2 q)A ...(7)
5
2
On multiplying Eq. (6) by cos q and subtracting it from Eq. (7), we get
7
5 Ï 7 M + (2 + 5 sin 2q )m ¸
mg sin q cos q = Ì ˝A
7 Ó 7 ˛
5m sin q cos q
or A = g
7 M + (2 + 5 sin 2q )m

Problem 3.141 A uniform circular cylinder of mass m and radius r is given an initial angular
velocity w0 and no initial translational velocity. It is placed in contact with a plane inclined
at an angle a to the horizontal. If there is a coefficient of friction m for sliding between the
cylinder and the plane. Find the distance the cylinder moves up before sliding stops. Also,
calculate the maximum distance it travels up the plane. Assume m > tan a.
Solution Given m > tan a fi mmg cos a > mg sin a
a = (mg cos a – g sin a)
( m mg cos a )r 2 m g cos a
a = =
1 2 r
mr
2
Slipping will stop when,
v = rw
or at = r (w0 – at)
rw 0 Ê rw 0 ˆ
\ t = =Á
a + ra Ë 3 m g cos a - g sin a ˜¯
1 2
d1 = at
2
2
1 Ê rw 0 ˆ
= ( m g cos a - g sin a ) Á
2 Ë 3 m g cos a - g sin a ˜¯
r 2w 02 ( m cos a - sin a )
= Ans.
2 g(3 m cos a - sin a )2
Ê rw 0 ˆ
v = at = (mg cos a – g sin a) Á
Ë 3 m g cos a - g sin a ) ˜¯
rw 0 ( m cos a - sin a )
=
(3 m cos a - sin a )
516പWŚLJƐŝĐƐĨŽƌ//dͲ:͗DĞĐŚĂŶŝĐƐ//

Once slipping stops, retardation in cylinder,


g sin a g sin a 2
a¢ = = = g sin a
I 1 3
1+ 2
1+ w0 a
mr 2 cos
a m mg
v2 3r 2w 02 ( m cos a - sin a )2 sin
d2 = = mg
2 a¢ (3 m cos a - sin a )2 ( 4 g sin a )
\ dmax = d1 + d2 a

r 2w 02 ( m cos a - sin a ) È 3( m cos a - sin a ) ˘ (a)


= Í1 + ˙
2 g(3 m cos a - sin a )2 Î 2 sin a ˚
a a
r 2w 02 ( m cos a - sin a )
= Ans.
4 g sin a (3 m cos a - sin a )

Note: Once slipping stops, pure rolling continues if


tan a
m > a
mr 2
1+
I (b)

tan a tan a Fig. 3.292


or m > or m>
1+ 2 3
2
and already in the question it is given that m > tan a. That’s why we have taken a¢ = g sin a.
3

Problem 3.142 A man pushes a cylinder of mass m1 F


m2
with the help of a plank of mass m2 as shown. There is
no slipping at any contact. The horizontal component
of the force applied by the man is F. Find: m1
(a) the accelerations of the plank and the centre of
mass of the cylinder; and
(b) the magnitudes and directions of frictional force
at contact points. (IIT-JEE, 1999) Fig. 3.293

Solution We can choose any arbitrary directions of frictional forces at different contacts. In the
final answer the negative values will show the opposite directions.
f1
a1
m1
F
f2
f1
(a) (b)

Fig. 3.294
ZŽƚĂƟŽŶപ517

Let f1 = frictional force between plank and cylinder


f2 = frictional force between cylinder and ground
a1 = acceleration of plank
a2 = acceleration of the centre of mass of cylinder
a = angular acceleration of the cylinder about its CM.
Directions of f1 and f2 are as shown here.
Since there is no slipping anywhere,
a1 = 2a2 ...(1)
(Acceleration of plank = Acceleration of top point of cylinder)
F - f1 a1 = 2a2
a1 = ...(2)
m2
a2
f1 + f 2
a2 = ...(3)
m1
( f1 - f 2 )R Fig. 3.295
a =
I
(I = moment of inertia of cylinder above CM)
( f1 - f 2 )R
\ a =
1
m1R2
2
2( f1 - f 2 )
a = ...(4)
m1R
2( f1 - f 2 )
For no slipping a2 = Ra = ...(5)
m1
Solving Eqs. (1), (2), (3) and (5), we get
8F
(a) a1 =
3m1 + 8 m2
4F
and a2 =
3m1 + 8 m2
3m1F
(b) f1 =
3m1 + 8 m2
m1F
f2 =
3m1 + 8 m2

Problem 3.143 Two balls of equal density and radii r and R = 2r, are placed with the centre
of the larger one at the middle of a cart of mass M = 6 kg and length L = 2m. The mass of the
smaller ball is m = 1 kg. The balls are made to roll, without slipping, in such a way that the
larger ball rests on the cart and a straight line connecting their centres remains at a constant
518പWŚLJƐŝĐƐĨŽƌ//dͲ:͗DĞĐŚĂŶŝĐƐ//

angle f = 60° to the horizontal. The cart is pulled by m


a horizontal force in the direction shown in Fig. 3.296.
(I) Find the magnitude of the force F.
(II) How much time elapses before the balls fall off f
the cart?
Solution Let the angular acceleration of the smaller
ball be a1, that of the larger one a2, their common M
F
horizontal acceleration is a1 and the acceleration of
the cart a2. As the balls are rolling without slipping,
Fig. 3.296
Ra2 = a2 – a1
a1
and Ra2 = ra1
and, because R = 2r, N1 a1
a -a
a1 = 2a2 = 2 1 mg
r
a2 N2 N2
The moment of inertia of the smaller ball
2
is mr2, while that of the larger one with N1
5 a1
the same density is
2 64 2 8mg
N
¥ 8 m ¥ ( 2 r )2 = mr
5 5
a2
Using the free-body diagram of Fig. Ffr Ffr
F
3.297, we can write the following equations
of motion:
F – Ffr. = Ma2, N
8mg + N1 – N = 0, Ffr. – N2 = 8ma1, Fig. 3.297
mg – N1 = 0, N2 = ma1,
2 2
N1r cos f – N2r sin f =mr a 1
5
64 2
2rFfr. + 2rN2r sin f – 2rN1r cos f = mr a 2
5
From these equations we can express the force F as
Ê 7 ˆ cos f
F = Á 9m + M ˜ g = 79 N
Ë 2 ¯ 1 + sin f
The acceleration of the balls relative to the cart is
5 cos f
Da = a2 – a1 = g
2 1 + sin f
At the time t when the balls fall from the cart, the distance they have moved relative to
the cart is L/2. As their initial velocities are zero,
L
t = = 0.55 s
Da
ZŽƚĂƟŽŶപ519

Problem 3.144 A semicircular ring of mass m and radius r is released from rest in the
position shown with its lower edge resting on a horizontal surface. Find the minimum
coefficient of static friction ms which is necessary to prevent any initial slipping of the ring.
Solution Centre of mass of semicircular ring lies at a dis-
2r
tance of from the centre.
p
2r
OC = , OP = r
p
Hence, CP = (OC )2 + (OP)2
r
or CP = 1.185r
If there is no slipping, then the instantaneous axis of
rotation passes through P.
Angular acceleration of the ring about P is
Fig. 3.298(a)
tp (mg )(CO)
a = =
Ip (2mr 2 )
Ê 2r ˆ
(mg ) Á ˜
Ëp¯ g
= 2
= ...(1) C
(2mr ) pr O IP = 2 mI2
Acceleration of the centre of mass,
mg N
a = (CP)(a) q

Ê g ˆ Ê 1.185 ˆ
= (1.185r) Á ˜ = Á g ...(2)
Ë p r ¯ Ë p ˜¯
f = ms N P
From Newton’s law,
ÂFy = may Fig. 3.298(b)
or mg – N = may
or mg – N = ma sin q
Ê 1.185 ˆ Ê 2r ˆ Ê 1 ˆ
or mg – N = m Á g
Ë p ˜¯ ÁË p ˜¯ ÁË 1.185r ˜¯
ax C
2 O x
= 2 mg = 0.2 mg q
p
y
or N = mg – 0.2 mg a
q
N = 0.8 mg ...(3) ay
ÂFx = max
or ms N = (m) (a cos q) P
Fig. 3.299
Ê 1.185 g ˆ Ê r ˆ
or ms(0.8 mg) = (m) Á
Ë p ˜¯ ÁË 1.185r ˜¯
or ms = 0.398
520പWŚLJƐŝĐƐĨŽƌ//dͲ:͗DĞĐŚĂŶŝĐƐ//

Problem 3.145 A small clamp of mass m is attached at B to a hoop of B


mass 3 m and radius r. The system is released from rest with q = 90°
and rolls without sliding. Determine: q

(a) the angular acceleration of the hoop.


(b) the horizontal and vertical components of the acceleration of B; and
(c) normal reaction and frictional force just after the release.
Solution The hoop rolls without sliding. Therefore, instantaneous
axis of rotation passes through the bottommost point. Fig. 3.300(a)
C is the centre of mass of the
hoop and the clamp x

r r/4 3r/4
where AC = M Y
4 B N = normal
C q
A r reaction
3r ac 45°
and CB =
4 3mg r
tp mgr aB
(a) a = = N q
mg
I p 6 mr 2 + 2mr 2
g
or a = (Ip = Ihoop + Iblock) P f = frictional force
8r a

Hence, the angular accelera- Fig. 3.300(b)


g
tion of hoop is .
8r
C
Ê gˆ 2g q
(b) aB = (PB)a = ( 2r ) Á ˜ =
Ë 8r ¯ 8

\ Horizontal component of a B is r
ac
2g g
cos 45° or Æ Fig. 3.300(c)
8 8

and vertical component of a B is

2g g
sin 45° or Ø
8 8
AC r/4 1
(c) tan q = = =
AP r 4

or q = 14°

r2
and PC = r2 + = 1.03r
16
ZŽƚĂƟŽŶപ521

Acceleration of centre of mass C is


aC = (PC)a
Ê gˆ g
= (1.03r) Á ˜ =
Ë 8 r ¯ 7.767

Horizontal component of a C is
g
ax = aC cos q = cos 14∞ = 0.125 g Æ
(7.767 )
\ f = (4m)ax = 0.5mg Æ

Similarly, vertical component of a C is
g
ay = aC sin q = sin 14∞ = 0.03g Ø
7.767
Now, 4mg – N = (4m)ay
Hence, N = 4mg – 4may
N = 4m(g – ay)
N = 4m(g – 0.03g)
N = 3.88mg ≠
Problem 3.146 Three bodies each of mass m can slide on fixed v0
m1 m2
frictionless circular tracks in the same horizontal plane as shown
in Fig. 3.301. Particle m1 moves with velocity v0 and hits particle q = 0.50
k
m2, the coefficient of restitution being e = 1/2. Assuming that m3
m2 and m3 are at rest initially and lie along a radial line before
the impact, and the spring is initially unstretched, find (a) the 2R
velocity of m3; and (b) maximum stretch of the spring. R

Solution First we apply conservation of momentum on m1 and


m 2.
m1v 0 = m 1v 1 + m 2v 2
As m1 = m2, we get v0 = v1 + v2 ...(1) Fig. 3.301(a)
where v1 and v2 are velocities of m1 and m2 immediately after the impact.
From definition of coefficient of restitution,
1 v -v
e = =- 2 1 or v0 = 2(v2 – v1) ...(2)
2 0 - v0
On solving Eqs. (1) and (2), we get m2
v 3v 2R
v1 = 0 and v2 = 0 3v 0
4 4 v2 =
4
The spring has maximum extension when angular velocity v3 = 0
w of m2 and m3 about O is same. Now we apply conservation R m3
of angular momentum.
Ê3 ˆ O
m Á v0 ˜ 2R = mR2w + m(2R)2w
Ë4 ¯ Fig. 3.301(b)
522പWŚLJƐŝĐƐĨŽƌ//dͲ:͗DĞĐŚĂŶŝĐƐ//

3 v0
or w =
2 5R
3
Velocity of m2 = 2Rw = v0
5
3
Velocity of m3 = Rw = v0
10
In order to determine maximum extension in the spring, we apply law of conservation
of energy.
2 2 2
1 Ê3 ˆ 1 Ê 3 v0 ˆ 1 Ê 3 v0 ˆ 1
m Á v0 ˜ = mR2 Á + m(2R)2 Á 2
+ k Dxmax
2 Ë4 ¯ 2 Ë 2 5R ˜¯ 2 Ë 2 5R ˜¯ 2
3 m
On solving for Dx, we get Dxmax = v0 .
4 5R

Problem 3.147 A spool of thread of mass


m and moment of inertia I is attached to the m
block of mass M. Along a horizontal line M R
passing through the centre of the spool, a r
constant force F is applied as shown. The O F
surface under the block is smooth while
beneath the spool it is rough. a b

I
Show that if M = , no frictional
r( R - r ) Fig. 3.302
force will act on the spool. Now suppose M
2I
= , find in which direction will the frictional force act? Also find in this case the rate of
r( R - r )
increase of length ab of the thread at time t after the application of force (no slipping occurs).
Solution For translational motion
F – T – f = maCM ...(1) FBD of the spool

For rotational motion m

(Tr + fR) = Ia ...(2) r


F
As per the given condition if frictional force is zero then, R
T
aCM b
fi a =
R f
a Fig. 3.303
Hence, Tr + fR = I CM
R
IaCM R
fi T = -f◊ ...(3)
Rr r
fi T = Ma¢
ZŽƚĂƟŽŶപ523

Since string ab remains tangent, hence acceleration of point a and that of FBD of M
point b is same. a¢
aCM
fi a¢ = (R - r )
R
aCM M
fi T =M (R - r ) ...(4) a T
R
From Eqs. (3) and (4), we get
aCM a R
M (R - r ) = I CM - f ◊ ...(5)
R R◊r r Fig. 3.304

Putting f = 0 in Eq. (5)


I
M =
r( R - r )
2I
Putting the value of M = in Eq. (5), we get
r( R - r )
- I ◊ aCM
f = ...(6)
R2
Negative sign indicates that friction is acting in the opposite direction of our assumption,
i.e., in forward direction.
From Eqs. (1), (3) and (6), we get
2I ◊ aCM I ◊ aCM
F– + = maCM
Rr R2
F F
fi aCM = fi a=
Ê 2I I ˆ Ê 2I I ˆ
ÁË m + - 2˜ RÁ m + + 2˜
Rr R ¯ Ë Rr R ¯
\ T = ma¢
d 2 ( ab) F( R - r ) 2IaCM
= a(R - r ) - a ¢ = -
dt 2 Ê 2 I I ˆ mRr
RÁm + - 2˜
Ë Rr R ¯
F( R - r ) 2I F
= -
Ê 2I I ˆ mRr Ê 2I I ˆ
RÁ m + - 2˜ RÁ m + - 2˜
Ë Rr R ¯ Ë Rr R ¯

F È 2I ˘
=
Ê 2I I ˆ ÍÎ R - r - mr ˙˚
RÁ m + - 2˜
Ë Rr R ¯

Ft È 2I ˘
\ Rate of increase in length ab (= vab) =
Ê 2I I ˆ ÍÎ R - r - mr ˙˚ .
RÁ m + - 2˜
Ë Rr R ¯
524പWŚLJƐŝĐƐĨŽƌ//dͲ:͗DĞĐŚĂŶŝĐƐ//

Problem 3.148 A sphere of radius R is projected with a reverse spin wo down a rough
inclined plane with a speed vo for which coefficient of friction is m > tan q, where q is the
5vo m
angle of the inclination. Show that it will turn back if wo > .
2R( m - tan q )

Solution ∵ m > tan q


fi m mg cos q ≥ mg cos q tan q
R f = m mg cos q
fi f ≥ mg sin q
w0
Hence, the sphere will turn back if at the moment its
centre of mass comes to stop it must have angular speed mg cos q
mg sin q q
in same direction as wo has
Let at time t, vCM = 0 Fig. 3.305
fi vo + (g sin q – mg cos q)t = 0
vo
fi t = …(1)
g( m cos q - sin q )

m mg cos q
Further, wo – t>0 …(2)
I CM
5vo m
From Eqs. (1) and (2), we get wo > .
2R( m - tan q )

DIRECTION OF FRICTION IN CASE OF


TRANSLATION AND ROTATION COMBINED
In case of translation motion we can decide the direction of friction force by direct observation.
But the direction of friction cannot be found by direct observation in case of rotational
motion, as the body is translating as well as rotating. The direction of friction force is
determined after deciding the motion tendency of the point of contact of the body under
consideration with the ground.
Let us take an example of analysis of direction of friction in this case.
In the following diagram (Fig. 3.306), a rolling object
of mass M and radius R is placed on a rough horizontal F R
surface. A force F is applied as shown in the figure. x
To find the direction of friction at P, let us consider the O
surface to be frictionless. aR at
Acceleration of point P due to translation only,
P
F
at = (Æ) Fig. 3.306
M
Acceleration of point P due to rotation only,
t FR
ar = aR = Rfi x(¨)
I I
ZŽƚĂƟŽŶപ525
  
Net acceleration of point P is a p = a t + a r
F FRx
ap = - (Æ) …(i)
M I
From Eq. (ii), it is clear that motion tendency at point P depends on both x and I.
Equation (i) can be written as
FÊ Rx ˆ
ap = Á 1 - 2 ˜¯ …(ii)
MË K
The motion tendency of point P will be in forward direction if
Rx Rx
1– > 0 fi 1 > 2 fi K 2 > Rx …(iii)
K2 K
If this condition is satisfied, friction will act in backward direction.
We can summarise the situation as
If K2 > Rx: friction will act in backward direction.
If K2 = Rx: no friction will act.
If K2 < Rx: friction will act in forward direction.
If force acts in lower diametric plane, friction will act in backward direction only.

WORKED PROBLEMS
Problem 3.149 Draw the direction of friction force in the
following cases. If the rolling object is F R
R/2
(i) a ring (ii) a disc
(iii) a solid sphere (iv) a hollow sphere O

Solution
R P
Case I: K2 = R2 fi Rx = R (MK2 = MR2)
2 Fig. 3.307
As K2 > Rx,
The friction will act in the backward direction
R2 R R2 F R
Case II: K2 = fi Rx = R = (MK2 = MR2/2) R/2
2 2 2
O
fi 2
K = Rx
No friction will act.
P
2 2 R R2 2
Case III: K2 = R fi Rx = R = (MK2 = MR2)
5 2 2 5 Fig. 3.308
fi 2
K < Rx
The friction will act in the forward direction.
2 2 R R2
Case IV: K2 = R fi Rx = R =
3 2 2
fi 2
K > Rx
The friction will act in the forward direction.
526പWŚLJƐŝĐƐĨŽƌ//dͲ:͗DĞĐŚĂŶŝĐƐ//

Problem 3.150 A cylinder of weight w and radius R is to be raised on to a horizontal


step of height h as shown in Fig. 3.309. A rope is wrapped around the cylinder and pulled
horizontally. Assuming the cylinder does not slip on the step, find the minimum force F
necessary to raise the cylinder and the reaction force at P exerted by the step on the cylinder.
C F

n
F
2R – h
O
O
q
R n
R–h P w
P
d
h q

w F
Q
(a) (b) (c)

Fig. 3.309

Solution Problem solving strategy: At the instant the cylinder is about to be raised, the
reaction at Q vanishes. Hence, there are only three forces acting on the cylinder, as shown in
Fig. 3.309(b). We choose the point P as the point about which to take the torque. The torque
of the normal reaction, N is zero, since its line of action passes through the point about which
we take the torque.
The moment arm d of the weight relative to the point P is

d= R2 - (R - h)2 = 2Rh - h 2
The moment arm of F relative to P is 2R – h.
The net torque about P is
wd – F(2R – h) = 0

w 2Rh - h 2 - F(2R - h) = 0
w 2Rh - h 2
F = ...(1)
2R - h
The components of normal reaction N can be determined from the first condition of
equilibrium. Figure 3.309(c) shows that the vector sum of the three external forces is zero.
ÂFx = F – N cos q = 0 or N cos q = F ...(2)
ÂFy = N sin q – w = 0 or N sin q = w ...(3)
Dividing Eq. (3) by Eq. (2), we get
w
tan q =
F
On eliminating q from Eqs. (2) and (3), we get

N = w2 + F2
ZŽƚĂƟŽŶപ527

Problem 3.151 Find the tension in the tape and the linear
acceleration of the cylinder up the incline, assuming there is no
slipping. (Take M = 4m and g = 10 m/s2) B
Solution Since the cylinder does not slip, the displacement, M
A
velocity and acceleration of the hanging mass are, respectively, m
equal to the displacement, velocity and acceleration of the upper a
point B of the cylinder as the string leaves the upper point
Fig. 3.310
tangentially.
Now v (velocity of the upper point) = vm + w r = 2vCM (\ w r = vm)
From conservation of energy, when m lowers through h
1 Ê1 1 ˆ
mgh = mv 2 + Á mvCM
2
+ Iw 2 ˜
2 Ë 2 2 ¯
(\ loss in PE = gain in KE)
2
1 1 Mv 2 1 Ê 1 ˆv
= mv 2 + + Á Mr 2 ˜ CM
2 2 4 2Ë 2 ¯ r2
Ê 1 2ˆ
ÁË∵ I = Mr ˜¯
2
1 1 1 v2
= mv 2 + Mv 2 + M
2 8 4 4
1 3
= mv 2 + Mv 2
2 16
16 mgh = 8 mv2 + 3 Mv2
16 mgh
fi v2 =
8m + 3 M
From v2 = 2ah
8 mg g
fi a = = = 4 m/s 2
8 m + 3 M 1 + 3 M/8 m
Considering downward motion of m,
mg – T = ma
or T = mg – ma = 2(10 – 4)
= 12 N
Problem 3.152 Figure 3.311 shows a girl on a swing. Assume that the centre of mass of the
girl in crouched position is at 1.2 m from the ground. The weight of the girl is 400 N and
her centre of mass is 3.7 m from the pivot of the swing in her crouched position. The swing
is released from rest and at the bottom of the arc the girl stands up instantaneously, thus
raising her centre of mass 0.6 m. What is the height of her centre of mass at the top of the arc?
Solution When the swing reaches position B, the line of action of gravitational force passes
through the pivot of the swing. The torque of this only external force is zero. The angular
momentum is conserved at B, while the girl stands up.
528പWŚLJƐŝĐƐĨŽƌ//dͲ:͗DĞĐŚĂŶŝĐƐ//

Considering the girl-swing system to be


a rigid body just before and after she stands m
0.6
up, we have
Iw = I¢w
where I = ICM + mR2 3.7 m
I¢ = I¢CM + mR¢2
h
R and R¢ are the distances from the pivot to B¢
the CM of the girl before and after, ICM and
1.2 m
I¢CM are the moments of inertia of the girl B 0.6 m
U=0
about CM before and after. Since R and R¢
are large compared with the dimensions of
the girl, we have ICM < mR2 and I¢CM < mR¢2.
Fig. 3.311
So we can write approximately
mR2w = mR¢2w¢
or mRvB = mR¢v¢B
where vB and v¢B are the speeds of centre of mass.
Thus, 3.7 mvB = 3.1 mv¢B or v¢B = 1.2vB
From conservation of energy,
1
mv2B = mg(1.2 – 0.6)
2
or vB = 3.43 m/s
So v¢B = 1.2vB = 4.1 m/s
Again we can apply conservation of energy.
1
mv¢B2 = mg(h – 1.2)
2
v¢ 2
or h = B + 1.2 = 2.1 m
2g

Problem 3.153 An elastic spherical ball of mass M and radius a moving with velocity v
strikes a rigid surface at an angle q to the normal. Assuming it skids while in contact with
the surface, the tangential frictional force being a constant fraction m of the normal reaction
force, show that:
(a) the ball is reflected at an angle f to the normal where (tan q – tan f) = 2m; and
5mv
(b) the angular velocity of the rebounding ball changes by an amount cos q.
a
You may assume that the component of velocity perpendicular to the surface is reversed
in direction without change of magnitude.
Solution
(a) We cannot assume that the ball is not initially rotating because if the ball was not
rotating, it could not skid against the surface; so the angle q and f would be identical.
ZŽƚĂƟŽŶപ529

Let us therefore assume that it has an initial angular


velocity w1 and a final angular velocity w2, as shown in
Fig. 3.312.
M u
It is given that the perpendicular component of the

1
w

w2
velocity is reversed by the impact; so (with the notation
of figure) v
q f
v cos q = u cos f ...(i)
The change in the perpendicular component of the mo-
Fig. 3.312
mentum is 2 Mv cos q, which is equal to the perpendicu-
lar component of the impulse delivered to the ball by the surface. Since the frictional
force is m times the perpendicular force, the component of the impulse parallel to the
surface must be 2 Mvm cos q.
If the ball is rotating clockwise as shown in Fig. 3.312, this impulse will be to the right,
but it would be to the left if the ball were rotating anticlockwise.
The parallel component of the ball’s momentum must thus change by 2 Mvm cos q; so
the parallel component of the velocity changes by 2vm cos q. Thus,
u sin f – v sin q = 2m v cos q ...(ii)
Equations (i) and (ii) give tan f – tan q = 2m but if the original angular velocity were
anticlockwise instead of clockwise, we could obtain
tan f – tan q = – 2m
so we may put |tan q – tan f| = 2m
(b) We have shown that the horizontal component of the impulse delivered to the ball is
2 Mvm cos q; and since the ball’s radius is a, the impulsive moment (angular impulse) is
2 Mv ma cos q. Since the impulsive moment is equal to the change in angular momentum
and the angular momentum is given by Iw, where I is the moment of inertia, we must
have
2 m Mva cos q 2 m Mva cos q 5 m v cos q
Dw = = =
I (2/5)Ma 2 a

Problem 3.154 A cone of height h and base radius R is


constrained to rotate about its vertical axis as shown in Fig. r
w0
3.313. A thin straight groove is cut in the surface of the cone g
starting from the apex to the base as shown in Fig. 3.313. The
cone is given an angular velocity w0 around its axis. m
A small bead of mass m is released at the top of the h
frictionless groove and is allowed to slide down under gravity.
The moment of inertia of the cone about its symmetry axis is
I 0. (IIT-JEE, 1971)
R
(a) What is the angular velocity of the cone when the bead
reaches the bottom?

(b) Find the speed of the bead in the laboratory frame at î v^
v||
the instant the bead leaves the cone.
Fig. 3.313
530പWŚLJƐŝĐƐĨŽƌ//dͲ:͗DĞĐŚĂŶŝĐƐ//

Solution
(a) Let w be the angular velocity of the system (bead + cone), when the bead reaches the
base of the cone. From conservation of angular momentum,
L i = Lf
I0w0 = (I0 + mR2)w
I 0w 0
Hence, w = ...(1)
(I 0 + mR2 )
(b) We assign x- and y-axis normal to the groove and parallel to the groove respectively.
When the bead reaches the base of the cone, it has two components of velocity:
v|| due to translation along groove,
v ^ due to rotation with cone.
Hence, velocity of the bead

v = v^ i + v||j

with |v| = v||2 + v^2 = v||2 + (Rw )2

As the energy of the system is conserved,


KEi + GPEi = KFf + GPEf
1 1 1
I 0w 02 = mgh = mv 2 + I 0w 2 + 0 ...(2)
2 2 2
where v2 = v2|| + v2^ ...(3)
From Eqs. (2) and (3),
1 1 1 1
mv||2 = I 0w 02 + mgh - I 0w 2 - mR2w 2 ...(4)
2 2 2 2
On substituting expression for w from Eq. (1), we obtain
(I 0 + mR2 ) I 02w 02
v|2| = I0w02 – ¥ + 2 gh
m (I 0 + mR2 )2
I 0w 02 R2
= + 2 gh
I 0 + mR2
Hence, the velocity of the bead when it reaches the bottom is

v = v^ i + v||j
I 0w 0 R  È I 0w 02 R2 ˘
= 2
i + Í 2
+ 2 gh ˙ j
I 0 + mR Î I 0 + mR ˚

2
  Ê I 0w 0 R ˆ I 0w 02 R2
Magnitude of v , |v| = Á I + mR2 ˜ + I + mR2 + 2 gh
Ë 0 ¯ 0
ZŽƚĂƟŽŶപ531

Problem 3.155 A plate of mass m is placed m


v
on a solid and hollow spheres each of mass
m. If the speed of the plate is v, assuming Solid Hollow
m sphere m
sphere
pure rolling of the spheres with all contacting
surfaces, find the kinetic energy of the system
(spheres + plate).
Fig. 3.314
Solution The total KE is
K = Krod + Khollow sphere + Ksolid sphere ...(i)
1
where Krod = mv 2 ...(ii)
2
v
Since the CM of each sphere moves with a velocity vC = ,
2
1 Ê K2 ˆ
Khollow sphere = mvC2 Á 1 + 2 ˜ m
2 Ë R ¯ v

2 v v
1 Ê vˆ Ê 2ˆ
= mÁ ˜ Á1 + ˜ 2 2
2 Ë 2¯ Ë 3¯

5mv 2 Fig. 3.315


= ...(iii)
24
2
1 Ê vˆ Ê 2ˆ
Ksolid sphere = mÁ ˜ Á1 + ˜
2 Ë 2¯ Ë 5¯
7
= mv 2 ...(iv)
40
Using the above four equations,
1 5 7
K = mv 2 + mv 2 + mv 2
2 24 40
53
= mv 2 Ans.
60

Problem 3.156 A uniform rod of mass m and length


l is suspended by two inextensible strings at A and
B. If the string at B suddenly snaps, determine the
tension T in the string at A immediately after the
break occurs.
60° l 60°
Solution When the string at B snaps, the rod has a
horizontal acceleration and a vertical acceleration. In A B
addition, the rod will have an angular acceleration Fig. 3.316
a about the centre of mass. Thus, the equations of
motion are
532പWŚLJƐŝĐƐĨŽƌ//dͲ:͗DĞĐŚĂŶŝĐƐ//

l Ê ml 2 ˆ
Ât = Y sin 60° =Á a ...(1)
2 Ë 12 ˜¯
ÂFy = mg – T sin 60° = may ...(2)
ÂFx = T cos 60° = max ...(3)
We have three equations in four unknowns. We will search for a kinematic relation. The
point A on the rod will move on a circle in the subsequent motion. It can have radial and
tangential acceleration. The radial acceleration (= v2/r) is zero at the instant the string snaps,
so the total acceleration of point A along the string is zero initially.
a l/2 T sin 60°
T

a
60° O
ax A T cos 60° B
ax
mg
ay
ay
(a) (b)

Fig. 3.317

al
Thus, we have ay cos 30° – cos 30° – ax cos 60° = 0
2
Solving Eqs. (1), (2) and (3), we get

Ê T ˆ 3T T
ÁË g - sin 60∞˜¯ cos 30∞ - sin 60∞ cos 30∞ - cos 2 60∞ = 0
m m m
T È 3 3 3 3 3 1 ˘ T È 3 9 1 ˘ T È 13 ˘
g cos 30° = Í + + ˙= + + =
mÎ 2 2 2 2 4 ˚ m ÍÎ 4 4 4 ˙˚ m ÍÎ 4 ˙˚

Ê 4ˆ
T = mg Á ˜ cos 30∞
Ë 13 ¯

2 3
T = mg
13
The point A has tangential acceleration; therefore, the relation t = la is not applicable
about point A. This relation t = la is valid either about a fixed point or about the centre of
mass of the object even though it is accelerated.
Problem 3.157 A uniform disc of radius r0 lies on a smooth horizontal plane. A similar disc
spinning with the angular velocity w0 is carefully lowered onto the first disc. How soon do
both discs spin with the same angular velocity if the friction coefficient between them is
equal to m?
ZŽƚĂƟŽŶപ533

Solution When the disc spins with its plane on the table as shown w0
in Fig. 3.318, to calculate frictional torque consider the disc to be
made of large number of concentric rings. The mass of a ring of
radius r and thickness dr will be
dM = (M/pR2)(2prdr)
So that force of friction on the ring will be
m Mg
df = m (dM)g = (2p r dr )
p R2 Fig. 3.318
and so frictional torque on the ring
2 mMg 2
dt = r(df) = r dr
R2
So, the total frictional torque on the disc,
R 2 m Mg 2 2
t = Ú0 2
R
r dr =
3
m MgR

2
So, Ia = mMgR [as t = Ia]
3
1 Ê dw ˆ 2 È 1 dw ˘
or MR2 Á - = m MgR Ías I = MR2 and a = -
2 Ë dt ˜¯ 3 Î 2 dt ˙˚
t 3 R 0
or Ú0 dt = -
4 m g Úw 0
dw

3 Rw 0
or t =
4 mg
Solving problems involving variable forces.
In some situations force is not constant. In such situations acceleration may be function
of time, displacement and velocity.
Problem 3.158 A string is wrapped several times on
a cylinder of mass M and radius R. The cylinder is R
pivoted about its axis of block symmetry. A block of
mass m tied to the string rests on a support positioned
so that the string has no slack. The block is carefully
lifted vertically a distance h and the support is
removed as shown in Fig. 3.319 Just before the string m
becomes taut (a) evaluate the angular velocity w0 of m h
the cylinder, the speed v0 of the falling body m and
the kinetic energy K0 of the system. (b) Evaluate the
corresponding quantities w1, v1 and K1 for the instant
just after the string becomes taut. (c) Why is K1 less
than K0? Where does the energy go? (d) If M = m, Fig. 3.319
what fraction of the kinetic energy is lost when the
string becomes taut?
534പWŚLJƐŝĐƐĨŽƌ//dͲ:͗DĞĐŚĂŶŝĐƐ//

Solution
(a) Just before the string becomes taut, the block falls freely, so v0 = 2gh. There is no
tension in the string, so nothing causes the cylinder to spin; so w0 = 0. The kinetic
energy of the system is
1 1
K0 = mv02 + Iw 02 = mgh
2 2
(b) When the string experiences a jerk, the large impulse developed is of very short
duration so that the contribution of weight mg can be neglected during this time
interval.
The angular momentum of the system is conserved, as the tension is internal force for
the system. Thus, we have
 
Li = L f
1
mv1R + MR2w1 = mv0R = m 2ghR
2
The string is inextensible, so v1 = Rw1. On solving for w1, we get
2 gh
w1 =
R[1 + ( M/2m)]

2 gh
v1 = Rw1 =
[1 + ( M/2m)]
The final kinetic energy K1 is given by
1 1
K1 = mv12 + Iw 12
2 2

ˆÊ v ˆ
2
1 1Ê 1
= mv12 + Á MR2 ˜ Á 12 ˜
2 2Ë 2 ¯ËR ¯

1Ê Mˆ 2
= Á m + ˜¯ v1
2Ë 2

1È mv02 ˘
= Í ˙
2 Î 1 + ( M/2m) ˚

K0
=
1 + ( M/2m)
(c) The situation in this case is analogous to the energy loss in completely inelastic two-
body collisions. The lost kinetic energy is converted to heat energy or elastic potential
energy of the string or in the two objects.
2K 0 Ê K - K1 ˆ 1
(d) For M = m, K1 = , so the fraction lost is Á 0 = .
3 Ë K 0 ˜¯ 3
ZŽƚĂƟŽŶപ535

Problem 3.159 The Fig. 3.320 shows a system consisting of (i) z


a ring of outer radius 3R rolling clockwise without slipping on
w
a horizontal surface with angular speed w and (ii) an inner disc
of radius 2R rotating anticlockwise with angular speed w/2. w/2
The ring and disc are separated by frictionless ball bearings. 3R P
R
The system is in the x–z plane. The point P on the inner disc 30° x
is at a distance R from the origin, where OP makes an angle O
2R
of 30° with the horizontal. Then with respect to the horizontal
surface: (IIT-JEE, 2012)
(a) the point O has a linear velocity 3Rwi

11  3 
(b) the point P has a linear velocity Rw i + Rw k Fig. 3.320
4 4
13 3
(c) the point P has a linear velocity Rw i - 
Rw k
4 4
Ê 3ˆ  1 
(d) the point P has a linear velocity Á 3 - ˜ Rw i + 4 Rw k
Ë 4 ¯
w
Solution VP = 3Rw i + (- j) ¥ (R cos 30∞i + R sin 30∞k
)
2
w  w  11  3
= 3Rw i + 3 Rk - Ri = Rw i + 
Rw k
4 4 4 4

Problem 3.160 A small sphere rolls down without slipping from the top of a track in a
vertical plane. The track has an elevated section and a horizontal part. The horizontal part
is 1.0 m above the ground level and the top of the track is 2.4 m above the ground. Find
the distance on the ground with respect to B (which is vertically below the end of the track)
where the sphere lands. During its flight as a projectile does the sphere continue to rotate
about its centre of mass. Explain. (IIT-JEE, 1987)
Solution Loss in PE when the sphere rolls from C to A
= Mg(2.4 – 1) = 1.4 Mg
This potential energy is converted into KE of rolling
1 1
K= Mv 2 + Iw 2
2 2
Now as for sphere I = (2/5) Mr2 and for rolling v = rw
1 2 7
So, K= Mv 2 + Mv 2 = Mv 2
2 5 10
So, by conservation of mechanical energy:
1.4 Mg = (7/10)Mv2 or v= 2g
536പWŚLJƐŝĐƐĨŽƌ//dͲ:͗DĞĐŚĂŶŝĐƐ//

2.4 m
A

1m

Fig. 3.321

Now at A as v is horizontal, time taken by the sphere to reach the ground

2h 2¥1 2
t= = =
g g g
So the horizontal distance moved by the sphere in this time
x = vt
= 2 g ¥ (2/g )
=2m
i.e., sphere falls at a distance of 2 m from B.
Now before leaving the point A the sphere has an angular momentum (= Iw) and as in
flight torque of mg will be zero (as mg will pass through the centre of mass of sphere), so
angular momentum will remain constant. This, in turn, implies that sphere will continue to
rotate about its centre of mass during flight.
Problem 3.161 A small solid marble of mass M and radius r rolls down along the loop
track, without slipping. Find the height h above the base, from where it has to start rolling
down the incline such that the sphere just completes the vertical circular loop of radius R.
Solution Here the centre of mass of marble will P
move in a circle of radius (R – r) so for just looping
the loop, at H H
v = g( R - r ) ...(i)
h O
Now as in rolling Q
R
K = KT + KR
L
1 1
= Mv 2 + Iw 2 Fig. 3.322
2 2
And here I = (2/5)Mr2 with v = rw
2
1 1 È2 ˘ Èv˘
So, K = Mv 2 + Í Mr 2 ˙ Í ˙
2 2 Î5 ˚Îr˚
ZŽƚĂƟŽŶപ537

1 1 7
i.e., K = Mv 2 + Mv 2 = Mv 2 ...(ii)
2 5 10
So in the light of Eqs. (i) and (ii) becomes
7
K = Mg(R - r ) ...(iii)
10
As this kinetic energy is provided by loss in PE, so applying conservation of mechanical
energy between P and H.
7
0 + Mgh = Mg(R – r) + Mg(2R – r)
10
1
or h = [27 R - 17 r ]
10
Note: In this problem due to rolling motion KE of sphere at any point is (7/10)mv2 and not
(1/2)mv2. So if we apply conservation of ME between P and L taking vL = 5gr the solution
1
will become wrong as in deriving it from vH = gr we have taken KE at L equal to mv2L.
2
Problem 3.162 State whether the statement given below is true of false giving reason in
brief.
“A ring of mass 0.3 kg and radius 0.1 m and a solid cylinder of mass 0.4 kg and of the same
radius are given the same kinetic energy and released simultaneously on a flat horizontal
surface such that they begin to roll as soon as released towards a wall which is at the same
distance from the ring and cylinder. The rolling friction in both the cases is negligible. The
cylinder will reach the wall first.”
Solution In case of rolling as
1 1
K = Mv 2 + Iw 2 with v = rw
2 2
1
So, K = Mv2 [1 + I/Mr2]
2
For ring I = Mr2
\ KR = Mv2R, i.e., vR = K R/0.3
For cylinder, I = (1/2) Mr2
KC = (3/4)MvC2 , i.e., VC = ( 4 KC/3 ¥ 0.4) = KC/0.3
Now according to the given problem KR = KC, so vR = vC and as the motion is uniform,
both will reach the wall simultaneously, i.e., given statement is wrong.
Problem 3.163 A body of mass M and radius r, rolling on a smooth horizontal floor with
velocity v, rolls up an irregular inclined plane up to a vertical height (3v2/4g). Compute the
moment of inertia of the body and comment on its shape. (IIT-JEE, 2007)
538പWŚLJƐŝĐƐĨŽƌ//dͲ:͗DĞĐŚĂŶŝĐƐ//

Solution The total kinetic energy of the body:


K = KT + KR
1 1
= Mv 2 + Iw 2 h
2 2
1
or K = Mv2[1 + (I/Mr2)] [as v = rw]
2 Fig. 3.323
2
When it rolls up an irregular inclined plane of height (h = 3v /4g), its KE is converted
into PE, so by conservation of mechanical energy
1 È I ˘ È 3v 2 ˘
Mv 2 Í1 + = Mg Í ˙
2 Î Mr 2 ˙˚ Î 4g ˚
which on simplification gives I = (1/2)Mr2. This result clearly.
Problem 3.164 A reeled carpet of radius R is gently
pushed towards right. If it rolls without sliding and
unreels, find the speed of the CM of the rolling position
when its radius is halved. Neglect the vertical motion
of the CM. (IIT-JEE, 1990)
Solution If r is the density of the material of the
carpet, initial mass of the carpet (cylinder) M will be
R
pR2Lr while when its radius becomes half the mass
of cylindrical part will be
MF = p(R/2)2 Lr = M/4
Fig. 3.324
So, initial PE of the carpet is MgR while final
(M/4)g (R/2) = MgR/8
So, loss in potential energy when due to unrolling radius changes from R to R/2
= MgR – (1/8)MgR = (7/8)MgR ...(i)

R
R
2

(a) (b)

Fig. 3.325
ZŽƚĂƟŽŶപ539

This loss in potential energy is equal to increase in rotational KE which is


1 1
K = K T + KR = Mv 2 + Iw 2
2 2
If v is the velocity when half the carpet has unrolled then as
2
R M 1 È M ˘ ÈR˘
v = w, M Æ and I = Í ˙ Í ˙
2 4 2 Î 4 ˚Î 2 ˚

1 È M ˘ 2 1 È MR2 ˘ È 2v ˘
2
K = v + Í ˙
2 ÍÎ 4 ˙˚ 2 Î 32 ˚ ÍÎ R ˙˚

1 1 3
i.e., K = Mv 2 + Mv 2 = Mv 2 ...(ii)
8 16 16
So from Eqs. (i) and (ii),
(3/16)Mv2 = (7/8)MgR

i.e., v = (14 gR/3)

Problem 3.165 A cone rolls without slipping along a w


plane. The axis of the cone rotates with a velocity w around B W
a vertical line passing through its apex. The height of the D1
cone is h and the angle between generatrix of the cone is r
C
a. What is the angular velocity with which the cone rotates h
r h tan a
around its axis? Also determine the linear velocity of an D2
arbitrary point on the diameter of the cone base lying in O a
the vertical plane. h/cos a A

Solution Since the cone rolls without slipping, the points Fig. 3.326
on the generatrix OA should be instantaneously at rest.
Point A is common to two rotations, one due to w and the other due to W. So for point A
we should have

Ê h ˆ
wÁ = W(h tan a)
Ë cos a ˜¯
w
or W =
sin a

The velocity of an arbitrary point D1 on diameter AB of the cone base is the sum of two
velocities:
rw
v1 = w (h cos a – r sin a) +
sin a
where r is the distance from the centre of the base C to the given point.
540പWŚLJƐŝĐƐĨŽƌ//dͲ:͗DĞĐŚĂŶŝĐƐ//

For point D2 below centre C, we have


rw
v2 = w(h cos a + r sin a) –
sin a

The velocity of the lowermost point is zero and of the uppermost point v = 2wh cos a.

Problem 3.166 A heavy plank of mass 102.5 kg is placed F


over two cylindrical rollers of radii R = 10 CM and r = 5
CM. Mass of rollers is 40 kg and 20 kg, respectively. Plank
10 cm
is pulled towards right by applying a horizontal force F
= 25 N as shown in Fig. 3.327. During the first second of
Fig. 3.327
motion, the plank gets displaced by 10 CM. If the plank
remains horizontal and slipping does not take place, calculate the magnitude and direction
of force of friction acting between
(a) plank and bigger roller
(b) plank and smaller roller
(c) bigger roller and floor
(d) smaller roller and floor
Solution Since force F is constant, therefore the plank moves with a constant acceleration.
Let that acceleration be a. For the motion of the plank,
u = 0, t = 1 second, s = 10 CM = 0.10 m
Using, s = ut + 1/2at2; a = 0.20 m/s2
Accelerations of the top points of the rollers are also equal to a because these points are
in contact with the plank and there is no slipping. Hence, angular acceleration a1 and a2 of
the bigger and smaller rollers (respectively) are given by a1 = a/2R and a2 = a/2r.
or a1 = 1 rad/s2 and a2 = 2 rad/s2
Acceleration of the centre of mass of the bigger and 102.5g
smaller rollers are a1 = Ra1 and a2 = ra2, respectively.
A B F
\ a1 = a2 = 0.10 m/s2
F1 F2
Now considering the free-body diagram (as shown N2
N1 102.5a
in Fig. 3.328).
Since, plank tries to slide (towards right) over the top N1 N2
surfaces of rollers, therefore the rollers exert frictional F1 F2
forces on the plank towards the left. l1 a 1
A B
For the vertical forces on the plank, 40g l 2a 2
20
N1 + N2 = 102.5g …(i) 40a1 20a2
C D
For the horizontal forces on the plank, F3 F3
v1 v2
F – F1 – F2 = 102.5a …(ii)
Big roller Small roller

Fig. 3.328
ZŽƚĂƟŽŶപ541

Since instantaneous axes of rotation of the bigger and smaller rollers are C and D,
respectively, therefore taking moments of forces acting on these rollers about their respective
instantaneous axes of rotation are as follows:
For bigger roller, F1 (2R) = I1 a1 ...(iii)
For smaller roller, F2 (2r) = I2 a2 ....(iv)

where F1 (3/2) (40) R2 – 0.6 kg m2 (moment of the inertia of the bigger roller about axis C)

3
Similarly, I2 = (20)r2 = 0.075 kg m2
2

From Eqs. (iii) and (iv), F1 = 3 N; F2 = 1.50 N


For the horizontal forces acting on the bigger roller,
F1 – F3 = 40a1 or F3 = F1 – 40a1
or F1 = – 1 N
(–ve sign indicates that friction between bigger roller and the floor is opposite to its
assumed direction)
\ |F3| = 1 N (rightwards)
For the horizontal forces acting on the smaller roller,
F2 – F4 = 20a2
\ F4 = – 0.50 N
or |F4| = 0.50 N (rightwards)

Problem 3.167 The assembly of two discs as shown in Fig.


3.329 is placed on a rough horizontal surface and the front
disc is given an initial angular velocity w0. Determine the final R w0
linear and angular velocity when both the discs start rolling. It
is given that friction is sufficient to sustain rolling in the rear
wheel from the starting of motion. Fig. 3.329

Solution Li = Lf (about bottommost point)


\ Iw0 = 2[Iw + mRv]

Ê1 2ˆ È1 2 ˘
or ÁË mR ˜¯ w 0 = 2 Í mR w + mR(w R)˙
2 Î 2 ˚
542പWŚLJƐŝĐƐĨŽƌ//dͲ:͗DĞĐŚĂŶŝĐƐ//

w0

v
w w

Fig. 3.330

Problem 3.168 The top in Fig. 3.331 has a moment of inertia A¢


equal to 4.00 ¥ 10–4 kgm2 and is initially at rest. It is free to rotate
about the stationary axis AA¢. A string wrapped around a peg r
F
along the axis of the top is pulled in such a manner as to maintain
a constant tension of 5.57 N. If the string does not slip while it is
unwound from the peg, what is the angular speed of the top after
80.0 CM of string has been pulled off the peg?
Solution Work done = FDr = (5.57 N)(0.800 m) = 4.46 J A

1 2 1 2 Fig. 3.331
and Work = DK = Iw f - Iw i
2 2
(The last term is zero because the top starts from rest).
1
Thus, 4.46 J = (4.00 ¥ 10–4 kg ¥ m2)w2f
2
and from this, wf = 149 rad/s.

Problem 3.169 Two skaters, each of


mass 50 kg, approach each other along v
parallel paths separated by 3 m. They w
have equal and opposite velocities
v
of 10 m/s. The first skater carries a
long light pole, 3 m long, and the
(a) (b)
second skater grabs the end of it as
he passes (assume frictionless ice). (a)
Describe quantitatively the motion of Fig. 3.332
the skaters after they are connected by
the pole. (b) By pulling on the pole the skaters reduce their distance apart to 1 m. What is
their motion then? (c) Compare the KE’s of the system in part (a.) Where does the change
come from?
ZŽƚĂƟŽŶപ543

Solution
(a) As the initial linear momentum of the system (skater + pole) is zero; the centre of mass
will be at rest before and after the collision.
The skaters and the pole will rotate around the centre of mass (at the mid-point of the
pole). Free rotation of a system always takes place about CM.
Applying the conservation of angular momentum about an axis through C and
perpendicular to the plane of the figure,
mvl + mvl = Iw where I = 2m(l)2
2mvl v
w = =
2ml 2 l
w = 20/3 rad/s
(b) When the skaters reduce their separation, no external torque acts on the system; hence
we can apply conservation of angular momentum about CM. Due to redistribution of
mass about the axis of rotation, moment of inertia changes.
Since the separation reduces to 2l¢ = 1 m,
Iw = I¢w¢ (conservation of angular momentum)

Iw 2ml 2w
w¢ = = = 9w = 60 rad/s fi angular velocity increases
I¢ 2ml ¢ 2
1
KE f I ¢w ¢ 2 (I ¢w ¢ )w ¢ w ¢
(c) = 2 = = =9
KEi 1 2 (Iw )w w
Iw
2
The kinetic energy increases because the skaters do positive work in pulling themselves
towards the centre of the pole.

Problem 3.170 A yo-yo of mass M lies on a smooth horizontal N

table as shown in Fig. 3.333(a). Assume that moment of inertia


1 F
about centre of mass is MR2. A string is pulled with force F from r
2
the inner radius r as shown in Fig. 3.333(a). R q
f
(a) In what direction will the yo-yo roll if q = 0, p/2 or p?
(b) For what value of q will the yo-yo slide without rolling
independent of the roughness (coefficient of friction) of the Mg
table or the magnitude of F?
Fig. 3.333(a)
(c) At what angle q will the yo-yo roll, independent of the
smoothness of the table?
544പWŚLJƐŝĐƐĨŽƌ//dͲ:͗DĞĐŚĂŶŝĐƐ//

Solution We assume that the yo-yo is at rest before the application of the force F.
(a) As there is no friction acting on the yo-yo the direction of rolling is only determined
by the direction of the torque of the applied force F about its centre.

w w w
F
Moment O
arm F O O

q=0 q = p/2 q=p

Fig. 3.333(b)

(b) The friction force on the yo-yo is f = mN. The yo-yo will slide without rolling if torque
of external force is balanced by torque of friction.
St = F ¥ r – mN ¥ R = 0 ...(1)
Also, SF = F cos q – mN = Ma ...(2)
From Eqs. (1) and (2),
Ma r
cos q = +
F R

If this condition is satisfied, q is independent of m. It still depends on F unless a = 0,


i.e., no motion takes place.
(c) Let a and a be the acceleration of the centre of mass and angular acceleration of the
yo-yo.
The equations of motion are:

S f = F cos q – f = Ma ...(3)

1
St = f ¥ R – F ¥ r = MR2a ...(4)
2
For pure rolling of the yo-yo,
a = – Ra ...(5)

2F Ê 1 ˆ
Eliminating a and a gives f =Á r - R cos q ˜¯
RË 2
Since, f £ mN £ m(Mg – F sin q)
For the yo-yo to roll without slipping irrespective of the smoothness of the table, i.e.,
independent of M, we require
ZŽƚĂƟŽŶപ545

Mg 2r
sin q = , cos q =
F R
R Mg
or tan q =
2r F
N
Problem 3.171 A wheel of radius R, mass m and moment of
inertia I = mR2 is pulled along a horizontal surface by application
of force F to a rope unwinding from the axle of radius r as shown F
in Fig. 3.334. Friction is sufficient for pure rolling of the wheel. r
(a) What is the linear acceleration of the wheel? R q
(b) Calculate the frictional force that acts on the wheel. f
Solution The equations of motion of the wheel are
F – f = ma ...(1) Mg
F ¥ r + f ¥ R = Ia ...(2)
Fig. 3.334(a)
In case of pure rolling, a = Ra ...(3)
N
I
F ¥ r + (F – ma)R = a
R
F
F(r + R)R
or a = r
(I + mR2 )
R
and the frictional force is f = F – ma
È ˘ f
Í (r + R)R ˙
= F Í1 - ˙
Í Ê I + R2 ˆ ˙ Mg
ÍÎ ÁË m ˜¯ ˙
˚ Fig. 3.334(b)
F[(I/m) - Rr ]
=
[(I/m) + r 2 ]
For I/m = Rr, frictional force is zero.
For I/m > Rr, frictional force is positive, i.e., it acts in backward direction.
For I/m < Rr, frictional force is negative, i.e., it acts in forward direction.
y
Problem 3.172 Three particles A, B and C each of mass m are connected x
A
to each other by three massless rigid rods to form a rigid, equilateral
triangular body of side l. This body is placed on a horizontal frictionless
table (x–y plane) and is hinged at point A so that it can move without
F
friction about the vertical axis through A (see Fig. 3.335). The body is
set into rotational motion on the table about this axis with a constant B l C
angular velocity w. (a) Find the magnitude of the horizontal force exerted Fig. 3.335
by the hinge on the body. (b) At time T, when side BC is parallel to
x-axis, force F is applied on B along BC (as in the Fig. 3.335). Obtain the x-component
and the y-component of the force exerted by the hinge on the body, immediately after
time T. (IIT-JEE, 2002)
546പWŚLJƐŝĐƐĨŽƌ//dͲ:͗DĞĐŚĂŶŝĐƐ//

Solution (a) The centre of mass of the system is at the centroid of a triangular assembly.
The CM moves along a circular path with constant angular velocity. Therefore, there must
be a horizontal centripetal force directed towards the axis at the hinge.
From Fig. 3.335, we find
l 3
AD = l sin 60° =
2
2 l
AO = AD = =r
3 3
Ê l ˆ
The centripetal acceleration ac = w2r = w 2 Á
Ë 3 ˜¯
Ê l ˆ
Tangential acceleration at = ar = a Á
Ë 3 ˜¯

Fy y
A
x
Fx w
r a
ac ac =w 2r

O at = r a
(CM) F O
B C
D

(a) (b)

Fig. 3.336

Let Fx and Fy be the forces applied by the hinges along x-axis and y-axis, respectively. The
system is in non-centroidal rotation. The three equations of motion are
Ê l ˆ
SFx = Fx + F = (3m)at = 3m Á a …(i)
Ë 3 ˜¯
Ê l ˆ 2
SFy = Fy = 3m Á w …(ii)
Ë 3 ˜¯

Ê 3 ˆ
St = F ¥ Á l˜ = 2ml 2a …(iii)
Ë 2 ¯
3F
From Eq. (iii), a =
4 ml
Ê l ˆ 3F 3F
From Eq. (i), Fx + F = 3m Á ˜ ¥ =
Ë 3 ¯ 4 ml 4
F
fi Fx =
4
From Eq. (ii), Fy = 3 mlw 2
ZŽƚĂƟŽŶപ547

Problem 3.173 Figure 3.337(a) shows two blocks m1 and m2, connected by light strings,
supported on two identical pulleys of moment of inertia I and radius R. Determine the ac-
celeration of each block and the tensions T1, T2 and T3 in the string. There is no slipping
between string and pulleys.
T2

T1 T3
+
T1 T3
m1 m2

+
m1 m2 m1g m2g

(a) (b)

Fig. 3.337

Solution Force diagrams for blocks and pulleys are shown in Fig. 3.337(a); we consider
downward direction for m2 as positive and clockwise angular acceleration of pulleys as
positive. Equations for blocks are:
m 2g – T 3 = m 2a ...(1)
T 1 – m 1g = m 1a ...(2)
Equations for pulleys are:
(T2 – T1)R = Ia ...(3)
(T3 – T2)R = Ia ...(4)
Adding Eqs. (3) and (4) gives
(T3 – T1)R = 2Ia ...(5)
Adding Eqs. (1) and (2) gives
(T3 – T1) = (m2 – m1)g – (m1 + m2)a ...(6)
From Eqs. (6) and (5), we obtain
[(m2 – m1)g – (m1 + m2)a]R = 2Ia where a = a/R.

(m2 - m1 ) g
Thus, a =
I
m1 + m2 + 2 2
R

n1 n2
T2 T2

T1 mpg mpg T3

Fig. 3.338
548പWŚLJƐŝĐƐĨŽƌ//dͲ:͗DĞĐŚĂŶŝĐƐ//

Note that we may obtain T1 and T3 from Eqs. (1) and (2) and T2 from Eq. (3) by substituting
for a and a. If m1 = m2, a = 0 and if m1 > m2, a is negative; then motion is reversed in direction.
Problem 3.174 A ball of radius R and mass m is rolling without
slipping on a horizontal surface with velocity of its centre of mass vCM h
q
vCM. It then rolls without slipping up a hill to a height h before
momentarily coming to rest. Find h. Fig. 3.339
Solution Problem solving strategy: Mechanical energy is
conserved because static friction does not perform any work. The initial kinetic energy,
1
which is the translational kinetic energy of the centre of mass, mv2CM, plus the kinetic
2
1
energy of rotation about the centre of mass, ICMw2, is converted to potential energy mgh.
2
Since the sphere rolls without slipping. Rolling constraint equation is applicable, vCM = Rw.
Apply conservation of mechanical energy with Ui = 0 and Kf = 0.
Ef = Ei or Uf = Ki
The total initial kinetic energy Ki is vCM and angular speed w.

1 2 1
Ki = mvCM + I CMw 2
2 2
vCM 2
Substitute w = and I CM = mR2 and solve for Ki in terms of the mass and vCM.
R 5
2
1 2 1Ê2 ˆÊv ˆ
Ki = mvCM + Á mR2 ˜ Á CM ˜
2 2Ë5 ¯Ë R ¯

7 2
= mvCM
10
Set this initial kinetic energy equal to the final potential energy mgh.

7 2
mvCM = mgh
10
2
7 vCM
h =
10 g

WHAT IS PERCUSSION?
Centre of Percussion
Let us consider a bat which is stationary. The bat is struck by an impulse J at a distance y
from the centre of mass. From impulse-momentum equation,
J = mvCM
ZŽƚĂƟŽŶപ549

Torque exerted by this impulse causes the bat to rotate about the centre of mass. From
angular impulse-angular momentum theorem
tDt = DL = ICMw
Angular impulse is related to linear impulse through the relation
tDt = Jnet y
yJnet = ICMw
Any point on the bat has two velocities, the centre of mass velocity VCM and the tangential
velocity Vt caused due to rotation about the centre of mass. As shown in Fig. 3.340, these
two velocities are in opposite directions. If they are of equal magnitude, the point x acquires
no net velocity due to impulse.

CM x x CM x CM
y
J

Fig. 3.340 After being struck by an impulse the resulting motion can be thought of as the superposition
of translational and rotational motion about the centre of mass.

VCM = Vt, VCM = hw


J net yJ
= h net
m I
I CM
y =
mh
The point whose position is given by y is called the centre of percussion.
More about instantaneous axis of rotation
Figure 3.341 shows a rod which is resting on the walls starts slipping
on two walls. As the rod slides the centre of mass translates and N1
the rod rotates about an axis passing through the centre of mass x
y
and perpendicular to it. Such type of motion can be analysed by N2
ax
making equations of translation and rotation separately.
ay
For translation:
N1 = max ...(i)
and mg – N2 = may ...(ii) mg

Fig. 3.341
Also, ax2 + ay2 = a ...(iii)

For rotation of rod about CM:


N1y – N2x = Ia ...(iv)
550പWŚLJƐŝĐƐĨŽƌ//dͲ:͗DĞĐŚĂŶŝĐƐ//

Now solve the equations and obtain the result the velocity y
and acceleration of any end can be calculated. I1
Motion of such an object may be looked as pure rotation
about a point has zero velocity to simplify the study. Such A I2
a point is called instantaneous centre and the axis passing
through this point and perpendicular to the plane of motion
is called Instantaneous Axis of Rotation (IAOR). B
x
In Fig. 3.342 shown I1 and I2 are the instantaneous centres
of rotation of rod at two different instants. Fig. 3.342
To understand instantaneous centre consider a rigid body
which has plane motion. Let A be a point having velocity vA at I
the instant considered. Now locate a point I on perpendicular to
the direction of vA at A at a distance rA. The motion of A can be
split into translation of I and rotation about I. Thus, we can write
rB
vA = vI + w rA, rA
v
If we take rA = A , then
w
vA
vA = vI + w ¥ B
w
vB
\ vI = 0 vA
v A
Thus, point I is selected at a distance A along the perpen-
w
dicular to the direction of A, the plane motion of point A can be Fig. 3.343
reduced to pure rotation about I. Hence, I is the instantaneous centre.
If B is any other point on the rigid body then its velocity will be given by
vB = vI + wrB
or vB = wrB (vI = 0)

Method of Locating Instantaneous Centre


Instantaneous centre can be located by any of the following two methods.
(i) If the angular velocity w and linear velocity vA are known, instantaneous centre can
be located at a distance vA/w along the perpendicular to the direction of vA at A, as
discussed earlier.
(ii) If the linear velocities of two points of rigid body are known, say vA and vB, drop
perpendiculars to them at A and B. The intersection point is the instantaneous centre
(see Fig. 3.343).

WORKED PROBLEMS
Problem 3.175 A rod AB of length 3 m which remains in the same vertical plane has its
ends A and B constrained to remain in contact with a horizontal floor and a vertical wall
respectively as shown in Fig. 3.344. Determine the velocity and acceleration of the end B at
the position shown in figure, if the point A has a velocity of 2 m/s and an acceleration of
1.6 m/s2 rightward.
ZŽƚĂƟŽŶപ551

Solution

vA
B
°

A
60

vB
vB
3m vA = 2 m/s
aA = 1.6 m/s2
60°

A vA
(a) (b)

Fig. 3.344

Velocity of End A:
Method I
The motion of B can be looked as: translation of end B with velocity vA and rotation of B
about A with tangential velocity vBA. If w is the angular velocity of B about A, then vBA = w
 
¥ 3 = 3w. Since the velocity vector of end B moves downwards, so resultant of v A and v BA

must be equal to v B (see figure).
From Fig. 3.345(b)
vA
= tan 60°
vB
vA 2
\ vB = =
tan 60∞ tan 60∞
= 1.16 m/s Ans.
vA 2
Also, vBA = = = 2.3 m/s
sin 60∞ sin 60∞

Method II
 I
Velocity of end A, v A is horizontal, while velocity of the end B, B
 w
v B is vertical downward. Drop perpendiculars to the directions
  vB
of v A and v B at point A and B respectively. The intersection
point I be the instantaneous centre.
For the end A, vA = w (IA)
v 2 vA
\ w = A = = 0.77 rad/s 60°
I A 3 sin 60∞ A

For the end B, vB = w (IB) = 0.77 ¥ 3 cos 60° Fig. 3.345


= 1.16 m/s Ans.
Acceleration of End B
The acceleration of the end B has three components.
(i) Due to translation of end A, aA = 1.6 m/s2.
(ii) Due to rotation of end B about A, i.e., tangential acceleration
552പWŚLJƐŝĐƐĨŽƌ//dͲ:͗DĞĐŚĂŶŝĐƐ//

at = arAB = a ¥ 3 = 3a m/s2
(iii) Radial acceleration (from B towards A)
2
vBA
an =
rAB

(2.3)2
= = 1.76 m/s 2
3
y
The component of the acceleration of the end B are shown
in Fig. 3.346. As the net acceleration of the end is vertically B aA
downward, so net horizontal component of the accelerations 30° 60°
must be zero. i.e., an
at aB
aA + an cos 60°– at cos 30°
ax = 0 = 1.6 + 1.76 cos 60° – 3a cos 30°
or a = 0.95 rad/s2 A aA

Considering vertical components, we get x

an sin 60° + at sin 30° Fig. 3.346

aB = ay = 1.76 sin 60° + 3a sin 30°


= 2.98 m/s2 Ans.
y
w = 5 rad/s
Problem 3.176 A truck wheel of radius 1 m rolls freely with A
an angular velocity of 5 rad/s and with an angular acceleration a = 4 rad/s2
of 4 rad/s2, both clockwise as shown in Fig. 3.347. Determine m
B
the velocity and acceleration of points A and B. 6
60° x
O
Solution Velocity: Translational velocity of centre of mass O
v0 = wr = 5 ¥ 1 = 5 m/s
The motion of points A and B may be looked as translational
of O and rotation about O.
   Fig. 3.347
For point A, v A = vO + v AO
or vA = vO + w (OA) = 5 + 5 ¥ 1
= 10 m/s Ans. v0
30°
  
For the point B, v B = vO + v BO
60° vBO
O
or vB = vO2 2
+ vBO + 2vO vBO cos 30∞
Here vBO = w (OB)
= 5 ¥ 0.6 = 3 m/s

\ vB = 52 + 32 + 2 ¥ 5 ¥ 3 ¥ cos 30∞ Fig. 3.348

= 7.74 m/s Ans.


Acceleration: The acceleration at any point on the wheel (except centre) has three compo-
nents.
ZŽƚĂƟŽŶപ553

(i) Due to translation of wheel a0 = 4 m/s2


(ii) Tangential acceleration at = ar = 4r m/s2
(iii) Radial acceleration, towards centre of wheel
2
vBO
an =
r
Acceleration of Point A
rA = 1 m
\ at = 4 ¥ 1 = 4 m/s2
2
vAO 52
an = = = 25 m/s 2
rA 1
The resultant acceleration of point A, y
A at
2
aA = ( a0 + at) + an2 a0
an
= ( 4 + 4)2 + 252 = 26.25 m/s 2 Ans.
Acceleration of Point B
x
rB = 0.6 m O
\ at = 4 ¥ 0.6 = 2.4 m/s2
v2 32
an = BO = = 15 m/s 2
rB 0.6
Fig. 3.349
Problem 3.177 A long uniform rod AB or a metre stick of
length L and mass M is pivoted about a horizontal, frictionless pin B
through one end. The rod is released from rest in a vertical position as
shown in Fig. 3.350. At the instant the rod is horizontal. Find (a) the
angular velocity of the rod, (b) its angular acceleration, (c) the x and
y component of the acceleration of its CM, and (d) the component of
reaction force at the pivot.
A
Solution PIN
(a) Let M be the mass and L be the length of the metre scale. When Fig. 3.350
the upper end of the rod strikes the floor, its centre of gravity
falls through height L/2.
L
\ Loss in PE = Mg ◊
2
B

CG I0
CG

L/2 d = L/2

A I

Fig. 3.351
554പWŚLJƐŝĐƐĨŽƌ//dͲ:͗DĞĐŚĂŶŝĐƐ//

MI of the scale about the lower end A,


I = MI of the scale about the parallel axis through c.g. + Md2
ML2 ML2 ML2 È L˘
= I0 + Md2 = + = ÍÎ∵ d =
12 4 3 2 ˙˚
v v
Also, w = =
r L
Gain in rotational KE
1 2 1 ML2 v 2 Mv 2
= Iw = ◊ ◊ =
2 2 3 L2 6
Now, gain in rotational KE = Loss in PE
Mv 2 L
= Mg ◊ or v2 = 3gl
6 2
or v = 3 gL = 3 ¥ 9.8 ¥ 1 = 5.4 ms - 1 Ans.
(b) Torque acting on the rod about left end
Rx

Ry Mg

Fig. 3.352

L
t = Mg ¥
2
By Newton’s second law
L
Mg = Ia
2
L Ê ML2 ˆ
or Mg =Á ˜a
2 Ë 3 ¯
3g
\ a =
2L
(c) ax = w2 r
3g L
= ¥
L 2
3g
=
2
and ay = a r
3g L
= ¥
2L 2
3g
=
4
ZŽƚĂƟŽŶപ555

Thus, Fx = Max
3 Mg
=
2
For the translation of the centre of mass along y-axis, we have
Mg – Ry = May
Ê 3g ˆ
Ry = M Á ˜
Ë 4¯
Mg
\ Ry = Ans.
4

Problem 3.178 Two small flat discs, one with mass of 3 kg and other with 3 m/s
mass of 2 kg, are connected by a massless stiff rod 1.5 m long and rest on a
frictionless horizontal surface. A third disc of mass 4 kg slides at a constant
velocity of 3 m/s in a direction perpendicular to the rod, strikes the 2 kg Top 1.5 m
view
mass, and sticks to it. (a) Determine the position and the velocity of CM of
the entire system following the impact. (b) What is the angular frequency of
rotation of the rod after impact? (c) Is energy conserved in this impact, if not
how much kinetic energy is lost? Fig. 3.353
Solution
(a) The position of CM 4 kg 2 kg
6 ¥ 0 + 3 ¥ 1.5
y=
6+3 y
= 0.5 m from top.
Velocity of combined mass (4 + 2) kg after collision: CM

4 ¥ 3 + 0 = (4 + 2) v
\ v = 2 m/s
The velocity of CM is given by
m1v1 + m2v2
vcm = m1 + m2
3 kg

Fig. 3.354
È6 ¥ 2 + 3 ¥ 0˘
= Í ˙˚
Î 6+3
4
= m/s Ans.
3
(b) Using conservation of angular momentum about CM we have
4 ¥ 3 ¥ y = Iw
or 4 ¥ 3 ¥ 0.5 = [6 ¥ 0.52 + 3 ¥ 12] ¥ w
4
\ w = m/s Ans.
3
556പWŚLJƐŝĐƐĨŽƌ//dͲ:͗DĞĐŚĂŶŝĐƐ//

Problem 3.179 A uniform rod of length 2a is held with one end resting on a smooth
horizontal table making an angle a with the vertical. Show that when the rod is released its
angular velocity when it makes an angle q with the vertical is given by
1/2
È 6 a(cos a - cos q ) ˘
w= Í 2 ˙
Î a(1 + 3 sin q ) ˚

Solution If w is the angular velocity about an axis through IAOR, then velocity of CM will
be w (a sin q). In the process, loss in PE will equal to gain in KE, therefore
1 2 1
mga (cos a – cos q) = Iw + m(w a sin q )2
2 2
Since there is no force acting in the horizontal direction, therefore no translation of CM
in that direction. 1/2
È 6 a(cos a - cos q ) ˘
After simplifying, we get w = Í -1 ˙
Î a(1 + 3 sin q ) ˚

Instantaneous centre of rotation

CM a CM

q
a
a
q v = w a sin q
Smooth table Smooth table
(a) (b)

Fig. 3.355

Problem 3.180 A thin rod is held resting on the ground with its length inclined at an angle
a to the horizontal. The coefficient of friction between the rod and the ground is m. Show
that when the rod let go, it will start slipping on the ground, if
3 sin a cos a
m<
1 + 3 sin a
Solution By Newton’s second law
mg – R = ma cos a ...(i)
For not slipping, mR > ma sin a
a sin a
Solving the above equations, we get mR > ...(ii)
g - a cos a
a sin a
a sin a R a
and for slipping, m < ...(iii) a cos a
g - a cos a a mR
Smooth table
Fig. 3.356
ZŽƚĂƟŽŶപ557

a can be calculated as
a
Ia = mg ¥ L cos a where a= ...(iv)
L
3 sin a cos a
After solving equation, we get m <
1 + 3 sin a

Problem 3.181 A constant force P is exerted on a bar of Instantaneous


mass M. The rod is supported by frictionless wall. If the rod center of rotation
starts from a position of rest when q = 45°, as shown in Fig. w
3.357, what is its angular speed when the end A has moved y
a distance 1.5 m? B
C
y/2
Solution The length of the rod = 32 + 32 = 18 m
x/2 3 m
When the end A moves a distance x = 1.5 m, let the end
B move a distance y, which can be obtained as
(3 – x)2 + (3 + y)2 = 18 P
A q = 45°
1.5 m
Substituting x = 1.5 m, y will be equal to 0.97 m. Thus, 3m
the distances of ends A and B from axis of rotation are 1.5
m and 3.97 m respectively. Let vA and vB be the velocities of Fig. 3.357
the ends A and B respectively, then
vB = 1.5w and vA = 3.97w
The velocity of the centre C, 2 2
Êv ˆ Êv ˆ
v2C = Á A ˜ + Á B ˜
Ë 2¯ Ë 2¯
or vC2 = 4.52w2
In this process the centre will rise = y/2 = 0.97/2 = 0.485 m
Using work-energy theorem, we have
Work done by the force = increase in PE of the rod + gain in translational KE + gain in
rotational KE
1 1
or P ¥ 1.5 = Mg(0.485) + MvC2 + Iw 2
2 2
ML2 18 3 M
where I = = M¥ =
12 12 2
Substituting the all known values in the above equation, we get
1/2
Ê 0.498 P ˆ
w = Á - 0.16˜ Ans.
Ë Mg ¯

Problem 3.182 A uniform thin rod of mass m and length l is standing on a smooth
horizontal surface. A slight disturbance causes the lower end to slip on the smooth surface
and the rod starts falling. Find the velocity of the centre of mass of the rod at the instant
when it makes an angle q with horizontal.
558പWŚLJƐŝĐƐĨŽƌ//dͲ:͗DĞĐŚĂŶŝĐƐ//

Solution As the floor is smooth, mechanical A


energy of the rod will remain conserved.
Further, no horizontal force acts on the rod, CM
1
hence the centre of mass moves vertically h = (1 - sin q )
IC 2
downwards in a straight line. Thus, velocities r w
vC
of CM and the lower end B are in the 1
directions shown in Fig. 3.358. The location sin q
2
of IC at this instant can be found by drawing q
 
perpendiculars to v C and v B at respective B r
vB
points. Now, the rod may be assumed to be
in pure rotational motion about IAOR passing Fig. 3.358
through IC with angular speed w.
Applying conservation of mechanical energy. Decrease in gravitational potential energy
of the rod = increase in rotational kinetic energy about IAOR
1
\ mgh = I IAORw 2
2
l 1 Ê ml 2 ml 2 ˆ
or mg (1 - sin q ) = Á + cos 2 q ˜ w 2
2 2 Ë 12 4 ¯
12 g(1 - sin q )
Solving this equation, we get w =
l(1 + 3 cos 2 q )
 Êl ˆ
Now, |v C| = Á cos q ˜ w
Ë2 ¯

3 gl(1 - sin q )cos 2 q


=
(1 + 3 cos 2 q )
1
k = IIAOR w2
2
1 2 1
= mvcom + I comw 2
2 2
l
vc = pw = cos q w
2
l2 1 1 ml 2
vc2 = cos q w 2 fi
2
mv 2c = cos 2 q w 2
4 2 2 4
1 2 1 ml 2 2
Iw = w
2 2 12

Problem 3.183 A rigid rod of length l and mass M rest on a


frictionless table with one end in contact with a vertical peg
v
(Fig. 3.359). A small mass m travelling with a uniform speed of
m
v hits the other end of the rod perpendicularly. Calculate (a) the l
angular velocity of the rod and the velocity of the mass m after
Fig. 3.359
collision; and (b) the impulsive reaction of the peg (assume an
elastic collision).
ZŽƚĂƟŽŶപ559

Solution After collision, the rod will rotate about the peg, and a reaction force is developed
on the peg (which is not known). The torque of the reaction force about the peg is zero, so
angular momentum of (rod + m) about the peg remain constant.
(a) Suppose v¢ is the velocity of mass m after collision and w be the angular velocity of
the rod.
The angular momentum and kinetic energy conservation gives
mvl = mv¢l + Iw
Ê Ml 2 ˆ
or mvl = mv¢l + Á ˜w ...(i)
Ë 3 ¯
1 1 1
and mv 2 = mv ¢ 2 + Iw 2
2 2 2
1 1 1 Ê Ml 2 ˆ 2
or mv 2 = mv ¢ 2 + Á w ...(ii)
2 2 2 Ë 3 ˜¯
Solving Eqs. (i) and (ii), we get
6 mv
w = (Proof on next page)
l( M + 3m)
( M - 3 m)
v¢ = – v Ans.
( M + 3 m)
(b) Let impulse of the force exerted by mass m be J, then
 
tDt = DL F1 F2
or (F1l)Dt = DL
or (F1Dt)l = DL l/2 l/2
or J1l = DL
Fig. 3.360
DL Iw - 0
\ J1 = =
l l

Ê Ml 2 ˆ
Á 3 w - 0˜
Ë ¯ Mlw
= =
l 3

Let the impulse of the force due to the peg reaction be J2. The change in the angular
momentum about an axis perpendicular to the rod passing through its centre of mass
is 

tDt = DL
Ê l lˆ
or ÁË F1 - F2 ˜¯ Dt = DL
2 2
l l
or J1 - J 2 = ICM w
2 2
560പWŚLJƐŝĐƐĨŽƌ//dͲ:͗DĞĐŚĂŶŝĐƐ//

Ml 2
= w
12
Mlw
or J2 = J1 –
6
Mlw Mlw
= -
3 6
Mlw
= Ans.
6
2
Ml
mvl = mv¢l + w …(i)
3
Ml 2
mvl – w = mv¢l
3
Ê M ˆ
l Á mv - lw ˜ = mv ¢l
Ë 3 ¯
Ê M ˆ
mv¢ = Á mv - lw ˜
Ë 3 ¯
M
mv - lw
v¢ = 3
m
M
v¢ = v – lw
3m
From Eq. (ii)
1 1 1 Ê Ml 2 ˆ 2
mv 2 = mv ¢ 2 + Á w
2 2 2 Ë 3 ˜¯
2
1 1 Ê M ˆ 1
mv 2 = m Á v - lw ˜ + Ml 2w 2
2 2 Ë 3m ¯ 6
1
Divide by m on both sides.
2 1 2 2
2 Ml w
Ê M ˆ 6
v = Áv -
2
lw ˜ +
Ë 3m ¯ 1
m
2
2
Ê M ˆ 1M 2 2
v2 = Á v - lw ˜ + lw
Ë 3m ¯ 3m

2M M2 2 2 1 M 2 2
v2 = v2 - lw v + lw + lw
3m 9m 2 3m
2M M2 2 2 1 M 2 2
lw v = lw + lw
3m 9m 2 3m
ZŽƚĂƟŽŶപ561

1M 1 M Ê l 2w 2 M ˆ 1 M 2
(2lw v) = + lw
3m 3 m ÁË 3m ˜¯ 3 m
l 2w 2 M 2 2
2lwv = +l w
3m
Ê lw M ˆ
lw (2v) = lw Á + lw ˜
Ë 3m ¯
lw M
2v = + lw
3m
ÊM ˆ
2v = lw Á +1
Ë 3m ˜¯

Ê M + 3m ˆ
2v = lw Á
Ë 3m ˜¯
6mv = lw (M + 3m)
6 mv
lw =
( M + 3 m)
6 mv
w =
l( M + 3m)

Problem 3.184 A rod of length L and mass M is hinged at point O. O


A small bullet of mass m hits the rod with velocity v, as shown in
Fig. 3.361. The bullet gets embedded in the rod. Find the angular velocity
of the system just after the impact. (IIT-JEE, 2005) M L

Solution Before impact, the bullet is moving. The initial angular v


momentum of the system about O is m
Li = mv ¥ L = mvL
After the bullet gets embedded in the rod, suppose the system attains Fig. 3.361
angular velocity w.
The moment of inertia of the bullet + rod system about the axis through O is
I = (MI of bullet + MI of rod) about the axis through O
1 M + 3m 2
= mL2 + ML2 = L
3 3
Final angular momentum of the system is
M + 3m 2
Lf = Iw = Lw
3
By conservation of angular momentum,
M + 3m 2
or L w = mvL
3
3mv
or w =
( M + 3m)L
562പWŚLJƐŝĐƐĨŽƌ//dͲ:͗DĞĐŚĂŶŝĐƐ//

Problem 3.185 A uniform rod of length L lies on a smooth horizontal table. The rod has a
mass M. A particle of mass m moving with speed v strikes the rod perpendicularly at one
of the ends of the rod and sticks to it after collision.
1. Find the velocity of the centre of mass C of the system constituting “the rod plus the
particle”.
2Mv 2mv
(a) (b)
M-m M+m
Mv mv
(c) (d)
M+m M+m
2. Find the velocity of the particle with respect to C before the collision.
Mv mv
(a) (b)
M+m M+m
2mv 2Mv
(c) (d)
M+m M+m
3. Find the velocity of the rod with respect to C before the collision.
Mv mv
(a) (b)
M+m M+m
2Mv 2mv
(c) (d)
m+ M m+ M
4. Find the angular momentum of the particle and of the rod about the centre of mass C
before the collision.
M 2 mv l m2 Mv l M 2 mv l Mm2vl
(a) , (b) ,
4(m + M )2 4(m + M )2 2( M + m)2 2( M + m)2
M 2 mvl m2 Mvl 2 M 2vml 2m2 Mvl
(c) , (d) ,
( m + M ) ( m + M )2
2
( m + M )2 ( M + m)2
5. Find the velocity of the centre of mass C and the angular velocity of the system about
the centre of mass after the collision.
2mv 3 Mv Mv 6 Mv
(a) , (b) ,
M + m ( M + 2m)L M + m ( M + 4 m)L
2 Mv 3mv mv 6 mv
(c) , (d) ,
M + m ( M + 4 m)L M + m ( M + 4 m)L
Solution Velocity of the centre of mass of the ‘rod + particle’ system.
mv
VCM =
( M + m)
  
v p,CM = v p - v c
mv
vc = and vp = v
M+m
ZŽƚĂƟŽŶപ563

 Mv m V A
v p,CM =
M+m
r1
   mv
v rod, CM = v rod - v CM = 0 - l/2
M+m C

mv
vrod, CM = –
M+m M O

Location of the centre of mass of the system ‘rod + particle’


from A B
m ¥ 0 + Ml/2
r1 = Fig. 3.362
(m + M )
Ml
=
2(m + M )
Angular momentum of the particle just before collision about C,
Lpc = m vpc r1

M 2 mvl
=
2( M + m)2
Angular momentum of the rod about the centre of mass of system C,
Êl ˆ
Lrod, CM = Mvrod, CM Á - r1 ˜
Ë2 ¯

Ê mv ˆ Ê ml ˆ
= MÁ
Ë M + m ˜¯ ÁË 2(m + M ) ˜¯

Mm2vl
=
2( M + m)2
Moment of inertia about the vertical axis passing through C,
2
Êl ˆ
Ic, rod = I0 + M Á - r1 ˜
Ë2 ¯
2
Ml 2 Ê ml ˆ
Ic, rod = + MÁ
12 Ë 2( M + m) ˜¯
2
Ê Ml ˆ
Ic, particle = mr12 = mÁ
Ë 2( M + m) ˜¯
2 2
Ml 2 Ê Ml ˆ Ê Ml ˆ
Ic = + MÁ ˜ + mÁ
12 Ë 2( M + m) ¯ Ë 2( M + m) ˜¯
M ( M + 4 m) 2
Ic = L
12( M + m)
564പWŚLJƐŝĐƐĨŽƌ//dͲ:͗DĞĐŚĂŶŝĐƐ//

As no external forces are acting on the system ‘rod + particle’, hence the velocity of the
centre of mass of the system will remain constant.
mv
vCM =
M+m
For angular velocity about C,
mvr1 = Icw
Ê ml ˆ
mv Á
mvr1 Ë 2( M + m) ˜¯ 6 mv
w = = =
Ic Ê M( M + 4 m)L ˆ
2 ( M + 4 m)L
Á 12( M + m) ˜
Ë ¯

Problem 3.186 A boy of mass m runs on ice with velocity v0 and steps M
on the end of a plank of length l and mass M which is perpendicular
to his path.
(a) Describe quantitatively the motion of the system after the boy l
is on the plank. Neglect friction with the ice.
(b) One point on the plank is at rest immediately after the collision.
Where is it? m
v0
Solution C is the CM of (M + m)
Fig. 3.363
Ê M ˆÊ lˆ
BC = Á
Ë M + m ˜¯ ÁË 2 ˜¯ A

Ê m ˆÊ lˆ
and OC = Á
Ë M + m ˜¯ ÁË 2 ˜¯ O

From conservation of linear momentum,


C v
(M + m)v = mv0 w

Ê m ˆ B
or v = Á v …(i)
Ë M + m ˜¯ 0
Fig. 3.364
From conservation of angular momentum about point C we have,
mv0 (BC) = Iw
È 2 2 2 ˘
Ê M ˆ Êl ˆ Ê m ˆ Ê l ˆ˙
2 2
mMv0 l Ml 2
or Í
= mÁ ˜ + + M Á ˜ w
2( M + m) Í Ë M + m ¯ ÁË 4 ˜¯ 12 Ë M + m ¯ ÁË 4 ˜¯ ˙
Î ˚
mv0
Putting = v from Eq. (i) we have
M+m
v l È 4m + M ˘
= Í
w 6 Î M + m ˙˚
v
Now, a point (say P) at a distance x = , from C (towards O) will be at rest. Hence, distance
of point P from boy at B will be w
ZŽƚĂƟŽŶപ565

BP = BC + x
Ê M ˆ Ê l ˆ l È 4 m + M ˘ 2l
= Á + = Ans.
Ë M + m ˜¯ ÁË 2 ˜¯ 6 ÍÎ M + m ˙˚ 3

Problem 3.187 A stick of length L and mass M lies on a frictionless horizontal surface on
which it is free to move in anyway. A ball of mass m moving with speed v as shown in Fig.
3.365 collides elastically with the stick. What must be the mass of the ball so that it remains
at rest immediately after collision.
Solution
(a) (i) As external force is zero so linear momentum is conserved. (ii) As M
torque is zero so angular momentum is conserved. (iii) As collision is
elastic mechanical energy is also conserved. (KE after collision is equal
to KE before collision).
C
(b) By conservation of linear momentum
d
mv = m ¥ 0 + MV ...(i)
v
By conservation of angular momentum m
mvd = Iw ...(ii)
Fig. 3.365
By conservation of mechanical energy
1 1 1
mv 2 = MV 2 + Iw 2 ...(iii)
2 2 2
Substituting the value of V and w from Eqs. (i) and (ii) respectively in
(iii)
2 2 V
1 1 È mv ˘ 1 È mvd ˘ w
mv 2 = M Í + I
2 2 Î M ˙˚ 2 ÍÎ I ˙˚
m md 2 È ML2 ˘ m
or 1 = + Ías I = ˙
M ( ML2/12) Î 12 ˚
Fig. 3.366
ML2
or m = 2
(L + 12d 2 )

Problem 3.188 Discuss the quantity which remains conserved in the collision between a
bullet and a bar in the following situations. Find the axis of rotation and velocity of the bar
just after collision. In each case the rod is placed on a smooth horizontal surface. The mass
and length of the rod are m and L respectively. (IIT-JEE, 1972)
Solution
(a) Bar is hinged at its one end and the bullet sticks to it after collision.
Here rod will rotate about the hinged end. As soon as the bullet hits the rod, a force R
is exerted by the hinge on A. Its moment of force about an axis passing through hinge
is zero. So in this case:
(i) Linear momentum does not remain conserved.
566പWŚLJƐŝĐƐĨŽƌ//dͲ:͗DĞĐŚĂŶŝĐƐ//

(ii) As collision is completely inelastic, so KE does not remain hinge


conserved. A R
(iii) Angular momentum about hinge remain constant, therefore
w
mv0L = Iw
mv0 L
or w =
I
È mL2 ˘
Here I = Í + mL2 ˙ m v0
Î 3 ˚ B

1 1 Fig. 3.367
Loss in KE = mv02 - Iw 2
2 2
(b) Bar is free to move in any way and the bullet sticks to it after collision.
Here no external force in the direction of motion and no torque along axis of rotation
are acting on the system. So in this case:
(i) Linear momentum remains conserved. Thus, A

mv0 = (m + m)v
v
or v = 0 L/4
2
w
(ii) As collision is completely inelastic, so KE does v L/4
not remain conserved.
(iii) Angular momentum about an axis passing m v0
through CM of the system remains constant. B
CM of the system
The position of CM of system from the centre of (rod + bullet)
bar. (a) (b)

m ¥ 0 + m ¥ L/2 L Fig. 3.368


yc = =
m+m 4
Thus, we have
L
mvo ¥ = Iw
4
mvo L
\ w =
4I
È ML2 Ê Lˆ ˘
2 2
Ê Lˆ
Here, I = Í + M Á ˜ ˙ + mÁ ˜
ÍÎ 12 Ë 4¯ ˙ Ë 4¯
˚
1 È1 1 ˘
Loss in KE = mv02 - Í (2m)v 2 + Iw 2 ˙
2 Î2 2 ˚
(c) Bar is free to move in any way, but collision between the bullet and the rod is
completely elastic and let bullet stop after collision. In this case:
(i) Linear momentum remains conserved, so we can write
mv0 + 0 = 0 + mv
fi v = v0
ZŽƚĂƟŽŶപ567

(ii) As collision is elastic so KE remains constant. We can


write
1 1 1
mv02 = mv 2 + Iw 2 w
2 2 2 v
(iii) By conservation of angular momentum, we have
L
mv0 = Iw m v0
2
mv0 L
or w = Fig. 3.369
2I
ML2
Here I =
12

Problem 3.189 A scale AB of mass M and length L is lying on a horizontal frictionless


surface. A particle of mass m travelling along the surface hits the end A of the scale with a
velocity v0 in a direction perpendicular to AB. The collision is completely elastic. After the
collision the particle comes to rest.
m
(a) Find the ratio
M
(b) A point P on the scale is at rest immediately after the collision. Find the distance
AP.
pL
(c) Find the linear speed of a point P at time after the collision.
3v0
Solution
(a) Let vc and w are the velocity of CM and angular velocity just after collision. Using
conservation of linear momentum, we have
mv0 = 0 + Mvc ...(i)
and by conservation of angular momentum
L
mv0 = Iw
2
L Ê ML2 ˆ
or mv0 = w ...(ii)
2 ÁË 12 ˜¯
From Eqs. (i) and (ii)
mv0 6 mv0
vc = and w =
M ML
Since collision is completely elastic, therefore KE before collision is equal to after
collision
1 1 1
or mv02 + 0 = Mvc2 + Iw 2
2 2 2
568പWŚLJƐŝĐƐĨŽƌ//dͲ:͗DĞĐŚĂŶŝĐƐ//

1 Ê ML2 ˆ Ê 6mv0 ˆ
2
1 1 Ê mv ˆ
or mv02 = M Á 0 ˜ vc2 + Á Á ˜
2 2 Ë M ¯ 2 Ë 12 ˜¯ Ë ML ¯
m 1 v
which gives = and vc = 0 Ans.
M 4 4
(b)

A
v0
m

vy
vC
vc vc
w w L/6 P L/6
y
wy vc
P P

B
(a) (b) (c)

Fig. 3.370

Velocity of point P immediately after collision is zero, let it be at a distance y from CM


\ vc – wy = 0
mv0 Ê 6 mv0 ˆ
or -Á ˜y =0
M Ë ML ¯
L
which gives y =
6

L L 2L
\ AP = + =
2 6 3
pL
(c) Angle rotated by rod in time
3v0
6 mv0 p L
q = wt = ¥
ML 3v0
p
= Ans.
2
p
The rod turns through , in this interval of time. The velocity of point P in y-direction
will be 2
6 mv0 L
vy = w y = ¥
ML 6
v
= 0
4
ZŽƚĂƟŽŶപ569

The resultant velocity of point P

v = vc2 + vy2

2 2
Ê v0 ˆ Ê v0 ˆ
= ÁË ˜¯ + ÁË ˜¯
4 4
v0
= Ans.
2 2
Problem 3.190 A homogeneous bar AB of length l = 1.8 m and mass M is pivoted at the
centre O in such a way that it can rotate freely in the vertical plane. The bar is initially in
the horizontal position. An insect S of the same mass M falls vertically with speed v on the
point C, midway between the points O and B. Immediately after falling, the insect moves
towards the end B such that the bar rotates with a constant angular velocity w.
(a) Determine the angular velocity w in terms of v and L.
(b) If the insect reaches the end B when the bar has turned through an angle of 90°,
determine v. (IIT-JEE, 1992)
Solution

Insect

v
O
A C B A B
L/2 L/4 L/4 x

Mg

(a) (b)

Fig. 3.371

(a) In the process the angular momentum of the system remains constant.
L
Mv ¥ + 0 = Iw ...(i)
4
where I is the moment of inertia of the system which is
2
ML2 Ê Lˆ 7 ML2
+ MÁ ˜ =
12 Ë 4¯ 48
570പWŚLJƐŝĐƐĨŽƌ//dͲ:͗DĞĐŚĂŶŝĐƐ//

Substituting this value in the above equation, we get


12v
w = ...(ii)
7L
Ans.
(b) Let the insect be at a distance x from the centre of the rod after time t, the moment of
ML2
inertia of the system will be = + Mx2, and therefore angular momentum at this
12
Ê ML2 ˆ
instant Á + Mx 2 ˜ w .
Ë 12 ¯
Torque exerted by the weight of the insect at that moment t = mgx.
dL
Since we have =t
dt
d Ê ML2 ˆ
Therefore, Á + Mx 2 ˜ w = Mgx ...(iii)
dt Ë 12 ¯
dx
or Mw 2x = Mgx
dt
dx g
or =v= ...(iv)
dt 2w
p (p/2)
The time taken by the rod to rotate through rad = , it is given that the rod
2 w
rotates with constant angular velocity. The time taken by the insect to reach the end
B = (L/4)v.
It is given that both times are equal, therefore
(p/2) (L/4)
=
w v
(p/2) (L/4)
or =
w ( g/2w )
After solving the above equation, we get
w = (p g/L)
Substituting this value in Eq. (ii), we get
7w L pg L
v = =7 ¥ where L = 1.8 m
12 L 12
= 4.4 m/s Ans.
Problem 3.191 A uniform bar of length 6a and mass 8 m lies on a smooth horizontal table.
Two point masses m and 2 m moving in the same horizontal plane with speed 2v and v
respectively strike the bar and stick to the bar after collision. Calculate:
ZŽƚĂƟŽŶപ571

(a) the velocity of the centre of mass after collision,


(b) angular velocity of rotation about the centre of mass, and
(c) total energy.
Solution
(a) In the process of collision the linear momentum remains constant
2mv – m2v + 0 = (2m + m + 8m)vcm
or vcm = 0 Ans.
(b) Angular momentum of the system also remains constant
\ 2mv ◊a + (2mv)◊2a + 0 = Iw ...(i)
where I is the moment of inertia of the system after collision.
I = MI of the rod + MI of the mass 2m + MI of the mass m
(8 m)(6 a)2 2m
or I = + 2 m ◊ a 2 + m ◊ ( 2 a )2 v a
12
3a 2a 2v
= 30 ma2 m
Substituting this value in Eq. (i), we get Fig. 3.372
w = v/5a Ans.
(c) Total energy after collision
1 2 1
E = Iw = 30 ma 2 ¥ (v/5 a)2
2 2
or E = 3mv2/5 Ans.
Problem 3.192 A uniform bar of length 6a and mass 8m 2m
lies on a smooth horizontal table. Two point masses m and
v
2m moving in the same horizontal plane with speeds 2v
C
and v respectively, strike the bar (as shown in Fig. 3.373 8m
and stick to the bar after collision. Calculate (a) velocity of
2a a 2a a
the centre of mass, (b) angular velocity about the centre of 2v
mass, the and (c) total kinetic energy, just after collision.
(IIT-JEE, 1991) m
Solution
Fig. 3.373
(a) As Fext = 0 linear momentum of the system is con-
served, i.e.,
– 2m ¥ v + m ¥ 2v + 0 = (2m + m + 8m) ¥ V
or V = 0, i.e., velocity of the centre of mass is zero.
(b) As text = 0 angular momentum of the system is conserved, i.e.,
m1v1r1 + m2v2r2 = (I1 + I2 + IB)w
2mva + m(2v)(2a) = [2m(a)2 + m(2a)2 + 8m ¥ (6a)2/12]w
i.e., 6mva = 30ma2w or w = (v/5a)
(c) As from part (1) and (2) it is clear that the system has no translatory motion but only
rotatory motion,
572പWŚLJƐŝĐƐĨŽƌ//dͲ:͗DĞĐŚĂŶŝĐƐ//

2
1 2 1 Èv˘ 3
E= Iw = (30 ma 2 ) Í ˙ = mv 2
2 2 Î 5a ˚ 5
Note: As initial KE of the system was
1 1
Ei = (2m)v 2 + m(2v)2 + 0 = 3mv 2
2 2
But as Ef = (3/5) mv2, there is loss of KE (= 2.4 mv2)
So in this problem mechanical energy is not conserved.
Problem 3.193 A thin uniform scale of length L is initially at rest with respect to an inertial
frame of reference. The scale is tapped at one end perpendicular to its length. How far the
centre of mass translates while the scale completes one revolution about its centre of mass.
Neglect gravitational effect.
Solution The impulse delivered perpendicular to the rod at one end gives some linear
momentum to the CM (centre of mass) of the rod and also some angular momentum about
the CM. The rod will rotate about CM. Free rotation of any rigid body always takes place
about CM.
Let Dt be the duration of impact, then F
 
FDt = DP
or FDt = (Mvc – 0) ...(i)
w
Also angular impulse is equal to the change in angular mo- L vc
mentum
Ê Fl ˆ
or ÁË ˜¯ Dt = IDw
2
= I(w – 0) ...(ii)
Dividing Eq. (i) by Eq. (ii), we get Fig. 3.374
vc l
=
w 6
Let t be the time in which rod rotates an angle 2p, then
vc t l
¥ =
w t 6
xc l
or =
q 6
xc l lp
or = fi xc = Ans.
2p 6 3

Problem 3.194 Two 2 kg balls are attached to the ends of a thin rod of negligible mass, 50
cm long. The rod is free to rotate in vertical plane without friction about a horizontal axis
through its centre. While the rod is horizontal, a 50 g putty wad was dropped onto one of
the balls with a speed of 3.0 m/s and sticks to it.
ZŽƚĂƟŽŶപ573

(a) What is the angular speed of the system just after the putty wad hits?
(b) What is the ratio of the kinetic energy of the entire system after the collision to that of
putty wad just before?
(c) Through what angle the system will rotate until momentarily stops?
Solution
(a) Let M be the mass of each ball and m be the mass of the putty wad given:
M = 2 kg m = 0.05 kg, Length of the rod, L = 0.05 m
Using the principle of conservation of angular momentum, we have
Ê Lˆ
mv Á ˜ + 0 = Iw ...(i) Putty wad
Ë 2¯
2 2 2
Ê Lˆ Ê Lˆ Ê Lˆ Rotational axis
where I = M Á ˜ + M Á ˜ + mÁ ˜
Ë 2¯ Ë 2¯ Ë 2¯
2
Ê Lˆ
= (2M + m) Á ˜
Ë 2¯ Fig. 3.375
Substituting this value in above equation, we get

Ê mvL ˆ 2mv
w = Á I=
Ë 2 ˜¯ (2 M + m)L
= 0.148 rad/s Ans.
1
(b) The initial KE = mv 2
2
1 2
Find KE = Iw
2
1 2 q 90°
Iw
Their ratio f = 2
1 L
mv 2 h= (1 - cos q )
2 2
A
After substituting the values of L and w, we get
Fig. 3.376
m
f = 0.0123 Ans.
( 2 M + m)
(c) As the rod rotates, its mechanical energy remains constant. If one on the balls is
lowered a distance d the other is raised the same distance, and the sum of the PE of the
balls does not change. Now using the principle of conservation of mechanical energy,
we have mechanical energy at A = mechanical energy at B
L 1 2 L
or mg + Iw = mg (1 - cos q )
2 2 2
2 2
L 1 Ê L ˆ È 2mv ˘ L
or mg + ( 2 M + m) Á ˜ Í = mg (1 - cos q )
2 2 Ë 2 ¯ Î 2 M + m ˙˚ 2
574പWŚLJƐŝĐƐĨŽƌ//dͲ:͗DĞĐŚĂŶŝĐƐ//

After substituting the all known values in above equation, we get


q = 92.6°
The total angle of the swing is = 90° + 92.6° = 182.6° Ans.
Problem 3.195 Two uniform thin rods P
A and B, of length 0.6 m each and of A 0.01 kg
masses 0.01 kg and 0.02 kg respectively,
L = 0.6 m ym
are rigidly joined, end to end. The com- A
bination is pivoted at the lighter end P
as shown in Fig. 3.377 such that it can
freely rotate about the point P in a ver- B cm of the system
tical plane. A small object of mass 0.05 L = 0.6 m
B
kg moving horizontally hits the lower 0.02 kg
end of the combination and sticks to y
m v
it. What should be the velocity of the
m = 0.05 kg
object so that the system could just be (a) (b)
raised to the horizontal position?
(JEE, 1994) Fig. 3.377

Solution: As initially torque about P is zero, angular momentum of the system is conserved,
i.e.,
mv ¥ 2L = Iw ...(i)
But here I = I P + IA + IB
È L2 Ê L ˆ ˘
2
L2
i.e., I = mP (LA + LB)2 + MB Í B + Á B + LA ˜ ˙ + M A A
ÍÎ 12 Ë 2 ¯ ˙
˚ 3

È 0.6 2 ˘ (0.6)2
i.e., I = 0.05(1.2)2 + 0.02 Í + (0.3 + 0.6)2 ˙ + 0.01
Î 12 ˚ 3

i.e., I = 0.0720 + 0.0168 + 0.0012 = 0.09 kg-m2 ...(ii)


So, Eq. (i) becomes
0.05 ¥ v ¥ 1.2 = 0.09 ¥ w,
i.e., w = (2/3)v ...(iii)
Now due to angular velocity w the system will rotate and kinetic energy of rotation will
be converted into PE. So gain in PE when the system becomes horizontal,
È L ˘ L
DU = mPghP + MBg ÍLA + B ˙ + M A g A
Î 2 ˚ 2
i.e., DU = 0.05 ¥ 1.2g + 0.02 ¥ 0.9g + 0.01 ¥ 0.3g
= 0.081 ¥ 9.8 J
As this energy is provided by rotational KE, by conservation of mechanical energy (after
collision), we have
1 2
Iw = DU = 0.081 ¥ 9.8 J
2
ZŽƚĂƟŽŶപ575

Substituting the values of I and w from Eqs. (ii) and (iii) in the above, we have
2
1 È2 ˘
¥ 0.09 ¥ Í v ˙ = 0.081 ¥ 9.8
2 Î3 ˚
or v2 = 8.1 ¥ 4.9, i.e., v = 6.3 m/s
Problem 3.196 A scale is resting on a frictionless horizontal surface. The centre of the scale
is fixed. The mass of the scale is M and length is L. A small particle of mass m strikes the
end with a velocity v in the direction perpendicular to the scale. Calculate the final angular
velocity of the scale. (BITSAT, 2014)
Solution Step 1: Initial angular momentum:
v
mvL w
Of particle =
2
Of scale = 0 since it is at rest.
mvL
fi Li =
2
Step 2: Final angular momentum:
Ê wLˆ Ê Lˆ 1 2
Of particle = m Á = mL w
Ë 2 ˜¯ ÁË 2 ˜¯ 4
1
Of scale = ML2w
12
1 1 1
fi Lf = mL2w + ML2w = w L2 ( M + 3m)
4 12 12 Initial Final
situation situation
The principle of conservation of angular mo-
mentum gives us Fig. 3.378
mvL 1
Li = Lf fi = wL2 ( M + 3m)
2 12
v Ê 6m ˆ
fi 6mvL = wL2[M + 3m] fi w = Á ˜
L Ë M + 3m ¯
v
Problem 3.197 A scale is resting on a frictionless horizontal w
surface. The mass of the scale is M and length is L. A small
particle of mass m strikes the end with a velocity v in the
direction perpendicular to the scale and gets stuck to it.
Calculate the final angular velocity of the scale.
vf
Solution Step 1: Initial situation: The particle at a velocity
is about to strike the scale. The scale is at rest.
Final situation: The particle is embedded in the scale at the
end. The scale moving on the surface with its centre at a
velocity vf and an angular velocity w.
Initial Final
situation situation
Fig. 3.379
576പWŚLJƐŝĐƐĨŽƌ//dͲ:͗DĞĐŚĂŶŝĐƐ//

Step 2: Initial angular momentum:


Of particle = 0 since lever arm = 0
Of scale = 0 since it is at rest.
fi Li = 0
Step 3: Final angular momentum:
Of particle = 0 since lever arm = 0
1
Of scale = – Mvf R + MR2w
12
1 1
fi Lf = – Mv f L + ML2w
2 12
The principle of conservation of angular momentum gives us
1 1 Lw
Li = Lf fi 0=– Mv f L + ML2w fi vf =
2 12 6
vf = vCM of the system – w ¥ (distance of CM from centre of the scale)
The conservation of linear momentum gives:
È m ˘
mvm = vCM (M + m) fi vcm = Í vm
Î m + M ˙˚

È m ˘ 1 È m ˘ Lw
fi vf = Í v - wL Í =
Î m + M ˙˚ 2 Î m + M ˙˚ 6

v È 6m ˘
(M + m) Lw = 6mv – 3mLw fi w=
L ÍÎ M + 4 m ˙˚

Problem 3.198 A uniform rod of length 2a is placed horizontally on the edge of a table.
Initially, the centre of mass of the rod is at a distance a/3 from the edge. The rod is released
from rest. If the rod slips after it has turned through an angle q, find the coefficient of friction
between the rod and the table.
Solution Initially, the rod rotates about the edge.
From the principle of conservation of energy
KEi + GPEi = KEf + GPEf
1 2 Ê a ˆ
0+0 = Iw + Á - Mg sin q ˜ ...(1)
2 Ë 3 ¯
2 2
M( 2 a) Ê aˆ
where I = + M Á ˜ from parallel axis theorem.
12 Ë 3¯
From Eq. (1),
3 g sin q
w2 = ...(2)
2a
ZŽƚĂƟŽŶപ577

mN
N
t-axis

2a q
CM
a
3 mg sin q
q
n-
mg cos q axis
mg
(a) (b)

Fig. 3.380

Differentiating the above expression for w w.r.t q, we obtain


dw 3g
= cos q
dq 4a
dw dw dq dw
Since, a = = ¥ =w
dt dq dt dq
Equations of motion along t-axis and n-axis are:
a
Sfy = may or mg cos q – N = m a
3
a Ma Ê dw ˆ Mg
Sft = Ia SFt = Mg cos q – N = M a = Áw ˜= cos q ...(3)
3 3 Ë dq ¯ 4
3 Mg
Therefore, N = cos q
4
Ê aˆ 3 g sin q
S Fn = mN – Mg sin q = M Á ˜ w 2 w2 =
Ë 3¯ 2a
Ma 3 g sin q
= ¥
3 2a

Mg sin q
=
2
3
Therefore, mN = Mg sin q
2
Ê 3 Mg ˆ 3
mÁ cos q ˜ = Mg sin q
Ë 4 ¯ 2
m = 2 tan q
578പWŚLJƐŝĐƐĨŽƌ//dͲ:͗DĞĐŚĂŶŝĐƐ//

Analytical Skill Problem


Mark the following statements as true or false:
1. Friction force on the rod immediately after release of the rod is zero.
2. Initial acceleration of CM is in vertical direction.
3. Angular acceleration of the rod is increasing as the rod rotates.
4. Normal reaction of the edge is increasing as the rod rotates.
8
5. Velocity of the right tip of the rod is ga sin q.
3
Problem 3.199 One-fourth length of a uniform rod of
mass m and length l is placed on a rough horizontal surface
and it is held stationary in horizontal position by means
l/4
of a light thread as shown in Fig. 3.381. The thread is then
burnt and the rod starts rotating about the edge. Find the
angle between the rod and the horizontal when it is about
to slide on the edge. The coefficient of friction between the
rod and the surface is m.
Solution w : Decrease in gravitational potential energy =
increase in rotational kinetic energy Fig. 3.381
l 1
\ mg sin q = I 0w 2
4 2

1 È ml 2 Ê lˆ 2
= Í + mÁ ˜ ˙w
2 ÍÎ 12 Ë 4¯ ˙˚

l/4 3l/4
È 24 g sin q ˘
\ w = Í ˙ ...(i)
Î 7l ˚ C

t l/4
a: a = O
I
l
mg cos q
= 4
È ml 2 Ê lˆ ˘
2
Í + mÁ ˜ ˙
ÍÎ 12 Ë 4¯ ˙ (a)
˚
12 g cos q w,a
= ...(ii) q
7l
SFy = may or mg cos q – N = mat O
or N = mg cos q – mat
l
= mg cos q – m a
4 mg

(b)
ZŽƚĂƟŽŶപ579

Substituting value of a from Eq. (ii), we get y

4
N = mg cos q ...(iii)
7 x mN N
Rod begins to slip when
mN – mg sin q = man q
an
4 l O
or mmg cos q – mg sin q = m w 2
7 4
Substitution value of w from Eq. (i), we get at
mg
4m
tan q =
13 (c)
Ê 4m ˆ
\ q = tan–1 ÁË ˜¯ Ans. Fig. 3.382
13

Problem 3.200 The angular velocity and angular acceleration of the


pivoted rod are given as w and a respectively. Find the acceleration of B. A

Solution Ground Frame Method:


w
Putting VA = 0, aAx = aAy = 0 y
q a

(because the rod is pivoted at A)


in the above two equations, we have
aBx = l(a cos q – w2 sin q) x
B
2
and aBy = l(a sin q + w cos q)
Fig. 3.383
Relative Motion Method:
     
a B = a BA + a A = a BAt + a BAr + a A
where aA = 0, aBAt = la
q
and aBAt = lw2 lw 2 cos q
la sin q
Then resolving the vectors in x and y directions, we have lw 2 q la
aBx = Ia cos q – lw2 sin q q
la cos q
and aBy = la sin q + lw2 cos q Ans. lw 2 sin q B

Fig. 3.384
Problem 3.201 A conical pendulum, a thin uniform rod of length L and mass M, rotate
uniformly about a vertical axis with angular velocity w (the upper end of the rod is hinged).
Find the angle q between the rod and the vertical.
Solution Choose an element of the rod of width dx at a distance x from the hinge.
m
Mass of the element, dm = dx. The centrifugal force on this element
l
dF = (dm) w2 (x sin q).
580പWŚLJƐŝĐƐĨŽƌ//dͲ:͗DĞĐŚĂŶŝĐƐ//

Its moment of force about the hinge


w
dt = dF ¥ x cos q
= (dm) w2 (x sin q) (x cos q) q x

Êm ˆ Ê sin 2q ˆ dmw 2(x sin q)


= Á dx˜ w 2 x 2 Á
Ë l ¯ Ë 2 ˜¯
/2
2
mw
= sin 2 q x 2 dx ...(i) mg
2l
For the moment of force of whole length of rod, integrat-
ing (i) Fig. 3.385
l
mw 2
t = sin 2 q Ú x 2 dx
2l 0
mw 2l 2
= sin 2q ...(ii)
6
In the rotating frame, apart from other forces the centrifugal force also acts. For rotational

equilibrium of the rod, we have S t = 0. Taking moment of all forces about the hinge and
putting their algebraic sum to zero, we get
l mw l 2
mg sin q = sin 2q
2 6
3g
or cos q = Ans.
2w 2l

Problem 3.202 Two skaters, each of mass 50 kg, approach each other along parallel paths
separated by 3 m. They have equal and opposite velocities of 10 m/s. The first skater carries
a long light pole, 3 m long, and the second skater grabs the end of it as he passes (assume
frictionless ice).
(a) Describe quantitatively the motion of the skaters after they are connected by the pole.
(b) By pulling on the pole the skaters reduce their distance to 1 m. What is their motion
then?
(c) Compare the KEs of the system in parts (a) and (b). Where does the change come from?
Solution
(a) As the net linear momentum of the sys- 2l w
m
tem (skater + pole) is zero, the centre Pole v
of mass will be at rest before and after v
C m
the collision.
m C
The skaters and the pole will rotate CM
v v
around the centre of mass (at the mid-
point of the pole). m
Applying the conservation of angular
momentum about an axis through C
and perpendicular to the plane of the
Fig. 3.386
ZŽƚĂƟŽŶപ581

Fig. 3.387,
mvl + mvl = Iw, where I = 2m(l)2
w = mvl/I = v/l
= 20/3 rad/s
(b) As the separation reduces to 2l¢ = 1 m
Iw = I¢w¢
Again applying conservation of angular momentum, we get
Iw 2ml ¢ 2w
w¢ = = = 9w = 60 rad/s
I¢ 2ml ¢ 2
Therefore, angular velocity increases.
1
KEi I ¢w ¢ 2 l ¢w ¢ 2
(c) = 2 = 2 2 =9
KE f 1 2 lw
Iw
2
The kinetic energy increases because the skaters do positive work in pulling themselves
towards the centre of the pole.

Problem 3.203 A force F = Ai + Bj is applied to a point whole radius vector relative to the
origin of co-ordinates O is equal to r = ai + bj , where a, b, A, B are constants, and i, j are
the unit vectors of the x- and y-axis. Find the moment N and the arm l of the force F relative
to the point O.
  
Solution Moment of force N = r ¥ F
= ( ai + bj) ¥ ( Ai + Bj)
= - bAk  + aBk


= (aB – bA) k
 
|r ¥ F|
Moment arm l = 
|F|
( aB - bA)
= Ans.
A2 + B2

Problem 3.204 Three forces are applied to a square plane as shown in B


F
F
the Fig. 3.387. Find the modulus, direction and the point of application of
the resultant force, if this point is taken on the side AB.
A C
Solution The given system of forces is like as shown in Fig. 3.388. Thus
Fx = F+F
D
= 2F F 2
and Fy = F–F
Fig. 3.387
= 0
582പWŚLJƐŝĐƐĨŽƌ//dͲ:͗DĞĐŚĂŶŝĐƐ//

B F
Resultant of these, R = Fx2 + Fy2
F 2F
= 2F
A C
Suppose the resultant force R passes from a point, a distance y x x
from A, then
Fx – 2Fy = 0 D F
x
fi y = F
2
Fig. 3.388
Thus, the resultant is applied at the mid-point of side BC.
Problem 3.205 The body of mass m as shown in Fig. O
3.389 slides down the frictionless surface and collides m
with the uniform vertical bar sticking to it. The bar q
pivots about O, through the angle q before momentarily M, l
coming to rest. Find q in terms of the other parameters
given in the Fig. 3.390.
Solution Velocity of the body just before striking the
bar v = (2gh) . In the process of collision the angular
momentum of the system remains constant Fig. 3.389
mvl + 0 = Iw ...(i)
O
Ml 2
where I = + ml 2
3 q
m
Substituting value of I in Eq. (i), we get y/2
M, l
mvl
w =
( Ml / 3 + ml 2 )
2 h
y
Let the bar deflect through an angle q before it
momentarily stops. The rotational KE of the bar + body is
Fig. 3.390
used to raise the body as well as to raise the c.g. of the bar.
1 2 Mgy
\ Iw = mgy + ...(ii)
2 2
where y = l(1 – cos q).
Solve the above equations to get the value of q.
Problem 3.206 A uniform solid cylinder of mass m rests on two horizontal planks. A thread
is wound on the cylinder. The hanging end of the thread is pulled vertically down with a
constant force F as shown in Fig. 3.391. Find the maximum magnitude of the force F which
still does not bring about any sliding of the cylinder, if the coefficient of friction between the
cylinder and the planks is equal to m. What is the acceleration amax of the axis of the cylinder
rolling down the inclined plane.
ZŽƚĂƟŽŶപ583

N2

(F2 = mN2)
N1
F2

mg

F2
(F1 = mN1)
F
(a) F (b)

Fig. 3.391

Solution The different forces are shown in Fig. 3.391. Here, we have
N1 + N2 = mg + F ...(1)
1 a
F ◊ R – (F1 + F2) R = Ia = mR2 ¥ ...(2)
2 R
and F1 + F2 = ma ...(3)
From Eq. (2),
m
F – (F1 + F2) = a
2
1
or F – (F1 + F2) =[F1 + F2]
2
2
or (F1 + F2) = F
3
Further F1 = mN1 and F2 = mN2
The condition for not sliding is
F1 + F2 £ (mN1 + m N2)
2
\ F £ m(N1 + N2) £ m (mg + F)
3
2
or F – mF £ mmg
3
( 2 - 3 m )F 3 m mg
or £ mmg or F £
3 (2 - 3m )
3 m mg
\ Fmax =
(2 - 3m )
m ( N1 + N 2 ) m
From Eq. (3), a = = [mg + F]
m m
m
amax = [mg + Fmax]
m
mÈ 3m m g ˘
= Ímg + ˙
mÎ (2 - 3 m ) ˚
584പWŚLJƐŝĐƐĨŽƌ//dͲ:͗DĞĐŚĂŶŝĐƐ//

È (2 - 3m ) + 3m ˘
= mg Í ˙
Î (2 - 3 m ) ˚
È 2m g ˘
= Í ˙
Î2 - 3 m ˚

Problem 3.207 A block of mass M is moving with a v1 v2


velocity v1 on a frictionless surface as shown in Fig. 3.392. M
It passes over to a cylinder of radius R and moment of
inertia I which has a fixed axis and initially at rest. When
it first makes contact with the cylinder, it slips on the
cylinder, but the friction is large enough so that slipping
Cylinder
ceases before it loses contact with the cylinder. Find if it
goes to the dotted position with velocity v2. Compute v2 in Fig. 3.392
terms of v1, M, I and R.
Solution By conservation of angular momentum, we have
Mv1R = Iw + Mv2R
MR2
where I =
2
v2
and w =
R
v
Thus, Mv1R = I ¥ 2 + Mv2 R
R
v1
\ v2 = Ans.
È I ˘
ÍÎ1 + MR2 ˙˚

Problem 3.208 A thread reel with a thread wound on it


is placed on an inclined smooth plane set at an angle a =
30° to the horizontal. The free end of the thread is attached
to the wall as shown in 3.393. The mass of the reel is m = T
300 g, its moment of inertia relative to its own axis I = 0.45
mg
g-m2, the radius of the would thread layer r = 3.0 cm. Find a sin
a
the acceleration of the reel axis.
Solution For translational motion of the cylinder, we have Fig. 3.393
mg sin a – T = macm ...(i)
and for rotational motion
Êa ˆ
Tr = I Á cm ˜ ...(ii)
Ë R ¯
On solving above equations, we get
g sin a
ain = Ans.
È I ˘
ÍÎ1 + MR2 ˙˚
ZŽƚĂƟŽŶപ585

Problem 3.209 A thread reel with thread wound on it, of mass m,


rests on a rough horizontal surface. Its moment of inertia relative R F
to its own axis is equal to I = gmR2, where g is a numerical factor r
a
and R is the outside radius of the spool. The radius of the wound
thread layer is equal to r. The spool is pulled without sliding by
the thread with a constant force F directed at an angle a to the
horizontal as shown in the Fig. 3.394. Find
(a) the projection of the acceleration vector of the spool axis on Fig. 3.394
the x-axis; and
(b) the work performed by the force F during the first t seconds after the beginning of
motion.
Solution If ax be the acceleration of the spool along x-axis, then
F cos a – f = max ...(i)
and fR – Fr = Ia ...(ii)
a
where a = x
R
and I = g mR2
(a) After simplifying the above equations, we get
R r F
F(cos a - r/R) a
ax =
m(1 + g ) f

(b) The displacement of the spool in t second Fig. 3.395

1 2
s =
ax t
2
Thus, work done W = Fs cos a

F 2t 2 (cos a - r/R)2
= Ans.
2m(1 + g )
Problem 3.210 At the bottom edge of a smooth wall, an inclined plane
is kept at an angle of 45°. A uniform ladder of length l and mass M rests A
on the inclined plane against the wall such that it is perpendicular to the l
incline.
(a) If the plane is also smooth, which way will the ladder slide? B
(b) What is the minimum coefficient of friction necessary so that the
ladder does not slip on the incline. 45°
O
Solution
Fig. 3.396
(i) The ladder has a tendency to slip by rotating clockwise about the
point A.
586പWŚLJƐŝĐƐĨŽƌ//dͲ:͗DĞĐŚĂŶŝĐƐ//

(ii) The free-body diagram of the ladder is shown in Fig. 3.397.


A
l R1
Balancing torque about point A, f l = Mg ( sin 45°) C
2
Mg 45° B f = mR2
\ f = …(i)
2 2 Mg
45° R2
Balancing forces in the vertical direction,
Mg = R2 cos 45° + f sin 45° …(ii) O
From Eqs. (i) and (ii), we get Fig. 3.397
3 Mg
R2 = …(iii)
2 2
From Eqs. (i) and (iii), f = mR2.

Mg 3 Mg 1
or = m or m=
2 2 2 2 3

DAILY PRACTICE PROBLEMS FOR JEE MAIN AND ADVANCE

‡˜‡Žͳ౨(Only one option is correct) Which of the vector correctly represents


total acceleration of a point on the rim
1. A body is rotating uniformly about a
of the disk
vertical axis fixed in an inertial frame.
The resultant force on a particle of the 2
body not on the axis is
(a) vertical
(b) horizontal and skew with the axis 3 1
(c) horizontal and intersecting the
axis
(d) none of these 4
2. Figure shows an assembly of three (a) 1 (b) 2
small spheres of the same mass that (c) 3 (d) 4
are attached to a massless rod with the 4. Figure shows a graph of the angular
indicated spacings. Consider the mo- velocity versus time for the rotating
ment of inertia I of the assembly about disk. For a point on the rim of the
each sphere, in turn. The sphere(s) disk, the only radial acceleration is
about which moment of inertia is represented by
greatest is
w
1 2 3
d 2d

(a) 1 (b) 2
(c) 3 (d) 1 and 3 a b c d
t
3. Figure shows the overhead view of a
disk rotating counterclockwise. The (a) ab (b) ac
angular speed of the disk is decreasing. (c) bc (d) cd
ZŽƚĂƟŽŶപ587

5. Figure is a graph of the angular posi- (a) w μ 1/r


tion of the rotating disk. The angular (b) w μ r
velocity of the disk at t = 3 s (c) w is independent of r
(d) all
q (rad)
10. Fly wheel is an important part of an
engine
(a) it gives strength to engine
(b) it accelerates the speed of the
wheel
t(s)
1 2 3 (c) it reduces the moment of inertia
(d) it helps the engine in keeping the
(a) Zero (b) Positive speed uniform
(c) Negative (d) None 11. Figure shows three particles of the
6. Which of the following equations can same mass and the same constant
be used for uniformly accelerated ro- speed moving as indicated by the ve-
tating body locity vectors. Points a, b, c and d form
(a) w = 5 a square with point e at the centre. The
(b) w = t – 3 points about which angular momen-
(c) w = 5t2 – 3 tum is greatest
(d) w = 4t2 + t + 3
7. Consider the following two equations a b

   dL
(A) L = I w (B) t =
in non-inertial frame dt e

(a) both A and B are true


d c
(b) A is true but B is false
(c) B is true but A is false
(d) both A and B are false (a) a (b) a, b
8. A uniform rod is kept vertically on a (c) c, d (d) e
horizontal smooth surface at O. If it is 12. Figure shows a particle moving at
rotated slightly and released, it falls constant velocity v and four points
down on the horizontal surface. The with their xy coordinates. If L1, L2, L3
lower end will remain and L4 are the angular momenta about
(a) at O the points a, b, c and d respectively,
l then
(b) at a distance less than from O
2
l r
(c) at a distance from O. a v
2 c
(9, 1)
l (–2, +1)
(d) at a distance larger than from
2
O.
b ∑ (–3, –1)
b ∑ (–2, –2)
9. A body is in pure rotation. The linear
speed v of a particle, the distance r of
(a) L1 < L3
the particle from axis and the angular
(b) L1 > L4
velocity w of the body are related as w
= v/r. Then (c) L1 = L3 = L4
(d) (L1 = L3) < L4 < L2
588പWŚLJƐŝĐƐĨŽƌ//dͲ:͗DĞĐŚĂŶŝĐƐ//

13. A bola, which consists of three heavy (a) The hollow sphere reaches the
balls connected to a common point by bottom first
identical lengths of a sturdy string, is (b) The solid sphere reaches the
readied for launch by holding one of bottom with greater speed
the balls overhead and rotating the (c) The solid sphere reaches the bot-
wrist, causing the other two balls to tom with greater kinetic energy
rotate in a horizontal circle about the (d) The two spheres will reach the
hand. The bola is then released, and bottom with same linear momen-
its configuration rapidly changes from tum
that of figure (a) to the figure (b). Thus, 16. A solid sphere, a hollow sphere and a
the rotation is initially around axis 1 disc, all having same mass and radius,
through the ball that was held, then are placed at the top of a smooth incline
it is around axis 2 through the centre and released. The friction coefficients
of mass. If w1 and w2 are the angular between the objects and the incline
speeds in two cases of figure (a) and are same and not sufficient to allow
(b) respectively, then pure rolling. Least time will be taken
in reaching the bottom by
(a) the solid sphere (BITSAT, 2009)
(b) the hollow sphere
2 w2 (c) the disc
1
w1
(d) all will take same time
17. A body is rotating nonuniformly about
(a) a vertical axis fixed in an inertial frame.
(b)
The resultant force on a particle of the
(a) w1 = w2 (b) w1 < w2 body not on the axis is
(c) w1 > w2 (d) none of (a) vertical
the above (b) horizontal and skew with the axis
14. The density of a rod gradually (c) horizontal and intersecting the
decreases from one end to the other. It axis
is pivoted at an end so that it can move (d) none of these
about a vertical axis through the pivot. 18. Four forces of the same magnitude act
A horizontal force F is applied on the on a square as shown in the figure. The
free end in a direction perpendicular square can rotate about point O; mid-
to the rod. The quantities, that do not point of one of the edges. The force
depend on which end of the rod is which can produce greatest torque is
pivoted are
(a) angular acceleration
F3 F2
(b) angular velocity when the rod
completes one rotation
(c) angular momentum, when the
rod completes one rotation F1
O
(d) torque of the applied force F4
15. A hollow sphere and a solid sphere
having same mass and same radii are (a) F1 (b) F2
rolled down a rough inclined plane (c) F3 (d) F4
ZŽƚĂƟŽŶപ589

19. Figure below shows an overhead view moment of inertia of the system about
of a horizontal bar that can pivot about an axis passing through the centre of
the point indicated. Two horizontal mass and perpendicular to the string is
forces act on the bar, but the bar is (a) (m1 + m2)r2
stationary. If the angle between the bar
and force F2 is now decreased from the m1m2 r 2
initial 90° and the bar still do not turn, (b)
m1 + m2
then: (BITSAT, 2003)
m1r 2
F1 F2 (c)
Pivot m2 (m1 + m2 )

m2 r 2
(d)
(a) F2 to be made smaller m1 (m1 + m2 )
(b) F2 to be made larger
(c) F2 to be remain as such 23. The moment of the force about the
(d) none of the above origin, from the data as shown in the
20. A rod of mass m and length l is bent figure is
into shape of L. Its moment of inertia y
F = 10 N
about the axis shown in figure
60°

m/2 1m
30°
O x

m/2 (a) 10k N-m (b) 5 3 j N-m


(c) 5k N-m (d) 5j N-m
ml 2 ml 2 24. A force (3i + 4j) N is applied to a point
(a) (b) whose radius vector relative to origin
6 3
O is equal to (2i + j) m. The moment
ml 2 arm of the force relative to point O is
(c) (d) None
2 (a) 5m (b) 3m
21. A uniform cylinder has a length l and (c) 5m (d) 1m
radius R. If moment of inertia of this 25. A solid cylinder of mass M and radius
cylinder about its own geometrical axis R rolls down an inclined plane with
is equal to moment of inertia of the height h without slipping. The speed
same cylinder about an axis passing of its centre of mass when it reaches
through the centre and perpendicular the bottom is (OJEE, 2012)
to its length is
4
(a) 2gh (b) gh
(a) l = R (b) l = 3 R 3
1 3
(c) l = R (d) l = 3 R (c) gh (d) 4gh
3 4
22. Two point masses m1 and m2 joined
by a massless string of length r. The
590പWŚLJƐŝĐƐĨŽƌ//dͲ:͗DĞĐŚĂŶŝĐƐ//

26. A ring rolls on a plane surface. The (a) remains constant


fraction of its total energy associated (b) goes on increasing
with its rotation is (EAMCET, 2007) (c) goes on decreasing
1 (d) zero
(a) (b) 1 31. Figure shows a smooth inclined
2
plane fixed in a car accelerating on
1 a horizontal road. The angle of the
(c) (d) 2
3 incline q is related to the acceleration
27. A body slides down on an incline and a of the car as a = g tan q. If the sphere
reaches the bottom with a velocity v. is set in pure rolling on the inclined
If the same body were in the form of a
ring, its velocity at the bottom would
have been (MHCET, 2006)
(a) v (b) v 2
v 2
(c) (d) v (a) it will continue pure rolling
2 5 (b) it will slip down the plane
28. Three point masses, each of mass m (c) its linear velocity will increase
are placed at corners of an equilateral (d) its linear velocity will decrease
triangle of side l then the moment of 32. A disc is rolling (without slipping) on
inertia of this system about an axis a horizontal surface C is the centre, P
along one side of the triangle is and Q are two points equidistant from
(a) ml2 (b) 3 ml2 C. Let vP, vQ and vC be the magnitudes
of velocities of points P, Q and C
3 2 2 2 respectively, then (IIT-JEE, 2004)
(c) ml (d) ml
4 3
29. Four spheres each of diameter 2r and
∑Q
the mass M are placed with their ∑C
centres on the four corners of a square P∑

of side l then the moment of inertia of


the system about an axis along one of (a) vQ > vC > vP
the sides of the square is
(b) vQ < vC < vP
È4 ˘ 1
(a) M Í r 2 + 2 l 2 ˙ (c) vQ = vP, vC = vP
Î5 ˚ 2
(d) vQ < vC > vP
È8 ˘
(b) M Í r 2 + 2 l 2 ˙ 33. If I is the moment of inertia of a
Î5 ˚
solid sphere about an axis parallel to
8
(c) Mr2 a diameter and at a distance x from it.
5 Which of the following graphs repre-
È4 ˘ sents the variation of I with x
(d) M Í r 2 + 4 l 2 ˙
Î5 ˚
I I
30. A mass is moving with constant veloc-
(a) (b)
ity parallel to x-axis. Its angular mo-
mentum with respect to origin O x O x
ZŽƚĂƟŽŶപ591

I direction, and O is at rest. The velocity


I
of point A is
(c) (d)
x y
O x O z
34. ABC is a rectangular plate of uniform
thickness. The side AB, BC and AC are A O x
in the ratio 4, 3, 5 as shown in figure.
IAB, IBC, and ICA are the moments of

interia of the plate about AB, BC and
CA respectively. Which one of the (a) 10 k m/s (b) –10 j m/s
following statements is correct (c) + 10 j m/s (d) 100 k m/s
A 37. A particle P with a mass 2.0 kg has
position vector r = 3.0 m and velocity
v = 4.0 m/s are shown. It is accelerated
4 5 by the force = 2.0 N. All these vectors
lie in a common plane. The angular
90° momentum vector is
B C
3 y

(a) ICA is maximum


(b) IAB > IBC
(c) IBC > IAB v P
(d) IAB + IBC = ICA 30°
r
35. A thin circular ring of mass M and 45°
radius R is rotating about its axis x
with a constant angular velocity w. (a) 12 kg m2/s out of the plane of the
Two objects each of the mass m are figure
attached gently to the opposite ends of (b) 12 kg m2/s in to the plane of the
a diameter of the ring. The wheel now figure
rotates with an angular velocity (c) Zero
(IIT-JEE, 1983) (d) 24 kg m2/s in to the plane of the
wM figure
(a) 38. Two particles, each of mass m and
M+m
moving with speed v in opposite
w( M - 2m) directions along parallel lines, are
(b)
M + 2m separated by a distance d. The vector
wM angular momentum of this system of
(c) particles will be
M + 2m
(a) Maximum when the origin is tak-
w( M - 2m) en beyond the two parallel lines
(d)
M on either side
36. A strip of length 1 m rotates about the (b) Minimum when the origin is tak-
z-axis passing through the point O in en beyond the two parallel lines
the xy-plane with an angular velocity on either side
of 10 rad/s in the counterclockwise
592പWŚLJƐŝĐƐĨŽƌ//dͲ:͗DĞĐŚĂŶŝĐƐ//

(c) Maximum when the origin lies 41. A uniform solid sphere of radius R
anywhere on the middle line having moment of inertia I about its
between the two. diameter is melted to form a uniform
(d) Same to matter where point is disc of thickness t and radius r. The
taken as the origin moment of inertia of the disc about
39. A sphere of mass M and radius R an axis passing through its edge and
moves on a horizontal surface with perpendicular to its plane is I, then the
a velocity v and then climbs up an radius of the disc is
inclined plane up to a height h where
2 2
it stops. The height up to which it rises (a) R (b) R
will be 15 5

2 2
(c) R (d) R
5 15
v h 42. The moment of inertia of a uniform
q
semicircular disc of mass M and radius
R about a line perpendicular to the
(a) directly proportional to the square
plane of disc through the centre is
of the velocity and inversely
proportional to the angle of the 2 1
(a) MR2 (b) MR2
inclination 5 4
(b) directly proportional to the veloc-
ity and inversely proportional to 1
(c) MR2 (d) MR2
its mass. 2
(c) directly proportional to the square 43. Two identical rods each of mass M and
of the velocity and independent length L are joined to form a symmetrical
of mass and the angle of the X. The smaller angle between the rods
inclination is q. The moment of inertia of the
(d) directly proportional to its velocity system about an axis passing through
and inversely proportional to the the point of intersection of the rods
angle of the inclination. and perpendicular to this plane is
40. Four point masses each of value m are
placed at the corners of a square ABCD ML2
(a)
of side l. The moment of inertia of this 12
system about an axis passing through ML2
A and parallel to BD is (b)
6
D C ML2
(c) sin2 q
6
G l
ML2
(d) cos2 q
l 2 6
A B 44. A disc of mass M and radius R is
rolling with angular velocity w on a
horizontal plane as shown in figure.
(a) 2ml2 (b) 3 ml2 The magnitude of angular momentum
(c) 3ml2 (d) ml2 of disc about the origin O is
ZŽƚĂƟŽŶപ593

y (a) up the incline while ascending


and down the incline while de-
w
scending
R (b) up the incline while ascending as
M well as descending
O x
(c) down the incline while ascending
and up the incline while descending
1 (d) down the incline while ascending
(a) MR2w (b) MR2w
2 as well as descending
3 47. One quarter section is cut from a
(c) MR2w (d) 2 MR2w uniform circular disc of radius R. This
2
section has a mass M. It is made to
45. An equilateral triangle ABC formed rotate about a line perpendicular to its
from a uniform wire has two small plane and passing through the centre
identical beads initially located at A. A of original disc. Its moment of inertia
triangle is set rotating about the vertical about the axis of rotation is
axis AO. Then the beads are released (IIT-JEE, 2001)
from rest simultaneously and allowed 1
to slide down. One along AB the (a) MR2
2
other along AC as shown. Neglecting
frictional effects, the quantities that are 1
(b) MR
conserved as beads slide down are 4
M
(IIT-JEE, 2001) 1
(c) MR2
w 8
A (d) 2 MR2

g
48. A child is standing with folded hands at
the centre of a platform rotating about
its central axis. The kinetic energy
of the system is K. The child now
B C stretches his arms so that the moment
O
of inertia of the system doubles. The
(a) Angular velocity and total energy kinetic energy of the system now is
(kinetic and potential) (IIT-JEE, 2009)
(b) Total angular momentum and
K
total energy (a) 2K (b)
(c) Angular velocity and moment of 2
inertia about the axis of rotation K
(c) (d) 4 K
(d) Total angular momentum and 4
moment of inertia about the axis 49. A horizontal force F is applied at the
of rotation top of an equilateral triangular block
46. A cylinder rolls up an inclined plane, having mass m and side a as shown
reaches some height and then rolls in figure. The minimum value of the
down (without slipping throughout coefficient of friction required to topple
these motions). The directions of the the block before translation will be
frictional force acting on the cylinder (VITEEE, 2011)
are
594പWŚLJƐŝĐƐĨŽƌ//dͲ:͗DĞĐŚĂŶŝĐƐ//

F (b) is directed opposite to the velocity


of the centre of mass throughout
the journey
(c) is directed up the plane throughout
the journey
(d) is directed down the plane
2 1 throughout the journey
(a) (b)
3 2 53. A ball of mass m moving with velocity
v, collide with the wall elastically as
1 1
(c) (d) shown in the figure. After impact the
3 3 change in angular momentum about P
50. A spool is pulled horizontally by two is P
equal and opposite forces on rough (a) 2 mvd sin q
surface. The correct statement is d
(b) 2 mvd cos q
q
(c) 2 mvd
F (d) zero
F 54. A disc of radius R rolls on a horizontal
surface with linear velocity v and
angular velocity w. This is a point P
on the circumference of the disc at an
(a) centre of mass of spool moves angle q which has a vertical velocity.
towards right Here q is equal to
(b) centre of mass of spool moves P w
towards left q
(c) centre of mass remains at rest v
(d) none of these
51. A solid cylinder is wrapped with
a string and placed on an inclined Ê v ˆ
(a) p + sin–1 Á
plane as shown in the figure. Then Ë w R ˜¯
the frictional force acting between the
p Ê v ˆ
cylinder and plane is (b) - sin - 1 Á
2 Ë w R ˜¯
(a) zero
Ê v ˆ
(c) p – cos–1 Á
(b) 5 mg Ë w R ˜¯
7 mg Ê v ˆ
(d) p + cos–1 Á
(c)
2 m = 0.4 Ë w R ˜¯
mg
60° 55. A disc is performing pure rolling on
(d) a smooth stationary surface with con-
5
stant velocity v. For the point of con-
52. A cylinder is in pure rolling up an tact of the disc
inclined plane. It stops momentarily
and then rolls back. Therefore, friction
v
(a) on the cylinder is zero throughout R
the journey
ZŽƚĂƟŽŶപ595

(a) velocity is v, acceleration is zero (a) Decreases continuously


(b) velocity is zero, acceleration is (b) Decreases initially and increases
zero again
(c) velocity is v, acceleration is v2/R (c) Remains unaltered
(d) velocity is zero, acceleration is (d) Increases continuously
v2/R 59. A hoop of radius 0.10 m and mass 0.50
56. A uniform cylinder oscillates without kg rolls across a table parallel to one
slipping on rough inclined plane. The edge with a speed of 0.50 m/s. Refer
cylinder is connected to a spring at the its motion to a rectangular coordinate
centre, whose other end is fixed. The system with the origin at the left rear
force of friction on the cylinder corner of the table. At a certain time t, a
line drawn from the origin to the point
of contact of the hoop with the table
has length 1 m and makes an angle of
30° with the x-axis (see figure below).
What is the spin angular momentum
q of the hoop with respect to the origin
at this time t?
(a) is always upwards
z
(b) is always downwards
(c) is initially upwards and then
downwards periodically 30°
(d) none of these y
57. A solid sphere is rolling on a friction- x
less surface, shown in figure with a
translational velocity v m/s. If the
sphere climbs up to height h then val-
ue of v should be (BITSAT, 2007)

(a) – 0.25 i kg m2/s


h (b) – 0.005 i kg m2/s
(c) – 0.025 i kg m2/s
v

(d) – 0.5 i kg m2/s


10 60. Consider the two bobs are shown in
(a) gh (b) 2gh the figure. The bobs are pivoted to the
7
hinges through massless rods. If tA is
10
(c) 2 gh (d) gh the time taken by the bob A to reach
7 the lowest position and tB be the time
58. A horizontal platform is rotating with taken by the bob B to reach the lowest
uniform angular velocity around the position (both bobs are released from
vertical axis passing through its centre. rest from a horizontal position) then
At some instant of time a viscous fluid ratio tA/tB is
of mass m is dropped at the centre and
is allowed to spread out and finally m m
A B
fall. The angular velocity during this
period l l/2
596പWŚLJƐŝĐƐĨŽƌ//dͲ:͗DĞĐŚĂŶŝĐƐ//

(a) 3 (b) 5 L L
(c) w 2 , mw 2
2 2
1
(c) 2 (d) (d) zero, mw2L
2
63. Linear acceleration of a cylinder of
61. A hemispherical shell of mass m and
mass m2 is a2. Then angular acceleration
radius R is hinged at point O and
a2 is (given that there is no slipping).
placed on a horizontal surface. A ball
of mass m moving with a velocity u
inclined at an angle q = tan–1 (1/2)
strikes the shell at point A (as shown m1 , R
in the figure) and stops. What is the
minimum speed u if the given shell is
to reach the horizontal surface OP? m2, R
O P

a2 ( a2 + g )
A (a) (b)
R R
m q
u
2( a2 + g )
(c) (d) None of
(a) Zero R
these
2 gR
(b) ‡˜‡Žʹ౨(Only one option is correct)
3
gR 1. Minimum coefficient of friction re-
(c) quired to cause pure rolling of a cyl-
5
inder down on an inclined plane of
(d) it cannot come on the surface for inclination q is
any value of u.
sin q
62. A particle of mass m is attached to a (a) sin q (b)
rod of length L and it rotates in a circle 3
with a constant angular velocity w. An tan q tan q
observer P is rigidly fixed on the rod (c) (d)
3 2
at a distance L/2 from the centre. The
acceleration of m and the pseudo force 2. End of the bar AB in figure rests on
on m from the frame of reference of P a horizontal surface, while end B is
must be respectively. hinged. A horizontal force P of 60 N is
exerted on end A. Neglect the weight
w
of the bar. The vertical component of
P the force exerted by the bar on the
m
hinge at B is
L
O
B

(a) zero, zero 5m 4m

L
(b) zero, mw2 P = 60 N
2 A
ZŽƚĂƟŽŶപ597

(a) 60 N (b) 80 N 7. In problem 6, if v1 is the speed of block


(c) 100 N (d) 120 N at B and v2 is the speed of sphere at the
3. Figure shows the motions of three same position, then
identical spheres on three different (a) v1 = v2 (b) v1 < v2
ramps of same inclination. (c) v1 > v2 (d) none of
the above
8. In the figure, a block slides down a
frictionless inclined plane and a sphere
rolls without sliding down a ramp of
the same angle q. The block and sphere
(a) (b) (c)
have the same mass, start from rest
The rough part of the ramps are shown at point A, and descend to point B. If
by bold line. KA, KB and KC are their work done by gravitational force on
kinetic energy at the bottom of the the block is W1 and that on sphere is
ramp, then W2, then
(a) KA < KB A A
(b) (KA = KB) < KC
(c) KA < (KB = KC)
(d) (KA = KB > KC B B
q q
4. Two spheres of radii R and 2R roll on
an inclined plane without slipping. If (a) W1 = W2 (b) W1 < W2
v1, v2 and w1, w2 are their velocities of (c) W1 > W2 (d) None of
centre of mass and angular velocities the above
of rotation after moving a distance l, 9. Figure shows overhead views of three
then structures on which three forces act.
(a) v1 = v2, w1 = w2 The direction of the forces are as in-
(b) v1 < v2, w1 < w2 dicated. If the magnitude of the forces
(c) v1 < v2, w1 = w2 are adjusted properly, which structure
can be in stable equilibrium
(d) v1 = v2, w1 > w2
5. If K1 is the translational kinetic energy
of block and K2 is the translational
kinetic energy of the sphere at B, then
(a) K1 = K2 (b) K1 < K2 A B C
(c) K1 > K2 (d) none of
the above (a) A (b) B
6. If E1 is the kinetic energy of the block (c) C (d) None
and E2 is the kinetic energy of sphere 10. Figure shows four rotating disks that
at the position B, then are sliding across a frictionless floor.
(a) E1 = E2 (b) E1 < E2 Three forces F, 2F or 3F act on each
(c) E1 > E2 (d) none of disk, either at the rim, at the centre,
the above or halfway between rim and centre.
Which disks are in equilibrium
598പWŚLJƐŝĐƐĨŽƌ//dͲ:͗DĞĐŚĂŶŝĐƐ//

F F moment of force (t ) relative to the


point O acting on the particle when
3F 2F 
2F
the angle between the vectors t and J
F is equal 45°
A B
a a
(a) (b) 2
F F b b
2F F a b
(c) 2b (d) 2a
F b a
2F
C D 13. Moment of inertia of an uniform
(a) A, D (b) A, B circular disc about a diameter is I.
(c) C, D (d) A, C Its moment of inertia about an axis
perpendicular to its plane and passing
11. A rod AB of length L slides in the xy-
through a point its rim will be
plane. If the rod makes an angle q with
the vertical, the angular velocity of the (a) 5I (b) 3I
rod will be (c) 6I (d) 4I
14. A uniform spherical shell of mass M
y
and radius R rotates about a vertical
axis on frictionless bearing as shown in
A
figure. A massless cord passes around
q L the equator of the shell over a pulley
of rotational inertia l and radius r and
is attached to a small object of mass
B x m that is otherwise free to fall under
the influence of gravity. There is no
friction of pulley axle. The cord does
(a) Directly proportional to the length
not slip on the pulley. The speed of
of the rod and the linear velocity
the object after it has fallen a distance
of the end A at that instant.
h from rest is
(b) Independent of the length of the
rod but will depend on the angle
q.
(c) Independent of q but will depend
on the length of the rod and linear
velocity of the end A at that
instant.
(d) Dependent upon the length of the
rod, the angle q and also on the
linear velocity of the end A of the
rod at that instant. (a) v2 = 2 gh
12. The angular momentum of a particle
relative to a point O varies with time as mgh
  (b) v2 =
J = a + bt2, where a and b are constant m I
 + 2 +M
 2 2r
vectors, with a perpendicular b . The
ZŽƚĂƟŽŶപ599

mgh (a) 2 v
(c) v2 = (b) 2 vt
m I M
+ +
2 2r 2 3 Ê vt ˆ
(c) 2v cos Á ˜
Ë 2r ¯
mgh
(d) v2 =
m I
+ +M Ê vt ˆ
2 r2 (d) 2v sin Á ˜
Ë 2r ¯
15. A block of mass M rests on a turntable 17. A rod of mass m and length is hinged
that is rotating at constant angular to its one end and held vertical. A point
velocity w. A smooth cord runs from mass m is attached to the other end, is
the block through a hole in the centre allowed to rotate about the hinge. The
of the table down to hanging block velocity of the rod when it becomes
of mass m. The coefficient of friction horizontal is (VITEEE, 2013)
between the first block and the
turntable is m (see figure). The smallest 3
(a) 3gl (b) gl
value of the radius r for which the first 2
block will remain at rest relative to the 3
turntable (c) gl (d) None
4
18. A uniform rod AB of length L is hinged
M
at one end A. The rod is kept in the
horizontal position by a massless
string tied to point B as shown in
figure. If the string is cut, the initial
m
angular acceleration of the rod will be
(VITEEE, 2007)
mg - m Mg
(a) r = g
Mw 2 (a)
L
mg + m Mg String
(b) r = 2g
Mw 2 (b)
L
mg A
(c) r = 2g B
Mw 2 (c) L
3L
(d) None of the above
3g
16. A hoop of radius r rolls on a horizontal (d)
2L
plane with constant velocity v without
slipping. The velocity of any point, t 19. A circular wooden hoop of mass m
second after it passes the top position and radius R rests flat on a horizontal
is frictionless surface. A bullet, also of
mass m, and moving with a velocity v,
w
strikes the hoop and gets embedded in
q it. The thickness of the hoop is much
v
smaller than R. The angular velocity
with which the system rotates after the
bullet strikes the hoop is
600പWŚLJƐŝĐƐĨŽƌ//dͲ:͗DĞĐŚĂŶŝĐƐ//

v 3v
(a) (c) counterclockwise
4R 4L
m
v v
(b) R (d) counterclockwise
3R L
2v v 22. A uniform cylindrical disc of radius R
(c) and mass M is pulled over a horizontal
3R m
frictionless surface by a constant
3v force. The force is applied by means
(d)
4R of a string wound around the disc as
shown in the figure. If it starts from
20. A straight bar of mass 15 kg and length
rest at t = 0, the linear and angular
2 m, at rest on a frictionless horizontal
displacements respectively at time t
surface, receives an instantaneous
are (IIT-JEE, 1999)
impulse of 7.5 N-s perpendicular to
the bar. If the impulse is applied at the y
centre of mass of the bar, the energy
transferred is (BITSAT, 2011)
(a) 3.2 J M
R
x
(b) 1.9 J
J F
(c) 3.8 J
(d) 2.5 J
Ê Fˆ Ê Fˆ
21. A bar of mass M and length L is in (a) Á ˜ t 2 , Á ˜ t 2
pure translatory motion with its centre Ë M¯ Ë M¯
of mass velocity v. It collides with and
sticks to another identical bar which Ê F ˆ Ê F ˆ 2
(b) Á t, t
is initially at rest. (Assume that it be- Ë 2 M ˜¯ ÁË 2 MR ˜¯
comes one composite bar of length 2L.)
The angular velocity of the composite Ê F ˆ 2 Ê F ˆ 2
(c) Á t , t
bar will be Ë 2 M ˜¯ ÁË MR ˜¯
M
Ê 2F ˆ Ê 2F ˆ 2
(d) Á ˜ t 2 , Á t
L Ë M ¯ Ë MR ˜¯

P 23. A uniform disc of mass M and radius


M R is mounted on an axle supported
L v on fixed frictionless bearings. A light
cord wrapped around the rim is
pulled with a force 5 N. On the same
system of pulley and string, instead
3v of pulling it down, a body of weight
(a) clockwise
4L 5 N is suspended. If the first process
4v is termed A and the second B, the
(b) clockwise tangential acceleration will be
3L
ZŽƚĂƟŽŶപ601

M M 26. A thin wire of length L and uniform


linear mass density r is bent into a
R R circular loop with centre at O as
shown. The moment of inertia of the
loop about the axis xx’ is
5 N (Pull) mg = 5 N (IIT-JEE, 2000)
(A) (B) x x¢

(a) equal in the processes A and B 90°


(b) greater in process A than in B
(c) greater in process B than in A O
(d) independent of the two processes
24. A hole of radius R/2 is cut from a thin
circular plate of radius R. The mass of
the remaining plate is M. The moment rL3 rL3
of inertia of the plate about an axis (a) (b)
8p 2 16p 2
through O perpendicular to the xy-
plane (i.e., about the z-axis) is (the 5 rL3 3 rL3
(c) (d)
mass of remaining disc is M). 16p 2 8p 2
y 27. A disc of radius R has mass of 9 m.
R
A hole of radius is cut from it as
3
shown in figure. The moment of inertia
R R/2
x of the remaining part about an axis
O
M passing through centre O of the disc
and perpendicular of the plane of disc
is (JEE, 2005)
5 7
(a) MR2 (b) MR2
7 12 2R
3
11 13 O
(c) MR2 (d) MR2
24 24 R
25. In the device shown in figure, the
cylinder is in the pure rolling motion.
At certain instant the angular speed of (a) 8 mR2 (b) 4 mR2
the cylinder is w. The velocity of the
40 37
block at that instant is (c) mR2 (d) mR2
9 9
(a) wR
2R
(b) 2wR R
28. A circular disc X of radius R is made
from an iron plate of thickness t and
(c) 3wR another plate Y of radius 4R is made
m
(d) 4wR t
from an iron plate of thickness . The
4
Iy
ratio between moments of inertia is
Ix
602പWŚLJƐŝĐƐĨŽƌ//dͲ:͗DĞĐŚĂŶŝĐƐ//

(a) 1 (b) 16 31. Consider a body consisting of two


(c) 32 (d) 64 identical balls, each of mass M, con-
29. A symmetrical lamina of mass M nected by a light rigid rod. If an im-
consists of square shape with a semi- pulse J = Mv is imparted to the body
circular section over each edge of the at one of its ends, what should be its
square as shown in figure. The side of angular velocity?
the square is 2 a. The moment of inertia
M M
of the lamina about an axis through its
J = Mv
centre of mass and perpendicular to
the plane is 1.6 Ma2. The moment of
inertia of the lamina about the tangent v 2v
(a) (b)
AB in the plane of the lamina is L L
(IIT-JEE, 1997) v v
(c) (d)
A B 3L 4L
a
32. A force F is applied at the top of a
a
ring placed on a rough horizontal
surface as shown in figure. Friction
is sufficient to cause pure rolling. The
frictional force acting on the ring is
F

(a) 0.8 Ma2 (b) 2.6 Ma2


(c) 3.6 Ma2 (d) 4.8 Ma2 F
(a) towards left
30. A circular platform is free to rotate in 2
a horizontal plane about a vertical axis
passing through its centre. A tortoise F
(b) towards right
is sitting at the edge of the platform. 2
Now the platform is given an angular
F
velocity w0. When the tortoise moves (c) towards left
along a cord of platform with a 3
constant velocity (with respect to the (d) zero
platform) the angular velocity of the 33. A time varying force F = 2t is applied
platform w(t) will vary with time t as on a spool as shown in figure. The
(IIT-JEE, 2002) angular momentum of the spool at
time t about the bottommost point is
w(t) w(t)
r 2t 2
(a) w0 (b) w0 (a) F
2 r
t t ( R + r )2 2 R
(b) t
w(t) w(t) r

(c) (d) (c) (R + r)t2


w0
w0
(d) none of these
t t
ZŽƚĂƟŽŶപ603

34. A cylinder rolls without slipping on a (a) mk2 + m(a + b)2 – ma2
plank is device shown in figure. The
( a + b )2
acceleration of the plank to keep the (b) mk2 + m
cylinder in a fixed position during the 2
2
motion is b
(c) mk2 + m
g 2
(a) sin q
2 a (d) mk2 + mb2
(b) g sin q 38. A small particle of mass m is given an
q initial high velocity in the horizontal
(c) 2 g sin q
plane and winds its cord around the
(d) 2g sin q fixed vertical shaft of radius a. All
35. A plank P is placed on a hollow cyl- motion occurs essentially in horizontal
inder C, which rolls on a horizontal plane. If the angular velocity of the
surface as shown. No slippage is there cord is w0 when the distance from the
at any of the surfaces in contact. Both particle to the tangency point is r0,
have equal mass say M (each) and if v then the angular velocity of the cord
is the velocity of the centre of mass of w after it has turned through an angle
the cylinder C, then the ratio of the ki- q is
netic energy of plank P to the cylinder
C is
P
C
m

(a) 1:1 (b) 2:1 a


O
(c) 3:8 (d) 8:11
a
36. A sphere is given some angular veloc- (a) w = w0 (b) w = w0
ity about a horizontal axis through the r0

(c) w = Ê
centre, and gently placed on a plank w0 ˆ
(d) w = w0q
with coefficient of friction m. The plank Á a ˜
rests on a smooth horizontal surface. Á1- q ˜
Ë r0 ¯
The initial acceleration of the sphere
relative to the plank is: (mass of sphere 39. A rectangular piece of dimension l
= mass of plank) ¥ b is cut out of central portion of a
uniform circular disc of mass m and
(a) mg
radius r. The moment of inertia of
7 the remaining piece about an axis
(b) mg
5 perpendicular to the plane of the disc
(c) 2 mg and passing through its centre is
(d) zero È lb 2 ˘
37. Moment of inertia of a uniform disc (a) m Ír 2 - ( l + b 2 )˙
Î 6p r 2 ˚
of mass m about an axis x = a is mk2,
where k is the radius of gyration. What mÈ 2 lb 2 ˘
is its moment of inertia about an axis x (b) Í r - ( l + b 2 )˙
2Î 6p r 2 ˚
=a+b
604പWŚLJƐŝĐƐĨŽƌ//dͲ:͗DĞĐŚĂŶŝĐƐ//

m È 2 (l 2 + b 2 ) ˘ 43. A thin uniform straight rod of mass


(c) Ír - ˙ 2 kg and length, 1 m is free to rotate
2Î 6 ˚
about its upper end, when at rest, it
(d) not determinable as mass of the recieves an impulse of 10 N-s at its
rectangular piece is not given lower end, normal to its length. The
40. A cubical block of side L rests on a kinetic energy of rod after impact is
rough horizontal surface with coeffi-
cient of friction m. A horizontal force
F is applied on the block as shown. If
the coefficient of friction is sufficiently 1m
high so that the block does not slide
before toppling, the minimum force 10 Ns
required to topple the block is (a) 75 J (b) 100 J
(IIT-JEE, 2000) (c) 200 J (d) 225 J
F 44. A rigid body can be hinged about any
M point on the x-axis. When it is hinged
such that the hinge is at a distance x,
the moment of inertia is given by
Mg
(a) infinitesimal (b) I = 2x2 – 12x + 15
4
Mg The x-coordinate of the centre of mass is
(c) (d) Mg (1 – m)
2 (a) x = 0 (b) x = 1
41. A uniform sphere of radius R is placed (c) x = 2 (d) x = 3
on a rough horizontal surface and 45. A hollow sphere of mass 2 kg is kept
given a linear velocity v0 and angular on a rough horizontal surface. A force
velocity w0 as shown. The sphere comes of 10 N is applied at the centre of the
to rest after moving some distance to sphere as shown in the figure. The
the right. It follows that minimum value of m so that the sphere
(BITSAT, 2003) starts pure rolling is (g = 10 m/s2)

w F
v0 30°

m
(a) v0 = w0R (b) 2v0 = 5w0R
(c) 5v0 = 2w0R (d) 2v0 w0R (a) 3 ¥ 0.16 (b) 3 ¥ 0.08
42. A uniform rod of mass M is hinged at its (c) 3 ¥ 0.1 (d) none of
upper end. A particle of mass m moving these
horizontally strikes the rod at its mid- 46. The spool shown in figure is placed on
point elastically. If the particle comes to a rough horizontal surface has inner
rest after collision, then M/m is radius r and outer radius R. The angle
2 3 q between the applied force and the
(a) (b)
3 4 horizontal can be varied. The critical
4 angle (q) for which the spool does not
(c) (d) none roll and remains stationary is given by
3
ZŽƚĂƟŽŶപ605

r 1
(a) sin q = (c) v0 = v1
R 5
r (d) v0 = 3v1
(b) cos q =
R 50. A cubical block of side a moving with
r F
R q velocity v on a horizontal smooth
r
(c) cos q = plane as shown in figure. It hits a ridge
R
at point O. The angular speed of the
2r block after it hits O is (IIT-JEE, 1999)
(d) cos q =
R 3v
(a)
47. A particle of mass 1 kg located at 4a a
the position (i + j)m has a velocity 2 3v
(b)
(i - j + k
 ) m/s. Its angular momentum
2a v
O
about z-axis is
3v
(a) zero kg-m2/s (c)
2a
(b) + 4 kg-m2/s
(d) zero
(c) – 4 kg-m2/s
51. A straight uniform metal rod of
(d) + 8 kg-m2/s
length 3l is bent through angle as
48. An impulse J is applied on a ring of shown. The bent rod is then placed
mass m along a line passing through on a rough horizontal table. A light
its centre O. The ring is placed on a string is attached to the vertex of the
rough horizontal surface. The linear right angle. The string is then pulled
velocity of centre of ring once it starts horizontally so that the rod translates
rolling without slipping is at constant velocity. Then the angle a
(WBJEE, 2013) which the side 2l makes with string is
r
J O
2l
a
(a) J/m (b) J/2m
(c) J/3m (d) J/4m
49. A sphere of radius r and mass m has a l
velocity v0 directed to the left and no
angular velocity as it is placed on a belt Top view
moving to the right with a constant
velocity v1. If after sliding on the belt
1
the sphere is to have no linear velocity (a) p – tan–1
relative to the ground as it starts rolling 2
on the belt without sliding. In terms of 1
(b) p – tan–1
v1, the velocity v0 is 4
v0 1
2 (c) r – sin–1
(a) v0 = v1 2
5
1
3 v1 (d) p – sin–1
(b) v0 = v1 4
5
606പWŚLJƐŝĐƐĨŽƌ//dͲ:͗DĞĐŚĂŶŝĐƐ//

52. Two particles A and B of mass m each (d) through the point at a distance 5
and moving with velocity v, hit the cm from A
ends of a rigid bar of the same mass 54. Two thin rods; each of mass m and
m and length l simultaneously and length l are joined to form L shape as
stick to the bar as shown in the figure. shown. The moment of inertia of rods
The bar is kept on a smooth horizontal about an axis passing through the free
plane. The linear and angular speed end (O) of a rod end and perpendicular
of the system (bar + particle) after the to both the ends is
collision are

Av
l
l l

v B

2 2 ml 2
12 v (a) ml (b)
(a) vcm = 0, w = 7 6
7 l 5 2
4v (c) ml (d) ml2
(b) vcm = 0, w = 3
l 55. A uniform rod AB of length l and mass
5v m is laying on a smooth table. A small
(c) vcm 0, w =
l particle of mass m strike the rod with
v a velocity is at C a distance x from the
(d) vcm 0, w = centre O. The particle comes to rest
5l
after collision. The value of x, so that
3 point A of the rod remains stationary
53. A block of mass kg is placed on
10 just after collision is
a rough horizontal surface as shown B
in the figure. A force of 1 N is applied
v0 C
at one end of the block and the block m x
remains stationary. The normal force O
exerted by the surface on the block acts
(g = 10 m/s2)
A
F=1N
30° (a) l/3 (b) l/6
(c) l/4 (d) l/12
20 cm

56. A spool is pulled vertically by a con-


stant force F (< Mg) as shown in figure.
The frictional force is best represented
B 20 cm A in
F
(a) through the centre of mass of the
block
(b) through point A
(c) through point B
ZŽƚĂƟŽŶപ607

F F
Ê 1 ˆ
(a) tan–1 Á
(a) (b) Ë 3 ˜¯

(b) tan–1 ÊÁ ˆ˜
f f
3
Ë 4¯
F
(c) tan–1 (1)
(c) (d) none Ê 4ˆ
(d) tan–1 Á ˜
Ë 3¯
f=0
59. A particle of mass m is attached to
57. A weight W rests on the bar AB as a disc of equal mass m by means
shown in figure. The cable connecting of a slack string as shown. The disc
W and B passes over frictionless pulleys. is hinged about its centre C, on a
If bar AB has negligible weight, the horizontal smooth table. The particle
vertical component of reaction at A is is projected with initial velocity v. Its
velocity when the string becomes taut
is

•C

a
m v
W m
B A
(a) v (b) v/3
L
3v
(c) 2v/3 (d)
4
L-a L+a
(a) W (b) W 60. A ring of mass m and radius R has four
L+a L-a
particles each of mass m attached to the
L - 2a L-a ring as shown in figure. The centre of
(c) W (d) W
L+a L + 2a ring has a speed v0. The kinetic energy
of the rolling ring at an instant shown
58. Three identical rods are hinged at
is
point A as shown. The angle made by
rod AB with vertical is m

A
m va m

B 90° m
D

90°
(a) mv02 (b) 3mv02
(c) 5mv02 (d) 6mv02
C
608പWŚLJƐŝĐƐĨŽƌ//dͲ:͗DĞĐŚĂŶŝĐƐ//

Multiple Correct Options (c) on the front wheels has larger


magnitude than the rear wheels
1. A wheel is rolling on a horizontal plane (d) on the car in backward direction
without slipping. At a certain instant, it 5. When a sphere of radius R is thrown
has velocity v and acceleration a of CM on a rough horizontal surface with
as shown in the figure. Acceleration of velocity v0, then
C (a) its linear velocity decreases and
becomes zero
a (b) its angular velocity increases to
B
v
v0
A
R

(a) A is vertically upwards (c) its velocity increases but cannot


(b) B may be vertically downwards v
be equal to 0
(c) C cannot be horizontal R
(d) some point on the rim may be (d) the frictional force acts in back-
horizontal leftwards. ward direction of v0.
2. A particle is moving along an ex- 6. Let IA and IB be moments of inertia of
panding spiral in such a manner that a body about CM and B respectively.
magnitude of normal acceleration of Then
particle remains constant. Choose the (a) IA < IB
correct options. (b) If the axes are parallel, IA < IB
(a) Linear speed of particle is increas- (c) If the axes are not parallel, IA £ IB
ing (d) none of these
(b) Linear speed of particle is de- 7. In the figure shown the lines of action
creasing and moment arms of two forces about
(c) Angular speed of particle is in- the origin O. Imagining these forces to
creasing be acting on a rigid body pivoted at O,
(d) Angular speed of particle is de- all vectors shown being the plane of the
ceasing figure, the magnitude and direction of
3. A sphere is rolled on a rough horizontal the resultant torque will be
surface. It gradually slows down and
stops. The force of friction tries to
(a) decreases the linear velocity
q2
(b) increases the angular velocity q1
(c) increases the angular momentum r1 r2 F2
(d) decreases the linear momentum. F1 O
4. In front-wheel drive cars, the engine
rotates the front wheels and the rear (a) (F2r2 sin q2 – F1r1 sin q1) out of the
wheels rotate only because the car plane of the page.
moves. If such a car accelerates on a
(b) (F1r1 sin q1 – F2r2 sin q2) out of the
horizontal road, the frictional force
plane of the page.
(a) on the front wheels is in the
forward direction (c) (F2r2 sin q2 – F1r1 sin q1) into the
(b) on the rear wheels is in the back- plane of the page.
ward direction (d) Zero
ZŽƚĂƟŽŶപ609

8. A ring of mass M and radius R can about that point. From this it follows
have moment of inertia that (IIT-JEE, 1998)
(a) MR2 (b) 2MR2 dL
2 (a) is perpendicular to L at all
(c) 3 MR (d) 4 MR2 dt
9. Let F be a force acting on a particle instants of time.

having vector r . Let be the torque of (b) the component of L in the direction
this force about the origin, then of A does not change with time.
  
(a) r ◊ t = 0 (b) F ◊ t = 0 (c) the magnitude of L does not
   
(c) r ¥ t = 0 (d) r ¥ t π 0 change with time.
10. A man stands at the centre of a turn- (d) L does not change with time
table and the turntable is rotating 15. A small ball is connected to a block
about its centre. If the man walk away by a light string of length l. Both
from the axis of rotation, then are initially on the ground. There is
(a) moment of inertia of the system sufficient friction on the ground to
increases prevent the block from slipping. The
(b) angular momentum of the system ball is projected vertically up with a
increases velocity u, where 2g l < u2 < 3gl. The
(c) angular velocity of the system de- centre of mass of the block + ball
creases system is C.
(d) kinetic energy of the system
increases l
11. In which of the following cases the
angular momentum is constant
(a) a particle moving on a circular (a) C will move along a circle
path with constant speed (b) C will move along a parabola
(b) a satellite revolving in elliptical (c) C will move along a straight line
orbit (d) The horizontal component of
(c) earth revolving around the sun the velocity of the ball will be
(d) during opening of a door by maximum when the string makes
applying force on the handle an angle q = sin–1 (u2/3gl) with the
12. The moment of inertia of a body de- horizontal
pends on 16. The moment of inertia of a thin square
(a) mass of body plate ABCD of uniform thickness about
(b) distribution of mass an axis passing through the centre O
(c) location of axis of rotation and perpendicular to the plane of the
(d) angular velocity of the body plate is (IIT-JEE, 1992)
I2
13. A body can have (a) I1 + I3 A B I3
(a) three moments of inertia
(b) I1 + 2I2
(b) infinite number of moments of
inertia (c) I1 + I2 I1
O
(c) infinite moments of inertia I1 + I 2 + I 3 + I 4
(d) None of the above (d) D C I4
2
14. The torque t on a body about a given
point is found to be equal to A ¥ L 17. In the figure shown, the plank is being
where A is a constant vector, and L is pulled to the right with a constant
the angular momentum of the body
610പWŚLJƐŝĐƐĨŽƌ//dͲ:͗DĞĐŚĂŶŝĐƐ//

velocity v. If the cylinder does not slip, mv 3


then (a) zero (b)
4 2g
mv 3
(c) (d) m 2 gh 3
R 2g
v 21. The angular acceleration of the
toppling pole shown in figure is given
(a) the speed of the centre of mass of by a = k sin q, where q is the angle
the cylinder is 2v. between the axis of the pole and the
(b) the speed of the centre of mass of vertical, and k is a constant. The pole
the cylinder is zero starts from rest at q = 0. Choose the
(c) the angular velocity of the cylinder correct options
is v/R a
(d) the angular velocity of the cylinder
is zero as
an
18. If a cylinder is rolling down the incline
with sliding l q

(a) after some time it may start pure


rolling
(b) after some time it will start pure
O
rolling
(c) it is possible that it never starts (a) The tangential acceleration of the
pure rolling upper end of the pole is lk sin q
(d) none of these (b) The centripetal acceleration of the
19. A sphere is rolling without slipping upper end of the pole is 2kl (1 –
on a fixed horizontal plane surface. In cos q)
figure, A is the point of contact, B is (c) The tangential acceleration of the
the centre of the sphere and C is its upper end of the pole is 2kl (1 –
topmost point. Then (IIT-JEE, 2009) cos q)
(d) The centripetal acceleration of the
C upper end of the pole is lk sin q
B 22. A disc is given an initial angular
velocity w0 and placed on rough
A horizontal surface as shown. The
    quantities which will not depend on
(a) v C - v A = 2(v B - v C ) the coefficient of friction is/are
   
(b) v C - v B = v B - v A
   
(c) |v C - v A|= 2|v B - v C|
   w0
(d) |v C - v A|= 4|v B|
20. A particle of mass m is projected with (a) The time until rolling begins.
a velocity v, making an angle 45° with (b) The displacement of the disc until
the horizontal. The magnitude of the rolling begins.
angular momentum of the particle (c) The velocity when rolling begins.
about the point of projection when the (d) The work done by the force of
particle is at its maximum height h is friction.
ZŽƚĂƟŽŶപ611

Passages and Matrix Matching of inertia I and 2I respectively about the


common axis. Disc A is imparted an initial
Passage for Q: 1–3 angular velocity 2 w using the entire potential
A non-uniform cylinder of mass 2 M and energy of a spring compressed by a distance
radius 2 R is released from the position x1. Disc B is imparted an angular velocity w
shown. The moment of inertia of the cylinder by a spring having the same spring constant
about longitudinal axis passing through A is and compressed by a distance x2. Both the
4 MR2. Point B (distance R from the axis) is discs rotate in the clockwise direction.
the centre of mass of the cylinder. Friction
4. The ratio x1/x2 is
between the cylinder and the horizontal
surface is sufficient to prevent slipping. 1 1
(a) (b)
2 2

(c) 2 (d) 2
R
B A 5. When disc B is brought in contact with
disc A they acquire a common angular
velocity in time t. The average frictional
torque on one disc by the other during
this period is
1. Initial angular acceleration of the
cylinder is 2Iw 9Iw
(a) (b)
g 3t 2t
(a) g/R (b)
3R
9Iw 3Iw
g g (c) (d)
(c) (d) 4t 2t
5R 6R
6. The loss of kinetic energy in the above
2. Frictional force acting on the cylinder
process is
just after its release is
Mg 1 2 1 2
(a) Mg (b) (a) Iw (b) Iw
3 2 3
2 Mg 4 Mg
(c) (d) 1 2 1 2
3 7 (c) Iw (d) Iw
4 5
3. Angular speed of the cylinder as the
line AB becomes vertical is Passage for Q: 7–9
A disc of radius R is spun to an angular
g g
(a) (b) speed w0 about its axis and then imparted a
R 3R
w R
horizontal velocity v0 = 0 (at t = 0) with
g g 4
(c) (d) its plane remaining vertical. The coefficient
5R 6R
of friction between the disc and plane is m.
The direction of v0 and w0 are shown in the
Passage for Q: 4–6
figure.
Two disc A and B are mounted coaxially
on a vertical axle. The discs have moment
612പWŚLJƐŝĐƐĨŽƌ//dͲ:͗DĞĐŚĂŶŝĐƐ//

A force F = 55 N is applied on the plank such


that the plank moves and causes the cylinder
v
to roll. The plank always remains horizontal.
There is no slipping at any point of contact.
10. Calculate the acceleration of cylinder
(a) 20 m/s2 (b) 10 m/s2
7. The disc will return to starting point at (c) 5 m/s2 (d) none of
time these
11. Find the value of frictional force at A
Ê 25 ˆ Rw 0 5 Rw 0
(a) Á ˜ (b) (a) 7.5 N (b) 5.0 N
Ë 48 ¯ m g 12 m g
(c) 2.5 N (d) none of
5 Rw 0 Rw 0 these
(c) (d) 12. Find the value of frictional force at B
48 m g 6m g
(a) 7.5 N (b) 5.0 N
8. The disc will start rolling without slip- (c) 2.5 N (d) none of
ping at time these
Rw 0 5Rw 0
(a) (b) Passage for Q: 13–15
6m g 12 m g
A rectangular rigid fixed block has a long
Rw 0 Rw 0 horizontal edge. A solid homogeneous
(c) (d) cylinder of radius R is placed horizontally at
mg 4m g
rest with its length parallel to the edge such
9. The angular momentum of the disc that the axis of the cylinder and the edge O
about the point of contact, when slip- of the block are in the same vertical plane
ping ceases is equal to as shown in the figure. There is sufficient
w0 w0 friction present at the edge so that a very
(a) MR2 (b) MR2 small displacement causes the cylinder to
2 12
roll of the edge without slipping.
w0 w0
(c) MR2 (d) MR2
4 3
R
Passage for Q: 10–12
Consider a cylinder of mass M = 1 kg and
radius R = 1 m lying on a rough horizontal
plane. It has a plank lying on its top as shown 13. Determine the angle q though which
in the figure. the cylinder rotates before it leaves
contact with the edge.
F
m = 1 kg 60° (a) cos–1 (3/4) (b) cos–1 (4/7)
–1
A (c) cos (5/7) (d) sin–1 (4/5)
14. Determine the speed of the centre of
mass of the cylinder before leaving
M R contact with the edge.

(a) 3 gR/7 (b) 2gR


B
(c) ( 4 gR/7 ) (d) 7/4 gR
ZŽƚĂƟŽŶപ613

15. Determine the ratio of the translational of mass m travelling along the surface hits
to rotational kinetic energies of the the end A of the rod with a velocity v0 in a
cylinder when its centre of mass is in direction perpendicular to AB. The collision
horizontal line with its edge. is completely elastic. After the collision the
(a) 3 (b) 6 particle comes to rest.
(c) 4 (d) 2 19. Find the ratio m/M.
Passage for Q: 16–18 (a) 1/4 (b) 1/3
(c) 1/2 (d) 3/4
A uniform rod AB of length 2l falls without
20. A point P on the rod is at rest
rotation on a smooth horizontal surface at an
immediately after the collision. Find
angle q to the horizontal. The speed of rod
the distance AP.
just before collision is v0 and the collision
(a) 3L/4 (b) (2/3) L
is elastic. The magnitude of the angular
(c) L/2 (d) 3L/8
velocity and magnitude of the velocity of
21. Find the linear speed of the point P at
centre of mass after collision are w and v¢
a time p/3v0 after the collision.
respectively.
(a) v0/2 (b) 3v0/2
B
(c) (v0/4) 2 (d) v0/2 2
Passage for Q: 22–24
A q
A uniform flat disc of mass M and radius R
rotates about a horizontal axis through its
16. The direction of force of impact on the centre with angular speed w0
rod is
(a) along the surface m
(b) vertically upwards
(c) along the rod
(d) any direction is possible
17. The relation between v0, w and v¢ is
(a) v0 = v¢ 22. What is its angular momentum?
(b) v0 = v¢ + lw
1
(c) v0 = v¢ – lw (a) MR2w0 (b) MR2w0
2
(d) v0 = v¢ + lw cos q 2
18. The angular momentum before collision (c) 2 MR2w0 (d) MR2w0
3
about the point on the ground at which
the rod strikes has the magnitude (m: 23. A chip of mass m breaks off the edge
mass of rod) of the disc at an instant such that the
chip rises vertically about the point at
mv0 l
(a) (b) mv0 l cos q which it broke off. How high above the
2 point does it rise before starting to fall?
(d) ÊÁ ˆ˜ 0
mv0 l ml v
(c) R2w 02 2R2w 02
2cos q Ë 3¯ l (a) h = (b) h =
g g
Passage for Q: 19–21
R2w 02 R2w 02
A rod AB of mass M and length L is lying on (c) h = (d) h =
2g 3g
a horizontal frictionless surface. A particle
614പWŚLJƐŝĐƐĨŽƌ//dͲ:͗DĞĐŚĂŶŝĐƐ//

24. What is the final angular momentum


and energy of the disc?
P
1
(a) MR w0, MR2w2
2
2
1
(b) mR2 w0, mR2w0 28. The angular velocity of the ring is
2
v v
ÊM ˆ ÊM ˆ (a) (b)
(c) Á - m˜ R2w 0 , Á - m˜ R2w 02 R 2R
Ë 2 ¯ Ë 2 ¯

1 ÈÊ M ˘ v 2v
ÊM ˆ ˆ (c) (d)
(d) Á - m˜ R2w 0 , ÍÁ - m˜ R2w 02 ˙ 2R R
Ë 2 ¯ Ë
2Î 2 ¯ ˚
29. The kinetic energy of ring is
Passage for Q: 25–27
A cylinder of mass 6 kg is suspended 1
(a) mv2 (b) mv2
through two ideal strings wrapped around 2
it symmetrically as shown in the figure. The
3
strings are taut and the cylinder is initially at (c) mv2 (d) 2mv2
rest. Take g = 10 m/s2. 2
30. The work done by friction on the ring
in a displacement pR is
(a) mmg ¥ R (b) mmg ¥ 2pR
(c) zero (d) none of
these
25. The acceleration of the cm of the
cylinder is Passage for Q: 31–33
A disc rolls over a horizontal surface with
20 velocity and acceleration as shown in the
(a) m/s2 (b) 10 m/s2
3 figure P1 and P2 are two points on the disc
10 (OP1 = OP2 = R/2)
(c) m/s2 (d) 12 m/s2
3 a

26. The tension in one of the strings is


(a) 60 N (b) 40 N P2
O v, a
(c) 10 N (d) 24 N P1
27. The velocity of the centre of the
cylinder after at falls through a height
7.5 m is
31. The speed of point P1 with respect to
(a) 10 3 m/s (b) 10 m/s
P2 is
(c) 6 5 m/s (d) 5 3 m/s
R 3
Passage for Q: 28–30 (a) w +v (b) Rw
2 2
A ring of mass m and radius R is rolling on a
R R
rough horizontal surface (coefficient friction (c) w (d) w
m) with constant velocity. The velocity of the 2 3
point P is v.
ZŽƚĂƟŽŶപ615

32. If the disc has mass m and radius R mR2


then torque acting on the disc about (c) a
3
centre of mass of the given instant is
(d) data insufficient
3
(a) mR2a 33. The magnitude of acceleration of the
2
point of disc, which is in contact with
mR2 ground is
(b) a (a) zero (b) a = Ra
2
(c) 2a (d) none of
these
Matrix Matching
34. Match Column I with Column II and select the correct answer:
Column I Column II
(Quantity) (Expression)
 
A. Angular momentum (p) r ¥ (mv )
1 2
B. Impulse (q) Iw
2
 
C. Torque (r) r ¥ F

D. Energy (s) mDv
35. Match the following columns:
Column I Column II
(Object) (Moment of inertia about any axis)
mR2
A. ring (p)
4
mR2
B. sphere (q)
2
3
C. disc (r) mR2
2
D. cylinder (s) 2 mR2
36. Match Column I (Body rolling on a surface without slipping) with Column II (Ratio
of Translational energy to Rotational energy) and select the correct answer using the
codes given below:
Column I Column II
A. Circular ring (p) 1/2
B. Circular disc (q) 1
C. Solid sphere (r) 3/2
D. Spherical shell (s) 5/2
616പWŚLJƐŝĐƐĨŽƌ//dͲ:͗DĞĐŚĂŶŝĐƐ//

37. Match the following columns:


Column I Column II
(Devices)
A. A planet around the sun in circular orbit (p) Angular momentum constant
B. A planet around the sun in elliptical orbit (q) Angular momentum changes

C. (r) Kinetic energy constant

A small particle is rotating in the horizontal


plane with decreasing radius in the
device shown

D. (s) Kinetic energy changes

A small particle is rotating in the horizontal


plane with decreasing radius in the
device shown
38. For the following statements, except gravity and contact force between the contact
surfaces, no other force is acting on the body.
Column I Column II
A. When a sphere is in pure rolling on (p) Upward direction
a fixed horizontal surface
B. When a cylinder is in pure rolling (q) vcm > Rw
on a fixed inclined plane
C. When a cylinder is in pure rolling (r) vcm < Rw
down a fixed incline plane, friction
force acts is
D. When a sphere of radius R is rolling (s) No frictional force acts
with slipping on a fixed horizontal
surface, the relation between vcm and
w is
ZŽƚĂƟŽŶപ617

39. In each, there is sufficient friction for regular rigid uniform body to undergo pure
rolling on a rigid horizontal surface. Now match the column I and II.
Column I Column II

F
h
A. R (p) The direction of static friction may
be backward or static friction may
disc be forward or friction may be zero

F
B. R (q) The direction of static friction is
towards backward
disc

R
C. h (r) The angular acceleration will be
F
clockwise
disc

h R

D. R (s) Acceleration of the centre of mass


will be along the direction of F
disc

40. Match Column I with Column II and select the correct answer:
Column I Column II
A. Moment of inertia of a solid uniform (p) MR2
sphere about the diameter
1
B. Moment of inertia of a thin uniform (q) MR2
spherical shell about the tangent 2
5
C. Moment of inertia of a uniform disc (r) MR2
through centre of a mass and 3
perpendicular to plane of the disc
2
D. Moment of inertia of disc about (s) MR2
tangent in the plane of disc. 5
5
(t) MR2
4

41. The following figures show different bodies which are either free to rotate or translate
on smooth horizontal surface. An impulse J is given to the bodies in the direction
shown in figure. Match the columns:
618പWŚLJƐŝĐƐĨŽƌ//dͲ:͗DĞĐŚĂŶŝĐƐ//

Column I Column II
M

A. l (p) Translation

J M

Dumbell with a massless rod placed


on smooth table

l
B. (q) Rotation occurs
M
J

C. l (r) Angular momentum about CM


increases

L-shaped strip not fixed anywhere


Hinge
l

D. (s) Linear momentum increases.


l

42. A rigid body of mass M and radius R rolls without slipping on an inclined plane of
inclination q, under gravity. Match the type of body with magnitude of the force of
friction.
Column I Column II

Mg sin q
A. For ring (p)
2.5
Mg sin q
B. For solid sphere (q)
3
ZŽƚĂƟŽŶപ619

Mg sin q
C. For solid cylinder (r)
3
Mg sin q
D. For hollow spherical shell (s)
2
43. A uniform solid cylinder of mass m and radius R is placed on a rough horizontal
surface where friction is sufficient to provide pure rolling. A horizontal force of
magnitude F is applied on cylinder at different positions with respect to its centre O in
each of four situations of column-I, due to which magnitude of acceleration of centre
of mass of cylinder is a. Match the appropriate results in column-II for conditions of
column-I.
Column I Column II

F
R
A. (p) Friction force on cylinder will not
be zero

F
R/2 F
B. O (q) a =
m

F O F
C. (r) a π
m

O
D. R/2 (s) The direction of friction force acting
F
on cylinder is towards left.

Answers Key
Level 1
1. (c) 2. (c) 3. (c) 4. (c) 5. (c) 6. (b) 7. (b) 8. (c)
9. (c) 10. (d) 11. (a) 12. (d) 13. (b) 14. (d) 15. (b) 16. (d)
17. (b) 18. (c) 19. (b) 20. (a) 21. (b) 22. (b) 23. (c) 24. (d)
25. (b) 26. (d) 27. (c) 28. (c) 29. (b) 30. (a) 31. (a) 32. (a)
33. (d) 34. (c) 35. (c) 36. (b) 37. (a) 38. (d) 39. (c) 40. (c)
41. (a) 42. (c) 43. (b) 44. (c) 45. (b) 46. (a) 47. (a) 48. (b)
49. (c) 50. (a) 51. (d) 52. (c) 53. (b) 54. (c, d) 55. (d) 56. (c)
57. (b) 58. (b) 59. (c) 60. (c) 61. (d) 62. (d) 63. (c)
Level 2
1. (c) 2. (b) 3. (d) 4. (d) 5. (c) 6. (a) 7. (c) 8. (a)
9. (c) 10. (d) 11. (d) 12. (c) 13. (c) 14. (c) 15. (a) 16. (c)
620പWŚLJƐŝĐƐĨŽƌ//dͲ:͗DĞĐŚĂŶŝĐƐ//

17. (c) 18. (d) 19. (b) 20. (b) 21. (c) 22. (c) 23. (b) 24. (d)
25. (c) 26. (d) 27. (b) 28. (d) 29. (d) 30. (c) 31. (a) 32. (d)
33. (c) 34. (d) 35. (b) 36. (c) 37. (a) 38. (c) 39. (b) 40. (c)
41. (c) 42. (b) 43. (a) 44. (d) 45. (b) 46. (b) 47. (c) 48. (b)
49. (a) 50. (a) 51. (d) 52. (a) 53. (b) 54. (c) 55. (b) 56. (a)
57. (a) 58. (b) 59. (c) 60. (c)
Multiple Correct Options
1. (a, b, c, d) 2. (a, d) 3. (a, b, d) 4. (a, b, c)
5. (c, d) 6. (b, c) 7. (b, c) 8. (a, b, c, d)
9. (a, b, d) 10. (a, c) 11. (a, b, c) 12. (a, b, c)
13. (b, c) 14. (a, b, c) 15. (a, d) 16. (a, c, d)
17. (b, c) 18. (a, c) 19. (b, c) 20. (b, d)
21. (a, b) 22. (c, d)
Passages and Matrix Matching
1. (d) 2. (c) 3. (a) 4. (c) 5. (a) 6. (b) 7. (a) 8. (b)
9. (c) 10. (b) 11. (a) 12. (c) 13. (b) 14. (c) 15. (b) 16. (b)
17. (d) 18. (b) 19. (a) 20. (b) 21. (d) 22. (a) 23. (c) 24. (d)
25. (a) 26. (c) 27. (b) 28. (c) 29. (a) 30. (c) 31. (c) 32. (b)
33. (d)
34. AÆ(p); BÆ(s); CÆ(r); DÆ(q)
35. AÆ(q, r, s); BÆ(q, r, s); CÆ(p, q, r, s); DÆ(p, q, r, s)
36. AÆ(q); BÆ(s); CÆ(t); DÆ(r)
37. AÆ(p, r); BÆ(p, s); CÆ(p, s); DÆ(q, r)
38. AÆ(s); BÆ(p); CÆ(p); DÆ(q, r)
39. AÆ(p, s, r); BÆ(q, r, s); CÆ(q, r, s); DÆ(p, r, s)
40. AÆ(s); BÆ(r); CÆ(q); DÆt
41. AÆ(p, q, r, s); BÆ(p, s); CÆ(p, q, r, s); DÆ(q, r, s)
42. AÆ(s); BÆ(r); CÆ(q); DÆ(p)
43. AÆ(p, r); BÆ(p, s, r); CÆ(p, r, s); DÆ(p, r, s)

SOLUTION AND HINTS

Level 1
1. (c) For a body with constant angular velocity, there is only centripetal force, which
passes through the axis of rotation.
2. (c) I1 = 0 + md2 + m(3d)2 = 10md2
I3 = 0 + m(2d)2 + m(3d)2 = 13md2
3. (c) For the decreasing speed of the disk, there are two accelerations an and (–at) as
shown.
ZŽƚĂƟŽŶപ621

at

an

4. (c) For only radial acceleration, w must be constant.


dq
5. (c) At t = 3s the slope is negative
dt
d(t - 3)
6. (b) a = (const)
dt
7. (b) In non-inertial frame, there is a torque of pseudo force, so (B) should be changed
accordingly.
8. (c) The only force acts on the rod is gravitational force, so its c.g. falls vertically at O.
v
9. (c) w = or v = w r in which w is constant and v μ r.
r
t
10. (d) Flywheel is the wheel of large moment of inertia. So a = becomes smaller.
I
11. (a) La = 2mv ¥ l; Lb = mv ¥ l
12. (d) L1 = L3 = 0; L2 = mv ¥ 3 and L4 = mv ¥ 2
13. (b) Clearly, I2 > I1 and I1w1 = I2 w2, so w2 < w1.
14. (d) t = F ¥ moment arm; about both the ends, moment arm remains constant.
15. (b) Acceleration of sphere will be greatest and so its speed will be greatest at the
bottom.
16. (d) In this case, the acceleration of each object will be g sin q.
17. (b) The body with non-uniform speed, has two forces; normal force and tangential
force, so their resultant will not pass through axis of rotation.
18. (c) The moment of arm of F3 is greatest. So its torque will be greatest.
19. (b) As t = F2r sin q; so if q decreases, F2 should be increased.
2
mÊ lˆ
Á ˜ 2
2 Ë 2¯ mÊ lˆ
20. (a) I = + Á ˜
3 2 Ë 2¯
2
ml
=
6
2
MR Ml 2 MR2
21. (b) = +
2 12 4
fi l = 3R
m2 r m1r
22. (b) r1 = and r2 =
m1 + m2 m1 + m2 r1 r2
Moment of inertia, I = m1r12 + m2r22 m1 m2

Ê mm ˆ
= Á 1 2 ˜ r2
Ë m1 + m2 ¯
622പWŚLJƐŝĐƐĨŽƌ//dͲ:͗DĞĐŚĂŶŝĐƐ//

23. (c) t = Fr sin q = 10 ¥ 1 ¥ sin 30°


|t| |(2i + j) ¥ (3i + 4j)|
= 5 N-m
24. (d) Moment arm = =
|F| 5
5
= =1m
5
2 gh 2 gh 4
25. (b) v = = 2
= gh
I mR /2 3
1+ 1+
mR2 mR2
2
1 1 1 1 Ê vˆ
mv 2 + Iw 2 mv 2 + (mR2 ) Á ˜
2 2 2 2 Ë R¯
26. (d) f = = 2
=2
1 2 1 Ê v ˆ
Iw 2
(mR ) Á ˜
2 2 Ë R¯

27. (c) In case of sliding, v = 2gh

In case of rolling of ring,

2 gh v
v¢ = = gh =
I 2
1+
mR2
28. (c) I = 0 + 0 + mr2
3 r
= ml2
4
M, r
È2 ˘ È2 ˘
29. (b) I = 2 Í Mr 2 ˙ + 2 Í Mr 2 + Ml 2 ˙
Î5 ˚ Î5 ˚
l
È8 ˘
= M Í r 2 + 2l 2 ˙
Î5 ˚
30. (a) If y is the distance from x-axis, then
L = mvy. As y is constant, so L is constant.
31. (a) In accelerated car the net force along the inclined becomes zero.
Also no torque is acting on the sphere, so it will continue pure rolling.
mg sin q

mg tan q

q mg sin q

32. (a) vp = vc – wr and vQ = vc + wr


so vQ > vc > vp
ZŽƚĂƟŽŶപ623

33. (d) IA = Icm + mx2, so (d) is the correct option


34. (c) As side AB > BC, so IBC > IAB
35. (c) In the process, Iiwi = If wf
or (MR2)w = (MR2 + 2mR2)wf
È Mw ˘
\ wf = Í
Î M + 2m ˙˚

36. (b) v A = wr (-j)
= 10 ¥ 1 (-j) = –10 j m/s
37. (a) The angular momentum,
L = mvr sin (180° – 30°)
1
=2¥4¥3¥ = 12 kg m2/s
2
38. (d) Angular momentum mv
L = mvd (constant)
As d constant d

1 1 mv
39. (c) mv 2 + Iw 2 = mgh
2 2
1 Ê 2mR2 ˆ Ê v ˆ
2
1
or mv 2 + Á Á ˜ = mgh
2 2 Ë 5 ˜¯ Ë R ¯
Clearly h becomes free from mass of the sphere.
2
Ê l ˆ
40. (c) I = 0 + 2 m Á + m( 2 l)2
Ë 2 ˜¯
= 3 ml2
2 3
41. (a) MR2 = mr 2
5 2
2R
fi r =
15

MR2
42. (c) I = M
2
R
Ml 2 ml 2
43. (b) I = 2 =
12 6

44. (c) I = Icm + mR2


MR2 3
= + MR2 = MR2
2 2
3
\ L = Iw = MR w2
2
624പWŚLJƐŝĐƐĨŽƌ//dͲ:͗DĞĐŚĂŶŝĐƐ//

45. (b) As no torque acts about the axis of rotation, so its angular momentum remains
constant.
46. (a) The sliding tendency of point of contact of cylinder in both the cases is downward,
so friction will act in upward direction.
47. (a) Its moment of inertia will be equal to the moment of inertia of the disc. So,
MR2
I=
2

48. (b) In the process L remains constant, so
Iw = (2I)w2
w
fi w2 =
2
1 2 1 2
K1 Iw Iw
= 2 = 2
K2 1 1
I 2w 22 (2I )(w/2)2
2 2
K
\ K2 =
2
49. (c) For topple F
F ¥ l sin 60° ≥ mg ¥ l/2
fi F ≥ mg/ 3 l
N
For translation
F = mmg 60°
mmg
mg
or = mmg mg
3
1
\ m =
3
50. (a) The sliding tendency of bottommost point of the spool is backward, so friction acts
rightwards. Therefore, centre of mass of spool moves rightwards.
51. (d) mg sin 60° – (T + f) = ma ...(i)
T
and fR – TR = Ia °
f
60
sin
Ia mg
or f–T = …(ii)
R2 60°

After simplifying, we get f = mg/5


52. (c) The tendency of sliding of point of contact is downwards, so friction will act
upwards.
53. (b) Li = mvd cos q and Lf = + mvd cos q
\ DL = 2mvd cos q
ZŽƚĂƟŽŶപ625

54. (c, d)
For velocity to be vertical,
v cos a = v
y = wR q
or wR cos a = v
Ê v ˆ a
v
\ cos a = Á ˜
Ë wR¯
55. (d) The velocity of point of contact
vp = v – wR = v – v = 0
centripetal acceleration, a = v2/R
56. (c) The force on the cylinder changes periodically, so frictional force also changes
periodically.
57. (b) For smooth surface,
1
mv 2 = mgh
2
\ v = 2gh
58. (b) As the fluid spreads out, the moment of inertia of the platform increases and so
angular velocity decreases (Iw = const). When fluid falls of the platform, its spread
again increases.
59. (c) The position vector of the centre of mass at the time t is

r = i(cos 30∞) + j(sin 30∞) + k
cm
 (0.10)

= 0.866 i + 0.5 j + 0.10 k
and the total momentum of the hoop is
 
p = mv cm = (0.50)(0.50 j) = 0.25 j
  
Thus, Lorb = rcm ¥ p
= (0.866i + 0.5j + 0.10k  ) ¥ 0.25 j
 kg-m2/s
= – 0.025i + 0.216k
To find the spin angular momentum, note that every element of mass of the hoop is
at the same distance from the centre of mass r¢ = 0.10 m, and every element rotates

about the centre of mass with a velocity v ¢ (of magnitude 0.50 m/s) perpendicular

to r ¢ . Thus,
    
L = r ¢ ¥ v ¢ dm = r ¢ v ¢( -i) dm
spin Ú Ú
= - mr ¢ v ¢ i = - 0.025 i kgm2/s
60. (c) Consider a situation when the bob A has fallen through an angle q.
Loss in PE = Gain in KE

l
626പWŚLJƐŝĐƐĨŽƌ//dͲ:͗DĞĐŚĂŶŝĐƐ//

1 2
fi Iw = mgl sin q
2
2mgl sin q 2mgl sin q
fi wA = =
I ml 2
2g sinq
wA =
l
In the similar position
4 g sinq w t 1 t
wB = fi A = B = fi A = 2
l w B tA 2 tB
61. (d) The angular momentum of the system about O = 0
fi w = 0.
62. (d) In the shown frame the particle appears to be at rest.
\ Net force on it must be zero. Therefore, pseudo force must be equal and opposite
to the tension.
w
P
m

L
O

63. (c) mg – T = ma2 ...(i)


and TR = I a2
Ia 2 R
or T =
R T
mR2 a 2
or T = ...(ii)
2 R T
or a1R = (a2 – a2R) ...(iii) R

Ê mR2 ˆ
and TR = Á ˜ a1 ...(iv)
Ë 2 ¯ mg

From equations, we get the answer.

Level 2
Iam
1. (c) On the inclined plane, the minimum friction needed for pure rolling, f =
R2
2 mR2
For cylinder a = g sin q, I =
3 2
mg sin q
\ f =
3
ZŽƚĂƟŽŶപ627

Also, N = mg cos q
f tan q
\ m = =
N 3
2. (b) Taking moment of forces about O, and put St = 0

Fv
O FH

4m

60 N
A 3m

or 60 ¥ 4 – Fv ¥ 3 = 0
\ Fv = 80 N
3. (d) In case (a) and (b), the sphere will move in pure rolling motion, while in (c), the
sphere will not be in pure rolling. So some part of mechanical energy will convert
into heat by friction.

2 gh 2 gh 10
4. (d) v = = 2
= gh
I 2 mR 7
1+ 1+
mR2 5mR2
Clearly velocity of CM does not depend on the radius of the sphere.
v v
w1 = and w2 =
R 2R
5. (c) K1 + 0 = mgh fi K1 = mgh
and K2 + Krotational = mgh fi K2 = mgh – Krotational
Clearly, K1 > K2
6. (a) E1 = E2 = mgh
2 gh
7. (c) v1 = 2 gh and v2 =
I
1+
Clearly, v1 > v2 mR2

8. (a) Work done by gravitational force in both the cases is W1 = W2 = mgh


9. (c) In (c) the net force and net torque can be zero.

 
10. (d) For the equilibrium,  F = 0 and  t = 0 which is possible in A and C.
628പWŚLJƐŝĐƐĨŽƌ//dͲ:͗DĞĐŚĂŶŝĐƐ//

11. (d) At any instant, y

vA
w = x O
x w
vA
q
vA l y
=
l sin q
 x
 dJ d 
12. (c) t = = ( ai + bt 2 j) = 2btj
dt dt
 
t◊J
cos 45° =
tJ
1 (2b + j) ◊ ( ai + bt 2 j)
or =
2 2bt ¥ a 2 + b 2t 4

a
or t =
b
 a
and t = 2b ¥ j
b
MR2
13. (c) Given I =
4
MR2 3
Itangent = + MR2 = MR2
2 2
3
= [4I ] = 6I
2
14. (c) By conservation of energy, we have
1 1 1
mgh = mv 2 + I pulleyw 12 + Ishellw 22
2 2 2
2 2
1 1 Èv˘ 1 Ê 2 ˆÊ vˆ
= mv 2 + I Í ˙ + Á MR2 ˜ Á ˜
2 2 Îr˚ 2 3Ë ¯ Ë R¯

È ˘
Í mgh ˙
\ v2 = Í ˙
Ím + I + M˙
ÍÎ 2 2r 2 3 ˙˚
15. (a) For m to be stationary
T = mg N
T mN
For M: T – mN = Mw2r
or mg – mMg = Mw2r Mg
T
mg - m Mg
\ r =
Mw 2 mg
ZŽƚĂƟŽŶപ629

v vt
16. (c) w = and q = wt =
r r
The velocity of any point of the periphery is given by
q
v = 2v cos
2
vt
or v = 2v cos
2r
17. (c) In the process,
l 1
mgl + mg = Iw 2 m
2 2
3 1 Ê ml 2 ˆ
or mgl = Á + ml 2 ˜ w 2
2 2Ë 3 ¯
3
w = g/l
2
3
\ v = wl/2 = gl
4
t mgL/2 3 g
18. (d) a = = =
I mL2 2L
3
19. (b) By conservation of angular momentum about CM
R
mv = Iw
2
È Ê Rˆ ˘
2 2
Ê Rˆ O
= ÍmR2 + m Á ˜ + m Á ˜ ˙ w
ÍÎ Ë 2¯ Ë 2¯ ˙
˚ c.m. w
v mv
or w =
3R
  
20. (b) We know that, J = m(v f - v i )
or 7.5 = 15(vf – 0)
\ vf = 0.5 m/s
1 1
Energy transferred, E = mvt2 = ¥ 15 ¥ (0.5)2
2 2
= 1.9 J
21. (c) By conservation of angular momentum about CM, we have
L
Mv = Iw
2
630പWŚLJƐŝĐƐĨŽƌ//dͲ:͗DĞĐŚĂŶŝĐƐ//

2 M ( 2 L )2
= w
12
3v
or w = , counterclockwise
4L
F 1 1 F 2
22. (c) a = ; s = at 2 = t
M 2 2M

t FR 2F
a = = =
I MR2 MR
2
1 2 1 2F Ft 2
\ q = at = ¥ ¥ t2 =
2 2 MR MR
t 5R 5R 2
23. (b) (A) a = = ; and at = aR =
I I I
t ¢ TR TR2
(B) a = = ; and at¢ = aR =
I I I
As T < 5 N, so at¢ < at
2
Ê Rˆ 3
24. (d) Area of the remaining disc = p R2 - p Á ˜ = p R2
Ë 2¯ 4

M ¥ p R2 4 M
Mass of whole disc M¢ = =
3 3
p R2
4
\ Mass of the removed disc,
M
m = .
3
Moment of interia
I = I whole disc – Iround disc

Ê 4M ˆ R
2 È M (R / 2)2 M Ê R ˆ 2 ˘
=Á ˜ -Í + Á ˜ ˙
Ë 3 ¯ 2 Î3 2 3 Ë 2¯ ˚
13
= MR2
24
25. (c) vblock = vtangent = vcm + wR
= w ¥ 2R + wR = 3wR
ZŽƚĂƟŽŶപ631

L
26. (d) L = 2pR Æ R =
2p x x¢
Ê mR2 ˆ
I = Á + mR2 ˜
Ë 2 ¯ O
2
3 3 Ê Lˆ
= mR2 = ( rL) Á ˜
2 2 Ë 2L ¯
3
3 rL
=
8 p2
p (R/3)2
27. (b) The mass of the hole removed = 9m ¥ =m
p R2
I = Iwhole disc – Iremoved disc

(9m)R2 È m(R/3)2 Ê 2R ˆ ˘
2
= -Í + mÁ ˜ ˙
2 2 Ë 3 ¯ ˙
ÎÍ ˚
2
= 4 mR
Ix m1R2/2 r(p R2 )tR2
28. (d) = =
Iy m2 ( 4 R)2/2 rp ( 4 R)2 t ¥ ( 4 R)2
1 4
=
64
29. (d) IAB = Ic.m. + Md2
1.6 Ma 2
= + M ( 2 a )2
2
= 4.8 Ma2
30. (c) Iw = L (constant)
L
or w = x
I
L
=
Ê MR 2

Á 2 + mx ˜
Ë ¯
When tortoise moves along a cord, x decreases and then increases. So w first
increases and then decreases.
L/2
31. (a)
Mv
J = Iw
È Ê Lˆ ˘
2
L
or Mv = Í2 M Á ˜ ˙ w
2 Ë 2¯ ˙
ÎÍ ˚
v
\ w =
L
632പWŚLJƐŝĐƐĨŽƌ//dͲ:͗DĞĐŚĂŶŝĐƐ//

32. (d) F + f = ma ...(i)


F
and FR – fR = Ia
Ia
or F–f = f
R2
mR2 a
=
R2
or F – f = ma ...(ii)
From the above equations, f = 0.
33. (c) Torque about bottom most point is
t = F (R + r) = 2t(R + r)
Angular momentum
t t
L = Ú t dt = Ú 2t(R + r )
0 0
2
= t (R + r)
34. (d) mg sin q – f = m¥0
f
f = mg sin q ...(i)
R
Also fR = Ia q
sin a
g
mR2 a a
=
or fR = ¥
2 R
ma
or f = ...(ii)
2
From Eqs. (i) and (ii), we get
a = 2g sin q
35. (b) Velocity of plank will be 2v.
Kinetic energy of plank,
1
Kplank = M(2v)2 = 2Mv2
2
Kinetic energy of hollow cylinder,
1 1
Kcylinder = Mv 2 + Iw 2
2 2
2
1 1 Ê vˆ
= Mv 2 + ( MR2 ) Á ˜
2 2 Ë R¯
= Mv2
K plank
\ =2
Kcylinder
ZŽƚĂƟŽŶപ633

36. (c) Acceleration of sphere


f
a = = mg
m
f
Acceleration of plank f
f
a¢ = = mg
m
Acceleration of sphere relative to plank
= a + a¢ = 2mg
37. (a) Ia = Ic.m. + ma2
or mk2 = Ic.m. + ma2
\ Ic.m. = mk2 – ma2
Now, I(a + b) = Ic.m. + m(a + b)2
= mk2 – ma2 + m(a + b)2. a
a+b
38. (c) r = r0 – aq
In the process v remains constant, so
w0r0 = wr
w 0 r0 w0
or w = =
r0 - aq Ê aq ˆ
ÁË 1 - ˜¯
r
39. (b) The mass of the rectangular piece
m(lb)
m¢ = r
p r2 b
mr 2 m¢ 2 l
Moment of inertia, I = - [l + b 2 ]
2 12
mr 2 mlb 2
= - [l + b 2 ]
2 12p r 2

mÈ 2 lb 2 ˘
= r - ( l + b 2 )˙
2 ÍÎ 6p r 2 ˚

40. (c) Taking moment O, we get


F
a
Fa = Mg ¥
2 a
Mg
or F = a/2 O
2
Mg
634പWŚLJƐŝĐƐĨŽƌ//dͲ:͗DĞĐŚĂŶŝĐƐ//

f
41. (c) a = = mg
m w0
t fR m mg ¥ R v0
and a = = =
I I 2
mR2 f
5
5 mg
=
2R
Using equation of motion, we have
0 = v0 – at ...(i)
and 0 = w0 – at ...(ii)
From the above equations, we get
v0 a
=
w0 a
mg 2R
= =
5 m g/2R 5
or 5v0 = 2w0R.
l
42. (b) mv ¥ = Iw
2
Ml 2 È v ˘
= Í ˙ v
3 Î l/2 ˚ m l
M 3
\ =
m 4
  
43. (a) Angular impulse, J = I (w f - w i )

ml 2
or 10 ¥ 1 = (w f – 0)
3
2 ¥ 12
= ¥w f
3
or wf = 15 rad/s
1 2
\ K = Iw
2
1 Ê ml 2 ˆ 2
= w
2 ÁË 3 ˜¯

1 È 2 ¥ 12 ˘ 2
= Í ˙ ¥ 15
2Î 3 ˚

= 75 J.
ZŽƚĂƟŽŶപ635

44. (d) Given I = 2x2 – 12x + 15


dl
As moment of inertia is minimum about CM so =0
dx
d
or (2 x 2 - 12 x + 15) = 0
dx
or 4x – 12 = 0
or x =3
45. (b) 10 cos 30° – f = ma ...(i) 10 sin 30°
= 2a
Also, fR = Ia N
10 cos 30°
Ia
or f = 2
R
f
2 a 2 4a
= mR2 2 = ¥ 2 a = ...(ii) mg = 20 N
3 R 3 3
From the above equations, we get
f = 2 3N
Normal reaction, N = 20 + 5 = 25 N
f
\ m = = 0.08 3
N
46. (b) For spool to be stationary, SF = 0 N

or F cos q – f = 0 ...(i)
F
Also, St = 0, r
q
or Fr – f R = 0 ...(ii)
From the above equations, f
r
cos q = mg
R

47. (c) Velocity of the particle perpendicular to z-axis is v = 2(i - j) . Angular momentum
about z-axis,   
L = m( r ¥ v )
= 1[(i + j) ¥ (2i - 2j)]

or L = - 2k  - 2k  = - 4k  kg-m2/s
48. (b) Using conservation of angular momentum about a fixed point at the level of
contact point of the ring, we have
J ¥ r = Iw
= (2mr2)w
J
J
or w = r
2mr f
J
\ v = wr =
2m
636പWŚLJƐŝĐƐĨŽƌ//dͲ:͗DĞĐŚĂŶŝĐƐ//

49. (a) With respect to the ground it has only rotation, so


v1 = wr
Now using conservation of angular momentum about a fixed point at the level of
the bottom of the sphere,
mv0r = Iw
v0
2 v
= mr 2 ¥ 1 v1
5 r
2
\ v0 = v1
5
50. (a) mvr = Iw

a È ma 2 Ê a ˆ ˘
2
or mv = Í + mÁ ˙w
2 ÍÎ 6 Ë 2 ˜¯ ˙
˚
3v
\ w =
4a
51. (d) For rotational equilibrium,
l
St0 = 0
2
2 1 1 m mg l
or m mg ¥ (l sin q ) - m mg ¥ cos q = 0 3
q a
3 3 2
1 1 O
or tan q = m mg l/2
4 3
l/2
52. (a) By conservation of linear momentum, we have
mv – mv = (m + m + m)vc.m.
or vc.m. = 0
Now using conservation of angular momentum, we get
mvl = Iw
È ml 2 ˘
= Í + 2m(l/2)2 ˙ w
Î 12 ˚
12v
\ w =
7l
53. (b) For equilibrium of the block N 1/2
 1 3
ÂF =0 or N= 3-
2 2

Torque of all the forces except N about A is found equal


to zero.
3 /2 x
ZŽƚĂƟŽŶപ637

54. (c) d = l 2 + (l/2)2

ml 2 È ml 2 ˘ l
I = +Í + md 2 ˙
3 12 l
Î ˚
5
= ml2
3
55. (b) mv0 + 0 = m ¥ 0 + mv
v = v0 B
ml 2 w
Also, mv0x = Iw = w v
12
12v0 x A
or w =
l2
For end A to be stationary
l
vA = v0 - w
2
v0 x l
or 0 = v0 - 12 ¥
l2 2
l
or x =
6
56. (a) The tendency of sliding of contact point is rightwards, so frictional force acts
leftwards.
57. (a) FBD of rod will be as shown below
TB

TB
a RAH
B A
C W
L
RAV

Summation of moments about A should be zero


S MA = 0 = – TB ¥ L – (TB – W) ¥ a = 0
Wa
\ TB =
( L + a)
Summation of moments about B should be zero
SMB = 0 = (TB – W) ¥ (L – a) + RAV ¥ L = 0
Ê Wa ˆ
ÁË (L + a) - W ˜¯ ¥ (L - a) + RAV ¥ L = 0
638പWŚLJƐŝĐƐĨŽƌ//dͲ:͗DĞĐŚĂŶŝĐƐ//

Ê Wa - WL - Wa ˆ
RAV ¥ L = – Á ˜¯ ¥ (L - a)
Ë ( L + a)

WL(L - a) ( L - a)
\ RAV = =W
( L + a) ¥ L ( L + a)
58. (b) Vertical line from hinge A must pass through cm of rod A
system. 2l/3
q
OP l/2
tan q = = P
AP 2l/3
B l/2
O D
3 Ê 3ˆ
tan q = fi q = tan - 1 Á ˜
4 Ë 4¯

59. (c) \ K = 5 mv02 C

mvR = Iw + mv¢R

mR2 Ê v ¢ ˆ
or mvR = Á ˜ + mv¢R
2 Ë R¯
\ v¢ = 2v/3
60. (c) K = Kring + Kparticles

È1 1 ˘ È1 1 1 ˘
= Í mv02 + Iw 2 ˙ + Í m( 2v0 )2 + m(2v0 )2 + m( 2v0 )2 + 0 ˙
Î2 2 ˚ Î2 2 2 ˚
v0
Also, w = , I = mR2
R

Multiple Correct Options


1. (a, b, c, d)
The acceleration is shown in figure.
aR
a
v2
aR R
a O a
v2 v2
R R
v2 aR
R
aR a

2. (a, d)
v2
Normal acceleration is an = . As r increases, v also increases to keep an constant.
r
ZŽƚĂƟŽŶപ639

3. (a, b, d)
w
v
f

Frictional force opposes the linear velocity, while its torque increases angular velocity
(see figure)
4. (a, b, c)
The frictional force acts opposite to the sliding tendency of point of contact.
5. (c, d)

v0

In this case friction acts in backward direction of v0. So v0 decreases to v. The angular
Êv v ˆ
velocity will be w = Á < 0 ˜
ËR R¯
6. (b, c)
In this case IA = ICM + md2 and I A + IB = I
fi IA < IB or IA £ IB
7. (b, c)
t0 = F1r1 sin q1 – F2r2 sin q2 out of plane of the page or t0 = F2r2 sin q2 – F1r1 sin q, into
the plane of the page.
8. (a, b, c, d)
The moment of inertia of the ring can have any value greater than MR2/2.
9. (a, b, d)
  
t is perpendicular to r and F .
10. (a, c)
In the process, angular momentum
Iw = constant
As man moves away from axis of rotation, moment of inertia increase, and so w
decreases.
11. (a, b, c)
In these cases, the torque about axis of rotation is zero and so angular momentum is
constant.
12. (a, b, c)
Mass and its distribution and axis of rotation
13. (b, c)
A body can be rotated about infinite axes.
14. (a, b, c)
       
Given t = A ¥ L. Clearly t is perpendicular to L . As t is perpendicular to L and A,
 
so it will cause no change in L or A
640പWŚLJƐŝĐƐĨŽƌ//dͲ:͗DĞĐŚĂŶŝĐƐ//

15. (a, d)
1
m (2gl) = mgh fi h = l
2
16. (a, c, d)
I 0 = I1 + I2 = I3 + I4
Also, I1 = I2 and I3 + I4
17. (b, c)
If vCM is the velocity of c.m. then w
v = vCM + wR v
For no slipping, v = wR, \ vCM = 0
18. (a, c)
If sufficient length of a rough inclined plane is available, then the cylinder will start
rolling.
19. (b, c)
 
Given, v c.m. = v B
  
v C = v B + wR
  
v A = v B - wR
From the above equations, we get
  
vB = v A + vC
2
20. (b, d)
Velocity of the particle at the highest point v = 2gh

v v

h
45°

Angular momentum
L = mv ¥ h

= m 2 gh ¥ h = m 2 gh 3

v 2 sin 2 45∞ mv 3
Also, h = ; then L =
2g 4 2g
21. (a, b)
at = ar = k sin q ¥ l = kl sin q
dv
Also, = kl sin q
dt
ZŽƚĂƟŽŶപ641

dv
or v = kl sin q
ds
dv
or v = kl sin q
Rdq
v q
vdv
or Ú R = kl Ú sin q dq
0 0
v2
or = 2kl(1 – cos q)
R
22. (c, d)
f t fR m mgR 2 m g
a = = m g , and a = = = =
m I I mR2/2 R
v = 0 + at and w = w0 – at
From the above equations, we get
a
v = (w 0 - w )
a
a
Here becomes free from m.
a
Ê1 1 ˆ Ê1 ˆ
W = DK = Á mv 2 + Iw 2 ˜ - Á Iw 02 ˜ = constant
Ë2 2 ¯ Ë 2 ¯

Passages and Matrix Matching


Passage (Q. 1–3)
1. (d) f = (2M)aCM
and 2MgR – f(2R) = Ia
a R
or 2MgR – f(2R) = (4MR2) CM
( 2 R) f
or 2MgR = – 4MaCMR = 2MRaCM 2 mg
g
fi aCM =
3
aCM g
so a = =
( 2 R) 6 R
2 Mg
2. (c) f = 2M ¥ g/3 =
3
1 2
3. (a) Iw = (2M)gR
2
1
or ( 4 MR2 )w 2 = 2MgR
2
g
w =
R
Passage (Q. 4–6)
1 2 1
4. (c) kx1 = I (2w)2
2 2
642പWŚLJƐŝĐƐĨŽƌ//dͲ:͗DĞĐŚĂŶŝĐƐ//

1 2 1
and kx2 = (2I)w2
2 2
x1
\ = 2
x2
5. (a) If w¢ is the final angular velocity, then
I(2w) + 2I(w) = (I + 2I)w¢
4w
or w¢ =
3
Thus, JB = (Lf – Li)/t
= [(2I)w¢ – 2Iw]/t

È 4w ˘
= Í 2I ¥ - 2Iw ˙/t
Î 3 ˚
2Iw
=
3t
6. (b) Loss = ki – kf

È1 1 ˘ È1 ˘
= Í I (2w )2 + (2I )w 2 ˙ - Í (I + 2I )w ¢ 2 ˙
Î2 2 ˚ Î2 ˚
1 2
= Iw
3

Passage (Q. 7–9)


7. (a) Let linear velocity of the disc become zero after a time t1. Then it starts moving in
backward direction and at time t2 it comes in pure rolling. When disc starts pure
rolling its linear and angular velocities will become constant and friction will be
zero.

Rw 0
v0 =
4
w

At time t1,
Rw 0
0 = – mg t1
4

È Rw ˘
t1 = Í 0 ˙ ...(i)
Î 4m g ˚
ZŽƚĂƟŽŶപ643

At time t2,
v = mg t2
mg t2 = Rw
È Rw ˘
t2 = Í ˙ ...(ii)
Î mg ˚
2mg
w = w0 – (t1 + t2)
R
2 m g È Rw 0 Rw ˘
w = w0 – Í + ˙
R Î 4m g m g ˚
w
w = w0 – 0 – 2w
2
w0 Èw ˘
3w = or, w = Í 0 ˙
2 Î 6 ˚
Rw 0
Now from (ii), t2 =
6m g
Maximum displacement of the disc in forward direction,
2
Rw 0 Rw 0 1 Ê Rw 0 ˆ
S = ¥ - mg
4 4 m g 2 ÁË 4 m g ˜¯

R2w 02 È 1 1 ˘ R2w 02
= - =
m g ÍÎ 16 32 ˙˚ 32 m g
The displacement of the disc when it starts pure rolling
2
1 R2w 02 1 Ê Rw 0 ˆ 5 (Rw 0 )2
= - mgÁ =
32 m g 2 Ë 6 m g ˜¯ 32 ¥ 9 m g

w0R 5 (w 0 R)2
\ ¥ t3 =
6 32 ¥ 9 m g

5 Ê w0R ˆ
48 ÁË m g ˜¯
or, t3 =

25 w 0 R
Total time t1 + t2 + t3 =
48 m g
8. (b) Time after which disc starts pure rolling.
Rw 0 Rw 0 5Rw 0
t = t1 + t2 = + =
4 m g 6 m g 12 m g
644പWŚLJƐŝĐƐĨŽƌ//dͲ:͗DĞĐŚĂŶŝĐƐ//

9. (c) Angular momentum of disc after it starts pure rolling,


L = MvR + Iw
È MRw 0 R MR2 w 0 ˘
= Í + ˙
Î 6 2 6 ˚

È 1 1 ˘ Ê MR w 0 ˆ
2
= MR2w 0 Í + ˙ = Á
Î 6 12 ˚ Ë 4 ˜¯

Passage (Q. 10–12)


10. (b); 11. (a); 12. (c)
Drawing the FBD of the plank and the cylinder
F sin q
N1

F cos q
f1

mg

f1
N1

Mg

N2
f2

Equations of motion are


F cos q – f1 = ma …(1)
F sin q + N1 = mg …(2)
f1 + f2 = MA …(3)
f 1R – f 2R = Ia …(4)
A = Ra …(5)
1
4 F cos q 4 ¥ 55 ¥
a = = 2 = 10 m/s2
3 M + 8 m [(3 ¥ 1) + (8 ¥ 1)]
1
3 MF cos q 3 ¥ 1 ¥ 55 ¥ 2
f1 = = = 7.5 N
3 M + 8m 3 ¥1+ 8 ¥1
1
MF cos q 1 ¥ 55 ¥
and f2 = = 2 = 2.5 N
3 M + 8m 3 ¥ 1 + 8 ¥ 1
ZŽƚĂƟŽŶപ645

Passage (Q. 13–15)


Suppose cylinder gets rotated through an angle before leaving contact. In the process

q R cos q

mg

1 1
mgh = Mv 2 + Iw 2
2 2
1 Ê MR2 ˆ Ê v ˆ
2
1
mgR (1 – cos q) = Mv 2 + Á ˜ ÁË ˜¯
2 2Ë 2 ¯ R
3
= Mv2
4
4
\ v = gR(1 - cos q ) ...(i)
3
For the motion of the cylinder, we have
mv 2
mg cos q – N =
R
To leave the contact, N = 0,
mv 2
\ mg cos q =
R
v2
or cos q = ...(ii)
gR
13. (b) After solving Eqs. (i) and (ii), we get
4
cos q = .
7
14. (c) From Eq. (i), we get
4 gR
v =
7

Ê 4ˆ 1 1
15. (b) We have mgR Á 1 - ˜ = mv 2 + mv 2
Ë 7¯ 2 4
1 mgR
\ mv 2 =
4 7
mgR
or Krot =
7
646പWŚLJƐŝĐƐĨŽƌ//dͲ:͗DĞĐŚĂŶŝĐƐ//

Total decrease in potential energy


= mgR
mgR
\ Ktrans = mgR –
7
6
= mgR
7
K trans
\ =6
K rot

Passage (Q. 16–18)


16. (b) The force of impact at A is vertically upwards.
17. (d)

A w
A
v0 l

vA = v¢ + lw cos q
∵ collision is elastic
\ v0 = v¢ + lw cos q
  
18. (b) Angular momentum L = r ¥ mv

|L| = mv0 l cos q.

Passage (Q. 19–21)


19. (a) Let vc and w be the velocities of CM and angular velocity just after collision.
Using conservation of linear momentum, we have
mv0 = 0 + Mvc ...(i)
and by conservation of angular momentum
L
mv0 = Iw
2
L Ê ML2 ˆ
or mv0 = Á ˜w ...(ii)
2 Ë 12 ¯
From Eq. (i) and (ii)
mv0 6 mv0
vc = and w =
M ML
Since collision is completely elastic, therefore KE before collision is equal to after
collision
1 1 1
or mv02 + 0 = Mvc2 + Iw 2
2 2 2
ZŽƚĂƟŽŶപ647

1 Ê ML2 ˆ Ê 6 mv0 ˆ
2
1 1 Ê mv ˆ
or mv02 = M Á 0 ˜ vc2 + Á Á ˜
2 2 Ë M ¯ 2 Ë 12 ˜¯ Ë ML ¯

m 1 v
which gives = and vc = 0
M 4 4
20. (b)
A
v0
m

vy
vC
VC VC
w w L/6 P L/6
y
P wy VC
P

Velocity of point P immediately after collision is zero, let it be at a distance y from


CM.
\ vc – wy = 0
mv0 Ê 6 mv0 ˆ
or -Á ˜y =0
M Ë ML ¯
L
which gives y =
6
L L 2L
\ AP = + =
2 6 3
pL
21. (d) Angle rotated by rod in time 3v
0

6 mv0 p L
q = wt = ¥
ML 3v0
p
=
2
p
The rod turns through , in this interval of time. The velocity of point P in y
direction will be 2
6 mv0 L
vy = wy = ¥
ML 6
v
= 0
4
The resultant velocity of point P

v = vc2 + vy2
648പWŚLJƐŝĐƐĨŽƌ//dͲ:͗DĞĐŚĂŶŝĐƐ//

2 2
Ê v0 ˆ Ê v0 ˆ
= ÁË ˜¯ + ÁË ˜¯
4 4
v0
=
2 2
Passage (Q. 22–24)
22. (a) Kinetic energy of the disc
1 2
k = Iw 0
2
1 Ê MR2 ˆ 2 1
= w 0 = MR2w 02
2 ÁË 2 ˜¯ 4
Angular momentum
L = Iw0

MR2
= w0
2
23. (c) In the process, mechanical energy remains constant and so
1
mv 2 = mgh
2
v2
\ h =
2g
(w 0 R)2 w 02 R2
= =
2g 2g
24. (d) Final angular momentum of the broken disc
L¢ = I¢w0
Ê MR2 ˆ
= Á - mR2 ˜ w 0
Ë 2 ¯

ÊM ˆ
= Á - m˜ R2w 0
Ë 2 ¯

1
Kinetic energy K¢ = I ¢w 02
2
1Ê M ˆ 2 2
= Á - m˜¯ R w 0
2Ë 2
ZŽƚĂƟŽŶപ649

Passage (Q. 25–27)


T T

mg

25. (a) mg – 2T = maCM ...(i)


2
mR Ê ac.m. ˆ
and 2TR = Ia = Á ˜
2 Ë R ¯
maCM
or 2T = ...(ii)
2
From the above equations, we get
2 g 20
aCM = = m/s2
3 3
20

ma CM 3
26. (c) T = = = 10 N
4 4

1 Ê mR2 ˆ Ê v ˆ
2
1
27. (b) mg ¥ 7.5 = mv 2 + Á Á ˜
2 2 Ë 2 ˜¯ Ë R ¯
fi v = 10 m/s
Passage (Q. 28–30)
28. (c) If 2 vCM = v
v
\ vCM =
2
vc.m. v
Angular velocity, w = =
R 2R
1 1
29. (a) K = mv 2CM + Iw 2 = mvCM
2
2 2
2
Ê v ˆ mv 2
= m ÁË ˜ =
2¯ 2
30. (c) In pure rolling, W = f ¥ s = 0
Passage (Q. 31–33)
31. (c) vP1 = v – wR/2 and vP2 = v + wR/2
Ê mR2 ˆ
32. (b) t = Ia = Á ˜a
2 Ë 2 ¯
v
33. (d) a =
R
650പWŚLJƐŝĐƐĨŽƌ//dͲ:͗DĞĐŚĂŶŝĐƐ//

Matrix Matching
34. A Æ (p); B Æ (s); C Æ (r); D Æ (q)
  
Impulse, J = DP = mDv
35. A Æ (q, r, s); B Æ (q, r, s); C Æ p, q, r, s); D Æ (p, q, r, s)

MR2
(A) Moment of inertia of ring ≥
2
2
(B) Moment of inertia of sphere ≥ MR2
5
MR2
(C) Moment of inertia of disc of cylinder ≥
4
36. Solution is given in the theory.
37. A Æ (p, r); B Æ (p, s) C Æ (p, s); D Æ (q, r)
(A) A planet revolves around the sun in circular orbit; its angular momentum remains
constant (t = 0). As its distance from the sun is same, so speed is also same.
(B) A planet in elliptical orbit, t = 0, so angular momentum is zero. But its distance
from sun changes, so speed changes.
(C) t = F ¥ r = F ¥ 0 = 0

In the process L is constant. F
(D) In this case t = Fr,
so angular momentum does not remain constant.

F r

38. A Æ (s); B Æ (p); C Æ (p); D Æ (q, r)


(A) In pure rolling on horizontal surface, friction is zero.
(B) (C) On inclined plane, the point of contact has the tendency of sliding downwards,
so friction acts in upward direction.
(D) In case of slipping vCM > wR
39. A Æ (p, s, r); B Æ (p, r, s); C Æ (q, r, s); D Æ (p, r, s)
(A, D) The sliding tendency of point of contact of the disc may be forward or backward
depending on relative value of F and Fh.
(B, C) Sliding tendency of point of contact is forward so friction acts in backward
direction. The acceleration of CM will be in the direction of F.
40. A Æ s; B Æ r; C Æ q; D Æ t
2
(A) MI of sphere = MR2
5
2 5
(B) MI of shell about tangent = MR2 + MR2 = MR2
3 3
ZŽƚĂƟŽŶപ651

MR2
(C) MI of disc =
2
MR2 5
(D) MI of disc about tangent in its plane = + MR2 = MR2
4 4
41. A Æ p, q, r, s; B Æ p, s; C Æ p, q, r, s; D Æ q, r, s
(A), (C) Dumbell experiences a force and net torque, so it has translation and rotation.

Also J = D P , so linear momentum increase.
 
Jl = DL , so angular momentum also increases.
(B) t = 0, so it experiences translation and linear momentum increases
(D) Due to the hinge strip will not move.
42. Solution is given in the theory.
43. A Æ p, r; B Æ p, s, r; C Æ p, r, s; D Æ p, r, s
Assume friction to be absent and horizontal force F is applied at a distance x above
the centre
F
\ a = ...(1)
m
mR2
and Fx = a
2
2Fx
or Ra = ...(2)
mR
R
If a = Ra then from Eqs. (1) and (2) x =
2
F
The friction force will be zero and a =
m
R F
If a > Ra or x < , friction force is towards left and a π
2 m
R F
If a < Ra or x > , friction force is towards right and a π
2 m
4 CHAPTER

Gravitation

KEPLER’S LAWS
On the basis of observations Johannes Kepler (1571–1630) presented three famous laws about
planetary motion.
  .CYQHQTDKVU -GRNGTŏUſTUVNCY
Each planet revolves around the sun in an elliptical orbit with the sun at one of the
foci.
B
Planet

P A 2b KE = min
KE = max Perihelion S¢
S Aphelion PE = max
PE = min

C
2a

Fig. 4.1

‡ϐ‹‹–‹‘‘ˆ–‡”•
Aphelion : A planet point fartherest from the sun.
Perihelion : A planet point nearest to the sun.
Apogee : The point in the orbit of a planet fartherest from the earth.
Perigee : The point in the orbit of a planet nearest to the earth.
Mass of the earth = 6 ¥ 1024 kg
Value of G = 6.67 ¥ 10–11 N-m2/kg2
Elliptical orbit of a planet, PA = 2a = major axis
BC = 2b = minor axis
The distance of planet closest to the sun is called perihelion and farthest to the sun is
called aphelion.
'ƌĂǀŝƚĂƟŽŶപ653

2. .CYQHCTGCU -GRNGTŏUUGEQPFNCY
The radius vector drawn from the sun to C
B
planet sweeps out equal areas in equal
intervals of time, i.e., the areal velocity of a
A
planet around the sun is constant. The linear S
speed of a planet is greater when it is closer
to the sun than its linear speed when away D
from the sun.
  .CYQHRGTKQFU -GRNGTŏUVJKTFNCY Fig. 4.2
The square to the time period of revolution of a planet around the sun is proportional
to the cube of the semimajor axis of its elliptical orbit.
If T is the time period of revolution of a planet and R is the length of the semimajor
axis of it, then
T 2 R3
T2 a R3 for two different planet 12 = 13
T2 R2
–‡’•‘™ƒ”†•ƒ™‘ˆ ”ƒ˜‹–ƒ–‹‘
On seeing a falling apple, Newton was inspired
to think about the law of gravitation. He realised
that for the falling apple on his head and the
overhead the moon, the same gravitational force R ac
Moon
was responsible for both phenomena. Newton
assumed that the moon revolved around the earth
Earth
in a circular orbit of radius r (= 3.84 ¥ 108 m). v
The moon completes a revolution around the
r
earth in 27.3 days, so
T = 27.3 ¥ 24 ¥ 60 ¥ 60 = 27.3 ¥ 86400 s. Fig. 4.3

2p r 2p ¥ (3.84 ¥ 108 )
Speed of the moon around the earth v = =
T 27.3 ¥ 86400
= 1.02 ¥ 103 m/s
Centripetal acceleration of the moon towards the centre of orbit
v 2 (1.02 ¥ 10 3 )2
ac or acentripetal = =
r 3.84 ¥ 108
= 2.72 ¥ 10–3 m/s2.
Acceleration due to gravity at the surface of the earth is g = 9.8 m/s2
Newton proposed that the gravitational force should be inversely proportional to the
square of the distance. If R is the radius of the earth, then
Acceleration of the moon a 1/r 2 R2
= c = =
Acceleration of body near the earth g 1/R2 r 2
R 1
Newton knew that =
r 60
654പWŚLJƐŝĐƐĨŽƌ//dͲ:͗DĞĐŚĂŶŝĐƐ//

2
R2 Ê 1ˆ
\ ac = g = Á ˜ ¥ 9.8
r 2 Ë 60 ¯
= 2.72 ¥ 10–3 m/s2
This value is in close argument with the value obtained otherwise. Thus, verify the inverse
square law it is called Newton moon test.

ǯ    


F F
“Every particle in the universe attracts every other m1 m2
particle with a force which is directly proportional to r
the product of their masses and inversely proportional Fig. 4.4
to the square of the distance between them.”
Consider two particles of masses m1 and m2 placed at a separation of r, the force between
them is given by

Gm1m2
F=
r2
where G is called universal gravitational constant. In SI system, its value is = 6.67 ¥ 10–11
N-m2/kg2. The value of G does not depend on the nature and size of the bodies. It also
does not depend on the nature of the medium between the bodies. That is why G is called
universal gravitational constant.

‡ –‘” ‘”‘ˆƒ™‘ˆ ”ƒ˜‹–ƒ–‹‘


Consider two particles of masses m1 and m2 r r
placed at a separation r. m1 F12 F21
m2
 r̂12 r̂21
Let r12 be the unit vector from particle 1 to

particle 2, r21 be the unit vector from particle 2
to particle 1, then Fig. 4.5
 mm
F21 = -G 1 2 2 r̂12
r

The negative sign indicates that the direction of F21 is opposite to the vector r̂12 .
 Gm1m2
Similarly, F12 = – r̂21
  r2
As r̂21 = – r̂12, so F21 = – F12
Clearly, the universal law of gravitation is consistent with Newton’s third law of motion.
 
As F12 and F21 are directed towards the centres of the particles so gravitational force is the
central force.
The following points about the gravitational force are to be noted:
1. The gravitational force does not depend on the medium between the bodies. In other
words the forces between the bodies remain the same whether, they are in air or in
water, other things remain same.
'ƌĂǀŝƚĂƟŽŶപ655

2. The gravitational force between two point masses is the central force. This force has
no angular dependence. Its magnitude depends on r. We can say that gravitational
force possesses spherical symmetry.
3. For a spherically symmetric body the force on an object outside the body acts as if the
entire mass of the body is concentrated at the centre. Newton proved this important
assertion, called shell theorem in his book Principia.
4. The gravitational force is a conservative force.
5. The law of gravitation holds for point masses.
6. For bodies of finite sizes if the distance between them is much greater than their sizes,
one may take r to be the distance between their centres of mass.
7. The resultant force acting on a particle m due toa number
  of masses M1, M2, ..... Mn;
we can use the principle of superposition. Let F1 , F2 , , Fn be the individual forces
due to M1, M2, ..., Mn, each given by the law of gravitation. The resultant force on the
particle m is given by;
    n 
F = F1 + F2 + + Fn = Â Fi
i =1

This shows that each force acts independently and is uninfluenced by the other bodies.
This is called the principle of superposition.

   


Shell theorems help us to simplify the analysis of the gravitational forces in certain cases.
Newton’s Shell Theorem. This theorem gives gravitational force on a point mass due to a
spherical shell or a solid sphere. It can be stated as follows:
Shell Theorem I: If a point mass lies outside a uniform spherical shell/sphere with a
spherically symmetric internal mass distribution, the shell/sphere attracts the point mass
as if the entire mass of the shell/sphere were concentrated at its centre.
M
GMm
R F= ,r ≥R
r2
O m
P
r

Fig. 4.6 Force on an outside particle due to a shell

M
GMm
R F= ,r ≥R
r2
O P
m
r

Fig. 4.7 Force on an outside particle due to a solid sphere


656പWŚLJƐŝĐƐĨŽƌ//dͲ:͗DĞĐŚĂŶŝĐƐ//

Shell Theorem II: If a point mass lies inside a uniform spherical M


shell, the gravitational force on the point mass is zero. But if a
F=0
point mass lies inside a homogeneous solid sphere, the force on R m
the point mass acts towards the centre of the sphere. This force is r
exerted by the spherical mass situated interior to the point mass.
M
R F = 0, r < R
E m Fig. 4.8 No force acts on
Mr
r
a particle inside
a shell

GMr m GMm È Mr r 3 ˘
F= = r Í∵ = ˙
r2 R3 Î M R3 ˚
Fig. 4.9 A net force acts towards the centre on a particle inside (r < R) a solid sphere

Is gravitational shielding possible?


Gravitational shielding is not possible. If we place a particle inside a shell, the
gravitational force on the particle due to shell is zero, yet the shell does not shield the
other bodies outside it which exert gravitational force on the particle lying inside it.

   ǯ


Kepler’s First Law: The orbit of a body under the r
v
gravitational force of a massive body is a circle, el- P
lipse, parabola or a hyperbola depends on the initial r r
condition. The proof of the first law at this level is r F
not possible.
Kepler’s Second Law: Let a planet P revolve around
S
the sun. It experience a force
 GMs mp
F =– r
r2 Fig. 4.10
 
GMs mp r Ê rˆ

=– ÁË r = ˜
r3 r¯

The torque exerted on the planet P about the sun S



    È - GMs mp r ˘
t = r ¥F = r ¥Í ˙
Î r3 ˚

È - GMs mp ˘    
=Í ˙ (r ¥ r ) = 0 (As r ¥ r = 0)
Î r3 ˚
Here MS and mp are the masses of the sun and the planet respectively.
'ƌĂǀŝƚĂƟŽŶപ657


The angular momentum L of planet about the sun is given by
  
L = r ¥P
 
= r ¥ mp v

 dL 
Since t = is zero, so momentum L is constant.
dt
This shows that the angular momentum of the planet about the sun remains constant
    
both in magnitude and direction. Since the direction of L(= r ¥ p) is fixed, r and v lie in

a plane normal to L . Moreover, it can be shown that the central force under the action of
which the planet moves varies as the square of the distance between the planet and the sun
and this orbit is an ellipse.

‡’އ”ǯ•‡ ‘†ƒ™‘ˆ–Ї‘•–ƒ ›‘ˆ”‡ƒŽ‡Ž‘ ‹–›‹•ƒ‘•‡“—‡ ‡‘ˆ


‰—Žƒ”‘‡–—‘•‡”˜ƒ–‹‘
 r
Consider a planet whose radius vector r sweeps out v
an area DA in time Dt. We know that
S r r
 1   r
D r = vDt
DA = ( r ¥ Dr ) r
2
 
Since Dr = vDt
 1  
\ DA = ( r ¥ vDt) Fig. 4.11
2

DA 1  
or = (r ¥ v)
Dt 2

DA 1  
or = ( r ¥ mv )
Dt 2m
 
DA L
or =
Dt 2m

 Ê dA ˆ
Since L is constant, so that areal velocity Á is constant.
Ë dt ˜¯

Note: Conservation of angular momentum will hold for any central force.
Kepler’s Third Law: Assuming that the orbit of planet is a circle of radius r with the sun
at the centre. The sun’s gravitational attraction is responsible for the planet’s centripetal
acceleration, i.e.,
GMs mp mp v 2
=
r2 r

GMs
which gives v = .
r
658പWŚLJƐŝĐƐĨŽƌ//dͲ:͗DĞĐŚĂŶŝĐƐ//

The time period of revolution


2p r 2p r r3
T = = = 2p
v GMs GMs
r T2
3
r
or T2 = 4p2
GMs
Here [4p2/GMs] is constant. R3

\ T 2 μ r 3. Fig. 4.12


Problem 4.1 In Fig. 4.13 a sphere of a L
mass M is a distance a from one end of
a uniform rod of length L and mass m.
What is the force between them. M
Solution Take an element of width
dx of the rod at a distance x from the
centre of the spherical body. The mass
of the element, x dx

Êm ˆ
dm = Á dx˜ . Fig. 4.13
ËL ¯
The force between the spherical body and the element of mass dm
GM(dm)
dF =
x2
Êm ˆ
GM Á dx˜
ËL ¯
=
x2
The force between the body and the whole rod
Êm ˆ ( a + L)
( a + L ) GM Á dx
Ë L ˜¯ GMm x - 1
F = Ú x2
=
L (-1)
a a
( a + L)
- GMm 1
=
L xa

- GMm È 1 1˘
= -
L ÍÎ a + L a ˙˚

- GMm È a - ( a + L) ˘
=
L ÍÎ ( a + L)( a) ˙˚
'ƌĂǀŝƚĂƟŽŶപ659

- GMm È - L ˘
=
L ÍÎ ( a + L)( a) ˙˚
GMm
=
( a + L) a
GMm
= Ans.
a( a + L)
GMm
If L << a, F  .
a2

Problem 4.2 A thin rod of mass M is bent in an arc of circle of radius R, which sustains an
angle 2 q at the centre. What is its gravitational force on a particle of mass m at the centre
of curvature.
Solution Take two identical elements are as shown in Fig. dm
4.14.
ÊM ˆ M
Mass of each element, dm = Á da ˜
Ë 2q ¯
R dm
The resultant force on the particle m due to the elements
= 2dF cos a dF
G(dm)m da a
where dF = q a dF
R2 da
m q
The resultant force on the particle due to whole arc.
q
Fig. 4.14
F = Ú (2dF cos a )
0
q
G(dM )m
= Ú2 R2
cos a
0
q q
2Gm Ê M ˆ 2Gm M
2 ÚÁ
R 2q Ú0
= ◊ da ˜ cos a = 2 cos a da
Ë
R 0 2q ¯
q
GMm sin a
=
R2 q 0

GMm Ê sin q ˆ
= Á ˜ Ans.
R2 Ë q ¯
sin q
For, q Æ 0°, =1
q
GMm
\ F =
R2
660പWŚLJƐŝĐƐĨŽƌ//dͲ:͗DĞĐŚĂŶŝĐƐ//

Problem 4.3 Consider a homogeneous ring of mass M and radius R. What gravitational
attraction does it exert on a particle of mass m located at a distance x from the centre of the
ring along its axis?

r = R2 + x2

R
dF
x q m
M
q
dF

Fig. 4.15

Solution Take an element of ring of length dl, its mass


Ê M ˆ
dm = Á dl
Ë 2p R ˜¯
The force between the element of mass (dm) and particle of mass m is
G(dm)m
dF =
r2
Ê M ˆ
GÁ dl m
Ë 2p R ˜¯
=
r2
Take another element of length dl, diametrically opposite to the first element. The resultant
force due to these two elements on the particle will be
= 2dF cos q
The effective force on the particle due to one element
= dF cos q
The force on the particle due to the whole ring
2p R
F = Ú dF cos q
0
Ê M ˆ
2p R G Á dl m
Ë 2p R ˜¯
= Ú r2
cos q
0
2p R
GMm cos q x
=
r 2 (2p R)
Ú dl cos q =
r
0

GMm Ê x ˆ [l]02p R GMm Ê x ˆ (2p R - 0)


= ¥Á ˜ ¥ = Á ˜
r2 Ë r ¯ 2p R r 2 Ë r ¯ 2p R
'ƌĂǀŝƚĂƟŽŶപ661

GMmx
=
r3
GMmx
= Ans.
[R2 + x 2 ]3/2
If x = 0, F = 0, i.e., when a particle is placed at the centre of a ring, it experiences zero
force. Find the gravitational force between the two. What will the force be if x >> R and x <
< R? For what value of x is the force maximum?
Now if x >> R, F  (GmM/x2), then for a distant point ring behaves as a point mass
(which is expected) and for x << R, F  (GmM/R3)x, i.e., force varies linearly with distance
x. Furthermore, F will be maximum when (dF/dx) = 0 i.e.,
d È GmMx ˘ d
Í ˙ =0 or [x(R2 + x 2 )-3/2 ] = 0
dx Î (R2 + x 2 )3/2 ˚ dx
[as GmM π 0]
2 2 –3/2 2 2 2 –5/2
i.e., [(R + x ) –3x (R + x ) ] =0
[as (R2 + x2)–5/2 π 0]
1 3x 2
- =0
(R2 + x 2 )3/2 (R2 + x 2 )5/2
R2 + x 2 - 3x 2
=0
(R2 + x 2 )5/2
R2 – 2x2 = 0
R2 = 2x2
R2
x2 =
2
R
x =
2
[Substituting this value of x in the expression for force F, Fmax = (2GMm/3 3R2 )].
Problem 4.4 A spherical hollow is made d – R/2
in a lead sphere of radius R. Such that its
surface touches the outside surface of the
lead sphere and passes through its centre.
The mass of the sphere before hollowing
R
was M. With what gravitational force will m
the hollowed-out lead sphere attract a small
sphere of mass m, which lies at a distance d
d from the centre of the lead sphere on the
straight line connecting the centres of the Fig. 4.16
sphere and of the hollow?
662പWŚLJƐŝĐƐĨŽƌ//dͲ:͗DĞĐŚĂŶŝĐƐ//

Solution Volume of the sphere removed


3
4 Ê Rˆ
V= pÁ ˜
3 Ë 2¯
Mass of the sphere removed
3
M 4 Ê Rˆ M
M¢ = ¥ pÁ ˜
4
pR 3 3 Ë 2¯ 8
3
The force on the small sphere of mass m
= Force due to sphere of radius R with the distance d
– force due to sphere of radius R/2 with the distance (d – R/2)
GMm GM ¢m
i.e., F = 2
-
d (d - R/2)2
Ê Mˆ
GÁ ˜ m
GMm Ë 8¯
= -
d2 (d - R/2)2
È1 1 ˘
= GMm Í 2 - 2˙
ÍÎ d 8(d - R/2) ˙˚

GMm È 1 ˘
= 2 Í
1- 2˙
. Ans.
d ÍÎ 8(1 - R/2d) ˙˚

      


A body attracts the other body by the gravitational force even if they are not in direct contact.
It can be explained by the concept of field.
The space surrounding a body in which other body experiences gravitational force is
called gravitational field.
The intensity of gravitational field at any point is the force experienced by a unit
mass placed at that point provided the presence of unit mass does not affect the original
gravitational field. The gravitational field intensity is a M

vector quantity, it can be denoted by E . Its direction
is towards the mass producing field. ur
Consider a body of mass M. The intensity of r E
gravitational field due to this body at a distance r is
given by Fig. 4.17

 F
E =
m
where m is the test mass, m << M.
'ƌĂǀŝƚĂƟŽŶപ663

According to Newton’s law of gravitation,


GMm
F=
r2

–‡•‹–›‘ˆ ”ƒ˜‹–ƒ–‹‘ƒŽ ‹‡Ž††—‡–‘–Їƒ”–Š


Let us assume the earth as a uniform sphere and non-rotating. The gravitational force acting
on a particle of mass m located outside the earth at a distance r from the centre of the earth,
is given by
GMm
F=
r2
In which M is the mass of the earth.
Intensity of gravitational field is given by
F GM
Eg = = 2 ...(i) R
m r m
If a particle is released, this gravitational M F
force produces an acceleration towards
the centre of the earth. We shall call this Earth
gravitational acceleration, and it is equal to
r
F GM
ag = = 2 ...(ii) Fig. 4.18
m r
Thus, for the assumed model of the earth, intensity of gravitational field and gravitational
GM
acceleration are equal. At the surface of the earth the gravitational acceleration g = , ( r = R)
R2
Note: The gravitational acceleration ag is due to the gravitational force exerted on the particle
by the earth. It differs from free fall acceleration that would measure for a falling particle,
because the earth is not spherical and also rotating.

ˆˆ‡ –‘ƒŽ—‡‘ˆg
1. The earth is not uniform
The density of the earth varies radially. The density is nearly 13.6 ¥ 103 kg/m3 at the
inner core of the earth and 2.3 ¥ 103 kg/m3 at the outer crust. This variation in density
affects the value of g when we go inside the earth.
2. Effect to shape of the earth
The earth is not spherical. It is approximately an ellipsoid Rp
flattened at the poles and bulging at the equator. Its equatorial Re
radius is nearly 21 km greater than its polar radius. If Re and
Rp are the radii at the equator and poles, then we have
Re  (Rp + 21) km. Fig. 4.19
664പWŚLJƐŝĐƐĨŽƌ//dͲ:͗DĞĐŚĂŶŝĐƐ//

The acceleration due to gravity,


GM
ge = (Assuming the earth is not rotating)
Re2
GM
and gP =
Rp2
Since R e > R p, \ g e < g p.
3. Effect of rotation of the earth
Now suppose the earth to be a perfect sphere
of radius R and mass M. It rotates with angular w
velocity w about polar axis. As the earth rotates, N
every particle lying on its surface revolves in a r
mw 2r
horizontal circle with the same angular velocity mg (180° – l)
w. R Fnet
W E
Consider a particle of mass m lying at a point l
Equatorial
whose latitude is l. The particle revolves in plane
a circle of radius r = R cos l. The particle
has centrifugal force Fc = mw2r in addition to S
gravitational force mg.
Let g¢ be the acceleration due to gravity in the Fig. 4.20
presence of rotation of the earth, then we have
Fnet
g¢ =
m
(mg )2 + (mw 2 R cos l )2 + 2(mg )(mw 2 R cos l )cos(180∞ - l )
=
m
= [g2 + R2w4 cos2 l – 2gRw2 cos2 l]1/2 (180° – l)
mw 2r
1/2 = mw 2 R cos l
È Ê Rw 2ˆ2
2Rw 2 ˘
= g Í1 + Á cos 2
l - cos 2 ˙
l
Í Ë g ˜¯ g ˙
Î ˚ mg Fnet
As R = 6.38 ¥ 10 m and g = 9.8 m/s
6 2
Fig. 4.21
2
Rw 2 6.38 ¥ 106 Ê 2p ˆ 1
\ = ¥Á ˜¯ =
g 9.8 Ë 24 ¥ 60 ¥ 60 291
Rw 2
Since is small, so its square is very small. Thus, we can write
g
1/2
È Rw 2 2 ˘
g¢  g Í1 - 2 cos l ˙
Î g ˚
Expanding by binomial theorem, we get
È Rw 2 ˘
g¢ = g Í1 - cos 2 l ˙
Î g ˚
or g¢ = g – w R cos2 l.
2
'ƌĂǀŝƚĂƟŽŶപ665

’‡ ‹ƒŽƒ•‡•ǣ
1. At poles l = 90° ; g¢ = g.
The highest value of g will occur at poles. It is nearly 9.83 m/s2
2. At the equator l = 0°; g¢ = g – w2R
or g – g¢ = w2R
2
Ê 2p ˆ
= Á ˜ R
Ë T¯
 0.034 m/s2.
Thus, the free-fall acceleration g¢(9.78 m/s2  9.8 m/s2) measured on the equator of
the rotatory the earth is slightly less than the gravitation acceleration g.
3. The effective value of g at the equator to be zero: Let w¢ be the angular velocity of
rotation of the earth, then
g – w¢2 R = 0
g
or w¢ =
R
This value is nearly 17 times the present value of rotation.
T 24
As w¢ = 17w, \ T¢ = = 1.41 h. Thus, if time period of rotation of the earth
17 17
becomes 1.41 h, the body of the equator will fly off.

      


What are the various factors on which the value of g at any place on the earth depends?
Factors on which g depends. The value of g changes from place to place. It depends on
various factors such as (i) altitude, (ii) depth, (iii) shape of the earth, and (iv) rotation of the
earth.

B gh
   g   ȍ  Ȏ
Effect of Altitude on g. Consider the earth to be a sphere of mass M, h
radius R and centre O. Then the acceleration due to gravity at a point
A on the surface of the earth will be
A g
GM
g = ...(i)
R2 R
If gh is the acceleration due to gravity at a point B at a height h from
the earth’s surface, then O M

GM
gh = ...(ii)
( R + h )2 Earth

Fig. 4.22
666പWŚLJƐŝĐƐĨŽƌ//dͲ:͗DĞĐŚĂŶŝĐƐ//

Dividing Eq. (ii) by Eq. (i), we get


gh GM R2
= 2
¥
g (R + h) GM

R2
or gh = g ◊ ...(iii)
( R + h )2
-2
gh R2Ê hˆ
or = = Á1+ ˜
g Ê h ˆ
2 Ë R ¯
R2 Á 1 + ˜
Ë R¯
g 2h
Expanding RHS by using binomial theorem, we get h = 1 - + terms containing higher
g R
h
powers of
R
h h
If h << R, then << 1, so that higher powers of can be neglected, we get
R R
gh 2h
=1–
g R

Ê 2h ˆ
or gh = g Á 1 - ˜ ...(iv)
Ë R¯
Both Eqs. (iii) and (iv) show that the value of acceleration due to gravity decreases with
the increase in height h, that is why the value of g is less at mountains than at plains. While
solving numerical problems, Eq. (iii) should be used when h is comparable to R and Eq. (iv)
should be used when h << R.
Value of g
(i) At a height of geostationary satellite gh = 0.225 m/s2.
(ii) At a distance as the moon from the earth’s centre gh = 0.0027 m/s2.
For Your Knowledge
s 4HE DECREASE IN THE VALUE OF g at height h is
2gh
g – gh =
R
Clearly g – gh μ h
s 4HE PERCENTAGE DECREASE IN THE VALUE OF g at height h is
g - gh 2h
¥ 100 = ¥ 100%
g R
s 4HE LOSS IN WEIGHT OF A BODY AT A HEIGHT h
2mgh
= mg – mgh =
R
s !T AN ALTITUDE h = 320 km, gh = 0.9 g, i.e., the value of g decreases by 10%
s ! TYPICAL SPACE SHUTTLE ALTITUDE IS h = 400 km.
'ƌĂǀŝƚĂƟŽŶപ667

   g   A g


d
Effect of depth on g. Consider the earth to be a sphere of mass B gd
M, radius R and centre O. The acceleration due to gravity at any R
point A on the surface of the earth will be R–d

GM O
g=
R2
Assuming the earth to be a homogeneous sphere of average
density r, then its total mass will be
4
M = Volume ¥ Density = p R3 r Fig. 4.23 Effect to depth
3 on g
4 3
G ¥ pR r
\ g = 3
R2
4
or g = p GRr
3
Let gd be the acceleration due to gravity at a point B at depth d below the surface of the
earth. A body at B is situated at the surface of inner solid sphere and lies inside the spherical
shell of thickness d. The gravitational force of attraction on a body inside a spherical shell
is always zero. Therefore, a body at B experiences gravitational force due to inner shaded
sphere of radius (R – d) and mass M¢, where
4
M¢ = p ( R - d )3 r
3
GM ¢ G 4
\ gd = 2
= 2
¥ p ( R - d )3 r
( R - d) ( R - d) 3
4
or gd = p G(R - d)r
3
4
p G(R - d)r
gd R-d d
= 3 = = 1-
g 4 R R
p GRr
3
Ê d ˆ g(R - d) gr
or gd = g Á 1 - ˜ = = R–d=r
Ë R¯ R R
Clearly, the acceleration due to gravity decreases with the increase in depth d. That is why the
acceleration due to gravity is less in mines than that on the earth’s surface.

’‡ ‹ƒŽƒ•‡•ǣ
1. At the centre of the earth, d = R; \ gd = 0
2. At the surface of the earth, d = 0; gd = g.
668പWŚLJƐŝĐƐĨŽƌ//dͲ:͗DĞĐŚĂŶŝĐƐ//

Variation in value of g:
M
GM
gh = 2 , r > R R
r
GM
g = ,r=R
R2
GM g
gd = r, r < R
R3
r
*Note: Intensity of gravitational field is towards R
 
the centre of the earth, i.e., g μ (- r ) . Therefore, it
is plotted on negative slide.
Fig. 4.24
For Your Knowledge
s 4HE ACCELERATION DUE TO GRAVITY DECREASES BOTH WITH THE INCREASE IN HEIGHT AND INCREASE
in depth. So it is maximum at the surface of the earth and zero at the centre of the earth.
s $ECREASE IN THE VALUE OF g at depth d is
d
g – gd = g
R
s 0ERCENTAGE DECREASE IN THE VALUE OF g at depth d is
g - gd d
¥ 100 = ¥ 100%
g R


Problem 4.5 What is the relation between height h and depth d for the same change in g?
Solution Relation between height h and depth d for the same change in g. Acceleration
due to gravity at a height h above the earth’s surface, ++6,''
Ê 2h ˆ
gh = g Á 1 - ˜
Ë R¯
Acceleration due to gravity at a depth d below the earth’s surface,
Ê dˆ
gd = g Á 1 - ˜
Ë R¯
For the same change in g, we have
gh = gd
2h d 2h d
\ 1- = 1- or = or d = 2 h
R R R R
Hence, the acceleration due to gravity at a height h above the earth's surface will be same as that
at depth d = 2h, below the earth’s surface. But this fact holds only when h << R.
'ƌĂǀŝƚĂƟŽŶപ669

Change in value of g due to change in radius of the earth, keeping mass of the
earth constant
(i) g at the surface of the earth
GM
g =
R2
= (GM)R – 2
Dg DR
\ = -2
g R
Dg DR
or ¥ 100 = - 2 ÊÁ ˆ
¥ 100˜
g Ë R ¯
Thus, 1% decrease in radius of the earth will increase the value of g by 2%.
(ii) For, r > R
GM
We have, g = 2
r
As r is not changing, so value of g remains constant at that point. Therefore, no effect
of change in radius of the earth will occur at that position.

    


What is meant by gravitational potential energy of a body? what is the zero level of potential
energy?
Gravitational potential energy. When two bodies are placed close to one another, they
interact through the gravitational force. Due to this, they possess mutual gravitational
potential energy. When the distance between the two bodies is changed, work is done either
by the gravitational force between the two bodies or against this force. In either case, the
gravitational potential energy of the bodies changes.
The gravitational potential energy of a body is the energy associated with it due to its position in
the gravitational field of another body and is measured by the amount of work done in bringing a
body from infinity to a given point in the gravitational field of the other.
When one body lies at infinity from another body, the gravitational force on it is zero.
Consequently, its potential energy is zero. This is called zero level of potential energy.

‘–‡–‹ƒŽ‡”‰›
As we have already defined, potential energy is the capacity of a body or system to do work
by virtue of its position or configuration. In other words, it is the amount of work which a
body (or a system) will do if it is left to itself without any constraints on it.

”ƒ˜‹–ƒ–‹‘ƒŽ‘–‡–‹ƒŽ‡”‰›ȋ Ȍ
Conventional Approach
GPE of a body at its present position is the work to be done against gravitational force by
an external agency to bring it from infinity to its present position without imparting any
acceleration to it.
670പWŚLJƐŝĐƐĨŽƌ//dͲ:͗DĞĐŚĂŶŝĐƐ//


Let F = Gravitational force on the body of mass m
when it is coming extremely slowly (i.e.,
quasistatically) from • and is at present at |dr | •
the position under consideration. r
O Q P
GME m Earth
= (See Fig. 4.25)
r2
Suppose it is brought to a nearby point Q at a distance
dr from P.
Fig. 4.25
\ dW = work done by an external agency to carry it
from P to Q.
dW = Wagent = – Wgravitation at force = U

GME m
= ¥ dr
r2
\ GPE at a point distant r from the earth’s centre
r r
GM m -2
= Ú r 2E dr = GME m Ú r dr
• •
r
È r -1 ˘ È 1˘
r
= GME m Í ˙ = GME m Í- ˙
Î -1 ˚ • Î r ˚•

È 1 Ê 1ˆ˘ GME m
= GME m Í- - Á - ˜ ˙ = -
Î r Ë ¯
• ˚ r

GME m
U = GPE = -
r
GMm
U =–
r
\ GPE of two point masses m1 and m2 kept r distance apart will be

Gm1m2
GPE = -
r
The minus sign indicates that the external force to be applied to do the displacement will
not be in the direction of the displacement (from • to the final position) but will be in an
opposite direction. This is so because if the body is just shifted from • to a point towards the
earth, the rest of the job will be automatically done by the earth’s gravitational force which
will attract the body towards its centre.
Hence, the external force must be applied in the opposite direction (just slightly less than
the earth’s gravitational force at all positions) so as the body does not gather any velocity in
the process. If it gathers some velocity too then the work done will be used up in increasing
the KE of the body also and, thus, will not be equal exactly to the change in its PE.
'ƌĂǀŝƚĂƟŽŶപ671

Šƒ‰‡‹‘–‡–‹ƒŽ‡”‰›
With respect to infinity as reference level the potential energy of the h
particle at the earth’s surface, r = R
-GMm
U =
R
R
The potential energy of particle at height h
- GMm M
Uh =
R+h
The change in potential energy
DU = Uh – U Fig. 4.26

GMm È - GMm ˘
=– -
R + h ÍÎ R ˙˚

È1 1 ˘
= GMm Í -
Î R R + h ˙˚

ÈR + h - R˘
or DU = GMm Í ˙
Î R(R + h) ˚
GMmh
=
Ê hˆ
R2 Á 1 + ˜
Ë R¯

mgh Ê GM ˆ
DU = g= 2 ˜
h ÁË R ¯
1+
R
h
For h << R, Æ0
R
\ DU  mgh.
GMm
Note: The potential energy given by equation U = – is a property of the system of two
r
particles rather than of either particle alone. For particle-earth system, M >> m we often
speak of “the potential energy of the particle”.

ƒš‹— ‡‹‰Š–––ƒ‹‡†„›ƒƒ”–‹ އ


Suppose a particle of mass m is projected vertically upwards with a speed v.
If h is the maximum height attained by the particle. h
By conservation of mechanical energy
m
1 GMm GMm
mv 2 - =–
2 R R+h M, R

2GM
v2 (R + h) – (R + h) = – 2GM
R Fig. 4.27
672പWŚLJƐŝĐƐĨŽƌ//dͲ:͗DĞĐŚĂŶŝĐƐ//

v2R2 + v2Rh – 2GMR – 2GMh = – 2GMR


h(v2R – 2GM) = – v2R2
v 2 R2
h =
2GM - v 2 R
v2
=
2GM v 2
-
R2 R
v2
h =
v2
2g -
R
(i) If v = ve
or v2e = 2gR, h Æ •
v2
Special Case: If v is small, then Æ0
R
v2
\ h =
2g


Problem 4.6 A particle is fired vertically upwards with a speed of 9.8 km/s. Find the
maximum height attained by the particle. Radius of the earth = 6400 km and g at the surface
= 9.8 m/s2. Consider only the earth’s gravitation.
Solution Initial energy of particle on the earth’s surface is
1 GMm
Ei = mv 2 -
2 R
If the particle reaches up to a height h above the surface of the earth then its final energy
will only be the gravitational potential energy.
GMm
Ef = –
R+h
According to energy conservation, we have
Ei = Ef
1 GMm GMm 1 2 gR2
or mv 2 - =– or v - gR = –
2 R R+h 2 R+h
2 gR2 2 ¥ 9.8 ¥ (6400 ¥ 10 3 )2
or h = 2
-R= – 6400 ¥ 103 = (27300 – 6400) ¥ 103 = 20900 km
2 gR - v 2 ¥ 9.8 ¥ 6400 ¥ 10 3 - (9.8)2

‹‡–‘‰‡––Їƒš‹— ‡‹‰Š–
Suppose at any height y, the speed of the particle is vy, so by conservation of mechanical
energy, we have
'ƌĂǀŝƚĂƟŽŶപ673

Ê1 1 ˆ mgy vy
– Á mvy2 - mv 2 ˜ =
Ë2 2 ¯ y
1+
R
1 1 mgy
or mv 2 - mvy2 =
2 2 y y
1+ v
R
1/2
È ˘
Í 2 2 gy ˙
vy = Í v -

or
Í 1+ ˙
Î R˚ Earth
1/2
È ˘
dy Í 2 2 gy ˙
= Ív -

or
dt Í 1+ ˙ Fig. 4.28
Î R˚
h t
dy
or ÚÈ ˘
1/2
= Ú dt
0 0
Í 2 2 gy ˙
Ív - y˙
Í 1+ ˙
Î R˚
After solving the above equation, we can get t.

‡Žˆ‡”‰›‘ˆ–Їƒ”–ŠȀ ‘ˆ–Ї—Ȁ‘Ž‹†’Ї”‡
The energy possessed due to interactions between particles of the M
dr
body is called self energy. R
Consider an element of thickness dr at a radial distance r. Let m1
r
and m2 be the masses of sphere of radius r and element respectively.
If M and R are the mass and radius of the earth, then

Ê ˆ
3
Á M ˜ 4 3 Mr
Mass of whole sphere, m1 = Á ¥
˜ 3 p r =
4 3
ÁË p R ˜¯ R3
3 Fig. 4.29

Ê ˆ 4 3
3 Mr 2 dr Volume = pr
Á M ˜ 2 3
Mass of the strip, m2 = Á ˜ ¥ 4p r dr =
4 3
ÁË p R ˜¯ R3 Surface area = 4p r2
3
The potential energy of system of masses m1 and m2 is given by
-Gm1m2
dU =
r
674പWŚLJƐŝĐƐĨŽƌ//dͲ:͗DĞĐŚĂŶŝĐƐ//

The total potential energy (self energy) of the earth


R
Gm1m2
U = -Ú
0
r
Ê Mr 3 ˆ Ê 3 Mr 2 ˆ
dr ˜
= – Ë R3 ˜¯ ÁË R3

¯

0
r
R
3GM 2 4
R6 Ú0
=– r dr

R
3GM 2 r 5
=–
R6 5 0
2
3 GM
or Uself = –
5 R

m

x = R2 + r 2
Problem 4.7 Find the potential energy of the system shown
in Fig. 4.30. R

r
Solution The mass of the ring can be assumed concentrated M
2 2
at a point on the ring. Its distance from M, will be x = R +r
\ Potential energy of the system
GMm
U =-
x Fig. 4.30

GMm
=- Ans.
R2 + r 2

‘–‡–‹ƒŽ‡”‰›‘ˆ–Ї›•–‡‘ˆƒ”–‹ އ• m3
If our system contains more than two particles, we consider
each pair of particles in turn, calculate the gravitational
potential energy of that pair as if the other particles were not
there and then sum the result. The potential energy of the r13 r23
system of three particles shown in Fig. 4.31.
Ê Gm1m2 Gm1m3 Gm2 m3 ˆ
U = -Á + +
Ë r12 r13 r23 ˜¯
m1 r12 m2
Note: If there are n particles in the system, then number of
Fig. 4.31
n(n - 1) 3¥2
pairs formed are . For three-particle system, it is
= 3. 2 2
'ƌĂǀŝƚĂƟŽŶപ675

‘–‡–‹ƒŽ‡”‰›‘ˆ—Ž–‹Ǧƒ”–‹ އ•›•–‡
In case the system has n-particles, the total gravitational potential energy of the system is
given by
ÏÔ mi m j ¸Ô ÏÔ mi m j ¸Ô
U=
 Ì- G
rij ˛Ô
˝ = - G ÌÂ
rij ˛Ô
˝
ÓÔ ÓÔ
all pairs, i π j

where the summation extends to all pairs of i and j excluding the case i = j, which contributes
nothing to the potential energy of the system. We have to count all possible pairs, which
obviously means counting each pair twice, first as i, j and then j, i (i.e., 1, 2, and 2, 1) excluding
the case i = j, thus the total number counted is twice the actual number of pairs. Therefore,
to get correct pairs, we divide the expression of U by 2. Thus, we have

1 n n mi m j
U = – GÂ Â
2 i = 1 j = 1 rij
jπ1

Potential energy of a galaxy consisting of n stars, each equal to mass m and with an
average distance r.
The gravitational potential energy of n particle system is given by

1 n n mi m j
U=– GÂ Â
2 i = 1 j = 1 rij
jπi
n n
Here, mi = mj = m and there are n equal terms in  and (n – 1) terms in Â
i =1 j =1
Therefore, we get jπi
1 m2
U = - G n(n - 1) .
2 r

   


The gravitational potential at a point in the gravitational field of a body is defined as the
amount of work done in bringing a body of unit mass from infinity to that point. Thus,
gravitational potential,
[W•P ]agent
VP =
m
Gravitational potential is a scalar quantity. Its SI unit is J/kg.

”ƒ˜‹–ƒ–‹‘ƒŽ‘–‡–‹ƒŽƒ–ƒ‘‹–†—‡–‘–Їƒ”–Š‘”‹ˆ‘”‘Ž‹†’Ї”‡
The work done in bringing a body of mass m from infinity to a point at a distance r from
-GMm
the centre of the earth is W = for r > R.
r
676പWŚLJƐŝĐƐĨŽƌ//dͲ:͗DĞĐŚĂŶŝĐƐ//

W
Hence, gravitational potential, V =
m
-GMm/r M
R
=
m
-GM
or V = ,r>R
r
–Ї”ƒ•‡• V

-GM
(i) V = ,r>R
r
-GM O r
(ii) V = ,r=R R
R – Gm/R
h. bolic

È 3R - r 2 2˘
-3 GM
(iii) V = - GM Í 3 ˙, r < R 2 R
p. bolic
Î 2R ˚
3 Ê - GM ˆ
(iv) V = Á ˜,r=0
2Ë R ¯ Fig. 4.32
3
or Vcentre = Vsurface
2

‘–‡–‹ƒŽƒ–ǡr < R
Potential at point P R uur
- dr
[W•P ]agent uur
VP = = - [W•P ]gravitational force O P Q F ds r=•
m r

È W•Q + WQP ˘
= –Í ˙
ÍÎ m ˙˚
Fig. 4.33
ÈR
1    ˘ ÈR ˘  
r r
1
= - Í Ú F ◊ ds + Ú F ◊ ds ˙ = - Í Ú Fds + Ú F ¢ds˙ [F
F ◊ ds = Fds cos 0∞ = Fds]
m ÍΕ R ˙˚ m ÍΕ R ˙˚

1 È GMm ˘
R r
GMmr
= - ÍÚ 2 ( - dr ) + Ú 3
(- dr )˙
m ÍΕ r R R ˙˚

1 È GMm( - dr ) GMmr( - dr ) ˘
R r

m ÍÎ •Ú Ú
= - Í + ˙ …(1)
r2 R R3 ˙˚
for solving for solving
we take we take
it as I1 it as I2
'ƌĂǀŝƚĂƟŽŶപ677

R
I1 = – Ú GMmr - 2 dr

R
È GMmr - 2 + 1 ˘
I1 = – Í ˙
ÍÎ -1 ˙˚ •

R
È GMm ˘ GMm GMm GMm
= Í ˙ = - =
Î r ˚• R • R

r r
GMmr(- dr ) GMm rdr È GMm r 2 ˘
I2 =
R 3
= - Ú R 3
= -Í 3
Î R
˙
2 ˚R
R

È GMm 2 ˘
=– Í ( r - R 2 )˙
Î 2R 3 ˚

GMm 2 GMmR2
= - r +
2R 3 2R 3
In Eq. (1) put the value of I1 and I2

1 È GMm GMmR2 GMm r 2 ˘


= - Í + - ˙
m ÍÎ R 2R 3 2R3 ˙˚

1 È 2R2 (GMm) + R2GMm - GMmr 2 ˘


= - Í ˙
mÎ 2R 3 ˚

1 È 2R 2 + R 2 - r 2 ˘
= - GMm Í ˙
m Î 2R 3 ˚

È 3R 2 - r 2 ˘
= - GM Í 3 ˙
Î 2R ˚

È 3R 2 - r 2 ˘
VP = - GM Í 3 ˙
Î 2R ˚

’Ї”‹ ƒŽŠ‡ŽŽ‘ˆƒ••Mƒ†ƒ†‹—•R
1. Intensity of gravitational field
GM
(i) E = – ,r>R
r2
678പWŚLJƐŝĐƐĨŽƌ//dͲ:͗DĞĐŚĂŶŝĐƐ//

GM
(ii) E = – ,r=R
R2
R
(iii) E = 0, r < R.
2. Gravitational potential
GM
(i) V = – ,r>R E
r

GM O r
(ii) V = – ,r=R
R

GM V
(iii) V = – ,r<R
R O r

‘–‡–‹ƒŽ†—‡–‘ƒ‹ˆ‘”‹‰‘‹–•š‹•
Consider a differential element dm on the ring. The Fig. 4.34
potential at P due to mass dm is
Gdm
dV = – x2 + R2
( x + R2 )1/2
2
R
The total potential is
x P
Gdm
V = -Ú
( x + R2 )1/2
2

◊G Fig. 4.35
= -
( x 2 + R2 )1/2
Ú dm
GM
= -
( x 2 + R2 )1/2
The gravitational field intensity at P can be determined from
 d
E = – (V )
dx
d È GM ˘
=– Í- 2 2 1/2 ˙
dx Î ( x + R ) ˚

GMx
=-
( x + R2 )3/2
2

‘–‡–‹ƒŽ†—‡–‘ƒ‹• ‘‹–•š‹•
Surface mass density of disc is
M
s=
p R2
'ƌĂǀŝƚĂƟŽŶപ679

Now we consider a differential ring of radius r and thickness dr.


dr
Mass, dm = (2pr dr)s
2 Mr dr R
= x P
R2
The potential due to differential ring
Fig. 4.36
Gdm
dV = –
x2 + r 2
2GMr dr
=–
R2 x 2 + r 2
Total potential can be obtained by integrating dV in the limits r = 0 to r = R.

2GM R r dr
R2 Ú0 x 2 + r 2
V=–

r dr
The integral I = Ú x 2 + R2
can be solved by substitution.

Substituting x2 + r2 = t2, 2r dr = 2t dt,


t dt
I = Ú t
= t = x2 + r 2

2GM È 2 2 ˘ R
V =– x +r ˙
R2 ÍÎ ˚0
2GM
= [x - R2 + x 2 ]
R2
The field intensity at P can be calculated from
dV
E =–
dx
d È 2GM ˘
=– [ x - R 2 - x 2 )˙
dx ÍÎ R2 ˚

2GM È x ˘
= 2 Í
1- ˙
R ÍÎ R2 + x 2 ˙˚

‡Žƒ–‹‘„‡–™‡‡ ”ƒ˜‹–ƒ–‹‘ƒŽ ‹‡Ž†ƒ†‘–‡–‹ƒŽ


Gravitational potential at any point is defined as

Wagent - Wgravitational force


V= =
m m
680പWŚLJƐŝĐƐĨŽƌ//dͲ:͗DĞĐŚĂŶŝĐƐ//

 
=–
Ú F ◊ dr
m

Ê Fˆ 
= – Ú Á ˜ ◊ dr
Ë m¯
 
V = - Ú E ◊ dr
Differentiating both sides of the above expression, we get
dv
E =–
dr
For three-dimensional field, we have
- ∂V ∂V ∂V
Ex = , Ey = - and Ez = -
∂x ∂y ∂z
Thus, resultant field can be written as
 È ∂V  ∂V  ∂V  ˘
E =– Í i+ j+ k˙
Î ∂x ∂y ∂z ˚


Problem 4.8 A non-homogeneous sphere of radius R has the following density variation:
r = r 0, r £ R/3
1 R 3R
= r 0, <r£
2 3 4
1 3R
= r 0, <r£R
8 4
Find the gravitational field intensity due to the sphere at r = R/4, R/2, 5R/6, and 2R?
(Gravitational field at a point is the force experienced by a unit mass at that point.)

Solution Gravitational field at a point due to a spherical body at distance r from the centre
GM
of the body is given by E = ,
r2
where M is the mass of the sphere under consideration and is given by Ú r 4pr2 dr.
R
%CUG K  At r = R/4, point lies inside the sphere. As< R/3, hence, density is r0 (given).
4
Here, M corresponds to the mass of the sphere having radius R/4. Let this mass be M1.
R/4
\ M1 = Ú r0 4p r 2 dr
0
'ƌĂǀŝƚĂƟŽŶപ681

R/4
È r3 ˘
= 4pr0 ◊ Í ˙
Î 3 ˚0

4 ÈÊ R ˆ 3 ˘ 4 Ê Rˆ
3
= pr0 ◊ ÍÁ ˜ - 0 ˙ = pr0 Á ˜
3 ÍÎË 4 ¯ ˙˚ 3 Ë 4¯

\ Gravitational field (I1) at r = R/4 is


3
4 Ê Rˆ
G ◊ pr0 Á ˜
3 Ë 4¯
E1 = 2
Ê Rˆ
ÁË ˜¯
4
4
p G r0 R 3
3 ( 4 )2 1
or E1 = 2
¥ 3 = p G r0 R
R ( 4) 3
= 0.33 pGr0R
%CUG KK  When r = R/2, then the point also lies inside the sphere and at the point under
consideration density is r0/2. Mass of the sphere under consideration will be the mass of
the sphere having radius R/2. In this sphere density up to radius R/3 is r0 and from R/3 to
R/2 is r0/2. Hence, the mass M2 of the sphere of radius R/2 will be
R/3 R/2
r0
M2 = Ú r0 4p r 2 dr + Ú 2
◊ 4p r 2 dr
0 R/3

R/3 R/2
È r3 ˘ 4pr0 È r3 ˘
= 4pr0 Í ˙ + Í ˙ p0 p0/2 p0/8
Î 3 ˚0 2 Î 3 ˚ R/3

4 ÈÊ R ˆ 3 ˘ 2pr0 ÈÊ R ˆ 3 Ê R ˆ 3 ˘
= pr0 ÍÁ ˜ - 0 ˙ + ÍÁ ˜ - Á ˜ ˙
3 ÍÎË 3 ¯ ˙˚ 3 ÍÎË 2 ¯ Ë 3 ¯ ˙˚
3 3 3
4 Ê Rˆ 2pr0 Ê R ˆ 2pr0 Ê R ˆ R/3
= pr0 Á ˜ + -Á ˜ - Á ˜
3 Ë 3¯ 3 Ë 3¯ 3 Ë 3¯ 3R/4

R
4 È1 1 1˘
= pr0 R3 Í + - ˙
3 Î 27 16 54 ˚ Fig. 4.37
4
= pr0 R3 [0.037 + 0.063 - 0.019]
3
4
= pr0 R3 [0.081] = 0.108pr0 R3
3
682പWŚLJƐŝĐƐĨŽƌ//dͲ:͗DĞĐŚĂŶŝĐƐ//

\ Gravitational field at r = R/2 will be


G(0.108pr0 R3 )
E2 = = 0.43Gpr0 R
(R/2)2
%CUG KKK  When r = 5R/6, then the point also lies inside the sphere. Mass under consideration
3R
will be the mass of the sphere of radius 5R/6. As this point lies between the range to R
hence density around this point is r /8. 4
0
If effective mass of the sphere of radius 5R is M3, then
R/3 3 R/4 5 R/6
r0 r
M3 = Ú r0 4p r 2 dr + Ú 2
◊ 4p r 2 dr + Ú 0 4p r 2 dr
8
0 R/3 3 R/4

R/3 3 R/4 5 R/6


È r3 ˘ 4pr0 È r 3 ˘ 4pr0 È r 3 ˘
= 4pr0 Í ˙ + Í ˙ + Í ˙
Î 3 ˚0 2 Î 3 ˚ R/3 8 Î 3 ˚ R/4

4 R/3 4 r 3 R/4 4 r 5 R/6


= pr0 [r 3 ]0 + p 0 [r 3 ]R/3 + p 0 [r 3 ]3 R/4
3 3 2 3 8

4 ÈÊ R ˆ 3 ˘ 4 r ÈÊ 3R ˆ 3 Ê R ˆ 3 ˘
= pr0 ÍÁ ˜ - 0 ˙ + p 0 ÍÁ ˜ - Á ˜ ˙
3 ÍÎË 3 ¯ ˙˚ 3 2 ÍÎË 4 ¯ Ë 3 ¯ ˙˚
4 r0 ÈÊ 5R ˆ Ê 3R ˆ ˘
3 3
+ p ÍÁ ˜ - Á ˜ ˙
3 8 ÍÎË 6 ¯ Ë 4 ¯ ˙˚
4 È1 27 1 125 27 ˘
= p R 3 r0 Í + - + - ˙˚
3 Î 27 128 54 1728 512
= 0.332 p R3r0
5R
\ Gravitational field (I3) at r = will be
6
G(0.332p R3 r0 )
E3 = 2
= 0.48 Gp Rr0
Ê 5R ˆ
ÁË ˜¯
6
%CUG KX  At r = 2R, the point lies outside the sphere, hence mass (M) corresponds to the
whole mass of the sphere, which is
3R
R/3 4 R
r r
Ú r0 4p r 2 dr + Ú 20 ◊ 4p r dr + 3ÚR 80 4p r dr
2 2
M =
0 R/3
4
R/3 3 R/4 R
È r3 ˘ 4p ◊ r0 È r3 ˘ 4p ◊ r0 È r3 ˘
= 4pr0 Í ˙ + Í ˙ + Í ˙
Î 3 ˚0 2 3
Î ˚ R/3 8 Î 3 ˚ 3 R/4
'ƌĂǀŝƚĂƟŽŶപ683

3 3 3
4 Ê Rˆ 4 r Ê 3R ˆ 4 r Ê Rˆ
= pr0 Á ˜ + p 0 Á ˜ - p 0 Á ˜
3 Ë 3¯ 3 2 Ë 4 ¯ 3 2 Ë 3¯
3
4 r0 4 r Ê 3R ˆ
+ p ( R )3 - p 0 Á ˜
3 8 3 8 Ë 4 ¯
4 È1 27 1 1 27 ˘
= p R 3 r0 Í + - + - ˙ = 0.402p R3 r0
3 Î 27 128 54 8 512 ˚
\ Gravitational field (I4) at r = 2R is

G(0.402p R3 r0 )
E4 = = 0.1p GRr0
( 2 R )2

 
 3 &GſPGGUECRGXGNQEKV[ Obtain an expression for the escape velocity of a body from the surface
of the earth?
Solution Escape Velocity. If we throw a ball into air, it rises to a certain height and falls
back. If we throw it with a greater velocity, it rises to a greater height. If we throw it with a
sufficient velocity, it will never come back. It will escape from the gravitational pull of the
earth. This minimum velocity is called escape velocity.
Escape velocity is the minimum velocity with which a body must be projected vertically upwards
in order that it may just escape the gravitational field of the earth.
Expression for Escape Velocity. Consider the earth to be a sphere of mass •
M and radius R with centre O. Suppose a body of mass m lies at point P at
distance x from its centre, as shown in Fig. 4.38. The gravitational force of Q
dx
attraction on the body at P is P
GMm
F=
x2
The small work done in moving the body through small distance PQ = dx
x
against the gravitational force is given by
A
GMm
dW = Fdx = dx R
x2
The total work done in moving the body from the surface of the earth O
(x = R) to a region beyond the gravitational field of the earth (x = •) will be Earth

GMm
W = Ú dW = Ú dx Fig. 4.38
R
x2
• •
È 1˘
= GMm Ú x - 2 dx = GMm Í- ˙
R Î x ˚R

È 1 1 ˘ GMm
= GMm Í- + ˙ =
Î • R˚ R
684പWŚLJƐŝĐƐĨŽƌ//dͲ:͗DĞĐŚĂŶŝĐƐ//

1
mv2e imparted to the body
If ve is the escape velocity of the body, then the kinetic energy
2
at the surface of the earth will be just sufficient to perform work W.
1 GMm 2GM
\ mve2 = or ve2 =
2 R R
2GM
Escape velocity ve = ...(i)
R
GM
As g = or GM = gR2
R2

2 gR2
\ ve = or ve = 2gR ...(ii)
R
If r is the mean density of the earth, then
4
M = p R3 r
3
2G 4 8pr G R2
\ ve = ¥ p R3 r = ...(iii)
R 3 3

Equations (i), (ii) and (iii) give different expressions for the escape velocity of a body.
Clearly, the escape velocity does not depend on the mass of the body projected.

‘”‡ƒ„‘—–• ƒ’‡‡Ž‘ ‹–›


1. It is clear from the expression that escape velocity does not depend on the mass of the
projected body. It depends on the mass and radius of the planet from which the body
is projected. Some escape velocities are given below.

Heavenly body Escape velocity


Earth 11.2 km/s
Moon 2.3 km/s
Jupiter 60 km/s
Sun 618 km/s
Neutron star 2 ¥ 105 km/s

2. In deriving the escape velocity, from the earth we have ignored the air resistance on
the body. So in actual practice, the value of escape velocity will be greater than 11.2
km/s.
3. The escape velocity does not depend on the angle of projection from the surface of
the earth. But it depends on the rotation of the earth about its axis. It becomes easier
to attain escape velocity if the body is projected in the direction in which the launch
site is moving.
'ƌĂǀŝƚĂƟŽŶപ685

4. If the body is projected at a place, h above the earth surface, then


- GMm 1
+ mve¢ 2 = 0 + 0
( R + h) 2
2GM
or v e¢ = < ve
( R + h)
5. When body is projected from between the planets: By conservation of mechanical
energy, we have
ve
È GM1m GM2 m ˘ 1 2
–Í + 2 ˙ + mve = 0 + 0
Î 1d d ˚ 2 M1
M2
È GM1 GM2 ˘ d1 d2
or ve = 2Í + 2 ˙
Î d1 d ˚
Fig. 4.39
6. A planet will have atmosphere if the root mean
square velocity, vrms < ve.
7. When body is projected with a velocity greater than the escape velocity kve, (k > 1), then
its velocity at infinity will not be zero. It can be easily calculated by using conservation
of mechanical energy. Let v be the velocity at infinity, then mechanical energy at earth’s
surface = mechanical energy at infinity
1 GMm 1
m(kve )2 - = mv 2 + 0
2 R 2
2GM
or v = k 2ve2 -
R
2GM
As = ve
R
\ v = k 2ve2 - ve2 = ve k 2 - 1

’‡ ‹ƒŽƒ•‡•ǣ
(i) For k = 1, v = 0
(ii) For k = 2, v = 3ve


Problem 4.9 Two uniform solid spheres of equal
radii R, but mass M and 4M have a centre-to- M m v 4M
R R
centre separation 6R, as shown in Fig. 4.40. The
O N C
two spheres are held fixed. A projectile of mass
m is projected from the surface of the sphere of
mass M directly towards the centre of the second r
sphere. Obtain an expression for the minimum 6R
speed v of the projectile so that it reaches the Fig. 4.40
surface of the second sphere.
686പWŚLJƐŝĐƐĨŽƌ//dͲ:͗DĞĐŚĂŶŝĐƐ//

Solution The two spheres exert gravitational forces on the projectile in mutually opposite
directions. At the neutral point N, these two forces cancel each other. If ON = r, then
GMm G( 4 M )m
2
=
r ( 6 R - r )2
or (6R – r)2 = 4r2 or 6R – r = ± 2r
or r = 2R or – 6R
The neutral point r = – 6 R is inadmissible.
\ ON = r = 2R
It will be sufficient to project the particle m with a minimum speed v which enables it to
reach the point N. Thereafter, the particle m gets attracted by the gravitational pull of 4M.
The total mechanical energy of m at the surface of the left sphere is:
Ei = KE of m + PE due to left sphere + PE due to right sphere
1 GMm 4GMm
= mv 2 - -
2 R 5R
At the neutral point, speed of the particle becomes zero. The energy is purely potential.
\ EN = PE due to left sphere + PE due to right sphere
GMm 4GMm
=– -
2R 4R
By conservation of mechanical energy,
Ei = EN
1 GMm 4GMm - GMm 4GMm
mu2 - - = -
2 R 5R 2R 4R
u2 GM 4GM - GM 4GM
or - - = -
2 R 5R 2R 4R
u2 GM GM 4GM 4GM
or = - - +
2 R 2R 4R 5R

GM GM GM 4GM 2GM È 2 1 ˘
= - - + = -
R 2R R 5R R ÍÎ 5 4 ˙˚
u2 2GM È 8 - 5 ˘
=
2 R ÍÎ 20 ˙˚

4GM È 3 ˘ 3 GM
or u2 = =
R ÍÎ 20 ˙˚ 5 R
1/2
È 3 GM ˘
\ u = Í
Î 5 R ˙˚
'ƌĂǀŝƚĂƟŽŶപ687

Problem 4.10 The distance between the centres of two stars is 10a. The masses of these
stars are M and 16 M and their radii a and 2a respectively. A body of mass m is fired straight
from the surface of the larger star towards the smaller star. What should be its minimum
initial speed to reach the surface of the smaller star? Obtain the expression in terms of G, M
and a. ++6,''
Solution Let the gravitational field 10a
due to the two stars be zero at some
point O lying at a distance x from the 8a
centre of smaller star.
Then m
M 16M
Mm (16 M )m a O P 2a
G 2 =G
x (10 a - x )2 x
Threshold
1 16
or 2
=
x (10 a - x )2 Fig. 4.41
2 2
or 16x = (10a – x)
or 4x = ± (10a – x)
The negative sign is inadmissible, so x = 2a
The body of mass m when fired from point P lying on the surface of heavier star must
cross the threshold (the point O), otherwise it would return.
The gravitational potential energies when the body of mass m lies at positions P and O
are given by
GMm G ¥ 16 M ¥ m 65 GMm
Up = - - =-
8a 2a 8a
GMm G ¥ 16 M ¥ m 5 GMm
UO = - - =-
2a 8a 2a
\ Increase in potential energy,
5GMm 65 GMm 45 GMm
Du = Up – UO = - + =
2a 8a 8a
If v is the minimum speed with which the body is fired from P so as to reach O, then
1 45 GMm
mv 2 = DU =
2 8a
45 GM 3 5GM
or v = =
4 a 2 a

Problem 4.11 A simple pendulum has a time period T1 when on the earth’s surface, and
T2 when taken to a height R above the earth’s surface where R is the radius of the earth.
What is the value of T2/T1?
Solution Let g and g¢ be the accelerations due to gravity on the earth’s surface and at a
height R above the earth’s surface respectively. Then
688പWŚLJƐŝĐƐĨŽƌ//dͲ:͗DĞĐŚĂŶŝĐƐ//

g g g
g¢ = 2
= 2
=
Ê hˆ Ê Rˆ 4
ÁË 1 + ˜¯ ÁË 1 + ˜¯
R R
Time period of simple pendulum,
l
T = 2p
g

T2 g g
\ = = =2
T1 g¢ g/4

Problem 4.12 Find the period of oscillation of a simple pendulum of length L suspended
from the roof of a vehicle which moves without friction down an inclined plane of
inclination a.
Solution The effective value of acceleration due to gravity,
g¢ = g cos a
l l
\ T = 2p = 2p
g¢ cos a

Problem 4.13 A spherical cavity is made inside a sphere of density r. If its centre lies at
a distance l from the centre of the sphere, show that the gravitational field strength of the
field inside the cavity is
4p
E = Glr
3
Solution In Fig. 4.42, the gravitational field strength at the
centre C of the cavity will be due to the mass of the shaded solid
sphere of radius OC = l.
Mass of the shaded sphere,
l
Cavity
4 O
M = p l3 r C
3
Gravitational field strength at the centre C of the cavity is
GM G 4 3
E = = 2 ¥ pl r
l2 l 3 Fig. 4.42

4
= p Gl r
3

      


A satellite is a body which continuously revolves around a much heavier body in a stable
orbit. Moon is the natural satellite of the earth, which in turn, is a satellite of the sun. A
manmade satellite is called an artificial satellite. Aryabhata, INSAT, etc., are the India’s
artificial satellites.
'ƌĂǀŝƚĂƟŽŶപ689

Ultra high frequency television


Satellite television transmission an antenna on the
signals are sent television set picks up the signal
from the from the television
television station
station to
houses via
satellite

Fig. 4.43

”‹ ‹’އ‘ˆƒ— Š‹‰ƒƒ–‡ŽŽ‹–‡ v0


Let us throw a body horizontally from the top of the tower.
The body describes a parabolic path under the influence of h
v < v0
gravity and hits the earth’s surface. At a certain horizontal
velocity, the body will not hit the earth, but it will always be R
in the state of free fall under gravity. The body will follow a
circular path around the earth and will become a satellite of
the earth. This horizontal velocity is called orbital velocity. Earth
Hence, to establish a satellite into an orbit around the earth,
it requires:
1. A minimum vertical velocity to take the satellite to its
orbit. Launching a satellite
2. Orbital velocity to make it move in a circular orbit Fig. 4.44
around the earth.

   v0

Let a satellite revolve in a circular orbit at a height h from h


the earth’s surface. The orbital radius of satellite, r = R + h.
The centripetal force required to keep the satellite in orbit
R
mv02
is , which is provided by the gravitational pull of the M
r
earth. Therefore,
Earth
mv02 GMm
=
r r2 r=R+h

GM
or v0 = .
r Fig. 4.45
690പWŚLJƐŝĐƐĨŽƌ//dͲ:͗DĞĐŚĂŶŝĐƐ//

If g is the acceleration due to gravity on the surface of the earth, then GM = gR2.

gR2 gR2
\ v0 = =
r ( R + h)
When satellite revolves close to the earth’s surface, then h << R and R + h  R.

\ v0 = gR
As g = 9.8 m/s2 and R = 6.4 ¥ 106 m

So, v0  9.8 ¥ 6.4 ¥ 106

= 7.92 km/s
v0 = 8 km/s (say).

’‘”–ƒ–‘‹–•
GM
It is clear from the formula v0 = that
r
(i) orbital velocity is independent of the mass of the satellite
(ii) it decreases with the increase in radius of the orbit
(iii) it depends on the mass and radius of the planet about which the satellite revolves
(iv) the escape velocity of a body from the earth’s surface is ve = 2gR . The orbital velocity
of a satellite revolving close to the earth’s surface is v0 = gR. Therefore, ve = 2v0

    


It is the time taken by a satellite to complete one revolution around the earth. It is given by
Circumference of the orbit
T =
Orbital velocity
2p r 2p r
or T = =
v0 GM
r

r3 GM
or T = 2p or Angular speed w =
GM r3
2
We have, GM = gR and r = R + h
( R + h )3
\ T = 2p
gR2
When a satellite revolves close to the earth’s surface, then h << R and R + h  R
R
\ T = 2p
g
'ƌĂǀŝƚĂƟŽŶപ691

Putting g = 9.8 m/s2 and R = 6.4 ¥ 106 m, we get

6.4 ¥ 106
T = 2p
9.8
= 5078 s
= 84.6 min = 1.41 hours
Angular momentum of satellite is given by L = m GMr

   


 K  -KPGVKE'PGTI[The kinetic energy of a satellite due to its orbital motion is
1
KE = mv 2
2
2
1 È GM ˘
= mÍ ˙
2 Î r ˚
GMm
=
2r
 KK  2QVGPVKCN 'PGTI[ Because of gravitational pull of the earth, the potential energy of
the satellite is given by
GMm
U = -
r
Total mechanical energy of the satellite
E =K+U
GMm È GMm ˘
= + Í-
2r Î r ˙˚
GMm
or E =–
2r
The total energy of the satellite is negative. It indicates that the satellite is bound to
the earth. It can be concluded that
U
K =– = -E
2

    
The energy required by a satellite to leave it from the bondage of K
the earth and escape it to infinity is called binding energy. The total
Energy

r
GMm
energy of a satellite is - . In order to escape to infinity, it must E
2r U
GMm
be supplied an extra energy equal to + so that its total energy
2r
becomes zero. Hence, binding energy of a satellite Fig. 4.46
692പWŚLJƐŝĐƐĨŽƌ//dͲ:͗DĞĐŚĂŶŝĐƐ//

GMm
BE =
2r

Energy required to establish a satellite into its orbit


The energy of the satellite at the earth’s surface
GMm
Ei = –
R
The energy of the satellite into its orbit
GMm
Ef = –
2r
The energy needed to establish the satellite into its orbit
DE = Ef – Ei
GMm È GMm ˘
=– - Í-
2r Î R ˙˚
GMm GMm
=– +
2(R + h) R

GMm(R + 2 h)
=
2R(R + h)

  ȍ Ȏ


A satellite which appears stationary to an observer on the earth’s surface is called
geostationary satellite. When such a satellite is used for communication purposes, it is
called communication satellite. The first communication satellite was sent by USA in space
in 1962 named Telstar.

‡ ‡••ƒ”›‘†‹–‹‘•ˆ‘”ƒ ‡‘•–ƒ–‹‘ƒ”›ƒ–‡ŽŽ‹–‡
1. The orbital plane of the geostationary satellite must lie in the equatorial plane of the
earth.
2. Its sense of rotation should be same as that of the earth, i.e., from west to east.
3. Its period of revolution around the earth is equal to that of the period of rotation of
the earth about its axis, i.e., 24 hours.
Note: For the stability of satellite into circular orbit, there should not be any torque produced
by the force acting on it about the centre of the orbit, when the centre of orbit of satellite does
not pass through the centre of the planet, the gravitational pull will cause a torque about the
centre and the satellite falls on the planet.

‡‹‰Š–‘ˆ ‡‘•–ƒ–‹‘ƒ”›ƒ–‡ŽŽ‹–‡
The time period of revolution of the satellite is given by
'ƌĂǀŝƚĂƟŽŶപ693

( R + h )3
T = 2p
gR2
1/3
È T 2 gR2 ˘
or h = Í 2 ˙
-R
ÍÎ 4p ˙˚
Here T = 24 h = 86400 s, R = 6.4 ¥ 106 m, g = 9.8 m/s2
\ h = 35920 ¥ 103m
= 36000 km (say)
Orbital radius r = R + h = 6800 + 35923  42000 km.

  


Q.1. From an inertial frame in space, we watch two identical uniform spheres falling
towards one another owing to their mutual gravitational attraction. Approximate
their initial speed as zero and take the initial gravitational potential energy of the two-
sphere system as Ur. When the separation between the two spheres is half the initial
separation, what is the kinetic energy of each sphere?
Ans. M1 = M2
U1 + K 1 = U2 + K 2
Gm2 Gm2 Ê1 ˆ
- +0 = - + 2 Á mv 2 ˜
R R/2 Ë2 ¯
Ui
or – Ui = – 2Ui + 2K or 2K = – Ui or K=–
2
Q.2. Calculate the gain in the potential energy of an object of mass 2 kg. When it is raised
from the surface of the earth to a height equal to the radius R of the earth.

Ans. DU = mgh mgh R mg R ◊ R mgR 2


= = = = gR = gR
1 + h/R h + R R+R 2 2
Q.3. Calculate the gravitational potential energy of a system of n point masses each of mass
m and separated from one another by the same effective distance r.
n(n - 1) Gm2
Ans. U = –
2 r
Q.4. At what rate should the earth rotate so that the apparent g at the equator becomes
zero? What will be the length of the day in this situation?
Ans. 84.6 min = T.
Q.5. At what height above and below the earth’s surface the gravitation field intensity has
same magnitude. $+65#6
Ans. gd = gh
Ê hˆ g
g Á1- ˜ =
Ë R¯ Ê hˆ
2

ÁË 1 + ˜

694പWŚLJƐŝĐƐĨŽƌ//dͲ:͗DĞĐŚĂŶŝĐƐ//

Ê
2
hˆ Ê hˆ Ê h2 ˆ Ê hˆ
or ÁË 1 + ˜¯ ÁË 1 - ˜¯ = 1 or Á 1 - 2˜Á
1+ ˜ = 1
R R Ë R ¯Ë R¯
h2 h h3 h2 h
or 1- + - =1 or + -1 = 0
R2 R R3 R2 R
h Ê 5 - 1ˆ Ê 5 - 1ˆ
or = Á ˜ or h= Á ˜R
R Ë 2 ¯ Ë 2 ¯
Q.6. Calculate the gravitational potential and field due to a point mass of 100 kg at a
distance of 1 m.
GM G(100) GM G(100)
Ans. g = 2
= 2
= 100 G; V = - =- = -100 G
R 1 R 1
Q.7. You move a ball of mass m away from a sphere of mass M.
(a) Does the gravitational potential energy of the ball–sphere increase or decrease?
(b) Is positive or negative work done by the gravitational force between the ball and
the sphere?
Ans. (a) Increase (b) Negative.
Q.8. A body stretches a spring by a particular length at the earth’s surface at the equator. At
what height above the south pole will it stretch the same spring by the same length?
Assume the earth to be spherical. '#/%'6
Ans. (gpole)h = gequator

Ê 2h ˆ
g Á 1 - ˜ = g – w 2R
Ë R¯
2gh w 2 R2
= w2 R or h = ª 10 km
R 2g
Q.9. Find the minimum colatitude which can directly receive a signal l
R l
from a geostationary satellite. ,%'%'
7R
Ans. From geometry
1
sin l = ª 0.15
7 Fig. 4.47
Q.10. The escape speed of a projectile on the earth’s surface is 11.2 km/s–1. A body is
projected out with thrice this speed. What is the speed of the body far away from the
earth? Ignore the presence of the sun and other planets. $+65#6

Ans. v = v 2 - ve2 = (3ve )2 - ve2 = 2 2 ve

Q.11. Calculate the velocity of a satellite which is revolving around the earth in a circular
orbit at a distance of 6400 km above the surface of the earth.
'ƌĂǀŝƚĂƟŽŶപ695

mv 2 GMm
Ans. =
r r2
v2 GM R2 gR2
= 2 2 = 2
r R r r

gR2 9.81 ¥ (6.4 ¥ 106 )2


v = = = 5602 ms–1 = 5.6 km s–1
r 2 ¥ 6.4 ¥ 106

 
A black hole is a body from the surface of which nothing can escape, even light. From
Einstein’s theory of relativity, speed of any object cannot exceed the speed of light, c = 3 ¥
108 m/s. Thus, c is the upper limit to the projectile’s escape velocity. Hence, for a body to
be a black hole,
ve ≥ c

2GM
or ≥c
R
2GM
or R £
c2


Problem 4.14 A mass 6 ¥ 1024 kg (= mass of the earth) is to be compressed in a sphere in
such a way that the escape velocity from its surface is 3 ¥ 108 m/s (equal to that of light).
What should be the radius of the sphere?
Solution As ve = (2GM/R , R = (2GM/ve2 )
2 ¥ 6.67 ¥ 10 - 11 ¥ 6 ¥ 10 24
\ R =
(3 ¥ 108 )2
= 9 ¥ 10–3 m = 9 mm
Note: If v < vc (= c) the particle cannot escape and as according to theory of relativity it is not
possible to accelerate a material particle to v ≥ c, so nothing can escape from such a dense
material. Such objects are known as black holes. A number of black holes exist in space and
even light cannot escape from these.
Problem 4.15 A spaceship goes into a circular orbit close to the earth’s surface. What
additional velocity must be imparted to the ship so that it is able to escape the gravitational
pull of the earth? (R = 6400 km, g = 9.8 m/s–2)
Solution The orbital velocity in a circular orbit close to the earth is v = gR .
The velocity required to escape ve = 2gR
Hence, additional velocity required is ve – v = ( 2 - 1) gR. Therefore,

ve – v = 0.414 ¥ 9.8 ¥ 6400 ¥ 10 3


= 3278.71 m/s = 3.278 km s–1
696പWŚLJƐŝĐƐĨŽƌ//dͲ:͗DĞĐŚĂŶŝĐƐ//

Problem 4.16 Find the minimum colatitude which can directly receive a signal from a
geostationary satellite.
Solution The farthest point on
the earth, which can receive signals P
from the parking orbit is the point
l
where a length is drawn on the
h l
earth’s surface from the satellite as
shown in Fig. 4.48.
Parking orbit
The colatitude l of point P can
be obtained from figure as
Re 1
sin l =
Re + h 7 AOR

We know for a parking orbit h Fig. 4.48


= 6Re
Ê 1ˆ
Thus, we have l = sin–1 Á ˜
Ë 7¯

Problem 4.17 If a satellite is revolving around the earth in a circular


Orbit
orbit in a plane containing the earth’s axis of rotation, if the angular speed
N
of satellite is equal to that of the earth, find the time it takes to move from
a point above the north pole of a point above the equator.
Solution A satellite which rotates with angular speed equal to the earth’s Re
rotation has an orbit radius 7 Re and the angular speed of revolution is
2p 2p
w= = = 7.27 ¥ 10 - 5 rad/s
T 86400 S
When satellite moves from a point above the north pole to a point AOR
above the equator, it traverses an angle p/2, time taken is
Fig. 4.49
p/2
t= = 21600 s = 6 hrs.
w
Problem 4.18 A satellite is orbiting around the
earth in an orbit in equatorial plane of radius 2Re
N Patch-1
where Re is the radius of the earth. Find the area on
P
the earth, this satellite covers for communication
purpose in its complete revolution. 725'' q
q 2Re C Re
Solution As shown in Fig. 4.50 when statelite S
revolves, it covers a complete circular belt on the
earth’s surface for communication. If the colatitude
of the farthest point on surface upto which signals
Patch-2
can be received (point P) is q then we have
Re 1 p AOR
sin q = = or q=
2Re 2 6 Fig. 4.50
'ƌĂǀŝƚĂƟŽŶപ697

During revolution, satellite leaves two spherical patches 1 and 2 on the earth’s surface at
north and south poles where no signals can be transmitted due to curvature. The areas of
these patches can be obtained by solid angles.
The solid angle subtended by a patch on the earth’s centre is
W = 2p (1 – cos q) = p (2 – 3 ) st.
Area of patch 1 and 2 is
Ap = WR2e = p(2 – 3 )R2e
Thus, total area on the earth’s surface to which communication can be made is
Ac = 4pR2e – 2Ap = 4pR2e – 2p(2 – 3 )R2e
= 2pR2e(2 – 2 + 3) = 2 3 p Re2
Problem 4.19 To what latitude does the syncoms coverage extend?
Solution The latitude of the coverage extends up to the S
tangent SP. From right D OPS, 1,'' l
h
OP
sin l =
OS
R R P
= R
R+h
l
O
6.4 ¥ 106
=
4.2 ¥ 107
or l = sin–1 (0.15) Ans.

”„‹–ƒŽ‡Ž‘ ‹–›‘ˆ ‡‘•–ƒ–‹‘ƒ”›ƒ–‡ŽŽ‹–‡ Fig. 4.51


Orbital velocity, v0 = wr
2p
= r
T
2p ¥ 42000 ¥ 10 3
= m/s
86400
= 3 km/s.

•‡‘ˆ ‡‘•–ƒ–‹‘ƒ”›ƒ–‡ŽŽ‹–‡‹
S2 S1
Ž‘„ƒŽ”ƒ•‹••‹‘
A satellite cannot make link over the entire the earth. This Earth
is because of the curvature of the earth keeps a large part
of the earth out of sight. However, three equally spaced
satellites placed in a geostationary orbit can be used
to establish link over the entire the earth. It can easily
be understood by Fig. 4.52. Such satellites are called
synchronous communication satellites (SYNCOMS). S3
Three equally spaced satellites are used for global
communication. Fig. 4.52
698പWŚLJƐŝĐƐĨŽƌ//dͲ:͗DĞĐŚĂŶŝĐƐ//

‘Žƒ”ƒ–‡ŽŽ‹–‡ Polar orbit, used by earth


observation satellites
A satellite whose orbital plane is perpendicular
to the equatorial plane is called polar satellite.
IERS is the India’s polar satellite. A polar
orbit passes over north and south poles of
the earth and has a height of 500–800 km.
The polar satellite successively passes across
the different parts of the earth’s surface as the
earth rotates about its axis. Thus, the polar
satellite can photograph entire surface of Geostationary orbit, used by Elliptical orbit, used
communications satellites by spy satellites
the earth. They are used in spying work for
military purposes. Fig. 4.53

”‘ƒ† ƒ•–‹‰‡‰‹‘‘ˆƒƒ–‡ŽŽ‹–‡
Now as we know from the height of a geostationary satellite we can easily find the area
of the earth exposed to the satellite or area of the region in which the comunication can be
made using this satellite.
Figure 4.54 shows the earth and its exposed area Axis of rotation
to a geostationary satellite. Here the angle q can be of earth
given as Re
Ê R ˆ
q = cos–1 Á e
Ë Re + h ˜¯
q
h
Re
Now we can find the solid angle W which the
exposed area subtends on the earth’s centre as
W = 2p (1 – cos q) Earth

Ê Re ˆ 2p h Fig. 4.54
= 2p Á 1 - =
Ë Re + h ˜¯ Re + h
Thus, the area of the earth’s surface to geostationary satellite is
2p hRe2
S = WR2e =
Re + h
Let’s take some examples to understand the concept in detail.
Problem 4.20 A satellite is revolving around the earth in an orbit of radius double that of
the parking orbit and revolving in the same sense. Find the periodic time duration between
two instants when this satellite is closest to a geostationary satellite.
Solution We know that the time period of revolution of a satellite is given as
4p 2 3
T2 = r [Kepler’s III law]
GMe
'ƌĂǀŝƚĂƟŽŶപ699

For satellite given in problem and for a geostationary satellite we have


3 3
T1 Êr ˆ Êr ˆ
= Á 1˜ or T1 = Á 1 ˜ ¥ T2 = (2)3 ¥ 24 = 192 hours
T2 Ë r2 ¯ Ë r2 ¯

ƒ— Š‹‰‘ˆƒ–‡ŽŽ‹–‡• Point of


projection
Depending upon the launching speeds,
a projectile can have trajectories of v < v 0 , v < (gr )
different shapes as shown in Fig. 4.55. Speed
In analysing the motion of a satellite
we will consider only the earth’s
gravitational force on it.
Earth
Thus, if projection speed v is,

la
bo
er
v > 2gr = Hyperbolic trajectory
yp
H
v = 2gr = Parabolic trajectory v > (2gr ), v > v e
Circular
2gr > v > gr = Ellipse
v = (gr ) = 8 km s -1 = v 0
v = gr = Circle la
Parabo

v< gr = It will come back on the


earth’s surface
Note: v is the projection speed v = (2gr ) = 11km s -1, v = v e Ellipse
v0 is the orbital velocity (2gr ) > v > (gr ), v e > v > v 0
ve is the escape velocity
Fig. 4.55
ƒ–‡ŽŽ‹–‡‹ŽŽ‹’–‹ ƒŽ”„‹–
c
Eccentricity of the elliptical path e = fi c = ea
a
rp = a – c = a – ea = a(1 – e).
Using conservation of angular momen- rp ra
tum at perigee and apogee, we have
va
mvprp = mvara ...(i) Earth
P
Perigee
vp ra 1 + e Apogee
fi = = ...(ii) vp
va rp 1 - e
vp ra a + c 1 + e c a
fi = = =
va rp a - c 1 - e Fig. 4.56
c
[As rp = a – c, ra = a + c and eccentricity e = ]
a
Mechanical energy of revolving satellite remains constant
GMm 1 - GMm 1
\ – + mvp2 = + mva2
rp 2 ra 2
700പWŚLJƐŝĐƐĨŽƌ//dͲ:͗DĞĐŚĂŶŝĐƐ//

È1 1˘
or vp2 – va2 = 2GM Í - ˙
ÍÎ rp ra ˚˙
vr
We have from Eq. (i), vp = a a
rp
2
Ê va ra ˆ 2
È1 1˘
\ Á ˜ - va = 2GM Í - ˙
Ë rp ¯ ÍÎ rp ra ˙˚

Ê ra2 - rp2 ˆ È ra - rp ˘
or va2 Á 2 ˜ = 2GM Í ˙
Ë rp ¯ ÍÎ rp ra ˙˚

2GMrp
or va2 =
ra (ra + rp )
We have ra = a + c and rp = a – c, therefore,
2GM È ( a - c) ˘
v a2 =
( a + c) + ( a - c) Î ( a + c) ˙˚
Í

GM È ( a - c) ˘
=
a ÍÎ ( a + c) ˙˚

GM È (1 - e ) ˘
=
a ÍÎ (1 + e ) ˙˚

Thus, the speeds of planet at apogee and perigee are

GM Ê 1 - e ˆ
va = Á ˜
a Ë 1+ e¯

GM Ê 1 + e ˆ
vp = Á ˜
a Ë 1- e¯
va È1 - e ˘
= Í
vp Î 1 + e ˙˚


Problem 4.21 The largest and the shortest distance of the earth from the sun are r1 and r2
respectively. Calculate the distance from the sun when it is at perpendicular distance to the
major axis of the orbit drawn from the sun.
c
Solution Since e = fi c = ea
a
\ r1 = a + c = a + ea = a(1 + e)
r2 = a – c = a – ea = a(1 – e)
'ƌĂǀŝƚĂƟŽŶപ701

r2 r1
2b 2
Formula of Latus rectum =
a
1 b2
Required distance = of Latus rectum =
2 a
b2 2b 2 L Sun
b
\ L= Latus rectum
a a
We have r1 + r2 = 2a c a
and r1 – r2 = a(1 + e) – a(1 – e) = 2ae
Fig. 4.57
Êr -r ˆ
fi e = Á 1 2˜
Ë r1 + r2 ¯
b2
Also, e = 1 - b 2/a 2 fi b 2 = a 2 (1 - e 2 ) e= 1- Formula of ellipse
a2
b 2 a2
\ L = = (1 - e 2 )
a a
= a(1 – e2)
Êr +r ˆÈ Êr -r ˆ ˘
2
= Á 1 2 ˜ Í1 - Á 1 2 ˜ ˙
Ë 2 ¯ Í Ë r1 + r2 ¯ ˙
Î ˚
Ê 2r r ˆ
= Á 12 ˜ Ans.
Ë r1 + r2 ¯

 
The weight of a body is the force with which it is attracted towards the centre of the earth.
When a body is stationary with respect to the earth, its weight equals the gravity. This weight
of the body is known as its static or true weight.
We become conscious of our weight, only when our weight (which is gravity) is opposed
by some other object. Actually, the secret of measuring the weight of a body with a weighing
machine lies in the fact that as we place the body on the machine, the weighing machine
opposes the weight of the body. The reaction of the weighing machine to the body gives the
measure of the weight of the body.
The sate of weightlessness can be observed in the following situations.
1. When objects fall freely under gravity: For example, a lift falling freely, or an airship
showing a feat in which it falls freely for a few seconds during its flight, are in state
of weightlessness.
2. When a satellite revolves in its orbit around the earth: Weightlessness poses many
serious problems to the astronauts. It becomes quite difficult for them to control their
movements. Everything in the satellite has to be kept tied down. Creation of artificial
gravity is the answer to this problem.
702പWŚLJƐŝĐƐĨŽƌ//dͲ:͗DĞĐŚĂŶŝĐƐ//

3. When bodies are at null points in outer space: On a body projected up, the pull of
the earth goes on decreasing, but at the same time the gravitational pull of the moon
on the body goes on increasing. At one particular position, the two gravitational pulls
may be equal and opposite and the net pull on the body becomes zero. This is zero
gravity region or the null point and the body in question is said to appear weightless.

Šƒ‰‡‹–Ї”„‹–‘ˆƒ–‡ŽŽ‹–‡
When a satellite is transferred to a higher orbit (r2 > r1) then variation in different quantities
can be shown by the following table.

Quantities Variation Relation with r


Orbital velocity Decreases 1

r
Time period Increases T μ r3/2
Linear momentum Decreases 1

r
Angular momentum Increases Lμ r
Kinetic energy Decreases 1

r
Potential energy Increases 1

r
Total energy Increases 1

r
Binding energy Decreases 1
BE μ
r

Note: Work done in changing the orbit W = E2 – E1

Ê GMm ˆ Ê GMm ˆ
W = Á- - - r2
Ë 2r2 ˜¯ ÁË 2r1 ˜¯ M
r1 Earth
GMm È 1 1 ˘
W = Í - ˙
2 Î r1 r2 ˚
Fig. 4.58
‡‹‰Š–އ••‡••‹ƒƒ–‡ŽŽ‹–‡
A satellite, which does not produce its own gravity moves around the earth in a circular
GM
orbit under the action of gravity. The acceleration of satellite is 2 towards the centre of
the earth. r
'ƌĂǀŝƚĂƟŽŶപ703

If a body of mass m is placed on a surface inside a Satellite


satellite moving around the earth. Then forces on the body ma
are h m
r
GMm
(i) The gravitational pull of the earth = R
r2 GMm
(ii) The reaction by the surface = N Earth r2 v

GmM
By Newton’s law – N = ma
r2
GmM Ê GM ˆ
- N = mÁ 2 ˜
r2 Ë r ¯
Fig. 4.59
\ N=0
Thus, the surface does not exert any force on the body and hence its apparent weight is
zero.
A body needs no support to stay at rest in the satellite and hence all positions are equally
comfortable. Such a state is called weightlessness.
Examples under condition of weightlessness:
(i) One will find it difficult to control his movement, without weight he will tend to float
freely. To get from one spot to the other he will have to push himself away from the
walls or some other fixed objects.
(ii) As everything is in free fall, so objects are at rest relative to each other, i.e., if a table
is withdrawn from below an object, the object will remain where it was without any
support.
(iii) If a glass of water is tilted and glass is pulled out, the liquid in the shape of container
will float and will not flow because of surface tension.
(iv) If one tries to strike a match, the head will light but the stick will not burn. This is
because in this situation convection currents will not be set up which supply oxygen
for combustion.
(v) If one tries to perform simple pendulum experiment, the pendulum will not oscillate.

It is because there will not be any restoring torque and so T = 2p (L/g¢ ) = •.


[As g¢ = 0]
(vi) Condition of weightlessness can be experienced only when the mass of the satellite is
negligible so that it does not produce its own gravity.
For example, the moon is a satellite of the earth but due to its own weight it applies
gravitational force of attraction on the body placed on its surface and hence weight of the
body will not be equal to zero at the surface of the moon.


Problem 4.22 Two satellites S1 and S2 revolve around a planet in coplanar circular orbits
in the same sense. Their periods of revolution are 1 h and 8 h respectively. The radius of the
orbits of S1 is 104 km. When S2 is closest to S1 find: ++6,''
(a) the speed of S2 relative to S1, and
(b) the angular speed of S2 as observed by an astronaut in S1.
704പWŚLJƐŝĐƐĨŽƌ//dͲ:͗DĞĐŚĂŶŝĐƐ//

Solution We have T2 μ r3
2 3
Ê T1 ˆ Ê R1 ˆ R2 v1
\ ÁË T ˜¯ = ÁË R ˜¯
2 2 v2

2 3
Ê 1ˆ Ê R1 ˆ S2
ÁË ˜¯ = Á ˜ fi R2 = 4R1 = 4 ¥ 10 km
4
or R1 S1
8 Ë R2 ¯
Let v1 and v2 be the linear speeds of S1 and S2 with
respect to the planet. Then
2p R1
v1 = = 2p ¥ 10 4 km/h
T1
Fig. 4.60
2p R2
and v2 = = p ¥ 10 4 km/h
T2
(a) At the closest separation, they are moving in the same direction. Therefore, the speed
of S2 with respect to S1 is
|v2 – v1| = p ¥ 104 km/s. Ans.
(b) As seen from S1 the satellite S2 is at a distance r = R2 – R1 = 3 ¥ 10 km. At the closest
4

separation
|v - v | p ¥ 10 4
w = 2 1 =
R2 - R1 3 ¥ 10 4
p
rad/h = Ans.
3
Problem 4.23 Three particles of mass m each are placed at the three corners of an equilateral
triangle of side a. Find the work which should be done on this system to increase the side
of the triangle to 2a. ,%'%'
Solution The work done, m
W = Uf – Ui
Gmm 3Gm2
where Ui = – 3 ¥ =- a a
a a
Gmm 3 Gm2
and Uf = – 3 ¥ =- m m
2a 2 a a

3 Gm2 Ê 3Gm2 ˆ Fig. 4.61


Thus, W = - -Á-
2 a Ë a ˜¯
3Gm2
= Ans.
2a
'ƌĂǀŝƚĂƟŽŶപ705

  ǧǧ


A two-body problem, involving central forces, can be y
reduced to a one-body problem and thereby simplifying m2
F21 r r r
calculations. r = r1 - r2
Suppose we have two particles of masses m1 and m2
  r
F12
whose positions at any time are r1 and r2 with respect r2
to origin O. m1
   r
The vector distance of particle m1 from m2 is r = r1 - r2 r1
The force exerted by the second particle on the first,
 
Ê d2 r ˆ
F12 = m1 Á 21 ˜ ...(i) x
Ë dt ¯ O

and Fig. 4.62


 the force exerted by the first particle on the second,
F21 is given by
 
Ê d2 r ˆ
F21 = m2 Á 22 ˜ ...(ii)
Ë dt ¯
Both the forces act along the line joining m1 and m2. Since no external forces are acting on
the system, and according to the with Newton’s third law of motion, we have
  
F12 = - F21 = F
The above equations now can be written as
 
Ê d2 r ˆ
m1 Á 21 ˜ = F ...(iii)
Ë dt ¯
 
Ê d2 r ˆ
and m2 Á 22 ˜ = - F ...(iv)
Ë dt ¯
Multiplying Eq. (iii) by m2 and Eq. (iv) by m1 and subtracting Eq. (iv) from Eq. (iii), we get
  
Ê d2 r d2 r ˆ
m1m2 Á 21 - 22 ˜ = (m1 + m2 )F
Ë dt dt ¯
d2   Ê m + m2 ˆ 
or ( r - r2 ) = Á 1 F
dt 2 1
Ë m1m2 ˜¯
   m1m2
Substituting r1 - r2 = r and = m, is called reduced mass, we have
m1 + m2
  
d2 r F d2 r 
or or m =F
dt 2 m dr 2

This equation will be seen as the equation of motion of a particle of mass m at a vector

distance r from one of the particles relative to other. In other words, take m2 as a fixed
706പWŚLJƐŝĐƐĨŽƌ//dͲ:͗DĞĐŚĂŶŝĐƐ//

 
particle which exerts a force F on m1 at vector distance r from it, we can get the relative
motion of m1 by using m in place of m1.
y y

Bo Fixed
m2 th m2
in par
mo tic r
r tio les r
r n
r r
r2 r2
m1 m1
r r
r1 r1

x x
O O

(a) Two-body problem (b) One-body problem

Fig. 4.63

”ƒ˜‹–ƒ–‹‘ƒŽ ‘” ‡
The gravitational force between two particles of masses m1 and m2 is given by
 mm
F = -G 1 2 2 r̂
r
where r̂ is the unit vector along r, the distance of particle m1 from m2. Thus, we have

Ê d2 r ˆ mm
m Á 2 ˜ = -G 1 2 2 r̂
Ë dt ¯ r

d2 r Gm1m2
or 2
=- r̂
dt mr 2
m1m2
as m =
m1 + m2

d2 r Ê m + m2 ˆ Gm1m2
\ = -Á 1 r̂
dt 2
Ë m1m2 ˜¯ r 2

d2 r m +m
or = - G 1 2 2 r̂
dt 2 r
Here the motion of particle m1 with respect to particle m2 would be the same as that of

the particle of mass m under the force of a fixed mass m2 at a vector distance r from it.


Problem 4.24 Two masses m1 and m2 at an infinite distance from each other are initially
at rest, start interacting gravitationally. Find their velocity of approach when they are at a
distance r apart.
'ƌĂǀŝƚĂƟŽŶപ707

Solution Let vr be their velocity of approach. From conservation of energy:


Increase in kinetic energy = Decrease in gravitational potential energy
1 2 Gm1m2
or mvr = ...(i)
2 r
Here, m = reduced mass
m1m2
=
m1 + m2
Substituting in Eq. (i) we get
2G(m1 + m2 )
vr =
r
‘–‹‘‘ˆƒ„‘†›‘ˆƒ••mƒ”‘—†ƒ„‘†›‘ˆƒ••M
In the absence of any external force acting on the system, the total linear momentum and
hence the velocity of the centre of mass remains constant in the inertial frame and zero in the
centre of mass frame. The two particles rotate about the centre of mass with same angular
velocity.
Let r1 and r2 be the distances of m and M from the centre of mass and r be the distance
between them, then we have
r1 + r2 = r ...(i)
and *mr1 = mr2 ...(ii)
Solving Eqs. (i) and (ii), we get
mr1 = M(r – r1)
mr1 = Mr – Mr1
mr1 + mr1 = Mr V
w
Mr M m
r1 = CM
m M+m v
Multiplying by
m
Ê mM ˆ r m r
r1 = Á =
Ë M + m ˜¯ m m
Again r1 + r2 = r r2 r1
mr1 = Mr2
Fig. 4.64
M(r – r2) = Mr2
mr = Mr2 + mr2
mr = r2(M + m)
mr
r2 =
M+m
M
Multiplying by
M
Ê mr ˆ M Ê mM ˆ r
r2 = Á =
Ë M + m ˜¯ M ÁË M + m ˜¯ M
mr
=
M
708പWŚLJƐŝĐƐĨŽƌ//dͲ:͗DĞĐŚĂŶŝĐƐ//

”„‹–ƒŽ‡Ž‘ ‹–›
Let v be the orbital velocity of m, then
GMm mv 2
2
=
r r1
GMm mv 2
or =
r2 Ê Mr ˆ
ÁË ˜
M + m¯

GM 2
\ v =
( M + m)r
Similarly, orbital velocity of M will be
GM 2
V = .
( M + m)r

‹‡‡”‹‘†
Method I:
Let T be the time period of revolution of m, then

Ê Mr ˆ
2p Á
2p r1 Ë M + m ˜¯
T = =
v GM 2
( M + m)r

Mr ( M + m)r
= 2p ¥
M+m GM 2

Mr ( M + m)1/2 ◊ r 1/2
= 2p
M + m G1/2 M 2 x1/2

M r 3/2 ◊ ( M + m)1/2
= 2p
M+m G1/2 M

r3
or T = 2p
G( M + m)

GM r3
For m << M, v = 0 and v  and T 2p
r GM
These are the results which we have obtained earlier.
Method II:
We can assume the body of mass m orbiting in a circle of radius r about a fixed mass M.
'ƌĂǀŝƚĂƟŽŶപ709

For the motion of m, we have


GMm
= mw2 r w r
m
r2 M
GMm Ê mM ˆ 2
= Á w r
r 2 Ë M + m ˜¯ Fig. 4.65

G( M + m)
\ w =
r3
2p r3
and T = = 2p .
w G( M + m)


1
Problem 4.25 If the radius of the earth contracts of its present value, what will be the
length of the day approximately? n $+65#6
2 2p
Solution Conservation of angular moment, L = Iw = MR2 ¥ = constant
5 T
fi T μ R2 [If M remains same]
2 2
T2 ÊR ˆ Ê R/n ˆ 1
fi = Á 2˜ =Á ˜ = 2
T1 Ë R1 ¯ Ë R ¯ n
24
fi T2 = hr [as T1 = 24 hr] Ans.
n2

Problem 4.26 A body of mass m is situated at a distance 4 Re above the earth’s surface,
where Re is the radius of the earth. How much minimum energy should be given to the
body so that it may escape?
Solution Potential energy of the body at distance 4Re from the surface of the earth
mgRe mgRe mgR
U=– = =- [as h = 4Re (given)]
1 + h/Re 1+ 4 5
mgRe
So, minimum energy required to escape the body will be Ans.
5
Problem 4.27 At what height the acceleration due to gravity decreases by 36% of its value
on the surface of the earth?
2
È R ˘
Solution As gh = g Í
Î R + h ˙˚
and gh = g – 36% of g
36
= g- g
100
710പWŚLJƐŝĐƐĨŽƌ//dͲ:͗DĞĐŚĂŶŝĐƐ//

64
\ gh = g
100
2
64 È R ˘
\ g = gÍ
100 Î R + h ˙˚
8 R
\ =
10 R+h
\ 8R + 8h = 10R
\ 8h = 2R
2R
\ h =
8
R
\ h =
4

Problem 4.28 A planet moves around the sun along an ellipse so that its minimum distance
from the sun is equal to r and the maximum distance to R. Making use of Kepler’s laws,
find its period of revolution around the sun. $+65#6
Solution The motion of the planet can be approximated to be along a circle of radius
Ê R + rˆ
ÁË ˜.
2 ¯
The time period will be given by
3
ÈR + r ˘
Í 2 ˙˚ r S R
T = 2p Î
G( M + m)
Here m is the mass of the planet and M is the mass
of the sun. 2a = R + r
As m << M
( R + r )3 Fig. 4.66
\ T  p
2GM

Problem 4.29 An artificial satellite of the moon revolves in a circular orbit of radius hR,
where R is the radius of the moon. During its orbit motion the satellite experiences resistance
due to cosmic dust. Assuming that this resistance is given by F = a v2, where a is a constant
and v is the velocity of satellite, determine how long the satellite will stay in the orbit until
it falls to the moon’s surface.
Solution Total energy in the circular orbit
GMm
=– ...(1)
2r
Rate of change of energy with radial distance r,
dE GMm
=
dr 2r 2
'ƌĂǀŝƚĂƟŽŶപ711

GMm
dE = dr ...(2)
2r 2
Energy lost per second,
dE
= – Fv = – av3
dt
3/2
Ê GM ˆ
= -a Á
Ë r ˜¯
3/2
Ê GM ˆ
dE = - a Á dt ...(3)
Ë r ˜¯
From Eqs. (1) and (3)
3/2
GMm Ê GM ˆ
dr = - a Á dt
2r 2 Ë r ˜¯

mr -1/2
dr = -a(GM )1/2 dt
2
Integrating the above expression, we get
R t
m
r -1/2 dr = - a(GM )1/2 Ú dt
2 hÚR 0

m| r|hRR = -a GM t

m[ hR - R ] = -a GM t

m [ hR - R ]
\ t =
a GM
m
t = R1/2 [ h - 1]
a (GM )1/2
On the surface of the moon,
GMm
= mg
R2
or (GM)1/2 = gR

Substituting it in the expression for t, we obtain


m
t = R1/2 [ h - 1]
a gR

m
= [ h - 1]
a gR
712പWŚLJƐŝĐƐĨŽƌ//dͲ:͗DĞĐŚĂŶŝĐƐ//

Problem 4.30 The gravitational field in a region is given by Eg = (5i + 10j) N/kg. Calculate
the work done by an external agent in carrying a mass of 0.4 kg from point (0, 0) to point
(3, 4) metre.
Solution Given, gravitational field Eg = (5i + 10j) N/kg

The gravitational force F = mE g = 0.4 (5i + 10j)
= (2i + 4j) N
The displacement s = (3i + 4j) m
The work done by the gravitational force
 
W = F◊s
= (2i + 4j) ◊ (3i + 4j) = 22 J
Thus, the work done by external agent = – 22 J. Ans.
Problem 4.31 A planet of mass m moves along an ellipse around the sun of mass Ms, so
that its maximum and minimum distances from the sun are r1 and r2 respectively. Find the
angular momentum L of this planet relative to the centre of the sun.
Solution By conservation of angular momentum at given two positions, we have
mv1r1 = mv2r2
v1r1
or v2 = ...(i)
r2
Applying the principle of conservation of v1
mechanical energy at the given positions, we have
1 GMs m 1 GMs m
mv12 - = mv22 - ...(ii) r2 r1
2 r1 2 r2 Sun

Solving Eq. (i) and (ii), we get


2Gms r2
v1 = v2
r1 (r1 + r2 )
Fig. 4.67
Angular momentum L = mv1r1
2Gms r1r2
= Ans.
(r1 + r2 )

Problem 4.32 Two particles of equal mass (m) go round a circle of radius R under the action
of their mutual gravitational attraction. Find the speed of each particle.
Solution If v is the speed of each particle, then by Newton’s second law,
we have v
Gmm mv 2 F F
F = = v R
( 2 R )2 R
Gm
\ v = Ans. Fig. 4.68
4R
'ƌĂǀŝƚĂƟŽŶപ713

Problem 4.33 The masses and the radii of the earth and the moon are M1, R1 and M2,
R2 respectively. Their centres are at a distance d apart. The minimum speed with which a
particle of mass m should be projected a point mid-way between the two centres so as to
escape to infinity is ....
Solution The potential energy of particle of mass m cated at P is

GM1m È GM2 m ˘ Earth


V =- + Í- ˙ M1 Moon
(d/2) Î (d/2) ˚
M2
m
2G m R1
=– [ M1 + M2 ] p
R2
d
KE of particle at P d
1
K= mves2 where ves = escape velocity Fig. 4.69
2
Total energy E =K+V
1 2 È 2G m ˘
= mves + Í- ( M1 + M2 )˙
2 Î d ˚
When the particle just escapes (reaches at infinite distance from M1 and M2), its total
energy is zero. Hence,
1 2 2G m
mves - [M1 + M2] = 0
2 d
4G
ves2 = [M1 + M2]
d
ÈÏ 4 G ¸˘
or ves = ÍÌ d ( M1 + M2 )˝˙
ÎÓ ˛˚

Problem 4.34 Binary stars of comparable masses m1 and m2 rotate under the influence of
each other’s gravity with a time period T. If they are stopped suddenly in their motions,
find their relative velocity when they collide with each other. The radii of the stars are R1
and R2 respectively. G is the universal constant of gravitation.
Solution Both the stars rotate about their centre of mass (CM).
m2
m1
R1 COM R2

r1 r2
r

Fig. 4.70
714പWŚLJƐŝĐƐĨŽƌ//dͲ:͗DĞĐŚĂŶŝĐƐ//

r1 r r +r r
For the position of CM = 2 = 1 2 = (r = r1 + r2)
m2 m1 m1 + m2 m1 + m2
Gm1m2 Gm2 Ê 2p ˆ
Also, m 1r 1w 2 = or w 2 = ÁË w = ˜
r 2
r1r 2 T¯
m2 r
But, r1 =
m1 + m2
G(m1 + m2 )
\ w2 =
r3
1/3
Ï G(m1 + m2 ) ¸
or r =Ì ˝ ...(i)
Ó w2 ˛
Applying conservation of mechanical energy we have
Gm1m2 Gm1m2 1 2
- =– + mv ...(ii)
r (R1 + R2 ) 2
m1m2
Here, m = reduced mass =
m1 + m2
and vr = relative velocity between the two stars.
From Eq. (ii), we find that
2Gm1m2 Ê 1 1ˆ
v2 = Á - ˜
m Ë R1 + R2 r ¯

2Gm1m2 Ê 1 1ˆ
= -
m1m2 ÁË R1 + R2 r ˜¯
m1 + m2
Ê 1 1ˆ
= 2G(m1 + m2) Á - ˜
Ë R1 + R2 r ¯
Substituting the value of r from Eq. (i), we get

È 1 Ô¸ ˘˙
1/3
Í ÔÏ 4p 2
v= 2G(m1 + m2 ) -Ì ˝
Í R1 + R2 ÔÓ G(m1 + m2 )T 2 ˛Ô ˙
Î ˚

Problem 4.35 Three particles each of mass m, are situated at the vertices of an equilateral
triangle of side length a. The only forces acting on the particles are their mutual gravitational
forces. It is desired that each particle moves in a circle while maintaining the original
separation a. Find the initial velocity that should be given to each particle and also the time
period of the circular motion. (IIT-JEE, 1988)
Solution Let the particles be located at vertices A, B and C of equilateral triangle. The centre
of their circular path is O. The radius of circular path
a/2 a
r = OA = OB = OC = =
cos 30∞ 3
'ƌĂǀŝƚĂƟŽŶപ715

Gmm
The force exerted by any particle on the other particle F = .
a2
Each particle experiences the force due to other two particles. The resultant force on any
of the particles
A
F¢ = F 2 + F 2 + 2 FF cos 60∞
Gmm F F
= 3F = 3 F¢
a2
This force provides the necessary centripetal force, i.e., r 0
2 30°
mv Gmm
= 3 B a C
r a2
Substituting the value of r and after solving, we get
Fig. 4.71
Gm
v =
a
2p r 2p ( a/ 3 ) 2p a 3/2
Time period = = = Ans.
v Gm (3Gm)1/2
a

Problem 4.36 The mass of a railroad car is 100 metric tonnes. What will be its weight when
it is (i) moving due east (ii) moving due west along the equator at 72 km/h? Radius of the
earth is 6400 km.
Solution Suppose W0 is the weight of the train when it is not moving; the earth is also not
rotating. Let W be the weight of the train when the earth is rotating with angular velocity
we; train is at rest with respect to the earth, we have
mve2
W = W0 –
R
mve2
or W0 = W + ...(i)
R
2p 2p
where v e= w eR and we = = = 7.27 ¥ 10–5 s
T 24 ¥ 60 ¥ 60
(i) Apparent weight of the train when it is moving along east, with velocity v w.r.t. the
earth
m(ve + v)2
W¢ = W0 -
R

Ê mve2 ˆ m(ve + v)2


= ÁW + - ...(ii)
Ë R ˜¯ R
716പWŚLJƐŝĐƐĨŽƌ//dͲ:͗DĞĐŚĂŶŝĐƐ//

Simplifying Eq. (ii) and noting that v << ve and W = mg, we get
Ê 2We v ˆ
W¢ = W Á 1 -
Ë g ˜¯

Here v = 72 km/h = 20 m/s and we = 7.27 ¥ 10–5 s

Ê 2 ¥ 7.27 ¥ 10 - 5 ¥ 20 ˆ
\ W¢ = 100 ¥ 1000g Á 1 - ˜
Ë 9.8 ¯
= 99.63 metric tonnes. Ans.
(ii) When the train is moving due west, its apparent weight will be

Ê 2We v ˆ
W¢ = W Á 1 +
Ë g ˜¯

Ê 2 ¥ 7.27 ¥ 10 - 5 ¥ 20 ˆ
= 100 ¥ 1000g Á 1 + ˜
Ë 9.8 ¯
= 100.31 metric tonnes. Ans.
Problem 4.37 A cord of length 64 m is used to connect a 100 kg astronaut spaceship
whose mass is much larger than that of the astronaut. Estimate the value of tension in the
cord assuming that the spaceship is orbiting near the earth’s surface. Also assume that the
spaceship and the astronaut fall in a straight line from the earth’s centre. The radius of the
earth R = 6400 km.
Astronaut
Solution If h is the length of the cord, then radius of path of GMm T
astronaut (neglecting height of spaceship) is (R + h). The gravi- (R + h )2 Spaceship
tational force on the astronaut due to the earth
R h
GMm
F=
( R + h )2
If T is the tension in the cord then by Newton’s second law Fig. 4.72
GMm
+ T = mw2 (R + h)
( R + h )2
GMm
or T = mw2 (R + h) – ...(i)
( R + h )2
For the astronaut,
GMm
= mw2 (R + h)
( R + h )2
GM gR2 g
or w2 = 3
= 3
= (As h << R).
( R + h) ( R + h) R

Substituting value of w2 and GM = gR2 in Eq. (i), we have


mg(R + h) mgR2
T= -
R ( R + h )2
'ƌĂǀŝƚĂƟŽŶപ717

Ê ˆ
mgh Á 1 ˜
= mg + - mg Á 2
˜
R ÁÊ hˆ ˜
ÁË ÁË 1 + ˜¯ ˜¯
R

ÈÊ -2
mgh hˆ ˘
= mg + - mg ÍÁ 1 + ˜ ˙
R ÍÎË R ¯ ˙˚

mgh ÈÊ 2h ˆ ˘
= mg + - mg ÍÁ 1 - ˜ ˙
R ÎË R ¯˚
mgh 2mgh
= +
R R
3mgh
=
R
3 ¥ 100 ¥ 10 ¥ 64
= = 3 ¥ 10 – 2 N Ans.
6400 ¥ 10 3

Problem 4.38 A solid sphere of uniform density and radius 4 m is y


located with its centre at the origin O of coordinates. Two spheres
of equal radius 1 m with their cavities at A (–2, 0, 0) and B (2, 0, 0)
respectively are taken out, leaving behind spherical cavities (Fig.
4.73). The mass of each sphere taken out is M. Calculate:
x
O
(a) the gravitational field at B; (IIT-JEE, 1993) A B
(b) the gravitational potential at any point on circle y2 + z2 = 36;
(c) the gravitational potential at any point on circle y2 + z2 = 4.
z
Solution Given, mass of sphere of radius 1 m = M. Fig. 4.73
\ Mass of the whole sphere of radius 4 m,
4
M ¥ p ( 4 )3
M0 = 3 = 64 M.
4
p (1)3
3
(a) The gravitation field at B = field due to the whole sphere – field due to sphere A
GM0 r GM
= -
R3 AB2
G(64 M ) ¥ 2 GM
or EB = - 2
43 4
31
= GM . Ans.
16
718പWŚLJƐŝĐƐĨŽƌ//dͲ:͗DĞĐŚĂŶŝĐƐ//

(b) The radius of the given circle is 4 m. Obviously, the point will lie outside the sphere
in yz-plane
P
The distance AP = BP = 6 2 + 22
= 40 m.
The potential at P = Potential due to whole sphere 6m

– 2 (potential due to cavity)


- GM0 È GM ˘ A 2 mO 2 m B
= + 2Í
OP Î AP ˙˚
- G(64 M ) 2GM
= + Fig. 4.74
6 40
= – 10.98 GM. Ans.
2 2
(c) The given circle, y + z = 4, has radius 2 m. Obviously, the point will lie inside the
sphere.
The distance AP = BP = 2 2 m.
y
The potential due to whole sphere is given by
GM0
= - ( 3R 2 - r 2 )
2R 3 P
- G(64 M )(3 ¥ 4 2 - 22 ) 2m
= x
2 ¥ 43 A 2 mO 2 m B

= – 22 GM.
GM GM
The potential due to each cavity = + =+ Fig. 4.75
AP 2 2
GM
Thus, net potential at P = – 22GM + 2 ¥
2 2
= – 21.29 GM. Ans.

R
Problem 4.39 There is a crater of depth on the surface of the Mars of R = radius of
100
Mars. A projectile is fired vertically upwards from the centre, with a velocity, which is equal
to escape velocity on the moon. Find the maximum height obtained by the projectile.
(JEE, 2003)
H
ve
Solution If M is the mass of the moon, then escape velocity from the
moon’s surface R
100
A
2GM R
v= M
R O

By conservation of mechanical energy, we can write Moon


Ki + Ui = Kf + Uf ...(i)
Fig. 4.76
'ƌĂǀŝƚĂƟŽŶപ719

1 1 Ê 2GM ˆ GMm
where Ki = mv 2 = m Á ˜=
2 2 Ë R ¯ R
Kf = 0.
È - GM ˘
Ui = m ¥ potential of point A = m Í 3
( 3 R 2 - r 2 )˙
Î 2R ˚
È - GM ÏÔ 2 Ê R ˆ ¸Ô˘
2
= mÍ Ì 3 R - ÁË R - ˜ ˝˙
3
ÍÎ 2R ÓÔ 100 ¯ ˛Ô˙˚

-1.01 GM
=
R
- GMm
If H is the height attained, then Uf =
R+H
On substituting these values in Eq. (i) and simplifying, we get
H = 99.5 R Ans.
Problem 4.40 A man made satellite is moving in a circular orbit around the earth with a
speed equal to half the magnitude of escape velocity from the earth:
(i) Determine the height of the satellite above the earth’s surface.
(ii) If the satellite is stopped suddenly in its orbit and allowed to fall freely into the earth,
find the speed with which it hits the surface of the earth.
Solution
(i) If M and R are the mass and radius of the earth, then orbital speed of the satellite
1 2GM
v0 =
2 R
If h is the height of satellite, then
GM
v0 =
R+h
GM 1 2GM
Thus, =
R+h 2 R
\ h = R. Ans.
(ii) By conservation of mechanical energy, we have
K i + Ui = Kf + Uf
GMm GMm 1
O- =- + mv 2
( R + h) R 2
GMm - GMm 1
or – = + mv 2
( R + R) R 2
After solving, we get
v = gR Ans.
720പWŚLJƐŝĐƐĨŽƌ//dͲ:͗DĞĐŚĂŶŝĐƐ//

Problem 4.41 A particle is fired vertically upwards from the earth’s surface and it goes
up to a maximum height equal to radius of the earth. Find the velocity of projection of the
particle.
Solution If v is the velocity of projection, then
1 GMm GMm
mv 2 - = 0-
2 R ( R + h)
Given, h = R,
1 GMm GMm
\ mv 2 - =-
2 R ( R + R)

GM
or v = = 7.9 km/s Ans.
R

Problem 4.42 Three point masses each of mass m are placed at the vertices of an equilateral
triangle of side l. What is the gravitational field and potential due to three masses at the
centroid of the triangle?
Solution The distance of each point mass from the centroid m

l/2 l
r= =
cos 30∞ 3
E l
The gravitational field produced by each point mass is equal, and
so their vector sum will be zero. Gravitational potential E E

30°
È Gm ˘
Vg = 3 ¥ Í - m l/2 m
Î r ˙˚
Fig. 4.77
È Gm ˘ Gm
= 3 Í- ˙ = -3 3 Ans.
Î l/ 3 ˚ l

Problem 4.43 A double star is a system of two stars moving around the centre of mass of
the system due to gravitation. Find the distance between the components of the double star,
if its total mass equals M and the period of revolution T.
Solution The time period is given by
r3
T = 2p
G( M + m)

Here (M + m) is the total mass of the system.


Problem 4.44 A planet A moves along an elliptical orbit around the sun. At the moment
when it was at the distance r0 from the sun its velocity was equal to v0 and the angle between
 
the radius vector r0 and the velocity vector v 0 was equal to a. Find the maximum and
minimum distance that will separate this plane from the sun during its orbital motion. Take
mass of the sun as ms.
'ƌĂǀŝƚĂƟŽŶപ721

Solution m0v0r0 sin a m1v1r1 = m2v2r2

1 GMm 1 GMm
and mv02 - = mv12 -
2 r0 2 r1
1 GMm
= mv22 -
2 r2
Solve the above equations for r1 and r2.
Problem 4.45 Two the earth’s satellites move in a common plane along circular orbits. The
orbital radius of one satellite r = 7000 km while that of the other satellite is Dr = 70 km less.
What time interval separates the periodic approaches of the satellite to each other over the
minimum distance?

GMe GMe
Solution w= 3
and w ¢ =
r (r - Dr )3/2
2p 2p
t1 = t1 = and t2 =
w¢ -w w¢ +w

Problem 4.46 If the radius of the earth contracts to half of its present value without change
in its mass, what will be the new duration of the day?
Solution Present angular momentum of the earth
2
L1 = Iw = MR2w
5
New angular momentum because of change in radius
2
2 Ê Rˆ
L2 = MÁ ˜ w¢
5 Ë 2¯
If external torque is zero then the angular momentum must be conserved
L1 = L2
2 1 2
MR2w = ¥ MR2w ¢, i.e., w¢ = 4 w
5 4 5
1
T¢ = T
4
1
= ¥ 24 = 6 h
4

Problem 4.47 The minimum and maximum distances of a satellite from the centre of the
earth are 2R and 4R respectively, where R is the radius and M is the mass of the earth. Find:
(a) its minimum and maximum speeds,
(b) radius of curvature at the point of minimum distance.
722പWŚLJƐŝĐƐĨŽƌ//dͲ:͗DĞĐŚĂŶŝĐƐ//

Solution
(a) Applying conservation of angular momentum v2
mv1 (2R) = mv2(4R) R 3R
v1 = 2v2 ...(i) A B
R
From conservation of energy
v1
1 GMm 1 GMm
mv12 - = mv22 - ...(ii)
2 2R 2 4R Fig. 4.78

Solving Eqs. (i) and (ii), we get

GM 2GM
v2 = , v1 =
6R 3R
(b) If r is the radius of curvature at point A
mv12 GMm
=
r ( 2 R )2
4v12 R2 8 R
r = = (Putting value of v1)
GM 3
Problem 4.48 An artificial satellite is put in the orbit just above the earth. Atmosphere
with a velocity 1.5 times the velocity for a circular orbit at that height. The initial velocity
imparted is horizontal what would be the maximum distance of the satellite from the earth
when it is in the orbit.
Solution If v1 and v2 are the velocities of satellite at closest S
point P and farthest point A respectively, then as force is central,
angular momentum will be conserved, i.e.,
R
v1 r2 P E A
mv1r1 = mv2r2 or = ...(i)
v2 r1
Also as gravitational force is conservative, mechanical energy is
also conserved, i.e., r1 r2

1 GMm 1 GMm Fig. 4.79


mv12 - = mv22 -
2 r1 2 r2

1 È Ê v ˆ2˘ È1 1˘
or mv1 Í1 - Á 2 ˜ ˙ = GMm Í - ˙
2
2 ÍÎ Ë v1 ¯ ˙˚ Î r1 r2 ˚

Substituting the value of (v2/v1) from Eq. (i) in it

È Ê r ˆ2˘ Èr - r ˘
v12 Í1 - Á 1 ˜ ˙ = 2GM Í 2 1 ˙ ...(ii)
ÍÎ Ë r2 ¯ ˙˚ Î r1r2 ˚
'ƌĂǀŝƚĂƟŽŶപ723

Now according to given problem

v1 = ( 1.5 )v0 = (3GM/2r1 ) [as v0 = GM/r ]


So, Eq. (ii) reduces to
3 GM È r22 - r12 ˘ È r2 - r1 ˘
Í ˙ = 2GM Í ˙
2 r1 Î r22 ˚ Î r1r2 ˚
or 3(r2 + r1) = 4r2 or r2 = 3r1
So the maximum distance of the satellite from the centre of the earth is 3R (with r1 = R)
and so maximum distance from the surface will be 3R – R = 2R.
Problem 4.49 A body is projected from the earth’s surface at an angle a = 30° to the
1.5 GM
horizontal at a speed v = . Neglecting air resistance and rotation of the earth, find
Re
height to which the body will rise.
Solution Suppose the body is launched at the point A with a velocity v1 at an angle 30°
1.5 GM
with the horizontal v1 =
Re
GM
The orbital velocity v0 =
Re
v1 > v0

2GM
The escape velocity ves =
Re
ves > v1 > v0
\ the body will move in an elliptical orbit around the earth and reach the highest point at B.
v1

30°

th
Ear

Elliptical path
v

Fig. 4.80
724പWŚLJƐŝĐƐĨŽƌ//dͲ:͗DĞĐŚĂŶŝĐƐ//

Applying the law of conservation of angular momentum,


mv1 cos 30° Re = mvr
where v is the velocity at the highest point.
Re v1 3
v =
r 2
Using the law of conservation of energy
1 GMm 1 GMm
mv12 - = mv 2 -
2 Re 2 r
1 2 GM 1 3 R2 GM
v1 - = ¥ v12 2e -
2 Re 2 4 r r
1 1.5 GM GM 3 1.5 GM Re2 GM
¥ - = ¥ -
2 Re Re 8 Re r2 r
3 1 9Re 1
- = -
4 Re Re 16 r 2 r
Solving 4r2 – 16 rRe + 9R2e = 0
16 Re ± 256 Re2 - 144 Re2 (4 + 7 )
r= = Re
8 2
Solution giving to lower value of r is neglected. Maximum height reached = r – Re
Ê (4 + 7 )ˆ Ê (2 + 7 ) ˆ
= Á ˜ Re - Re = Á Re
Ë 2 ¯ Ë 2 ˜¯

Problem 4.50 A cosmic body A moves to the sun with velocity


v0 (when far from the sun) and aiming parameter l the arm of
the vector v0 relative to the centre of the sun (Fig. 4.81). Find Sun
the minimum distanced by which this body will get to the sun.
Mass of the sun is M.
l
Solution At the minimum separation with the sun, the cosmic A
body’s velocity is ^ to its position vector relative to the sun. v0
Using the conservation of angular momentum about the sun Fig. 4.81
located at C
vl
m v0 l = mvrmin fi v = 0
rmin
Also the total mechanical energy is conserved. rmin
C V
1 GMs m 1
mv02 = - + mv 2
2 rmin 2
Ms Æ mass of the sun A r
v0
Fig. 4.82
'ƌĂǀŝƚĂƟŽŶപ725

2 G Ms v02 t 2
or v20 + - 2 =0
rmin rmin
or v20 r2min + 2 GMs rmin – v02 l2 = 0
Solving the quadratic equation,
È 2 ˘
G Ms Í Ê lv02 ˆ ˙
rmin = 1 + ÁGM ˜ - 1
v02 Í Ë s ¯ ˙
ÍÎ ˙˚

Problem 4.51 A space vehicle is in circular orbit about the earth. The mass of the vehicle
is 300 kg, and the radius of the orbit is 2 Re. It is desired to transfer the vehicle to a circular
orbit of radius 4 Re :
(a) What is the minimum energy required for the transfer?
(b) If the transfer accomplished through an elliptical orbit, what initial and final velocity
changes are required.
Take g = 10 m/s2 at the earth’s surfaces and Re = 6400 km (radius of the earth).
Solution See Fig. 4.83.
Ê GM ˆ
For the first orbit, v1 = ÁË 2R ˜¯ 4Re
e
va
Ê gRe ˆ
\ v1 = ÁË ˜ = 5656 m/s
2 ¯ 3Re
E
For the second orbit a
Ê gRe ˆ
v2 = ÁË ˜ = 4000 m/s
4 ¯ c

Let the velocity of the satellite is increased to vp. vp


Now it moves along the elliptical orbit with semi- Transfer
major axis as a = 3 Re. The distance of its focus c, is Re. orbit
Applying the principle of conservation of angular
momentum at apogee, we have
Fig. 4.83
m va (a + c) = m vp (a – c)
or va (3Re + Re) = vp (3Re – Re)
va = vp/2 ...(1)
Applying the principle of conservation of energy at apogee and perigee, we get
1 GMm 1 GMm
mvp2 - = mva2 -
2 2 Re 2 4 Re
G M gRe
or vp2 – va2 = = ...(2)
2 Re 2
726പWŚLJƐŝĐƐĨŽƌ//dͲ:͗DĞĐŚĂŶŝĐƐ//

From Eqs. (1) and (2), we have


vp2 gRe
vp2 – =
4 2
3vp2 gRe Ê 2 gRe ˆ
= or vp = Á
4 2 Ë 3 ˜¯
\ vp = 6532 m/s
and va = vp/2 = 3266 m/s
Now, initial and final velocity changes are
vp – v1 = 6532 – 5656 = 876 m/s
v2 – va = 4000 – 3266 = 734 m/s,
Further, minimum energy required is
1 1 1 1
E= mvp2 - mv12 + mv22 - mva2
2 2 2 2
Substituting the values and solving it, we get
E = 2.4 ¥ 109 J
mv 2 GMm
=
R R2
GM È GM ˘
i.e., v0 = = gr ÍÎas g = R2 ˙˚
R

i.e., v0 = (9.8 ¥ 6.4 ¥ 106 ª 8 km/s


and for escaping from close to the surface of the earth,
GMm 1
= mve2
R 2
2GM È GM ˘
i.e., ve =
R
= 2gR ÍÎas g = R2 ˙˚

i.e., ve = 2 ¥ v0 = 1.41 ¥ 8 km/s


= 11.2 km/s
So additional velocity to be imparted to the orbiting satellite for escaping
= 11.2 – 8 = 3.2 km/s
Problem 4.52 A 470 kg communication satellite is released from a space shuttle at a height
of 280 km above the surface of the earth. From this height a rocket engine boosts it into a
geosynchronous orbit. What is the energy spent by the engine in transfer ring the orbit?
Solution The orbit period of a geosynchronous satellite is one day, i.e., the satellite travels
once around the earth in the same time that the earth spins once on its axis. We can apply
Kepler’s third law to find the radius of the orbit.
'ƌĂǀŝƚĂƟŽŶപ727

T2 = K Er 3
For a satellite around the earth,
GME ms ms v 2
=
r2 r
As orbital speed of planet is 2pr/T, where T is time period of revolution,
GME (2p r/T )2
=
r2 r
Ê 4p 2 ˆ 3
T2 = Á ˜r
Thus, constant Ë GME ¯
4p 2
KE =
GME
= 9.89 ¥ 10–14 s2/m3

T2 (86400)2
r =3 =3
KE (9.89 ¥ 10 - 14 )
= 4.23 ¥ 107 m
Which is the radius of the final orbit from the centre of the earth.
Radius of initial orbit, Ri = RE + 280 ¥ 103 m
= 6.65 ¥ 106 m
Total energies in initial and final orbits are
GME ms
Ei = –
2Ri
GME ms
Ef = –
2R f
Thus, the energy supplied by the booster to change the orbit is
DE = Ef – Ei
GME ms È 1 1˘
=– Í - ˙
2 ÍÎ R f Ri ˙˚

(6.67 ¥ 10 -11 )(5.98 ¥ 10 24 )( 470)


=-
2
È 1 1 ˘
¥Í -
Î 4.23 ¥ 107 6.65 ¥ 106 ˙˚
= 1.19 ¥ 1010 J
728പWŚLJƐŝĐƐĨŽƌ//dͲ:͗DĞĐŚĂŶŝĐƐ//

The change in kinetic energy of satellite is

GME ms È 1 1˘
DK = Í - ˙
2 ÍÎ R f Ri ˙˚
= – 1.19 ¥ 1010 J
The change in potential energy of the satellite
Ê 1 1ˆ
DU = – GMEmS Á - ˜
Ë R f Ri ¯
= 2.38 ¥ 1010 J
Thus, the firing of engine results in increase of potential energy and decrease of kinetic
energy, but total mechanical energy is increased.

Problem 4.53 All masses are released. What is initial acceleration of A with respect to C?

m l m
A B

D C
m m

Fig. 4.84
m F m
Solution We need resultant force on A. A
F1
2Gm2 Gm2 F
The resultant force = 2 F + F1 = + 2
l2 2l
2Gm Gm
Acceleration of m at A = + 2 m m
l2 2l
2Gm Gm
Acceleration value of mass at C is also = + 2 Fig. 4.85
l2 2l
The direction of acceleration of masses at A and C are in opposite a
direction.
The relative acceleration (initial) will be a

2 2Gm 2Gm
2a = + 2
l2 2l Fig. 4.86

Problem 4.54 The masses are released. What is the initial acceleration
m 3m
of mass m with respect to 3m?
a
3Gm2
Solution Gravitational force, F =
a2 Fig. 4.87
'ƌĂǀŝƚĂƟŽŶപ729

3Gm
Acceleration of m, a1 = (Æ)
a2
Gm
Acceleration of 3m, a2 = 2 (¨)
  a
We need a1 - a 2 .
Vector subtraction for finding relative acceleration of m w.r.t. 3m.
  4Gm
a1 - a 2 =
a2
Problem 4.55 What angular speed should be given to each mass
2m m
such that they perform circular motion?
3r0
Solution The masses will rotate about the centre of mass with same
angular speed. Fig. 4.88
3r m
Distance of centre of mass from 2m = 0 = r0
3m
The gravitational force will provide the radial force
G(2m)(m) 2Gm2
2
=
(3r0 ) 9r02
2Gm2
For mass 2m, 2
= 2mw 2 (r0 )
9r0
Gm
\ w = Speed = wx (radius)
9r03
Gm
i.e., to perform circular motion each mass is given angular speed
9r03
Linear speed for mass m = w (2r0) and linear speed for mass 2m = w (r0).
Problem 4.56 Two satellites of mass m and 2m are doing circular
motion in same orbit but in opposite direction. At some point they
collide completely inelastic. Find the speed of combined mass when it Me r = 2Re
falls back on surface of the earth. Re m

Solution We should know the orbit speed at which they rotate in the r = 2Re
orbit before collision,
GMe gRe2 gRe 2m
V0 = = =
2Re 2Re 2
Fig. 4.89
Momentum will be conserved just before and after collision,

gRe gRe 1 gRe


2m -m = 3mV1 fi V1 =
2 2 3 2
Let V be the combined when it falls back on the surface of the earth.
730പWŚLJƐŝĐƐĨŽƌ//dͲ:͗DĞĐŚĂŶŝĐƐ//

Now applying energy conservation,


3mV12 GMe (3m) 3mV 2 GMe (3m)
- = -
2 2Re 2 2Re
3V12 GMe (3m) 3V 2 GMe (3)
fi - = -
2 2Re 2 Re

3V12 gRe2 (3) 3V 2 gRe2 (3) 19 gRe


fi - = - \ V=
2 2Re 2 Re 18

Problem 4.57 What is gravitational field intensity at origin?


Gm2 Ê 25 ˆ Gm2 Ê 17 ˆ
(a) Á ˜ (b) Á ˜
a 2 Ë 36 ¯ a 2 Ë 36 ¯ m m m
(–a, 0) (2a, 0) (3a, 0)
Gm2 Ê 23 ˆ Gm2 Ê 19 ˆ
(c) Á ˜ (d) Á ˜
a 2 Ë 36 ¯ a 2 Ë 36 ¯ Fig. 4.90

Solution We will find E due to all three masses.


Gm2
E1 due to mass at (– a, 0) = (¨)
a2
Gm2
E2 due to mass at (2a, 0) = (Æ)
4 a2
Gm2
E3 due to mass at (3a, 0) = (Æ)
9a2
13 Gm2
Resultant of E2 and E3 = (Æ)
36 a 2

Problem 4.58 Initial velocities of two masses are shown in Fig. 4.91.
V0 2V0
What is the maximum separation between them during motion. a
m m
Solution At maximum separation velocity of each would become
same. Based on energy and momentum conservation concept, Fig. 4.91
mV0 + 2mV0 = 2mV
3V0
\ V =
2
Energy conservation,
2
Gm2 mV02 m(2V0 )2 Gm2 2m Ê 3V0 ˆ
– + + =– + Á ˜
a 2 2 r 2 Ë 2 ¯

Gm2 5mV02 9mV02 Gm2


fi – + - =–
a 2 4 r
'ƌĂǀŝƚĂƟŽŶപ731

Gm V02 Gm
- =
a 4 r
4Gma
\ r =
4Gm - V02 a

Problem 4.59 There are two cavities in the solid ball as shown in Fig.
4.92. Find (a) the potential at point P due to remaining part (b) the field M; R
intensity at point Q (OQ = R/2).
Solution Q¢ Q P
GM
(a) The potential due to complete ball = – on the surface. O
R
M
The mass of cavity portion will be .
8
GM/8 Fig. 4.92
Potential due to cavity part(right) =
R/2
GM/8
Potential due to cavity part(left) =
3R/2
The actual potential of ball after cavities are made
GM GM GM 2GM
fi Vremaining = - + + =-
R 4 R 12R 3R
GM(R/2)
(b) Field due to complete solid ball (¨)
R3
Field due to cavity ball of right portion = 0.
It is zero as the point is at the centre of the cut ball.
GM/8
Field due to cavity ball of left portion = (¨)
R2
The point is outside with distance R.
The net field due to ball after the cuts are made
 GM GM 3GM
Eremaining = - =
2R 2 8 R 2 8R2
C
Problem 4.60 A satellite is to be launched from the surface of the
earth to a circular orbit, height h above the surface of the earth. A
What is the extra energy required for launching a satellite and to
bring it to orbital altitude?
R
Solution In order to place a satellite in a stable orbit at an insertion
point C, first it is necessary to bring the satellite to the altitude and h
then the satellite must be given the required orbital velocity. The
orbital velocity at r = R + h can be found by equating the centripetal
force and gravitational force.
Fig. 4.93
732പWŚLJƐŝĐƐĨŽƌ//dͲ:͗DĞĐŚĂŶŝĐƐ//

GMm mv 2
=
r2 r
1/2
Ê GM ˆ Ê GM ˆ
1/2
v =Á ÁË ˜
Ë r ˜¯ R + h¯
1/2 - 1/2
Ê GM ˆ Ê hˆ

Ë R ˜¯ ÁË 1 + ˜¯
R
1/2
Ê GM ˆ Ê 2h ˆ

Ë R ˜¯ ÁË 1 - ˜¯ for h << R
R
The minimum initial velocity that a satellite must have at launch so that it has proper
orbital velocity at insertion point C after fuel of rocket has exhausted is called characteristic
velocity.
The energy at launch point A must be equal to the total energy of a circular orbit at
height h.
1 GMm GMm
mvc2 - =
2 R 2(R + h)
GM Ê 1 ˆ
v2c = Á 2-
R Ë (1 + h/R) ˜¯

GM(1 + 2 h/R)
=
R(1 + h/R)
GM Ê hˆ
ª ÁË 1 + ˜¯ if h << R
R R
1/2
Ê GM ˆ Ê h ˆ
or vc ª Á ÁË 1 + ˜ for h << R
Ë R ˜¯ 2R ¯
The kinetic energy at the launch point
1
KEc = mv 2
2
GMm (1 + 2 h/R)
=
2R (1 + h/R)
The kinetic energy for orbiting at height h,
1 2
KEorbit = mvins. .
2
GMm Ê 1 ˆ
=
2R Ë 1 + h/R) ˜¯
Á

The extra kinetic energy required,


GMm Ê 2 h/R ˆ
KEc – KEo =
2R ÁË 1 + h/R ˜¯
'ƌĂǀŝƚĂƟŽŶപ733

Problem 4.61 Use the conservation of mechanical energy and show that if an object is in
an elliptical orbit about a planet, then its distance r from the planet and speed v are related
Ê 2 1ˆ
by v2 = GM Á - ˜ ; where a is the semi-major axis.
Ë r a¯
Solution The situation is shown in Fig. 4.94. If vA is the velocity of the object at A, then at A

GMm mvA2 v
q
2
=
a a

1 GMm r a
A
\ mvA2 =
2 2a
Potential energy at Fig. 4.94
GMm
A = -
a
Thus, total mechanical energy at A
GMm GMm GMm
=K+U= - =-
2a a 2a
By conservation of mechanical energy, we can write
Mechanical energy at any position = Mechanical energy at A
- GMm 1 -GMm
\ + mv 2 =
r 2 2a
After simplifying, we get
Ê 2 1ˆ
v2 = GM Á - ˜ .
Ë r a¯

Problem 4.62 A satellite revolving in a circular equatorial orbit of radius r = 2.00 ¥ 104 km
from west to east appear over a certain point at the equator every t = 11.6 hours. Using this
data, calculate the mass of the earth. The gravitational constant is supposed to be known.
Solution Here the absolute angular velocity of satellite is given by
w = ws + wE
where wE is the angular velocity of the earth, which is from west to east.
2p 2p
or w = + [where t = 11.6 hr and T = 24 hr]
t T
GM
From Kepler’s third law, we have w = 3/2
r
GM 2p 2p
Thus, we have 3/2 = +
r t T
2 2
4p 2 r 3 È 1 1 ˘ 4p 2 (2 ¥ 107 )3 È 1 1 ˘
or M = + = + = 6.0 ¥ 10 24 kg
G ÍÎ t T ˙˚ (6.67 ¥ 10 - 11 ) ÍÎ 11.6 ¥ 3600 24 ¥ 3600 ˙˚
734പWŚLJƐŝĐƐĨŽƌ//dͲ:͗DĞĐŚĂŶŝĐƐ//

Problem 4.63 A satellite of mass m is moving in a circular orbit of radius r. Calculate its
angular momentum with respect to the centre of the orbit in terms of the mass of the earth.
Solution The situation is shown in Fig. 4.95. r
v
The angular momentum of the satellite with respect to the centre
of orbit is given by   
L = r¥mv Satellite
 m
where r is the position vector of satellite with respect to the centre r
 r
of orbit and v is its velocity vector of satellite.
 
In case of circular orbit, the angle between r and v is 90°. Hence M
L = mvr sin 90° = mvr ...(1)
Earth
The direction is perpendicular to the plane of the orbit.
We know orbital speed of satellite is
GM Fig. 4.95
v = ...(2)
r
From Eq. (1) and (2), we get

GM
L = m fi L = (GMm2 r)1/2
r

Problem 4.64 An artificial satellite is moving in a circular orbit around the earth with a
speed equal to half the magnitude of escape velocity from the earth.
(i) Determine the height of the satellite above the earth’s surface.
(ii) If the satellite is stopped suddenly in its orbit and allowed to fall freely onto the earth,
find the speed with which it hits the surface of the earth.
Solution
(i) Let M and R be the mass and radius of the earth respectively. If m is the mass of
satellite, then escape velocity from the earth vc = 2gRe

gRe
velocity of satellite =
2
Further, we know orbital speed of satellite at a height h is

Ê GMe ˆ Ê Re2 g ˆ R2 g
vs = ÁË ˜ = Á R + h˜ or v2s =
r ¯ Ë e ¯ R+h

From equation written above, we get


h = R = 6400 km
(ii) Now total energy at height h = total energy at the earth’s surface (principle of
conservation of energy)
m 1 m
or 0 – GMe = mv 2 - GMe
R+h 2 Re
'ƌĂǀŝƚĂƟŽŶപ735

1 GMe m GMe m
or mv 2 = - [As h = R]
2 Re 2Re
Solving we get v = gRe

or 9.8 ¥ 6400 ¥ 10 3 = 7.919 km/s

Problem 4.65 A sky lab of mass 2 ¥ 103 kg is first launched from the surface of the earth
in a circular orbit of radius 2R (from the centre of the earth) and then it is shifted from this
circular orbit to another circular orbit of radius 3 R. Calculate the minimum energy required
(a) to place the lab in the first orbit (b) to shift the lab from the first orbit to the second orbit.
Given, R = 6400 km and g = 10 m/s2. (IIT Roorkee, 1985)
Solution
(a) The energy of the sky lab on the surface of the earth
Ê GMm ˆ GMm
Es = KE + PE = 0 + Á - ˜ =-
Ë R ¯ R
And the total energy of the skylab in an orbit of radius 2R is
GMm
E1 = -
4R
So the energy required to place the lab from the surface of the earth to the orbit of
radius 2R is given as
GMm È GMm ˘ 3 GMm
E1 – E s = - - Í- =
4R Î R ˙˚ 4 R
3m 3 È GM ˘
or DE = ¥ gR2 = mgR ÍÎAs g = R2 ˙˚
4R 4
3 3
or DE = (2 ¥ 103 ¥ 10 ¥ 6.4 ¥ 106) = (12.8 ¥ 1010) = 9.6 ¥ 1010 J
4 4
(b) As for 2nd orbit of radius 3R the total energy of skylab is
GMm GMm
E2 = - =-
2(3R) 6R
GMm Ê GMm ˆ 1 GMm
or E2 – E 1 = - -Á- ˜=
6R Ë 4 R ¯ 12 R
1 1
or DE = mgR = (12.8 ¥ 1010) = 1.1 ¥ 1010 J
12 12

Problem 4.66 Calculate the mass and density of the earth. Given that gravitational constant
G = 6.67 ¥ 10–11 Nm2/kg2, the radius of the earth = 6.37 ¥ 106 m and g = 9.8 m/s2.
Solution The acceleration due to gravity on the earth’s surface is given as
GMe g R2 9.8 ¥ (6.37 ¥ 106 )2
ge = 2 or Me = s e = = 6 ¥ 1024 kg
Re G 6.67 ¥ 10 - 11
736പWŚLJƐŝĐƐĨŽƌ//dͲ:͗DĞĐŚĂŶŝĐƐ//

If r is the density of the earth, then


4 3M 3 ¥ (6 ¥ 10 24 )
M= p R3 ¥ r or r= = = 5.5 ¥ 103 kg/m3
3 4p R3 4 ¥ 3.14 ¥ (6.37 ¥ 106 )3

Problem 4.67 If the radius of the earth were to shrink by one per cent, its mass remaining
the same, what would happen to the acceleration due to gravity on the earth’s surface?
(IIT-JEE, 1981)
Solution Consider the case of body of mass m placed on the earth’s surface (mass of the
earth M and radius R).
If g is acceleration due to gravity, then we know that
GM
g = ...(1)
R2
Now, when the radius is reduced by 1%, i.e., radius becomes 0.99 R, let acceleration due
to gravity be g¢, then
GM
g¢ = ...(2)
(0.99R)2
From Eqs. (1) and (2), we get
2
g¢ R2 1 Ê 1 ˆ
= = or g¢ ª g ¥ Á or g¢ = 1.02 g
g (0.99R) (0.99)2 Ë 0.99 ˜¯
Thus, the value of g is increased by 2%.
Problem 4.68 At what rate should the earth rotate so that the apparent g at the equator
becomes zero? What will be the length of the day in this situation?
Solution At the earth’s equator effective value of gravity is
geq = gs – w2Re
If geff at equator is zero, we have
gs
gs = w 2R e or w=
Re

2p Re 6.4 ¥ 106
Thus, length of the day will be T = = 2p = 2 ¥ 3.14 = 5074.77 s  84.57 min
= 1.41 hr. w gs 9.8

Problem 4.69 Calculate the acceleration due to gravity at the surface of the Mars if its
diameter is 6760 km and mass is one-tenth that of the earth. The diameter of the earth is
12742 km and acceleration due to the gravity on the earth is 9.8 m/s2.
Ê GM ˆ
We know that g = Á 2 ˜
Ë R ¯
2 2
g M Ê M M ˆ Ê RE ˆ Ê 1 ˆ Ê 12742 ˆ gM
So = Á ˜ Á ˜ = ÁË ˜¯ ÁË ˜¯ fi = 0.35 or gM = 9.8 ¥ 0.35 = 3.48 m/s2
gE Ë ME ¯ Ë RM ¯ 10 6760 gE
'ƌĂǀŝƚĂƟŽŶപ737

Problem 4.70 Calculate the apparent Re cos l


weight of a body of mass m at a latitude
l when it is moving with speed v on the
N
surface of the earth from west to east at the
Re cos l m l
same latitude. mw 2Re cos l
Solution If W be the apparent weight of mgs
body at a latitude l then from Fig. 4.96
l R
shown, we have e

W = mg – mw2R cos2l ...(1) Equator


When the body moves at speed v from
west to east relative to the earth, its net
angular speed w can be given as
v we
w = we +
R cos l Fig. 4.96
[we Æ the earth’s angular velocity]

Now from Eq. (1) we have


2
È v ˘ 2
W = mg – m Íw e + ˙ R cos l
Î R cos l ˚

È v2 2w e v ˘
or W = mg - m Íw e2 + 2 2
+ 2
˙ R cos l
ÍÎ R cos l R cos l ˙˚

mv 2
= mg – mw e2R cos2l – – 2mwe v cos l
R
Ê W 2 R cos 2 l 2w e v cos l ˆ mv 2
= mg Á 1 - e - ˜ [Neglecting as being very small]
Ë g g ¯ R

Problem 4.71 A satellite revolves around a planet in an elliptical orbit. Its maximum and
minimum distances from the planet are 1.5 ¥ 107 m and 0.5 ¥ 107 m respectively. If the speed
of the satellite at the farthest point is 5 ¥ 103 m/s, calculate the speed at the nearest point.
Solution In case of elliptical orbit, the r
speed of satellite varies constantly as v1
shown in Fig. 4.97. Thus, according to
the law of conservation of angular mo- r r
mentum, the satellite must move faster r2 r1
Appogee
Perigee
at a point of closest approach (perigee) m
than at a farthest point (appogee).
  
We know that L = r ¥ m v r
v2
Hence, at the two points, L = m v1r1
= m v2 r2 Fig. 4.97
738പWŚLJƐŝĐƐĨŽƌ//dͲ:͗DĞĐŚĂŶŝĐƐ//

v1 r
or = 2
v2 r1
Substituting the given values, we get
5 ¥ 10 3 0.5 ¥ 107
= fi v2 = 1.5 ¥ 104 m/s
v2 1.5 ¥ 107

Problem 4.72 Imagine a light planet revolving around a massive star in a circular orbit of
radius r with a period of revolution T. On what power of r, will the square of time period
depend if the gravitational force of attraction between the planet and the star is proportional
to r–5/2.
Solution As gravitation provides centripetal force
mv 2 K K
= 5/2 , i.e., v2 =
r r mr 3/2

2p r mr 3/2 4p 2 m 7/2
So that T= = 2p r or T2 = r ; so T2 μ r7/2
v K K

Problem 4.73 Estimate the mass of the sun, assuming the orbit of the earth around the sun
to be a circle. The distance between the sun and the earth is 1.49 ¥ 1011 m and G = 6.66 ¥
10–11 Nm2/kg2. (BITSAT, 2015)
Solution Here the revolving speed of the earth can be given as
GM
v= [Orbital speed]
r
where M is the mass of the sun and r is the orbit radius of the earth.
We known time period of the earth around the sun is T = 365 days, thus we have
2p r r 4p 2 r 3 4 ¥ (3.14)2 ¥ (1.49 ¥ 1011 )3
T= or T = 2p r or M = = = 1.972 ¥ 1022 kg
v GM GT 2
(365 ¥ 24 ¥ 3600)2 ¥ (6.66 ¥ 10 - 11 )

Problem 4.74 If the earth is one-half of its present distance from the sun, how many days
will be in one year?
Solution If orbit of the earth’s radius is R, in previous example we’ve discussed that time
period is given as
r 2p 3/2
T = 2p r = r
Gm GM
r
If radius changes or r¢ = , new time period becomes
2
2p
T¢ = r¢ 3/2
GM
From the above equations, we have
3/2 3/2 3/2
T Ê rˆ Ê r¢ ˆ Ê 1ˆ 365
= Á ˜ or T¢ = T Á ˜ = 365 Á ˜ = days
T¢ Ë r¢ ¯ Ë r¯ Ë 2¯ 2 2
'ƌĂǀŝƚĂƟŽŶപ739

Problem 4.75 A satellite of mass Ms is orbiting the earth in a circular orbit of radius R.
It starts losing energy slowly at a constant rate C due to friction. If Me and Re denotes the
mass and radius of the earth respectively, show that the satellite falls on the earth in a time
GMs Me Ê 1 1 ˆ
t= - .
2C ÁË Re R ˜¯

Solution The energy of the satellite in its orbit


GMe Ms
Ei = , and close to the earth surface
2R
it is Ef = – GMeMs/2Re.
If t is the required time, then

È - GMe Ms ˘ -GMe Ms
- Ct + Í ˙ =
Î 2R ˚ 2Re
- GMs Me È 1 1˘
\ t = Í - ˙ – ve sign as losing energy.
2C Î Re R ˚

Problem 4.76 A satellite is revolving around the earth v + v0


in a circular orbit of radius a with velocity v0. A particle
is projected from the satellite in forward direction with Me
a
relative velocity v = ( 5/4 - 1)v0 . Calculate, during subse-
quent motion of the particle its minimum and maximum Re
C
distances from the earth’s centre.
Solution The corresponding situation is shown in Fig.
4.98.
Ê GM ˆ
Initial velocity of satellite, v0 = ÁË ˜
a ¯
When the particle is thrown with the velocity v relative r
to the satellite, the resultant velocity of the particle will
become v1
vR = v0 + v
Fig. 4.98
Ê 5ˆ Ê 5 GM ˆ
= ÁË ˜¯ v0 = ÁË ˜
4 4 a ¯

As the particle velocity is greater than the velocity required for circular orbit, hence the
particle path deviates from circular path to elliptical path. At position of minimum and
maximum distance velocity vectors are perpendicular to instantaneous radius vector. In this
elliptical path the minimum distance of the particle from the earth’s centre is a and maximum
speed in the path is vR and let the maximum distance and minimum speed in the path be
r and v1 respectively.
Now as angular momentum and total energy remain conserved. Applying the law of
conservation of angular momentum, we have
740പWŚLJƐŝĐƐĨŽƌ//dͲ:͗DĞĐŚĂŶŝĐƐ//

mv1r = m (v0 + v)a [m = mass of particle]

(v0 + v)a a È Ê 5 GM ˆ ˘ 1 È Ê 5 ˆ˘
or v1 = = Í Á ˜ ˙ = Í Á ¥ GMa˜ ˙
r r ÍÎ Ë 4 a ¯ ˙˚ r ÍÎ Ë 4 ¯˙
˚
Applying the law of conservation of energy
1 GMm 1 GMm
mv12 - = m(v0 + v)2 -
2 r 2 a

1 Ê 5 GMa ˆ GMm 1 Ê 5 GM ˆ GMm


or mÁ ˜ - = mÁ ˜-
2 Ë4 r ¯
2
r 2 Ë4 a ¯ a

5 a 1 5 1 1 3
¥ 2 - = ¥ - =-
8 r r 8 a a 8a
5a
or 3r2 – 8ar + 5a2 = 0 or r = a or
3
Thus, minimum distance of the particle = a
5a
And maximum distance of the particle =
3

Problem 4.77 A rocket is fired vertically from the surface of Mars with a speed of 2 km s–1.
If 20% of its initial energy is lost due to Martian atmospheric resistance, how far will the
rocket go from the surface of the Mars before returning to it? Mass of the Mars = 6.4 ¥ 1023
kg, radius of the Mars = 3395 km, G = 6.67 ¥ 10–11 Nm2 kg–2.
Solution Total energy of the rocket
1 GMm
E= mv 2 -
2 R
Since 20% energy is lost, hence energy remained
80 4 4 È1 GMm ˘
= 80% or E = E = E = Í mv 2 -
100 5 5 Î2 R ˙˚
At the highest point, distant h from the surface of the Mars, its total energy will be
potential. Hence,
4 È1 GMm ˘ GMm
mv 2 - =–
5 ÍÎ 2 R ˙˚ R+h

2 Ê 2 2GM ˆ GMm
or mÁ v - ˜ =–
5 Ë R ¯ R+h

2 v 2 R - 2GM GM
or ◊ =–
5 R R+h

5 RGM
or R+h =–
2(v 2 R - 2 GM )
'ƌĂǀŝƚĂƟŽŶപ741

But R = 3395 km = 3395 ¥ 103 m,


G = 6.67 ¥ 10–11 Nm2 kg–2
M = 6.4 ¥ 1023 kg, v = 2 km s–1 = 2 ¥ 103 ms–1
\ R+h
5 ¥ 3395 ¥ 10 3 ¥ 6.67 ¥ 10 - 11 ¥ 6.4 ¥ 10 23
=–
2[(2 ¥ 10 3 )2 ¥ 3395 ¥ 10 3 - 2 ¥ 6.67 ¥ 10 - 11 ¥ 6.4 ¥ 10 23 ]
7.25 ¥ 10 20
=–
2[1.36 ¥ 1013 - 8.54 ¥ 1013 ]
7.25 ¥ 107
= = 5.05 ¥ 106 m
2 ¥ 7.18
or h = 5.05 ¥ 106 – 3395 ¥ 103
= 1655 ¥ 103 m = 1655 km.
Problem 4.78 In Kepler’s law of periods: T2 = Kr3, the constant k = 10–13 s2 m–3. Express
the constant k in days and kilometres. The moon is at a distance of 3.84 ¥ 105 km from the
earth. Obtain its time-period of revolution in days. (VITEEE, 2011)
s2
Solution Given k = 10–13
m3
1 1
As 1s = day and 1 m = km
24 ¥ 60 ¥ 60 1000
1 1
\ k = 10 - 13 ¥ 2
d2 km - 3
(24 ¥ 60 ¥ 60) (1/1000)3
= 1.33 ¥ 10–14 d2 km–3.
For the moon, r = 3.84 ¥ 105 km.
\ T2 = kr3 = 1.33 ¥ 10–14 ¥ (3.84 ¥ 105)3
= 27.3 days.
Problem 4.79 A uniform copper sphere of radius R and mass m
surrounded by a thin uniform spherical shell of same mass and
radius 4R. The centre of the shell C falls on the surface of the inner
sphere. Find the gravitational fields at points A and B. B A
∑ ∑ C
Solution
(i) At point A, field strength due to shell will be zero.
Net field is only due to metal sphere. Distance between the AC = 3R, BC = 5R
centre of metal sphere and point A is 4R.
Fig. 4.99
G(m) Gm
\ EA = 2
=
( 4 R) 16 R2
(ii) At point B, net field is due to both, due to shell and due to copper sphere.
Gm Gm 61Gm
\ EB = 2
+ 2
= Ans.
( 5 R) ( 6 R) 900 R2
742പWŚLJƐŝĐƐĨŽƌ//dͲ:͗DĞĐŚĂŶŝĐƐ//

Problem 4.80 An artificial satellite is moving in a circular orbit around the earth with a
speed equal to half the magnitude of escape velocity from the surface of the earth. (Radius
of the earth = 6400 km)
(a) Determine the height of the satellite above the earth’s surface.
(b) If the satellite is stopped suddenly in its orbit and allowed to fall freely on the earth,
find the speed with which it hits the surface of the earth.
Solution
ve
(a) vo =
2
2GM
GM R
\ =
r 2
\ r = 2R
or h = r – R = R or height = radius of the earth.
(b) Increase in kinetic energy = decrease in potential energy
1 mgh 2 hg
\ mv 2 = \ v=
2 h h
1+ 1+
R R
Substituting the values we have,

2 ¥ 9.81 ¥ 6400 ¥ 10 3
v =
R
1+
R
= 7924 m/s ª 7.92 km/s Ans.
Problem 4.81 A spaceship is launched into a circular orbit close to the earth’s surface. What
additional velocity has to be imparted to the spaceship to overcome the gravitational pull?
(JEE, 2016)

GM 2GM
Solution V0 = Ve =
R R
Required additional velocity is
Dv = Ve ~ V0
GM
= ( 2 - 1)
R
GMR R
= ( 2 - 1) multiply by
R2 R
GM
= ( 2 - 1) gR g =
R2
Here M = 6 ¥ 1024 kg
R = 6.4 ¥ 106 m
g = 9.8 m/82
Dv = 3.27 vm/s approx.
'ƌĂǀŝƚĂƟŽŶപ743

Problem 4.82 The density of the core of a planet is r1 and that of the
outer shell is r2, the radii of the core and that of the planet are R and
2R respectively. The acceleration due to gravity at the surface of the
r r2 r1
planet is same as at a depth R. Find the ratio of 1 .
r2
Solution Let m1 be the mass of the core and m2 the mass of outer
shell.
gA = gB (given)
Fig. 4.100
Gm1 G(m1 + m2 ) B
Then 2
=
R ( 2 R )2
\ 4m1 = (m1 + m2) A

Ï4 ¸ 4 Ï4 4 ¸
or 4 Ì p R3 r1 ˝ = p R3 ◊ r1 + Ì p (2R)3 - p R3 ˝ r2 R
Ó 3 ˛ 3 Ó 3 3 ˛
\ 4r1 = r1 + 7r2
2R
r1 7
\ = Ans.
r2 3 Fig. 4.101

Problem 4.83 Three balls each of mass 0.5 kg are kept at the vertices of an isosceles right
triangle with its hypotenuse of 2 2 m. What will be the net gravitational force on the ball
kept at the vertex of right angle?
Solution Let BA = BC = r
A
0.5 kg
\ r 2 + r 2 = ( 2 2 )2
\ 2r2 = 4 ¥ 2
\ r =2m
   2 2m
Fnet on B = FBA + FBC
 G(0.5)(0.5) 6.67 ¥ 10 - 11 ¥ 0.25
|FBA| = = = 4.17 ¥ 10 - 12 N
22 4 0.5 kg 0.5 kg
B C
 G(0.5)(0.5) 6.67 ¥ 10 ¥ 0.25- 11
and |FBC| = = = 4.17 ¥ 10 - 12 N Fig. 4.102
22 4
   
|Fnet|2 = |FBA|2 +|FBC|2 = 2|FBA|2

\ |Fnet| = 2 ( 4.17 ¥ 10 - 12 )N = 5.84 ¥ 10–12 N

Problem 4.84 A projectile is projected with velocity kve vertically upward direction from
the ground into the space (ve is escape velocity and k < 1). If air resistance is considered to
be negligible then find the maximum height from the centre of the earth, to which it can go?
(R = radius of the earth)
744പWŚLJƐŝĐƐĨŽƌ//dͲ:͗DĞĐŚĂŶŝĐƐ//

Solution From the law of conservation of energy difference in potential energy between
the ground and maximum height = Kinetic energy at the point of projection.
mgh 1 1 1
= (kve )2 = mk 2ve2 = mk 2 ( 2 gR)2 ve = 2gR
1 + h/R m 2 2
Rk 2
By solving height from the surface of the earth, h =
1 - k2
Rk 2 R
So, height from the centre of the earth, r = R + h = R + = Ans.
1 - k2 1 - k2

Problem 4.85 Find the duration of the day if a body at the equator will be weightless.
(BITSAT, 2007)
Solution N = m(g – mRw2)
Since the body becomes weightless, we have N = 0
or 0 = m(g – Rw2)
g
or w =
R
R
or T = 2p = 84.4 min
g
T
Then, the duration of the day = = 42.2 min Ans.
2

Problem 4.86 Find the potential energy of a system of four particles each of mass 2 m kept
at the vertices of a square of side x. Also find the potential at the centre of the square.
2m x 2m
Solution Distance of diagonal pair is = x 2 + x 2 = 2 x
Now, potential energy of system = potential energy of pairs of mass
at the ends of the sides + Potential energy of masses at diagonal ends
È - G(2m)2 ˘ È - G(2m)2 ˘
= 4Í ˙ + 2 Í ˙
ÍÎ x ˙˚ ÍÎ 2 x ˙˚ 2m 2m
2 2
Gm 8Gm Fig. 4.103
= -16 -
x 2x
Gm2
= – 21.64
x È ˘
Í - G(2m) ˙ Gm
Now, the gravitational potential at the centre will be V(centre) = 4 Í ˙=-8 2
ÍÊ 2 ˆ ˙
x x
ÍÎ ÁË ˜
2 ¯ ˙˚
Problem 4.87 A satellite of mass 240 kg is in a circular orbit at a height of 1760 km above the
earth’s surface. The satellite loses energy at a uniform rate of 1.2 x 105 joule per revolution,
maintaining circular orbits of diminishing radii. Find (a) the distance from the earth and
'ƌĂǀŝƚĂƟŽŶപ745

speed of the satellite at the end of 1000 revolutions, (b) the average retarding force during
this period.
Solution Total energy of the satellite in the orbit of radius r = R + h = 6400 + 1760 = 8160 km
GMm
E = -
2r
dE = 1.2 ¥ 105 joule per revolution
GMm
dE = - dr
2r 2
dE dr
= -
E r
dE
|dr| = r
E
GMm
E = -
2r
6.67 ¥ 10 - 11 ¥ 6 ¥ 10 24 ¥ 240
= -
2 ¥ 8.16 ¥ 106
= 5.885 ¥ 109 J
1.2 ¥ 10 5 dE
Thus, dr = 8.16 ¥ 106 ¥ 9
as |dr| = Dr = r
5.885 ¥ 10 E
= 166.4 m
(a) After 1000 revolutions, change in radius
Dr = 166.4 km
The new radius,
r¢ = r – Dr
= (8160 – 166.4) km
= 7993.6 km
New velocity
GM
v =

= 7.076 ¥ 103 m/s
(b) Average radius
8160 + 7993.6
=
2
= 8076.8 km
Average distance travelled
= 2p ¥ 8.0768 ¥ 106
= 5.07 ¥ 107 m
Energy lost per revolution
= 1.2 ¥ 105 J
746പWŚLJƐŝĐƐĨŽƌ//dͲ:͗DĞĐŚĂŶŝĐƐ//

Therefore, average retarding force


1.2 ¥ 10 5
=
5.07 ¥ 107
= 0.237 ¥ 10–2 N
Problem 4.88 Two satellites of the earth move in a common plane along circular orbits, the
radii being r and r – Dr(Dr << r). What is the time interval between their periodic approaches
to each other over the minimum distance? Take Me to the mass of the earth. (Me = 6 ¥ 1024
kg, r = 7000 km, Dr = 70 km)
Solution Time period of a satellite in circular orbit is given by

r3
T = 2p = 1.62 hr
GM
2p 3
dT = ¥ r 1/2 dr
GM 2
dT 3 dr
=
T 2 r
3 70
dT = ¥ ¥T
2 7000
= 0.015T.
Let the first satellite catch up with the second after n revolutions, then
nT = (n + 1)(T – dT)
n
= 0.985
n+1
fi n = 65.67
\ Periodic time of approach = nT = 106.38 hrs.
Problem 4.89 Two concentric shells of masses M1 and M2 are M1
situated as shown in Fig. 4.104. Find the force on a particle of mass M2
B
m when the particle is located at (a) r = a (b) r = b and (c) r = c. The b
O
distance r is measured from the centre of the shell. a
A
C
c
Solution We know that attraction at an external point due to a
spherical shell of mass M is (GM/r2) while at an internal point is
zero. So
Fig. 4.104
(a) For r = a, the point is external to both the shells; so
G( M1 + M2 )
EA =
a2
So, FA = mEA = Gm(M1 + M2)/a2
'ƌĂǀŝƚĂƟŽŶപ747

(b) For r = b, the point is external to the shell of mass M2 and internal to the shell of mass
M1; so
GM2
EB = + 0 , i.e., FB = mEB = (GMm/b2)
b2
(c) For r = c, the point is internal to both the shells, so
EC = 0 + 0 = 0, i.e., FC = mEC = 0
Problem 4.90 Jupiter has a mass of 318 times that of the earth and its radius is 11.2 times
the earth’s radius. Estimate the escape velocity of a body from Jupiter’s surface. Given that
the escape velocity from the earth’s surface is 11.2 km/sec. Does your answer throw light on
why the atmosphere of Jupiter contains light gases, mostly hydrogen, whereas, the earth’s
atmosphere has little of hydrogen gas?
Solution If Me and Mj are masses of the earth and Jupiter and Re and Rj are their radii
respectively, then, If vej is the escape velocity on Jupiter and vee is escape velocity on the
earth, then
2GM j
Escape velocity on Jupiter vej = ...(i)
Rj

2GMe
and vee = ...(ii)
Re
Dividing Eq. (i) by Eq. (ii), we get
vej M j Re
= ◊ ...(iii)
vee Me R j

where M = 318 Me
Rj = 11.2 Re (all given)
and vee = 11.2 km/sec
Substituting these values in Eq. (iii), we get
318 Me Re
vej = 11.2 ◊ km s - 1
Me 11.2Re

1
or vej = 11.2 318 ¥ km s - 1
11.2
= 59.679 km s–1 = 59.68 km s–1
Since the escape velocity on the earth is less than 1/5 th of the escape velocity on the Jupiter,
hence lighter gases like hydrogen get escaped from the surface of the earth overcoming the
gravitational pull of the earth (ve = 11.2 km s–1). On the Jupiter, escape velocity is very high
(59.68 km s–1) hence, even the lighter gases which have greater molecular speeds at a given
temperature, cannot escape from the surface of the Jupiter. That is why hydrogen is held
back on the Jupiter and is major constituent of its atmosphere.
748പWŚLJƐŝĐƐĨŽƌ//dͲ:͗DĞĐŚĂŶŝĐƐ//

Problem 4.91 Light from a cirus star suffers gravitational red shift, i.e., its wavelength
changes towards the red end due to the gravitational attraction of the cirus star. Obtain
the formula for this gravitational red shift using the simple consideration that a photon of
hv
frequency v has energy hv (h is the Planck’s constant) and mass 2 . Estimate the magnitude
c
of the red shift for light of wavelength 5000 Å from a star of mass 1032 kg and radius 106 km.
Use the known values of G (G = 6.67 ¥ 10–11 Nm2kg–2 and c = 3.00 ¥ 108ms–1).
(IIT Roorkee, 1977)
Solution If hv is the energy of the photon neglecting the effect of gravitational attraction
and hv¢ is the energy after the red shift (i.e., when gravitational attraction is considered).
GM Ê hv ˆ
Then, hv¢ = hv - Á ˜
R Ë c2 ¯
hv
where is the mass of the photon
c2
GMv
or v¢ = v -
Rc 2
c c GM c
or = - 2◊
l¢ l Rc l

c c È GM ˘
or = Í1 - 2 ˙
l¢ l Î Rc ˚
-1
È GM ˘
or l¢ = l Í1 - 2 ˙
Î Rc ˚
If GM/Rc2 << 1, then
È GM ˘
l¢ = l Í1 - 2 ˙
Î Rc ˚
lGM
or |l¢ – l| =
Rc 2
5000 ¥ 10 - 10 ¥ 6.67 ¥ 10 - 11 ¥ 10 32
= Å
109 ¥ 9 ¥ 1016
= 0.3706 Å
Problem 4.92 An artificial satellite is moving in a circular orbit around the earth with a
speed equal to half the magnitude of escape velocity from the earth. (i) Determine the height
of the satellite above the earth’s surface. (ii) If the satellite is stopped suddenly in its orbit
and allowed to fall freely on to the earth, find the speed with which it hits the surface of the
earth. Take g = 98 ms–2, radius of the earth = 6400 km.
Solution
(i) Orbital velocity of a satellite at a height h above the earth’s surface is
GM
v0 =
R+h
'ƌĂǀŝƚĂƟŽŶപ749

Escape velocity from the earth’s surface

GM
ve =
R
ve
Given, v0 =
2
GM 1 2GM
\ =
R+h 2 R

GM 1 2GM GM
or = ¥ =
R+h 4 R 2R
or h = R = 6.4 ¥ 106 m.
(ii) Let V be the speed with which the satellite hits the earth when suddenly stopped. If
m is the mass of the satellite, then by the conservation of energy,
Initial PE at height h (= R) = Final PE on the surface of the earth
= KE of the satellite.
GMn GMn 1
or - =- + mV 2
2R 2R 2

GM gR2
or V = = = gR
R R

= 9.8 ¥ 6.4 ¥ 106


= 7.92 ¥ 103 ms–1 = 7.92 km s–1.
Problem 4.93 A particle is projected upwards from the surface of the earth (radius R) with
KE equal to half the minimum value needed for it to escape. To which height does it rise
above the surface of the earth? (IIT-JEE, 1997)
Solution For the particle to escape, KE = PE
1 GMm
mve2 =
2 R
1 1 GMn
But supplied KE = ¥ mve2 =
2 2 2R
Suppose the particle rises to a height h, then
1 1 GMn
¥ mve2 =
2 2 R+h
GMn GMn
or =
2R R+h
\ h =R
750പWŚLJƐŝĐƐĨŽƌ//dͲ:͗DĞĐŚĂŶŝĐƐ//

Problem 4.94 A uniform ring of mass m and radius a is placed directly above a uniform
sphere of mass M and of equal radius. The centre of the ring is at a distance 3a from the
centre of the sphere. Find the gravitational force exerted by the sphere on the ring.
Solution dF = force on a small mass dm of the ring by the
sphere. dm q a q dm

Net force on ring = S(dF sin q) or Ú dF sin q dF dF


3a 2a
GM(dm) 3
= S ¥
( 2 a )2 2
3GM M
= 2
S(dm)
8a
But S(dm) = m, the mass of whole ring.
3GMm Fig. 4.105
\ Net force = 2
Ans.
8a

Problem 4.95 Imagine a planet whose diameter and mass are both one half of those of the
earth. The day’s temperature of this planet surface reaches up to 800 K. Make calculation
and tell whether oxygen molecules are possible in the atmosphere of the planet. (Escape
velocity from the earth’s surface= 11.2 km/s, k = 1.38 ¥ 10–23 J/K, mass of oxygen molecule
= 5.3 ¥ 10–26 kg)
Option: (a) 0.79 km/s (b) 0.89 km/s (c) 0.5 km/s (d) 0.2 km/s
2GM
Solution As ve =
R
vP MP RE 1 2
So, = ¥ = ¥ =1
vE ME RP 2 1
i.e., vP = vE = 11.2 km/s ...(i)
Now according to kinetic theory of gases,

3RT 3(Nk )T
vrms = =
M Nm
[as R = Nk and M = Nm]

3kT 3 ¥ 1.38 ¥ 10 -23 ¥ 800


i.e., vrms = = = 0.79 km/s ..(ii)
m 5.3 ¥ 10 -26

From Eqs. (i) and (ii) it is clear that velocity of oxygen molecule in the atmosphere of
planet (~ 0.8 km/s) is less than escape velocity from the planet (= 11.2 km/s); so oxygen
molecules cannot escape and so the planet’s atmosphere may contain oxygen.
'ƌĂǀŝƚĂƟŽŶപ751

Problem 4.96 Inside a fixed sphere of radius R and uniform density


R
r, there is spherical cavity of radius such that surface of the cavity
2
passes through the centre of the sphere as shown in Fig. 4.106. A A B
particle of mass m0 is released from rest at centre B of the cavity. R/2
R
Calculate velocity with which particle strikes the centre A of the
sphere. Neglect the earth’s gravity. Initially, sphere and particle are
at rest.
Solution Applying conservation of mechanical energy, Increase in Fig. 4.106
kinetic energy
= decrease in gravitational potential energy
1
or m0v2 = UB – UA = m0(VB – VA)
2
\ v = 2(VB - VA ) ...(i)

Potential at A
VA = potential due to complete sphere – potential due to cavity
1.5 GM È Gm ˘
= - - Í- ˙
R Î R/2 ˚
2Gm 1.5 GM
= - ...(ii)
R R
3
4 Ê Rˆ pr R3
Here, m = pÁ ˜ r= (cavity)
3 Ë 2¯ 6
4
and M = pR3r (complete sphere)
3
In Eq. (ii) substituting the values, we get
G È pr R3 ˘ 5
VA = Í - 2pr R3 ˙ = - p Gr R3
RÎ 3 ˚ 3
Potential at B
GM È Ê R ˆ ˘ 1.5Gm
2
2
VB = - Í1 .5 R - 0 .5 ÁË ˜¯ ˙ +
R3 Î 2 ˚ R/2
11 GM 3Gm
=- + ...(iii)
8 R R
G È pr R3 11 ˘
Again put values of m, M = Í - ◊ pr R3 ˙
RÎ 2 6 ˚
4
= - p GrR2
3
1
\ VB – VA = p GrR2
3
So, from Eq. (i)
2
v = p Gr R2 Ans.
3
752പWŚLJƐŝĐƐĨŽƌ//dͲ:͗DĞĐŚĂŶŝĐƐ//

Problem 4.97 A projectile of mass m is fired from the surface of a


v0
the earth at an angle a = 60° from the vertical. The initial speed v0
GMe
is equal to . How high does the projectile rise? Neglect air
Re
Re
resistance and the earth’s rotation.
rmax
Solution Let v be the speed of the projectile at the highest point and
rmax its distance from the centre of the earth. Applying conservation
of angular momentum and mechanical energy,
Fig. 4.107
mv0 sin a = mvrmax ...(i)
1 GMe m 1 GMe m
mv20 – = mv 2 - ...(ii)
2 Re 2 rmax
Solving these two equations with the given data we get,
3Re
rmax =
2
or the maximum height
Re
hmax = rmax – Re = Ans.
2

Problem 4.98 A man can jump vertically to a height of 1.5 m on the earth. Calculate the
radius of a planet of the same mean density as that of the earth from whose gravitational
field he could escape by jumping. Radius of the earth is 6.41 ¥ 106 m.

u2
Solution h=
2 ge
\ u = 2g e h ...(i)
For the asked planet this u should be equal to the escape velocity from its surface.

\ 2g e h = 2g p Rp
or g eh = g pR p

GMe GM p
◊h = ◊ Rp
Re2 Rp2

Ê4 3ˆ Ê4 3ˆ
ÁË p Re ˜¯ r h ÁË p Rp ˜¯ r Rp
3 3
or =
Re2 Rp2

or RP = Re h

= (6.41 ¥ 106 )(1.5)


= 3.1 ¥ 103 m Ans.
'ƌĂǀŝƚĂƟŽŶപ753

Problem 4.99 Two planets of equal mass orbit a much more


massive star (Fig. 4.108). Planet m1 moves in a circular orbit
m1
of radius 1 ¥ 108 km with 2 years period. Planet m2 moves in
Star
an elliptical orbit with closest distance r1 = 1 ¥ 108 km and r2 r1
A P
farthest distance r2 = 1.8 ¥ 108 km, as shown in Fig. 4.108.
(a) Using the fact that the mean radius of an elliptical
orbit is the length of the semi-major axis, find the m2
period of m2’s orbit.
(b) Which planet has the greater speed at point P? Which Fig. 4.108
has the greater total energy?
(c) Compare the speed of planet m2 at P with that at A.
Solution
(a) Mean radius of planet,
r1 + r2
m2 = = 1.4 ¥ 108 km
2
Now, T μ r3/2
\ Time period of m2: 3/2
Ê 1.4 ¥ 108 ˆ
T2 = Á
Ë 108 ˜¯
or T2 = 2(1.4)3/2 = 3.31 years Ans.
(b) For m2, point P is perigee position. So speed at this point is greater than orbital speed
for circular orbit.
\ Um2 = Um1
\ Em2 > Em1
Km2 > Km1
(c) vr = constant
Problem 4.100 Two concentric spherical shells have masses M1, M2 and radii R1, R2 (R1
< R2). What is the force exerted by this system on a particle of mass m1 if it is placed at a
R1 + R2
distance from the centre?
2
R1
Solution Field at point P P
R2
GM1 4 GM1
g = 2
=
Ê R1 + R2 ˆ (R1 + R2 )2
ÁË ˜
2 ¯
R1 + R2
4 GM1 m 2
\ F = mg =
(R1 + R2 )2 Fig. 4.109
754പWŚLJƐŝĐƐĨŽƌ//dͲ:͗DĞĐŚĂŶŝĐƐ//

Problem 4.101 A satellite of mass 100 kg is placed initially in a temporary orbit 800 km
above the surface of the earth. The satellite is to be placed now in a permanent orbit at 2000
km above the surface of the earth. Find the amount of work done to move satellite from
temporary to permanent orbit. The radius of the earth is 6400 km.
Solution The energy of a satellite at a distance r is given by
GMm 1
Er = – + mv 2
r 2
where v = (GM / r )
GMm 1 Ê GMm ˆ GMm
\ Er = + Á ˜=-
r 2Ë r ¯ 2r
If R be the radius of the earth and h, the distance of the satellite from the earth’s surface,
then
r = (R + h)
GMm gR2 m
\ Er = - =-
2(R + h) 2(R + h)
Now, work done is given by
W = Eh = 2000 – Eh = 800

gR2 m È 1 1 ˘
= -
2 ÍÎ R + 2000 R + 800 ˙˚

gR2 m È 1 1 ˘
= -
2 Î 8400 km 7200 km ˙˚
Í

Substituting the values and solving we get


25600
W = = 406.35 joules
63

Problem 4.102 A satellite orbiting in a circle of radius r around the earth has a time period
T. If its radius slightly increases by Dr, determine the change in its time period DT.
Solution From Kepler’s law, we know that
T = kr3/2 where k is a constant
On differentiating, we get
dT 3 3 kr 3/2 3 Ê T ˆ
= kr 1/2 = = Á ˜
dr 2 2 e 2Ë r¯

Ê dT ˆ 3ÊTˆ
Thus, DT = Á ˜ Dr = Á ˜ Dr
Ë dr ¯ 2Ë r¯
'ƌĂǀŝƚĂƟŽŶപ755

Problem 4.103 From a solid sphere of mass M and radius R, a


spherical portion of radius R/2 is removed, as shown in Fig. 4.110.
Taking gravitational potential V = 0 at r = •, the potential at the centre
of the cavity thus formed is: (JEE Main, 2015)
(G = gravitational constant)
2GM 2GM GM GM
(a) (b) (c) (d)
3R R 2R R
Fig. 4.110
Solution Due to complete solid sphere, potential at point P

-GM È 2 Ê R ˆ ˘
2

Vsphere = 2R 3 Í 3 R - ÁË ˜¯ ˙
ÍÎ 2 ˙˚

-GM Ê 11R 2 ˆ GM
= 3 Á ˜ = - 11
2R Ë 4 ¯ 8R
Due to cavity part potential at point P
GM
3 8 3GM
Vcavity = - =-
2 R 8R
2
So potential at the centre of cavity

11GM Ê 3 GM ˆ -GM
= Vsphere – Vcavity = - - Á- =
8R Ë 8 R ˜¯ R
Ans. (d)
Problem 4.104 In a double star, two stars (one of mass m and the other of 2m) distance
d apart rotate about their common centre of mass. Deduce an expression for the period of
revolution. Show that the ratio of their angular momenta about the centre of mass is the
same as the ratio of their kinetic energies.
d 2d
Solution The centre of mass C will be at a distance and from the masses 2m and
3 3
m respectively as shown in Fig. 4.111. Both the stars rotate with same angular velocity w
around C in their respective orbits. Here the gravitational force acting on each star due to
other supplies the necessary centripetal force.
Gravitational force on each star = G (2m) m/d2.
Centripetal force of star (mass m)

Ê 2d ˆ
= mrw2 = m Á ˜ w 2
Ë 3¯
G(2m)m Ê 2d ˆ
\ = mÁ ˜ w2
d 2 Ë 3¯

Ê 3 GM ˆ
w = ÁË 2 ˜¯
d
756പWŚLJƐŝĐƐĨŽƌ//dͲ:͗DĞĐŚĂŶŝĐƐ//

2p Ê d2 ˆ
\ T = = 2p Á
w Ë 3 GM ˜¯
w
Ratio of angular momentum
2
Ê dˆ d 2d
2m Á ˜ w
(Iw )big Ë 3¯ 1 3 3
= = 2
= 2m m
(Iw )small Ê 2d ˆ 2 C
mÁ ˜ w
Ë 3¯
Ê 1 2ˆ w
ÁË Iw ˜¯
2 big
Ratio of kinetic energies =
Ê 1 2ˆ
ÁË Iw ˜¯
2 small
2 Fig. 4.111
Ê dˆ
2m Á ˜
(I )big Ë 3¯ 1
= = 2
=
(I )small Ê 2d ˆ 2
mÁ ˜
Ë 3¯
This is same as the ratio of their angular momenta.
Problem 4.105 Four particles of equal mass M move along A
a circle of radius R under the action of their mutual gravita- M
45°
tional attraction. Find the speed of each particle.
(IIT-JEE, 2014)
Solution Hint. See Fig. 4.112. D
O
B
GM M M
Gravitational force on A due to B =
( R 2 )2 R
The direction being along AB
2
GM
Gravitational force on A due to D = along AD M
( R 2 )2
C
GM 2
Similarly, force on A due to C = along AC Fig. 4.112
4 R2
GM 2 GM 2
Resultant = 2 cos 45∞ +
( R 2 )2 4 R2
The resultants is along AC.
GM 2 È 2 2 + 1 ˘
= Í ˙
R2 Î 4 ˚

Mv 2 GM 2 È 2 2 + 1˘
Now, = Í ˙
R R2 Î 4 ˚
1/2
È GM Ï 2 2 + 1 ¸ ˘
v =Í Ì ˝˙
Î R Ó 4 ˛˚
'ƌĂǀŝƚĂƟŽŶപ757

Problem 4.106 A sphere of uniform density r has within it a spherical cavity whose centre
is a distance a from the centre of the sphere. Show that the gravitational field within the
cavity is uniform and determine its magnitude and direction.
Solution The resultant gravitational field in the cavity is equal to the gravitational field of
the original sphere minus the gravitational field of the material which has been removed.
The gravitational field inside a solid sphere of density r is given by
 4 
g = – p Gr r
3 P
The field is directed inwards. Now we consider the sphere with a r
R r r r
spherical cavity as shown in Fig. 4.113. Consider a point P within the r+a
r
cavity such that the position vector of P relative to centre of cavity a

is r . The position vector of P relative to centre of original sphere is
 
a + r . The gravitational field due to original sphere at point P is
4  
– p Gr ( a + r )
3 Fig. 4.113
The gravitational field due to a hypothetical negative mass occupying cavity is
4 
p Gr r
3
The resultant field at P is
 4   4 
g = - p Gr ( a + r ) + p Gr r
3 3
4 
= - p Gr a
3
4
Thus, the field within the cavity is uniform. The magnitude of field intensity is p Gra.
3
It depends only on the position, and not the size, of the cavity and its direction is parallel to
the line joining the centre of the cavity to the centre of the original sphere.
Problem 4.107 Find approximately the third cosmic velocity v3, i.e., the minimum velocity
that has to be imparted to a body relative to the earth’s surface to drive it out of the solar
system. The rotation of the earth about its own axis is to be neglected.
Solution The escape velocity in the field is
2GMs
ve =
rs
Ms = mass of the sun.
rs = The distance between the sun and the earth.
ve = 2vo
Here vo = orbital velocity around the sun.
GMs
=
rs
758പWŚLJƐŝĐƐĨŽƌ//dͲ:͗DĞĐŚĂŶŝĐƐ//

The relative velocity of escape of an object in the field of the sun with respect to the earth is
vrel = 2vo - vo
when v0 orbital velocity of the earth around the sun.
vrel = ( 2 - 1) vo
But the object is in the field of the earth.
The mechanical energy of the object in the field of the earth is
1 GmMe
E= mv32 -
2 Re
For escaping
The energy E provides required kinetic energy for escaping
1
E = mvrel2
2
1 GmMe 1 2
mv32 – = mvrel
2 Re 2
1 GmMe 1
mv32 – = m ( 2 - 1)2 vo2 ...(i)
2 Re 2

For orbital velocity of object in field around the earth is


GMe
vo = ...(ii)
Re
After solving Eq. (i) and (ii)
We get
v3 = ( 2 - 1)vo2 + 2vo2
Putting the value we get
v3 = 17 km/s.
Problem 4.108 Equal masses (m) hung from strings of
different lengths on a balance at the surface of the earth
as shown in Fig. 4.114. The strings have negligible mass
and differ in length by h. Find the error in weighing due
to the fact that W1 is nearer to the centre of the earth
m
than W. Given mean density of the earth = d.
h m
[Hint. Let R be the distance of W1 from the centre of the (W )
earth. (IIT Roorkee, 1994) (W1 )

Mm Mm Fig. 4.114
Solution Then, W1 = G 2
and W = G 2
R ( R + h)
Mm GMm
\ W1 – W = G -
R ( R + h )2
'ƌĂǀŝƚĂƟŽŶപ759

=G
Mm È
R ÍÎ
1- 1-
2h
R {˘
ºº˙
˚ }
2 GMnh
=
R3
2 G ( 4 / 3 ◊ p R3 d)mh 8
= 3
= p Gdmh
R 3

Problem 4.109 An the earth satellite is revolving in a circular orbit of radius a with velocity
v0. A gun is in the satellite and is aimed directly towards the earth. A bullet is fired from the
gun with muzzle velocity v0/2. Neglecting resistance offered by cosmic dust and recoil of
gun, calculate the maximum and minimum distance of bullet from the centre of the earth
during its subsequent motion.
v0 A
Solution The velocity of projection of bullet w.r.t. the earth, v0
v0
2 2
Êv ˆ
vA = v02 +Á 0˜ B a
Ë 2¯ r
vB
GM
Here, v0 =
a
By conservation of angular momentum between A and B, we have
mv0a = mvr ...(i)
Fig. 4.115
From conservation of mechanical energy, we have
1 GMm 1 GMm
mvA2 - = mv 2 - ...(ii)
2 a 2 r
Solving equations for r to get required result.
1 Ê 2 v02 ˆ GMm 1 GMm
m Á v0 + ˜ - = mv 2 -
2 Ë 4¯ a 2 r
5 2 GM v 2 GM
v0 - = - ...(iii)
8 a 2 r
GM
Here, v0 =
a
av0
v =
r
Put the value is Eq. (iii)
5 GM GM a 2 Ê GM ˆ GM
- = 2Á ˜-
8 a a r Ë 2a ¯ r
760പWŚLJƐŝĐƐĨŽƌ//dͲ:͗DĞĐŚĂŶŝĐƐ//

3 a 1
- = 2 -
8a 2r r
– 3r2 = 4a2 – 8ar
3r2 – 8ar + 4a2 = 0
8 a ± 64 a 2 - 48 a 2
r =
6
8a ± 4a
r =
6
2a
r = 2a and
3

Problem 4.110 If the radius of the earth were to shrink by one percent, its mass remaining
the same, the acceleration due to gravity on the earth’s surface would (i) decrease, (ii) remain
unchanged, (iii) increase. (IIT-JEE, 1981)
Solution Consider the case of a body of mass m placed on the earth’s surface (mass of the
earth M and radius R). If g is acceleration due to gravity, then
Mm GM
mg = G 2
or g= ...(1)
R R2
where G is universal constant of gravitation.
Now, when the radius is reduced by 1%, i.e., radius becomes 0.99 R, let acceleration due
to gravity be g¢, then
GM
g¢ = ...(2)
(0.99R)2
From Eq. (1) and (2), we get
g¢ R2 1
= 2
=
g (0.99R) (0.99)2
2
Ê 1 ˆ
\ g¢ = g ¥ Á
Ë 0.99 ˜¯
or g¢ > g
Thus, the value of g is increased.
Problem 4.111 Earth’s mass is 80 times that of the moon and their diameters are 12800
and 3200 km respectively. What is the value of g at the moon? g on the earth = 980 cm/sec2.
Solution Let the mass of the moon be m, then the mass of the earth M = 80 m.
If ge is the acceleration due to gravity on the earth then
GM G(80 m)
ge = =
R12 (6400)2
Similarly, if gm is the acceleration due to gravity on the moon then
Gm Gm
gm = =
R22 (1600)2
'ƌĂǀŝƚĂƟŽŶപ761

gm Gm (6400)2 (6400)2 1
\ = 2
¥ = 2
=
ge (1600) G (80 m) (1600) ¥ 80 5
1 980
gm = ¥ ge = = 196 cm/sec2
5 5

Problem 4.112 Find the intensity of gravitational field when a force of 100 N acts on a body
of mass 10 kg in the gravitational field. (EAMCET, 2011)
Solution Here F = 100 N, m = 10 kg
Intensity of gravitational field,
F 100 N
E= = = 10 N kg–1.
m 10 kg

Problem 4.113 Two bodies of masses 10 kg and 1000 kg are at a distance 1 m apart. At
which point on the line joining them will the gravitational field intensity be zero?
Solution Let the resultant gravitational intensity be zero at distance x from the mass of 10
kg on the line joining the centres of the two bodies. At this point, the gravitational intensities
due to the two bodies must be equal and opposite.
G ¥ 10 G ¥ 1000
\ 2
=
x (1 - x )2
or 100x2 = (1 – x)2 or 10x = 1 – x
or 11x = 1 or x = 1/11 m.
Problem 4.114 Two masses, 800 kg and 600 kg, are P
at a distance 0.25 m apart. Compute the magnitude
of the intensity of the gravitational field at a point
EB
m

distance 0.20 m from the 800 kg mass and 0.15 m EA 0.1


20

5m
0.

from the 600 kg mass.


Solution Let A and B be the positions of the two
masses and P the point at which the intensity of the A B
800 kg 0.25 m 600 kg
gravitational field is to be computed.
Gravitational intensity at P due to mass at A, Fig. 4.116
GM 800
EA = 2
=G = 20,000 G, along PA
r (0.20)2
Gravitational intensity at P due to mass at B,
600 80 , 000
EB = G 2
= , along PB
(0.15) 3
In DAPB,
PA2 + PB2 = AB2
\ –APB = 90°
762പWŚLJƐŝĐƐĨŽƌ//dͲ:͗DĞĐŚĂŶŝĐƐ//

Hence, the magnitude of resultant gravitational intensity at P is


2
Ê 80 , 000 ˆ
EA2 + EB2 = G (20 , 000)2 Á
Ë 3 ˜¯
E =

10 , 000
= 6.666 ¥ 10 -11 ¥ = 2.22 ¥ 10–6 N.
3

Problem 4.115 A planet A moves along an elliptical orbit ur


v0
around the sun. At the moment when it was at a distance r0 A
from the sun its velocity was equal to v0 and the angle between a
r0
the radius vector r0 and the velocity vector v0 was equal to a. C B
r2 r1
Find the maximum and minimum distances that will separate S
this planet from the sun during its orbital motion.
Solution See Fig. 4.117. Fig. 4.117
Angular momentum at point A
LA = r0 ¥ mv0 = mv0r0 sin a
Angular momentum at point B
LB = r1 ¥ mv1 = mv1r1
According to law of conservation of angular momentum
mv0 r0 sin a = mv1r1
v0 r0 sin a
\ v1 = ...(1)
r1

Applying the law of conservation of total energy at points A and B, we get


1 Gm Ms 1 Gm Ms
mv02 – = mv12 –
2 r0 2 r1
2G Ms 2G Ms
or v02 – = v12 - ...(2)
r0 r1
Substituting the value of v1 from Eq. (1) in Eq. (2), we get

2G Ms v 2 r 2 sin 2 a 2G Ms
v 02 – = 0 0 2 -
r0 r1 r1
Ê 2 2G Ms ˆ 2
ÁË v0 - r ˜¯ r1 = v0 r0 sin a – 2G Ms r1
2 2 2
or
0

Ê 2 2G Ms ˆ 2
ÁË v0 - r ˜¯ r1 + 2G Ms r1 – v0 r0 sin a = 0
2 2 2
or
0
'ƌĂǀŝƚĂƟŽŶപ763

Ê 2G Ms ˆ 2 2
- 2G Ms ± 4G 2 Ms2 + 4 Á v02 - (v0 r0 sin 2 a )
Ë r0 ˜¯
\ r1 =
Ê 2G Ms ˆ
2 Á v02 -
Ë r0 ˜¯

Ê 2G Ms ˆ
GMs ± G 2 Ms2 - Á - v02 ˜ (v02 r02 sin 2 a )
Ë r0 ¯
=
Ê 2G Ms ˆ
ÁË r - v02 ˜
0 ¯

v02 r02 sin 2 a Ê 2 2ˆ


1± 1- ÁË r - v0 ˜¯
G Ms 0
=
Ê2 v02 ˆ
ÁË r -
0 G Ms ˜¯

[ ± 1 - (2 - h) h sin 2 a ] r0 v02
= where h=
(2 - h) GMs

Problem 4.116 Two heavy spheres each of mass E1 E2


P
100 kg and radius 0.1 m are placed 1.0 m apart on B
a horizontal table. What is the gravitational field
and potential at the mid-point of the line joining the
0.5 m 0.5 m
centres of the spheres? Take G = 6.67 ¥ 10–11 Nm2 kg–2.
Solution Let A and B be the positions of the two Fig. 4.118
spheres and P be the mid-point of AB.
Gravitational field at P due to mass at A,
GM G ¥ 100
E1 = 2
= , along PA
r (0.5)2
Gravitational field at P due to mass at B,
G ¥ 100
E2 = , along PB
(0.5)2
As E1 and E2 have equal magnitudes but opposite directions, so the resultant gravitational
field at P is zero.
As gravitational potential is a scalar quantity, so total potential at P is
GM GM 2GM
V = VA + VB = - =-
r r r
2 ¥ 6.67 ¥ 10 -11 ¥ 100
= = – 2.668 ¥ 10– 8 J kg–1.
0.5
764പWŚLJƐŝĐƐĨŽƌ//dͲ:͗DĞĐŚĂŶŝĐƐ//

Problem 4.117 A geostationary satellite orbits around the earth in a circular orbit of radius
36,000 km. Then, what will be the time period of a spy satellite orbiting a few hundred km
above the earth’s surface (Rearth = 6,400 km)? (BITSAT, 2006)
Option: (a) 4 h (b) 2 h (c) 3 h (d) 1.7 h
Solution As T μ R and Re = 6400 km, therefore
2 3

3
T Ê 6400 ˆ
=Á ˜ or T = 1.7 h.
(24)2 Ë 36000 ¯
For the spy satellite, R is slightly greater than Re.
So Ts > T or Ts  2 h.
Problem 4.118 Two stars each of 1 solar mass (= 2 ¥ 1030 kg) are approaching each other
for a head-on collision. When they are at a distance 109 km, their speeds are negligible. What
is the speed with which they collide? The radius of each star is 104 km. Assume the stars to
remain undistorted until they collide. Use the known value of G.
Solution Mass of each star, M = 2 ¥ 1030 kg
Radius of each star, R = 107 m
Initial PE of the two stars when they are 1012 m apart,
GM ¥ M GM 2
Ei = = - 12 [r = 109 km = 1012 m]
r 10
When the stars are just to collide, distance between their centres
= twice the radius of each star = 2R = 2 ¥ 107 m
Final PE of the two stars when they just collide,
GM ¥ M GM 2
Ef = =-
2R 2 ¥ 107
Change in PE of the stars
GM 2 Ê GM 2 ˆ
= Ei – Ef = - 12
-Á- 7˜
10 Ë 2 ¥ 10 ¯

GM 2 GM 2 GM 2 È GM 2 GM 2 ˘
= - Í∵ << ˙
2 ¥ 107 1012 2 ¥ 107 Î 1012 2 ¥ 107 ˚
Let v be the velocity of each star just before collision. Then
Change in KE of the stars
= Final KE – Initial KE
1
= 2 ¥ Mv2 – 0 = Mv2
2
By conservation of energy,
GM 2
Mv2 =
2 ¥ 107
'ƌĂǀŝƚĂƟŽŶപ765

GM 2 6.67 ¥ 10 -11 ¥ 2 ¥ 10 30
or v = =
2 ¥ 107 2 ¥ 107
= 2.6 ¥ 106 ms–1.
Problem 4.119 At a point above the surface of the earth, the gravitational potential is – 5.12
¥ 107 J kg–1 and the acceleration due to gravity is 6.4 ms–2. Assuming the mean radius of the
earth to be 6400 km, calculate the height of this point above the earth’s surface.
(DCE, 2010)
Option: (a) 1600 km (b) 1400 km (c) 8000 km (d) 6000 km
Solution Let r be the distance of the given point from the centre of the earth. Then
Gravitational potential,
GM
V =- = – 5.12 ¥ 107 J kg–1 ...(i)
r
Acceleration due to gravity,
GM
g = = 6.4 ms–2 ...(ii)
r2
Dividing (i) by (ii),
5.12 ¥ 107
r = = 8 ¥ 106 m = 8000 km
6.4
Height of the point from the earth’s surface
= 8000 – 6400 = 1600 km.
Problem 4.120 The radius of the earth is 6.37 ¥ 106 m, its mean density is 5.5 ¥ 103 kg m–3 and
G = 6.66 ¥ 10–11 Nm2 kg–2. Determine the gravitational potential on the surface of the earth.
Solution Here R = 6.37 ¥ 106 m,
r = 5.5 ¥ 103 kgm–3, G = 6.66 ¥ 10–11 Nm2 kg–2
Mass of the earth,
4
M = Volume ¥ density = pR3r
3
Gravitational potential on the earth’s surface
GM G 4 4
V= = - ¥ p R3 r = - p GR2 r
R R 3 3
4
= – ¥ 3.14 ¥ 6.66 ¥ 10–11 ¥(6.37 ¥ 106)2 ¥ 5.5 ¥ 103
3
= – 6.22 ¥ 107 J kg–1.
Problem 4.121 Three mass points each of mass m are placed at the vertices of an equilateral
triangle of side l. What is the gravitational field and potential due to three masses at the
centroid of the triangle?
Solution In Fig. 4.119, three mass points, each of mass m, are placed at three vertices of
equilateral DABC of side l. If O is the centroid of the triangle, then OA = OB = OC.
766പWŚLJƐŝĐƐĨŽƌ//dͲ:͗DĞĐŚĂŶŝĐƐ//

From right DODB, A


M
BD l /2
cos 30° = =
OB OB
l/2 l/2 1 ur
or OB = = = ur E1
cos 30∞ 3 /2 3 E E E
ur
Gravitational fields at O due to mass points at A, B and 120° E3
C are as follows: ur O
 E2
Gm Gm 3Gm
E1 = 2
= 2
= 2 , along OA 30°
(OA) (l / 3 ) l m m
B D C
Gm 3Gm  l/2
E2 = 2
= 2 , along OB l
(OB) l
 Fig. 4.119
Gm 3Gm
E3 = = , along OC
(OC )2 l2
 
Angle between E1 and E 2 is 120°. Their resultant is

E = E12 + E22 + 2E1E2 cos 120∞

3Gm Ê 1 ˆ 3Gm 
= 1 + 1 + 2 ¥ Á - ˜ = 2 , along OF
l 2 Ë 2¯ l
 
Clearly, E is equal and opposite to E 3 , hence the resultant gravitational field at O is zero.
As gravitational potential is a scalar quantity, so the total gravitational potential at O is
Gm Gm Gm
V = V1 + V2 + V3 = – - -
OA OB OC

3Gm 3Gm l
=- =- ∵ OA = OB = OC =
OA l/ 3 3

Gm
or V = -3 3 .
l
m m
Problem 4.122 Find the potential energy of a system of four A B
particles, each of mass m, placed at the vertices of a square
of side l. Also obtain the potential at the centre of the square.
(IIT-JEE, 2013)
l
O
Solution In Fig. 4.120,
AB = BC = CD = DA = l
D C
\ AC = BD = l 2 + l 2 = 2l m l m

OA = OB = OC = OD = 2 l / 2 = l / 2 Fig. 4.120
'ƌĂǀŝƚĂƟŽŶപ767

By the principle of superposition, total potential energy of the system of particles is


U = UBA + ( UCB + UCA) + (UDA + UDB + UDC)
= 4UBA + 2UDB
[∵ UBA = UDA = UDC = UCB, UCA = UDB]

Ê Gmm ˆ Ê Gmm ˆ
= 4Á- ˜¯ + 2 Á -
Ë l Ë l/ 2 ˜¯

2 Gm2 È 1 ˘ 2 Gm2
=- Í 2 + 2 ˙ = - l [2 + 0.707 ]
l Î ˚

5.41 Gm2
=-
l

Problem 4.123 Consider two satellites, A and B, of equal mass m


moving in the same circular orbit or radius r around the earth, of mass A r B
Me, but in opposite sign of rotation and therefore on a collision course
(See Fig. 4.121). Earth
(a) In terms of G, Me, m and r, find the total mechanical energy EA +
EB of two-satellite plus-earth system before collision.
(b) If the collision is completely inelastic so that wreckage remains
as one piece of tangled material (mass = 2 m), find the total Fig. 4.121
mechanical energy immediately after collision.
(c) Describe the subsequent motion of the wreckage.
Solution
(a) The total mechanical energy of each satellite
- GMe m
=
2r
The total mechanical energy of the two satellite-earth system

Ê -GMe m ˆ
= 2¥Á
Ë 2r ˜¯
GMe m
=-
r
(b) For collision to occurs, mv – mv = (2m)v¢
\ v¢ = 0
Thus, the wreckage of mass 2m will fall vertically.
(c) The KE of the satellite will vanish, the potential energy
Ê - GMe m ˆ
= 2Á ˜¯
Ë r
768പWŚLJƐŝĐƐĨŽƌ//dͲ:͗DĞĐŚĂŶŝĐƐ//

- GMe m
=
r
- 2GMe m
Thus, total mechanical energy = Ans.
r

Problem 4.124 Given a thin homogeneous disc of radius a and mass m1. A particle of mass
m2 is placed at a distance l from the disc on its axis of symmetry. Initially, both are motionless
in free space but they ultimately collide because of gravitational attraction. Assuming a <<
l, show that the relative velocity at the time to collision is given by
1/2
È Ê 2 1ˆ ˘
Í 2G (m1 + m2 ) ÁË a - l ˜¯ ˙
Î ˚

Solution See Fig. 4.122


Let dm be the mass of the ring considered (shown
dotted in Fig. 4.122)
dm = (2prdr) ¥ surface change density r
= 2prdr ¥ (m1/p a2) = (2m1r dr/a2) a m2
We know that the potential due to a circular ring l
of radius r and width dr is given by
m1 dr
G dm 2 Gm1 r dr
df = =-
r 2 + l2 a2 (r 2 + l 2 ) Fig. 4.122
The total potential f at a distance l from the disc is given by
a 2 Gm1 r dr
f = -Ú
0 2
a (r 2 + l 2 )
2G m1
=– 2
[ a 2 + l 2 - l]
a
The initial potential energy
2G m1 m2
Vi = f m2 = – [ ( a 2 + l 2 ) - l]
a2
2G m1 m2
Now, Vf = (∵ l = 0)
a

G m1 m2
Further Vi = a << l
l
Change in potential energy = Vi – Vf
Ê 2 1ˆ
= G m 1 m2 Á - ˜
Ë a l¯
'ƌĂǀŝƚĂƟŽŶപ769

Applying the law of conservation of energy


Total KE before impact = Vi – Vf
1 1 Ê 2 1ˆ
\ m1 v12 + m2 v22 = G m1 m2 Á - ˜
2 2 Ë a l¯
1/2
È 2G Ê 2 1ˆ ˘ m1
v1 = m2 Í ÁË - ˜¯ ˙ ; v2 = – v1
Î m1 + m2 a l ˚ m2

Relative velocity at impact v1 – v2

Ê m ˆ
\ v1 – v2 = Á 1 + 1 ˜ v1
Ë m2 ¯
1/2
È Ê 2 1ˆ ˘
= Í 2 G (m1 + m2 ) Á - ˜ ˙
Î Ë a l¯˚

Problem 4.125 Find the potential due to the system of five particles each of mass m placed
at the vertices of a regular hexagon at O.
m l m

l l

m O

l l

m l m

Fig. 4.123 (a)


2 1

Solution V = V1 + V2 + V3 + V4 + V5 3l l

3 2l
Gm Gm Gm Gm Gm
=- - - - -
l 3l 2l 3l l 3l l

Gm Ê 1 2 ˆ 4 5
=- 2+ +
l ÁË 2 ˜
3¯ Fig. 4.123 (b)
m m
Problem 4.126 Eight identical particles each of mass m
are situated at the vertices of a cube of side l. Find the
gravitational potential energy of interaction of the system m m
of eight particles. 1

Solution In a cube we have eight corners having equal


masses.
Hence, V1 = V2 = ... = V8 and m1 = m2 = ... = m8 = m
m m
1
Then using U = SmiVi,
2 m m

Fig. 4.124
770പWŚLJƐŝĐƐĨŽƌ//dͲ:͗DĞĐŚĂŶŝĐƐ//

Ê1 ˆ
we have U = Á m1V1 ˜
Ë2 ¯
where m1 = m and V1 = potential at point 1.
This gives U = 4mV1
mÊ3 3 1 ˆ 12Gm2 Ê 1 1 ˆ
V1 = - G Á + + ˜ , we have U = - ÁË 1 + + ˜ Ans.
l Ë1 2 3 ¯ l 2 3 3¯

Problem 4.127 The density inside a solid sphere of radius a is given by r = r0a/r, where r0
is the density at the surface and r denotes the distance from the centre. Find the gravitational
field due to this sphere at a distance 2a from its centre.
Solution Volume of mass strip dV = (4pr2) dr mass of this
strip dm = (r) (dV)
Ê r aˆ
= Á 0 ˜ (4pr2)dr
Ë r ¯
= (4p ar0) (rdr) r
a
\ Mass of whole sphere = Ú0 (4p ar0)r dr dv
m = (2p a3r0)
Gm G(2p a 3 r0 ) Fig. 4.125
E = 2
= 2
( 2 a) 4a
p G r0 a
=
2
R C
Problem 4.128 Two concentric shells of mass M1 and M2 are M2
concentric as shown in Fig. 4.126. Calculate the gravitational force
M1
on m due to M1 and M2 at points P, Q and R. Q
a
Solution At P, F =0 b
P
GM1 m
at Q, F =
b2
G ( M1 + M2 )m
at R, F = Fig. 4.126
c2
Problem 4.129 A meteorite of mass m collides with a satellite of mass 10 m orbiting around
a planet in a circular path of radius R. Due to perfectly inelastic collision, the meteorite
sticks to the satellite and the satellite now follows an orbit whose minimum distance from
the planet is R/2. Determine the velocity v of the meteorite before collision. Mass of the
planet is M.
'ƌĂǀŝƚĂƟŽŶപ771

Solution After collision the orbit of the satellite becomes elliptical.


v0
v¢ q

v m
10 m 11 m
R R/2

Fig. 4.127

Let the velocity vector of the satellite be at an angle q as shown in Fig. 4.128.
From conservation of momentum, we get
mv = 11mv¢ sin q ...(1)
and 11mv0 = 11mv¢ cos q ...(2)
From conservation of angular momentum just after the collision and point of nearest
approach, we get
R
11mv¢ cos qR = 11mu ...(3)
2
From conservation of energy, we get
GM ◊ 11m 1 GM ◊ 11m 1
- + 11mv¢ 2 = - + ◊ 11mu¢ 2 ...(4)
R 2 R/2 2
From Eq. (1),
v
v¢ sin q =
11

From Eq. (2),


10v0
v¢ cos q =
11
On squaring and adding, we have
v 2 + 100v0
v¢2 =
121
Also from Eq. (3),
20
u = 2v¢ cos q = v0
11
Substituting the values of v¢2 and u in Eq. (4), we have

GM 1 Ê v 2 + 100v02 ˆ 2GM 1 400 2


- + Á ˜¯ = - + ◊ v0
R 2Ë 121 R 2 121
772പWŚLJƐŝĐƐĨŽƌ//dͲ:͗DĞĐŚĂŶŝĐƐ//

But orbital speed in a circular orbit is given by


GM
v0 =
R
On substituting v0 and solving for v, we get

58GM
\ v =
R

Problem 4.130 Two earth’s satellites move in a common plane along circular orbits. The
orbital radius of one satellite r = 7000 km while that of the other satellite is Dr = 70 km less.
What time interval separates the periodic approaches of the satellites to each other over the
minimum distance?
Solution We know that centripetal force = mv2/r
Now for the first satellite which is revolving about the earth (mass M and radius r)
Gm1 M m1v 2
2
=
r r
GM
\ v = ...(1)
r
Let T1 and T2 be the time periods for the first and the second satellites respectively. Then
2p r 2p 3/2
T1 = = r
v GM
2p
and T2 = (r - Dr )3/2
GM
{∵ second satellite is revolving in a radius (r – Dr)}
The period interval (T1 – T2) is given by
2p
T1 – T 2 = [r3/2 – (r – Dr)3/2]
GM
2p È 3/2 Dr ˆ ˘
3/2
3/2 Ê
= Í r - r ÁË 1 - ˜¯ ˙
GM Î r ˚

2p È Ê 3 Dr ˆ ˘
= r 3/2 Í1 - Á 1 - ˜
GM Î Ë 2 r ¯ ˙˚
2p Ê 3 Dr ˆ
r 3/2 Á
Ë 2 r ˜¯
= .
GM
Problem 4.131 A uniform hollow sphere has internal radius a and external radius b. Taking
the potential at infinity to be zero, show that the ratio of the gravitational potential at a point
on the outer surface to that on the inner surface is
'ƌĂǀŝƚĂƟŽŶപ773

2(b 3 - a 3 )
3b(b 2 - a 2 )

Solution Let the density of the hollow sphere be r. The mass M of the
hollow sphere is a
4 b
M = pr(b3 – a3)
3
GM
The gravitational potential on the outer surface is – ; so we have
b
4
G ◊ p r(b 3 - a 3 ) Fig. 4.128
V(b) = 3
b
Now we consider a point at a distance r from the centre of the sphere such that a < r <
b. The gravitational field at this point is due to the mass contained within the radius r, this
mass is
4
pr(r3 – a3)
3
The gravitational field strength is
4
G ◊ p r(r 3 - a 3 )
g= 3
r2
From the relation between the field intensity g and the gravitational potential, we have
a
V(a) – V(b) = – Úb g dr
4prG a r 3 - a 3
3 Úb r 2
= dr
a
4prG È r 2 a 3 ˘
= Í + ˙
3 Î2 r ˚b

4prG È a 2 b 2 a 3 a 3 ˘
= Í - + - ˙
3 Î2 2 a b˚

4prG È 3 a 2 b 2 a 3 ˘
= Í - - ˙
3 Î 2 2 b˚
After using the expression for V(b), we obtain

4prG Ê 2 a 3 3 a 2 b 2 a 3 ˆ
V(a) = Á-b + + - - ˜
3 Ë b 2 2 b¯

4prG Ê 3b 2 3 a 2 ˆ
= Á- + ˜
3 Ë 2 2 ¯
774പWŚLJƐŝĐƐĨŽƌ//dͲ:͗DĞĐŚĂŶŝĐƐ//

4prG Ê b 3 - a 3 ˆ
- Á ˜
V (b ) 3 Ë b ¯
So that =
V ( a) 4prG Ê 3b 3 3 a 2 ˆ
Á- + ˜
3 Ë 2 2 ¯
2(b 3 - a 3 )
=
3b(b 2 - a 2 )

Problem 4.132 A box is dumped from a rocket travelling outward from the earth at a speed
of 1800 m/s when 1600 km above the earth’s surface. Estimate the speed of the box when it
reaches the earth’s surface. Estimate the speed of the box when it reaches the earth’s surface,
ignoring air resistance.
Option: (a) 5320 m/s (b) 6000 m/s (c) 7000 m/s (d) 4000 m/s
Solution At the instant of projection speed of box relative to the earth is equal to the speed
of rocket.
Applying conservation of energy,
1 GME m 1 GME m
mv12 = = mv22 -
2 r1 2 r2
where v1 = 1.80 ¥ 103 m/s,
r1 = 1.60 ¥ 106 m + 6.38 ¥ 106 m
= 7.98 ¥ 106 m
and r2 = 6.38 ¥ 106 m (the radius of the earth).
On solving for v2, we have

Ê 1 1ˆ
v2 = v12 - 2GME Á - ˜
Ë r1 r2 ¯
Ê 1 1 ˆ
= (1.80 ¥ 10 3 )2 - 2(6.67 ¥ 10 -11 ) ¥ (5.98 ¥ 10 24 ) ¥ ÁË 7.98 ¥ 106 - 6.38 ¥ 106 ˜¯
= 5320 m/s
The direction of the velocity of rocket is not required to solve this problem. This is the
advantage of energy methods.
Problem 4.133 A projectile is fired straight up from the surface of the earth with an initial
speed v1 = 9 km/s. Neglecting air resistance, find the maximum height to which the projectile
will rise.
Solution We will determine the maximum height by applying the energy conservation law.
We take the earth’s surface as initial point and maximum height as final point.
Ui + KEi = Uf + KEf
GME m 1 GME m
- + mvi2 = +0
RE 2 r
'ƌĂǀŝƚĂƟŽŶപ775

1 2 GME Ê RE ˆ
vi = ÁË 1 - ˜
2 RE r ¯

Ê R ˆ
= gRE Á i - E ˜ ...(1)
Ë r ¯
GME
where =g
RE2
Eq. (1) becomes
RE vi2
1– =
r 2 gRE
RE
r = ...(2)
1 - vi2 / 2 gRE

vi2 (9000)2
=
2 gRE 2(9.81)(6.40 ¥ 106 )
= 0.645
RE
Thus, r =
1 - 0.645
= 2.81RE
h = r – RE = 1.81RE
Problem 4.134 Consider the motion of a planet in elliptical orbit about the sun. Assuming
the mass of the planet to be considerable in comparison to the sun, obtain expression for (a)
angular momentum of planet, (b) time period, and (c) binding energy.
Solution
(a) The magnitude of angular momentum about the centre of mass of the planet and the
sun,
dq
LP = mv^r = mr 2 …(1)
dt
where v^ is the component of planet’s velocity perpendicular to r.
dq
v^ = r
dt
Areal velocity,
dA 1 dq LP
= r2 = ...(2)
dt 2 dt 2m
where m is the reduced mass.
The period of orbit is
Ê dA ˆ
T = (area of ellipse) ∏ Á
Ë dt ˜¯
...(3)

Area of ellipse = p (major axis) ¥ (minor axis)


776പWŚLJƐŝĐƐĨŽƌ//dͲ:͗DĞĐŚĂŶŝĐƐ//

= p a2 1 - e 2 ...(4)

2 pma2 1 - e 2
T = ...(5)
LP
The inward acceleration due to gravitational force is given by
L2P 1
a = ...(6)
m a (1 - e ) r 2
2 2

Thus, Newton’s second law of the internal motion of the sun and planet becomes
GMS MP L2P 1
2
= ...(7)
r ma (1 - e ) r 2
2

Hence, Lp2 = GMsMpma(1 – e2) ...(8)


Eliminating Lp from Eq. (8) and (5),
4p 2 ma 3
T2 =
GMS MP
4p 2 a 3
=
G( MS + MP )

Ê MS MP ˆ
Since the reduced mass, m = Á
Ë MS + MP ˜¯
(b) Distance of planet at perihelion = a(1 – e)
Angular momentum,
Lp = mva(1 – e)
1 L2P
Kinetic energy = mv2 =
2 2ma 2 (1 - e 2 )
(c) The binding energy EB of the moving planet is the binding energy if it were stationary
at perihelion minus kinetic energy.
GMS MP L2P
EB = -
a(1 - e ) 2ma 2 (1 - e 2 )
Substituting expression for L2p, we obtain
GMS MP GMS MP (1 - e 2 )
EB = -
a(1 - e ) 2 a(1 - e 2 )

GMS MP Ê 1 (1 + e ) ˆ
= -Á -
a Ë1- e
2
2(1 + e ) ˜¯
GMS MP
E =
2a
'ƌĂǀŝƚĂƟŽŶപ777

Problem 4.135 Two identical spheres each of mass m and radius r are at rest with their
centre 4r apart on a smooth horizontal surface. When they are released they move towards
each other due to gravitational attraction. How long after will they collide?
Solution From conservation of energy, when the separation between centers is x < 4r, we
have
2
Ê1 ˆ Gm Gm2
2 Á mv 2 ˜ - = -
Ë2 ¯ x 4r
Ê 4r - x ˆ
v2 = Gm Á
Ë x ˜¯
or

1 dx ( 4r - x)
or - = v = Gm
2 dt 4 rx
4r x = 2r x
or Ú dt =
2
Ú
Gm x = 4 r ( 4r - x)
dx

r 4r x
or t = Ú
Gm 2 r ( 4r - x)
dx

r
= [r(p + 2)]
Gm
r3
= (p + 2)
Gm

Problem 4.136 The distance between the earth and the moon is about 3.8 ¥ 105 km. At what
point or points will the gravitational field strength of the earth-moon system be zero? Given
mass of the earth is 81 times the moon’s mass.
Solution In this problem, the earth and the moon will be treated as point masses as they
are far apart and as in case of a point mass E = GM/r2, the field due to both the masses will
be zero at r = •. In addition to it, the field will also be zero at a point in between the earth
and the moon where the pull of the earth balances the pull of the moon. If this point is at a
distance x from the earth,
GME GM M 81 1
2
= or =
x (3.8 ¥ 108 - x )2 x2 (3.8 ¥ 108 - x )2
[as ME = 81 MM]
or 9(3.8 ¥ 10 – x) = x
8

or x = 3.42 ¥ 108 m = 3.42 ¥ 105 km


Note:
(i) The field due to the earth or the moon at this point will be
GME (6.67 ¥ 10 -11 ) (6 ¥ 10 24 )
E= = = 3.42 ¥ 10–3 N/kg
x2 (3.42 ¥ 108 )2
778പWŚLJƐŝĐƐĨŽƌ//dͲ:͗DĞĐŚĂŶŝĐƐ//

(ii) This problem also indicates that an astronaut in going from the earth to the moon will
face weightlessness when he is at a distance of 3.42 ¥ 105 km from the earth, between
the earth-moon system and the engines of his spaceship are inactive.
Problem 4.137 Find the speeds of a planet of mass m in its perihelion and aphelion
positions. The semimajor axis of its orbit is a, eccentricity is e and the mass of the sun is M.
Also find the total energy of the planet in terms of the given parameters.
Solution Let v1 and v2 be the speeds of the planet at v2
perihelion and aphelion positions.
r1 = a(1 – e) r1 r2
P A
S
and r2 = a(1 + e) ...(i)
Applying conservation of angular momentum of the
v1
planet at P (perihelion) and A (aphelion)
Fig. 4.129
mv1r1 sin 90° = mv2r2 sin 90°
or v1r 1 = v 2r 2 ...(ii)
Applying conservation of mechanical energy in these two positions, we have
1 2 GMm 1 GMm
mv1 – = mv22 - ...(iii)
2 r1 2 r2
Solving Eqs. (i), (ii) and (iii), we get

GM Ê 1 + e ˆ
v1 =
a ÁË 1 - e ˜¯

GM Ê 1 - e ˆ
and v2 =
a ÁË 1 + e ˜¯
Further, total energy of the planet
1 GMm
E = mv12 -
2 r1
1 È GM Ê 1 + e ˆ ˘ GMm
= m -
2 ÍÎ a ÁË 1 - e ˜¯ ˙˚ a(1 - e )

GMm ÈÊ 1 + e ˆ ˘
= Í ÁË ˜¯ - 1˙
a(1 - e ) Î 2 ˚
GMm Ê e - 1ˆ
= Á ˜
a(1 - e ) Ë 2 ¯
GMm
or E =
2a
'ƌĂǀŝƚĂƟŽŶപ779

Problem 4.138 Find the work done in transferring a point mass m from the centre A of the
ring to the centre B of the thin spherical shell.
M
M

A 3R B

2R R

Fig. 4.130

Solution W = m (vB – vA)


Ê GM GM ˆ -6GM
vB = - Á + ˜=
Ë 5R R ¯ 5R

Ê GM GM ˆ -4GM
or vA = - Á + ˜=
Ë R 3R ¯ 3R

-2GMm
or W = Ans.
15R

Problem 4.139 What is the fractional decrease in the value of free-fall acceleration g for a
particle when it is lifted from the surface to an elevation h? (h << R).
GM
Solution g= fi
R2
dg -2GM
=
dR R3
dg -2GM 1
fi = ◊
h R3 R
dg Ê hˆ
fi = - 2Á ˜
g Ë R¯

Problem 4.140 A body is projected vertically upwards from the surface of the earth with
a velocity sufficient to carry it to infinity. Calculate the time taken by it to reach height h.
Solution If at a distance r from the centre of the earth the body has velocity v, by conserva-
tion of mechanical energy

1 Ê GMm ˆ 1 Ê GMm ˆ
mv 2 + Á 2
˜¯ = mve + ÁË - ˜
2 Ë r 2 R ¯

2GMm Ê R ˆ
fi v2 = v2e + Á - 1˜¯
R Ër
780പWŚLJƐŝĐƐĨŽƌ//dͲ:͗DĞĐŚĂŶŝĐƐ//

GM
Now, ve = 2gR and g=
R2
fi v2 = 2gR + 2gR (R/r – 1)

2 gR2
v =
r
dr 2g
fi =R
dt r
t R+h
1
fi Ú dt = R 2g
Ú rdr
0 R

1 2R ÈÊ hˆ
3/2 ˘
fi t = ÍÁË 1 + ˜¯ - 1˙
3 g Î R ˚

Problem 4.141 An object weighs 10 N at the north pole of the earth. In a geostationary
sattellite distant 7R from the centre of the earth (of radius R) what will be its (a) true weight,
and (b) apparent weight.
Solution
(a) The true weight of a body is given by mg and with height g decrease. Let it be g¢

WS mg ¢ 1 È g ˘
So, = = as g ¢ =
WE mg [1 + ( h / R)]2 ÍÎ 2˙
[1 + ( h / R)] ˚

But here h = 7R – R = 6R, i.e., h/R = 6


WE
So, WS =
[1 + 6]2
10
=  0.2 N
49
(b) If g¢ is the acceleration due to gravity of the earth at the position of satellite, the
apparent weight of a body in the satellite will be
Wapp = m(g¢ – a)
But as satellite is a free falling body, i.e., g¢ = a
So, Wapp = 0

Problem 4.142 What should be the length of the day so that the weight of a body on the
equator of the earth becomes zero? Given that radius of the earth is 6400 km and acceleration
due to gravity on its surface is 9.8 m/s2.
'ƌĂǀŝƚĂƟŽŶപ781

Solution The effective value of g at equator is zero let w¢ be the angular velocity of rotation
g – Rw¢ 2 = 0
g 9.8 1 rad
or w¢ = = =
R 6400 ¥ 10 3 808 s

2p R
So, T¢ = = 2p
w¢ g
i.e., T ¢ = 6.28 ¥ 808s  84.6 min  1.4 hr
Note: As the present length of the day is 24 hrs and T μ (1/w),
w¢ T 24
= = = 17
w T ¢ 1.4
i.e., if the earth starts spinning about 17 times of its present value, the weight of a body on
the equator will become zero.
Problem 4.143 A solid sphere of mass m and radius r is placed inside
a hollow thin spherical shell of mass M and radius R as shown in M
Fig. 4.131. A particle of mass m¢ is placed on the line joining the two
centres at a distance x from the point of contact of the sphere and the O
shell. Find the magnitude of the resultant gravitational force on this R x
particle due to the sphere and the shell if mO
r
1. r < x < 2r
Gmm¢(2r - x ) Gmm¢( x - r ) Fig. 4.131
(a) 3
(b) 3
2r 2r
Gmm¢( x - r ) Gmm¢(2 x - r )
(c) 3
(d)
r r3
2. 2r < x < 2R
Gmm¢ Gmm¢ Gmm¢ 2Gmm¢
(a) 2
(b) 2
(c) 3
(d)
4( x - x ) ( x - x) (x - r) ( x - r )2
3. x > 2R
2Gmm¢ Gmm¢ GMm¢ 2Gmm¢
(a) 2
+ 2
(b) 2
+
(x - r) (x + r) 2( x - R) ( x - r )2
GMm¢ Gmm¢ GMm¢ Gmm¢
(c) + (d) +
( x + R )2 ( x + r )2 ( x - R )2 ( x - r )2
782പWŚLJƐŝĐƐĨŽƌ//dͲ:͗DĞĐŚĂŶŝĐƐ//

Solution 1. (c), 2. (b), 3. (d). When 2r < x < 2R


m
then the force will be due to sphere only.
Gmm¢ M
F= x
( x - r )2
m
When x > 2R the sphere and the shell both will contribute to the
Fig. 4.132
GMm¢
F sphere =
( x - r )2
M
GMm¢
Fshell =
( x - R )2 O
R x
GMm¢ GMm¢ mO
r
F = 2
+
( x - R) ( x - r )2
Fig. 4.133
Problem 4.144 A uniform metal sphere of radius a and mass M is
surrounded by a thin uniform spherical shell of equal mass and
radius 4a. The centre of the shell falls on the surface of the inner
sphere. a
o
1. Find the gravitational field intensity at point P1.
4a
GM GM
(a) (b)
16 a 2 8 a2 P1
a
P2 a
GM GM
(c) (d)
2a2 4 a2 Fig. 4.134
2. Find the gravitational field intensity at point P2.
21GM 61GM 61GM 61GM
(a) 2
(b) 2
(c) 2
(d)
900 a 450 a 900 a 1800 a 2
GM GM GM GM 61GM
Solution 1. (a) EP = = 2. (c) EP = + =
( 4 a )2 16 a 2 ( 6 a )2 5a2 900 a 2
v0
Problem 4.145 A spaceship is sent to investigate a planet m
of mass M and radius R. While hanging motionless in space R
q M
at a distance 5R from the centre of the planet, the spaceship
fires an instrument package of mass m, which is much
5R
smaller than the mass of the spaceship. For what angle q
will the package just graze the surface of the planet?
Fig. 4.135
'ƌĂǀŝƚĂƟŽŶപ783

Solution Let the speed of the instrument package v0


be v when it grazes the surface of the planet. q R
Conserving angular momentum of the package m
about the centre of the planet
mv0 ¥ 5R sin q = mvr fi v = 5v0 sin q …(1)
Fig. 4.136
Conserving mechanical energy,
GMm 1 GMm 1
- + mv02 = - + mv 2
5R 2 R 2
1 4GMm
fi m (v2 – v02) =
2 5R
8GM
v2 – v02 = …(2)
5R
Substituting the value of v from Eq. (1) in Eq. (2)
8GM
25 v20 sin2 q – v02 =
5R
1 8GM
fi sin q = 1+ 2
5 5v0 R
È1 Ê 8GM ˆ ˘
or q = sin–1 Í Á 1 + 2 ˜ ˙
ÍÎ 5 Ë 5v0 R ¯ ˙˚

Problem 4.146 A body is projected from the earth’s surface


at an angle of projection μ with the horizontal. Show that the
semi-major axis of the elliptical orbit of the body is independent
2a v
of the angle of projection. v0
Solution Let v0 be the velocity of projection and r be the a
maximum and minimum separations from the earth.
From conservation of angular momentum, we have
rmax
m(v0 cos a)R = mvr ...(1)
From conservation of energy, we have
rmin
1 GMm 1 GMm
mv02 – = mv 2 - ...(2) Fig. 4.137
2 R 2 r
Now we substitute the expression for v from Eq. (1) in (2) to get
2
È v cosa R ˘ Ê r - Rˆ È GM ˘
v02 - Í 0 = 2GM Á ÍÎas g = R2 ˙˚
Î r ˙
˚ Ë rR ˜¯

or v 02r2 – v02R2 cos2 a = 2gRr(r – R)


784പWŚLJƐŝĐƐĨŽƌ//dͲ:͗DĞĐŚĂŶŝĐƐ//

The roots of this quadratic equation are rmax and rmin.


rmax + rmin
The semi-major axis a =
2
Sum of roots of quadratic equation
=
2
1 È 2 gR ˘ 2
= Í ˙
2 ÍÎ 2 gR - v02 ˙˚

gR2
=
2 gR - v02
So, we can see that the semi-major axis is independent of the angle of projection a.
Problem 4.147 The density of atmosphere at a certain altitude V
is approximately 1.6 ¥ 10–11kg/m3. Determine the aerodynamic
drag experienced by a satellite with a cross-sectional area of
0.5 m2 and mass 10 kg orbiting at this height. What will be the A
velocity and height of the satellite per revolution? Assume the
circular orbit close to the earth’s surface.
Solution When a solid body moves through air, the collision Fig. 4.138
between molecules and the body is mainly inelastic. If A is the
frontal area of the body, the volume swept per second is AV. Mass of the air particles striking
per second is rAV. Momentum gained per second is rAv2.
Therefore, the frictional force acting on the satellite is
F = rAv2
Assuming low circular orbit, the orbital velocity is 8 cm/sec.
F = 1.6 ¥ 10–11 ¥ (0.5)(8 ¥ 103)2
ª 5.12 ¥ 10–4 N
The total energy of the satellite on a circular orbit of radius R is
1
E = - mv 2
2
GME m
=-
2RE
dE GmME
=+
dR 2RE2

(dE)(2RE2 )
dR =
GmME
whereas change in velocity is
dE
dv = -
mv
'ƌĂǀŝƚĂƟŽŶപ785

The energy of the satellite will change due to drag by DE = – 2pREF per revolution.
Therefore, the orbit radius will be decreased by
4p RE3
dR = F
GmME
3F
ª–
Gmr
where r is the mean density of the earth.
r = 5500 kg/m3
dR ª 0.4 km
The increment in velocity,
2p RE F
dv =
mv
= 0.5 m/sec
Problem 4.148 A small satellite revolves around a heavy planet in a circular orbit. At a
point on its orbit an impulse acts suddenly and instantaneously increases its kinetic energy
k times without change in its direction of motion. Show that in its subsequent motion the
Ê k ˆ
ratio of its maximum and minimum distance from the planet is Á assuming the mass
Ë 2 - k ˜¯
of the satellite is negligibly small compared to that of the planet.
Solution Let v0 be the velocity of the satellite at A when it v0
is in a circular orbit. From Newton’s second law,
GMm mv02 A
M
A
2
= R P
r r r

GM
or v02 = ...(1)
r
Fig. 4.139
Due to impulse the velocity v0 is increased to v such that
v2 = kv02. Let the maximum distance at A¢ of the satellite from P be R.
From conservation of angular momentum, we have
mVR = mvr
vr
or V =
R
From the law of conservation of energy, we have
GMm 1 GMm 1
- + mV 2 = - + mv 2
R 2 R 2
On eliminating V, we get

Ê 2GM ˆ
ÁË - v 2 ˜ R2 – 2GMR + v2r2 = 0
r ¯
786പWŚLJƐŝĐƐĨŽƌ//dͲ:͗DĞĐŚĂŶŝĐƐ//

v2 r v2 r rk
Therefore, R = = 2 =
2
GM
- v 2 2v0 - v
2
2-k
r
R k
or =
r 2-k

Problem 4.149 A man made satellite moves in an elliptical orbit around the earth. It was
put in the orbit at a point with a speed 1.2v, where v is the speed for a circular orbit at that
point. Find the ratio of maximum to minimum distance of the satellite from the earth.
Solution The satellite moves in an elliptical orbit with r1 v1
and r2 the minimum and maximum distances, then

GME r1 r2 B
v =
r1
GME = v2r1 ...(1) v2

From conservation of energy between points A and B, Fig. 4.140


we have
1 GME m 1 GME m
m(1.2v)2 - = m22 -
2 r1 2 r2
1.44v 2 GME v 2 GME
- = 2 - ...(2)
2 r1 2 r2
From conservation of angular momentum,
1.2vr1 = v2r2
1.2vr1
v2 = = 1.2vk ...(3)
r2
where k = r1/r2.
Now substituting v2 and GME in Eq. (2), we obtain
1 – k = 0.721(1 – k2)
which on solving for k gives k = 2.57
r2
Thus, = 2.57
r1

Problem 4.150 A satellite of mass 2m is in a circular orbit of radius 2R 2R


around the earth. By mistake, another satellite of mass m is put in the
same orbit and having opposite sense of rotation. The collision between
R
the two satellites is perfectly inelastic. Will the combined mass hit the
earth’s surface? If yes, at what angle to the horizontal? If no, what is its
minimum distance from the surface of the earth?
Fig. 4.141
'ƌĂǀŝƚĂƟŽŶപ787

Solution Mass of the earth = M, radius of the earth = R, speed of both the satellites
GM
before collision = u = , velocity of the combined mass after collision = v.
2R
By conservation of momentum
2mu – mu = 3mv
u 1 GM
fi v = =
3 3 2R
Total mechanical energy just after collision
2
1 Ê uˆ GM(3m)
= ( 3 m) Á ˜ -
2 Ë 3¯ 2R
mu2 3 GMm 17 GMm
= - =-
6 2 R 12 R
When length of the minimum radius vector = r
Total angular momentum of the combined mass = L (conserved)
L
then velocity can be expressed as =
(3m)(r )
At this moment total energy
1 L2 GM(3m)
= ( 3 m) 2 2
-
2 ( 3 m) r r
Ê GM ˆ
Putting L = Á 2m R˜
Ë 2R ¯
2 Ê GM ˆ R2 3GMm 17 GMm
Total energy = mÁ ˜ - =-
3 Ë 2R ¯ r 2 r 12 R
fi 2
17r – 36Rr + 4R = 02
2R
fi r = 2R;
17
2R
Hence, as r = < R, the combined mass hits the earth.
17
Now when it hits the earth, let its linear momentum = p
Therefore, its kinetic energy
p2 p2
= =
2(3m) 6 m
p2 3m
Total energy = - GM
6m R
2
p 3GMm 17 GMm
Hence, - =-
6m R 12 R
P2 19 GMm
fi =
6 m 12 R
19 GM
fi P = m
12 R
788പWŚLJƐŝĐƐĨŽƌ//dͲ:͗DĞĐŚĂŶŝĐƐ//

Hence, angular momentum = Rp cos q where q = angle at which the combined mass hits
the earth’s surface.
Angular momentum being conserved.
Ê GM ˆ
Rp cos q = 2m Á
Ë R ˜¯
R

Ê 19 GM ˆ Ê GM ˆ
fi RÁ m ˜¯ cos q = 2m ÁË ˜R
Ë 2 R R ¯
2 2 4 2 2
fi cos q = ¥ = =
2 19 19 2 19
Ê 2 2ˆ
fi q = cos–1 Á
Ë 19 ˜¯
.

Problem 4.151 If a planet was suddenly stopped in its orbit supposed to be circular show
that it would fall onto the sun in a time ( 2/8) times the period of the planet’s revolution.
Solution If the mass of the sun is M and radius of the planet’s orbit is r, then as v0 =
(GM / r ) ,
2p r r
T = = 2p r
v0 GM
4p 2 r 3
i.e., T2 = …(1)
GM
Now if the planet (when stopped in the orbit) has velocity v when it is at a distance x
from the sun, by conservation of mechanical energy,
1 Ê GMm ˆ GMm
mv 2 + Á - ˜¯ = 0 -
2 Ë x r
2
Ê dx ˆ 2GM È r - x ˘
or ÁË - ˜ =
dt ¯ r ÍÎ x ˙˚
dx 2GM (r - x )
i.e., - =
dt r x
t r 0È x ˘
or Ú0 dt = - Ú Í ˙ dx
2GM r Î (r - x ) ˚
Substituting x = r sin2 q and solving the RHS.
1/2
0 Ê r sin q ˆ
2
0 x
-Ú dx = - Ú Á
r (r - x) p /2 Ë r - r sin 2 q ˜
¯
0
= -rÚ (1 – cos 2q)dq
p /2
0
È 1 ˘ pr
= - r Íq - sin 2q ˙ =
Î 2 ˚ p /2 2
r Ê prˆ
t = ¥Á ˜
2GM Ë 2 ¯
'ƌĂǀŝƚĂƟŽŶപ789

which in the light if Eq. (1) reduces to


1
t = T
4 2
Ê 2ˆ
t =Á
Ë 8 ˜¯
i.e., T

Method 2: The time it takes for a mass m to fall from a distance r


to say, the sun can be approximated in the following way. Imagine Path of
imaginary
m in a very eccentric elliptical orbit (e ª 1) about the sun as shown comet
in Fig. 4.142. The semi-major axis r¢ of the orbit then period of our Sun
Earth
imaginary comet T¢ according to Kepler’s law will be given by r¢
(T ¢ )2 T2 r
3
= 3
= constant
(r ¢)
r 3
Ê r¢ ˆ
or T¢2 = T 2 Á ˜
Ë r¯ Fig. 4.142
2
T
or T¢2 = [as r¢ = r/2]
8
The time which the planet will take to fall onto the sun,
T¢ T Ê 2ˆ
t = = =Á ˜T
2 2 8 Ë 8 ¯

Problem 4.152 Determine the height h above the surface where the value of g
(i) falls to 1% of the value at the surface; and
(ii) falls by 1% of the value of the surface.
Solution
R2 g h R2
(i) gh = g 2 fi = 2
r g r
It is given that gh = 0.01g
R2
So, = 0.01
r2
r = 100 R2 = 10R
r = R+h
10R = R+h
h = 10R – R = 9R

(ii) gh = g ÊÁ 1 -
2h ˆ
Ë ˜
R¯ gh
=1-
2h
g R
2h g - g h
=
R g
g - gh
Given, = 0.01
g
2h R
0.01 = ,h=
R 200
We know that R = 6400 km, so h = 32 km.
790പWŚLJƐŝĐƐĨŽƌ//dͲ:͗DĞĐŚĂŶŝĐƐ//

Problem 4.153 In a double star, two stars one of mass m1 and another of mass m2, with a
separation d, rotate about their common centre of mass. Find:
(a) an expression for their time period of revolution,
(b) the ratio of their kinetic energies,
(c) the ratio of their angular momenta about the centre of mass,
(d) the total angular momentum of the system, and
(e) the kinetic energy of the system
Solution COM
m1 m2
r1 r2
(a) r 1 + r2 = d ...(i)
m1r 1 = m 2r 2 ...(ii) d
Solving these two equations we get Fig. 4.143

Ê m2 ˆ Ê m2 ˆ
r1 = Á ˜ d or r2 = Á d
Ë 1
m + m2¯ Ë m1 + m2 ˜¯
The centripetal force is provided by gravitational force,
Gm1 m2 Gm1 m2
m 1r 1w 2 = m 2r 2w 2 = 2

d d2
Gm1 m2 Ê m2 ˆ
= m1 Á dw 2
d2 Ë m1 + m2 ˜¯
Solving these equations, we get

G(m1 + m2 )
w =
d3
2p
\ T =
w
d3
= 2p Ans.
G(m1 + m2 )
1
I1w 2
K1 2 I m r2
(b) = = 1 = 1 12
K2 1 I 2 m2 r2
I 2w 2
2 2 2
Êm ˆÊr ˆ Êm ˆÊm ˆ m
= Á 1˜Á 1˜ = Á 1˜Á 2˜ = 2 Ans.
Ë m2 ¯ Ë r2 ¯ Ë m2 ¯ Ë m1 ¯ m1
L1 I1w I1 m2
(c) = = = Ans.
L2 I 2w I 2 m1
(d) L = L1 + L2 = (I1 + I2)w
= (m1r12 + m2r22)w

È m m2 d 2 m2 m12 d 2 ˘
= Í 1 2 2 + ˙w
ÍÎ (m1 + m2 ) (m1 + m2 )2 ˙˚
'ƌĂǀŝƚĂƟŽŶപ791

È m m2 d 2 + m2 m12 d 2 ˘
=Í 1 2 ˙w
ÍÎ (m1 + m2 )2 ˙˚

d 2 m1 m2w È m2 + m1 ˘
fi Ím + m ˙
m1 + m2 Î 1 2˚

= mwd2 Ans.
m1 m2
where, m = = reduced mass
m1 + m2
1 1
(e) K = (I1 + I2)w2 = mw2d2 Ans.
2 2

Problem 4.154 The distance between the earth and the moon is about 3.8 ¥ 105 km. At what
point will the gravitational field strength of the earth-moon system be zero? Given mass of
the earth is 81 times the moon’s mass.
Solution Let r1 be the distance of point from the earth
m1 r r r
r1 = = =
m1 + m2 m2 1
1+ 1+
m1 81
9r 9
= = ¥ 3.8 ¥ 105 km
10 10

Problem 4.155 In Fig. 4.144 a central particle of mass M is sur- 2M


M
4M

rounded by a square array of other particles, separated by either 7M 5M


distance d or distance d/2 along the perimeter of the square. What
are the magnitude and direction of the net gravitational force on the 3M
M
central particle due to the other particles? (EAMCET, 2007)
5M 7M
G(3 M )( M ) GM 2
Solution F= 2
= 12 4M 2M
Ê dˆ d2 M
ÁË ˜¯
2 Fig. 4.144
 
Problem 4.156 The gravitational field in a region is given by E = (2 i + 3 j) N/kg. Show that
no work is done by the gravitational field when a particle is moved on the line 3y + 2x = 5.
3
Solution Direction of field intensity tan q1 =
2
2
Direction of line tan q2 = -
3
Ê 3ˆ Ê 2ˆ
(tan q1) (tan q2) = Á ˜ Á - ˜ = –1
Ë 2¯ Ë 3¯
i.e., line is perpendicular to field. So, the line is a equipotential line. Hence, work done by
gravitational field in moving the object along the line is zero.
792പWŚLJƐŝĐƐĨŽƌ//dͲ:͗DĞĐŚĂŶŝĐƐ//

Problem 4.157 Three uniform spheres each having a mass M and radius a are kept in such
a way that each touches the other two. Find the magnitude of the gravitational force on any
of the spheres due to the other two. (BITSAT, 2009)
Solution Let F be force between any two particles, then
GM 2
F =
( 2 a )2 F F
60
\ Fnet = 2F cos
2
Gm2 3 3 Gm2
=2 2
◊ = Fig. 4.145
4a 2 4 a2

Problem 4.158 A smooth tunnel is dug along the radius of the earth that ends at the centre.
A ball is released from the surface of the earth along tunnel. Coefficient of restitution for
collision between soil at centre and ball is 0.5. Calculate the distance travelled by ball just
before the second collision at centre. Given mass of the earth is M and radius of the earth is R.
Solution Let mass of the ball be m.
1 2 4=0
mv = m(VA – VB)
2 A
È GM Ê GM ˆ ˘
= m Í- - Á - 1.5 ˜
R ¯ ˙˚
B
Î R Ë v
GMm
=
2R
Fig. 4.146
GM
\ v =
R
Velocity of ball just after collision,
1 GM
v¢ = ev =
2 R
Let r be the distance from the centre up to where the ball reaches after collision. Then,
1
mv¢2 = m[V (r) – V(centre)]
2
1 GMm È 3GM GM Ê 3R2 r 2 ˆ ˘
or = mÍ - 3 Á - ˜˙
8 R Î 2R R Ë 2 2 ¯˚
2
1 3 3 r
or = - +
8 2 2 2R 2
r2 1
\ 2
=
R 4
R
or r =
2
'ƌĂǀŝƚĂƟŽŶപ793

\ Time desired distance,


R R
s = R+ + = 2R Ans.
2 2

Problem 4.159 A smooth tunnel is dug along the radius of the earth that ends at the centre.
A ball is released from the surface of the earth along the tunnel if the coefficient of restitution
is 0.2 between the surface and ball then the distance travelled by the ball before the second
collision at the centre
6 7
(a) R (b) R
5 5
9 3
(c) R (d) R
5 2
GM A
Solution (b) a =- 3
x
R
a = – w 2x O

GM R
So, w = vC r
3
R C
GM
= wA = ¥R
R3
After collision velocity of ball towards A,
Fig. 4.147
GM 1 GM
vc = evc = 0.2 =
R3 5 R
Let now amplitude be A¢, then
1 GM
vC 5 R R
A¢ = = =
w GM 5
3
R
7R
Net distance = R + (R /5) + (R/5) =
5
Problem 4.160 The density of the core of a planet is r1 and that of the
outer shell is r2. The radii of the core and that of the planet are R and 2R
respectively. The acceleration due to gravity at the surface of the planet R
2R
is same as at a depth R. The ratio of density r1/r2 will be:
(a) 7/3 (b) 5/3 r1
(c) 8/3 (d) 1/3 r2
Solution (a) Let M, m be the mass of the planet and that of the core,
Fig. 4.148
then
GM Gm
=
( 2 R )2 R2
M
fi =m
4
794പWŚLJƐŝĐƐĨŽƌ//dͲ:͗DĞĐŚĂŶŝĐƐ//

4 3 4
pR r1 + p[8R3 – R3] r2 = 4(4/3pR3 r1)
3 3
r1 + 7r2 = 4r1
r1/r2 = 7/3
Problem 4.161 A certain triple-star system consists of two stars, each of mass m, revolving
about a central star of mass M in the same circular orbit of radius r. The two stars are always
at opposite of a diameter of the circular orbit (see Fig. 4.149). Derive
an expression for the period of revolution of the stars. m
r
Solution The centripetal force acting on each star of mass m
È GMm GMm ˘ M
F= Í 2 +
Î r (2r )2 ˙˚
By Newton’s second law, we have m
2
GMm Gm Fig. 4.149
mw2r = 2
+ 2
r ( 2r )
GM Gm
\ w = 3
+ Ans.
r 4r 3

Problem 4.162 Two equal masses of M are separated by a distance 2 d. A small body of
mass m is released from a point P, equidistance from the two masses and a distance d from
the line joining them. Calculate:
(a) the velocity of this body when it passes through Q; and P
(b) the acceleration of this body at P and Q. M d M

Solution
d Q d
(a) By conservation of mechanical energy, we have
(K + U)p = (K + U)Q Fig. 4.150

È Ê - Gm ˆ ˘ 1 È Ê -GMm ˆ ˘ m
or 0 + Í2 Á ˜ ˙ = mv 2 + Í 2 Á ˜˙ P
Î Ë 2d ¯ ˚ 2 Î Ë d ¯˚
2d q
GM Ê 1 ˆ
\ v =2 Á 1- ˜
d Ë 2¯ M M
d Q
(b) The force on the body
Fp = 2F cos q Fig. 4.151

Ê GMm ˆ d
= 2Á ˜ ¥
Ë 2d ¯
2
2d
\ acceleration at P,
FP GM
ap = =
m 2 d2
At Q the net force on the body becomes zero, and so
aQ = 0.
'ƌĂǀŝƚĂƟŽŶപ795

DAILY PRACTICE PROBLEMS FOR JEE MAIN AND ADVANCE

‡˜‡Žͳ౨(Only one option is correct) 6. The escape velocity from the earth is
1. A body of mass m is moved to a height about 11 km/sec. Assuming the mass
equal to the radius of the earth R. The and radius of the earth to be about 81
increase in its potential energy is and 4 times the mass and radius of the
moon. The escape velocity in(km/sec)
1 from the surface of the moon will be
(a) mgR (b) mgR
2 (BITSAT, 2010)
1 (a) 2.44 (b) 0.34
(c) 2 mgR (d) mgR
4 (c) 11 (d) 49.5
7. The escape velocity from the earth is
2. Two planets have radii r1 and r2 and about 11 km/sec. The escape velocity
densities d1 and d2 respectively. The from a planet having twice the radius
ratio of acceleration due to gravity on and same mean density as the earth is
them will be (WBJEE, 2001) (a) 22 km/sec
(a) r1 d2 : r2 d1 (b) r12 d1 : r22 d2 (b) 11 km/sec
(c) r1 d1 : r2 d2 (d) r1d12 : r2 d22 (c) 5.5 km/sec
(d) 15.5 km/sec
3. The value of g will be 1% of its value 8. The distance of two planets (Jupiter
at the surface of the earth at a height of and Saturn) from the sun are 1013 and
(Re = 6400 km) 1012 m respectively. The ratio of time
(a) 6400 km (b) 2560 km period is (VITEEE, 2009)
(c) 57600 km (d) 64000 km (a) 10 10 (b) 1/ 10
4. If the change in the value of g at a (c) 100 (d) 10
height h above the surface of the earth 9. The orbital velocity of an artificial
is same as at a depth x below it then satellite in a circular orbit just above
(both h and x being below the surface the earth’s surface is v. For a satellite
of the earth is) (DCE, 2004) orbiting at an altitude of half the
(a) x = 2h (b) x = h earth’s radius the orbital velocity is
(c) x = 1/2h (d) x = h (EAMCET, 2001)
5. The acceleration due to gravity varies (a) 3/2 v (b) 2/3 v
as one moves away from the earth’s
centre is given by (BITSAT, 2003) (c)
3
v (d) 2/3 v
2
g g
10. A satellite rotating in the same sense as
(a) (b) the earth in an equitorial orbit has a time
r r period of 6 hours. At a certain instant
there is an observer on the equator
g g of the earth (which is overhead). It is
(c) (d) directly overhead the observer again
after a time t the current value of t is
r r
(a) 8 hr (b) 4 hr
(c) 12 hr (d) 24 hr
796പWŚLJƐŝĐƐĨŽƌ//dͲ:͗DĞĐŚĂŶŝĐƐ//

‡˜‡Žʹ౨(Only one option is correct) due to gravity at the latitude of 30° and
g at the equator. The value of g – g30 is
1. A particle is projected with a speed
v0 from the surface of the earth the 3 2 1 2
(a) wR (b) wR
maximum height reached by the 4 4
particle is equal to the radius R of the
1 2
earth if g is the acceleration due to (c) w2R (d) wR
gravity on the surface of the earth then 2
(a) v02 < 2gR (b) v02 > 2gR 7. Four particles of masses m are placed
2
(c) v0 = 2gR (d) v02 = gR at the four corners of a square of side
2. At what distance from the centre of the ‘a’. Forces exerted by this system on
moon is the point at which the strength other particles of mass m placed at the
of the resultant field of the earth’s and midpoint of a side of square is
the moon’s gravitational force is equal
to zero. The earth’s mass is 81 times 16GM 2 16GM 2
(a) (b)
that of the moon and the distance 5 3a2 5 5a2
between centre of the earth and the
16GM 2
moon is 60 R where R is the radius of (c) (d) zero
the earth. 5a2
(a) 4R (b) 6R 8. What should be the angular velocity of
(c) 3R (d) 5R the earth about its own axis so that a
3. The radius of the earth is 6400 km and person’s weight at the equator will be
g = 9.8 m/sec2. If the body placed at 3/5 of his weight at poles (OJEE, 2012)
the equator has to become weightless
2g 2g
the earth should make one complete (a) (b)
rotation in (WBJEE, 2004) 3R 5R
(a) 12 hours (b) 6 hours g 3g
(c) 1.4 hours (d) 24 hours (c) (d)
R 2R
4. The orbital period of a satellite in
a circular orbit of radius r about a 9. A satellite is moving in its orbit around
spherical planet of mass M and mean the earth. Its kinetic energy is K. The
density r for a low altitude orbit (r = satellite is given additional KE and
rr) will be when its kinetic energy becomes K¢
it escapes the gravitation pull of the
(a) 3p Gr (b) 3p/Gr earth then
(c) p/Gr (d) 2/Gr (a) K¢ = 4 K (b) K¢ = 2 K
(c) K¢ = 3 K (d) K¢ = 1.5 K
5. A body is projected vertically upward
10. Suppose the gravitational force varies
from the surface of a planet of radius R
inversely as the n power of distance
with a velocity equal to half the escape
then time period of a planet in circular
velocity for that planet. The max height
orbit of radius R around the sun will
attained is
be proportional to (IIT-JEE, 2016)
(a) R/2 (b) R/5 (a) Rn (b) Rn–1/2
R n+1
(c) R/3 (d)
4 (c) R 2 (d) R–n
6. If the earth is supposed to be a sphere of 11. A spherical symmetric gravitational
radius R. If g30 is value of acceleration system of particles has a mass density:
'ƌĂǀŝƚĂƟŽŶപ797

r = r0 for r£R 2GM


(a) ( 4 2 - 5)
0 for r>R 7R
where r0 is constant. A test mass can -2GM
undergo circular motion under the (b) ( 4 2 - 5)
7R
influence of the gravitational field of
particles. Its speed (v) as a function of GM
(c)
distance r (0 < r < •) from the centre 4R
of the system is represented by:
2GM
(IIT-JEE, 2008) (d) ( 2 - 1)
5R
v
Multiple Correct Options
(a) 1. A satellite is revolving in an elliptical
r orbit in free space; then which of the
R
following statements are true.
v (a) Its mechanical energy is constant
(b) Its linear momentum is constant
(b) (c) Its angular momentum is constant
r (d) Its areal velocity constant
R
2. A satellite is revolving in a circular
v orbit around the earth and an object is
placed inside the satellite. Then the
(c) (a) satellite and object both are
weightless
r
R (b) satellite has weight while object
appears weightless
v (c) object has weight while satellite
appears weightless
(d) (d) satellite behaves as a freely falling
r body
R
3. A spherical shell of radius R is cut along
12. A thin uniform annular disc (see figure) a chord AB and slightly displaced as
of mass M has outer radius 4R and shown in figure. Two points P and Q
inner radius 3R. The work required to are such that with P in lower part and
take a unit mass from point P on its Q in upper part of the shell. Then
axis to infinity is: (IIT-JEE, 2010) Q
A B
P
P

4R

(a) the gravitational field at P and Q


3R are zero
4R
(b) the sum of gravitational fields at
P and Q is zero
798പWŚLJƐŝĐƐĨŽƌ//dͲ:͗DĞĐŚĂŶŝĐƐ//

(c) the gravitational potentials at P 6. The spherical planets have the same
and Q are equal mass but densities in the ratio 1:8. For
(d) the gravitational potentials at P these planets, the
and Q are unequal (a) acceleration due to gravity will be
4. A solid sphere of uniform density in the ratio 1:4
and radius 4 units is located with its (b) acceleration due to gravity will be
centre at the origin O of coordinates. in the ratio 4:1
Two spheres of equal radii 1 unit with (c) escape velocities from their sur-
their centres at A (–2, 0, 0) and B (2,
0, 0) respectively are taken out of the faces will be in the ratio 1 : 2
(d) escape velocities from their
solid leaving behind spherical cavities
as shown in figure. Then surfaces will be in the ratio 2 : 1
Y
7. A geostationary satellite is at a height
h above the surface of earth. If the
earth’s radius is R.
A B
O X
R
Z q
R h

(a) the gravitational force due to this


sphere at the origin is zero
(b) the gravitational force at the point
(a) The minimum colatitude on the
B (2, 0, 0) is zero
earth up to which the satellite can
(c) the gravitational potential is the
be used for communicate is sin–1
same at all points of the circle y2
R/(R + h)
+ z2 = 36
(b) The maximum colatitude on the
(d) the gravitational potential is the
earth up to which the satellite can
same at all points on the circle y2
be used for communicate is sin–1
+ z2 = 4.
R/(R + h)
5. The magnitude of the gravitational
(c) The rear on the earth escaped
field strengths at distances r1 and r2
from this satellite is given as 2pR2
from the centre of uniform sphere of
(1 + sin q)
radius R and mass M are F1 and F2
(d) The area on the earth escaped
respectively, then
from this satellite is given as 2pR2
F1 r1 (1 + cos q)
(a) = if r1 < R and r2 < R
F2 r2 8. Due to a solid sphere magnitude of
2 (a) gravitational potential is maxi-
F1 Ê r2 ˆ
(b) = if r1 > R and r2 > R mum at the centre
F2 ÁË r1 ˜¯ (b) gravitational potential is mini-
F1 r1 mum at the centre
(c) = if r1 > R and r2 > R (c) field strength is maximum at the
F2 r2
2
centre
F1 Ê r1 ˆ (d) field strength is minimum at the
(d) = if r1 < R and r2 < R
F2 ÁË r2 ˜¯ centre
'ƌĂǀŝƚĂƟŽŶപ799

9. Two objects of masses m and 4 m (c) centre of mass of system will


are at rest at an infinite separation. remain at rest in all the reference
They move towards each other under frame
mutual gravitational attraction. G is (d) total energy of system remains
the universal gravitational constant. constant
Then at separation r 13. A tunnel is dug along a cord of the
(a) the total energy of the two objects earth at a perpendicular distance
is zero R/2 from the earth’s centre. The wall
(b) their relative velocity of approach of the tunnel may be assumed to be
1/2 frictionless. A particle is released from
Ê 10Gm ˆ one end of the tunnel. The pressing
is Á in magnitude.
Ë r ˜¯ force by the particle on the wall and
(c) the total kinetic energy of the the acceleration of the particle varies
2 with x (distance of the particle from
objects is 4Gm the centre) according to
r
(d) net angular momentum of both Pressing
the particles is zero about any force
point.
(a)
10. Two tunnels are dug from one side of
the earth’s surface to the other side,
one along a diameter and the other X = R/2 X = R
X
along a cord. Now two particles are
dropped from one end of each of the Pressing
force
tunnels. Both the particles oscillate
simple harmonically along the tunnels.
(b)
Let T1 and T2 be the time periods and
v1 and v2 be the maximum speed of the
particles in the two tunnels. Then X
X = R/2 X = R
(a) T1 = T2 (b) T1 > T2
Acceleration
(c) v1 = v2 (d) v1 > v2
11. Identical point masses each equal to
m are placed at x = 0, x = 1, x = 2, x = (c)
4, ... The total gravitational force on
mass m at x = 0 due to all other masses X = R/2 X = R
X
is
(a) infinite (b) (4/3) GM2 Acceleration
2
(c) (4/3) Gm (d) zero
12. Two masses m1 and m2 (m1 < m2)
are released from rest from a finite (d)
distance. They start under their mutual
gravitational attraction X = R/2 X = R
X
(a) acceleration of m1 is more than
that of m2 14. A double star is a system of two stars
(b) acceleration of m2 is more than of masses m and 2m, rotating about
that of m1 their centre of mass only under their
800പWŚLJƐŝĐƐĨŽƌ//dͲ:͗DĞĐŚĂŶŝĐƐ//

mutual gravitational attraction. If r respectively with r1 < r2. Plane of motion of


is the separation between these two the two are same. At position 1, A is given an
stars then their time period of rotation impulse in the direction of velocity by firing
about their centre of mass will be a rocket so that it follows an elliptical path to
proportional to meet B at position 2 as shown. Focal lengths
(a) r3/2 (b) r of the elliptical path are r1 and r2 respectively.
(c) m 1/2
(d) m–1/2 At position 2, A is given another impulse so
15. Assuming the earth to be a sphere of that velocities of A and B at 2 become equal
uniform density the acceleration due and the two move together.
to gravity For any elliptical path of the satellite time
(a) at a point outside the earth is period of revolution is given by Kepler’s
inversely proportional to the planetary law as T2 μ r3 where a is semi-
square of its distance from the r +r
major axis of the ellipse which is 1 2 in
centre this case. 2
(b) at a point outside the earth is Also angular momentum of any satellite
inversely proportional to its revolving around the earth will remain a
distance from the centre constant about the earth’s centre as force
(c) at a point inside is zero of gravity on the satellite which keeps it in
(d) at a point inside is proportional to elliptical path is along its position vector
its distance from the centre. relative to the earth’s centre.
Passages and Matrix Matching 3. When A is given its first impulse at
that moment
Passage for Q: 1–2
Two satellites S1 and S2 revolve round a
planet in coplanar circular orbits in the same
r2
sense. Their periods of revolution are 1 h and
Earth
8 h respectively. The radius of the orbit of S1 B 1 2
is 104 km. When S2 is closest to S1 find A
r1

1. The speed of S2 relative to S1


(a) 2 ¥ 104 km/h
(b) p ¥ 104 km/h (a) A, B and centre of the earth are in
(c) 2p ¥ 104 km/h same straight line
(b) B is ahead of A angularly
(d) 104 km/h
(c) B is behind A angularly
2. The angular speed of S2 as observed by
(d) None of these
an astronaut in S1 is
4. If the two have same mass:
(a) p rad/h
(a) A would have more potential
(b) 2p/3 rad/h
energy than B while on their
(c) p/4 rad/h initial circular paths
p (b) A would have less kinetic energy
(d) rad/h
3 than B while on their initial
circular paths
Passage for Q: 3–5 (c) Relative of the earth’s centre,
Two satellites A and B are revolving around angular momentum of A when it
the earth in circular orbits of radius r1 and r2
'ƌĂǀŝƚĂƟŽŶപ801

is in elliptical path would be less (a) – 21.29 GM (b) –10.98 GM


than angular momentum of B (c) – 8.98 GM (d) none of
(d) During the whole process angular these
momentum of B would be more
Passage for Q: 9–10
than angular momentum of A
5. If r2 = 3 r1 and time period of revolution An artificial satellite is moving in a circular
for B be T than time taken by A in orbit around the earth with a speed equal to
moving from position 1 to position 2 is half the magnitude of escape velocity from
the earth. (radius of earth is R)
3
(a) T (b) T 3 9. Height of the satellite above earth’s
2 2 surface is
(a) R (b) R/2
T 2 T 2
(c) (d) 3R
3 3 3 (c) 2R (d)
2
Passage for Q: 6–8 10. If the satellite is slopped suddenly in
A solid sphere of uniform density and radius its orbit and allowed to fall freely onto
4 m is located with its centre at the origin O the earth, the speed with which it hits
of coordinates. Two spheres of equal radius the surface of the earth
1 m with their cavities at A (–2, 0, 0) and B (a) 2gR (b) gR
(2, 0, 0) respectively are taken out, leaving
behind spherical cavities. The mass of each R gR
(c) g (d)
sphere taken out is M. Calculate 2 2
(IIT-JEE, 1993)
Passage for Q: 11–12
y A body is projected vertically upwards
from the surface of the earth with a velocity
sufficient to carry it to infinity.
x 11. The velocity of the body at a height h
A O B
2 gR2
(a) gR (b)
z 3(R + h)
6. The gravitational field at B is 2 gR2
(c) (d) zero
( R + h)
21GM
(a) GM (b) 12. The time, the body takes to reach
5
height h
31 31 ˘
2R ÈÊ
3/2
(c) GM (d) GM hˆ
16 8 (a) ÍÁ 1 + ˜ - 1˙
g ÍÎË R¯ ˙˚
7. The gravitational potential at any
1 2R ÈÊ hˆ
3/2 ˘
point on circle y2 + z2 = 36 is (b) ÍÁ 1 + ˜ - 1˙
3 g ÍÎË R¯ ˙˚
(a) –7.90 GM (b) –10.98 GM
3/2
(c) –8.98 GM (d) zero RÊ hˆ
(c) Á 1 + ˜¯ (d) none
8. The gravitational potential at any gË R
point on circle y2 + z2 = 4 is
802പWŚLJƐŝĐƐĨŽƌ//dͲ:͗DĞĐŚĂŶŝĐƐ//

Matrix Matching
13. Column I describes some situations in which a small object moves. Column II describes
some characteristics of these motions. Match the situations in column I with the
characteristics in column II.
Column I Column II
A. The object moves on the x-axis under (p) The object executes a simple
a conservative force in such a way harmonic motion.
that its ‘speed’ and ‘position’ satisfy
v = c1 c2 - x 2 , where c1 and c2 are
positive constants.
B. The object moves on the x-axis in (q) The object does not change in
such a way that its velocity and its direction.
displacement from the origin satisfy v
= – kx, where k is a positive constant.
C. The object is attached to one end of (r) The kinetic energy of the object
a massless spring of a given spring keeps on decreasing.
constant. The other end of the spring
is attached to the ceiling of an elevator.
Initially, everything is at rest. The
elevator starts moving upwards with a
constant acceleration a. The motion
of the object is observed from the
elevator during the period it maintains
this acceleration.
D. The object is projected from the earth’s (s) The object can change its direction
surface vertically upwards with a of motion only once.
speed 2 GM/R , where M is the mass
of the earth and R is the radius of
the earth. Neglect forces from objects
other than the earth.
14. On the surface of the earth acceleration due to gravity is g and gravitational potential
is V. Match the following:
Column I Column II
A. At height h = R, value of g (p) decreases by a factor 1/4
B. At depth h = R/2, value of g (q) decreases by a factor 1/2
C. At height h = R/2, value of g (r) decreases by a factor 3/4
D. At depth h = R/4, value of g (s) decreases by a factor 2/3
(t) None
'ƌĂǀŝƚĂƟŽŶപ803

15. Two concentric spherical shells are shown in figure. Match the following:
∑B
∑A
D

∑C

Column I Column II
A. Potential at A (p) greater than B
B. Gravitational field at A (q) less than B
C. As one moves from C to D (r) potential remains constant
D. As one moves from D to A (s) gravitational field decreases
(t) None
16. A satellite is revolving around the earth in a circular orbits of radius a with velocity v0.
A particle of mass m is projected from the satellite in forward direction with relative
È 5 ˘
velocity v = Í - 1˙ v0. During subsequent motion of particle, match the following
Î 4 ˚
Column I Column II
3GMe m
A. Total energy of particle (p) –
8a
5 GMe m
B. Minimum distance of particle from (q)
the earth 8 a
C. Maximum distance of particle from (r) 5a/3
the earth
D. Kinetic energy (s) a
17. Assume v0 is the velocity of projection of a body from the surface of the earth enables
it to become a satellite of the earth close to the earth’s surface. If R is the radius of
GM
the earth then v0 = gR where g = ; G – universal gravitational constant and M
R2
v 1
is the mass of the earth. If ve be the escape velocity on the earth then 0 = . If v is
ve 2
the velocity of projection of body on the surface of earth then, match the following
Column I Column II
A. v = v0 (b) Path of body is circular
B. v < v0 (q) Path of body is elliptical and returns
to the earth
C. v0 < v < ve (r) Path of body is elliptical but not
return to the earth
D. v = ve (s) Path of the body is parabolic
804പWŚLJƐŝĐƐĨŽƌ//dͲ:͗DĞĐŚĂŶŝĐƐ//

Answers Key
Level 1
1. (b) 2. (c) 3. (c) 4. (a) 5. (b) 6. (a) 7. (a) 8. (a)
9. (b) 10. (a)
Level 2
1. (d) 2. (b) 3. (c) 4. (b) 5. (c) 6. (b) 7. (b) 8. (b)
9. (b) 10. (c) 11. (c) 12. (a)
Multiple Correct Options
1. (a, c, d) 2. (b, d) 3. (b, d) 4. (a, c, d)
5. (a, b) 6. (a, c) 7. (a, c) 8. (a, d)
9. (a, b, c, d) 10. (a, d) 11. (b) 12. (a, d)
13. (b, c) 14. (a, d) 15. (a, d)
Passages and Matrix Matching
1. (b) 2. (d) 3. (b) 4. (c) 5. (c) 6. (c) 7. (b) 8. (a)
9. (a) 10. (b) 11. (c) 12. (b)
13. AÆ(p); BÆ(q), (r); CÆ(p), (s); DÆ(r) 14. AÆ(q); BÆ(q); CÆ(s); DÆ(r)
15. AÆ(q); BÆ(t); CÆ(r); DÆ(s) 16. AÆ(p); BÆ(s); CÆ(r); DÆ(q)
17. AÆ(p); BÆ(q); CÆ(r); DÆ(s)

SOLUTION AND HINTS

Level 1
mgh mgR 1
1. (b) DU = = = mgR
È h ˘ È R ˘ 2
ÍÎ1 + R ˙˚ ÍÎ1 + R ˙˚

g1 GM1/r12 ( 4/3)p r13 d1/r12 r1d1


2. (c) = = =
g2 GM2/r22 ( 4/3)p r23 d2/r22 r2 d2
g¢ R2 1 R2
3. (c) = 2
fi =
g ( R + h) 100 (R + h)2

fi h = 9R = 57600 km
4. (a) gh = gx
Ê 2h ˆ Ê xˆ
g Á1- ˜ = g Á1- ˜ (∵ x << R)
Ë R¯ Ë R¯
fi x = 2h
Ê GM ˆ
5. (b) Inside the earth: gin = Á 3 ˜ r
Ë R ¯
fi gin μ r
'ƌĂǀŝƚĂƟŽŶപ805

GM
Outside the earth: gout =
r2
1
gout μ
r2
Therefore, option (c) is correct.

vM 2GM M/RM ÊM ˆ Ê R ˆ
6. (a) = = Á M˜ ¥Á E ˜
vE 2GME/RE Ë ME ¯ Ë RM ¯

vM 1 2
= ¥4 =
11 81 9
2
vM = ¥ 11 = 2.44
9

GM G( 4/3)p R3 r 4
7. (a) g = = = p Gr R
R2 R2 3
4 Ê 8 ˆ
\ ve = 2 gR = 2 ¥ p Gr R ¥ R = Á p Gr ˜ R
3 Ë 3 ¯
\ ve μ R. As density r is same but radius becomes double, the escape velocity
will become double.
8. (a) According to Kepler’s 3rd law, T2 μ r3
3/2
T1 Ê r1 ˆ
\ =
T2 ÁË r2 ˜¯

9. (b) v0 = gR2/(R + h)
v0¢ R 2
\ = =
v0 (R + R/2) 3

Ê 2ˆ Ê 2ˆ
v¢0 = Á v = v
Ë 3 ˜¯ 0 ÁË 3 ˜¯
TSTE
10. (a) t = . if rotating in same sense.
TE - TS

Level 2
1. (d) Initial KE = Final PE

1 mgh
mv02 =
2 Ê hˆ
ÁË 1 + ˜¯
R
806പWŚLJƐŝĐƐĨŽƌ//dͲ:͗DĞĐŚĂŶŝĐƐ//

1 mgR
mv02 =
2 Ê Rˆ
ÁË 1 + ˜¯
R
v02 = gR
2. (b) Let the gravitational field strength be zero at point P having distance x from the
moon. Then,
Iearth = Imoon
GME GM M
2
=
(60 R - x ) x2
81 1
2
= 2
(60 R - x ) x
9 1
=
60R - x x
x = 6R
3. (c) The body placed at the equator will be weightless if weight of body is balanced
by centrifugal force, i.e.,
mRw2 = mg
w= g/R

R
T = 2p
g
22 6.4 ¥ 106
= 2¥ ¥
7 9.8
 5080 second
 1.4 hour
4. (b) For low altitude orbit, orbital velocity is

Ê4 ˆ
G Á p R3 ˜ r Ê ˆ
GM Ë3 ¯ 4
v0 = = =Á p Gr ˜ R
R R Ë 3 ¯

2p R 2p R 3p
\ T= = =
v0 Ê 4 ˆ Gr
ÁË p Gr ˜ R
3 ¯
1 Ê h ˆ
5. (c) mv 2 = mgh / Á 1 + ˜
2 Ë R¯

1 1
Here, v= ve = 2gR
2 2
'ƌĂǀŝƚĂƟŽŶപ807

1 Ê1 ˆ mgh
\ m Á gR˜ =
2 Ë2 ¯ Ê hˆ
ÁË 1 + ˜¯
R
R
h =
3
6. (b) g¢ = g – Rw2 cos2 f
At equator, f = 0°. Therefore,
g = geq = g – Rw2
At latitude, f = 30°
3
g30° = g – Rw2 cos2 30° = g – Rw2
4
1 1
\ g – g30° = – Rw 2 = Rw2 (in magnitude)
4 4
7. (b) From the figure, the net force on mass m placed at P is given by
Gm2 a 16Gm2
= 2F cos q = 2 ◊ =
(5 a /4) a 5/2 5 5 a 2
2

m m
a 5/2
a F
2
q q 2F cos q
a q q
P
F

m m

8. (b) g¢ = g – Rw 2 cos2 f
At poles, f = 90°
\ Wpole = mg¢p = mg
At equator, f = 0°.
Weq = mg¢eq = mg – mRw2
3
or mg = mg – mRw2
5
2g
w=
5R
1 1 1
9. (b) K = mv02 = m( gR )2 = mgR
2 2 2
1 1 1
K¢= mve2 = m( 2 gR )2 = (2mgR)
2 2 2
\ K¢= 2K
808പWŚLJƐŝĐƐĨŽƌ//dͲ:͗DĞĐŚĂŶŝĐƐ//

GMm
10. (c) mRw2 = …(i)
Rn 4p 2 GMm
mR 2 =
T Rn
R 1
2
μ n
T R
T2 μ Rn+1
T μ R(n+1)/2
11. (c) For radius r £ R, the intensity will be due to mass within radius r.

È4 ˘
G Í p r 3 r0 ˙ m 2
Therefore, Iinside = Î 3 ˚ = mv
2
r r
\ v μr
For r > R, the intensity will be due to total mass.

È4 ˘
G Í p r 3 r0 ˙ m
mv 2
Therefore, Ioutside = Î 3 2 ˚ =
r r
1
\ v μ
r
Thus, the graph will be as shown in option (c).
4R
(G(s )(2p y dy )
12. (a) The potential at point P is given by V = – Ú y2 + x2
3R

= – 2p GsR [4 2 - 5]

x2 + y 2
x

dy 3R 4R

M M
But, s = =
p (16 R - 9R ) 7p R2
2 2

2GM
\ Work = mV = (1) V = ( 4 2 - 5)
7R
'ƌĂǀŝƚĂƟŽŶപ809

Multiple Correct Options


1. (a, c, d)
Explained in theory.
2. (b, d)
The satellite experiences force due to the planet. Also the satellite is freely moving
(falling) object. An object experiences weightlessness inside the satellite, because
N=0
3. (b, d)   
The gravitational field inside shell, E g = 0. So If E1 and E 2 are the gravitation field
strength at P and Q, then
 
E1 + E 2 = 0.
4. (a, c, d)
See solved example

5. (a, b)
GM F r
For r1 < R, F = 2
,\ 1 = 1
r F2 r2

GM F1 r22
For r1 £ R, F = ,\ =
r2 F2 r12
6. (a, c)
1/3
4 Ê 3M ˆ
M= p R3 r or R = Á
3 Ë 4pr ˜¯
2/3
GM GM Ê 4pr ˆ
(i) g = = = GM Á
R 2
(3 M/4pr ) 3/2 Ë 3 M ˜¯
2/3 2/3
g Êr ˆ Ê 1ˆ 1
i.e., g μ r 2/3
\ 1 = Á 1˜ =Á ˜ =
g2 Ë r2 ¯ Ë 8¯ 4

2GM (2GM )1/2


(ii) ve = =
R (3 M/4pr )1/6
1/6 1/6
v Êr ˆ Ê 1ˆ 1
or 1 = Á 1 ˜ =Á ˜ =
v2 Ë r2 ¯ Ë 8¯ 2
7. (a, c)
From figure
A
R
q
q
R h
q

B
810പWŚLJƐŝĐƐĨŽƌ//dͲ:͗DĞĐŚĂŶŝĐƐ//

R
Min. colatitude, sin q =
R+h
Curved area AB on the earth = 2pR2(1 – sin q)
Area on the earth escaped from satellite
= 4pR2 – 2pR2(1 – sin q) = 2pR2 (1 + sin q)
8. (a, d)
-1.5GM
At centre, V =
R
and E =0
9. (a, b, c, d)
Initially, potential and kinetic energies are zero and from conservation of mechanical
energy total energy of the two objects is zero.
Further, decrease in gravitational potential energy = increase in kinetic energy
G(m)( 4 m) 1
or = mvr2 ...(1)
r 2

(m)( 4 m) 4 m
Here, m = reduced mass = =
m + 4m 5
Substituting in Eq. (1), we get
vr = relative velocity of approach
10Gm
=
r
From Eq. (1) total kinetic energy
G(m)( 4 m)
=
r
4Gm2
=
r
Net torque of two equal and opposite forces acting on two objects is zero. Therefore,
angular momentum will remain conserved. Initially, both the objects were stationary
i.e., angular momentum about any point was zero. Hence, angular momentum of both
the particles about any point will be zero at all instants.
10. (a, d)
Time period in both the cases

R2
T1 = T2 = 2p = 84.6 min
GM
But v1 > v2. Because the difference in potential energy between the extreme position
and mean position will be more in the first case.
'ƌĂǀŝƚĂƟŽŶപ811

11. (b)
O m m m
x=0 x=1 x=2 x=4

Gm2 Gm2 Gm2


F0 = + + + ºº
12 22 42
È1 1 1 ˘
F0 = Gm2 Í 2 + 2 + 2 + ºº˙
Î1 2 4 ˚

It is a G.P. with common ratio 1/4.

È 1 ˘ 4Gm2
\ F0 = Gm2 Í ˙ fi F0 =
Î 1 - 1/4 ˚ 3
12. (a, d)
Same force acts on both masses.
1
Hence, a μ (F = ma)
m
In the absence of external force (remember mutual gravitational force is an internal
force for the system) total energy remains constant.

13. (b, c)
mgx
Net force towards centre of the earth = mg¢ =
R
Normal force N = mg¢ sin q
mgx R
Thus, pressing force N =
R 2x
mg
N= constant and independent of x.
2
Tangential force F = ma = mg¢ cos q

R2
- x2
gx 4 N
Q = g¢ cos q =
R x
mg q R/2
gx
a= R2 - 4 x 2 x
R
R
Curve is parabolic and at x = ,a=0
2
812പWŚLJƐŝĐƐĨŽƌ//dͲ:͗DĞĐŚĂŶŝĐƐ//

14. (a, d)
2mr 2r
r2 = =
m + 2m 3

4p 2 r23
T22 =
GM r2 r1

32p r2 3 m 2 1 2m
T22 = r c.m.
27GM
T2 μ r3/2; T2 μ m–1/2

15. (a, d)
GM GM
Outside the earth g = 2
, inside the earth g = r
r R3
Passages and Matrix Matching
Passage (Q. 1–2)
We have T2 μ r3 R2
v1
2 3 v2
Ê T1 ˆ Ê R1 ˆ
\ ÁË T ˜¯ = ÁË R ˜¯
2 2 R1 S2
S1
2 3
Ê 1ˆ Ê R1 ˆ
or ÁË ˜¯ = Á ˜
8 Ë R2 ¯
fi R2 = 4R1 = 4 ¥ 104 km.
Let v1 and v2 be the linear speeds of S1 and S2 with respect to the planet.
Then
2p R1
v1 = = 2p ¥ 104 km/h
T1
2p R2
and v2 = = p ¥ 104 km/h
T2
1. (b) At the closest separation, they are moving in the same direction. Therefore, the
speed of S2 with respect to S1 is |v2 – v1|= p ¥ 104 km/h.
2. (d) As seen from S1 the satellite S2 is at a distance r = R2 – R1 = 3 ¥ 104 km. At the
closest separation.
|v - v | p ¥ 10 4
w = 2 1 =
R2 - R1 3 ¥ 10 4
p
= rad/h
3
Passage (Q. 3–5)
3. (b) When A is given its first impulse at that moment B is ahead of A angularly.
-GMm GMm
4. (c) U = and KE =
r 2r
for same m, if r1 > r2, KE1 < KE2 and U1 > U2
'ƌĂǀŝƚĂƟŽŶപ813

Also, angular momentum of A before reaching the position 2 (during elliptical


path) was less than that of B. Since, some impulse is needed (at position 2) for A
in direction of its motion to equal its speed and angular momentum equal to that
of B.
5. (c) In the continuation with same comprehension.
(r2 + r2/3)
r= = 2r2/3
2
Let time period of A in elliptical path be T0.
2 3
Ê T0 ˆ Ê rˆ T (2 2 )
\ ÁË ˜¯ = Á ˜ \ T0 =
T Ë r2 ¯ 3 3
\ Time taken by A to move from 1 to 2,
T 2
t = T0/2 =
3 3
Passage (Q. 6–8)
Given, mass of sphere of radius 1 m = M.
\ Mass of the whole sphere of radius 4 m,
4
M ¥ p ( 4 )3
M0 = 3 = 64 M.
4
p (1)3
3
6. (c) The gravitational field at B
= field due to whole sphere – field due to sphere A
GM0 r GM
= -
R3 AB2
G(64 M ) ¥ 2 GM
or EB = - 2
43 4
31
= GM
16
7. (b) The radius of the given circle is 6 m. Obviously the point will lie outside the sphere
in yz-plane.
P

6m

A 2 m O 2 mB
814പWŚLJƐŝĐƐĨŽƌ//dͲ:͗DĞĐŚĂŶŝĐƐ//

The distance AP = BP = 6 2 + 22
= 40 m
The potential at P = Potential due to whole sphere – 2 (potential due to cavity)
-GM0 È GM ˘
= + 2Í
OP Î AP ˙˚

-G(64 M ) 2 GM
= +
6 40
= – 10.98 GM
2 2
8. (a) The given circle, y + z = 4, has radius 2 m. Obviously the point will lie inside the
sphere.
y

2m
x
A 2 mO 2 m B

The distance AP = BP = 2 2 m.
The potential due to whole sphere is given by
GM0
=– ( 3R 2 - r 2 )
2R 3
G(64 M )(3 ¥ 4 2 - 22 )
=
2 ¥ 43
= – 22 GM.
GM GM
The potential due to each cavity = + =+
AP 2 2
GM
Thus, net potential at P = – 22 GM + 2 ¥
2 2
= – 21.29 GM
Passage (Q. 9–10)
GM
9. (a) v0 =
r
2GM
R GM
or =
2 ( R + h)
\ h =R
'ƌĂǀŝƚĂƟŽŶപ815

GMm 1 GMm
10. (b) 0– = mv 2 -
( R + h) 2 R
- GMm 1 GMm
or = mv 2 -
( R + R) 2 R

\ v = gR

Passage (Q. 11–12)

1 GMm 1 GMm
11. (c) mv 2 - = mv ¢ 2 -
2 R 2 ( R + h)

1 Ê 2GM ˆ GMm 1 GMm


or mÁ ˜¯ - = mv ¢ 2 - ...(i)
2 Ë R R 2 R+h

2 gR2
\ v¢ =
( R + h)
dy
12. (b) In Eq. (i), put v¢ =
dt
and solving, we can get the required time

1 2R ÈÊ ˘
3/2

t = ÍÁ 1 + ˜ - 1˙
3 g ÍÎË R¯ ˙˚

Matrix Matching
13. A Æ (p); B Æ (q), (r); C Æ (p), (s); D Æ (r)
14. A Æ (q); B Æ (q); C Æ (s); D Æ (r)
GM
g= ,
R2
g g
At height h = R: g¢ = =
h 2
1+
R
1
i.e, g¢ decreases by a factor
2
2
Similarly, at height h = R/2, g¢ = g
3
R Ê hˆ Ê 1ˆ g
At depth h = : g¢ = g Á 1 - ˜ = Á 1 - ˜ =
2 Ë R¯ Ë 2¯ 2
3
Similarly, at h = R/4, g¢ = g
4
816പWŚLJƐŝĐƐĨŽƌ//dͲ:͗DĞĐŚĂŶŝĐƐ//

15. A Æ (q); B Æ (t); C Æ (r); D Æ (s)


GM
Inside a shell V = = constant and E = 0
R
GM GM
Outside the shell, V = – and E = 2
r r
As r increases, V increases and E decreases.
16. A Æ (p); B Æ (s); C Æ (r); D Æ (q)
Angular momentum of particle
5 GMe
= m(v0 + v)a = mv0 a … v0 =
4 a
Total energy of particle
1 GMe m 1 5 GMe m
= m(v0 + v)2 - = ¥ mv02 -
2 a 2 4 a
5 GMe m GMe m 3GMe m
= - =-
8 a a 8a
1 GMe m
At any distance r TE = mu2 -
2 r
But angular momentum is conserved,
5GMe 5 GMe a
mur = m ◊a fi u =
4a 4 r2

1 5 GMe a GMe m
TE at any distance r = m -
2 4 r2 r
But through conservation of energy, total energy
1 5 GMe a GMe m 3GMe m
= m - =-
2 4 r2 r 8a
On solving 3r2 – 8ar + 5a2= 0 fi (r – a) (3r – 5a) = 0
fi r = a, r = 5a/3
Minimum distance = a, Maximum distance = 5a/3
17. A Æ (p); B Æ (q); C Æ (r); D Æ (s)
5 CHAPTER

Mechanical Properties of Matter

INTRODUCTION
Matter
Everything that has mass and occupies space

Mixtures
Pure Substances
∑ Variable composition
∑ Fixed composition
∑ Components retain their ∑ Cannot be separated into simpler
characteristic properties substances by physical methods
Physical
∑ May be separated into pure change ∑ Properties do not vary
components by physical methods
∑ Mixtures of different composition,
may have widely different properties

Homogeneous Homogeneous
Mixtures Mixtures Compounds Elements
∑ Have same ∑ Do not have same can be Cannot be
composition composition decomposed decomposed
throughout throughout into simpler into
∑ Components are ∑ Components are substances by simpler
indistinguishable, indistinguishable, chemical Chemical substances by
for example, for example, changes, always change chemical
a gaseous mixture carbon and sulphur at constant changes
or a liquid solution mixture (gun powder) composition

Fig. 5.1 Classification of matter

Elastic Behaviour in Terms of Interatomic Forces


The interatomic forces are electrical in nature. As you know atoms consist of very small
positive charged nuclei (size n 10–15 m) and negative charged electrons distributed over
a distance of n 10–10 m (atomic size). Each atom is electrically neutral. Now if they are
818പWŚLJƐŝĐƐĨŽƌ//dͲ:͗DĞĐŚĂŶŝĐƐ//

electrically neutral how do they attract each other. The force acting between the atoms of
a molecule due to electrostatic interaction between the charges of the atoms is known as
interatomic force. It arises due to interaction between the nuclei of two atoms, their electron
clouds and between nucleus of one atom and the electron cloud of the other atom. Attractive
forces begin to act between the nucleus of one atom and the electrons of the other atom.
Repulsive forces begin to act between the two nuclei as well as between the electrons of
two atoms.
The potential energy U and interatomic force F vary with interatomic separation r as
follows:
A B
U = n- m ...(i)
r r
For most of the substances n = 12 and m = 6, A and B are constants
dU nA mB
and F =- = - ...(ii)
dr r n + 1 r m + 1

U
F

Repuls
r0 = Equilibrium distance
which corresponds to F = 0
i on
r0
r
O
r0 r
O
U0 tion
Interatomic ttrac
A

Fig. 5.2 Potential energy Fig. 5.3 Force F between two identical atom as a
function of interatomic separation r

INTERMOLECULAR FORCES
The force acting between the two molecules of a substance due to electrostatic interaction
between their oppositely charged ends is known as intermolecular. These forces are short
range forces. They operate over distances of molecular size (10–9 m).

ORIGIN OF INTERMOLECULAR FORCES


When two molecules approach each other, the centres of positive and negative charges shift
due to their mutual electric fields of electron clouds. Each molecule thus becomes an induced
dipole. They now interact due to oppositely charged ends. This intermolecular interaction is
called van der Waals attraction, which is much weaker than interatomic attraction.

+ – + –

Fig. 5.4
DĞĐŚĂŶŝĐĂůWƌŽƉĞƌƟĞƐŽĨDĂƩĞƌപ819

For the intermolecular separation r > r0, the intermolecular force varies as
A
F = - ...(i)
r7
For distance r < r0, the intermolecular forces become repulsive and given by
B
F = ...(ii)
r9

r0 r
ction
tra
At

Fig. 5.5

Similarities and Dissimilarities between Interatomic and Intermolecular


Forces
Points of similarity
(i) Both the forces are electrical in nature.
(ii) Both are repulsive at very small separations, but attractive at large separations. Both
cease to exist at a critical separation r0.
(iii) Both exist only at short distance.
Point of dissimilarity
(i) Interatomic forces are stronger than intermolecular force, 50 to 100 times.
(ii) Interatomic forces depend only upon the distance between the two atoms, whereas
intermolecular forces depend upon the relative orientations of the molecules also, i.e.,
when the molecules are not exactly spherical.
(iii) The critical value of separation (r0) at which these force are zero (or PE is minimum)
is smaller in case of atoms than in case of molecules.
(iv) Molecules are not restricted to attract just one molecule only, whereas interatomic
forces cannot bind the atoms of another molecule.
(v) Interatomic and intermolecular forces are electrostatic in atom but they do not obey
Coulomb’s law.

What is Deforming Force? Explain


When force is applied in such a way that the body has no translation motion, but molecules
of body are forced to undergo a change in their relative position, is called deforming force.
As a result of this force, the body may undergo a change in length, volume or shape. The
force can be applied by the following ways:
820പWŚLJƐŝĐƐĨŽƌ//dͲ:͗DĞĐŚĂŶŝĐƐ//

(i) When the body is subjected to two equal and opposite forces.
(ii) When one end of the body is connected to a fixed support and the other end is
subjected to a force. In this case there develops an equal amount of reaction force on
the fixed end.

F F F T T F

R=F F F T T F

Fig. 5.6

In both the cases, the force on any section of the body will be T – F = 0 or T = F. It is
not 2F, or zero.
(iii) When force is applied at one end of the body, the inertia plays a significant role to
stretch the body. Let a body of mass m is subjected to force F at its one end. The force
T at any section can be calculated as:
F
Acceleration a =
m

F T T F

l x l–x

Fig. 5.7

Mass of the x length of the body


m
m¢ = x
l
By Newton’s second law for the mass m, we have
T = m¢A
Ê m ˆ F Fx
= Á x˜ =
Ë l ¯m l
For x = 0, T = 0
l F
For x = ,T =
2 2
For x = l, T = F
DĞĐŚĂŶŝĐĂůWƌŽƉĞƌƟĞƐŽĨDĂƩĞƌപ821

Stress
When an external force deforms a solid. The intermolecular distance gets increased because
of which attractive intermolecular force gets developed. Their resultant is called the restoring
force.
Restoring force per unit area is called stress.
Applied force F
Stress (f) = =
Area A
The SI unit of stress is N/m2. This is also called Pascal 1 N/m2 = 1 Pa. Dimensional
formula is ML–1 T–2.

Strain
Application of external deforming force results in some deformation so the body undergoes
a change in size and shape
Change in dimension
Strain (e) = Original dimension

Strain is a dimensionless quantity.

TYPES OF STRESS AND STRAIN


Depending on the force applied on the body, there are three types of stresses and
corresponding strains.
1. Direct or longitudinal stress and strain
The force is applied along one direction of the body in such a way that its shape will
not change. It may be tensile force or compressive force. Accordingly, there are two
types of direct stresses:
(i) Tensile stress
(ii) Compressive stress

F F F F
A
(a) Body subjected to tensile force Tensile stress
FBD

F F F F

(b) Body subjected to compressive force Compressive stress


FBD

Fig. 5.8

F
Direct stress f =
A
Dl
Longitudinal strain e =
l
where Dl is the change in length of the body and l is the original length of the body.
822പWŚLJƐŝĐƐĨŽƌ//dͲ:͗DĞĐŚĂŶŝĐƐ//

2. Normal stress and volumetric strain


When the body is subjected to a uniform force from all F V
possible directions and shape of body does not change, then
the corresponding stress is called normal stress. When the F
(V – DV ) F
body is placed inside a fluid, it subjected to uniform pressure
from all round.
Consider a body of volume V and surface area A. When it is F
subjected to uniform force F from all sides, then
Fig. 5.9
F
Normal stress f = = pressure (P)
A
Change in volume
Volumetric strain e =
Original volume
DV
=
V
3. Shear stress and shear strain Dl
When a force acts in such a way that it changes the
shape of the body, the corresponding stress is called
shear stress.
F
F f
f
Shear stress f = A
A
Shear stress Fixed face
Dl
Shear strain e = f tan f =
l Fig. 5.10

(a) (b)

Fig. 5.12
(a) (b)
Examples of Shear Stress. Shear stress is produced
Fig. 5.11
whenever the applied loads cause (or tend) one section of
the body to slide past its adjacent section.
Examples of Normal Stress
(a) A river resists shear across its cross-sectional area.
(a) A load is suspended by a wire. (b) The fibres of a shaft resist shear stress when it is twisted
Tensile stress is produced in by an external torque.
the wire.
(b) A column is supporting a ver-
tical load. Compressive stress
is produced in the wire.
DĞĐŚĂŶŝĐĂůWƌŽƉĞƌƟĞƐŽĨDĂƩĞƌപ823

WHAT IS MEANT BY ELASTIC LIMIT?


The maximum stress within which body can regain its original size and shape after removal
of the deforming force is called elastic limit. If the body exceed this limit, then it will not get
original size and shape completely, it will get permanent set and is said to be overstrained.

HOOKE’S LAW
It states that within elastic limit, the stress is directly proportional to the strain. Thus,
Stress μ Strain
or Stress = E ¥ strain
Stress
or =E
Strain
where E is a constant called modulus of elasticity or coefficient of elasticity. It is a material
property which does not depend on size and shape of the body.

DIFFERENT TYPES OF MODULII OF ELASTICITY


Depending on three types of stresses and strains, there are three types of modulii of elasticity,
viz., Young’s modulus, bulk modulus and shear modulus. Their values depend on the nature
of material of the body and manner in which it is deformed.
1. Young’s modulus of elasticity
It is defined as:
Longitudinal stress
Y=
Longitudinal strain
Suppose a wire of length l and cross-sectional area A is subjected to
external force F, then Young’s modulus
f F/A
Y= = l
e Dl/l
Fl
or Y= ...(i)
ADl
Fl
Also, Dl = ...(ii) A Dl
AY
For a circular cross section with a suspended load Mg, we have F = F
Mg and A = pr2. Fig. 5.13
Mgl
\ Y= ...(iii)
p r 2 Dl
Note: The above equation can be written as

YA
F= Dl .
l
824പWŚLJƐŝĐƐĨŽƌ//dͲ:͗DĞĐŚĂŶŝĐƐ//

YA
or, if is replaced by a single constant k and elongation Dl is represented by x, then
l
È YA ˘
F = kx ÍÎk = l ˙˚

Hooke’s law was originally stated in this form, rather than in terms of stress and strain.

Determination of Young’s Modulus by Searle’s Method


Experiment shows a metal wire is subjected to a gradually increasing load. It is found that
the strain is directly proportional to the corresponding stress up to a certain limit only and
beyond that the relation is non-linear. In investigating the mechanical properties of material
beyond this limit, the relationship between the strain and corresponding stress is usually
represented by tensile test diagram.

Stress (f )
0
Metal wire 1
2 Ultimate or
breaking S
3 Fracture point
strength Q
4 R
5 Scale P
A Æ Proportional limit
B Æ Elastic limit or yield point
C Æ Plastic behaviour

Permanent set
O <1% E Strain (e)

Fig. 5.14

Explanation of different parts of stress-strain curve


(i) OP: In this region, the stress is proportional to strain up to A and Hooke’s law is
obeyed. Point P is called proportional limit. Slope of line OP gives Young’s modulus
of material of wire.
(ii) PQ: From P to Q strain is not proportional to stress, but if the load is removed at
any point between O and Q, the curve will be retraced and the material will return
to its original length. In the region OQ, the material is said to be elastic and the
point Q is called elastic limit. Up to point Q, the elastic forces of the material are
conservative; when the material returns to its original shape, work done in producing
the deformation is recovered.
(iii) Beyond Q: If the wire is loaded further, the strain increases rapidly than the
corresponding stress. If the load is removed from any point beyond Q (say R), the
wire does not come back to its original length but traverses the dotted line RE as
shown in Fig. 5.14. Even on reducing the stress zero, a small strain equal to OE is left
in the wire, and the material is said to have a permanent set.
DĞĐŚĂŶŝĐĂůWƌŽƉĞƌƟĞƐŽĨDĂƩĞƌപ825

(iv) Beyond R: If the wire is loaded beyond R, a large Stress


increase in strain is observed until point S is reached High carbon steel
at which fracture (failure) takes place. From Q to S,
the material is said to undergo plastic flow, which is Cast iron
irreversible. The stress corresponding to the fracture Aluminium
point is called ultimate or breaking strength. Concrete
Figure 5.15 shows the stress-strain curves for differ- Strain
ent materials.
Fig. 5.15
Important:
1. For the hard materials like high carbon steel, cast-iron, points P and Q are indifferentiable.
2. The material with large region BD will be more ductile and small region of QS is called
brittle material.

ELASTIC HYSTERESIS Stress


B
Some materials like rubber can be stretched to over
Loading curve
six to seven times its original length. These materials
A Unloading curve
are called elastomers. When unloaded, the material is
restored to its original length. But any stage of loading C
and unloading, strain is not proportional to stress. The
fact that stress-strain curve is not retraced on unloading, O Straim
is known as elastic hysteresis.
Fig. 5.16
Important:
1. Rubber has large amount of elasticity, but it does not obey Hooke’s law.
2. The area of the loop OABCO represents the energy lost as heat during the loading-
unloading.

WORKED PROBLEMS
Problem 5.1 The stress-strain graphs for materials P and Q are shown in Fig. 5.17.
Stress Stress

P Q

Strain Strain
(a) (b)

Fig. 5.17
826പWŚLJƐŝĐƐĨŽƌ//dͲ:͗DĞĐŚĂŶŝĐƐ//

The graphs are drawn to the same scale.


(a) Which of the material has greater Young’s modulus?
(b) Which material is more ductile?
(c) Which is more brittle?
(d) Which of the two is stronger?
Solution
(a) As the slope of stress-strain curve P is greater, so Young’s modulus A will be greater
than that of Q.
(b) As plastic region for P is greater, so material P is more ductile.
(c) More ductile material will be brittle, so material A is less brittle.
(d) The breaking strength of material P is greater than material Q, so material P is stronger
than Q.
Problem 5.2 Two different types of rubber are found to have the stress-strain curves as
shown in Fig. 5.18.
(a) A heavy machine is to be installed in a factory. To absorb the vibrations of the machine,
a block of rubber, is placed between the machinery and the floor. Which of the two
rubbers A and B would you prefer to use for this purpose? Why?
(b) Which of the two rubber materials would you choose for car tyres?
Stress Stress

Rubber A Rubber B

Strain Strain
(a) (b)

Fig. 5.18

Solution It is clear from the figures that area enclosed in hysteresis loop is larger for B than
A. So rubber B will absorb more energy than A.
(a) In heavy machines, the energy of vibrations is quite large, so rubber B is suitable for
the purpose.
(b) In a car tyre small energy is to be absorbed by tyres (more by air inside it) so rubber
A is suitable for the purpose.
Problem 5.3
(a) It is possible to double the length of a metallic wire by applying a force over it?
(b) Is elastic limit a property of the material of the wire?
(c) Which is more elastic: water or air?
(d) Why does a wire get heated when it is bent back and forth?
DĞĐŚĂŶŝĐĂůWƌŽƉĞƌƟĞƐŽĨDĂƩĞƌപ827

(e) The ratio stress/strain remains constant for a small deformation. What happens to this
ratio if deformation is made very large?
(f) Why are electric/telephone poles given hollow structure?
Solution
(a) No, because elastic limit strain is about 10–3 and wire actually breaks much before it
is stretched to double its length.
(b) No. It also depends on the radius of the wire.
(c) Water is more elastic than air. More strain is produced in air compared to water for
some amount of stress, so ratio stress/strain is small for air.
(d) When a wire is bent back and forth, its deformations are beyond elastic limit, so the
work done against interatomic forces is no longer stored entirely in the elastic potential
energy and some part of it will convert into heat.
(e) When the deforming force exceeds the elastic limit, the strain increases more rapidly
than stress. Hence, the ratio stress/strain decreases.
(f) Hollow poles have greater moment of inertia in comparison to solid poles made from
the same amount of material and so less deformation.
Problem 5.4 Two wires of diameter 0.25 cm, one made of steel and other made
of brass are loaded as shown in Fig. 5.19. The unloaded length of the steel wire
is 1.5 m and that of brass is 1.0 m. Young’s modulus of steel is 2.0 ¥ 1011 Pa.

1.5 m
Compute the elongations of steel and brass wires. (IIT Roorkee, 1976) Steel

Solution For brass wire:


4 kg
l = 1.0 m, r = 0.125 ¥ 10–2 m

1.0 m
F = 6 ¥ 9.8 N, Y = 0.91 ¥ 1011 N/m2 Brass
Fl
We know Dl = 6 kg
AY
(6 ¥ 9.8) ¥ 1.0 Fig. 5.19
=
p (0.125 ¥ 10 -2 )2 ¥ (0.91 ¥ 1011 )
= 1.32 ¥ 10–4 m
For steel wire:
l = 1.5 m, r = 0.125 ¥ 10–2 m
F = (6 + 4) ¥ 9.8 N = 98 N, Y = 2.0 ¥ 1011 N/m2
Fl
We know Dl =
AY
98 ¥ 1.5
=
p (0.125 ¥ 10 -2 )2 ¥ (2.0 ¥ 1011 )
= 1.5 ¥ 104 m Ans.
Problem 5.5 Read each of the statements below carefully and state, with reasons, if it is
true or false.
828പWŚLJƐŝĐƐĨŽƌ//dͲ:͗DĞĐŚĂŶŝĐƐ//

(a) The modulus of elasticity of rubber is greater than that of steel.


(b) The stretching of a coil is determined by its shear modulus.
(c) Elastic restoring forces are strictly conservative only when Hooke’s law is obeyed.
Solution
(a) False. When steel and rubber are subjected to same amount of deforming force, large
È stress ˘
amount of strain will be produced in the rubber. The ratio Í and hence, Young’s
modulus will be smaller for rubber. Î strain ˙˚
(b) True. When a coil spring is subjected to a force, its shape will change.
(c) False. Elastic forces are conservative even material subjected to these forces does not
obey Hooke’s law.
Problem 5.6 A bar of cross section A is subjected to equal and opposite tensile forces at
its ends. Consider a plane section of the bar whose normal makes an angle q with the axis
of the bar.
(a) What is the tensile on this plane?
(b) What is the shearing stress on this plane?
(c) For what value of q is the tensile stress maximum?
(d) For what value of q is the shearing stress maximum?
Solution
A/cos q

Fn
A A

F F
F
q

q Ft
B

Fig. 5.20

AB = A¢
AB cos q = A
A¢ cos q = A
A
A¢ =
cos q
A
The area of the given section A¢ =
cos q
The tensile force F can be resolved into two perpendicular components, Fn and Ft, where
Fn = F cos q and Ft = F sin q.
DĞĐŚĂŶŝĐĂůWƌŽƉĞƌƟĞƐŽĨDĂƩĞƌപ829

Fn F cos q F
(a) Tensile stress = = = cos 2 q
A¢ ( A/cos q ) A
Ft F sin q F
(b) Shearing stress = = = sin q cos q
A¢ ( A/cos q ) A
F
= sin 2q
2A
(c) Tensile stress will be maximum when cos2 q is maximum, i.e., cos q = or q = 0°
(d) Shear stress will be maximum when sin 2q is maximum, i.e., sin 2q = 1 or 2q = 90° or
q = 45°.
Problem 5.7 Four identical hollow cylindrical columns of steel support a big structure of
mass 50,000 kg. The inner and other radii of each column are 30 cm and 40 cm respectively.
Assuming the load distribution to be uniform, calculate the compressional strain of each
column. The Young’s modulus of steel is 2.0 ¥ 1011 Pa.
50000 ¥ 9.8
Solution The load wear by each column = = 1.225 ¥ 10 5 N
4
Resisting area of each column = p (R2 – r2)
= p [(0.4)2 – (0.3)2] = 0.22 m2
Stress
Compressional strain =
Young’s modulus
F/A F
= =
Y YA
1.225 ¥ 10 5
=
(2.0 ¥ 1011 ) ¥ (0.22)
= 2.87 ¥ 10 – 6

Extension Due to Self Weight


Consider a rod of length l and cross-sectional area A. If density of its
material is r, then weight of the rod W = rAlg. The rod is hanging from a
rigid support. The specimen extends due to self weight.
Let us consider an element of thickness dx at a distance x from the free
end.
dx
The weight of the element W¢ = xArg.
The extension of the element due to this load x
W ¢(dx )
dl =
AY
Fig. 5.21
( xAr g )dx r g
= = xdx
AY Y
830പWŚLJƐŝĐƐĨŽƌ//dͲ:͗DĞĐŚĂŶŝĐƐ//

l l
rg
The total extension Dl = Údl = Y Ú0
xdx
0

rgl 2
or Dl =
2Y
Substituting W = rAlg in the above equation, we get
Wl
Dl =
2 AY
Note: If the rod is subjected to an external force in addition to its weight, then its total
extension
Fl Wl
Dl = +
AY 2 AY

Thermal Stress
Consider a specimen which is clamped between two rigid supports
and subjected to change in temperature by DT. If the specimen is free
to expand, its length will increase by la DT. A

Dl laDT
The strain produced e = =
l l Dl
= a DT
A
This strain is prevented by the supports and therefore stress is
induced in each section of the specimen.
Fig. 5.22
Thus, thermal stress fth = eY = a DTY
or fth = YAa DT
The force exerted by the support F = fth A = YAa DT.

WORKED PROBLEM
Problem 5.8 A steel wire of uniform cross section 1 mm2 is heated to 70°C and stretched by
tying it two ends rigidly. Calculate the change in tension of the wire when the temperature
falls from 70° C to 35° C. Coefficient of linear expansion of steel is 1.1 ¥ 10–5/°C and the
Young’s modulus is 2.0 ¥ 1011 N/m2.
Solution We know that thermal stress
fth = YaDT
here Dt = 70 – 35 = 35°C
\ fth = 2.0 ¥ 1011 ¥ 1.1 ¥ 10–5 ¥ 35
= 7.7 ¥ 107 N/m2
The change in tension (decrease because temperature falls)
= fth A = (7.7 ¥ 107) ¥ (1 ¥ 10 –6)
= 77.0 N
DĞĐŚĂŶŝĐĂůWƌŽƉĞƌƟĞƐŽĨDĂƩĞƌപ831

Bulk Modulus of Elasticity


Let a three-dimensional solid body is subjected to great outside pressure p on it. So, because
of this pressure, the volume of the body will shrink. The intermolecular distances will
decrease, repelling forces will come into play which will give rise to internal restoring forces.

Volumetric strain
Fig. 5.23

Let A= Original surface area of the body


DA = Reduction in surface area
p= Original pressure
Dp = Increase in pressure due to reduction in surface area
V= Original volume
DV = Reduction in volume
K or B= Bulk Modulus
= Volumetric stress/volumetric strain
\ p Æ p + Dp causes
V Æ V – DV
\ Force [(p + Dp) A – p ◊ A] causes a change of (– DV) in volume.
fi Force [p ◊A + Dp ◊ A] causes a volumetric strain (–DV/V).
Ê DV ˆ
fi Force [Dp ◊ A] causes strain Á - .
Ë V ˜¯
È Dp ◊ A ˘ Ê DV ˆ
fi Stress Í ˙ causes strain ÁË - V ˜¯ .
Î A ˚
stress Dp
\ B = =
strain - DV/V
Dp ◊ V
B=-
DV
832പWŚLJƐŝĐƐĨŽƌ//dͲ:͗DĞĐŚĂŶŝĐƐ//

The SI unit of bulk modulus is Nm–2 or Pascal. Its dimensional formula is [ML–1 T – 2]. SI
unit of compressibility is N–1 m2.
Note: Sometimes B is written for bulk modulus and K for compressibility. If the value of
K is given, you can easily determine from the units whether K stands for bulk modulus or
compressibility.

The Bulk Modulus and Compressibility of Various Materials

Material Bulk Modulus (N/m2) Compressibility (m2/N)


Aluminium 7.7 ¥ 1010 0.13 ¥ 10–10
Copper 14 ¥ 1010 0.07 ¥ 10–10
Steel 16 ¥ 1010 0.06 ¥ 10–10
Marble 7 ¥ 1010 0.14 ¥ 10–10
Granite 4.7 ¥ 1010 0.21 ¥ 10–10
Mercury 27 ¥ 109 0.37 ¥ 10–10
Water 2.0 ¥ 109 4.9 ¥ 10–10
Ethyl alcohol 0.9 ¥ 09 11 ¥ 10–10

Important: The bulk modulus in differential form can be written as


dP
B=
Ê dV ˆ
ÁË - ˜
V ¯
Here negative sign indicates that with increase in pressure, the volume of body will
decrease.

Compressibility (k)
Reciprocal of bulk modulus is called compressibility (k).
1
\ k=
B
Shear Modulus
It is defined as:
Shear stress
h =
Shear strain
( F/A) F
= =
f Af
Dl
Here f is the shear strain, which is equal to .
l
F l
\ h = ◊
A Dl
DĞĐŚĂŶŝĐĂůWƌŽƉĞƌƟĞƐŽĨDĂƩĞƌപ833

Important Points
1. Solids possess all the three modulii of elasticity.
2. Liquids and gases possess only bulk modulus.
3. For a perfectly rigid body, strain produced will be zero and hence modulii of elasticity
will be •.
4. Shear occurs in case of twisting, cutting, and tearing.
5. Elasticity of the material gets affected by the following:
(i) Hammering and rolling – Increases.
(ii) Annealing (formation of large crystal) – Decreases.
(iii) Presence of impurities – May increase or decrease, depending on the mixing
material.
(iv) With increase in temperature, elasticity of most of the material decreases.

POISSON’S RATIO
2r
In the previous part of our study we have considered the change in
Dr
dimension of the body in the direction of applied force. We have ignored
l
the change in lateral direction of the body. When a force is applied along
Dl
the length of the specimen, the longitudinal strain e = . If radius r is
decreased by Dr, then l
- Dr
Lateral strain = Dl
r
Lateral strain
Poisson’s ratio, s =
Longitudinal strain F

- Dr/r Fig. 5.24


or =
Dl/l
The negative sign indicates that longitudinal and lateral strains are in opposite sense.

What is Lateral Strain?


Whenever a rod is subjected to longitudinal stress its length increases, but its diameter
decreases. Decrease in radius per unit radius is called lateral strain.
Important:
1. The theoretical value of s lies between –1 to 0.5. But in practice, the value of s lies
between 0.2 to 0.5. The value of s for rubber is 0.5.
9hB 3B - 2n
2. Y = 3B(1 – 2s), Y = 2h (1 + s), Y = ,s=
h + 3B 2n + 6B
Change in Volume
Consider a rod of circular cross section of radius r and length l. The volume of the rod
V = pr2 l ...(i)
834പWŚLJƐŝĐƐĨŽƌ//dͲ:͗DĞĐŚĂŶŝĐƐ//

We can write
DV 2Dr Dl
= + ...(ii)
V r l
- Dr/r
We know that s =
Dl/l
Dr Dl
\ = -s
r l
Substituting this value in Eq. (ii), we get

DV Ê Dl ˆ Dl
= 2Á -s ˜ +
V Ë l ¯ l

DV Dl
or = (1 - 2s )
V l
For no change in volume of the rod, DV = 0
Dl
\ (1 - 2s ) = 0
l
or s = 0.5
Thus, a material having Poisson’s ratio 0.5 suffers no change in volume when a force is
applied on it.

Change in Density
Consider a body of density r, is subjected to all round P
pressure P. Its initial density
M,
M
r = ...(i) M,V (V – DV)
V
M
and final density r¢ = ...(ii)
(V - DV )
P Fig. 5.25
We have B =
Ê DV ˆ
ÁË ˜
V ¯
PV
\ DV =
B
Substitute the value of DV in Eq. (ii), we have
M M
r¢ = =
Ê PV ˆ Ê Pˆ
ÁË V - ˜ V ÁË 1 - ˜¯
B ¯ B
DĞĐŚĂŶŝĐĂůWƌŽƉĞƌƟĞƐŽĨDĂƩĞƌപ835

M
From Eq. (i), r =
V
r
\ r¢ =
P
1-
B
r
If a body is inside water at a depth h, then P = rw gh and r¢ = .
Ê rw gh ˆ
ÁË 1 - ˜¯
B
WORKED PROBLEMS
Problem 5.9 Compute the bulk modulus of water from the following data: Initial volume =
100 litre, pressure increase = 100 atm, final volume = 100.5 litres (1 atm = 1.013 ¥ 105 N/m2).
Solution Given, increase in pressure P = 100 ¥ 1.013 ¥ 105 N/m2
Increase in volume DV = Vf – Vi
= (100.5 – 100) ¥ 10 – 3 m3
= 0.5 ¥ 10 – 3 m3
We know that bulk modulus
P 100 ¥ 1.013 ¥ 10 5
B = =
Ê DV ˆ Ê 0.5 ¥ 10 - 3 ˆ
ÁË ˜¯ Á -3 ˜
V Ë 100 ¥ 10 ¯
= 2.026 ¥ 109 N/m2
Problem 5.10 The edge of an aluminium cube are 10 cm long. One face of the cube is firmly
fixed to a vertical wall. A mass of 100 kg is then attached to the opposite face of the cube.
The shear modulus of aluminium is 25 GPa. What is the vertical deflection of this face?
Solution Shearing force F = mg F = 100 g
10 cm
= 100 ¥ 10 = 1000 N Dl
q
F 1000
Shear stress f = =
A 0.12
= 1 ¥ 105 N/m2 q
We know that, the shear modulus
f Longitudnal stress 1000 N
h = =
e Longitudnal strain Fig. 5.26
f 1 ¥ 10 5
\ e = = 9
= 4 ¥ 10 - 6
h 25 ¥ 10
If Dl is the vertical deflection of the face, then
Dl
e =
l
836പWŚLJƐŝĐƐĨŽƌ//dͲ:͗DĞĐŚĂŶŝĐƐ//

\ Dl = el = (4 ¥ 10– 6) ¥ (0.10)
= 4 ¥ 10 –7 m
Problem 5.11 A light rod of length 2.00 m is suspended from the ceiling horizontally by
means of two vertical wires of equal length tied to its ends. One of the wires is made of steel
and is of cross section 10–3 m2 and the other is of brass of cross section 2 ¥ 10–3 m3. Find out
the position along the rod at which a weight may be hung to produce:
(i) equal stresses in both wires, and
(ii) equal strains in both wires.
Young’s modulus of brass = 1011 N/m2
Young’s modulus of steel = 2 ¥ 1011 N/m2.
Solution Suppose a1 and a2 are the cross sectional areas, and
Y1 and Y2 are the Young’s modulii of steel and brass wires
Steel Brass
respectively. Let T1 and T2 be the tensions in the steel and brass wire wire
wires respectively.
Let x be the position of the hanging weight from the steel wire. T1 T2
(i) First case: For equal stresses in both wires, we have A
C
B
x 2–x
Applied force T1 T2
Stress = = W
Area a1 a2
Fig. 5.27
T1 T2
or -3
=
10 2 ¥ 10 - 3
or T2 = 2T1 ...(i)

As the whole system is in equilibrium, so  t = 0.
Taking moment of all the forces acting on the rod about C, we have
T1x – T2(2 – x) = 0 ... (ii)
Anticlockwise is taken + ve
Solving Eqs. (i) and (ii), we get
4
x = m
3
(ii) Second case: For equal strains in both the wires
e1 = e 2
f1 f
= 2
Y1 Y2
T1/a1 T /a
or = 2 2
Y1 Y2
T1 T2
or -3 11
= -3
10 ¥ 2 ¥ 10 2 ¥ 10 ¥ 1011
or T1 = T2 ...(iii)
From Eqs. (ii) and (iii), we get
x =1m Ans.
DĞĐŚĂŶŝĐĂůWƌŽƉĞƌƟĞƐŽĨDĂƩĞƌപ837

Problem 5.12 Two rods of equal cross sections, one of copper and the other of steel are
joined to form a composite rod of length 2 m at 20°C, the length of the copper rod is 0.5 m.
When the temperature is raised to 120°C, the length of the composite rod increases to 2.002
m. If the composite rod is fixed between two rigid walls and thus not allowed to expand,
it is found that the length of the component rods also does not change with increase in
temperature. Calculate the Young’s modulus and the coefficient of linear expansion of steel.
Given Young’s modulus of copper = 1.3 ¥ 1011 N/m2 coefficient of linear expansion of copper
ac = 1.6 ¥ 10 – 5/°C.
Solution The change in the length of the copper rod due to change in temperature from
20°C to 120°C
Dl1 = lcac DT as Dl = la DT
= 0.5ac(120 – 20) = 50ac
For steel, Dl2 = ls as DT
= 1.5ac (120 – 20) = 150a s
Total change in length Dl = Dl1 + Dl2 = 50ac + 150a s
It is given that Dl = 0.002 m
\ 50ac + 150a s = 0.002
0.002 - 50a c
or as =
150
0.002 - 50 ¥ 1.6 ¥ 10 - 5
=
150
= 0.8 ¥ 10–5/°C
If there is no change in the length of individual rod, then stress in both the rods must be
equal.
So, fth = yaDT
fsteel = fcopper
or Ysas DT = Ycac DT
Yca c
or Ys =
as
1.3 ¥ 1013 ¥ 1.6 ¥ 10 - 5
=
0.8 ¥ 10 - 5
= 2.6 ¥ 1015 N/m2
Problem 5.13 A steel wire of diameter 0.8 mm and length 1 m is clamped firmly at two
points A and B which are 1 m apart and in the same plane. A body is hung from the middle
point of the wire such that the middle point sags 1 cm lower from the original position.
Calculate the mass of the body. Given that Young’s modulus of the material of wire is 2 ¥
1011 N/m2.
Solution As shown in Fig. 5.28, for equilibrium of mass M,
Mg = 2T sin q ...(i)
838പWŚLJƐŝĐƐĨŽƌ//dͲ:͗DĞĐŚĂŶŝĐƐ//

But from Fig. 5.28, A L L B


sin q = tan q = (x/L) ...(ii)
q q
x
and by definition of Young’s modulus, T T
YA YA 2 YAx 2
T = DL = [(L + x 2 )1/2 - L] ...(iii)
L L 2L2
So substituting the values of sin q and T from Eqs. (ii) Mg
and (iii) in Eq. (i), we get Fig. 5.28
YAx 2 x YAx 3
Mg = 2 ¥ ¥ , i.e., M= ...(iv)
2L2 L gL3
Now as 2L = 1 m, x = 1 cm = 10–2 m
and A = pr2 = p(0.8/2) ¥ 10–3] 2 = p ¥ (4 ¥ 10 – 4)2 m2

2 ¥ 1011 ¥ p( 4 ¥ 10 - 4 )2 ¥ (10 - 2 )3
So, M = kg = 82 kg
9.8 ¥ (1/2)3
1/3 1/3
Ê Mg ˆ x Ê Mg ˆ
Note: As from Eq. (iv) x = L Á so q  =Á
Ë YA ˜¯ ˜
L Ë YA ¯

Problem 5.14 1 kg weight is suspended by a rubber cord 2.00 m long and of cross section
0.5 cm2. It is made to describe a horizontal circle of radius 50 cm in 4 times a second. Find
the extension of the cord. (Young’s modulus Y = 5 ¥ 108 N/m2)
Solution Given frequency of rotation, n = 4 per second
\ Angular frequency, w = 2p n
=2¥p¥4
= 8 p rad/s q
If T is the tension in the cord and it makes angle q with
the vertical, then by Newton’s second law l

T sin q = mw2r ...(i) T cos q


and T cos q = mg ...(ii) q T
Squaring and adding the above equations, we get O
2 2 2
T = (mw r ) + (mg ) mg

The extension of the cord due to tension force T, Fig. 5.29


Fl Tl
Y = Dl =
ADl AY

(mw 2 r )2 + (mg )2
= ¥2
AY
DĞĐŚĂŶŝĐĂůWƌŽƉĞƌƟĞƐŽĨDĂƩĞƌപ839

{1 ¥ (8p )2 ¥ (0.5)}2 + (1 ¥ 9.8)2


= ¥2
(0.5 ¥ 10 - 4 ) ¥ (5 ¥ 108 )
= 2.53 ¥ 10 –2 m

ELASTIC POTENTIAL ENERGY OF A STRAINED BODY: STRAIN ENERGY


Suppose a wire of length l and cross-sectional area A is fixed at one end and is stretched by
an external force applied at the free end. The force is adjusted in such a way that it stretched
F x
slowly. When the extension is x, the wire under direct stress and strain .
A l
f F/A Longitudinal stress
\ Y = = =
e x/l Longitudinal strain

AY
or F = x
l
Work done by external force in further extension dx, is
Ê AYx ˆ
dW = Fdx = Á dx
Ë l ˜¯
F
The total work done by the external force in an extension from 0 to Dl, is Fig. 5.30
Dl
Ê AY ˆ AY 2
W = Ú ÁË x˜ dx =
l ¯ 2l
Dl
0

This work is stored in its elastic potential energy.


AY 2
\ U = Dl
2l
1 Ê Dl ˆ Ê Dl ˆ
or U = Á Y ˜ ¥ Á ˜ ¥ ( Al)
2Ë l ¯ Ë l ¯
1
or U = ¥ Stress ¥ Strain ¥ Volume
2
Note:
1. Many times it is more useful to write elastic potential energy per unit volume (u). So
we can write
1 1 f f2
u = ¥ f ¥e = ¥ f ¥ =
2 2 Y 2Y
1 e 2Y
= ¥ (eY ) ¥ e =
2 2
840പWŚLJƐŝĐƐĨŽƌ//dͲ:͗DĞĐŚĂŶŝĐƐ//

2. Elastic potential energy due to shear of a body can be written as


f2
u =
2h
U 1 f 2 e 2Y
Note: u = = ¥ f ¥e = =
Volume 2 2Y 2

WORKED PROBLEMS
Problem 5.15 A 80 kg boy whose legs are 4 cm2 in area and 50 cm long falls through a
height of 2 m without breaking his leg bones. If the bones can withstand a stress of 0.9 ¥ 108
N/m2. Calculate the Young’s modulus for the material of the bone.
Solution In the process of fall mechanical energy of the body remains constant. Thus,
Loss in PE = Gain in elastic potential energy by both the legs
Ê1 ˆ
or mgh = 2 ¥ Á ¥ Stress ¥ Strain ¥ Volume˜
Ë2 ¯

f2
or mgh = 2 ¥ ¥ ( Al)
2Y
(0.9 ¥ 108 )2
or 80 ¥ 9.8 ¥ 2 = ¥ ( 4 ¥ 10 - 4 ¥ 0.50)
Y
After solving, we get, Y = 1.025 ¥ 109 N/m2
Problem 5.16 A steel rod of cross-sectional area 1 m2 is acted upon by forces shown in
Fig. 5.31. Determine the total elongation of the bar. Take Y = 2.0 ¥ 1011 N/m2.
A B C D
60 kN 10 kN 20 kN 20 kN

1.5 m 1m 2m

Fig. 5.31

Solution The action of forces on each part of rod is shown in Fig. 5.32.
10 kN 20 kN
60 kN 50 kN
60 kN 50 kN 50 kN 50 kN

60 kN 60 kN 50 kN 50 kN
70 kN 70 kN

Fig. 5.32
DĞĐŚĂŶŝĐĂůWƌŽƉĞƌƟĞƐŽĨDĂƩĞƌപ841

We know that the extension due to external force F is given by


Fl
D =
AY
(60 ¥ 10 3 ) ¥ 1.5
\ DAB = = 4.5 ¥ 10 - 7 m
1 ¥ 2 ¥ 1011
(70 ¥ 10 3 ) ¥ 1
DBC = = 3.5 ¥ 10–7 m
1 ¥ 2 ¥ 1011
(50 ¥ 10 3 ) ¥ 2
and DCD = = 5.0 ¥ 10–7 m
1 ¥ 2 ¥ 1011
The total extension D = DAB + DBC + DCD
= 4.5 ¥ 10 – 7 + 3.5 ¥ 10– 7 + 5.0 ¥ 10– 7
= 13 ¥ 10 – 7 m
Problem 5.17 A copper rod, 25 mm in diameter is encased in a steel tube of 30 mm inner
diameter and 35 mm external diameter. The ends are rigidly attached. The composite bar is
500 mm long and is subjected to an axial pull of 30 kN. Find the stresses induced in the rod
and the tube. Take Y for steel = 2 ¥ 105 N/mm2 and Y for copper 1 ¥ 105 N/mm2.
l = 0.5 m
Tube

F = 30 kN F = 30 kN
25 mm 35 mm Rod 30 mm

Tube

Fig. 5.33

Solution Let us use suffix 1 for copper rod and 2 for steel tube.
A1 = p(12.5)2 = 490.6 mm2
A2 = p[17.52 – 152] = 255.1 mm2
If f1 and f2 are the stresses in the rod and tube respectively, then
F
f=
A
f1A1 + f2A2 = F = 30000 N
or f1(490.6) + f2(255.1) = 30000 ...(i)
As both rod and tube are joined together, so
Dl1 = Dl2
f1l1 f l
= 22 (As l1 = l2)
Y1 Y2
Y
or f1 = 1 f 2
Y2
842പWŚLJƐŝĐƐĨŽƌ//dͲ:͗DĞĐŚĂŶŝĐƐ//

1 ¥ 10 5
or f1 = f 2 = 0.5 f2 ...(ii)
2 ¥ 10 5
Solving Eqs. (i) and (ii), we get
f1 = 29.97 N/mm2
and f2 = 59.95 N/mm2

Bending of Beam
Beam is a structural member which can carry transverse load. A simple supported beam is
supported at its ends. A cantilever beam is fixed at one end.

W W
l/2 l/2
d
b
d d

(a) Simply supported beam (b) Cantilever beam

Fig. 5.34

&GƀGEVKQPQH$GCO Deflection of beam at its centre due to load placed as shown in Fig. 5.34.
Wl 3
d = for simply supported beam
48YI
Wl 3
and d = for cantilever beam
3YI
d
where I is called geometric moment of area.
(i) For rectangular cross section
b
bd 3
I = Fig. 5.35
12
(ii) For circular cross section
p r4 r
I =
4

TORSION Fig. 5.36

Torsion is a special case of shearing. In this case, instead of a shearing force, a torque is
applied to twist the specimen.

Torsion of Cylinder
If the upper end of a cylinder is clamped and a torque is applied at the lower end the cylinder
gets twisted by angle q.
DĞĐŚĂŶŝĐĂůWƌŽƉĞƌƟĞƐŽĨDĂƩĞƌപ843

Simultaneously, shearing strain f is produced in the cylinder.


(i) The angle of twist q is directly proportional to the distance
from the fixed end of the cylinder. r P
Q
At fixed end q = 0° and at free end q = maximum.
(ii) The value of angle of shear q is directly proportional to the l
radius of the cylindrical shell. f
At the axis of cylinder f = 0 and at the outermost shell f =
O
maximum. q
A B
(iii) Relation between angle of twist (q) and angle of shear (f)
Fig. 5.37
rq
AB = rq = fl \ f =
l
(iv) Twisting couple per unit twist or torsional rigidity or torque required to produce unit
twist is denoted by C
phr 4
C = q as
2l
phr 4
q =1 so C = \ C μ r 4 μ A2
2l
(v) Work done in twisting the cylinder through an angle q is

1 phr 4q 2
W= Cq 2 = .
2 4l

TWISTING OF A SHAFT IS SAME AS TWISTING COUPLE ON CYLINDER


Let us consider a shaft of length l and radius r, whose one end is
rigidly clamped and torque t is applied at the free end. Because t
of this the free end is twisted by an angle q.
r
From Fig. 5.38, arc f s q

s = rq = lf
l
where q Æ angle of twist and
f Æ angle of shear Fig. 5.38
t phr 4
Torsional rigidity of shaft =
q 2l
where h Æ modulus of rigidity.

WORKED PROBLEMS
Problem 5.18 A uniform elastic plank moves over a smooth F0 M
horizontal plane due to a constant force F0 distributed m
uniformly over the end face. The surface of the end face is dx
x
equal to A and Young’s modulus of the material is Y. Find
Fig. 5.39
the compressive strain of the plank in the direction of the
acting force.
844പWŚLJƐŝĐƐĨŽƌ//dͲ:͗DĞĐŚĂŶŝĐƐ//

Solution The force at any section is due to the inertia behind the section. The stress
therefore increases from zero to maximum at the end where force is applied.
Consider a small element of length dx at a distance x from the free end. The force
Fx = ma Fx Fx
ÊM ˆ F Fx
= Á x˜ ¥ 0 = 0 dx
Ë L ¯ M L Fig. 5.40
Elongation of the element
Ê F0 x ˆ
dx
Fx (dx ) ÁË L ˜¯
dl = =
AY AY
Total elongation
L
F
Dl = Ú ALY
0
xdx
0

F0
= L2
2 ALY
F0 L
or Dl =
2 AY

Problem 5.19 A slightly conical wire of length l and radii r1 and r2 is stretched by two forces
applied parallel to the length in opposite directions and normal to end forces. If Y denotes
the Young’s modulus, find the elongation of the wire.
Solution

r2
r1 r

F F
r
F F

x
dx
l dx
(a) (b)

Fig. 5.41

Consider an element of length dx at distance x as shown in Fig. 5.41. The radius of the
section
Êr -r ˆ
rx = r1 + Á 2 1 ˜ x
Ë l ¯
DĞĐŚĂŶŝĐĂůWƌŽƉĞƌƟĞƐŽĨDĂƩĞƌപ845

Differentiate with respect to x


r2 - r1
drx = dx
l
drx l
= dx
r2 - r1
F ◊ dx
From equation dl =
p rx2Y
F ◊l dr
dl = ◊ 2x
(r2 - r1 )p Y rx
The total elongation of the rod
r2
F ◊l drx dr -1
Dl =
(r2 - r1 )p Y Ú rx2 Ú r2 = r
r1

r
F ◊l È 1˘2
= Í- ˙
(r2 - r1 )p Y Î rx ˚ r
1

F ◊l È - 1 - 1˘
= Í - ˙
(r2 - r1 )p Y Î r2 r1 ˚

F ◊l È1 1˘
= Í - ˙
(r2 - r1 )p Y Î r1 r2 ˚

F◊l È r2 - r1 ˘
= Í ˙
(r2 - r1 ) p Y Î r1 r2 ˚

F ◊l
= .
p Yr1 r2

Problem 5.20 A horizontally oriented copper rod of length l is rotated about a vertical
axis passing through its middle. Calculate the rotated frequency at which the rod ruptures.
Breaking or rupture strength of copper takes as s and density of copper r.
Solution

A F F + dF

x
dx

Fig. 5.42
846പWŚLJƐŝĐƐĨŽƌ//dͲ:͗DĞĐŚĂŶŝĐƐ//

The stresses are zero at the free end and maximum at the axis. Therefore, the rod will
rupture at the middle.
Let us consider an element of rod at a distance x from the axis, the mass of element
dm = rAdx
m = Density ¥ Volume
w
Applying Newton’s second law
w 2x
F – (F + dF) = (dm)an F F + dF
2
or – dF = (rAdx)w x x
where w is the rotation speed. dx

Fig. 5.43
\ F = – Ú dF = - Ú ( r Adx )w 2 x

x2
= – r Aw 2 +C
2
at x = l/2, F = 0

r Aw 2l 2
\ C =
8
Aw 2 Ê l 2 ˆ
Now, F =r Á - x2 ˜
2 Ë4 ¯

Aw 2l 2 F rw 2l 2
at x = 0, F = r and f = =
8 A 8

Rupture of rod will occur when f = s

rw 2l 2
\ =s
8
8s
fi w =
rl 2

1 8s
or n =
2p rl 2

Problem 5.21 A ring of radius R made of lead wire breaking strength s and density d, is
rotated about a stationary vertical axis passing through its centre and perpendicular to the
plane of the ring. Calculate the number of rotation at which the ring ruptures.
Solution Due to rotation, each part of the ring experiences an outward force (centrifugal
force). Because of this force, the ring will rupture.
DĞĐŚĂŶŝĐĂůWƌŽƉĞƌƟĞƐŽĨDĂƩĞƌപ847

dmw 2R
R q/2 q/2
T q/2 q/2 T

(a) (b)

Fig. 5.44

Let us consider a small part of the ring, which subtends angle q at the centre.
Mass of the element (dm) = d dv = d(ARq)
\ 2T sin q/2 = (dm)w2R
A
When q is small sin q/2 q/2
f
\ 2T ¥ q/2 = (dARq)w2R f

fi T = dAw2R2 Fig. 5.45

The stress at any section of the ring


T d Aw 2 R2
f = =
A A
2 2
= dw R
Rupture takes place when f =s
\ s = dw2R2
s
fi w =
d R2
1 s 1 s
and n = 2
=
2p dR 2p R d
Important Point:
Breaking stress = Breaking force/Areas of cross section
Breaking stress and strain are constant for a material.
Problem 5.22 Two beams are made of the same material and having equal length resting
at their ends and subjected to same load at their centre. One beam square in cross section
other circular cross section. If their cross-sectional areas are equal, then calculate the ratio
of deflection at their centres.
Wl 3
Solution We have d =
48YI r
d square a
I
\ = circular [p r 2 = a 2 ]
d circular I square
a
p r 4/4 3p r 4 3p r 4 3
= = = = (a) (b)
Ê a4 ˆ a 4 2 2
(p r ) p Fig. 5.46
Á 12 ˜
Ë ¯
848പWŚLJƐŝĐƐĨŽƌ//dͲ:͗DĞĐŚĂŶŝĐƐ//

Problem 5.23 Two cylinders A and B of the same


material have same length, their radii being in the
ratio 1:2 respectively. The two are joined end to end A B
as shown. One end of cylinder A is rigidly clamped
while the free end of cylinder B is twisted through an
angle q. Find the angle of twist of cylinder A.
Fig. 5.47
Solution Let q1 and q2 be the angles of twist produced
in cylinders A and B respectively.
Given, q 1 + q2 = q ...(i)
On being in series, the torque acting at their free ends are equal.
phr 4q
We have t =
2l
phr 4q1 ph(2r )4 q 2
\ =
2l 2l
fi q1 = 16q2 ...(ii)
From Eqs. (i) and (ii), we have
16
q1 = q
17
Problem 5.24 The length of a metal wire is l1 when the tension is T1 and l2 when the tension
is T2. Find the unstretched length of the wire.
Solution Let l is the unstretched length of the wire.
Using F = kx, we have
T1 = k (l1 – l) and T2 = k(l2 – l)
After solving, we get = (T2 l1 – T1l2)/(T2 – T1).

SURFACE TENSION
Cohesive and Adhesive Forces: The force of attraction between molecules of same substance
is called cohesive force or cohesion.
The force of attraction between molecules of different substances is called adhesive force
or adhesion.
These forces are of electromagnetic nature. They vary with eight power of distance
between the molecules.
These forces are responsible for surface tension.
Surface Tension: The property of liquid by which a
liquid tends to acquire minimum surface area, is called
F Imaginary
surface tension. line
Mathematically, surface tension of liquid is the force
acting on a unit length of an imaginary line drawn on F
free surface of the liquid. Thus, if F is the total force
acting on l length of the line, then surface tension Surface film
F Fig. 5.48
T=
l
DĞĐŚĂŶŝĐĂůWƌŽƉĞƌƟĞƐŽĨDĂƩĞƌപ849

EXAMPLES BASED ON SURFACE TENSION


(i) Needle supported on water surface
When a needle is placed gently on the water surface, it remains F F
floating on it. The floating needle causes a slight depression.
The surface tension force F acts tangentially to the curved W
surface as shown in Fig. 5.49. The vertical components of the
Fig. 5.49
forces balance the weight of the needle.
(ii) 'PFNGUUYGVVJTGCFQPCUQCRſNO
Take a circular wire frame and dip it into soap
solution, a thin soap film is formed on the frame.
If a wet endless thread loop is gently placed over
the film, it takes any irregular shape. But when the
film is pricked inside, the loop stretches outwards
and takes a circular shape. This is because for a
given length a circle has the maximum surface (a) (b)
area and so the soap film tries to get maximum
possible area due to surface tension. Fig. 5.50 A soap film in a circular
wire frame
(iii) Small mercury droplets are spherical
Small mercury droplets are spherical because surface
tension force tends to decrease their surface area to a
minimum value and a sphere has minimum surface
Small drop Large drop
area for a given volume.
Larger mercury drops become flat due to the large Fig. 5.51
gravitational force on them. Here the shapes of the
droplets are such that the sum of the gravitational potential energy and the surface
potential energy is minimum. To get this, centre of gravity of droplets moves down.

MOLECULAR THEORY OF SURFACE TENSION


The maximum distance up to which the force of attraction between two molecules is
appreciable is called molecular range (ª 10 – 9 m). A sphere with a molecule as centre and
radius equal to molecular range is called the sphere of influence. The liquid enclosed
between the free surface (PQ) of the liquid and an imaginary plane (RS) at a distance r (equal)
to molecular range) from the free surface of the liquid forms a liquid film.
To understand the concept of tension acting on the free surface of a liquid, let us consider
four liquid molecules like A, B, C and D. Their spheres of influence are shown in Fig. 5.52.
1. Molecule A is well within the liquid, so it is attracted equally D
in all directions. Hence, the net force on this molecule is C B
P Q
zero and it moves freely inside the liquid. S
R
2. Molecule B is little below the free surface of the liquid
and it is also attracted equally in all directions. Hence, the A
resultant force acting on it is also zero.
3. Molecule C is just below the upper surface of the liquid film
and the part of its sphere of influence is outside the free Fig. 5.52
850പWŚLJƐŝĐƐĨŽƌ//dͲ:͗DĞĐŚĂŶŝĐƐ//

liquid surface. So the number of molecules in the upper half (attracting the molecules
upward) is less than the number of molecules in the lower half (attracting the molecule
downward). Thus, the molecule C experiences a net downward force.
4. Molecule D is just on the free surface of the liquid. The upper half of the sphere of
influence has no liquid molecule. Hence, the molecule D experiences a maximum
downward force.
Thus, all molecules lying on surface film experience a net downward force. Therefore,
free surface of the liquid behaves like a stretched membrane.

Pressure Difference Across a Liquid Surface


Pressure difference due to curvature of liquid surface: Let us take three types of surfaces:
plane, concave and convex as shown in Fig. 5.53.
A plane surface is formed when a liquid is stationary in a container. At the free surface,
the surface molecules pull any surface molecule P with equal amount of force from all sides
in the surface. Then, the net surface tension force acting on the molecule P is zero. Hence,
the force per unit area at the point P is equal to the normal cohesion pressure.
P T F
P
T T
T F=0
T T
P
F
For a flat surface, the net For concave surface, the For concave surface, the net
surface tension force molecules experience a net surface tension force on
acting on the molecule is zero upward surface tension force the molecule is downward

Fig. 5.53

If the surface of a liquid is concave (water meniscus in a capillary tube), the resultant
surface tension force on the surface molecule P will be upwards. Hence, the cohesion
pressure inside the liquid decreases as the surface is pushed up by the additional force F.
In the case of liquid drops and soap bubbles, the surface is convex. We can see that the net
surface tension force F acting on the surface molecule P (due to the other surface molecules)
is vertically downward, or radially inward. This increases the downward cohesion force
by F. Hence, the cohesion pressure increases. Let us now find the increase or decrease of
cohesion pressure because of the curvature of liquid surface.
Important:
For finding pressure difference use this technique.

PA – PB = 0 2T 2T
PA - PB = PB - PA =
R R
T A T A
T A T
T B T
B
B
(a) (b) (c)

Fig. 5.54
DĞĐŚĂŶŝĐĂůWƌŽƉĞƌƟĞƐŽĨDĂƩĞƌപ851

SURFACE ENERGY AND SURFACE TENSION


The extra energy possessed by the molecules of the surface film in comparison to the energy
of the molecules inside the liquid is called surface energy.
Surface tension can be defined as the work done in increasing the area of the surface film
by unit amount. Thus, surface tension
Work done
T=
Increase in surface area
The SI unit of surface tension is J/m2.

WORK DONE IN INCREASING THE AREA OF THE SURFACE FILM


Consider a wire frame ABCD in which AB is movable. Dip the frame into the soap solution.
A thin film of soap solution is formed inside the frame. The surface tension force on the
movable wire AB, F = 2Tl. Here the factor 2 is taken because the soap film has two free
surfaces.
Work done in displacing the wire from AB to A¢B¢ D A A¢

W =F¥x T

= 2 Tlx = T(2lx)
l F
or W = TDA
where DA is the effective increase in area of the film.
C B B¢
Work Done in Breaking a Liquid Drop Soap x
film
Let a liquid drop of radius R breaks into n-identical drops.
Fig. 5.55
If r is the radius of each small drop, then
4 4
p R3 = n ¥ p r 3
3 3
R
fi r = 1/3
n
The increase in surface area
DA = [n ¥ 4pr2 – 4pR2]
Work done W = TDA = T[n ¥ 4pr2 – 4pR2]
Note:
1. If breaking of drop takes place in self process the energy of resulting drops will
decrease and so the temperature of the drops decrease.
2. If number of small drops coalesce to form a big drop, the internal energy of the
resulting drop will increase and so its temperature increases.

Work Done in Blowing a Soap Bubble


A soap bubble of radius R is to be blown. The effective increase in surface area DA = 2 ¥ 4pR2.
852പWŚLJƐŝĐƐĨŽƌ//dͲ:͗DĞĐŚĂŶŝĐƐ//

Work done, W = T DA
= T ¥ 2(4pR2)
= 8 pTR2

PRESSURE DIFFERENCE
Liquid Drop: Consider a liquid drop of radius R. Let Pi be the pressure inside the drop and
Po be the pressure outside the drop. Consider the equilibrium of the half of the drop. The
force due to pressure difference (Pi – Po) acts on the projected area in upward direction. The
force due to surface tension acts all over the perimeter in downward direction.

Po

Po Pi

Pi
(Pi – Po) ¥ pR2

T ¥ 2pR

Fig. 5.56 Fig. 5.57

The force due to pressure difference = (Pi – Po)pR2


The force due to surface tension = T ¥ 2pR
For the vertical equilibrium of the drop
(Pi – Po)pR2 = T ¥ 2pR
2T
fi (Pi – Po) =
R

Note: When a drop or bubble tries to contract the internal pressure is increased that also
prevents collapse of the drop in equilibrium. Inside pressure is greater than outside pressure.
This pressure difference is called excess pressure.
This excess pressure is due to hydrostatic pressure within the drop or gauge pressure of
air in case of a bubble.
Let Pi and Po denote the inside and outside pressures.
Excess pressure DP = Pi – Po
Method II: Consider a spherical liquid drop of radius R. Let T be the surface tension of the
liquid. Suppose P is an excess pressure inside the drop over that on outside, i.e., P = Pi – Po.
Let the radius of the drop increase from R to (R + dR) due to the excess pressure P.
DĞĐŚĂŶŝĐĂůWƌŽƉĞƌƟĞƐŽĨDĂƩĞƌപ853

Initial surface area = 4pR2


Final surface area = 4p (R + dR)2 dR

= 4p(R2 + 2R dR + dR2)
P
On being small dR2 can be neglected
\ Final surface area = 4pR2 + 8pRdR
Increase in surface area = (4pR2 + 8pRdR) – 4pR2 = 8pRdR
Work done in increasing the surface area = Surface tension ¥ Fig. 5.58
Increase in surface area
or W = T ¥ 8pRdR
But work done W = Pressure ¥ Change in volume
= P ¥ 4pR2dR
Hence, (P ¥ 4pR2)dR = T ¥ 8pRdR
2T
\ P = .
R
Soap Bubble: Consider a soap bubble of radius R. Let Pi be the pressure inside the bubble
and Po be the pressure outside the bubble. Consider the equilibrium of the half of the bubble.
The force due to pressure difference (Pi – Po) acts on the projected area in upward direction.
The force due to surface tension acts all over the perimeter and in downward direction. The
force due to pressure difference = (Pi – Po)pR2.
The force due to surface tension force = 2T ¥ 2pR
Here the factor 2 is taken because soap film has two free surfaces.

Po
Po
2
(Pi – Po) pR

Pi
R Pi

2T ¥ 2pR
T T

(a) (b) (c)

Fig. 5.59

Thus, for the vertical equilibrium of the bubble


(Pi – Po)pR2 = 2T ¥ 2pR
4T
fi P i – Po =
R
854പWŚLJƐŝĐƐĨŽƌ//dͲ:͗DĞĐŚĂŶŝĐƐ//

2T
Note: Pressure difference in a film having one free surface is Pi – Po = . For a film having
R
radii of curvatures R1 and R2 the pressure difference for one free surface.
Ê 1 1ˆ
Pi – P o = T Á + ˜
Ë R1 R2 ¯

O1 R1

R2

O2

Fig. 5.60

Note:
s Air bubble inside water: Let an air bubble of radius R be at a depth h below the free
surface of water. The pressure difference
Pa
2T
Pi – P o =
R h
Pi
2T 2T
fi + Po =
Pi = + ( r gh + Pa )
R R Fig. 5.61
s 7ORK DONE IN FORMING A DROP OR BUBBLE IN A LIQUID WILL BE
W = T ¥ 4pr2 [as D = 4pr2 – 0]
while for a bubble in air

Surface Surface

urfac urfac
S
S

e
e

Drop Bubble in a liquid Bubble in air


1-surface 1-surface 2-surface

Fig. 5.62

W = T ¥ 4pr2 ¥ 2 [as bubble in air has 2-surfaces]


This surface energy becomes evident when a film breaks scattering the liquid droplets
with considerable velocity. Here the potential energy of the film is transformed into
kinetic energy of the scattered particles.
DĞĐŚĂŶŝĐĂůWƌŽƉĞƌƟĞƐŽĨDĂƩĞƌപ855

Bubble in Air Bubble in Liquid

4T 2T
DP DP = DP DP =
R R

Bubble at depth h below the free surface of Cylindrical liquid surface


liquid of density r.

R
h 2T T
DP = + hr g DP =
DP R R

Liquid surface of unequal radii Liquid film of unequal radii

È1 1 ˘ È1 1 ˘
DP DP = T Í + ˙ DP DP = 2T Í + ˙
Î R1 R2 ˚ R
Î 1 R2˚

WORKED PROBLEMS
Problem 5.25 A light open rigid paper frame as shown in Fig. 5.63 floats on the surface
of water.

Fig. 5.63
F
What will happen to the frame if some soap solution is dropped inside
it? What force will act on the frame and in what direction will it act?
Solution If T1 and T2 are the surface tensions of the water and the soap
film, then the frame will be acted with a force
T1l
F = T 1l – T 2l
= (T1 – T2)l T2l
The frame will begin to move in the direction of the force F.
Fig. 5.64
856പWŚLJƐŝĐƐĨŽƌ//dͲ:͗DĞĐŚĂŶŝĐƐ//

Surface Tension Force on Different Objects


l
1. Wire or thin rod

F = 2Tl Fig. 5.65


r
2. Circular disc

F = 2prT
Fig. 5.66
3. Annular disc
ri = r
ro = R T
Fig. 5.67
F = 2prT + 2pRT
4. Ring
F = 2pRT + 2pRT T
= 4pRT Fig. 5.68
5. Rectangular plate (a ¥ b)

F = T(2a + 2b)
Fig. 5.69
6. Wire frame (a ¥ b)

F = T ¥ 2(2a + 2b)

Fig. 5.70
Shape of Meniscus: Shape of meniscus depends on the relative values of adhesive and
cohesive forces.
1. If Fc = 2Fa , the net force acts vertically downwards. The liquid meniscus is horizontal.
or plane. Example, pure water contained in silver capillary tube.

Fa Fc

45°

Fc = 2Fa

Fig. 5.71
DĞĐŚĂŶŝĐĂůWƌŽƉĞƌƟĞƐŽĨDĂƩĞƌപ857

2. If Fc < 2Fa , the net force is directed outside the liquid. The liquid meniscus is concave
upwards. This is possible in case of liquids which wet the walls of the capillary tube,
example, water in glass capillary tube.

Fa

Fc

Fig. 5.72

3. If Fc > 2Fa , the net force is directed inside the liquid. The meniscus is convex
upwards. This is possible in case of liquids which do not wet the walls of the capillary
tube, example, mercury in glass capillary tube.

Fa

Fig. 5.73

ANGLE OF CONTACT
It is defined as the angle between the tangent to the liquid surface at the point of contact to
the solid surface inside liquid.
(i) Angle of contact is the property of the materials in contact.
(ii) It decreases with the increase in temperature.
(iii) It decreases with the addition of soluble impurities, like soap, detergent, etc.

q q
q q > 90°
q < 90°

(a) (b) (c)

Fig. 5.74
858പWŚLJƐŝĐƐĨŽƌ//dͲ:͗DĞĐŚĂŶŝĐƐ//

s 0URE WATER IN SILVER TUBE


s 7ATER IN GLASS TUBE q = 8°)
s -ERCURY IN GLASS TUBE q = 140°)
s ,IQUID DOES NOT WET THE SOLID SURFACE
s ,IQUID WETS THE SOLID SURFACE
s ,IQUID DOES NOT WET SOLID SURFACE

CAPILLARY RISE
When one end of a narrow tube (capillary) dipped into a liquid, the liquid rises or falls in
the tube. This phenomenon is called capillarity.
Let us consider a tube of radius r whose one end is dipped into liquid of surface tension
T. Surface tension force acts all over the perimeter of the tube at the meniscus in upward
direction. Because of this force liquid will rise in the tube till the weight of the liquid in the
tube is equal to the surface tension force.

T R T T cos q ¥ 2pr
r
q q

h h

(pr 2h)rg
(a)
(b)

Fig. 5.75

Method I: By load balancing method

T cos q ¥ 2pr = pr2hrg


2T cos q
or h =
rr g
If R is the radius of the meniscus, then
r
R =
cos q
2T
\ h =
Ê r ˆ
ÁË cos q ˜¯ r g

2T
or h =
Rr g
DĞĐŚĂŶŝĐĂůWƌŽƉĞƌƟĞƐŽĨDĂƩĞƌപ859

Method II: By pressure balance method


In case of capillary rise the shape of meniscus is concave upward. The pressure just below
2T
the free surface of the liquid is less than the pressure at point A by an amount . So to
R
compensate this pressure difference, liquid will rise in the tube. From Fig. 5.76:
PA = Pa
2T
PB = PA -
R
A
Also, PD = PB + hrg
B
Ê 2T ˆ
or PD = Á PA - ˜ + hr g h
Ë R¯
Since PD = PC = PA = Pa A C D
B
2T
\ = hrg
R Fig. 5.76
2T
or h =
Rr g

ƒ’‹ŽŽƒ”›—„‡‘ˆ •—ˆϐ‹ ‹‡–‡‰–Š


2T
Theoretically, the rise of liquid in the tube is h = . If the length of the tube above the
Rr g
liquid is l, if less than h, then the liquid will rise to full length of the tube and the free surface
of the liquid will acquire larger radius of curvature in such a way, that the product hR = lR¢.
2T
We have h =
Rr g
R
2T q R¢
or hR = r g (constant)
q
h 
For the tube of length l < h, let the radius of curvature
be R¢, then hR = lR¢
hR
or R¢ =
l
Fig. 5.77
As l < h, \ R¢ > R.
Note:
1. The energy required to raise the liquid in the capillary tube is obtained from the surface
energy of the air glass surface. In case when liquid (mercury) gets depressed inside
it, below its level outside the tube, the glass-liquid surface decreases, whereas the air-
glass surface increases by an equal amount, resulting in a net increase in the surface
energy of the whole system. This energy is derived from the depression of the liquid
in the tube, i.e., by decreasing gravitational potential energy by equal amount.
860പWŚLJƐŝĐƐĨŽƌ//dͲ:͗DĞĐŚĂŶŝĐƐ//

2. In deriving the formula of capillary rise, we have ignored the volume of the liquid in
the shaded portion (see Fig. 5.78). If it is taken into account then:
Volume of shaded portion = Volume of cylinder of radius r and height r – volume of
the hemisphere
2 1 Ê 4 3 ˆ p r3
= (p r )r - Á p r ˜ = r
2Ë 3 ¯ 3 r
Thus, we write
Ê p r3 ˆ
T cos q ¥ 2pr = (p r 2 h)r g + Á ˜ rg h
Ë 3 ¯
2T cos q r
or h = -
rr g 3
Fig. 5.78
3. Capillary rise in tube of square cross section:
If a is the width of the tube, then
T cos q ¥ (4a) = (a2h)rg T cos q

h
4T cos q
\ h =
ar g

Apparent Angle of Contact Fig. 5.79


We know that
2T cos q
h = ...(i)
rr g

For the tube of insufficient length l < h, so we have

2T cos q ¢
l = ...(ii)
rr g

Here q ¢ is the apparent angle of contact.


From Eqs. (i) and (ii), we get
l
cos q¢ = cos q
h
Important: Since l < h, so the value of q ¢ will be greater than q.

Tube Inside Liquid


The rise of liquid in the capillary tube does not depend on the portion of the tube inside the
liquid. But total length of the liquid remains depends on the length of the tube.
(i) Suppose l length of the tube is inside the liquid, which is less than the rise of the liquid
in the tube. When the tube is taken out of the liquid the length of the liquid in the tube
will be (h + l).
DĞĐŚĂŶŝĐĂůWƌŽƉĞƌƟĞƐŽĨDĂƩĞƌപ861

h h

 

(a)
(b)

Fig. 5.80

(ii) If length of the tube inside the liquid is greater to h, the length of the T T
liquid in the tube will be (h + l) = 2h. Pa
It can be explained as: The pressures at points A and B are:
A
2T 2T
PA = Pa - and PB = Pa +
R R

Clearly, PB = PA + rgh¢
Ê 2T ˆ Ê 2T ˆ T T
or ÁË Pa + ˜¯ = ÁË Pa - ˜ + r gh ¢ B
R R¯
Pa
4T
or h¢ = = 2h
r gR Fig. 5.81

CONCEPT OF CAPILLARITY
When a capillary tube opens at both ends is dipped into a liquid, then it is observed that:
(i) if the liquid wets the glass (like water, alcohol and benzene), the liquid rises in the
tube above its level outside the tube. Narrower is the tube, greater is the rise; and
(ii) if the liquid does not wet the glass (like mercury), the liquid is depressed below its
level outside the tube. Narrower is the tube, greater is the depression.
The phenomenon of rise or depression of liquids in capillary tube is called capillarity.

Explanation:
(i) For liquids which wet the glass (rise of the liquid).
In this case, the surface is concave upward. The pressure at a point A (Fig. 5.82a) just
above the meniscus is equal to the atmospheric pressure P. If we consider the pressure

at point B just below the meniscus, it is ÊÁ P -


2T ˆ
Ë ˜ because pressure on convex side of

a curved surface is less than that on concave side by (2T/R). But the pressure at the
points outside the tube at the same level as that of B is P. Thus, the liquid is pushed
up in the tube to a height h (Fig. 5.82b) such that the total pressure at B, now, is P. In
862പWŚLJƐŝĐƐĨŽƌ//dͲ:͗DĞĐŚĂŶŝĐƐ//

Ê 2T ˆ h
P P P ÁË P - R ˜¯
A

P B B
P

Ê 2T ˆ
ÁË P - R ˜¯

(a) (b)

Fig. 5.82

other words, the liquid column of height h adds a pressure (2T/R) at B. The pressure
due to a liquid column of height h is hrg, where r is the density of the liquid and g
is acceleration due to gravity. Hence,
2T
= h rg
R
2T
or h = ...(1)
Rr g

(ii) For liquids, which do not wet the glass (depression of liquid)
Consider the liquid to be mercury. Since the meniscus of mercury in the tube will be
convex upwards, the excess of pressure (2T/R) will be directed inwards. Thus, if at
a point A just above the meniscus (Fig. 5.83) the pressure is atmospheric pressure P,
Ê 2T ˆ
then at point B¢ just below the meniscus will be Á P + ˜ . It should be remembered
Ë R¯
that the pressure at points outside the tube in the same level as that of B is atmospheric
pressure P. Therefore, the level of mercury will be depressed to a height h such that
the pressure at a point C outside the tube and at point B¢ (Fig. 5.83), both at the same
level, is same. Thus,
P
P A P

B
Ê 2T ˆ
ÁË P + R ˜¯ P h

C B¢

(a) Ê 2T ˆ (b)
ÁË P + R ˜¯

Fig. 5.83
DĞĐŚĂŶŝĐĂůWƌŽƉĞƌƟĞƐŽĨDĂƩĞƌപ863

Pressure at B¢ = Pressure at C
Ê 2T ˆ
ÁË P + ˜ = P + Pressure due to mercury column h

Ê 2T ˆ
or ÁË P + ˜ = P + hrg

2T
\ h = ...(2)
Rr g
r = density of mercury.

Formula
From Fig. 5.84 radius r of capillary tube is given by r
r = R cos q q
where q = angle of contact R R
r
\ R =
cos q q
From Eqs. (1) or (2)
2T cos q
h = ...(3)
rr g Fig. 5.84
Now we consider the following two cases:
(i) If capillary tube is of insufficient length l (i.e., l < h), then the liquid rises to a full height
l and spreads such that the meniscus occupies a radius R¢ such that
hR = lR¢ ...(4)
(ii) When the capillary is tilted from the vertical by an angle a, then the vertical height h
of the liquid column remains the same, of course, the length of the liquid in capillary
increases such that
h
cos a =
l
h
or l = ...(5)
cos a

Important Points:
1. For liquids where the adhesive forces prevail over cohesive forces capillary rise is
observed.
2. For liquids where the cohesive forces prevail over the adhesive forces capillary dip is
observed.
3. In capillary rise, the angle of contact is less than 90°.
4. The flow of ink through a nib is due to capillarity.
5. The walls get damped in rainy season due to the absorption of water by bricks through
capillary action.
6. The pores in blotting paper act as capillaries.
864പWŚLJƐŝĐƐĨŽƌ//dͲ:͗DĞĐŚĂŶŝĐƐ//

7. Due to capillary rise, the plants absorb sap from the ground.
8. In the kerosene lamp, kerosene is supplied through the wick by capillary rise.
9. The towel absorbs water due to capillary action.
10. When a candle burns, the molten wax rises up through the wick and there it burns.

Inclined Tube
If the tube is inclined with the vertical, the rise of liquid in
the tube will be same, but length of the liquid in the tube will
increase.
If h is the rise of liquid, then length of liquid in the tube l h a 
h
= .
cos a

Work Done by Surface Tension Force Fig. 5.85

In the process of rise of liquid in the tube, the liquid pressure at the
wall of the tube increases from zero to rgh. So average pressure,
rgh
Pav =
2 h
The change in volume of air
DV = pr2h
2T cos q
Also, h =
rr g Fig. 5.86
Work done in increasing the glass-liquid surface (decreasing glass-air surface)

W = Pav DV = ÊÁ
r gh ˆ
(p r 2 h)
Ë 2 ˜¯
2
p pr gr 2 Ê 2T cos q ˆ
= r gr 2 h 2 =
2 2 ÁË r r g ˜¯
2T 2p cos 2 q
=
rg

Capillary Tube of Varying Radius r2


(r1 – r2)

Suppose radius of the tube varies from r1 and r2 in its total length
r
l. The radius at the position of meniscus

Êr -r ˆ
r = r1 - Á 1 2 ˜ h h
Ë l ¯
r1
The rise of the liquid in the tube can be obtained by the formula
2T cos q
h= , by placing the value of r in terms of r1 and r2.
rr g Fig. 5.87
DĞĐŚĂŶŝĐĂůWƌŽƉĞƌƟĞƐŽĨDĂƩĞƌപ865

Force required to separate the plates having some


liquid between them
Po
When some liquid is put between the plates, the
pressure between the plates Pi becomes less than the
outside pressure Po. So an outward force is required r2
to separate them. Let d be the separation between
the plates and r1 and r2 are the radii of curvatures of r1
two sides of the surface film. The pressure difference
Po
Ê 1 1ˆ
Po – P i = T Á + ˜ Fig. 5.88
Ë r1 r2 ¯

d 1 1
Here, r1 = also r2 >> r1, so <<
2 r2 r1
T
\ Po – Pi =
r1
T 2T
= =
d/2 d
Force required F = (Po – Pi) ¥ Area of plate

2T 2TA
= ¥A=
d d

WORKED PROBLEMS
Problem 5.26 Two soap bubbles of radii r1 and r2 coalesce to form a single bubble under
isothermal condition. Find the radius of resulting bubble.
Solution As the process is isothermal, so by Boyle’s law, we have
P1V1 + P2V2 = PV
If r is the radius of the resulting bubble, then

Ê 4T ˆ 4 3 Ê 4T ˆ 4 3 Ê 4T ˆ 4 3
ÁË r ˜¯ ¥ 3 p r1 + ÁË r ˜¯ ¥ 3 p r2 = ÁË r ˜¯ ¥ 3 p r
1 2

or r12 + r22 = r2
Problem 5.27 Two soap bubbles come together to form a double bubble. Find the radius
of curvature at the contact point. (IIT Roorkee, 1989)
Solution Pressure inside smaller bubble is greater, so curvature on this side will be concave.
Suppose two soap bubbles of radii r1 and r2 come in contact and r is the radius of contact
point
Ê 4T ˆ Ê 4T ˆ
P1 = Á + Pa ˜ and P2 = Á + Pa ˜
Ë r1 ¯ Ë r2 ¯
866പWŚLJƐŝĐƐĨŽƌ//dͲ:͗DĞĐŚĂŶŝĐƐ//

For resulting curvature P1 – P2 = P as r 1 < r2


P1 > P2
P1 P2
r r1
Ê 4T ˆ Ê 4T ˆ 4T
or ÁË r + Pa ˜¯ - ÁË r + Pa ˜¯ = r r2
1 2

1 1 1 r -r
or - = fi= 2 1 Ans. Resulting curvature
r1 r2 r r1 r2
Fig. 5.89
Concept
Excess pressure is inversely proportional to the radius of bubble (or drop), i.e., pressure
inside a smaller bubble (or drop) is higher than inside a larger bubble (or drop). This is
why when two bubbles of different sizes are put in communication with each other, the air
will rush from smaller to larger bubble, so that the smaller will shrink while the larger will
expand till the smaller bubble reduces to droplet. (IIT-JEE, Conceptual)

Fig. 5.90

Problem 5.28 Under isothermal condition two soap bubbles of radii r1 and r2 coalesce of
form a single bubble of radius r. The external pressure is Po. Find the surface tension of the
soap in terms of the given parameters.
Solution As mass of the air is conserved,
\ n1 + n 2 = n (as PV = nRT)
P1V1 P2V2 PV
\ + =
RT1 RT2 RT
As temperature is constant,

P2
P1
+ =
r1 r
P2

Fig. 5.91
DĞĐŚĂŶŝĐĂůWƌŽƉĞƌƟĞƐŽĨDĂƩĞƌപ867

T1 = T2 = T
\ P1V1 + P2V2 = PV
Ê 4S ˆ Ê 4 ˆ Ê 4S ˆ Ê 4 ˆ
\ = Á P0 + ˜ Á p r13 ˜ + Á P0 + ˜ Á p r23 ˜
Ë r1 ¯ Ë 3 ¯ Ë r2 ¯ Ë 3 ¯

Ê 4S ˆ Ê 4 ˆ
= Á P0 + ˜ Á p r 3 ˜
Ë r ¯Ë3 ¯
Solving this, we get
P0 (r 3 - r13 - r23 )
S =
4(r12 + r22 - r 2 )

Note: To avoid confusion with the temperature surface tension here is represented by S.
Problem 5.29 In a capillary rise, find the heat developed taking all standard notations as
described in the foregoing section.
Solution As the liquid rises, positive work is done by surface tension in pulling the liquid
up by a distance h which is given as
WST = Fy h, where Fy = (T cos q)2pr
This gives WST = (2prT cos q)h
2T cos q
Substituting h=
r gr CM

h
4p T 2 cos 2 q
We have WST = ...(i)
rg y

The work done by gravity during the capillary rise is,


Wgr = – DU = mgyCM
h
where yCM = and m = (pr2 h)r
2
Fig. 5.92
p r 2 r gh 2
Then, DU =
2
2T cos q
where h =
r gr

2p T 2 cos 2 q
This gives DU = ...(ii)
rg
Applying the first law of thermodynamics, the heat dissipated Q can be given as
WST = Q + DU ...(iii)
Using Eqs. (i), (ii) and (iii), we have
2p T 2 cos 2 q
Q =
rg
868പWŚLJƐŝĐƐĨŽƌ//dͲ:͗DĞĐŚĂŶŝĐƐ//

Problem 5.30 A vertical U-tube contains a liquid of density r and surface tension T. If the
radii of the meniscus of liquid in the limbs of the U-tube are R1 and R2, find the difference
in the heights of liquid column in the limbs.
Solution Let the heights of liquid column P0
in the limbs be h1 and h2. Using the formula
2T
DP = for the meniscus in the limbs, we
R
P0
have the pressures at the points A and B h2
given as
h1 A C
2T
PA = P0 - ...(i)
R1
2T
and PB = P0 - ...(ii)
R2 Fig. 5.93
Using the formula DP = rgh, we have the pressures at A and C
PA (= PC) – PB = rg(h2 – h1) ...(iii)
Substituting PA from Eq. (i), from Eq. (ii) in Eq. (iii)
Ê 2T ˆ Ê 2T ˆ
We have ÁË P0 - R ˜¯ - ÁË P0 - R ˜¯ = rgDh
1 2
2T (R1 - R2 )
This gives Dh =
r gR1R2

Student Task
s 7HEN WE USE THE ABOVE U-tube as a manometer, find the pressure difference in the
limbs, if the difference in heights of the liquid columns in the limbs is h.
Problem 5.31 Two vertical parallel plates are
partially submerged in water. The distance between
the plates is d and their width is l. Assume that the
water between the plates does not reach the upper
edges of the mutual attraction.
P0 P0
Solution The plates seem to attract due to pressure A
difference between an inside point A and a point h
outside the plates.
F = (Po – Paverage) ¥ lh ...(i) B

PA + PB PA + PA + r gh d
Paverage = =
2 2
Fig. 5.94
rgh
= PA +
2
DĞĐŚĂŶŝĐĂůWƌŽƉĞƌƟĞƐŽĨDĂƩĞƌപ869

Putting this value in Eq. (i),


Ê r gh ˆ
F = Á P0 - PA - ˜ lh
Ë 2 ¯

Ê 2T r gh ˆ
F = Á - ˜ lh ...(ii)
Ë d 2 ¯
2T
where is pressure due to surface tension.
d
In equilibrium the upward force on the liquid due to surface tension and the weight of
liquid column is balanced.
2T
2lT = rglhd or h = ...(iii)
r gd
On substituting h in Eq. (ii),
È 2T 1 Ê 2T ˆ ˘ 2Tl
F = Í - rÁ g ˜˙ ¥
ÍÎ d 2 Ë r gd ¯ ˙˚ rdg

2T 2l
F =
r gd 2

Problem 5.32 A bubble having surface tension T and radius R is formed on the ring of
radius b(b << R). Air is blown inside the tube with velocity v as shown. The air molecule
collides perpendicularly with the wall of the bubble and stops. Calculate the radius at which
the bubble separates from the ring. (IIT-JEE, 2003)

R
b
v A
v

Fig. 5.95

T sin q
Solution The bubble will separate from the tube when
thrust force exerted by the air is equal to the force due to R
excess pressure.
T b q
2pb ¥ 2T sin q = rAv2
b T
fi 4pbT ¥ = rpb2v2
R
4T
fi R = T sin q
rv 2
Fig. 5.96
870പWŚLJƐŝĐƐĨŽƌ//dͲ:͗DĞĐŚĂŶŝĐƐ//

Problem 5.33 A capillary tube of radius 0.50 mm is dipped vertically in a pot of water.
Find the difference between the pressure of the water in the tube 5.0 cm below the surface
and atmospheric pressure. Surface tension of water = 0.075 N/m. (VITEEE, 2011)
Solution PA = Pa
2T
PB = Pa -
r A
and PC = PB + hrg B
h
Ê 2T ˆ
= Á Pa - ˜ + hr g C
Ë r ¯
2T
\ PC – Pa = h r g -
r
2 ¥ 0.075
= 0.05 ¥ 103 ¥ 9.8 – Fig. 5.97
0.5 ¥ 10 - 3
= 190 N/m2
Problem 5.34 A wire forming a loop is dipped into a soap solution and taken out so that
a film of soap solution is formed. A loop of l long thread is gently put on the film and the
film is pricked with a needle inside the loop. The thread loop takes the shape of a circle.
Find the tension in the thread. Surface tension of soap solution is T.
Solution If r the radius of the loop formed, then
l = 2pr
l
fi r =
2p T
Surface tension force on the half loop
= T ¥ Projected length of the loop
= T ¥ 2r T
Now consider the equilibrium of the half loop, we have
2F = T ¥ 2r
r
fi F = Tr F F
Tl Fig. 5.98
or F =
2p

Problem 5.35 If a number of little droplets of water of surface tension T, all of the same
radius r combine to form a single drop of radius R and the energy released is converted into
kinetic energy. Find the velocity acquired by the bigger drop.
Solution By conservation of volume
4 4
p R3 = n p r 3
3 3
DĞĐŚĂŶŝĐĂůWƌŽƉĞƌƟĞƐŽĨDĂƩĞƌപ871

R3
or n =
r3
Mass of the bigger drop, m = Volume ¥ Density
4
= p R3 ¥ 1 [Density = 1 g/cm3]
3
4
= p R3
3
Energy released in the process = Surface tension ¥ Decrease in surface area
= T ¥ 4p (nr2 – R2)
È R3 ˘
= 4pT Í 3 r 2 - R2 ˙
Îr ˚
ÈR ˘
= 4pTR2 Í - 1˙
Îr ˚

ÈR - r ˘
= 4pTR2 Í
Î r ˙˚
Increase in KE = Decrease in energy of the system
1 ÈR - r ˘
or mv 2 = 4pTR2 Í
2 Î r ˙˚

1Ê 4 3ˆ 2 2 ÈR - r ˘
or Á p R ˜¯ v = 4pTR Í
2Ë 3 Î r ˙˚

6T (R - r )
\ v =
Rr
Here v will be in cm/s.
Problem 5.36 A glass plate of length 10 cm, breadth 4 cm and thickness 0.4 cm, weight 20
g in air is held vertically with long side horizontal and half the plate immersed in water.
What will be its apparent weight? Surface tension of water = 70 dyne/cm.
Solution The forces acting on the plate are:
(i) Weight of the plate vertically downwards,
W = mg = 20g-f
(ii) Buoyant force due to liquid, Fb = Vrlg

Ê lbt ˆ
= Á ˜ rl g
Ë 2¯
872പWŚLJƐŝĐƐĨŽƌ//dͲ:͗DĞĐŚĂŶŝĐƐ//

Ê 10 ¥ 4 ¥ 0.4 ˆ

Ë ˜¯ ¥ 1g-f
2
= 8g-f
(iii) Force due to surface tension, vertically downwards
F = T ¥ Perimeter of plate in contact with water
70
= ¥ 2(10 + 0.4)g -f
980
= 1.5g-f
Apparent weight of the plate = (W + F) – Fb
= (20 + 1.5) – 8
= 13.5g-f

Problem 5.37 A glass U-tube is such that the diameter of one limb is 3.0 mm and that of
the other is 6.00 mm. The tube is inverted vertically with the open ends below the surface
of water in a beaker. What is the difference between the height to which water rises in the
two limbs? Surface tension of water is 0.07 N/m. Assume that the angle of contact between
water and glass is 0°.
Solution Let PA and PB are the pressures at points A and B respectively. The pressure at
point C,
2T
PC = PA -
R1 A
r1
where R1 = = r1 C h B R
cos 0∞
D
2T
The pressure at point D, PD = PB - Y
R2
r2
where R2 = = r2 Fig. 5.99
cos 0∞
If h is the difference between heights of rise in two limbs, then
PD – PC = hrg
Ê 2T ˆ Ê 2T ˆ
or ÁË PB - R ˜¯ - ÁË PA - R ˜¯ = hr g
2 1
As PA = PB and R1 = r1 = 1.5 mm, R2 = r2 = 3.0 mm, so
Ê 1 1ˆ
2T Á - ˜ = hrg
Ë r1 r2 ¯

Ê 1 1 ˆ
0.2 ¥ 0.07 Á - = h ¥ 1000 ¥ 9.8
Ë 1.5 ¥ 10 - 3 3 ¥ 10 - 3 ˜¯
After solving, we get
h = 4.76 ¥ 10 – 3 m
DĞĐŚĂŶŝĐĂůWƌŽƉĞƌƟĞƐŽĨDĂƩĞƌപ873

The Technique in Finding


T T
Pressure difference A
B
2T
PA – PB = Fig. 5.100
R

Problem 5.38 Two rods of different metals, having the same area of cross section A, are
placed end to end between two massive walls as shown in Fig. 5.101. The first rod has a
length l1, coefficient of linear expansion a1 and Young’s modulus Y1. The corresponding
quantities for the second rod are l2, a2 and Y2. The temperature of both the rods is now
raised by T degrees. (a) Find the force with which the rods act on each other at the higher
temperature in terms of the given quantities. (b) Also find the lengths of the rods at the
higher temperature. Assume that there is no change in the cross-sectional area of the rods
and the rods do not bend. There is no deformation of walls.
Solution
(a) Due to heating the increase in length of the composite rod will be,
(DL)I = (L1a1 + L2a2)T [as DL = Laq]
and due to compressive force F from the walls due to elasticity, decrease in length will
be
ÈL L ˘ F È FL ˘
(DL)D = Í 1 + 2 ˙ ÍÎas DL = AY ˙˚
Î Y1 Y2 ˚ A
As the length of the composite rod remains unchanged, the increase in length due to
heating must be equal to the decrease in length due to compression, i.e.,
F È L1 L2 ˘
Í + ˙ = [L1a1 + L2a2]T
A Î Y1 Y2 ˚
A(L1a 1 + L2a 2 )T
or F = ...(i)
[(L1/Y1 ) + (L2/Y2 )]

(b) As initially the length of one rod is L1 and due to


heating it increases by (DL1)H = (a1L1T) while due to L1 L2
compression it decreases by a1 Y1 a2 Y2

(DL1)C = (FL1/AY1)
So, its final length Fig. 5.101

L¢1 = L1 + (DL1)H – (DL1)C = L1[1 + a1T – (F/AY1)]


Similarly, for the other rod,
L¢2 = L2 + (DL2)H – (DL2)C = L2 [1 + a2T – (F/AY2)]
where F is given by Eq. (i).
Problem 5.39 A vessel, whose bottom has round holes with diameter of 1 mm is filled with
water. Assuming that surface tension acts only at holes, find the maximum height to which
water can be filled in the vessel without leakage. Given that surface tension of water is 75
¥ 10– 3 N/m and g = 10 m/s2.
874പWŚLJƐŝĐƐĨŽƌ//dͲ:͗DĞĐŚĂŶŝĐƐ//

Solution As shown in Fig. 5.102 here the vertical force due to sur- T T
face tension at the hole T cos q ¥ L = T cos q ¥ 2pr will balance the q q
weight mg, i.e., pr2 hrg, i.e.,
T cos q 2pr = pr2 hrg
or h = (2T cos q/rrg)
This h will be maximum when cos q = maximum = 1
2 ¥ 75 ¥ 10 - 3
So, (h)max = Fig. 5.102
10 3 ¥ 5 ¥ 10 - 4 ¥ 10
= 0.03 m = 3 cm
Note: Historically, this method was used by Quincke to determine the surface tension of
liquids and is called ‘drop weight method’.
Problem 5.40 A ring is cut from a platinum tube of 8.5 F
cm internal diameter. It is supported horizontally from
a pan of a balance so that it comes in contact with water F
in a glass vessel. What is the surface tension of water of
an extra 3.97 g weight is required to pull it away from
water? (g = 980 cm/s2) T T

Solution The ring is in contact with water along its Cross


Section
inner and outer circumference; so when pulled out the
total force on it due to surface tension will be
F = T(2pr1 + 2pr2)
mg
So, T = [as F = mg]
2p(r1 + r2 )
3.97 ¥ 980 Fig. 5.103
i.e., T = = 72.13 dyne/cm
3.14 ¥ (8.5 + 8.7 )
2a
Problem 5.41 A container of width 2a is filled with a liquid. A thin wire
of weight per unit length l is gently placed over the liquid surface in the y
middle of the surface as shown in Fig. 5.104. As a result the liquid surface
is depressed by a distance y (y << a). Determine the surface tension of the
liquid. (IIT-JEE, 2004)
Solution Let l be the length of the wire. As l is the mass per unit length
of the wire, so weight of the wire Fig. 5.104

W = llg
a a
If T is the surface tension, then surface tension force = Tl.
The weight of the wire is balanced by the vertical component of the q q
T T
surface tension force Tl, so
2Tl cos q = W W

W l lg lg Fig. 5.105
\ T = = =
2l cos q 2l cos q 2 cos q
DĞĐŚĂŶŝĐĂůWƌŽƉĞƌƟĞƐŽĨDĂƩĞƌപ875

From Fig. 5.105


y y
cos q = = (As y << a)
a +y2 2 a
lg l ga
\ T = =
2( y/a) 2 y

Problem 5.42 A capillary tube of radius r and height h is connected to


a broad tube as shown in Fig. 5.106. The broad tube is gradually filled
with drops of water falling at equal intervals. Plot the changes in the
levels of water in both tubes with time and changes in the difference
between these levels. Calculate the maximum water level in the broader h1
tube and the maximum difference in the levels. The surface tension of
water is T.
Solution From the moment the filling begins to the moment of time Fig. 5.106
A the water level will uniformly rise in the capillary tube (curve I)
and remain at the same level in the broad tube (curve II). The difference in the levels will
2T
constantly increase. At the moment of time tA the difference in the levels will reach h0 = .
r gr
h (h1 – h2)

(h0 + h1) II

h1 I
h0

h0
O tA tB tC tD t

O tA tB tC tD –h0
t
(a) (b)

Fig. 5.107

From this moment up to the moment of time tB the levels in the capillary and broader tube
will rise with the same velocities while the difference in the levels will remain constant and
equal to h0. At the moment of time tB the water level in the capillary tube will reach the end
of the capillary and will stop at a height h1. From the moment tB to the moment tD the water
level will continuously rise in the broader tube. The water level in the capillary will remain
constant but the meniscus will change its shape from concave to convex of same radius r at
the moment tD. The difference in the levels in the section BC will decrease to zero and in the
section CD it will change its sign and will increase to h0. At the moment tD the water will
begin to flow out of the capillary tube and from this moment onwards all the levels will be
constant. The maximum height to which the water rises in the broader tube is h0 + h1. The
maximum difference in the levels is h0.
876പWŚLJƐŝĐƐĨŽƌ//dͲ:͗DĞĐŚĂŶŝĐƐ//

Problem 5.43 Two spherical soap bubbles coalesce. If V is the consequence change in
volume of the contained air and S the change in total surface area, show that
3PV + 4ST = 0,
where T is the surface tension of soap bubble and p is atmospheric pressure.
Solution Suppose a and b are the radii of the two soap bubbles and Pa, Pb are the pressure
inside them. Let c be the radius and Pc the pressure inside the resulting bubble. Then
4T 4T 4T
Pa = P +
, Pb = P + and Pc = P +
a b c
If Va, Vb and Vc are the volumes of two soap bubbles and that of the resulting bubble,
then by Boyle’s law, we have
PaVa + PbVb = PcVc

Ê 4T ˆ 4 3 Ê 4T ˆ 4 3 Ê 4T ˆ 4 3
or ÁË P + ˜¯ ¥ p a + ÁË P + ˜¯ ¥ p b = ÁË P + ˜ ¥ pc
a 3 b 3 c ¯ 3
Ê4 4 4 ˆ 4T
or Á p a 3 + p b 3 - p c 3 ˜ + ( 4p a 2 + 4p b 2 - 4p c 2 ) = 0
Ë3 3 3 ¯ 3
Ê 4 3 4 3 4 3ˆ
But ÁË p a + p b - p c ˜¯ = V (change in volume)
3 3 3
and (4pa2 + 4pb2 – 4pc2) = S (change in surface)
On substituting these values in the above equation, we get
4T
PV + S =0
3
or 3PV + 4TS = 0
Problem 5.44 Soapy water drips from a capillary. When the drop
breaks away, the diameter of its neck is D. The mass of the drop is Capillary
F
m. Find the surface tension of soapy water.
Solution When the drop breaks away from the capillary, weight of
the drop = force of surface tension
Soapy water
or mg = pD ¥ T
mg
mg
or T =
pD Fig. 5.108

Problem 5.45 A mercury drop shaped as a round table of radius R and thickness h is
located between two horizontal glass plates. Assuming that h << R, find the mass m of a
weight which has to be placed on the copper plate to diminish the distance between the
plates n-times. The contact angle equals q. Calculate m if R = 2.0 cm, h = 0.38 mm, n = 2.0
and q = 135°.
Solution We know that pressure inside a film is greater than the outside pressure by an
Ê 1 1ˆ
amount T Á + ˜ .
Ë r1 r2 ¯
DĞĐŚĂŶŝĐĂůWƌŽƉĞƌƟĞƐŽĨDĂƩĞƌപ877

h
If q is the angle of contact then h = 2r1 cos q or r1 = . Since the table is between the
plates, so r2 = R. Thus, pressure difference 2 cos q

Ê 1 1ˆ È 1 1˘
P = TÁ + ˜ = T Í + ˙
Ë r1 r2 ¯ Î h/2 cos q R ˚

1 1 2T cos q
As h is small in comparison to R, so << , \ P =
R h h
The total force exerted by mercury drop on the upper glass plate is nearly
F = P ¥ Projected area of the drop
q
Ê 2T cos q ˆ 2 180° – q
= Á
Ë ˜¯ ¥ p R
h h
R
2p R2T cos q
= ...(i)
h Fig. 5.109

Let R¢ become the new radius of curvature when the distance between the plates is
decreased by n-times. Assuming mercury to be incompressible, we have

Ê hˆ
pR2h = p R¢ 2 Á ˜
Ë n¯
fi R¢ = nR
The force exerted by the mercury drop now becomes

2T (p nR)2 T cos q
F¢ =
( h/n)
= n 2F ...(ii)
If mg is the weight placed on the upper plate then
F¢ = F + mg
F ¢ - F F(n2 - 1) 2p R2T cos q 2
\ m = = = (n - 1)
g g gh

Problem 5.46 What is the pressure inside a drop of mercury of radius 3.00 mm at room
temperature?
Surface tension of mercury at that temperature (20°C) is 4.65 ¥ 10–1 N/m. The atmospheric
pressure is 1.01 ¥ 105 Pa. Also give the excess pressure inside the drop.
Solution If Pi and Po are the inside and outside pressures of the drop, then
2T
Pi – P o =
R
2T
\ Pi = Po +
R
878പWŚLJƐŝĐƐĨŽƌ//dͲ:͗DĞĐŚĂŶŝĐƐ//

2 ¥ 4.65 ¥ 10 - 1
= 1.01 ¥ 10 5 +
3.00 ¥ 10 - 3
= 1.013 ¥ 10 Pa
5

Problem 5.47 A water drop falls in air with a uniform velocity. Find the difference between
the curvature radii of the drop’s surface at the upper and lower points of the drop separated
by the distance h = 2.3 mm.
Solution Suppose R1 and R2 be the radii of curvatures at the upper point and lower point
of the drop respectively. The pressure inside the drop at the upper end,
2T P0
PA = Po +
R1 A
and pressure at the lower end R1

2T h = 2.3 mm
PB = Po +
R2 R2
B
where Po is the atmospheric pressure.
As the drop is falling with uniform velocity, so P0

PB = PA + rgh Fig. 5.110

2T 2T
or Po + = Po + + r gh
R2 R1
Ê 1 1ˆ
or 2T Á - ˜ = rgh
Ë R2 R1 ¯
(R1 - R2 )
or 2T = rgh
R1R2
It can be assumed that R1 – R2 and R1 + R2 = h
So, R1 = R2 = h/2. Therefore, we get
2T(R1 – R2) = (rgh)R1R2
Ê h hˆ
= rgh ¥ Á ¥ ˜
Ë 2 2¯
rgh 3
R1 – R2 =
8T
At point A the technique is
2T
PB – PA =
R
A
T B T

Fig. 5.111
DĞĐŚĂŶŝĐĂůWƌŽƉĞƌƟĞƐŽĨDĂƩĞƌപ879

At point B the technique is


2T
PA – PB =
R

T T
A
B

Fig. 5.112

Problem 5.48 Two blocks of masses 1 kg and 2 kg are connected by


a metal wire going over a smooth pulley as shown in Fig. 5.113. The
breaking stress of the metal is 2 ¥ 109 N/m2. If the wire is not to break,
its minimum radius should be (IIT Roorkee, 1986)
(a) 4.6 ¥ 10– 5 m (b) 6.4 ¥ 10– 5 m T
(c) 4.6 ¥ 10– 6 m (d) 6.4 ¥ 10– 6 m
T
Solution The equations of motion of two blocks are 1 kg

T – 10 = 1 ¥ a
20 – T = 2 ¥ a m1g 2 kg
Solving we get, T = (40/3) newton
( 40/3)
\ Stress = m2g
p r2
Fig. 5.113
(∵ Stress = Tension/area of cross section)
( 40/3)
Now, 2 ¥ 109 =
p r2
r = 4.6 ¥ 10– 5 metre

VISCOSITY A
v + dv
Viscosity is the property of a fluid by virtue of F
which an internal force of friction comes into v
y + dy F
play in a moving fluid and which opposes the
relative motion between the adjacent layers. y
This opposing force is called viscous force or
viscous drag. Viscous force also acts between the Velocity profile
solid surface and liquid layers due to relative Fig. 5.114
motion between them. Viscosity is primarily due
to cohesion and molecular momentum exchange between fluid layers, and as flow occurs,
these defects appear as shearing stresses between the moving layers of the fluid.
Consider a liquid moving over a stationary horizontal surface. The liquid can be assumed
moving in the form of layers, one layer moves over the other. The layer in contact with the
surface is at rest and the velocity of every other layer increases upward. The velocity profile
will be parabolic (see Fig. 5.114).
880പWŚLJƐŝĐƐĨŽƌ//dͲ:͗DĞĐŚĂŶŝĐƐ//

Newton’s Law of Viscosity


Consider two liquid layers at distances y and y + dy from the stationary surface and moving
dv
with velocity v + dv respectively. The change in velocity with the height of the liquid is
and is called velocity gradient. dy

According to Newton’s viscous force F acting tangentially between two layers is


(i) proportional to the area A of the layer in contact
F μA
(ii) proportional to the velocity gradient between the layers
Ê dv ˆ
F μ Á ˜
Ë dy ¯
Ê dv ˆ
\ F = - hA Á ˜
Ë dy ¯
where h is the coefficient of viscosity of the liquid. Negative sign shows that the
viscous force acts opposite to the motion of the liquid.

‹–•‘ˆ‘‡ˆϐ‹ ‹‡–‘ˆ‹• ‘•‹–›


(i) The CGS unit of h is dyne-s/cm2 and is called poise.
(ii) The SI unit of h is N-s/m2 = Pa – s and 1 N-s/m2 = 10 poise.
Consider a liquid moving over a stationary horizontal surface. The liquid can be assumed
moving in the form of layers, one layer moves over the other. The layer in contact with the
surface is at rest and the velocity of every other layer increases upwards. The velocity profile
will be parabolic (see Fig. 5.115).

A
v + dv
F

F v
y + dy
y

Velocity profile

Fig. 5.115

Newton’s Law of Viscosity


Consider two liquid layers at distances y and y + dy from the stationary surface and moving
with velocity v and v + dv respectively. The change in velocity with the height of the liquid is
dv
and is called velocity gradient. According to Newton’s viscous force F acting tangentially
dy
between two layers is
DĞĐŚĂŶŝĐĂůWƌŽƉĞƌƟĞƐŽĨDĂƩĞƌപ881

(i) proportional to the area A of the layer in contact


FμA
(ii) proportional to the velocity gradient between the layers
Ê dv ˆ
F μ Á ˜
Ë dy ¯
Ê dv ˆ
\ F = - hA Á ˜
Ë dy ¯
where r1 is the coefficient of viscosity of liquid. Negative sign shows that the viscous
force acts opposite to the motion of the liquid.

‹–•‘ˆ‘‡ˆϐ‹ ‹‡–‘ˆ‹• ‘•‹–›


(i) The CGS unit of h is dyne-s/cm2 and is called poise.
(ii) The SI unit of h is N-s/m2 = Pa – s and 1 N-s/m2 = 10 poise.

WORKED PROBLEMS
Problem 5.49 A metal plate of area 0.10 m2 is connected to a 0.01 kg mass via a string that
passes over an ideal pulley (considered massless and frictionless), as shown in Fig. 5.116.
A liquid with a film thickness of 0.3 mm is placed between the plate and the table. When
released the plate moves to the right with a constant speed of 0.085 m/s. Find the coefficient
of viscosity of the liquid.
T
Thin film

T
m = 0.01 kg

mg

Fig. 5.116

Solution As the metal plate moves with constant velocity, so


mg – T = 0 fi T = mg
If F is the viscous force on the plate, then
F=T
= mg = 0.01 ¥ 9.8 = 9.8 ¥ 10– 2 N
882പWŚLJƐŝĐƐĨŽƌ//dͲ:͗DĞĐŚĂŶŝĐƐ//

By Newton’s law of viscosity


dv
F = hA
dy
dv 0.085
where = = 28.3/s
dy 0.3 ¥ 10 - 3
\ 9.8 ¥ 10– 2 = h ¥ 0.10 ¥ 28.3
or h = 3.4 ¥ 10– 3 N-s/m2

FACTOR AFFECTING VISCOSITY


Temperature
(i) With the increase in the temperature of the liquid, cohesion between the liquid
molecules decreases and hence viscosity of the liquid decreases.
Stotle empirical formula for the variation of the viscosity of a liquid is given by
h0
ht =
1 + at + bt2
where h0 and ht are the viscosities at 0°C and t°C respectively, and a and b are
constants.
(ii) Viscosity of the gases is due to the collisions between the molecules. With increase
in temperature, collisions between the molecules of the gases increases and hence
viscosity increases, h μ T .

Pressure
(i) Viscosity of liquids increases with increase in pressure. The viscosity of water decreases
with increase in pressure.
(ii) The viscosity of the gases does not depend on the pressure.

VISCOSITY VERSUS FRICTION


Viscous Force Friction
1. Viscous force is directly proportional to 1. Friction does not depend on the area of
the area of layer. contact.
2. It is directly proportional to the relative 2. It is almost independent of the velocity of the
velocity between the layers. object.
3. It is independent on the normal reaction 3. It is directly proportional to the normal
between the layers. reaction on the body.

POISEUILLE’S EQUATION
Consider a viscous liquid flowing in a horizontal pipe with constant velocity. Assuming a
cylinder of liquid of radius r and length l. It has outer surface 2prl.
DĞĐŚĂŶŝĐĂůWƌŽƉĞƌƟĞƐŽĨDĂƩĞƌപ883

The viscous drag on the cylinder is


dv
Fv = -h(2p rl)
dr
Since there is no acceleration, so this force is balanced by the force on the cylinder due
to pressure difference. The pressure difference at input and output is Pi – Po = DP and the
force it generates is DP ¥ p r2.
dv
\ -h(2p rl) = DP ¥ p r2
dr v
Pi P0
DP
fi dv = - rdr 
2hl
0
DP
R Fig. 5.117
or Ú dv = - 2hl Úr
rdr
v

DP 2 2
or v = (R - r )
4hl

4CVGQHƀQY Rate of flow in small element of cylinder,


dQ = vdA
= v ¥ 2p rdr
R dr
\ Q = Ú v ¥ 2p rdr r
0
v
R
DP Fig. 5.118
Ú 4hl (R
2
= - r 2 ) ¥ 2p rdr
0

pDPR 4
or Q =
8hl
The above equation can be written as
DP Pi - Po
Q = =
RF RF
where RF is the resistance of the pipe and is equal to,
8hl
RF = .
p R4

WORKED PROBLEMS
Problem 5.50 A liquid is flowing in a horizontal pipe of length l under pressure difference
P1 – P2. Calculate the pressure at a distance x from one end.
884പWŚLJƐŝĐƐĨŽƌ//dͲ:͗DĞĐŚĂŶŝĐƐ//

Solution Suppose P is the pressure at a distance x from left end. As the rate of flow is equal
in each part of the pipe, so
P
P1 - P P - P2 P1 P2
=
R1 R2
8hx 8h(l - x )
where R1 = , R2 =
p r4 p r4
x ( – x)
P1 - P P - P2
\ =
Ê 8hx ˆ 8h(l - x ) Fig. 5.119
ÁË 4 ˜¯
pr p r4
P1 - P P - P2
=
x (l - x )

È P (l - x ) + P2 x ˘
or P = Í 1 ˙˚
Î l

Stokes’ Law and Terminal Velocity


When a body moves in a viscous fluid, it experiences a viscous force Fb
or drag force, which is proportional to the velocity of the body.
h
Stoke experimentally found that, when a spherical body falls in
a viscous liquid, it experiences a viscous force 6 phrv (this value is
mg
different from the value theoretically obtained from Newton’s law).
Let us consider a spherical body of radius r is placed just inside the Vrg
6phrv
liquid of density r. It experiences gravitational force (mg) and buoyant
force both of constant values and viscous force of variable value.
Because of the net downward force the body starts accelerating and
viscous force starts increasing. At a certain velocity the net force on mg
the body becomes zero, and thereafter the body moves with constant Fig. 5.120
velocity, is called terminal velocity.
Fv + Fb = mg
or 6phrvt + Vrg = mg
v
4 4 vt
or 6phrvt + p r 3 r g = p r 3s g
3 3
2 r 2 (s - r ) g
VT =
9 h
Note: From the above expression we can see that terminal
velocity of a spherical body is directly proportional to the
difference in the densities of the body and fluid (s – r). If the t
density of fluid is greater than that of the body (i.e., r > s), the O
terminal velocity is negative. This means that the body instead
Fig. 5.121 Variation of ve-
of falling, moves upwards. This is why air bubbles rise up in locity of the body with time
water.
DĞĐŚĂŶŝĐĂůWƌŽƉĞƌƟĞƐŽĨDĂƩĞƌപ885

WORKED PROBLEMS
Problem 5.51 A sliding fit cylindrical of mass of 1 kg drops vertically down at a constant
velocity of 5 cm/s. Find the viscosity of the oil.
Solution As the body moves with constant velocity
\ mg = Fv
14.9 cm
dv
or 1g = hA
dy Oil 15 cm
1 kg
dv
or 1g = h(2p rl)
dy 15 cm

Ê 0.05 - 0 ˆ
or 1g = h(2p ¥ 7.5 ¥ 10 – 2 ¥ 15 ¥ 10 – 2) Á
Ë 0.05 ¥ 10 - 2 ˜¯
N-s
After solving, we get h = 1.4 Fig. 5.122
m2

Problem 5.52 A powder comprising particles of various sizes is stirred up in a vessel filled
to a height of 10 cm with water. Assuming the particles to be spherical, find the size of the
largest particle that will remain in suspension after 1 hour. (Density of powder = 4 g/cm3,
viscosity of water = 0.01 poise).
Solution Terminal velocity of the largest particle which is just about to settle at the bottom
of the vessel
10 ¥ 10 - 2
vt = m/s
3600
Let r be the radius of that particle, then
2 2 (s - r )
vt = r g
9 h

0.01 N - s
where s = 4 ¥ 103 kg/m3 and h=
10 m2
After solving, we get r = 2.0 ¥ 10– 6 m Ans.
Problem 5.53 Through a very narrow gap of height h, a thin plate of large extension is
pulled at a velocity v. On one side of the plate is oil of viscosity m1 and on the other side oil
of viscosity m2. Calculate the position of the plate so that (i) the shear force on the two sides
of the plate is equal, and (ii) the pull required to drag the plate is minimum.
Solution Let y be the distance of the plate from one of the surface.
(i) Force per unit area of the upper surface of the plate
dv v
f 1 = m1 = m1
dy ( h - y)
and force per unit area on the bottom surface of the plate
886പWŚLJƐŝĐƐĨŽƌ//dͲ:͗DĞĐŚĂŶŝĐƐ//

dv v
f2 = m 2 = m2
dy y
(h – y)
Equating the two, we get m1
v v v
m1 = = m2 y m2
( h - y) y

m2 h
fi y = Fig. 5.123
( m1 + m 2 )
(ii) Let F be the force required to pull the plate. Suppose A is the area of the plate, then
F = f 1A + f 2A
Ê v ˆ Ê vˆ
F = Am1 Á ˜ + Am 2 Á ˜
Ë h - y¯ Ë y¯
For F to be minimum,
dF
=0
dy
Am1v m v
or - 2 =0
( h - y )2 y 2
h
fi y =
m1
1+
m2

Problem 5.54 A large bottle is filled with a siphon made of capillary glass tubing. Compare
the time taken to empty the bottle when it is filled (i) with water (ii) with petrol of density
0.8 cgs unit. The viscosity of water and petrol are 0.01 and 0.02 cgs units respectively.
Solution The volume of liquid flowing in time t is through a tube is given by
p Pr 4
V = Qt = t
8hl
If t1 and t2 be the times taken by water and petrol respectively, then
p ( r1 gh)r 4
For water, V1 = t1
8h1l
p ( r2 gh)r 4
For petrol, V2 = t2
8h2 l
But V1 = V2
p ( r1 gh)r 4 p ( r2 gh)r 4
or t1 = t2
8h1l 8h2 l
t1 h r
\ = 1¥ 2
t2 h2 r1
0.01 0.8
= ¥ = 0.4
0.02 1.0
DĞĐŚĂŶŝĐĂůWƌŽƉĞƌƟĞƐŽĨDĂƩĞƌപ887

Problem 5.55 The level of liquid in a cylindrical vessel is kept constant at 30 cm. It has
three identical horizontal tubes of length 39 cm, each coming out at heights 0, 4 and 8 cm
respectively. Calculate the length of a single overflow tube of the same radius as that of
identical tubes which can replace the three when placed horizontally at the bottom of the
cylinder.
Solution

30 cm 30 cm
8 cm
4 cm

39 cm 
(a) (b)

Fig. 5.124

Pressure heads for first tube = 30 – 0 = 30 cm


Pressure head for second tube = 30 – 4 = 26 cm
Pressure head for third tube = 30 – 8 = 22 cm
Length of the each tube l = 39 cm
Let radius of the each tube = r cm
Rate of flow of liquid through the tubes
p P1r14 p ( r g ¥ 30) 4
Q1 = = ¥r
8hl 8hl

p ( r g ¥ 26) ¥ r 4
Similarly, Q2 =
8hl
p ( r g ¥ 22) ¥ r 4
and Q3 =
8hl
The total rate of flow
Q = Q1 + Q2 + Q3

pr gr 4
= (30 + 26 + 22)
8hl

pr gr 4 pr gr 4
= ¥ 78 = ...(i)
8h ¥ 39 4h
888പWŚLJƐŝĐƐĨŽƌ//dͲ:͗DĞĐŚĂŶŝĐƐ//

If l¢ is the length of the equivalent tube, then

p ( r g ¥ 30)r 4
Q = ...(ii)
8hl ¢
From Eqs. (i) and (ii)
p ( r g ¥ 30)r 4 pr gr 4
=
8hl ¢ 4h

or l¢ = 15 cm
C D
Problem 5.56 In Fig. 5.125 there is a pipe net- Q2
work of uniform cross-sectional area. Pressures
at inlet and outlet are 2P0 and P0 respectively. /2
Find ratio of volume of flow in two branches of A B
the pipes. The length of the each pipe is shown 2P0 Q1 P0
in Fig. 5.125. Also find pressure at points C 
and D.
Fig. 5.125

Solution Suppose resistance of the pipe AB is r then r


C D
resistance of pipes AC, CD and DB will be r/2, r Q2
and r/2 respectively. The given pipe network can be
treated as electrical circuit as shown in Fig. 5.126.
r/2 r/2
The total resistance of the circuit,
2r ¥ r Q1
R = A B
2r + r 2P0 P0
r
2r Fig. 5.126
=
3
DP
The total input Q =
R
2P0 - P0 3P0
or Q1 + Q2 = = ...(i)
2r/3 2r
As pipe ACDB is parallel to AB, so
Q1 ¥ r = Q2 ¥ 2r ...(ii)
or Q1 = 2Q2
Solving Eqs. (i) and (ii), we get
P0 P
Q1 = and Q2 = 0
r 2r
Q1
\ =2
Q2
DĞĐŚĂŶŝĐĂůWƌŽƉĞƌƟĞƐŽĨDĂƩĞƌപ889

Pressure difference:
r
PA – PC = Q2 ¥
2
P0 r P0
= ¥ =
2r 2 4
Given, PA = 2P0
7 P0
\ PC =
4
r
Similarly, PD – PB = Q2 ¥
2
P r P
= 0¥ = 0
2r 2 4
Given, PB = P0

5P0
\ PD =
4

Problem 5.57 A thin ring of radius R is made of a material of density r and Young’s
modulus Y. If the ring is rotated about its centre in its own plane with angular velocity w,
find the small increase in its radius. (IIT Roorkee, 1985)
Solution Consider an element PQ of length dl. Let T be the tension and A the area of cross
section of the wire.
Mass of element dm = Volume ¥ Density
= A(dl)r
The component of T, towards the centre provides the necessary centripetal force

Êqˆ
\ 2T sin Á ˜ = (dm)Rw2 …(i)
Ë 2¯

q q (dl/R)
For small angles sin ª =
2 2 2
Substituting in Eq. (i), we have T cos (q/2) T cos (q/2)
dl
T◊ = A(dl)rRw2 q/2 q/2
R F
P Q
or T = Arw2R2 q

Let DR be the increase in radius,


O
Dl D(2p R) DR T T
Longitudinal strain = = =
l 2p R R
T/A
Now, Y =
DR/R Fig. 5.127
890പWŚLJƐŝĐƐĨŽƌ//dͲ:͗DĞĐŚĂŶŝĐƐ//

TR
\ DR =
AY
( Arw 2 R2 )R
=
AY
rw 2 R3
or DR =
Y

Problem 5.58 A vertical capillary is brought in contact with the water surface. What
amount of heat is liberated while the water rises along the capillary? The wetting is assumed
to be complete. The surface tension equals T.
Solution If h is the capillary rise, then
2T
h =
rr g
Pressure P = r gh
2T
=
r
0+P T
Average pressure Pav = =
2 r
The volume of water rise = p r2h
Heat liberated = Work done by surface tension force
= Pav ¥ DV
T
= ¥ p r 2 h = p Tr h
r
Ê 2T ˆ
= p Tr ¥ Á
Ë r r g ˜¯
2p T 2
=
rg

Problem 5.59 The lower end of a capillary tube of diameter 2.00 mm is dipped 8 cm below
the surface of water in a beaker. What is the pressure required in the tube in order to blow a
hemispherical bubble at its end in water? The surface tension in water at the temperature of
the experiment is 7.30 ¥ 10–2 N/m. 1 atmospheric pressure = 1.01 ¥ 105 Pa, density of water
= 1000 kg/m3, g = 90 m/s2. Also calculate the excess pressure.
Solution The pressure difference in air bubble
2T
Pi – P o =
R
2 ¥ 7.30 ¥ 10 - 2
= = 146 Pa
1 ¥ 10 - 3
DĞĐŚĂŶŝĐĂůWƌŽƉĞƌƟĞƐŽĨDĂƩĞƌപ891

Pressure inside bubble


2T
Pi = Po +
R 8 cm

2T
= (Pa + hr g ) +
r

= 1.02 ¥ 103 Pa Fig. 5.128

Problem 5.60 What is the excess pressure inside a bubble of soap solution of radius 5.00
mm? Given that the surface tension of soap solution at the temperature 20°C is 2.50 ¥ 10–2
N/m. If an air bubble of the same dimension is formed at a depth of 40 cm inside a container
containing the soap solution (of relative density 1.20), what would be the pressure inside
the bubble? (1 atm = 1.01 ¥ 105 Pa).
4T
Solution Excess pressure P =
R
4 ¥ 2.50 ¥ 10 - 2
=
5 ¥ 10 - 3
= 20 N/m2
4T
At a depth h Pi – Po =
R
4T
\ Pi = P0 +
R
4T
= (Pa + r gh) +
R
4 ¥ 2.50 ¥ 10 - 2
= 1.01 ¥ 105 + 12000 ¥ 9.8 ¥ 0.4 +
5 ¥ 10 - 3
= 105724 N/m2
Problem 5.61 A sphere is dropped under gravity through a fluid of viscosity h. Taking the
average acceleration as half of the initial acceleration, show that the time taken to attain the
terminal velocity is independent of the fluid density.
Solution The initial acceleration of the drop
mg - Fb
=
m
È V r g - Vs g ˘
= Í ˙
Î Vr ˚
(r - s )g
=
r
892പWŚLJƐŝĐƐĨŽƌ//dͲ:͗DĞĐŚĂŶŝĐƐ//

(r - s )g
The average acceleration, a =
2r

2 2 (r - s )g
The terminal velocity, v = r
9 h
Now using v = u + at, we have

v 4 r2r
t = =
a 9 h

Problem 5.62 Two tubes A and B of length 1 m and 0.5 m have radii 0.1 mm and 0.2 mm
respectively. If a liquid is passing through the two tubes, entering A at a pressure of 0.8 m of
mercury and leaving B at a pressure of 0.76 m. Find the pressure at the junction of A and B.

p (PA - P)rA4 p (P - PB )rB4


Solution =
8hlA 8hlg

\ P = 0.7612 m of Hg
Problem 5.63 In Millikan’s oil drop experiment, what is the terminal speed of a drop of
radius 2.0 ¥ 10– 5 m and density 1.2 ¥ 103 kg/m3? Take the viscosity of air at the temperature
of the experiment to be 1.8 ¥ 10– 5 Ns/m2. How much is the viscous force on the drop at that
speed? Neglect buoyancy of the drop due to air.
Solution Terminal speed is given by
2 2 (s - r )
v = r g
9 h
= 5.8 cm/s
Viscous force, Fb = 6 phr v
= 3.9 ¥ 10–10 N.
Problem 5.64 Two capillary tubes AB and BC are joined end to end at B, AB is 16 cm long
and of diameter 4 mm whereas BC is 4 cm long and of diameter 2 mm. The composite tube
is held horizontally with A connected to a vessel of water giving a constant head of 3 cm
and C is open to the air. Calculate the pressure difference between B and C.
Solution
PA = 3 cm PB
PC = 0

16 cm 4 cm

Fig. 5.129
DĞĐŚĂŶŝĐĂůWƌŽƉĞƌƟĞƐŽĨDĂƩĞƌപ893

8hl1
The resistance of two tubes are R1 =
p r14
8hl2
and R2 =
p r24
As tubes are connected in series, and so,
Q1 = Q2
PA - PB P -0
or = B
R1 R2
(3 - PB ) PB
or =
Ê 8hl1 ˆ Ê 8hl2 ˆ
ÁË p r 4 ˜¯ ÁË p r 4 ˜¯
1 2

r14 P r4
or (3 - PB ) = B2
l1 l2
After substituting the value and simplifying, we get
PB = 2.4 cm
Problem 5.65 A capillary tube of radius 0.50 mm is dipped vertically in a pot of water.
Find the difference between the pressure of the water in the tube 5.0 cm below the surface
and the atmospheric pressure. Surface tension of water = 0.075 N/m.
Solution The pressure just below the free surface of water

Ê 2T ˆ
PA = Á Pa - ˜
Ë R¯
The pressure difference A
h = 5 cm
PB – PA = rgh
B

Ê 2T ˆ
or PB - Á Pa - ˜ = rgh
Ë R¯
2T
\ PB – Pa = rgh -
R
2 ¥ 0.075
= 1000 ¥ 9.8 ¥ 0.05 – Fig. 5.130
0.5 ¥ 10 - 3
2
= 190 N/m
Problem 5.66 A glass rod of diameter d1 = 1.5 mm is inserted symmetrically into a glass
capillary with inside diameter d2 = 2 mm. Then the whole arrangement is vertically oriented
and brought in contact with the surface of water. To what height will the water rise in the
capillary?
894പWŚLJƐŝĐƐĨŽƌ//dͲ:͗DĞĐŚĂŶŝĐƐ//

Solution If h is the height of water rise, then


Weight of water in the tube = surface tension force
p 2
(d2 - d12 )hr g = p(d2 + d1)T
4
4T
\ h = = 6 cm
r g(d2 - d1 )

Problem 5.67 Consider the situation shown in Fig. 5.131. The force F is equal to m2g/2.
If the area of the cross section of the string is A and its Young’s modulus Y, find the strain
developed in it. The string is light and there is no friction anywhere.

F m1

m2

Fig. 5.131

Solution If a is the acceleration of the blocks, then


m1
T – F = m 1a
and m 2g = m 2a
m2 g(2m1 + m2 ) T
After simplifying, we get T =
2(m1 + m2 ) m2

Stress
Strain = m2g
Y
m2 g(2m1 + m2 )
= Fig. 5.132
2 AY(m1 + m2 )

Problem 5.68 Each of the three blocks P, Q and R shown in Fig. 5.133 has a mass of 3 kg.
Each of the wires A and B has cross-sectional area 0.005 cm2 and Young’s modulus 2 ¥ 1011 N/
m2. Neglect friction. Find the longitudinal strain developed in each wire. (Take g = 10 m/s2)

A B
P Q

Fig. 5.133
DĞĐŚĂŶŝĐĂůWƌŽƉĞƌƟĞƐŽĨDĂƩĞƌപ895

Solution If a is the acceleration of the blocks, then T2 T1

3g – T1 = 3a
T1 – T2 = 3a T1
and T2 = 3a
After simplifying, we get 3g
g 10
a = m/s2 = m/s2
3 3 Fig. 5.134
T1 = 2g
= 20 N
and T2 = 10 N
T1/A
Strain developed, e1 =
Y
20
=
0.005 ¥ 10 - 4 ¥ 2 ¥ 1011

= 2 ¥ 10 – 4
T2/A
e2 =
Y
10
=
0.005 ¥ 10 - 4 ¥ 2 ¥ 1011

= 1 ¥ 10 – 4
Problem 5.69 A body of mass 3.14 kg is suspended from one end of a wire of length 10 m.
The radius of the wire is changing uniformly from 9.8 ¥ 10 – 4 m at one end to 5.0 ¥ 10 – 4 m at
the other end. Find the change in the length of the wire. What will be the change in length
if the ends are interchanged? Young’s modulus of the material of the wire is 2 ¥ 1011 N/m2.
(IIT Roorkee, 1989)
Solution The change in length of the wire is given by
FL
DL =
p r1r2Y
(3.14 ¥ 9.8) ¥ 10
=
p (9.8 ¥ 10 ) ¥ (5 ¥ 10 - 4 ) ¥ (2 ¥ 1011 )
-4

= 10 – 3
Problem 5.70 Find the attraction force between two parallel glass plates separated by a
distance h = 0 .10 mm, after a water drop of mass m = 70 mg was introduced between them.
The wetting is assumed to be complete.
Solution Force needed to pull the plate, each of area A
2TA
F=
d
896പWŚLJƐŝĐƐĨŽƌ//dͲ:͗DĞĐŚĂŶŝĐƐ//

Here d = h,
Ahr = m
m
\ A =
hr
Ê mˆ
2T Á ˜
Ë hr ¯ 2Tm
\ F = =
h rh2

Problem 5.71 Eight rain drops of radius 1 mm each falling down with terminal velocity of
5 cm/s coalesce to form a bigger drop. Find the terminal velocity of the bigger drop.
Solution If R is the radius of bigger drop, then
4 4
8 ¥ p r 3 = p R3
3 3
\ R = 2r = 2 ¥ 1 = 2 mm.
We know that, v1 a r2
v2 R2
\ = 2
v1 r
R2 22
or v2 = v1 = ¥ 5 = 20 cm/s
r2 12

Problem 5.72 Two large glass plates are placed vertically and parallel to each other inside
a tank of water with separation between the plates equal to 1 mm. Find the rise of water in
the space between the plates. Surface tension of water = 0.075 N/m.
Solution If h is the rise in the tube, then
Surface tension force = Weight of liquid rise
2Tl = r(lbh)g
2T 2 ¥ 0.075
\ h = =
rbg 1000 ¥ 10 - 3 ¥ 9.8
= 1.5 ¥ 10 – 2 m
Problem 5.73 In the bottom of a vessel with mercury there is a round hole of diameter d
= 70 mm. At what maximum thickness of the mercury layer will the liquid still not flow out
through this hole?
Solution The height above the hole is given by

2T 2T 4T
h= = = = 21 cm
r r g d r g dr g
2
DĞĐŚĂŶŝĐĂůWƌŽƉĞƌƟĞƐŽĨDĂƩĞƌപ897

Problem 5.74
(i) Two wires PQ and QR, one of aluminium and the other P
of steel, each 1 m long are joined end to end to form a
composite wire of length 2 m. The radius of each wire is
1 mm. Calculate the total length of the composite wire
if a mass of 10 kg is attached at end C. YAl = 7 ¥ 1010 Q
N/m2, Ysteel = 2.1 ¥ 1011 N/m2.
(ii) What is the elastic energy per unit volume produced in
each wire? R
(iii) What load at end C would produce an expansion 0.32 10 kg
mm in both wires combined?
Fig. 5.135
Solution
(i) The change in the length of the wire
FL
DL1 = [L = 1 m]
AY1

FL
and DL2 =
AY2
\ DL = DL1 + DL2

FL È 1 1˘
= Í + ˙
A Î Y1 y2 ˚

(10 ¥ 10) ¥ 1 È 1 1 ˘
= -3 2 Í
+
p (10 ) Î 7 ¥ 10 10
2.1 ¥ 1011 ˙˚
= 0.000606 m
Thus, total length of the wire
= 2 + 0.000606 = 2.000606 m.
mg 10 ¥ 10
(ii) Stress in each wire f = =
pr 2
p (10 - 3 )2
= 3.18 ¥ 107 N/m2
Elastic energy per unit volume is given by
f 2 (3.18 ¥ 107 )2
u1 = =
2Y1 2 ¥ 7 ¥ 1010
= 7.22 ¥ 103 J
f 2 (3.18 ¥ 107 )2
and u2 = =
2Y2 2 ¥ 2.1 ¥ 1011

= 2.41 ¥ 103 J
898പWŚLJƐŝĐƐĨŽƌ//dͲ:͗DĞĐŚĂŶŝĐƐ//

Problem 5.75 A steel rod of length l1 = 30 cm and two F


identical brass rods of length l2 = 20 cm each, support a
light horizontal plateform as shown in Fig. 5.136. Cross-
sectional area of each of the three rods is A = 1 cm2.
Calculate stress in each rod when a vertically downward
force F = 5000 N is applied on the plateform. Given, Brass Steel Brass 20 cm
Young’s modulus of elasticity for steel Ys = 2 ¥ 1011 Nm– 2
and brass Yb = 1 ¥ 1011 Nm– 2.
Solution If stresses in brass and steel are fb and fs 10 cm
respectively, then
2fb A + fs A = 5000 ...(i)
The change of length of rods are equal and so Fig. 5.136
DLb = DLs
fb Lb fL
or = s s ...(ii)
Yb Ys
After simplifying and substituting the given values, we have
fb = 1.5 ¥ 107 N/m2
fs = 2 ¥ 107 N/m2
Problem 5.76 Three capillary tubes of the same radius r but of lengths l1, l2 and l3 are fitted
horizontally to the bottom of a tall vessel containing a liquid at constant head and flowing
through these tubes. Calculate the length of a single outflow tube of the same radius r which
can replace the three capillaries.
Solution In this case the tubes are in parallel, so
1 1 1 1
= + +
R R1 R2 R3
As resistance is proportional to the length of the tube (constant) and so
1 1 1 1
= + +
l l1 l2 l3

1 Èl l + l l + l l ˘
= Í 12 13 2 3˙
l Î l1l2l3 ˚
È l1l2l3 ˘
\ l = Í ˙
Î l1l2 + l1l3 + l2l3 ˚
Problem 5.77 A solid copper cylinder of length L = 65 cm is placed on a horizontal surface
and subjected to a vertical compressive force F = 1000 N directed downwards and distributed
uniformly over the end face. What will be the resulting change of the volume of the cylinder
in cubic millimetres?
DĞĐŚĂŶŝĐĂůWƌŽƉĞƌƟĞƐŽĨDĂƩĞƌപ899

r
l

Fig. 5.137

Solution When the force is applied, let DD be the increase in diameter and Dl be the
decrease in length. Then from the definition of Poisson’s ratio, we have
Lateral strain
v =-
Longitudinal strain

( DD/D) - ( Dr/r )
=- =
( Dl/l) ( Dl/l)
Dr Dl
or = –v
r l
Volume of cylinder is
V = pr2l
dV dr dl
= 2 +
V r l
dl dl
= - 2v +
l l
dl
= (1 – 2v)
l
From definition of Young’s modulus, we have
Stress ( - F/p r 2 )
Y = =
Strain ( DV/V )

DV F
or Y = - 2
V pr
Fl(1 - 2v)
or DV = -
Y
On substituting numerical values, we get
DV = 1.6 mm3
900പWŚLJƐŝĐƐĨŽƌ//dͲ:͗DĞĐŚĂŶŝĐƐ//

Problem 5.78 A sphere of 0.25 g is attached to one end of a steel wire, having cross-sectional
area of 2.5 mm2 and one metre long. In order to form a conical pendulum, the other end
is attached to a vertical shaft which is set rotating about its axis. Calculate the number of
revolutions necessary to extend the wire by 2 mm. (IIT Roorkee, 1978)
Young’s modulus of elasticity of steel = 2 ¥ 1011 N/m2 and g = 9.8 m/sec2.
Solution The situation is shown in Fig. 5.138.
Let T be the tension in the wire when extension is 2 mm. Hence,
T/A
Y =
l/L
Y ◊ Al
\ T = q
L
Here l = 2 mm = 2 ¥ 10 – 3 m, L = 1.0 m,
L=1m
A = 2.5 mm2 = 2.5 ¥ 10 – 6 m2
Substituting these values, we get
T cos q
(2 ¥ 1011 )(2.5 ¥ 10 - 6 ) ¥ (2 ¥ 10 - 3 ) T
T =
1
= 1.0 ¥ 103 N r = L sin 0
T sin q
From Fig. 5.138, T cos q = mg
mg 0.25 ¥ 9.8
\ cos q = = 3
= 2.45 ¥ 10 - 3 mg
T 1 ¥ 10
Fig. 5.138
2 -3 2
sin q = (1 - cos q ) = [1 - (2.45 ¥ 10 ) ]

ª 1.0
Further, from figure T sin q = mv2/r

T sin q ¥ r (1.0 ¥ 10 3 )(1)L sin q


\ v2 = =
m 0.25
(1.0 ¥ 10 3 )(1)(1)
or v2 = = 4.0 ¥ 10 3
0.25
or v = 63.25 m/s

2p r 2p
Period of revolution = = = 0.0993 s
v 63.25
1 1
Frequency = =
Time period 0.0993
= 10.07 rev/sec.
Hence, the sphere must take about 10 revolutions per second to extend the wire by 2 mm.
DĞĐŚĂŶŝĐĂůWƌŽƉĞƌƟĞƐŽĨDĂƩĞƌപ901

Problem 5.79 A steel wire 1 m long and of radius 1 mm has a sphere of mass 2.5 kg attached
to one of its ends. It is whirled in a vertical circle and has an angular velocity of 2 rev/s at
the bottom of the circle. Calculate the elongation of the wire when the weight is at the lowest
point of its path. Young’s modulus of steel is 20 ¥ 1010 N/m2 and g = 9.8 m/s2.
Solution The centripetal force at the lowest point is given by
mw2 = T – mg ...(i)
Here T = Tension in the wire when the sphere is at the bottom w = Angular speed and
r = Radius of the circle.
Let Dl be the elongation of the wire due to tension T, then
Stress F/A T/A
Y = = =
Strain Dl/l Dl/l
Tl
\ Dl =
AY
Given that, r = 1 mm = (1 ¥ 10 – 3 m).
\ A = p (1 ¥ 10 – 3)2 = p ¥ 10 – 6 m2
From Eq. (1).
T = mg + m(2 p n)2 r
= 2.5 [9.8 + 4p2 ¥ 4 ¥ 1] = 419.3 N
Substituting these values in Eq. (2), we get
419.3 ¥ 1.0
Dl =
(p ¥ 10 - 6 )(20 ¥ 1010 )
= 6.67 ¥ 10 – 4 m = 0.067 cm.
Problem 5.80 One end of a wire 100 cm long (without load) is fixed and to the other end is
attached a mass of 2 kg. The mass is kept moving with uniform speed in a horizontal circle
of radius 60 cm. Find the strain and w of the wire, if the radius of the wire is 0.032 cm and
Young’s modulus 2 ¥ 1011 dynes/cm2.
Solution The situation is shown in Fig. 5.139. A

Given, AC = 1 metre, OC = 0.6 m


q
\ AO = [(1)2 - (0.6)2 ] = 0.8 m
100 cm
mg 2 ¥ 9.8
From Fig. 5.139 T = = = 24.5 N
cos q 0.8 T

24.5
Stress in wire = B C
p (0.032 ¥ 10 - 2 )2 O

Stress mg
Strain =
Y Fig. 5.139
902പWŚLJƐŝĐƐĨŽƌ//dͲ:͗DĞĐŚĂŶŝĐƐ//

24.5
=
p (0.032 ¥ 10 - 2 )2 ¥ (2 ¥ 1010 )
\ Y = 2 ¥ 1011 dynes/cm2
= 2 ¥ 1010 N-m2
Strain = 0.32 ¥ 10 – 2 m
If w be the angular velocity, then

Ê g ˆ Ê gˆ Ê 9.8 ˆ
w = ÁË L cos q ˜¯ = ÁË r ˜¯ = ÁË 0.6 ˜¯

= 4 rad/sec.
Problem 5.81 A mass 2 kg is attached to one end an elastic string of natural length 1.5 m,
whose other end is fixed at a point A. The elastic modulus of the string is such that the 2 kg
mass hanging vertically would stretch the string by 3 cm. The mass is held at A and allowed
to fall vertically. How far below A will it come to rest? (IIT Roorkee, 1976)
0.03
Solution Here Strain =
1.5
If A be the area of cross section of the wire, then
2 ¥ 9.8
Stress = n/m2
A
Stress
\ Young’s modulus Y =
Strain
2 ¥ 9.8 1.5
= ¥
A 0.03
980
= n/m2
A
After falling the mass through a distance 1.5 m, the velocity is given by
v2 = 2 ¥ 9.8 ¥ 1.5
1
\ Kinetic energy = ¥ 2 (2 ¥ 9.8 ¥ 1.5)
2
= 29.4 J
Now the wire will be stretched. Let l be the increase in length, then the strain is given
by (l/1.5)
\ Tension in the string = Y ¥ A ¥ Strain
980 l
= ¥ A¥
A 1.5
DĞĐŚĂŶŝĐĂůWƌŽƉĞƌƟĞƐŽĨDĂƩĞƌപ903

980 l
=
1.5
1
Now, Work done = ◊F ◊l
2
1 980 l 2 980 l 2
= ◊ = joule
2 1.5 3
Loss of potential energy = 2 ¥ 9.8 ¥ l joule
= 19.6 l joule
Loss of kinetic energy = 29.4 joule

980 l 2
\ = 29.4 + 19.6 l
3
Solving it for l and taking positive value, we get l = 0.33 so the mass comes to rest at 1.5
+ 0.33 = ODGNQYA.
Problem 5.82 A copper wire 2 metre long is stretched by 1 mm. If the energy stored in the
stretched wire is converted into heat, calculate the rise in temperature of the wire. Given
Y = 12 ¥ 1011 dynes/cm2, density of copper = 9, specific heat of copper = 0.1 and T = 4.2 ¥
107 ergs/cal.
Solution The energy stored per unit volume is given by
1 1
= Stress ¥ Strain = ◊ Y ◊ (strain)2
2 2
1
= ¥ 12 ¥ 1011 (0.1/200)2 = 1.5 ¥ 105 ergs
2
If V be the volume of the wire, then energy E stored is given by
1.5 ¥ 105 ¥ V ergs
When this energy is converted into heat, then heat produced
E 1.5 ¥ 10 5 V
= cal
J 4.2 ¥ 107
If m is the mass of the wire, c specific heat and DT, rise in temperature then

1.5 ¥ 10 5 V
m cDT =
4.2 ¥ 107

1.5 ¥ 10 5 V
or Vr c DT = (∵ m = Vr)
4.2 ¥ 107
1.5 ¥ 10 5 1
\ DT = 7
= °C
( 4.2 ¥ 10 ) ¥ 9 ¥ 0.1 252
904പWŚLJƐŝĐƐĨŽƌ//dͲ:͗DĞĐŚĂŶŝĐƐ//

Problem 5.83 Uniform rod of length L and density r is being L


pulled along a smooth floor with a horizontal acceleration
a (see Fig. 5.140). What is the magnitude of the stress at the a
transverse cross-section through the mid-point of the rod.
(IIT-JEE, 1999)
Fig. 5.140
Solution Let A be the area of cross section of the rod. The
mass m of the rod will be L A r
Force at the end of the rod = ma = LAra
1
Force at the middle = [LAra]
2
1 È L A ra ˘
Stress = Force per unit area =
2 ÍÎ A ˙˚
1
= r a L.
2

Problem 5.84 A thin rod of negligible mass and area of cross-section 4 ¥


10 – 6 m2 suspended vertically from one end has a length of 0.5 m at 400°C.
The rod is cooled to 0°C but prevented from contracting by attaching a mass
at the lower end. Find (i) this mass, and (ii) the energy stored in the rod.
Given for the rod, Young’s modulus = 1011 N/m2. Coefficient of linear m
expansion = 10 –5 k –1 = 10 m/s2 .
Fig. 5.141
Solution Length of the rod at 100°C = 0.5 m.
Change in length when temperature is 0°C
Dl = 0.5 ¥ 10 –5 ¥ 100
Dl 0.5 ¥ 10 - 5 ¥ 100
Now, Strain = = = 10 - 3
l 0.5
Stress = Young’s modulus ¥ Strain
= 1011 ¥ 10 –3 = 108 N/m2
Force = Stress ¥ Area
= 108 ¥ (4 ¥ 10 – 6) = 400 N
\ mg = 400 N

400
(i) Suspended mass m = = 40 kg
10
1
(ii) Energy stored in rod = ¥ F ¥ Dl
2
1
= ¥ 400 ¥ (0.5 ¥ 10 –3)
2
= 0.1 J
DĞĐŚĂŶŝĐĂůWƌŽƉĞƌƟĞƐŽĨDĂƩĞƌപ905

Problem 5.85 The Young’s modulus for steel is 2.0 ¥ 1011 newtons/m2. If the inter-atomic
spacing for the metal be 2.8 Å, compute (i) inter-atomic force constant in newtons per Å,
and (ii) increase in the inter-atomic spacing for a force of 109 newton/m2.
Solution
(i) The inter-atomic force constant k is given by
k = Y r0 = (2.0 ¥ 1011) (2.8 ¥ 10 – 10)
(∵ 2.8 Å = 2.8 ¥ 10–10 m)
– 10
= 56 newton/metre = 56 ¥ 10 PGYVQP—
(ii) As the distance between two atoms is r0, hence the area of one chain at atoms is r02.
If F is the inter-atomic force, then
F = Stress ¥ r02 = 109 ¥ (2.9 ¥ 10–10)2
= 7.84 ¥ 10 –11 newton.
Let Dr be the increase in the inter-atomic spacing, then
F = k ¥ Dr

F 7.84 ¥ 10 - 11
or Dr = = -9
= 1.4 ¥ 10 - 2 —
k 5.6 ¥ 10

Problem 5.86 A lift is tied with thick iron wires and its mass is 1000 kg. What should be
the minimum diameter of the wire if the maximum acceleration of lift is 1.2 m/sec2 and the
maximum safe stress of the wire is 1.4 ¥ 108 nt/m2. (Given g = 9.8 m/sec2).
Solution The tension T in the rope of the lift when it goes upwards is given by
T = m(g + a) = 1000 (9.8 + 1.2)
= 11000 newtons.
Let r be the radius of the wire, then maximum stress will be T/pr2.
Hence, T/pr2 = 1.4 ¥ 108
T
or r2 =
p ¥ 1.4 ¥ 108

11000 1
= =
3.14 ¥ (1.4 ¥ 108 ) 4 ¥ 10 4
1 1
Now, r = 2
= = 0.005 metre.
2 ¥ 10 200
\ Diameter of the wire = 2r = 0.01 metre.
906പWŚLJƐŝĐƐĨŽƌ//dͲ:͗DĞĐŚĂŶŝĐƐ//

Problem 5.87 A thin uniform rod of length L and mass M


rotates with constant angular velocity w in a horizontal plane dx
x
about a vertical axis passing through one of its ends, as shown
in Fig. 5.142. Determine the total elongation in the rod if the L
Young’s modulus of the material of rod is Y. 2
w x
Solution In this case, the stress in each element of the rod
F F + dF
decreases as one moves away from the axis of rotation. dm
Let us consider a small element of length dx at a distance
Fig. 5.142
x from the axis of rotation as shown in Fig. 5.142. Applying
Newton’s second law on the free body diagram of the element, we get
F – (F + dF) = (dm)w2x
or – dF = (dm)w2x
M
Since dm = dx , therefore
L
ÊM ˆ
– dF = Á dx˜ w 2 x
Ë L ¯
M 2
or dF = - w x dx
L
Integrating both the sides, we get
Mw 2 Mw 2 x 2
L Ú
F = - xdx = - +c
2L
where c is the constant of integration
MLw 2
Since at x = L, F = 0, therefore, C =
2
Mw 2 L Ê x2 ˆ
or F = 1 -
2 ÁË L2 ˜¯
If dl is the differential elongation of the differential element dx, then

F Mw 2 L2 Ê x2 ˆ
dl = dx = 1 - dx
YA 2YA ÁË L2 ˜¯
The total elongation of the rod is
Mw 2 L2 È ˘
L L 2
x
dl = Í Ú dx - Ú 2 dx ˙
2YA ÍÎ 0 0
L ˙˚

Mw 2 L2 È L˘
or l =
2YA ÍÎL - 3 ˙˚

Mw 2 L3
or l =
3YA
DĞĐŚĂŶŝĐĂůWƌŽƉĞƌƟĞƐŽĨDĂƩĞƌപ907

Problem 5.88 A ring of radius 0.1 m is made out of a thin metallic wire of area of cross
section 10–6 m2. The ring has a uniform charge of p coulomb. Find the change in the radius of
the ring when a charge of 10– 8 coulomb is placed at the centre of the ring. Young’s modulus
of the metal is 2 ¥ 1011 N/m2.
Solution The situation is shown in Fig. 5.143.
Consider an element of length Dl making an angle Dq Dl
at the centre. There will be a tangential tension T at each
end of this element. The horizontal components of tensions
are cancelled while the vertical components are added up. F
Hence, T Dq T
Dq Ê Dl ˆ
F 2T sin ª T Dq ª T Á ˜
2 Ë R¯
FR O
\ T = ...(1)
Dl
Charge per unit length of the ring
q p 1
s = = = = 5 C/m
2 p R 2 p R 2 ¥ 0.1
Charge on the element of length Dl = 5Dl C/m Fig. 5.143

Charge on the centre of ring = 10 – 8 coulomb


1 5 Dl ¥ 10 - 8
\ Force of Dl, F = ¥
4 p e0 (0.1)2
5 Dl ¥ 10 - 8
= (9 ¥ 109)¥
(0.1)2
= 4.5 ¥ 104 Dl newton ...(2)
Substituting this value in Eq. (1), we get
( 4.5 ¥ 10 4 )Dl ¥ R
T = = (4.5 ¥ 104) (0.1)
Dl
= 4.5 ¥ 103 newton
Strain along the circumference of the wire
Stress T/A
= =
Young’s modulus Y
Charge in circumference = Strain ¥ Circumference
T/A
= ¥ 2p R
Y
Change in circumference
\ Change in radius =
2p

TR ( 4.5 ¥ 10 3 )(0.1)
= = = 2.25 ¥ 10– 3 m.
A 10 - 6 ¥ (2 ¥ 1011 )
908പWŚLJƐŝĐƐĨŽƌ//dͲ:͗DĞĐŚĂŶŝĐƐ//

Problem 5.89 A simple pendulum is made by attaching a 1 kg bob to 5 m long copper wire
of diameter 0.08 cm and it has a certain period of oscillation. Next a 10 kg bob is substituted
for the 1 kg bob. Calculate the change in time period, if any. (Young’s modulus of copper =
12.4 ¥ 1010 N/m2). (IIT Roorkee, 1975)
Solution The time period of a simple pendulum is given by

T = 2p (l/g )
where l is the length of the simple pendulum.
In case of copper wire, the change in length when 1 kg bob is attached
Dl1 F 1 mg 1
= ◊ = ◊
l A Y p r2 Y
1 ¥ 9.8 ¥ 5 1
\ Dl1 = -2 2
¥
3.14 ¥ (0.04 ¥ 10 ) 12.4 ¥ 1010
= 7.86 ¥ 10 – 4 m
Similarly, the change in length when 10 kg is substituted for one kg bob

10 ¥ 9.8 ¥ 5 1
Dl2 = ¥
3.14 ¥ (0.04 ¥ 10 - 2 )2 12.4 ¥ 1010

= 7.86 ¥ 10– 3 m
Time period in the first case,
Ï l + Dl1 ¸
T1 = 2p Ì 1 ˝
Ó g ˛

ÏÔ (7.86 ¥ 10 - 4 ) ¸Ô
= 2 ¥ 3.14 Ì5 + ˝
ÔÓ 9.8 Ô˛
= 4.488 sec.
Time period in the second case,
Ï l + Dl2 ¸
T2 = 2p Ì 1 ˝
Ó g ˛

ÏÔ 5 + (7.86 ¥ 10 - 3 ) ¸Ô
= 2 ¥ 3.14 Ì ˝
ÓÔ 9.8 ˛Ô
= 4.4915 sec
\ Change in time period is given by
DT = T2 – T1
= 4.4915 – 4.488 = 0.0035 sec.
DĞĐŚĂŶŝĐĂůWƌŽƉĞƌƟĞƐŽĨDĂƩĞƌപ909

Problem 5.90 A sphere of mass 50 g is attached to one


end of a steel wire 0.315 mm in diametre and one metre
long. In order to form a conical pendulum, the other end is
attached to a vertical shaft which is set rotating about its axis. q
Calculate the number of resolutions necessary to break it. The
wire will break when the stress exceeds 4.5 ¥ 109 dyne/cm2,
g = 9800 cm/s2. l

Solution See Fig. 5.144. T cos q


From figure T sin q = mrw2 = m(l sin q)w2 T
T sin q
T
\ 2
w = ...(1) mrw 2 r
ml
T = Breaking stress ¥ Area
2 mg
Ê 0.0315 ˆ
= (4.5 ¥ 109) ¥ p Á
Ë 2 ˜¯
m = 50 g and l = 100 cm. Fig. 5.144
Substituting these values in Eq. (1), we get w (radian/sec)
w
Now frequency = revolutions/sec.
2p
Problem 5.91 A copper rod of length l is suspended from the ceiling
by one of its ends. Find: x

(a) the elongation Dl of the rod due to its own weight; l dx


DV
(b) the fractional decrement of its volume - ; and
V
(c) the elastic potential energy stored in the rod due to its own
weight. Fig. 5.145
Solution
(a) The increment in length dx due to weight (l – x) length of the rod downside
Dx F/A m(l - x ) g
= =
dx Y AY
m = Mass per unit length
m(l - x )
or, Dx = g dx
AY
\ In elongation
mg l
Dl = Ú Dx = AY Ú0 (l - x) dx
mg Ê 2 l 2 ˆ mg l 2 r g l 2
= l - ˜= = ...(i)
AY ÁË 2 ¯ AY 2 2Y
m
where, r = is the density of the rod.
A¥1
910പWŚLJƐŝĐƐĨŽƌ//dͲ:͗DĞĐŚĂŶŝĐƐ//

p D2
(b) Initial volume of the rod = l=V
4
DD/D b
\ Poisson’s ratio, s = + =
Dl/l a
\ Final volume,
pD2l ÏÔÊ Dl ˆ ¸Ô
2
DD ˆ Ê
V¢ = ÌÁË 1 - ˜¯ ÁË 1 + ˜¯ ˝
4 ÓÔ D l ˛Ô
DV
\ – = Fractional change in volume
V
∫ Volume strain,
2
V¢ - V Ê DD ˆ Ê Dl ˆ
= = Á1 - ˜ Á 1+ ˜ -1
V Ë D¯ Ë l ¯

p D2
\ V = l
4
Ê 2 DD ˆ Ê Dl ˆ
= Á1- ˜¯ ÁË 1 + ˜¯ - 1
Ë D l
DD
as <<1
D
Dl
<<1
l
Dl 2 DD
∫ 1+ - -1
l D
Dl
= (l - 2s ) ...(ii)
l
r gl
= (l - 2 s ) ...(iii)
2Y
1
(c) Energy density at x = ¥ Stress ¥ Strain
2
1 (l - x )mg (l - x )mg
= ¥ ¥
2 p r 2Y p r2
where r = radius of the rod
1 ( l - x )2
= ¥ ¥ r 2 (r = density)
2 Y
Energy stored in the volume of length element dx

Ï 1 ( l - x )2 ¸Ô
= ÔÌ ¥ ¥ r 2 ˝ ¥ (p r 2 dx )
ÔÓ 2 Y Ô˛
DĞĐŚĂŶŝĐĂůWƌŽƉĞƌƟĞƐŽĨDĂƩĞƌപ911

Total potential energy stored


1 p r2r2 l
Ú0 (l - x)
2
U = dx
2 Y

1 p r 2 r 2 l3
=
6 Y

Problem 5.92 Show that work performed to make a hoop out of a steel band (Young’s
modulus Y) of length l, width b and thickness d is equal to

1 Ê p 2 Y b d3 ˆ
6 ÁË l ˜
¯

Solution While making a hoop out of a steel l


band, the central layer remains unstretched. The
layers above the central layer are extended in
length while the lower layers are compressed R
in length. Let R be the radius of the central
b
unstretched layer (shown dotted in Fig. 5.146).
Thus, l = 2pR. d
b
Consider a coaxial layer of radius x. The
d
strain produced in this layer will be
2p x - 2p R Êx ˆ
= Á - 1˜
2p R ËR ¯
Fig. 5.146
Êx ˆ
Hence, stress at this layer = Y ¥ Á - 1˜
ËR ¯
Now, the energy density (u) at his layer is given by
1
u = ¥ Stress ¥ Strain
2
2
1Ï Ê x ˆ ¸Ï x ¸ 1 Êx ˆ
= ÌY ¥ ÁË - 1˜¯ ˝ Ì - 1˝ = Y ÁË - 1˜¯
2Ó R ˛Ó R ˛ 2 R

The energy stored in an elementary volume at x


2
1 Êx ˆ
dU = u ¥ Volume = Y Á - 1˜ {(2 p x dx ) b}
2 ËR ¯
d 2
R+ 1 Êx ˆ
\ U = ÚR-
2
d
2
◊ Y ◊ Á - 1˜ ◊ 2 p x dx b
ËR ¯
2

d
p Y b R+ 2
R2 ÚR - 2
2
= d ( x - R) x dx
912പWŚLJƐŝĐƐĨŽƌ//dͲ:͗DĞĐŚĂŶŝĐƐ//

p Y b È R+ 2 3 ˘
d

R2 ÍÎ ÚR - 2
= Í d ( x - 2 R x 2
+ R 2
x ) dx ˙
˙˚

p Y b d3
= (∵ d << R)
12 R
p Y b d3
= (∵ l = 2pR)
12(l/2p )

1 Ê p 2 Y b d3 ˆ
=
6 ÁË l ˜
¯

Problem 5.93 A sphere of radius 0.1 m and mass 8p kg is attached to the lower end of a
steel wire of length 5.0 m and diameter 10 – 3 m. The wire is suspended from 5.22 m high
ceiling of a room. When the sphere is made to swing as a simple pendulum, it just grazes the
floor at its lowest point. Calculate the velocity of the sphere at the lowest position. Young’s
modulus of the steel is 1.994 ¥ 1011 N/m2.
Solution The situation is shown in Fig. 5.147.
Ceiling
Let DL be the extension of wire at mean position when
oscillating, then
DL = 5.22 – (L + 2r)
5m
= 5.22 – (5 + 2 ¥ 0.1) = 0.02 m
If T is the tension in wire at mean position when
oscillating, then 5.22 cm
T
DL
T/A TL Y A DL
Y = = or T=
DL/L A DL L 2r
r

Y (p r 2 )DL
\ T = Floor
L
Mg
1.994 ¥ 1011 ¥ (p ¥ 5 ¥ 10 - 4 )2 ¥ 0.02
= Fig. 5.147
5
= 199.4 p newton ...(1)
2
Mv
At mean position, T – Mg = ...(2)
R
where R = Radius of circular path of oscillating sphere
= 5.22 – 0.1 = 5.12 m
Mg = 8p kg ¥ 9.8 N/kg = 78.4 p newton
From Eq. (2), we have
(8 p )v 2
(199.4 p – 78.4 p) =
5.12
DĞĐŚĂŶŝĐĂůWƌŽƉĞƌƟĞƐŽĨDĂƩĞƌപ913

121.0 ¥ 5.12
or v2 = = 72.44
8
or v = 8.8 m/s.
Problem 5.94 What internal pressure (in the absence of
an external pressure) can be subtained Ds

(a) by a glass-tube; and


(b) by a glass spherical flask, if in both cases the wall Dr
thickness is equal to Dr = 1.0 mm and the radius
of the tube and the flask equals r = 25 mm?
Solution Df

(a) Consider a transverse section of the tube of length


l that subtends an angle Df at the axis of the tube.
Then a tensile force (acting toward centre)
Df 1 Ds Fig. 5.148
= 2s (Dr ◊ l) sin= 2 s Dr l ◊
2 2 Dr
The pressure acting outwards = p Ds l
1 Ds
In equilibrium: 2 s Dr l ◊ = p Ds ◊ l
2 r
Dr 10 - 3 ¥ 0.05 ¥ 109
or p = s=
r 2.5 ¥ 10 - 2
= 2 ¥ 106 N/m2 ª 20 atms
[1 atm. = 1 atmosphere pressure – 105 N/m2]
(b) Let us divide the spherical flask into two hemispherical shells and consider the
equilibrium of one-half. Since the pressure p acts everywhere normal to the inside
surface, the sum of all forces is p r2 p. In equilibrium, this must be equal to tensile force
= (2p rdr) s.
2 dr
\ p r2 p = 2pr dr s fi p = s = 40 atm
r

Fig. 5.149
914പWŚLJƐŝĐƐĨŽƌ//dͲ:͗DĞĐŚĂŶŝĐƐ//

Problem 5.95 A glass capillary sealed at the upper end is of length 0.11 m and internal
diameter 2 ¥ 10– 5 m. The tube is immersed vertically into a liquid of surface tension 5.06 ¥
10–2 N/m. To what length has the capillary to be immersed so that the liquid level inside
and outside the capillary becomes the same? What will happen to the water level inside the
capillary if the seal is now broken?
Solution If A is the cross-sectional area of the tube and L its length,
the initial volume of air inside it will be V1 = AL while pressure p1 =
p0 = atmospheric pressure. (L – x) P P
2 0
Now when the tube is immersed in water with its length x in water,
the level of water inside and outside is same; so the volume of air in the
tube will be V2 = A(L – x). Further, if p2 is the pressure of gas in the tube.
2T 2T p2 -
2T
p2 - = p 0, i.e., p2 = p0 + r
r r
Now if temperature is constant, Fig. 5.150

P 1V 1 = P 2V 2

È 2T ˘ È rp ˘
p0AL = Í p0 + A(L - x ) or x Í1 + 0 ˙ = L
Î r ˙˚ Î 2T ˚
È 1.012 ¥ 10 5 ¥ 1 ¥ 10 - 5 ˘
i.e., x Í1 + ˙ = 0.11
ÍÎ 2 ¥ 5.06 ¥ 10 - 2 ˙˚
0.11
or x = = 0.01 m
11
If the seal is broken the pressure inside the capillary will become atmospheric, i.e., p0
while capillarity will take place and the rise will be

2T 2 ¥ 5.06 ¥ 10 - 2
h= = -5 = 1.03 m
r r g 10 ¥ 10 3 ¥ 9.8

However, the length of the tube outside the water is 0.11 – 0.01 = 0.1 m; so the tube will
be of insufficient length and so the liquid will rise to the top of the tube and will stay there
with radius of meniscus,
hR 1.03 ¥ 10 5
r= = = 1.03 ¥ 10 – 4 m
L 0.1

Problem 5.96 A conical glass capillary tube of length 0.1 m has diameters 10 –3 and 5 ¥ 10 – 4
m at the ends. When it is just immersed in a liquid at 0°C with larger diameter in contact
with it, the liquid rises to 8 ¥ 10–2 m in the tube. If another cylindrical glass capillary tube
B is immersed in the same liquid at 0°C, the liquid rises to 6 ¥ 10–2 m height. The rise of
liquid in the tube B is only 5.5 ¥ 10–2 m when the liquid is at 50°C. Find the rate at which the
surface tension changes with temperature considering the change to be linear. The density
of the liquid is (1/14) ¥ 104 kg/m3 and angle of contact is zero. Effect of temperature on the
density of liquid and glass is negligible. (IIT Roorkee, 1994)
DĞĐŚĂŶŝĐĂůWƌŽƉĞƌƟĞƐŽĨDĂƩĞƌപ915

Solution If r is the radius of the meniscus in the conical A


r1
B
tube, then as shown in Fig. 5.151, q
r
r - r1 r2 - r1
tan q = =
L-h L L
-4 -4 h
r - 2.5 ¥ 10 (5 - 2.5) ¥ 10
i.e., =
0.1 - 0.08 0.1
r2
i.e., r ¥ 104 – 2.5 = 0.2 ¥ 2.5
i.e., r = 3 ¥ 10 – 4 m
Now as capillarity is independent of the shape of tube so
at same temp. q = 0°C, Fig. 5.151
hArA = hBrB = (2T0/rg) = constant
rB = (0.08 ¥ 3 ¥ 10– 4)/(6 ¥ 10–2) = 4 ¥ 10–4 m
Now as from h = (2T/rrg) for cylindrical tube,
h0 r gr 1 È 1 ˘
T0 = = Í6 ¥ 10 - 2 ¥ ¥ 10 4 ¥ 9.8 ¥ 4 ¥ 10 - 4 ˙
2 2Î 14 ˚
= 8.4 ¥ 10–2 N/m
Now as for a given tube and liquid T μ h (as T = hrgr/2)
T50 h
= 50
T0 h0

5.5 ¥ 10 - 2
So, T50 = -2
¥ 8.4 ¥ 10 - 2 = 7.7 ¥ 10– 2 N/m
6 ¥ 10
So, the rate of change of surface tension with temperature assuming linearity,
DT T - T0
= 50
Dq 50 - 0
(7.7 ¥ 8.4) ¥ 10 - 2
=
50
= –1.4 ¥ 10–2 N/m°C
Negative sign shows that with the rise in temperature surface tension decreases.
Problem 5.97 A body of mass 3.14 kg is suspended from one end of a wire of length 10.0
m. The radius of the wire is changing uniformly from 9.8 ¥ 10– 4 m at one end to 5.0 ¥ 10– 4
m at the other end. Find the change in the length of the wire. What will be the change in
the length if the ends are interchanged? Young’s modulus of the material of the wire is 2 ¥
1011 N/m2.
916പWŚLJƐŝĐƐĨŽƌ//dͲ:͗DĞĐŚĂŶŝĐƐ//

Solution Let L be the length of the wire, suppose r1 and r2 be r1


the radii of the upper and lower ends of the wire respectively.
Now the radius r of the wire at a distance y from the upper end y
is given by
r -r
r = r1 + 2 1 y dy L
L

Stress at a distance y is given by


Mg Mg r2 – r 1
2
= r2
pr p[r1 + (r2 - r1 )y/L]2
Fig. 5.152
Corresponding strain
Stress
=
Young’s modulus
Mg
=
Yp[r1 + (r2 - r1 )y/L]2

Consider a differential element dy of the wire. Extension Dl in this element is


Mg dy
Dl =
Yp[r1 + (r2 - r1 )y/L]2
Total extension in the length L of the wire,
L L Mg dy
l = Ú0 Dl = Ú0 Yp[r1 + (r2 - r1 )y/L]2
3.14 ¥ 9.8 L dy
(2 ¥ 1011 )(3.14) Ú0 [(5.0 ¥ 10 - 4 ) + ( 4.8 ¥ 10 - 4 )y/10.0]2
=

10 dy
= 4.9 ¥ 10 - 3 Ú
0 (5.0 + 0.48 y )2
10
4.9 ¥ 10 - 3
È 1 ˘
= - Í
–3
˙ = 10 m
0.48
Î ( 5 .0 + 0 .48 y ) ˚0
On interchanging the ends, the change in length remains the same.
Problem 5.98 A rod AD consisting of three segments AB, BC and CD joined together is
hanging vertically from a fixed support at A. The lengths of the segments are respectively
0.1 m, 0.2 m and 0.15 m. The cross section of the rod is uniformly 10– 4 m2. A weight of 10
kg is hung from D. Calculate the displacements of points B, C and D if YAB =2.5 ¥ 1010 N/
m2, YBC = 4 ¥ 1010 N/m2 and YCD = 1 ¥ 1010 N/m2. (Neglect the weight of the rod.)
Solution By definition of Young’s modulus,
FL MgL
DL = = [as F = Mg]
AY AY
DĞĐŚĂŶŝĐĂůWƌŽƉĞƌƟĞƐŽĨDĂƩĞƌപ917

10 ¥ 9.8 ¥ 0.1 A
So, for rod AB DL1 =
2.5 ¥ 1010 ¥ 10 - 4
0.1 m Y = 2.5 ¥ 1010
= 3.92 ¥ 10– 6 m B
10 ¥ 9.8 ¥ 0.2 Y = 4 ¥ 1010
and for rod BC DL2 = 0.2 m
4 ¥ 1010 ¥ 10 - 4
= 4.90 ¥ 10– 6 m C
Y = 1 ¥ 1010
10 ¥ 9.8 ¥ 0.15 0.15 m
and for rod CD DL3 =
1 ¥ 1010 ¥ 10 - 4 D
= 14.7 ¥ 10 m–6

So, displacement of B = DL1


= 3.92 ¥ 10– 6 m
10 kg
displacement of C = DL1 + DL2
= 8.82 ¥ 10– 6 m Fig. 5.153
and displacement of D = DL1 + DL2 + DL3
= 23.5 ¥ 10– 6 m
Problem 5.99 Calculate the elongations of (a) a copper wire of 1.4 mm
diameter and (b) an aluminium wire of 1 mm diameter as shown in Fig.
5.153. If Young’s modulii for copper and aluminium are 11 ¥ 1010 N/m2
Aluminium 1m
and 7 ¥ 1010 N/m2 respectively.
FL
Solution By definition of Young’s modulus, DL =
YA M2 4 kg
(a) As tension in copper wire will be due to load of 7 kg, Copper 0.5 m
i.e., TCu = M1g = 7 ¥ gN
M1 7 kg
7 ¥ 9.8 ¥ 0.5
So, (DL)Cu =
(22/7 )(0.7 ¥ 10 - 3 )2 ¥ 11 ¥ 1010 Fig. 5.154

9.8 ¥ 5
i.e., (DL)Cu = ¥ 10 - 3 = 0.2 ¥ 10 – 3 m
22 ¥ 11
(b) However, the tension in the aluminium wire will be
TAl = M2g + TCu = (M2 + M1)g
( 4 + 7 ) ¥ 9.8 ¥ 1
So, (DL)Al =
(22/7 )(0.5 ¥ 10 - 3 )2 ¥ 7 ¥ 1010
= 1.96 ¥ 10 – 3 m
Problem 5.100 A uniform pressure p is exerted on all sides of a solid cube at temperature
t°C. By what amount should the temperature of the cube be raised in order to bring its
volume back to the volume it had before the pressure was applied, if the bulk modulus and
coefficient of volume expansion of the material are B and g respectively.
918പWŚLJƐŝĐƐĨŽƌ//dͲ:͗DĞĐŚĂŶŝĐƐ//

Solution As by definition of bulk modulus B = –V (Dp/DV), with increase in pressure


decrease in volume of the cube will be given by,
p
– DV = V[as Dp = p] ...(i)
B
Now with the rise in temperature due to thermal expansion volume increases, so if Dq is
the rise in temperature,
DV = Vg Dq [as V¢ = V (1 + g Dq)]
As the volume of the cube remains constant,
p p
V = Vg Dq , i.e., Dq =
B gB

Problem 5.101 Calculate the force F needed to punch a 1.46


cm diameter hole in a steel plate 1.27 cm thick (Fig. 5.155). F
The ultimate shear strength of steel is 345 MN/m2.
Solution In punching, shear elasticity is involved, the hole
will be punched if
È F|| ˘
Í ˙ > Ultimate shear stress
Î A˚ Fig. 5.155
i.e., F|| > (Shear stress) ¥ (Area)
So, (F||)min = (3.45 ¥ 108)(2prL) [as here A = 2prL]
i.e., (F||)min = (3.45 ¥ 10 )(2 ¥ 3.14) ¥ (0.73 ¥ 10 )(1.27 ¥ 10 )  200 kN
8 –2 –2

Problem 5.102 Assuming that shear stress at the base of a mountain is equal to the force
per unit area due to its weight, calculate the maximum possible height of a mountain on the
earth if breaking shear stress for a typical rock is 30 ¥ 107 N/m2 and its density 3 ¥ 103 kg/m3.
Solution For a mountain of height h and base area A, weight W = Ahrg. So pressure at the
base due to its own weight will be
W
p= = hrg
A
The mountain will exist if,
hrg < Breaking shear stress

30 ¥ 107
i.e., h <
3 ¥ 10 3 ¥ 10
or (h)max = 10 km
which is nearly the height of Mount Everest!
Problem 5.103 Compute the bulk modulus of water if its volume changes from 100 litres
to 99.5 litres under a pressure of 100 atmosphere. Compare it with that of air.
DĞĐŚĂŶŝĐĂůWƌŽƉĞƌƟĞƐŽĨDĂƩĞƌപ919

Solution By definition of bulk modulus,


Dp (100 ¥ 1.013 ¥ 10 5 )
BW = - V = - 100 ¥
DV (99.5 - 100)
= 2.026 ¥ 109 N/m2
Now as isothermal elasticity of a gas is equal to its pressure,

BA = Eq = P0 = 1.013 ¥ 1015 N/m2


BW k 2.026 ¥ 109 È 1˘
So that = A = = 2 ¥ 10 4 Ías k = ˙
BA kW 1.013 ¥ 10 5 Î B ˚

i.e., bulk modulus of water is very large as compared to air. This means that air is about
20,000 times more compressive than water, i.e., the average distance between air molecules
is much larger than between water molecules.
Problem 5.104 Find the depth of a lake at which the density of water is 1% greater than
at the surface, if compressibility of water is 50 ¥ 10 – 6/atm [take g = 10 m/s2 and 1 atm =
105 N/m2].
Solution The variation of density with pressure is given by r¢ = r[1 + (Dp/B)]. But as Dp
= hrg and (1/B) = k, so
r¢ = r(1 + khrg) or (r¢ – r)/r = khrg
Dr 1 1 1
So, h = ¥ = ¥
r k r g 100 (50 ¥ 10 - 6/10 5 ) ¥ 10 3 ¥ 10
i.e., h = 2 ¥ 103 m = 2 km
Problem 5.105 A solid sphere of radius R made of a material of bulk modulus B is
surrounded by a liquid in a cylindrical container. A massless piston of area A floats on the
surface of the liquid. Find the fractional change in the radius of the sphere (dR/R) when a
mass M is placed on the piston to compress the liquid. (IIT-JEE, 1988)
Solution As for a spherical body,
4 DR 1 DV
V = p R3 , = ...(i)
3 R 3 V
Now by definition of bulk modulus,
Dp DV Dp Mg È Mg ˘
B = -V i.e., = = Ías Dp = A ˙
DV V B AB Î ˚
dR Mg
So, =
R 3 AB

Problem 5.106 A 5 m long cylindrical steel wire with radius 2 ¥ 10–3 m is suspended
vertically from a rigid support and carries a bob of mass 100 kg at the other end. If the bob
gets snapped, calculate the change in temperature of the wire ignoring radiation losses.
920പWŚLJƐŝĐƐĨŽƌ//dͲ:͗DĞĐŚĂŶŝĐƐ//

(For the steel wire: Young’s modulus = 2.1 ¥ 1011 Pa; density 7860 k/m3: Specific heat
capacity = 420 J/kg-K)
Solution Energy stored per unit volume is given by
1
= Stress ¥ Strain
2
When the bob gets snapped, this stored energy is released, thereby raising temperature
of the wire, i.e.,
1
Stress ¥ Strain ¥ Volume = mcDq
2
(mg )2 L
or = (rLA)cDq
2 AY
(mg )2
or Dq = = 0.00457 K
2r A2Yc

Problem 5.107 A load of 31.4 kg is suspended from a wire of radius 10–3 m and density
9 ¥ 103 kg/m3. Calculate the change in temperature of the wire if 75% of the work done is
converted into heat. The Young’s modulus and heat capacity of the material of the wire are
9.8 ¥ 1010 N/m2 and 490 J/kg K respectively.
Solution As work done in stretching an elastic body per unit volume is given by

W 1 1 Stress 2 È Stress ˘
= Stress ¥ Strain = ÍÎas Y = Strain ˙˚
V 2 2 Y

2
1 È Mg ˘ V È F Mg ˘
So, W =
2 ÍÎ A ˙˚ Y Ías stress = A = A ˙
Î ˚

Now according to the given problem H = (75/100)W


2
3 1 È Mg ˘ V
mcDq = ¥ Í ˙ [as H = mcDq]
4 2 ÍÎ p r 2 ˙˚ Y

2
3 1 È Mg ˘ V
or (rV)c Dq = ¥ Í 2 ˙ [as m = rV]
4 2 ÍÎ p r ˙˚ Y

2
3 È 31.4 ¥ 9.8 ˘ 1 1
\ Dq = Í -6 ˙ 10
¥ 3
8 ÍÎ p ¥ 10 ˙˚ 9.8 ¥ 10 9 ¥ 10 ¥ 490

1
i.e., Dq = K = 8.33 ¥ 10 – 3 K
120
DĞĐŚĂŶŝĐĂůWƌŽƉĞƌƟĞƐŽĨDĂƩĞƌപ921

Problem 5.108 A flat plate of area 0.1 m2 is placed on a flat surface and is separated from
it by a film of oil 10– 5 m thick whose coefficient of viscosity is 1.5 Nsm– 2. Calculate the force
required to cause the plate to slide on the surface at a constant speed of 1 mm s–1.
Solution Using formula, we get v = 1 mm s–1

dv d = 10–5 m
F = hA
dy
Rest
Here, h = 1.5 Nsm– 2; A = 0.1 m2
Fig. 5.156
-1
dv v 1 mms 10 - 3 ms - 1
= = -5
= = 10 2 s - 1
dy d 10 m 10 - 5 m

\ F = (1.5)(0.1)(102) = 15 N
Problem 5.109 An oil having a relative density of 0.75 and dynamic viscosity of 8 ¥ 10– 3
Nsm– 2 flows through a horizontal pipe of 2.0 cm diameter and 20 m length. Calculate the
pressure drop in order to maintain the flow of 0.15 litre per second.
Solution According to Poiseuille’s
8hQL 128hQL D
p1 – p2 = = (where R = )
p R4 p D4 2

Here, D = 2.0 cm = 2.0 ¥ 10–2 m


h = 8 ¥ 10– 3 Nsm– 2
L = 20 m
Q = 0.15 l/s = 1.5 ¥ 10– 4 m3s–1
(128)(8 ¥ 10 - 3 )(1.5 ¥ 10 - 4 )(20)
\ p1 – p2 =
(3.14)(2 ¥ 10 - 2 )4

or p1 – p2 = 6.11 ¥ 103 Nm– 2


Problem 5.110 An oil of relative density 0.9 and viscosity 0.12 kg/ms flows through a 2.5
cm diameter pipe with a pressure drop of 38.4 kNm–2 in a length of 30 m.
(a) Determine the discharge
(b) Shear stress at the pipe wall
(c) Power required to maintain the flow.
Solution
(a) Using Poiseuille’s, we get
( p1 - p2 )p D 4
Q=
128h L
Here, p1 – p2 = 38.4 ¥ 103 Nm– 2; D = 2.5 cm = 2.5 ¥ 10–2 m
L = 30 m; h = 1.12 Nsm–2
922പWŚLJƐŝĐƐĨŽƌ//dͲ:͗DĞĐŚĂŶŝĐƐ//

(38.4 ¥ 10 3 )(3.14)(2.5 ¥ 10 - 2 )4
\ Q = = 1.0 ¥ 10– 4 m3/s.
128(0.12)(30)
(b) We know
dp r
t= -
dx 2
At the wall of the pipe, i.e., r = R
dp R Ê p1 - p2 ˆ D
tmax = - =Á ˜
dx 2 Ë L ¯ 4

Ê 38.4 ¥ 10 - 3 ˆ 2.5 ¥ 10 - 2
\ tmax = Á ˜
Ë 30 ¯ 4

= 8 ¥ 10– 6 Nm– 2
(c) We get
128hQ 2 L
P = (p1 – p2)Q
p D4
\ P = (38.4 ¥ 103) (1 ¥ 10–4)
or P = 3.84 W
Problem 5.111 Find the maximum possible mass of a greased needle floating on water
surface.
Solution Let the mass of the needle be m. As the liquid surface is distorted, the surface
tension forces acing on both sides of the needle make an angle q, say, with vertical. Since the
forces acting on the needle are F, F and mg, resolving the forces vertically for its equilibrium,
we have
SFy = F cos q + F cos q – mg = 0
F q F
q
2F cos q
This gives m = m
g
where F = Tl

2Tl cos q mg
Then, m =
g
Fig. 5.157
For m to be maximum, cos q = 1
2Tl
Hence, mmax =
g

Problem 5.112 A film of water is formed between two straight parallel wires each 10
cm long and at separation 0.5 cm. Calculate the work required to increase 1 mm distance
between the wires. Surface tension of water = 72 ¥ 10 –3 N/m.
DĞĐŚĂŶŝĐĂůWƌŽƉĞƌƟĞƐŽĨDĂƩĞƌപ923

Solution Initial surface area = 2 ¥ length ¥ separation


= 2 ¥ 10 cm ¥ 0.5 cm = 10 cm2 = 10 ¥ 10– 4 m2
Final surface area = 2 ¥ 10 cm ¥ (0.5 + 0.1) cm
= 2 ¥ 10 ¥ 0.6 cm2 = 12 ¥ 10– 4 m2
The required work,
W = TDA = 72 ¥ 10 –3 ¥ (12 ¥ 10– 4 – 10 ¥ 10 – 4)J
= 72 ¥ 10 – 3 ¥ 2 ¥ 10– 4 = 144 ¥ 10–7 J
Problem 5.113 A balloon ascends vertically slowly unreeling a long copper wire. Estimate
the amount by which the wire has stretched when 1 km of initially unstretched wire has
been unreeled. The density of copper r is 9.0 ¥ 103 kg/m3 and its Young’s modulus Y = 1.2
¥ 1011 N/m2.
Solution Consider an element of wire of length dx at a height x from the A
ground as shown in Fig. 5.158.
B
Load on dx = weight of the part AB
dx
= (Axr)g
If e be the elongation of element dx, then
x
Stress ( A x r ) g/A
Y = =
Strain eldx
r g x dx A
\ e =
Y Fig. 5.158
r g x dx
Stretched length = dx + e = dx +
Y
Ê r g xˆ
= Á1+ ˜ dx
Ë Y ¯

HÊ r g xˆ
Entire length = Ú0 ÁË 1 +
Y ¯
˜ dx

H
r g È x2 ˘ r g H2
= H+ Í ˙ =H+ ¥
Y Î 2 ˚0 Y 2

9 ¥ 10 3 ¥ 9.8 106
= 10 3 + ¥
1.2 ¥ 1011 2
[∵ H = 1 km = 103 m]
3
= 10 + 0.3675
\ Increase in length = 0.3675 m.
924പWŚLJƐŝĐƐĨŽƌ//dͲ:͗DĞĐŚĂŶŝĐƐ//

NUMERICAL QUESTIONS
Q.1. In a vessel equal mass of alcohol (sp. gravity 0.8) and water are mixed together. A
capillary tube of radius 1 mm is dipped vertically in it. If the mixture rises to a height
5 cm in the capillary tube, then calculate the ST of the mixture; (assuming angle of
contact is 0°).
rhr g
Ans. 2p rT cos q = pr2hr g \ T=
2 cos q
2 r1 r2 2 ¥ 0.8 ¥ 1 1.6 8
r = + = = gm/cm3
r1 + r2 1 + 0.8 1.8 9
8
0.1 ¥ 5 ¥ ¥ 980
\ T = 9 = 217.9 dyne/cm
2
Q.2. A capillary tube of radius r is dipped in water vertically. Calculate the work done by
the force of S.T. if water rises up to height h.
Ans. pr2r g h2.
Q.3. In the above question calculate the amount of heat evolved as water rises in the
capillary tube.
Ans. PE of the water column of height h
1 1
= mgh = p r 2 hr gh
2 2
p r 2 r gh 2
Hence, heat evolved =
2
Q.4. The excess pressure inside a soap bubble is found to be five times that inside another
bubble of the same liquid, compare their volumes.
ÈV Ê r ˆ 3 Ê p ˆ 3 ˘
Ans. 1:125; Í 1 = Á 1 ˜ = Á 2 ˜ ˙
Í V2 Ë r2 ¯ Ë p1 ¯ ˙
Î ˚
Q.5. Calculate the rise of water in a capillary tube of radius 2 mm, assuming the angle of
contact to be 0°; S. T. of water is 72 ¥ 10 –3 N/m. (VITEEE, 2013)
Ans. Here, 2p rT cos q = p r hr g; If q = 0°, cos q = 1
2

2T 2 ¥ 72 ¥ 10 - 3
then, h = = = 7.34 ¥ 10 – 3 metre.
r r g 2 ¥ 10 - 3 ¥ 10 3 ¥ 9.8
Q.6. A capillary tube of radius 2 mm is dipped in mercury kept in a vessel. Calculate the
depression of the mercury in the capillary tube with respect to the level in the vessel
[angle of contact of Hg with glass is 135°, and ST of Hg is 465 dyne/cm.]
2T cos q 2T cos 135∞ 2T cos (90∞ + 45∞)
Ans. Here, h = = =
rrg rrg rrg
2T cos 45∞ 2 ¥ 455 ¥ 1/ 2
= - =- = 24.67 cm.
rrg 0.2 ¥ 13.6 ¥ 9.8
DĞĐŚĂŶŝĐĂůWƌŽƉĞƌƟĞƐŽĨDĂƩĞƌപ925

WORKED PROBLEM
Problem 5.114 Two soap bubbles are stuck together with an intermediate film separating
them. Compute the radius of curvature of this film given that the radii of the bubbles in this
arrangement are r1 and r2 respectively. If r1 > r2 state clearly which way the intermediate
film will bulge. For the case when r1 = r2 = 2 cm, calculate the radius of the bubble formed
by bursting the intermediate film. The volume of a spherical dome of radius R and height
h is p b2 (3R-b)/3.
Solution Excess pressure inside a soap bubble is 4T/r.
If P is the atmospheric pressure
A
4T 4T
P1 = - P , P2 = -P r1 r2
r1 r2

Ê 1 1 ˆ 4T r12
\ P1 – P2 = 4T Á - ˜ =
Ë r1 r2 ¯ r12 P1
P2
B
1 1 1 rr
\ = - fi r12 = 1 2
r12 r1 r2 |r1 - r2| Fig. 5.159

If r1 > r2, P2 > P1 and the intermediate film bulges towards the centre of a circle of radius r1.
If r1 = r2 = 2 cm, the intermediate film is a plane symmetrically positioned. The 3 films
due to the two bubbles and the intermediate film meet at a common circle with diameter
AB. These 3 films produce 3 forces at every points of this circle.
Since the surface tension T is the same for all the films, F1 F2
|F1| = |F2| = |F12| A
Equilibrium at any point such as A is possible only if the r r
three forces are at 120° to one another. 30°
O1 O2
1 C
\ O 1C = r F12
2
1 3
b = r+ r= r Fig. 5.160
2 2
2 3
pÊ3 ˆ Ê 3 ˆ 9p r
Volume of each bubble before bursting = Á r ˜ Á 3r - r ˜¯ =
3Ë2 ¯ Ë 2 8
If R is the radius of the combined bubble after bursting the intermediate film,

4p R3 9p r 3 9p r 3
= 2¥ =
3 8 4

NUMERICAL QUESTIONS
Q.7. A substance breaks down by a stress of 109 Nm–2. If the density of the substance is 3
¥ 103 kg m–3, find the length of the wire made of the same substance, by which it will
break under its own weight when suspended?
Ans. 34 km.
926പWŚLJƐŝĐƐĨŽƌ//dͲ:͗DĞĐŚĂŶŝĐƐ//

Q.8. The star Sirius of mass 7 ¥ 1030 kg is at a distance of 8 ¥ 1016 m from the earth of mass
6 ¥ 1024 kg. Calculate the cross section of a steel cable that can just withstand the
gravitational force between the star and the earth.
[G = 6.67 ¥ 10–11 Nm–2 kg–2, breaking stress = 1010 Nm–2].
F Gm1 m2/r 2 Gm1 m2
Ans. Breaking stress = = =
A A Ar 2
Gm1 m2
\ A = 2
= 44 m2
r (Breaking stress)
Q.9. A fractional change in the volume of oil is 1 per cent when a pressure of 2 ¥ 107 Nm–2
is applied. Calculate the bulk modulus and its compressibility.
Dp 2 ¥ 107
Ans. Bulk modulus B = = = 2 ¥ 109 Nm– 2
DV/V 1/100
1 1
Compressibility = = = 5 ¥ 10–10 m2 N– 1
B 2 ¥ 109
Q.10. A cube of gelatin 7 cm on an edge is resting in a dish. Pushing horizontally on the top
face with a force 0.21 N causes the top to undergo a displacement of 3 mm. Calculate
the rigidity modulus of gelatin.
F/A F F F
Ans. h = = = 2 =
q Aq x ◊ Dx/x x Dx
0.21
= -2 -3
= 1000 Nm– 2
7 ¥ 10 ¥ 3 ¥ 10
Q.11. A long wire hangs vertically with its upper end clamped. A torque of 8 Nm applied
to the free end twists it through 45°. Calculate the PE of the twisted wire.
1 1 p
Ans. PE = tq = ¥8¥ = joule
2 2 4
Q.12. Calculate Poisson’s ratio for silver from the following data: Young’s modulus = 7.25 ¥
1010 Nm– 2, bulk modulus = 11 ¥ 1010 Nm– 2. If a uniform pressure of 108 Nm– 2 acts on
the surface of a silver sphere, determine the fractional change in its volume.
(VITEEE, 2011)
1 Y
Ans. \ Y = 3B (1 – 2s) \ s= - = 0.39
2 6B

p DV p 109 1
Again ∵ B = \ = = =
Ê DV ˆ V B 11 ¥ 1010 110
ÁË ˜
V ¯
Q.13. The limiting stress of a typical human bone is 0.9 ¥ 108 Nm– 2, while Young’s modulus
is 1.4 ¥ 1010 Nm– 2. How much energy can be absorbed by two leg bones (without
breaking) If each has a typical length of 50 cm and an average cross-sectional area of
5 cm2?
DĞĐŚĂŶŝĐĂůWƌŽƉĞƌƟĞƐŽĨDĂƩĞƌപ927

F l l sl
Ans. \ Y = ◊ = s ; Dl =
A Dl Dl Y
1 1 sl 1 lAs2
PE = ¥ F ¥ Dl = ¥ sA ¥ =
2 2 Y 2 Y

1 1 ¥ 5 ¥ 10 - 4 ¥ (0.9 ¥ 108 )2
= ¥ = 145 J.
2 1.4 ¥ 1010
[∵ The leg bone is double \ l = 2 ¥ 50 cm = 1 m]
Two legs together will be able to absorb 290 J.
Q.14. A drop of radius 4 cm is broken into 125 equal small drops. Calculate the work done
if surface tension of water is 75 ¥ 10 –3 Nm.
Ans. Let us suppose that,
R = Radius of the big drop and r = radius of the small drop.
Since volume remains constant, V = nV1
1
4 4
p R3 = n p r 3 \ R = n3r
3 3
Hence, increase in surface area
1
R2 2 2
2 2
= 4p r n – 4pR = 4p 2
n - 4p R = 4p R (n 3 - 1)

Therefore work done n3


1 È 1 ˘
= 4pR2 (n 3 - 1)T = 4p R2 Í(53 ) 3 - 1˙ T = 4p R2 [5 - 1]T = 16p R2T
ÍÎ ˚˙
or W = 6.03 ¥ 10–3 J
Note:
(a) Work done in splitting a drop
È 1 ˘ 2
W = 4p R Ín 3 - 1˙ T
ÍÎ ˙˚
(b) Work done in splitting a bubble
È 1 ˘
W = 2 ¥ 4pR Ín 3 - 1˙ T
2
ÍÎ ˙˚
where,
R = Radius of big drop, n = No. of small drops
Q.15. Calculate the work done in breaking a water drop of radius 4 cm into 64 small drops,
surface tension of water is 75 ¥ 10–3 N/m.
1 1
Ans. W = 4pR2 [n 3 - 1]T = 4p R2 [( 4 3 ) 3 - 1]T
2
Ê 4 ˆ
= 12 pR2T = 12 ¥ p ¥ Á ¥ 75 ¥ 10 - 3 = 4.52 ¥ 10 – 3 joule
Ë 100 ˜¯
928പWŚLJƐŝĐƐĨŽƌ//dͲ:͗DĞĐŚĂŶŝĐƐ//

Q.16. Calculate the work done in breaking a soap-bubble of radius 10 cm into 512 small
bubbles; ST of soap solution is 30 ¥ 10 –3 N/m.
1 1
Ans. W = 2 ¥ 4pR2 [n 3 - 1] T = 2 ¥ 4p R2 [(8 3 ) 3 - 1] T = 56pR2T
or, W = 56 ¥ p (0.1)2 ¥ 30 ¥ 10 –3 = 5.27 ¥ 10 – 2 J
Q.17. Calculate the surface energy released if eight small drops of water each of radius 6 cm
combine to form a big drop.
Ans. Let r and R be the radii of small drop and big drop respectively, then as the volume
remains constant
1
4 4
p R3 = n p r 3 \ R = n 3 r
3 2
Total surface area of n small drops = 4pr2n and surface area of big drop = 4pR2
Total release of surface energy
2
= [4pr2n – 4pR2] T = 4p [r 2 n - n 3 r 2 ] T
2 2
= 4pr2 [n - n 3 ] T = 4p r 2 [8 - (23 ) 3 ] T

= 4pr2 [8 – 4]T = 16pr2 T


Note:
(a) Release of surface energy if n small drops combine to form a single big drop.
È 2˘
= 4p r 2 Ín - n 3 ˙ T
ÍÎ ˙˚
(b) For soap bubble; energy released when n small bubbles combine to form a big
bubble.
È 2˘
= 2 ¥ 4p r 2 Ín - n 3 ˙ T
ÍÎ ˙˚
where
n = No. of small drops, r = radius of small drops.
Q.18. If 216 small identical drops of water combine to form a big drop, then find the ratio
of final surface energy to the total initial surface energy.

È 2 ˘
Í Sf n3 1 ˙
Ans. 1 : 6 ÍHint: = = 1˙
Í S n
n 3 ˙˚
i
Î
Q.19. Suppose that 64 raindrops combine into a single drop. Calculate the ratio of the total
surface energy of the 64 drops to that of a single drop. For water, T = 0.072 nm–1 =
0.072 Jm– 2.
DĞĐŚĂŶŝĐĂůWƌŽƉĞƌƟĞƐŽĨDĂƩĞƌപ929

4 4 4
Ans. p R3 = 64 ¥ p r 3 = p ( 4 r )3 \ R = 4r
3 3 3
\ S1 = 64 ¥ 4pr2 ¥ T, S2 = 4pR2 ¥ T
2
S1 64 ¥ 4p r 2 ¥ T Ê rˆ 64
\ = = 64 Á ˜ = =4
S2 2
4p R ¥ T Ë R¯ 16

Q.20. Calculate the energy required to double the radius of a soap bubble of radius r, the
surface tension of the soap solution is T (assume that temperature remains constant).
Ans. W = 2 ¥ 4p [2r)2 – (r)2]T = 8pr2 [4 – 1]T = 24p r2T
Note:
(a) W = 2 ¥ 4pr2 (n2 – 1)T for a bubble
(b) W = 4pr2 (n2 – 1) T for a drop
where n = no. of times radius is increased.
Q.21. Calculate the work done in order to triple the radius of a water drop of radius 2 cm.
The surface tension of water is 75 dyne/cm.
Ans. W = 4pr2 [n2 – 1]T = 4pr2 ¥ 8T = 32pr2T
= 32 ¥ p(2)2 ¥ 75 = 30159.2 ergs

WORKED PROBLEMS
Problem 5.115 A copper wire of negligible mass, 1 m length and cross-sectional area 10– 6
m2 is kept on a smooth horizontal table with one end fixed. A ball of mass 1 kg is attached
to the other end. The wire and the ball are rotating with an angular velocity of 20 rad/sec. If
the elongation in the wire is 10–3 m, obtain the Young’s modulus. If on increasing the angular
velocity to 100 rad/sec, the wire breaks down, obtain the breaking stress.
Solution According to the given problem, for vertical equilibrium of ball,
R = mg ...(i)
w
and for motion of mass m in a circle of radius r at angular
frequency w in a horizontal plane, the centripetal force
required
F = mrw2 ...(ii) R
O
Here this force is provided by the elasticity of the wire, T
YA mg
i.e., F =T= DL ...(iii)
L
So, equating F from Eqs. (ii) and (iii), Fig. 5.161

mrLw 2 mL2w 2
Y = = [as r = L + DL  L]
A DL A DL
1 ¥ (1)2 ¥ (20)2
So, Y = = 4 ¥ 1011 N/m2
(10 - 6 ) ¥ (10 - 3 )
930പWŚLJƐŝĐƐĨŽƌ//dͲ:͗DĞĐŚĂŶŝĐƐ//

Further as the wire breaks at wmax = 100 rad/s,


Breaking force = mrw2max = 1 ¥ 1 ¥ (100)2 = 104 N
And as cross-section of wire is 10– 6 m2,
Breaking force
Breaking stress =
Area
10 4
= -6
= 1010 N/m2
10
Problem 5.116 A cylindrical vessel of area of cross section A and filled with liquid to a
height of h1 has a capillary tube of length l and radius r protruding horizontally at its bottom.
If the viscosity of liquid is h, density r and g = 9.8 m/s2, find the time in which the level of
water in vessel falls to h2.
Solution Let h be the height of water level in the vessel at instant t which decreases by dh
in time dt.
\ Rate of flow of water through capillary tube,
Ê dh ˆ
V = - AÁ ˜ ...(i)
Ë dt ¯
Further, the rate of flow as given by Poiseuille formula,

p pr 4
V = ...(ii)
8hl

dh pr ghr 4
From Eqs. (i) and (ii), -A = [∵ p = rgh]
dt 8hl

8hlA dh
\ dt = -
pr gr 4 h
Required time is obtained by integrating,
8hlA h2 dh
t = -
pr gr 4 Úh
1 h

8hlA h
= 4
log e 1
pr gr h2

Problem 5.117 A brass bar, having cross- A B C D


sectional area 10 cm2 is subjected to axial 5t 3t 1t 1t
forces as shown in Fig. 5.162. Find the
total elongation of the bar. (Take Y = 8 ¥ 60 cm 100 cm 120 cm
102 t/cm2)
Fig. 5.162
DĞĐŚĂŶŝĐĂůWƌŽƉĞƌƟĞƐŽĨDĂƩĞƌപ931

Solution Given, A = 10 cm2, Y = 8 ¥ 102 t/cm2


Let, Dl = total elongation of the bar. For the sake of simplicity the force of 3t acting at
B may be split into two forces of 5t and 2t as shown in Fig. 5.163. Similarly, the force of 1t
acting at C may be split into two forces of 2t and 1t.
A B B C C D
5t 5t 2t 2t 1t 1t

Fig. 5.163

1
Using the equation, Dl = (F1l1 + F1l2 + F3l3) with usual notations
AY
1
Dl = [5 ¥ 60 + 2 ¥ 100 + 1 ¥ 120]
10 ¥ 8 ¥ 10 2
= 0.0775 cm Ans.
Problem 5.118 Water flows through a capillary tube of radius r and length l at a rate of 40
mL per second, when connected to a pressure difference of h cm of water. Another tube of
the same length but radius r/2 is connected in series with this tube and the combination is
connected to the same pressure head. Calculate the pressure difference across each tube and
the rate of flow of water through the combination.
Solution The volume (quantity) of liquid flowing through a capillary tube per second,

p Pr 4
Q = = 40 ...(i)
8hl

where P : pressure difference at the two ends of tube


l : length of tube
r : radius of tube
h : coefficient of viscosity of liquid.
When tubes are connected in series, the amount of liquid flowing through them is equal,
i.e.,
p P1r 4 p P2 (r/ 2 )4
Q = = ...(ii)
8hl 8hl
and P1 + P2 = P = rgh ...(iii)
From Eq. (ii),
P2
P1 = or P2 = 16P1,
16
Substituting for P2 in Eq. (iii),
rgh
P1 + 16P1 = rgh or P1 =
17
16 rgh
and P2 =
17
932പWŚLJƐŝĐƐĨŽƌ//dͲ:͗DĞĐŚĂŶŝĐƐ//

Now, substituting for P1 or P2 in Eq. (ii),

p ( r gh)r 4 1 p ( r gh)r 4 40
Q= = ¥ = mL/sec
17 ¥ 8hl 17 8hl 17

Problem 5.119 Two equal drops of water are falling through air with a steady velocity v.
If the drops coalesce, what will be the new velocity?
Solution Let r be the radius of each drop. The terminal velocity vT of a drop of radius r is
given by
2 r 2 (s - r ) g
vT = ...(i)
9 h
Now when two drops each of radius r coalesce to form a new drop, the volume of
coalesced drop will be given by
4 4 4
p R3 = p r 3 + p r 3
3 3 3
So, the radius of the coalesced drop will be
R = (2)1/3 r
Hence, the new terminal velocity of the coalesced drop
2 [(2)1/3 r ]2 (s - r ) g
v¢T = ...(ii)
9 h
So dividing Eqs. (ii) by (i)
vT¢
= (2)2/3 or v¢T = (2)2/3 v [as vT = v]
vT

Problem 5.120 A spherical ball of radius 1 ¥ 10 – 4 m and density 104 kg/m3 falls freely
under gravity through a distance h before entering a tank of water. If after entering the water
the velocity of the ball does not change, find h. (The viscosity of water is 9.8 ¥ 10– 6 Ns/m2).
Solution After falling a height h the velocity of the ball will become v = 2gh . As after
entering the water this velocity does not change, this velocity is equal to terminal velocity,
2 2 Ès - r ˘
i.e., 2gh = r g
9 ÍÎ h ˙˚
2
È2 (10 4 - 10 3 ) ¥ 9.8 ˘
or 2gh = Í ¥ (10 - 4 )2 ˙
Î9 9.8 ¥ 10 - 6 ˚

20 ¥ 20
or h =
2 ¥ 9.8
= 20.41 m
+ORQTVCPV
In this problem viscosity of water is given extremely low which is usually 10–3 Ns/m2.
DĞĐŚĂŶŝĐĂůWƌŽƉĞƌƟĞƐŽĨDĂƩĞƌപ933

Problem 5.121 A long capillary tube of radius 0.2 mm is placed vertically inside a beaker
of water.
(a) If the surface tension of water 7.0 ¥ 10–2 Nm–1 and the angle of contact between glass
and water is zero, then determine the height of water column in the tube.
(b) If the tube is now pushed into water so that only 5.0 cm of its length is above the
surface, then determine the angle of contact between the liquid and the glass surface.
Take g = 10 ms–2 .
Solution
(a) Here T = 7.0 ¥ 10–2 Nm– 2; r = 0.2 ¥ 10–3 m; r = 103 kg m– 3
we know that the height of liquid rise in the tube is
2T cos q
h =
rRg
where R is the radius of the meniscus, and q is the angle of contact.
Since q = 0 (given), therefore, radius of the meniscus is equal to the radius of the
capillary tube, i.e., R = r
2(7.0 ¥ 10 - 2 )
\ h = = 0.07 m
(10 3 )(0.2 ¥ 10 - 3 )(10)
or h = 7.0 cm.
(b) When the length of the capillary tube above the free
surface of the liquid is less than the height of liquid p = p0
rise in the tube, then radius R of the free surface is not r
R
equal to the radius of the tube. It is greater than r as q
the surface tends to be flatter, as shown in Fig. 5.164. 5 cm
p0 p0
The pressure difference across the surface is given by p2

2T cos q
Dp =
R
If p1 and p2 are the pressures just above and below the Fig. 5.164
meniscus, then
p1 – p2 = rgh0
2T
\ rgh0 = …(i)
R
In part (a), we have seen that when h0 = h; q = 0 then R = r
2T
and rgh = …(ii)
r
Therefore, dividing Eqs. (i) by (ii), we have
r h
= 0
R h
934പWŚLJƐŝĐƐĨŽƌ//dͲ:͗DĞĐŚĂŶŝĐƐ//

From Fig. 5.164, it is clear that


r
cos q =
R
\ The angle of contact is
h0 Ê 5ˆ
q = cos - 1 = cos - 1 Á ˜ ª 44∞
h Ë 7¯

Problem 5.122 Spherical particles of pollen are shaken up in water and allowed to settle.
The depth of the water is 2 ¥ 10– 2 m. What is the diameter of the largest particles remaining
in suspension after one hour?
Density of pollen = 1.8 ¥ 103 kg m– 3
Viscosity of water = 1 ¥ 10– 2 poise and
Density of water = 1 ¥ 103 kg/m3
Solution For pollen particles not reaching the bottom in 1 hour,

2 ¥ 10 - 2 10 - 4
v£ = m/s
60 ¥ 60 18
Due to viscosity effects, the particles will move with terminal velocity v given by
4 3 4
p r s g = p r 3 rg + 6 phrv
3 3
9hv
r2 =
2 g(s - r )

Substituting, h = 1 ¥ 10– 2 poise = 10–3 Pl, s = 1.8 ¥ 103 kg/m3


10 - 4
r = 1 ¥ 103 kg/m2, g = 10 m/s2 and v = m/s, we have
18

9 ¥ 10 - 3 10 - 4 1
r2 = ¥ ¥
2 18 10(1.8 - 1) ¥ 10 3
\ r = 1.77 ¥ 10– 6 m
and diameter = 2r = 3.54 ¥ 10– 6 m
Problem 5.123 An air bubble of radius 1 mm is allowed to rise through a long cylindrical
column of a viscous liquid of radius 5 cm and travels at a steady rate of 2.1 cm per sec.
If the density of the liquid is 1.47 g per cc, find its viscosity. Assume g = 980 cm/sec2 and
neglect the density of air.
Solution Here due to the force of buoyancy the bubble will move up and so viscous force
which opposes the motion will act downwards and as weight of bubble is zero, in dynamic
equilibrium, Th = F,
DĞĐŚĂŶŝĐĂůWƌŽƉĞƌƟĞƐŽĨDĂƩĞƌപ935

4
i.e., pr3rg = 6phrvT
3
2 rr 2 g 2 1.47 ¥ (0.1)2 ¥ 980
or h = = ¥
9 vT 9 2.1

i.e., h = 1.524 poise


Problem 5.124 A bar of cross section A is subjected to
equal and opposite tensile forces at its ends. Consider a
plane section of the bar whose normal makes an angle q F q F
with the axis of the bar.
(a) What is the tensile stress on the plane?
(b) What is the shearing stress on this plane? Fig. 5.165
(c) For what value of q is the tensile stress maximum?
(d) For what value of q is the shearing stress maximum?
Solution
(a) The resolved part of F along the normal is the tensile force on this Â
A sec q
plane and the resolved part parallel to the plane is the shearing force
on this plane. q
Force F sin q F F
Therefore, tensile stress = = = cos 2 q A q
Area A sec q A
(∵ area of section = A sec q)
Fig. 5.166
Force F sin q F
(b) Shearing stress = = = sin 2q
Area A sec q 2 A
(c) Obviously, tensile stress on the plane is maximum when cos2 q is maximum, that is,
cos q = 1 or q = 0°.
(d) Obviously, shearing stress is maximum when sin 2q is maximum, that is
sin 2q = 1 or 2q = 90°
or q = 45°
Problem 5.125 Anvils made of single crystals of diamond, with the
shape as shown in Fig. 5.167, are used to investigate behaviour of
materials under very high pressures. Flat faces at the narrow end of
the anvil have a diameter of 0.5 mm and the wide ends are subjected
to a compressional force of 50,000 N. What is the pressure at the tip
of the anvil? Diamond
cones Metal
gasket
Solution Here r = 0.25 mm = 0.25 ¥ 10 –3
m
F = 50,000 N
Pressure at the tip of the anvil
Force F 50 , 000
= = =
Area p r 2 3.14 ¥ (0.25 ¥ 10 - 3 )2 Fig. 5.167
= 2.55 ¥ 1011 Nm– 2.
936പWŚLJƐŝĐƐĨŽƌ//dͲ:͗DĞĐŚĂŶŝĐƐ//

Problem 5.126 Two strips of metal are riveted together at their ends by four rivets, each
of diameter 6.0 mm. What is the maximum tension that can be exerted by the riveted strip
if the shearing stress on the rivet does not exceed 2.3 ¥ 109 Pa? Assume that each rivet is to
carry one quarter of the load.
Solution Let the tension exerted by riveted strip be F.
This tension would provide shearing force on the four rivets, which share it equally.
F
\ Shearing force on each rivet =
4
F/4 F
and shearing stress on each rivet = =
A 4A

Fig. 5.168

As the maximum shearing stress on each rivet is given to be 2.3 ¥ 109 Pa, so we have
Fmax
= 2.3 ¥ 109
4A
or Fmax = 4A ¥ 2.3 ¥ 109 = 4 ¥ pr2 ¥ 2.3 ¥ 109
22
=4¥ ¥ (3.0 ¥ 10– 3)2 ¥ 2.3 ¥ 109
7
= 260.2 ¥ 103 N = 260 kN.
Problem 5.127 If a 5 cm long capillary tube with 0.1 mm internal diameter opens at both
ends is slightly dipped in water having surface tension 75 dyne cm– 1. State whether (i) water
will rise half way in the capillary, (ii) water will rise up to the upper end of capillary, and
(iii) water will overflow out of the upper end of capillary. Explain your answer.
0.1
Solution Radius, r = mm = 0.05 mm = 0.005 cm
2
Surface tension, T = 75 dyne cm– 1
density, r = 1 gcm– 3; angle of contact, q = 0°
Let h be the height to which water rises in the capillary tube. Then
2T cos q 2 ¥ 75 ¥ cos 0∞
h= = = 30.58 cm.
rrg 0.005 ¥ 1 ¥ 981
DĞĐŚĂŶŝĐĂůWƌŽƉĞƌƟĞƐŽĨDĂƩĞƌപ937

Given length of capillary tube, h¢ = 5 cm



(i) As h > , so the first possibility is ruled out.
2
(ii) As the tube is of insufficient length, so the water will rise up to the upper end of the
tube.
(iii) The water will not overflow out of the upper end of the capillary. It will rise only up to
the upper end of the capillary. The liquid meniscus will adjust its radius of curvature
R¢ in such a way that
È 2T ˘
R¢h¢ = Rh Í∵ hR = = costant ˙
Î rg ˚
where R is the radius of curvature that the liquid meniscus would possess if the
capillary tube were of sufficient length.
Rh rh 0.005 ¥ 30.58
\ R¢ = = = = 0.0306 cm.
h¢ h¢ 5
È r r ˘
Í∵ R = = = r˙
Î cos q cos 0∞ ˚

DAILY PRACTICE PROBLEMS FOR JEE MAIN AND ADVANCE

Level 1 (c) 1.015 ¥ 105 Pa


1. A wire elongates by l mm when a (d) 1.55 ¥ 106 Pa
load W is hanged from it. If the wire 4. A wire suspended vertically from one
goes over a pulley and two weights of the ends is stretched by attaching a
W each are hung at the two ends, the weight of 200 N to the lower end. If the
elongation of the wire (in mm) will be weight stretches the wire by 1 mm, the
(VITEEE, 2001) elastic energy stored in the wire is
(a) l (b) 2l (WBJEE, 2011)
(c) zero (d) l/2 (a) 20 J (b) 10 J
2. If s is stress and Y is Young’s modulus (c) 0.2 J (d) 0.1 J
of the material of a wire. The energy 5. A wire of length L and cross-sectional
stored in the wire per unit volume is area A is made of a material of Young’s
modulus Y. If the wire is stretched by
s2 the amount x, the work done is
(a) (b) 2s2Y
2Y (IIT-JEE, 2001)
2Y 1 2 YAx 2 YAx 2
(c) (d) Ys (a) (b)
s2 2 2L L
YAx
3. When the pressure on a fluid is (c) (d) YAx2L
2L
changed from 1.01 ¥ 105 Pa to 1.65 ¥
6. The end of a uniform wire of length L
105 Pa, the volume changes by 10%,
and of weight W is attached rigidly to
the bulk modulus of the fluid is
a point in the roof and a weight W1 is
(a) 1.55 ¥ 105 Pa suspended from its lower end. If s is
(b) 0.015 ¥ 105 Pa the area of cross section of the wire, the
938പWŚLJƐŝĐƐĨŽƌ//dͲ:͗DĞĐŚĂŶŝĐƐ//

3L 10. A steel ring of radius r and cross-


stress in the wire at a height from sectional area A is fitted on to a wooden
its lower end is 4
disc of radius R (R > r). If Young’s
W modulus of steel is Y, then the force
W1 W1 +
(a) (b) 4 with which the steel ring expands is
s s
R R-r
3W (a) AY (b) AY
W1 + W1 + W
r r
(c) 4 (d)
s s Y Ê R - rˆ Yr
(c) Á ˜ (d)
7. An elastic string of unstretched length AË r ¯ AR
L and force constant k is stretched by 11. Two rods of different materials having
a small length x. It is further stretched coefficient of linear expansion a1, a2
by another small length y. The work and Young’s modulii Y1, Y2 respectively
done in the second stretching is are fixed between two rigid massive
1 2 walls. The rods are heated such that
(a) ky they undergo the same increase in
2
temperature. There is no bending of
1
(b) k (x2 + y2) the rods. If a1:a2 = 2:3, the thermal
2 stresses developed in the two rods are
1 equal provided Y1:Y2 is equal to
(c) k(x + y)2
2 (JEE, 1989 S)
1 (a) 2 : 3 (b) 1 : 1
(d) ky(2x + y) (c) 3 : 2 (d) 4 : 9
2
12. The stress versus strain graphs for
8. The ratio (stress/strain) remains wires of two materials A and B are
constant for a small deformation of as shown in the figure. If YA and YB
a material. When the deformation is are the Young’s
Stress

made larger, this ratio (MHCET, 2011) moduli of the


(a) increases materials, then A
(b) decreases
(c) remains constant (a) YB = 2YA B
(d) becomes zero
(b) YA = YB
9. Two rods A and B of the same 30° 60°
O Strain
material and length have radii r1 and (c) YB = 3YA
r2 respectively. When they are rigidly
fixed at one end and twisted by the (d) YA = 3YB
same couple supplied at the other end, 13. To break a wire, a force of 106 N/m2 is
the ratio: required. If the density of the material
(Angle of twist at end of A) is 3 ¥ 103 kg/m3, then the length of
is the wire which will break by its own
(Angle of twist at end of B)
weight will be
(a) r12/r22 (b) r23/r12 (a) 34 m (b) 30 m
(c) r24/r14 (d) r14/r24 (c) 300 m (d) 3 m
14. The value of Poisson’s ratio lies
between (VITEEE, 2009)
DĞĐŚĂŶŝĐĂůWƌŽƉĞƌƟĞƐŽĨDĂƩĞƌപ939

1 3 1 allow for appropriate safety factors.


(a) –1 to (b) - to - Its maximum upward acceleration is
2 4 2
1.5 m/s2. If the cable has to support
1 the total weight of 2000 kg of a loaded
(c) - to 1 (d) 1 to 2
2 elevator, the area of cross section of the
15. The isothermal bulk modulus of a gas cable should be
at atmospheric pressure is (a) 3.28 cm2 (b) 2.38 cm2
2
(a) 1 mm of Hg (c) 0.328 cm (d) 8.23 cm2
(b) 13.6 mm of Hg 21. The diagram shows stress v/s strain
curve for the materials A and B. From
(c) 1.013 ¥ 105 N/m2
the curves we infer that
(d) 2.026 ¥ 105 N/m2 (JEE Adv., 2015)
16. When a force is applied on a wire of
uniform cross-sectional area 3 ¥ 10–6 A
m2 and length 4 m, the increase in

Stress
length is 1 mm. Energy stored in it will B
be (Y = 2 ¥ 1011 N/m2)
(a) 6250 J (b) 0.177 J Strain
(c) 0.075 J (d) 0.150 J
17. A rod fixed between two points at 20°C. (a) A is brittle but B is ductile
The coefficient of linear expansion of (b) A is ductile and B is brittle
material of rod is 1.1 ¥ 10–5/°C and (c) Both A and B are ductile
Young’s modulus is 1.2 ¥ 1011 N/m2. (d) Both A and B are brittle
Find the stress developed in the rod if 22. The potential energy U between two
temperature of rod becomes 10°C molecules as a function of the distance
(a) 1.32 ¥ 107 N/m2 x between them has been shown in the
(b) 1.10 ¥ 1015 N/m2 figure. The two molecules are
(c) 1.32 ¥ 108 N/m2 U
(d) 1.10 ¥ 106 N/m2
18. The breaking stress of a wire depends
0 x
upon
(a) length of the wire
A B C
(b) radius of the wire
(c) material of the wire (a) attracted when x lies between A
(d) shape of the cross section and B and are repelled when x lies
19. The temperature of a wire of length I between B and C
metre and area of cross section 1 cm2 is (b) attracted when x lies between B
increased from 0°C to 100°C. If the rod and C and are repelled when x lies
is not allowed to increase in length, the between A and B
force required will be (a = 10–5/°C and (c) attracted when they reach B
Y = 1011 N/m2) (d) repelled when they reach B
(a) 103 N (b) 104 N 23. The adjacent graph shows the extension
5 (D l) of a wire of length 1 m suspended
(c) 10 N (d) 109 N
from the top of a roof at one end with
20. An elevator cable is to have a
a load W connected to the other end.
maximum stress of 7 ¥ 107 N/m2 to
If the cross-sectional area of the wire is
940പWŚLJƐŝĐƐĨŽƌ//dͲ:͗DĞĐŚĂŶŝĐƐ//

10–6 m2, calculate the Young’s modulus (a) becomes concave towards A
of the material of the wire (JEE, 2003 S) (b) becomes convex towards A
Dl( ¥ 104) m
(c) remains in the initial position
(d) either (a) or (b) depending on the
4 size of A w.r.t. B
3
26. A thin metal disc of radius r floats on
2
water surface and bends the surface
1
downwards along the perimeter
20 40 60 80 W(N) making an angle q with vertical edge
of the disc. If the disc displaces a
(a) 2 ¥ 1011 N/m2 weight of water W and surface tension
(b) 2 ¥ 10–11 N/m2 of water is T, then the weight of metal
(c) 3 ¥ 10–12 N/m2 disc is
(d) 2 ¥ 10–13 N/m2 (a) 2 prT + W
24. The diagram shows a force-extension (b) 2 prT cos q – W
graph for a rubber band. Consider the (c) 2 prT cos q + W
following statements
(d) W – 2 prT cos q
I. It will be easier to compress this
27. The terminal speed of a sphere of gold
rubber than expand it
(density = 19.5 kg/m3) is 0.2 m/s in a
II. Rubber does not return to its
viscous liquid (density = 1.5 kg/m3),
original length after it is stretched.
find the terminal speed of sphere of
III. The rubber band will get heated if
a silver (density = 10.5 kg/m3) of the
it is stretched and released.
same size in the same liquid
Which of these can be deduced from
(WBJEE, 2006)
the graph?
(a) 0.4 m/s (b) 0.133 m/s
(c) 0.1 m/s (d) 0.2 m/s
Extension

28. Water flows in a streamlined manner


through a capillary tube of radius a.
The pressure difference is p and rate
Force of flow is Q. If the radius is reduced
(a) III only (b) II and III to a/2 and the pressure is increased to
(c) I and III (d) I only 2p, then the rate of flow becomes
25. A thread is tied slightly loose to a wire (a) 4Q (b) Q
frame as in figure and the frame is
Q Q
dipped into a soap solution and taken (c) (d)
out. The frame is completely covered 2 g
with the film. When the portion A is 29. Spherical balls of radius R are falling
punctured with a pin, the thread in a viscous fluid of viscosity h with a
velocity v. The retarding viscous force
A Frame acting on the spherical ball is
(a) directly proportional to radius
B R but inversely proportional to
Thread
velocity v
(b) directly proportional to both
radius R and velocity v
DĞĐŚĂŶŝĐĂůWƌŽƉĞƌƟĞƐŽĨDĂƩĞƌപ941

(c) inversely proportional to both remain in the capillary then the surface
radius R and velocity v tension of the liquid will be
(d) inversely proportional to radius (a) 5 ¥ 10–3 N/m
R but inversely proportional to (b) 5 ¥ 10–2 N/m
velocity v (c) 5 N/m
30. An isolated and charged spherical (d) 50 N/m
soap bubble has a radius r and the 33. In a capillary tube experiment, a
pressure inside is atmospheric. If T is vertical 30 cm long capillary tube is
the surface tension of soap solution, dipped in water. The water rises up
then charge on drop is to a height of 10 cm due to capillary
action. If this experiment is conducted
2rT in a freely falling elevator. The length
(a) 2
e0 of the water column becomes
(a) 10 cm (b) 20 cm
(b) 8p r 2rTe 0 (c) 30 cm (d) zero
(c) 8p r rTe 0 34. Water rises in a vertical capillary tube
up to a height of 2.0 cm. If the tube
2rT is inclined at an angle of 60°with the
(d) 8p r vertical, then up to what length the
e0
water will rise in the tube
31. A cubical block of side a and density r (BITSAT, 2011)
slides over a fixed inclined plane with
(a) 2.0 cm (b) 4.0 cm
constant velocity v.
4
(c) cm (d) 2 2 cm
3
35. The liquid meniscus in capillary tube
will be convex, if the angle of contact
q is
(a) greater than 90°
There is a thin film of viscous fluid of (b) less than 90°
thickness t between the plane and the (c) equal to 90°
block. Then the coefficient of viscosity (d) equal to 0°
of the thin film will be 36. A liquid does not wet the sides of a
solid, if the angle of contact is
r agt sin q (a) zero
(a) h =
v (b) obtuse (more than 90°)
(c) acute (less than 90°)
r gt sin q
(b) (d) 90°
av 37. Two droplets merge with each other
v and forms a large droplet. In this
(c) r agt sin q process
(a) energy is liberated
(d) None of these (b) energy is absorbed
32. Radius of a capillary is 2 ¥ 10–3 m. A (c) neither liberated nor absorbed
liquid of weight 6.28 ¥ 10–4 N may (d) some mass is converted into
energy
942പWŚLJƐŝĐƐĨŽƌ//dͲ:͗DĞĐŚĂŶŝĐƐ//

38. A square frame of side L is dipped in (a) h


a liquid. On taking out, a membrane is (b) h6
formed. If the surface tension of liquid (c) 6h
is T, the force acting on the frame will (d) information is insufficient
be 43. In a surface tension experiment with a
(a) 2 TL (b) 4 TL capillary tube water rises up to 0.1 m.
(c) 8 TL (d) 10 TL If the same experiment is repeated on
an artificial satellite, which is revolving
39. The correct curve between the height
around the earth, water will rise in the
or depression h of liquid in a capillary
capillary tube up to a height of
tube and its radius is
(a) 1.0 m
h h
(b) 0.2 m
(a) (b) (c) 0.98 m
(d) full length of capillary tube
r r 44. The work done in increasing the size
h h of a soap film from 10 cm ¥ 6 cm to 10
cm ¥ 11 cm is 3 ¥ 10–4 joule. The surface
(c) (d)
tension of the film is
r r (a) 1.5 ¥ 10–2 N/m
40. A soap film is slowly blown under (b) 3.0 ¥ 10–2 N/m
isothermal conditions in air at NTP to (c) 6.0 ¥ 10–2 N/m
a radius R. How much work is done? (d) 11.0 ¥ 10–2 N/m
Surface tension of film is T:
Level 2
(a) 4 pR2 T (b) 8 pR2T
2 1. The velocity of small ball of mass M
(c) 18 pR T (d) Zero
and density d1 when dropped in a
41. A soap bubble is very slowly blown on container filled with glycerine becomes
the end of a glass tube by a mechanical constant after sometime. If the density
pump which supplies a fixed volume of glycerine is d2, the viscous force
per minute whatever the internal pres- acting on the ball will be
sure may be. Which graph represents
Md1 g Ê d ˆ
the variation of excess pressure p in- (a) (b) Mg Á 1 - 2 ˜
side the bubble with time: d2 Ë d1 ¯
p p M(d1 + d2 )
(c) (d) md1d2
g
(a) (b)
2. The table gives the areas of the four
t t surfaces and the magnitude of a force
p p that is applied perpendicular to the
surface and uniformly across it. Which
(c) (d) surface(s) has greatest stress
t t Surface Area Force
42. What will be the height of the liquid A 0.25 A F
column in the capillary on the surface B 0.50 A 2F
of moon if it is h on the surface of the C 2A 4F
earth D 3A 5F
DĞĐŚĂŶŝĐĂůWƌŽƉĞƌƟĞƐŽĨDĂƩĞƌപ943

(a) A (b) A, B (a) Zero (b) l + h


(c) B, C (d) D (c) 2h (d) h
3. Four cylindrical rods are stretched by 7. A capillary tube (A) is dipped in water.
applying forces at their ends. The force Another identical tube (B) is dipped
magnitudes, the areas of end faces, in a soap-water solution. Which of the
the changes in length, and the initial following shows the relative nature of
lengths are given in the table. The rod the liquid columns in the two tubes.
which is of greatest Young’s modulus (JEE, 2008)
is
Length Initial A B
Rod Force Area
change length (a)
1 F A DL L
2 2F 2A 2DL L
3 F 2A 2DL 2L A B
4 3F 1.5 A DL 2L (b)
4. A solid sphere of radius R made
of material of bulk modulus K is
surrounded by a liquid in a cylindrical A B
container. A massless piston of area
A floats on the surface of the liquid (c)
when a mass m is placed on the piston
to compress the liquid, the fractional
change in the radius of the sphere A B
dR/R is (JEE, 1988, 2018)
mg mg (d)
(a) (b)
AK 3 AK
mg 3mg
(c) (d) 8. If the ratio of lengths, radii and
A AK
Young’s modulus of steel and brass
5. A uniform cylindrical wire (Young’s wires shown in the figure are a, b, and
modulus 2 ¥ 1011 N/m2) is subjected c, respectively. The ratio between the
to a longitudinal tensile stress of 5 ¥ increase in length of steel and brass
107 N-m2. If the overall volume change wires would be
in the wire is 0.02%, the fractional
decrease in the radius of the wire, is
Brass
(a) 1.5 ¥ 10–4 (b) 1.0 ¥ 10–4
(c) 0.5 ¥ 10 –4
(d) 0.25 ¥ 10–4 2 kg
6. By inserting a long capillary tube up
to a depth I in water, the water rises Steel
to a height h. If the lower end of the
capillary is closed inside water and the 2 kg
capillary is taken out and closed end
opened, to what height the water will b2 a bc
(a) (b)
remain in the tube 2c 2a2
944പWŚLJƐŝĐƐĨŽƌ//dͲ:͗DĞĐŚĂŶŝĐƐ//

ba 2 a 12. The length of an elastic string is a


(c) (d) metre when the longitudinal tension is
2c 2b 2c 4 N and b metre when the longitudinal
9. A student performs an experiment to tension is 5 N. The length of the string
determine the Young’s modulus of in metre when the longitudinal tension
a wire, exactly 2 m long, by Searle’s is 9 N is
method increase in the length of the (a) a – b (b) 5b – 4a
wire to be 0.8 mm with an uncertainly 1
of ± 0.05 mm at a load of exactly 1.0 (c) 2b – a (d) 4a – 3b
4
kg. The student also measures of
diameter of the wire to be 0.4 mm with 13. A wire of length L and radius r is
an uncertainty of ± 0.01 mm. Take g = rigidly fixed at one end. On stretching
9.8 m/s2 (exact). The Young’s modulus the other end of the wire with a force F,
obtained from the reading is the increase in its length is l. If another
(a) (2.0 ± 0.3) ¥ 1011 N/m2 wire of same material but of length 2L
and radius 2 r is stretched with a force
(b) (2.0 ± 0.2) ¥ 1011 N/m2
of 2F, the increase in its length will be
(c) (2.0 ± 0.1) ¥ 1011 N/m2 (VITEEE, 2013)
(d) (2.0 ± 0.05) ¥ 1011 N/m2
(a) l (b) 2l
10. A light rod of length 2 m suspended
l 1
from the ceiling horizontally by means (c) (d)
of two vertical wires of equal length. 2 4
A weight W is hung from a light rod 14. A capillary tube of radius R is
as shown in figure. The rod hung by immersed in water and water rises in
means of a steel wire of cross-sectional it to a height H. Mass of water in the
area A1 = 0.1 cm2 and brass wire of capillary tube is M. If the radius of the
cross-sectional area A2 = 0.2 cm2. To tube is doubled, mass of water that
have equal stress in both wires, T1/ will rise in the capillary tube will now
T2 = be (WBJEE, 2016)
(a) M (b) 2 M
(c) M/2 (d) 4 M
T1 T2
15. A glass capillary tube of inner diameter
Steel Brass
2m 0.28 mm is lowered vertically into
water in a vessel. The pressure to be
W applied on the water in the tube so that
water level in the tube is same as that in
(a) 1/3 (b) 1/4
the vessel in N/m2 is (surface tension
(c) 4/3 (d) 1/2
of water = 0.07 N/m, atmospheric
11. A rod of length l and radius r is joined
pressure = 105 N/m2)
to a rod of length l/2 and radius r/2
of same material. The free end of small (a) 103 (b) 99 ¥ 103
rod is fixed to a rigid base and the free (c) 100 ¥ 10 3
(d) 101 ¥ 103
end of larger rod is given a twist of q°, 16. A cylinder with movable piston con-
the twist angle at the joint will be tains air under pressure P0 and a soap
(a) q/4 (b) q/2 bubble of radius r. The surface tension
(c) 5q/6 (d) 8q/9 of soap solution is T and the temper-
ature of the system is kept constant.
The pressure to which the air should
DĞĐŚĂŶŝĐĂůWƌŽƉĞƌƟĞƐŽĨDĂƩĞƌപ945

be compressed by slowly pushing the 2T Ê 1 1 ˆ T Ê 1 1ˆ


piston into the cylinder for the soap (a) - (b) -
r g ÁË r2 r1 ˜¯ r g ÁË r1 r2 ˜¯
bubble to reduce its size by half is
È 3T ˘ È T˘ 2T Ê 1 1 ˆ T Ê 1 1ˆ
(a) 8 ÍP0 + (b) ÍP0 + ˙
Î r ˙˚ Î r˚ (c) +
r g ÁË r1 r2 ˜¯
(d) +
r g ÁË r1 r2 ˜¯
È T˘ È 7T ˘
(c) 8 ÍP0 + ˙ (d) 8 ÍP0 + 19. A large number of droplets, each of
Î r ˚ Î r ˙˚ radius a, coalesce to form a bigger drop
17. A thin movable plate is separated from of radius b. Assume that the energy
two fixed plates P1 and P2 by two released in the process is converted
highly viscous liquids of coefficient of into the kinetic energy of the drop.
viscosity n1 and n2 as shown, where n2 The velocity of the drop is (S = surface
= 9n1. Area of contact of movable plate tension and r = density of drop)
with each fluid is same. If the distance
1/2
between two fixed plates is h, then È 3S Ê 1 1 ˆ ˘
the distance h1 of movable plate from (a) Í Á - ˜ ˙
Î r Ë a b¯ ˚
upper plates such that movable plate
can be moved with a finite velocity by 1/2
È 6S Ê 1 1 ˆ ˘
applying the minimum possible force (b) Í Á - ˜ ˙
on movable plate is (assume only linear Î r Ë a b¯ ˚
velocity distribution in each liquid) 1/2
È 2S Ê 1 1 ˆ ˘
P1 (c) Í Á - ˜ ˙
n1 Î r Ë a b¯ ˚
h 1/2
n2 È S Ê 1 1ˆ ˘
(d) Í Á - ˜ ˙
P2
Î r Ë a b¯ ˚
20. Drops of liquid of density r are floating
h h half immersed in a liquid of density s.
(a) (b)
4 2 If the surface tension of liquid is T, the
h h radius of the drop will be
(c) (d)
6 3
3T 6T
18. In the figure shown, radius of the (a) (b)
g(3 r - s ) g(2 r - s )
limbs of the manometer are r1 and r2
(< r1). The surface tension of the liquid
is T. The difference of the heights of 3T 3T
(c) (d)
the liquid column h is equal to (assume g(2 r - s ) g( 4 r - 3s )
that the angle of contact is zero).
21. A glass tube of uniform internal radius
(r) has a valve separating the two
r1 r2
identical ends. Initially, the valve is in
h a tightly closed position End 1 has a
A B
hemispherical soap bubble of radius r.
End 2 has hemispherical soap bubble
as shown in figure. Just after opening
the valve (IIT-JEE, 2008)
946പWŚLJƐŝĐƐĨŽƌ//dͲ:͗DĞĐŚĂŶŝĐƐ//

(d) heat is produced during elonga-


tion.
2. When a wire is stretched to double its
length
2 1
(a) strain is unity
(b) stress is equal to Young’s modulus
(a) air from end 1 flows towards end of elasticity
2. No change in the volume of the (c) its radius is halved
soap bubbles (d) Young’s modulus is equal to twice
(b) air from end 1 flows towards end the elastic potential energy per
2. Volume of the soap bubble at unit volume.
end 1 decreases 3. A metal wire of length L is suspended
(c) no change occurs vertically from a rigid support. When
(d) air from end 2 flows towards end a body of mass M is attached to the
1. Volume of the soap bubble at lower end of the wire, the elongation
end 1 increases. of the wire is l.
22. Two soap bubbles A and B are kept (a) The loss is gravitational potential
in a closed chamber where the air is energy of mass M is Mgl
maintained at pressure 8 N/m2. The (b) The elastic potential energy stored
radii of bubbles A and B are 2 cm ad 4 in the wire is Mgl
cm, respectively. Surface tension of the (c) The elastic potential energy stored
soap-water used to make bubbles is 1
0.04 N/m. Find the ratio nB/nA where in the wire is Mgl
2
nA and nB are the number of moles of 1
air in bubbles A and B, respectively. (d) Heat produced is Mgl
2
[Neglect the effect of gravity].
4. The wires A and B shown in figure, are
(IIT-JEE, 2009)
made of the same material and have
(a) 2 (b) 9 radii rA and rB respectively. A block of
(c) 8 (d) 6 mass m is connected between them.
When a force F is mg/3, one of the
Multiple Correct Options wires breaks.
1. A metal wire of length L, area of cross (a) A will break before B if
section A and Young’s modulus Y is rA < 2rB
stretched by a variable force F such (b) A will break before B if A
that F is always slightly greater than rA = rB
m
the elastic forces of resistance in the (c) Either A or B will
wire. When the elongation of the wire break if rA = 2rB B
is l (d) The lengths of A and
YAl 2 B must be known to
(a) the work done by F is
L decide which wire will F
break
YAl 2
(b) the work done by F is 5. A vertical bar of uniform section is fixed
2L at both its ends and a load W = 5000
(c) the elastic potential energy stored N is applied axially at an intermediate
section as shown in figure. Choose the
YAl 2
in the wire is correct options.
2L
DĞĐŚĂŶŝĐĂůWƌŽƉĞƌƟĞƐŽĨDĂƩĞƌപ947

8. Two identical rods each of cross-


sectional area A are placed on a smooth
2l
horizontal surface. These are acted
upon by forces as shown in figure. The
breaking strength of material of each
W
3l rod is F/A, then
A B
2F
(a) Reaction at the top support is
3000 N C D
(b) Reaction at the bottom support is F F
2000 N
(c) Reaction at the top support is (a) rod AB will break left of the centre
1000 N (b) rod AB will break right of the
(d) Reaction at the bottom support is centre
3000 N (c) rod CD will break left of the centre
6. Four rods, A, B, C and D of the same (d) rod CD will break right of the
length and material but of different centre
radii r, r 2 , r 3 and 2r respectively 9. An oil drop falls through air with a
are held between two rigid walls. The terminal velocity of 5 ¥ 10–4 m/s.
temperature of all rods is increased Viscosity of oil is 1.8 ¥ 10–5 N-s/m2 and
through the same range. If the rods density of oil is 900 kg/m3. Neglecting
do not bend, then density of air as compared to that of
(a) the stress in the rods A, B, C and the oil
D are in the ratio 1 : 2 : 3 : 4 (a) radius of the drop is 6.20 ¥ 10–2 m
(b) the forces on them exerted by the (b) radius of the drop is 2.14 ¥ 10–6 m
wall are in the ratio 1 : 2 : 3 : 4 (c) terminal velocity of the drop at
(c) the energy stored in the rods due half of this radius is 1.25 ¥ 10–4
to elasticity are in the ratio 1 : 2 : m/s
3:4 (d) terminal velocity of the drop at
(d) the strains produced in the rods half of this radius is 2.5 ¥ 10–4 m/s
are in the ratio 1 : 2 : 3 : 4 10. The viscous force acting on a solid ball
7. Two wires A and B have equal lengths of surface area A moving with terminal
and are made of the same material, but velocity v is proportional to
the diameter of A is twice that of wire (a) A (b) A1/2
B. Then, for a given load
(c) v (d) v1/2
(a) the extension of B will be four
times that of A 11. Choose the correct option(s)
(b) the extension of A and B will be (a) With the rise in temperature
equal viscosity decreases.
(c) the strain in B is four times that in (b) Viscosity of liquid increases with
A increase in pressure but in case
(d) the strains in A and B will be of water, viscosity decreases with
equal rise in pressure.
948പWŚLJƐŝĐƐĨŽƌ//dͲ:͗DĞĐŚĂŶŝĐƐ//

(c) Viscosity of liquid is about 100 16. When a capillary tube is dipped into a
times greater than that of gases. liquid, the liquid neither rises nor falls
(d) Viscosity of gases is independent in the capillary. The
of pressure. (a) surface tension may be zero.
12. When a capillary tube is dipped in a (b) surface tension of the liquid must
liquid, the liquid rises up to a height be zero
h in the tube. The free liquid surface (c) contact angle may be 90°
inside the tube is hemispherical in (d) none of these
shape. The tube is now pushed down 17. The contact angle between a solid and
so that the height of the tube outside a liquid is a property of the
the liquid is less than h. (a) material of the solid
(a) The liquid will ooze out of the (b) material of the liquid
tube slowly. (c) shape of the solid
(b) The liquid will come out of the (d) mass of the solid
tube like in a small fountain. 18. The capillary rise of water in a tube
(c) The free liquid surface inside the depends on the
tube will not be hemispherical. (a) outer radius of the tube
(d) The liquid will fill the tube but not (b) inner radius of the tube
come out of its upper end. (c) maternal of the tube
13. The rise of liquid in a capillary tube (d) length of the tube
depends on 19. An air bubble in a water tank rises
(a) the material of the tube and nature from the bottom to the top. Which of
of liquid the following statements are true?
(b) the length of tube (a) Bubble rises upwards because
(c) the outer radius pressure at the bottom is less than
(d) the inner radius of the tube that at the top.
14. When a drop splits up into a number (b) Bubble rises upwards because
of drops pressure at the bottom is greater
(a) area increases than that at the top.
(b) volume increases (c) As the bubble rises, its size
(c) energy is absorbed increases.
(d) energy is liberated (d) As the bubble rises, its size
15. A solid sphere moves at a terminal decreases.
velocity of 10 m/s in air at a place
where g = 10 m/s2. The sphere is taken Passages and Matrix Matching
in a gravity free hall having air at the
2CUUCIGHQT3Ō
same pressure and pushed down at a
speed of 10 m/s. When a viscous liquid flows, adjacent layers
(a) Its initial acceleration will be 10 oppose their relative motion by applying a
m/s2 downward. viscous force given by
dv
(b) Its initial acceleration will be 10 F = -hA
m/s2 upward. dy
(c) The magnitude of acceleration
will decreases as time passes.
(d) It will eventually stop.
DĞĐŚĂŶŝĐĂůWƌŽƉĞƌƟĞƐŽĨDĂƩĞƌപ949

When h = coefficient of viscosity, A (a) F μ r2 (b) F μ r


= surface area of adjacent layers in (c) F μ 1/r (d) F μ 1/r2
contact. 3. The momentum of the liquid confined
dv in the tube is
= velocity gradient
dy (a) rpR2lv0 (b) rpR2lv0/2
Now, a viscous liquid having coeffi- (c) 2rpR2lv0 (d) rpR2lv0/4
cient of viscosity h is flowing through 2CUUCIGHQT3Ō
a fixed tube of length l and radius R The axle of a pulley of mass 1 kg is attached
under a pressure difference P between to the end of an elastic string of length
the two ends of the tube. 1 m, cross-sectional area 10–3m2 and Young’s
Consider a cylindrical volume of liq- modulus 2 ¥ 105 N/m2 whose other end is
uid of radius r. Due to steady flow, fixed to the ceiling. A rope of negligible mass
net force on the liquid in cylindrical is placed on the pulley such that its left end
volume should be zero. is fixed to the ground and its right end is
dv hanging freely from the pulley which is at
-h 2p rl = Ppr2 rest in equilibrium. The free end of the rope
dr
A starts pulling with constant force F = 10 N.
0 R
P Friction can be neglected between the rope
- Ú dv =
2hl Úr
r dr
and the pulley.
v

(layer in contact with the tube is


stationary)
1m
Ê r2 ˆ PR2
v = v0 Á 1 - 2 ˜ , where v0 =
Ë R ¯ 4hl
The volume of the liquid per second
through the tube, A
R R
Ê r ˆ
2 F
Q = Ú v ◊ 2p r dr = Ú v0v0 ÁË 1 - R2 ˜¯ 2p r dr
0 0
R 4. The elongation of the string before
È r2 r4 ˘
= v0 2p Í - 2˙
applying force is
Î 2 4R ˚0 (a) 0.05 cm (b) 0.5 cm
(c) 5 cm (d) 50 cm
È R2 R2 ˘ v0p R2 p PR 4 5. The greatest elongation of the string is
= v0 2p Í - ˙= =
Î 2 4 ˚ 2 8hl (a) 20 cm (b) 25 cm
(c) 30 cm (d) 35 cm
This is called Poiseuille’s equation
6. The maximum displacement of point
1. Force acting on the tube due to the A after applying F
liquid is (a) 30 cm (b) 40 cm
(a) p h lv0 (b) 2p h lv0 (c) 60 cm (d) 70 cm
(c) 4p h lv0 (d) 6p h lv0 2CUUCIGHQT3Ō
2. The viscous force on the cylindrical A copper rod of length 2 m and cross-
volume of the liquid varies as sectional area 2.0 cm2 is fastened end to end
950പWŚLJƐŝĐƐĨŽƌ//dͲ:͗DĞĐŚĂŶŝĐƐ//

to a steel rod of length L and cross-sectional 2CUUCIGHQT3Ō


area 1.0 cm2. The compound rod is subjected The two wires shown in figure are made
to equal and opposite pulls of magnitude 3 ¥ of the same material which has a breaking
104 N at its ends. stress of 8 ¥ 108 N/m2. The area of the cross-
Ysteel = 2.0 ¥ 1011 N/m2 section of the upper wire is 0.006 cm2 and
Ycopper = 1.1 ¥ 1011 N/m2 that of the lower wire is 0.003 cm2. The mass
m1 = 10 kg, m2 = 20 kg and the hanger is light,
7. The length L of the steel rod if the
elongation of the two rods are equal is
(a) 1.82 m (b) 2.20 m
(c) 3.04 m (d) 3.84 m
8. The stress in copper rod is m2
(a) 1.20 ¥ 108 N/m2
(b) 1.50 ¥ 108 N/m2
(c) 2.00 ¥ 108 N/m2 m1
(d) 3.20 ¥ 108 N/m2
9. The strain in steel rod is
(a) 1.5 ¥ 10–3 (b) 2.25 ¥ 10–3
(c) 3.0 ¥ 10 N/m
–3 2
(d) 4.5 ¥ 10–3 13. Find the maximum load that can be
put on the hanger without breaking
2CUUCIGHQT3Ō lower wire
A steel wire has the following properties: (a) 7 kg (b) 10 kg
Length = 5 cm (c) 14 kg (d) 20 kg
Cross section = 0.5 cm2 14. For m1 = 10 kg, m2 = 36 kg; find the
Young’s modulus = 1.8 ¥ 1011 N/m2 maximum load that can be put on the
Shear modulus = 0.6 ¥ 1011 N/m2 hanger without breaking upper wire is
Proportional limit = 3.6 ¥ 108 N/m2 (a) 2 kg (b) 5 kg
Breaking stress = 7.2 ¥ 108 N/m2 (c) 7 kg (d) 14 kg
The wire is fastened at its upper end and
2CUUCIGHQT3Ō
hangs vertically.
When a liquid medicine of density r is to
10. The maximum load W that can be put in the eye, it is done with the help of
supported without exceeding the a dropper. As the bulb on the top of the
proportional limit is dropper is pressed, a drop forms at the
(a) 600 N (b) 1200 N opening of the dropper. We wish to estimate
(c) 1800 N (d) 2100 N the size of the drop. We first assume that
11. The extension of the wire under this the drop formed at the opening is spherical
load W is because that requires a minimum increase
(a) 0.01 m (b) 0.02 m in its surface energy. To determine the size,
(c) 0.04 m (d) 0.05 m we calculate the net vertical force due to the
12. The maximum load the wire can carry surface tension T when the radius of the
without breaking is drop is R. When this force becomes smaller
(a) 1800 N (b) 2400 N than the weight of the drop, the drop gets
(c) 3000 N (d) 3600 N detached from the dropper.
(IIT-JEE, 2010)
DĞĐŚĂŶŝĐĂůWƌŽƉĞƌƟĞƐŽĨDĂƩĞƌപ951

15. If the radius of the opening of the detaches from the dropper is approxi-
dropper is r, the vertical force due mately
to the surface tension on the drop of (a) 1.4 ¥ 10–3 m
radius R (assuming r << R) is (b) 3.3 ¥ 10–3 m
(a) 2p rT (b) 2pRT (c) 2.0 ¥ 10–3 m
(d) 4.1 ¥ 10–3 m
2p r 2T 2p R2T 17. After the drop detaches, its surface
(c) (d)
R r energy is
16. If r = 5 ¥ 10–4 m, r = 103 kgm–3, T = 0.11 (a) 1.4 ¥ 10–6 J (b) 2.7 ¥ 10–6 J
Nm–1, the radius of the drop when it (c) 5.4 ¥ 10 J
–6
(d) 8.1 ¥ 10–6 J

Matrix Matching
18. Match the columns correctly.
Column I Column II
A. Bernoulli’s theorem (p) Elasticity
B. Stokes’ law (q) Speed of efflux
C. Torricelli’s theorem (r) Venturimeter
D. Hooke’s law (s) Viscosity
(t) Conservation of energy
19. Column II depends on physical quantity/law given in column I. Match the columns
correctly.
Column I Column II
A. Stokes’ law (p) Radius
B. Terminal velocity (q) Density of the material of body
C. Excess pressure inside mercury drop (r) Coefficient of viscosity
D. Viscous force (s) Surface tension
(t) Velocity gradient
20. Match the columns correctly.
Column I Column II
A. With rise in temperature forces (p) Elastic force
that decreases
B. Forces involved in capillary action (q) Force due to surface tension
C. Water flows in a continuous stream (r) Friction force
down a vertical pipe whereas it breaks
into drops when falling freely because of
D. Terminal velocity of rain drop (s) Viscous force
(t) Gravitational force
21. A copper wire (Y = 1011 N/m2) of length 8 m and steel wire (Y = 2 ¥ 1011 N/m2) of
length 4 m each of 0.5 cm2 cross-section are fastened end to end and stretched with a
tension of 500 N.
Column I Column II
A. Elongation in copper wire in mm (p) 0.25
B. Elongation in steel wire in mm (q) 1.0
952പWŚLJƐŝĐƐĨŽƌ//dͲ:͗DĞĐŚĂŶŝĐƐ//

C. Total elongation in mm (r) 0.8


1
D. Elastic potential energy of the system (s) th the elongation in
4
in joules copper wire
22. Column I Column II
A. Larger Reyonlds number (p) Drops in mist
2r 2 g( r - s )
B. Time to acquire terminal speed (q) v =
9h
C. Radius of drops of water < 0.01 mm (r) Less viscous force
D. Greater velocity of flow of a liquid (s) Independent of the density
of liquid

Answers Key
Level 1
1. (a) 2. (a) 3. (a) 4. (d) 5. (a) 6. (c) 7. (d) 8. (b)
9. (c) 10. (b) 11. (c) 12. (d) 13. (a) 14. (a) 15. (c) 16. (c)
17. (a) 18. (c) 19. (b) 20. (a) 21. (b) 22. (b) 23. (a) 24. (a)
25. (a) 26. (c) 27. (c) 28. (d) 29. (b) 30. (b) 31. (a) 32. (b)
33. (c) 34. (b) 35. (a) 36. (b) 37. (a) 38. (c) 39. (b) 40. (b)
41. (b) 42. (c) 43. (d) 44. (b)
Level 2
1. (b) 2. (b) 3. (d) 4. (b) 5. (d) 6. (c) 7. (b) 8. (d)
9. (b) 10. (d) 11. (d) 12. (b) 13. (a) 14. (b) 15. (b) 16. (a)
17. (a) 18. (a) 19. (b) 20. (c) 21. (b) 22. (d)
Multiple Correct Options
1. (b, c) 2. (a, b, d) 3. (a, c, d) 4. (a, b, c)
5. (a, b) 6. (b, c) 7. (a, c) 8. (b, c, d)
9. (b, c) 10. (b, c) 11. (a, b, c, d) 12. (c, d)
13. (a, b, d) 14. (a, c) 15. (b, c, d) 16. (a, c)
17. (a, b) 18. (b, d) 19. (b, c)
Passages and Matrix Matching
1. (c) 2. (a) 3. (b) 4. (c) 5. (c) 6. (c) 7. (a) 8. (b)
9. (a) 10. (c) 11. (a) 12. (d) 13. (c) 14. (a) 15. (c) 16. (c)
17. (b) 18. AÆ(q, r, t); BÆ(s); CÆ(q, t); DÆ(p)
19. AÆ(p, r); BÆ(p, q, r); CÆ(p, s); DÆ(r, t) 20. AÆ(p, q, r, s); BÆ(q, t); CÆ(q); DÆ(s, t)
21. AÆ(r); BÆ(s); CÆ(q); DÆ(p) 22. AÆ(r); BÆ(s); CÆ(p, q); DÆ(r)
DĞĐŚĂŶŝĐĂůWƌŽƉĞƌƟĞƐŽĨDĂƩĞƌപ953

SOLUTION AND HINTS

Level 1
1. (a) In the second case, the deforming force is also W.
So, the elongation of the wire is l.
1 f 2 s2
2. (a) U = =
2 Y 2Y
DP 0.155 ¥ 10 5
3. (a) Bulk modulus, B = =
Ê DV ˆ 0.1
ÁË ˜
V ¯
= 1.55 ¥ 105 N/m2
Fx 200 ¥ 10 - 3
4. (d) U = = = 0.1 J
2 2
1 2 1 Ê YA ˆ 2
5. (a) W = kx = Á ˜x
2 2Ë L ¯

F W1 + 3W/4
6. (c) f = =
A s

3L
4

W1

1 2
7. (d) W1 = kx
2
1
and W2 = k ( x + y )2
2
1 1
\ W = W2 – W1 = k( x + y )2 - kx 2
2 2
1
= ky(2 x + y )
2
Ê Stress ˆ
8. (b) The ratio Á
Ë Strain ˜¯
decreases (see figure)
f

e
O
954പWŚLJƐŝĐƐĨŽƌ//dͲ:͗DĞĐŚĂŶŝĐƐ//

t phr 4
9. (c) As, =
q 2l
qA r4
\ = 24
qB r1

Dr R - r
10. (b) Strain e = =
r r
Ê R - rˆ
Stress f = eY = Á Y
Ë r ˜¯

Ê R - rˆ
Force needed F = fA = Á YA
Ë r ˜¯
11. (c) f 1 = f2
or Y1a1DT = Y2a2DT
Y1 a 3
\ = 2 =
Y2 a1 2
12. (d) YA = tan 60° = 3
1
and YB = tan 30° =
3
\ YA = 3YB.
13. (a) W = mg = (Al)rg
Thus, fA =W
or 106 A = Alrg
or 106 = l ¥ 3 ¥ 103 ¥ 10
or l  34 m.
1
14. (a) The theoretical value of Poisson’s ratio lies between –1 and
2
15. (c) The isothermal bulk modulus of a gas is equal to the pressure of the gas.
DY 1 ¥ 10 - 3
16. (c) e = =
l 4
2
e 2Y Ê 10 - 3 ˆ 2 ¥ 1011
u = =Á ˜ ¥
2 Ë 4 ¯ 2
= 0.075 J.
17. (a) f = YaDT
= 1.2 ¥ 1011 ¥ 1.1 ¥ 10–5 ¥ (20 – 10)
= 1.32 ¥ 107 N/m2.
18. (c) It is the material property, so does not depend on size and shape of the specimen.
DĞĐŚĂŶŝĐĂůWƌŽƉĞƌƟĞƐŽĨDĂƩĞƌപ955

19. (b) F = YaDTA


= 1011 ¥ 10–5 ¥ 100 ¥ 10–4
= 104 N.
20. (a) Given, the breaking strength of cable fu = 7 ¥ 107 N/m2
The force carried by the cable,
F = m(g + a)
= 2000(9.8 + 1.5) = 22600 N
F 22600
The area of cross section, A = =
fu 7 ¥ 107
= 3.28 ¥ 10–4 m2.
21. (b) For brittle material, there is no yield point.
22. (b) For x < r0 Æ repulsion and x > r0 Æ attraction
f 80/10 - 6
23. (a) Y = = = 2 ¥ 1011 N/m2
e 4 ¥ 10 - 4
24. (a) The difference in work done in expanding and compressing rubber will appear as
heat.
25. (a) The thread spreads out due to surface tension.
26. (c)
Fb

q
T T

For floating disc, Fnet = 0


or Fb + 2prT cos q = W¢
or W + 2prT cos q = W¢
v1 ( r g - rl ) 19.5 - 1.5
27. (c) = =
v2 ( rs - rl ) 10.5 - 1.5
v1 0.2
\ v2 = = = 0.1 m/s
2 2
p Pa 4
28. (d) Q =
8hl
p (2P)( a/2)4 Q
and Q¢ = =
8hl 8
29. (b) Fv = 6phRv
956പWŚLJƐŝĐƐĨŽƌ//dͲ:͗DĞĐŚĂŶŝĐƐ//

4T
30. (b) The inside pressure must be greater than the outside pressure in the bubble.
r
This excess pressure is provided by charge in bubble. pa

pa
4T s2
= 2e
r 0

4T Q2 È Q ˘
r
= 2 2
16p r ¥ 2e 0 ÍÎs = 4p r 2 ˙˚

Ê vˆ
31. (a) mg sin q = hA Á ˜
Ë t¯

Ê vˆ
or ra3g sin q = ha 2 Á ˜
Ë t¯

r agt sin q
\ h =
v
32. (b) W = mg = r(pr2h)g
Ê 2T ˆ
or W r(p r 2 ) ¥ Á g
Ë rrg ˜¯
or W = 2prT

W 6.28 ¥ 10 4
\ T = =
2p r 2p ¥ 2 ¥ 10 - 3

= 5 ¥ 10–2 N/m
33. (c) In freely falling elevator, the effective value of gravity becomes zero, so the liquid
will rise up to full length of the tube.
h 2
34. (b) l = = = 4 cm
cos q cos 60∞

h q
l

35. (a) For the convex meniscus, the angle of contact should be greater than 90°.
DĞĐŚĂŶŝĐĂůWƌŽƉĞƌƟĞƐŽĨDĂƩĞƌപ957

36. (b) For the liquid does not wet the solid, the angle of contact should be greater than
90°.
37. (a) In this process, surface area of the drop decreases and so energy will liberate.
38. (c) F = 2T ¥ perimeter
= 2T ¥ 4 l = 8 Tl
2T cos q
39. (b) h = , or hr = constant (rectangular hyperbola).
rr g
40. (b) W = TDA
= T ¥ 2[4pR2 – 0]
= 8pR2T.
4T
41. (b) We know that P = . With increase in time r increases and so pressure decreases.
r
2T
42. (c) As h =
Rr g
2T
\ – h¢ = = 6h
Rr g/6
43. (d) In satellite there is weightlessness, so water will rise to full length of the tube.
44. (b) W = TDA
W
\ T =
DA
3 ¥ 10 - 4
=
2(10 ¥ 11 - 10 ¥ 6) ¥ 10 - 4

= 3 ¥ 10–2 N/m2

Level 2
1. (b) Fb + Fv = Mg
Ê F ˆ
or Fv = Mg – Fb = Mg Á 1 - b ˜
Ë Mg ¯
Ê d ˆ
= Mg Á 1 - 2 ˜
Ë d1 ¯
958പWŚLJƐŝĐƐĨŽƌ//dͲ:͗DĞĐŚĂŶŝĐƐ//

F 4F 2F 4F
2. (b) fA = = ; fB = =
0.25 A A 0.50 A A
4 F 2F 5F
fC = = ; fD =
2A A 3A
3. (d) We know that
FL
DL =
AY
FL
\ Y =
ADL
Clearly, it is greatest for case 4.
mg
4. (b) f =
A
Volumetric strain, m
DV DP
=
V K
DR mg/A
or 3 =
R K
DR
= ÊÁ
mg ˆ
\
R Ë 3KA ˜¯
5. (d) Longitudinal strain,
f 5 ¥ 107
e = = = 2.5 ¥ 10–4
Y 2 ¥ 1011
For cylindrical wire, V = pr2l
DV 2Dr Dl
\ = +
V r l
0.02 Dr
or = 2 + 2.5 ¥ 10 - 4
100 r
Dr
\ = – 0.25 ¥ 10–4
r
6. (c) One free surface raises the water by h, then two free surfaces raise the water by 2h.
7. (b) The surface tension of soap water is smaller, so it rises to a small height.
Dlsteel F l /p r 2Y
8. (d) = 1 1 12 1
Dlbrass F2l2/p r2 Y2
2
2 Ê 1ˆ 1
= ¥a¥Á ˜ ¥
4 Ë b¯ c
a
=
2b 2c
DĞĐŚĂŶŝĐĂůWƌŽƉĞƌƟĞƐŽĨDĂƩĞƌപ959

MgL
9. (b) Y =
p r 2l
1 ¥ 9.8 ¥ 2
=
p (0.2 ¥ 10 - 3 )2 ¥ 0.8 ¥ 10 - 3

= 2 ¥ 1011 N/m2
DY Dr Dl
Also = 2 +
Y r l
0.01 0.05
= 2 + = 0.2
4 0.8
Use 1 Æ steel, 2 Æ brass
10. (d) For stress to be equal,
T1 T
= 2
A1 A2
T1 A 1
\ = 1 =
T2 A2 2
11. (d)
q1
t

l/2
l

q1 + q2 = q0 ...(i)
We know that
t phr 4
=
q 2l
q1 l r4 l/2 ¥ r 4
\ = 1 ¥ 24 = =8 ...(ii)
q2 l2 r1 l ¥ (r/2)4
After solving the above equations, we get
8q
q1 =
9
12. (b) Using Hooke’s law, F = kx we can write
4 = k(a – l0) ...(i)
and 5 = k(b – l0) ...(ii)
If l is the length under tension 9 N, then
9 = k(l – l0)
After solving the above equations, we get
l = (5b – 4a)
960പWŚLJƐŝĐƐĨŽƌ//dͲ:͗DĞĐŚĂŶŝĐƐ//

13. (a) Increase in length of the wire


F◊L
l =
p r 2Y
2 F ¥ (2l)
and l¢ = =l
p ( 2 r )2 Y

Ê 2T cos q ˆ
14. (b) M = (pr2h)r = p r 2 Á r
Ë rrg ˜¯

Ê 2T cos q ˆ
and M¢ = p (2r )2 Á r
Ë r ¥ 2r ¥ g ˜¯
= 2M.

2T 2 ¥ 0.07
15. (b) = = 103 N/m2
r 0.14 ¥ 10 - 3
2T
Pressure applied = Pa –
r
= 105 – 103 = 99 ¥ 103 N/m2

Ê 4T ˆ Ê 4T ˆ V
16. (a) Á P0 + ˜ V = ÁP +
Ë r ¯ Ë r/2 ˜¯ 8

Ê 3T ˆ
fi P = 8 Á P0 + ˜
Ë r ¯
È v v ˘
17. (a) F = Ín1 + n2 ˙A
Î h1 ( h - h1 ) ˚
dF
=0
dh1
h
h1 =
4
2T 2T
18. (a) h1 = and h2 =
r1r g r2 r g

2T Ê 1 1 ˆ
\ h = h 2 – h1 = -
r g ÁË r2 r1 ˜¯

4 4
19. (b) n p a 3 = pb 3
3 3
b3
\ n =
a3
DĞĐŚĂŶŝĐĂůWƌŽƉĞƌƟĞƐŽĨDĂƩĞƌപ961

W = TDA = S[n ¥ 4pa2 – 4pb2]

1 È b3 ˘
or mv 2 = S Í 3 ¥ 4p a 2 - 4p b 2 ˙
2 Îa ˚

1 Ê4 ˆ
3
È b3 ˘
or r ¥ Á p b˜ v 2 = S Í 3 ¥ 4p a 2 - 4p b 2 ˙
2 Ë3 ¯ Îa ˚
6S Ê 1 1 ˆ
\ v = Á - ˜
r Ë a b¯
20. (c) T ¥ 2pr + mg = Fb
fb

T ¥ 2p r

mg

È4 3˘
4 3 Í pr ˙
or T ¥ 2pr + r p r g = Í 3 ˙s g
3 Í 2 ˙
Î ˚
21. (b) The radius at end 1 is smaller than at end 2, so pressure at end 1 is greater. Air
blows from this end.
22. (d) From PV = nRT, we have
PAVA n
= A
PBVB nB
Ê 4T ˆ 4 3
ÁË 8 + r ˜¯ ¥ 3 p rA n
A
fi = A
Ê 4T ˆ 4 3 nB
ÁË 8 + r ˜¯ ¥ 3 p rB
B

After substituting values, we get


nB
= 6.
nA

Multiple Correct Options


1. (b, c)
e 2Y
W =U= ¥ vol.
2
2
1Ê l ˆ YAl 2
= ÁË ˜¯ Y ¥ AL =
2 L 2L
962പWŚLJƐŝĐƐĨŽƌ//dͲ:͗DĞĐŚĂŶŝĐƐ//

2. (a, b, d)
Dl l
Strain, e = = =1
l l
Stress, f = eY = 1 ¥ Y = Y
e 2Y (1)2 Y
Potential energy, U = = =Y
2 2
3. (a, c, d)

Mg

M l Dl
O

The decrease in PE = Mgl


l
Elastic potential energy stored = Mg
2
l
Heat produced, H = Mgl - Mg
2
l
= Mg
2
4. (a, b, c)
mg 4 mg
FA = mg + =
3 3
mg
and FB =
3
FA 4 mg/3
fA = 2
=
p rA p rA2
FB mg/3
and fB = 2
=
p rB p rB2
For rA = 2rB fi fA = fB = f; so either of them will break.
For rA < 2rB ; fA > f, A will break before B.
For rA = rB; fA > f, so A will break before B.
5. (a, b) F1
F1 = F2 = 5000
Also Dl1 = Dl2
2l
F1 (2l) F (3l)
or = 2
AY AY 5000 N
or 2F1 = 3F2 3l

After solving, we get F2


F1 = 3000 N and F2 = 2000 N
DĞĐŚĂŶŝĐĂůWƌŽƉĞƌƟĞƐŽĨDĂƩĞƌപ963

6. (b, c)
F = (YaDT) pr2
\ F 1 : F2 : F3 : F4 = 1 : 2 : 3 : 4
1 1 Fl F 2l F 2l
Energy stored = fx = F ¥ = =
2 2 AY 2 AY 2p r 2Y
F2
or U μ
r2
7. (a, c)
Fl
Dl =
p r 2Y
DlA r2
Clearly, = B2 = (2)2 = 4
DlB rA
8. (b, c, d)
The force at the middle of rod AB will be F. So stress right of middle will be greater
than F/A. The force at each section of rod CD is F. So stress at each section is F/A.
9. (b, c)
As vt μ r2
v1 ( r )2 v
\ = 2
fi v2 = 1
v2 (r/2) 4
10. (b, c)
Viscous force, Fv = 6phrv
Clearly, Fv μ v and Fv μ r
A
As A = pr2; r = , so Fv μ A
p
11. (a, b, c, d)
Solution in theory
12. (c, d)
The liquid will rise to the full height of the tube and will get meniscus of larger radius.
13. (a, b, d)
2T cosq
h=
rr g
h = Clearly, h = depends on T, r and length of the tube.
14. (a, c)
When a drop splits, surface area increases and energy is to be given in the process.
15. (b, c, d)
Initially, mg = 6phr ¥ 10
In gravity free space, mg = 0, so unbalanced force acts in upward direction. Therefore
mg
a = = g m/s2
m
Because of upward acceleration, the downward speed decreases and sphere will stop
momentarily.
964പWŚLJƐŝĐƐĨŽƌ//dͲ:͗DĞĐŚĂŶŝĐƐ//

16. (a, c)

2T cos q
h= . For h to be zero, either T = 0 or q = 90°
rr g
17. (a, b)
Angle of contact is the property of materials in contact.
18. (b, d)
2T cos q
h=
rr g
1
Clearly, h μ . Also in case of tube of insufficient length, h = l
r
19. (b, c)
PtopV = PbottomV¢
Pbottom = (Ptop + rgh)
As Pbottom > Ptop; \ V > V¢

Passages and Matrix Matching


Passage (Q. 1–3)
Ê dv ˆ È - 2Rv0 ˘
1. (c) f = 2hp Rl Á ˜ = - 2hp Rl Í 2 ˙
; F = 4phlv0
Ë dr ¯ Î R ˚
È - 2rv0 ˘
2. (a) F = – h2prl Í 2 ˙
fi F μ r2
Î R ˚
3. (b)
l
v
R

dr dm = rdV

dm = r dV = rdAl; dm = r l 2pr dr
So momentum of mass dm
p R
dp = v dm; Ú dp = Ú v dm
0 0
R R
Ê r2 ˆ È r2 r4 ˘
p = 2prlv0 Ú r Á 1 - 2 ˜ dr ; 2prlv0 Í - 2˙
0 Ë
R ¯ Î 2 4R ˚0

È R2 R2 ˘ 2prlv0 R2 rp R2lv0
= 2prlv0 Í - ˙= ;p=
Î 2 4 ˚ 4 2
DĞĐŚĂŶŝĐĂůWƌŽƉĞƌƟĞƐŽĨDĂƩĞƌപ965

Passage (Q. 4–6)


F = 30N
Fl 10 ¥ 1
4. (c) Dl = = -3 = 5 cm
AY 10 ¥ 2 ¥ 10 5

1 2 2 F 2 ¥ 30
5. (c) Fx = kx \ x = =
2 k (YA/l) 10N 10N
2 ¥ 30 10N
= = 30 cm
2 ¥ 10 5 ¥ 10 - 3
6. (c) When the pulley moves down by 30 cm, the string will loosen from its both sides,
so the point A moves down by 60 cm.
Passage (Q. 7–9)
7. (a) Dlcopper = Dlsteel
F¥2 F¥L
or -4
=
11
(1.1 ¥ 10 ) ¥ 2 ¥ 10 2.0 ¥ 1011 ¥ 1 ¥ 10 - 4
or L = 1.8 m
4
F 3 ¥ 10
8. (b) fcopper = = = 1.50 ¥ 108 N/m2
A 2 ¥ 10 - 4
Ê 3 ¥ 10 4 ˆ 11
9. (a) esteel = fsteel/Ysteel = Á ˜ 2 ¥ 10
Ë 1 ¥ 10 - 4 ¯
= 1.5 ¥ 10–3
Passage (Q. 10–12)
10. (c) W = fA = 3.6 ¥ 108 ¥ 0.5 ¥ 10–4 = 1800 N
Wl 1800 ¥ 0.05
11. (a) Dl = =
AY 0.5 ¥ 10 - 4 ¥ 1.8 ¥ 1011
= 0.01 m
12. (d) Wmax = fuA = 7.2 ¥ 108 ¥ 0.5 ¥ 10–4 = 3600 N
Passage (Q. 13-14)
Suppose m is the load put on the hanger.
(20 + 10 + m) g
13. (c) The stress in the upper wire 8 ¥ 108 =
0.006 ¥ 10 - 4
\ m = 18 kg
(10 + 18) g
Stress in the lower wire f =
0.003 ¥ 10 - 4
= 9.3 ¥ 109 N/m2
Thus, the lower wire will break by 18 kg load, and so to prevent its breaking let,
m¢ be the required mass.
966പWŚLJƐŝĐƐĨŽƌ//dͲ:͗DĞĐŚĂŶŝĐƐ//

(10 + m) g
\ 8 ¥ 108 =
0.003 ¥ 10 - 4
or m¢ = 14 kg
(10 + 36 + m)
14. (a) The stress in the upper wire 8 ¥ 104 =
0.006 ¥ 10 - 4
\ m¢ = 2 kg.
The stress in the lower wire corresponding to this load
(10 + 2)
f = = 4 ¥ 108 N/m2, (safe)
0.003 ¥ 10 - 4

Passage (Q. 15–17)


15. (c) The vertical component of face
r
Fv = F sin q
 F tan q
r
= (T ¥ 2pr) ¥ F sin q
R F
2p r 2T
=
R q
R
16. (c) Fv = mg O
2
2p r T 4
or = p R3 r
R 3
r/4
Ê 3r 2T ˆ
\ R = Á ˜
Ë 2r g ¯
17. (b) Surface energy = T (4pR2) = 2.7 ¥ 10–6 J
Matrix Matching
18. A Æ (q, r, t); B Æ (s); C Æ (q, t); D Æ (p)
(A) Bernoulli’s equation is based on conservation of energy. From Bernoulli’s equation,
we can get speed of efflux and rate of flow (venturimeter)
(B) Fv = 6phrv
(C) Toricelli’s theorem, ve = 2gh , which can be obtained by conservation of energy.
(D) Hooke’s law: stress μ strain
19. A Æ (p, r); B Æ (p, q, r); C Æ (p, s); D Æ (r, t)
(A) Stokes’ law, Fv = 6phrv
2 r 2 (r - s )g
(B) Terminal velocity, vt =
9 h
2T
(C) Excess pressure, p =
r
(D) Viscous force, F = hA(–dv/dy).
DĞĐŚĂŶŝĐĂůWƌŽƉĞƌƟĞƐŽĨDĂƩĞƌപ967

20. A Æ (p, q, r, s); B Æ (q, t); C Æ (q); D Æ (s, t)


21. A Æ (r); B Æ (s); C Æ (q); D Æ (p)
Fl 500 ¥ 8
(A) Dlcopper = = = 0.8 mm
AY 0.5 ¥ 10 - 4 ¥ 1011
Fl 500 ¥ 4
(B) Dlsteel = =
AY 0.5 ¥ 10 - 4 ¥ 2 ¥ 1011
= 0.2 mm
(C) Dl = Dlcopper + Dlsteel = 1.0 mm

È e 2Ycopper e 2Y ˘
(D) U = Í + steel
˙ ¥ Vol = 0.25 J
ÍÎ 2 2 ˙
˚
22. A Æ (r); B Æ (s); C Æ (p, q); D Æ (r)
rvD
(A) RN = , clearly less value of h indicates large value of RN.
h
(B) Time required for terminal velocity does not depend on the density of body or
liquid.
2 2 (r - s )g
(C) Radius is related to v = r
9 h
pr 2
(D) Velocity of flow, v =
8hl
6 CHAPTER

Hydrostatics or Fluid Statics

INTRODUCTION
Fluid is that which flows, i.e., liquids and gases.
Hydrostatics also called ƀWKFUVCVKEU is the study of fluids and objects-in-fluids, at rest.
The study of fluid flow or objects moving in fluids is called hydrodynamics or fluid dynamics.
Both combined are called fluid mechanics or mechanics of fluids.

Shape and Size


Solids have definite shape and size under normal conditions. Liquids have definite size, i.e.,
volume but not shape. They assume the inside shape of their containers. Gases have neither
definite shape nor size. They take up the shape and size of their containers.

Tangential Forces/Modulus of Rigidity


Fluids cannot withstand tangential stress. They yield without resistance to such stresses
either rapidly as in the case of water, or slowly as in the case of grease, honey or coal tar.
Thus, fluids do not have any modulus of rigidity.

Normal Forces
Fluids do withstand forces applied at right angles to their surfaces.

Orderliness of Atoms
As we have already discussed all solids (except amorphous and semi-crystalline solids) have
regular crystalline structures. Atoms of fluids are invariably disordered.

Compressibility
Within practical limits liquids are incompressible. They have free surfaces of their own.
Gases are compressible and occupy all the space which is made available to them.

Ideal Liquid
An ideal liquid has zero compressibility and zero viscosity. No liquid is an ideal liquid.
,LJĚƌŽƐƚĂƟĐƐŽƌ&ůƵŝĚ^ƚĂƟĐƐപ969

Ideal Gas
For the purpose of thermodynamics, a gas is ideal if it obeys Boyle’s law as well as Joule’s
law of internal energy, i.e., its internal energy does not depend on the volume occupied by
the gas.

Different Units of Pressure


Various units of pressure are as follows:
(i) SI unit of pressure: Nm–2 or Pascal (Pa)
(ii) Atmosphere (atm)
1 atm = 1.013 ¥ 105 Pa = 76 cm of Hg
(iii) In medicine and physiology, pressure is measured in “torr” and mm of mercury.
1 torr = 133 Pa
1 mm of Hg = 1 torr
(iv) In meteorology, the atmospheric pressure is measured in bar and millibar.
1 bar = 105 Pa
1 millibar = 100 Pa

MASS DENSITY AND SPECIFIC WEIGHT


Density of any material is defined as the mass per unit volume of the material. Thus,
Mass
Density =
Volume
M
or r =
V
SI unit of density is kg/m3. CGS unit of density is g/cm3.

’‡ ‹ϐ‹ ‡‹‰Š–


Specific weight of any material is the weight per unit volume of the material.
Weight
Specific weight =
Volume
W
or g =
V
SI unit of specific weight is N/m3.

Relative Density
Relative density of a substance is defined as the ratio of the density of the substance to the
density of water at 4°C.
970പWŚLJƐŝĐƐĨŽƌ//dͲ:͗DĞĐŚĂŶŝĐƐ//

Density of substance
Thus, RD =
Density of water at 4°C
Relative density is a dimensionless quantity. Clearly, density of a substance = R.D. ¥
Density of water at 4°C.

’‡ ‹ϐ‹  ”ƒ˜‹–›


Specific gravity is the modern name of relative density, it is the density of a substance as
compared to that of water.
Density of substance
Specific gravity, R.D. =
Density of water
Specific weight of substance
=
Specific weight of water
Note:
1. Numerically relative density and specific weight are equal.
2. Relative density can be calculated as:
Weight of substance in air
R.D. =
Loss in weight in water
Wair
or R.D. =
Wair - Wwater
3. If ro is the density of substance at 0°C, then its density at t°C is given by
ro
rt = ª ro (1 – g t)
(1 + g t)

Densities of some common fluids at STP

Fluid Density (kg/m3)


Water 1.00 ¥ 103
Seawater 1.03 ¥ 103
Ice 0.92 ¥ 103
Mercury 13.6 ¥ 103
Blood 1.06 ¥ 103
Air 1.29
Black hole 1019
Best laboratory vacuum 10– 29
,LJĚƌŽƐƚĂƟĐƐŽƌ&ůƵŝĚ^ƚĂƟĐƐപ971

Density of Mixture
If m1, m2, ..., mn are the masses and V1, V2, ... Vn are the volumes of the different substances,
then density of their mixture is defined as
M m1 + m2 + ºº + mn
r= =
V V1 + V2 + ºº + Vn
(i) Case of two liquids, each of the same volume V but of different densities r1 and r2
mixed together.
Total mass = Vr1 = Vr2
Total volume = 2V
V r1 + V r2
Density of mixture, r =
2V
r1 + r2
or r=
2
(ii) Case of two liquids, each of same mass m but of different densities r1 and r2 mixed
together.
Total mass = 2m
m m
Total volume = +
r1 r2
2m
Density of mixture, r =
m m
+
r1 r2
2 r1 r2
or r=
r1 + r2
(iii) Case of two liquids of different masses m1 and m2 mixed together. Let r1 and r2 be
their respective densities.
Total mass = m1 + m2
m1 m2
Total volume = +
r1 r2
m1 + m2
Density of mixture, r =
m1 m2
+
r1 r2
(iv) Case of two liquids of different volumes V1 and V2 mixed together. Let r1 and r2 be
their respective densities.
Total mass = V1 r1 + V2 r2
Total volume = V1 + V2
V1r1 + V2 r2
Density of mixture, r =
V1 + V2
972പWŚLJƐŝĐƐĨŽƌ//dͲ:͗DĞĐŚĂŶŝĐƐ//

Density of gases varies considerably with pressure but not of liquids, because while
gases are readily compressible, liquids are not.

THRUST OR NORMAL FORCE OF A LIQUID


When a certain liquid is held in static equilibrium against the boundary surfaces, the forces
exerted by liquid are always perpendicular to the surface in contact. This is so because
a liquid at rest cannot resist any tangential force. To understand this, consider a liquid
contained in a vessel in equilibrium, as shown in Fig. 6.1. Suppose the liquid exerts a force
F at the bottom of the vessel, whose direction makes angle q with the horizontal. The bottom
surface of the vessel exerts an equal reaction force R on water.
The reaction force has two components:
(i) Tangential component = R cos q
(ii) Normal component = R sin q
R R sin q
Since the liquid cannot sustain any tangential
force, so it begins to flow along tangentially. But the
liquid is at rest, so tangential component of R must
be zero, i.e., F
R cos q q
R cos q = 0
As R π 0, \ cos q = 0 or q = 90° Fig. 6.1
Hence, a liquid always exerts force perpendicular to the surface of the container at every
point.

PRESSURE
The pressure or intensity of pressure at any point may be DF
defined as the normal force exerted on a unit area around
that point. If force DF acts normally over a flat area DA,
then pressure
DF
P=
DA DA
Pressure at any point can be defined as
DF
P = lim
DAÆ0 DA
dF Fig. 6.2
or P=
dA

Units of Pressure
(i) SI unit of pressure = N/m2 or pascal (Pa), i.e., 1 Pa = 1 N/m2
(ii) In metrology the pressure is measured in bar and millibar.
1 bar = 105 N/m2
Also, 1 torr = 1 mm of Hg height
,LJĚƌŽƐƚĂƟĐƐŽƌ&ůƵŝĚ^ƚĂƟĐƐപ973

(iii) Atmospheric (atm): It is the pressure exerted by 76 cm of mercury column or 10.3 m


of water column.
1 atm = 1.013 ¥ 105 N/m2

Some Pressures (in Pa)

Atmosphere at sea level 1.01 ¥ 105

Systolic blood pressure (gauge) 1.60 ¥ 104 (120 torr)

Highest laboratory pressure 1012

Exceptional laboratory vacuum 10–12

Blood Pressure: When the heart contracts to its smallest size, the pumping of blood is the
hardest and the pressure of blood flowing in major arteries is nearly 120 mm of Hg (120
torr). This is known as systolic pressure. When the heart is expanded to its largest size the
blood pressure is nearly 80 mm of Hg (80 torr). This is known as diastolic pressure. The
sphymomanometer is a device, which measures these extreme pressures.

WORKED PROBLEMS
Problem 6.1 The two thigh bones, each of cross-sectional area 10 cm2 support the upper
part of a human body of mass 80 kg. Estimate the average pressure sustained by the bones.
Take g = 10 m/s2.
5QNWVKQP Total cross-sectional area of the thigh bones
A = 2(10 ¥ 10– 4) = 2 ¥ 10– 3 m2
Force acting on the bones = mg = 80 ¥ 10
= 800 N
F 800
\ Pav = = = 4 ¥ 105 N/m2
A 2 ¥ 10 - 3

Problem 6.2 A cylindrical vessel containing liquid is closed by a P0


smooth piston of mass m. The area of cross section of the piston is A.
If the atmospheric pressure is Po, find the pressure of the liquid just
below the piston.
mg
5QNWVKQP The pressure due to weight of the piston, P =
A
Atmospheric pressure = Po
mg
\ Total pressure just below the piston = + Po
A

Fig. 6.3
974പWŚLJƐŝĐƐĨŽƌ//dͲ:͗DĞĐŚĂŶŝĐƐ//

VARIATION OF LIQUID PRESSURE WITH DEPTH


Consider a liquid of density r in static equilibrium.
Take a liquid sample contained in an imaginary (P + dP)A
cylindrical element of area A and thickness dy, at a
dy
height y from the bottom of the container. Various
forces acting on the element are: y
dW
y
(i) Upward force PA on the bottom of the element. PA
(ii) Downward force (P + DP) A on the top of the
element. Fig. 6.4
(iii) Weight of the liquid in the cylindrical element. P2
If is dW = mg = (Ady)rg
h
As the liquid element is in equilibrium, so
P1
PA – (P + dP)A = dW y2
y1
or (– dP)A = A(dy)rg
or – dP = rg(dy)
dP Fig. 6.5
fi = – rg
dy
If P1 and P2 are the pressures at elevation y1 and y2 respectively and assuming r and
g constant, then on integrating the above equation, we get
P2 y2

Ú dP = Ú - rg dy
P1 y1

y
[P]PP12 = -rg[ y]y12

P2 – P1 = – rg(y2 – y1)
Pressure at a depth h below the free surface can be obtained as
y2 – y 1 = h
If P2 = Po (Pressure at free surface of liquid), then pressure P at a depth h,
P = Po + hrg (P1 = P)
Note:
1. If Po = 0, then P = hrg.
2. Pressure at any point inside the liquid depends on depth h.
hrg
3. Pressure does not depend on the area of cross section or the shape of the
vessel. Fig. 6.6

Now consider two points A and B in the same horizontal line inside the liquid. Imagine
a small vertical area DA containing the point A and a similar vertical area DA containing
,LJĚƌŽƐƚĂƟĐƐŽƌ&ůƵŝĚ^ƚĂƟĐƐപ975

the point B. If the pressure at A and B are P1 and P2


respectively, then the forces on element AB are:
B
(i) P1 DA towards right and A
(ii) P2 DA towards left. P1DA P2DA
As the liquid is in static equilibrium, so
P1DA = P2DA fi P1 = P2
Thus, the pressure is same at different points on Fig. 6.7
the same horizontal level.

PASCAL’S LAW
This law tells about the transmission of pressure in a liquid. It can be stated in the following
equivalent ways:
P

P1 = P2 = P3 = P4 P1 = P2

P3

P2 P1 P

P4 1 2

(i) (ii) (iii)

Fig. 6.8

(i) The pressure exerted at any point on an enclosed liquid is transmitted equally in all
directions.
(ii) A change in pressure applied to an enclosed incompressible liquid is transmitted
undiminished to every point of the liquid and the walls of the container.
(iii) The pressure in a liquid at rest is same at all points if we ignore gravity.

Proof of Pascal’s Law F


Let us imagine the volume of liquid which is inside a C
FA sin q
very small right angled prism shown in enlarged view
in Fig. 6.9. Let it have uniform thickness t. FA
q
\ Areas of the three forces of prism will be as a
FB
FA cos q
Face ADEB = AB ¥ t = ct b

Face ADFC = AC ¥ t = bt
D E
Face BCFE = BC ¥ t = at
FC q
Let FC, FB and FA are the forces being acted upon these A t
B
three forces respectively. FA can be resolved into two
c
components FA cos q and FA sin q.
Fig. 6.9
976പWŚLJƐŝĐƐĨŽƌ//dͲ:͗DĞĐŚĂŶŝĐƐ//

Applying Newton’s first law that as this prism is in equilibrium at rest, hence no net
external force would be acting on it.
\ FC = FA cos q ...(i)
FB = FA sin q ...(ii)
and PA = Pressure on face BCFE
FA
= ...(iii)
at
PB = Pressure on face ADFC
FB FA sin q
= =
bt bt
FA ¥ b/a FA
= = ...(iv)
bt at
PC = Pressure on face ADEB
FC FA cos q
= =
ct ct
FA ¥ c/a FA
= = ...(v)
ct at
Thus, from Eqs. (iii), (iv) and (v) we get
PA = PB = PC
This is what Pascal’s law states.
How will you experimentally verify the Pascal’s law of transmission of fluid pressure?
'ZRGTKOGPVCNXGTKſECVKQPQH2CUECNŏU.CY. As shown in
Area = A/2
Fig. 6.10, take a vessel having three openings A, B and C 2F F/2
and provided with frictionless and watertight pistons. Let
Area = 2A
A B C
their cross-sectional areas be A, 2A and respectively.
2
Fill the vessel with water and apply an additional force F
Water
on piston A. To keep the pistons B and C in their positions,
F
forces equal to 2F and respectively have to be applied
2 A
Area = A
on them. This shows that the pressure P is transmitted F

equally in all directions because Fig. 6.10 Experimental verification


of Pascal’s law
F 2 F F/2
P= = =
A 2 A A/2
,LJĚƌŽƐƚĂƟĐƐŽƌ&ůƵŝĚ^ƚĂƟĐƐപ977

APPLICATIONS OF PASCAL’S LAW


Hydraulic Lift or Jack
It is used to lift heavy load (cars, trucks) for small f
height. A piston of small cross-sectional area a is W
used to exert a small effort f on a liquid such as a A
f
oil. The pressure P = is transmitted to a larger
a
cylinder equipped with a larger piston of area A
through a pipe (see Fig. 6.11).
According to Pascal’s law, the same pressure P
is also transmitted to the larger piston of cross-
Fig. 6.11
sectional area A.
Pressure at smaller piston = Pressure at larger piston
f W
or =
a A
Ê Aˆ
fi W = fÁ ˜
Ë a¯
As A > a, \ W > f.
Ê Aˆ
Hence, by making Á ˜ larger, heavy loads can be lifted by applying small effort.
Ë a¯

Hydraulic Brakes
The hydraulic brakes used in vehicles are also based on the same principle as that of the
hydraulic lift. When the driver of the vehicle puts pressure on the brake paddle, the lever
system moves the piston into the master cylinder containing brake fluid.

Drum Brake lining


Brake shoes
Brake
paddle
P1

Spring

P2
Master
cylinder

To other
wheels
Brake fluid

Fig. 6.12
978പWŚLJƐŝĐƐĨŽƌ//dͲ:͗DĞĐŚĂŶŝĐƐ//

The brake fluid from the master cylinder is led through strong pipes to cylinders provided
with pistons (P1, P2) of larger cross-sectional areas. Thus, a small force applied over the
brake paddle is transmitted by the brake fluid as a large force to the piston for brake and as
a result, the brake shoes open tend to up. They press against the brake linings of the drums
of the wheels and bring them to rest.
When the paddle is released, 2-spring system brings the brake shoes back to their normal
positions and the brake fluid returns to the master cylinder.

Examples of Liquid Pressure


6[TGUQHJGCX[NQCFVTWEMUCTGOCFGYKFG
force
Obviously, weight of entire truck falls on tyres. Further, pressure = ; hence if tyres are
area
made of small area; the pressure on them will be very large. They will get damaged soon.
However, if tyres have large area in contact with ground, there will be less pressure on them,
hence they will remain intact, in good condition for a long time.
(QWPFCVKQPUQHJKIJTKUGDWKNFKPIUCTGOCFGYKFG
The weight of entire building falls on the foundation. If the foundation is made wide, the
area will be large, hence the pressure exerted by them on the earth will be small. So the
building will remain safe.
&KXGTUYGCTOGVCNNKEEQXGTUYJKNGIQKPIFQYPKPVQVJGUGC
The water pressure increases with increase of depth of water. The increase in pressure is 1
atmosphere per 10 m increase of depth of water. If this order of pressure falls on the diver
directly, he cannot tolerate it. Therefore, if a diver wants to go to a great depth, he wears the
metallic cover. The pressure of water column falls on the metallic cover while the pressure
inside the cover remains nearly equal to the atmospheric pressure.
9CNNUQHCFCOCTGOCFGVJKEMCVVJGDQVVQOCPFVJKPWRYCTFU
The pressure at a point inside a liquid depends on the depth of point from the free surface
of liquid, therefore, the pressure is very high at the bottom of the dam. To take care of this
pressure, the walls of dam are made thick at the bottom.

WORKED PROBLEMS
Problem 6.3 Two pistons of a hydraulic machine have diameters of 30.0 cm and 2.5 cm.
What is the force exerted on the larger piston when 40 kg wt. is placed on the smaller piston?
If the smaller piston moves in through 6 cm, how much does the other piston move out?
5QNWVKQP For smaller piston, Area a1 = p ¥ (1.25)2 cm2
For larger piston, Area a2 = p ¥ (15)2 cm2
a2
Mechanical advantage at the larger piston is
a1
a2
\ F2 = ¥ F1
a1
,LJĚƌŽƐƚĂƟĐƐŽƌ&ůƵŝĚ^ƚĂƟĐƐപ979

p (15)2 225
= ¥ 40 kg wt = ¥ 40 ¥ 9.8 N
p (1.25) 2
1.25 ¥ 1.25
= 56,448 N
This is the force exerted on the larger piston.
The liquids are considered incompressible. Therefore, volume covered by the movement
of smaller piston inwards is equal to that moved outwards by larger piston.
fi l1a 1 = l 2a 2
a1 (1.25)2
fi l2 = l1 = ¥ 6 cm
a2 (15)2
So, the distance moved out by the larger piston is 0.042 cm.
Problem 6.4 In a hydraulic press, the two pistons are of diameters of 30.0 cm and 2.5 cm.
Estimate the force exerted by the larger piston when 50.0 kg wt. is placed on smaller piston.
When the stroke of the smaller piston is 4.0 cm, what is the distance through which the larger
piston would move after 10 strokes. (IIT Roorkee, 1978)
5QNWVKQP
 A1 = p(2.5/2) cm2;
A2 = p(30/2)2 cm2; F1 = 50 kg wt.

F1 50 ¥ p ¥ (30/2)2
Now, F2 = ¥ A2 =
A1 p (2.5/2)2
50 ¥ 30 ¥ 30
= = 7200 kg wt.
2.5 ¥ 2.5
In one stroke
Input work done = Output work done
So, F 1l 1 = F 2l 2
F1l1 50 ¥ 4
or l2 = = = 0.028
F2 7200
\ Distance covered after 10 strokes
= 0.028 ¥ 10 = 0.28 cm.
Problem 6.5 In a car lift compressed air exerts a force F1 on a small piston having a radius
of 5 cm. This pressure is transmitted to a second piston of radius 15 cm. If the mass of the
car to be lifted is 1350 kg, what is F1? What is the pressure necessary to accomplish this
task? Take g = 9.81 m/s2.
5QNWVKQP As the pressure through air is transmitted equally on both the pistons, so
F1 F
= 2
A1 A2
980പWŚLJƐŝĐƐĨŽƌ//dͲ:͗DĞĐŚĂŶŝĐƐ//

Here F2 = mg = 1350 ¥ 9.81 N


A1
\ F1 = F2
A2
p r12 r12
= F2 = F2
p r22 r22

52
= ¥ 1350 ¥ 9.81
152
= 1.47 ¥ 103
Required air pressure,
F1 1.47 ¥ 10 3
P = =
A1 p (0.05)2
= 1.87 ¥ 105 N/m2
Pressure Head: The vertical height of the free surface above any point in a liquid at rest is
P
known as pressure head, it is expressed as h = .
rg
Since the pressure at any point in a liquid depends on the height of the free surface above
the point, so it is convenient to express a liquid pressure in terms of pressure head.
Hydrostatic Paradox: It was experimentally demonstrated by Pascal that the pressure
exerted by a liquid depends only on the height of the liquid and not on the shape of the
containing vessel. Figure 6.13 shows three vessels of different shapes. When the vessels
are filled with the same liquid up to the same height, the pressure meters record the same
pressure in all the three vessels, even the amount of liquid is different in the vessels. This
apparent result is known as hydrostatic paradox.

A B C

hrg hrg hrg

(a) (b) (c)

Fig. 6.13

Explanation: In vessel A, the pressure exerted by a liquid is normal to the wall. The pressure
acts horizontally on the walls. The reaction R of the walls is also horizontal.
,LJĚƌŽƐƚĂƟĐƐŽƌ&ůƵŝĚ^ƚĂƟĐƐപ981

V
R R
R R
Liquid H Liquid H
Liquid
R R
V

Container wall (a) (b) (c)


Container wall Container wall

Fig. 6.14

In vessel B, the reaction R of the wall is inclined upwards. The vertical component V of
which decreases the downward thrust due to extra liquid.
In vessel C, the reaction R is involved in the downward direction. The vertical component
V of which increases the downwards thrust of the liquid.
Note: If W is the weight of liquid in vessel A and if base area of all the vessels are equal then
W
thrust at the bottom of vessel PA = .
A
If WB and WC are weights of liquid in vessels B and C respectively, then thrust at bottom
of vessel B,
W -V
PB = B
A
WC + V
and PC =
A
As PA = PB = PC
\ WB – V = W fi WB = W + V
Also, WC + V = W fi WC = W – V

Atmospheric Pressure
Earth is surrounded by different gases. The Vacuum
pressure exerted by atmospheric gases is called A
PA = 0
atmospheric pressure. The atmospheric pressure
at sea level is 1.013 ¥ 105 N/m2.
h
MEASURING PRESSURE Pa Pa

Mercury Barometer
Figure 6.15 shows a mercury barometer which was C B
first made by Torricelli. A long glass tube (may be
1 m long) is filled with mercury and inverted with Mercury
its open end in a dish of mercury. The mercury
level in the tube falls and comes to rest at a vertical
height of 76 cm above the mercury level in the dish. Fig. 6.15
982പWŚLJƐŝĐƐĨŽƌ//dͲ:͗DĞĐŚĂŶŝĐƐ//

The space above mercury in the tube is almost a vacuum, so PA = 0. At point C there is
only atmospheric pressure, so Pc = Pa which is equal to the pressure at B, because points B
and C are at the same level. Thus,
PC = PB
= PA + rgh
As PA = 0, \ PC = rgh
For mercury barometer, h = 76 cm = 0.76 m, r = 13.6 ¥ 103 kg/m3
Pc = Pa = 0.76 ¥ 13.6 ¥ 103 ¥ 9.81
= 1.013 ¥ 105 N/m2

Open Tube Manometer


Pa
It is used to measure the gauge pressure of the gas. It
consists of an U-tube containing a liquid (mercury),
with one end of the tube connected to the vessel whose
pressure is to be measured.
The total pressure P of the gas is equal to the pressure
at A.
Gas h
By Pascal’s law, PA = PB
Pressure
or P = Pa + rgh P
or P – Pa = rgh
B
A
or P – Pa = rgh
Here P – Pa is the gauge pressure of the gas.

Absolute Pressure and Gauge Pressure


Fluid pressure may be measured with respect to any
arbitrary reference level (datum). The two most commonly
used datum are (i) absolute zero pressure and (ii) local Fig. 6.16
atmospheric pressure. When pressure is measured above
absolute zero, it is called an absolute pressure. When it is measured either above or below
the atmospheric pressure as a datum, it is called gauge pressure. Thus,
Gauge pressure = Absolute pressure – Atmospheric pressure
or Pgauge = Pabsolute – Pa
or we can write Pabsolute = Pgauge + Pa

For Your Knowledge


s 7HILE DESCRIBING A mUID WE ARE CONCERNED WITH PROPERTIES THAT VARY FROM POINT TO
point and not with properties associated with a specific piece of matter. So the role of
force in a solid is replaced in a fluid by pressure and that of mass by density.
,LJĚƌŽƐƚĂƟĐƐŽƌ&ůƵŝĚ^ƚĂƟĐƐപ983

s ! mUID EXERTS PRESSURE NOT ONLY ON A SOLID PIECE IMMERSED IN mUID OR ON THE WALLS
of container, fluid pressure exists at all points in a fluid. A volume element (of fluid)
inside a fluid is in a equilibrium because the pressure exerted on its various faces get
balanced.
s 0RESSURE AT A POINT IN A LIQUID ACTS EQUALLY IN ALL DIRECTIONS
s 0RESSURE IN A LIQUID IS THE SAME FOR ALL POINTS AT THE SAME HORIZONTAL LEVEL
s 0RESSURE IN A LIQUID INCREASES WITH DEPTH h according to the relation, P = Pa + h r g
This expression is valid only for incompressible fluids i.e., liquids.
s ,IQUID PRESSURE IS INDEPENDENT OF THE AREA AND THE SHAPE OF THE CONTAINING VESSEL
s 4HE MEAN PRESSURE ON THE WALLS OF A VESSEL CONTAINING LIQUID UP TO HEIGHT h is hr g/2.
s -OST OF THE PRESSURE MEASURING DEVICES MEASURE THE PRESSURE DIFFERENCE BETWEEN THE
true pressure and the atmospheric pressure. This difference is called gauge pressure and
the pressure is called absolute pressure.
Absolute pressure = Gauge pressure + Atmospheric pressure
i.e., P = P g + Pa
s 4HE GAUGE PRESSURE MAY BE POSITIVE OR NEGATIVE DEPENDING ON P > Pa or P < Pa. In
inflated tyres or the human circulatory system, the absolute pressure is greater than
atmospheric pressure, so gauge pressure is positive, called the overpressure. However,
when we suck a fluid through a straw, the absolute pressure in our lungs is less than
atmospheric pressure and so the gauge pressure is negative.
s ! DIVER IN WATER AT A DEPTH OF  M IS UNDER TWICE THE ATMOSPHERIC PRESSURE
s !T A DEPTH OF  KM IN THE SEA THE INCREASE IN PRESSURE IS  ATM 3UBMARINES ARE
designed to withstand such high pressures.
s 4HE PRESSURE AT THE CENTRE OF THE EARTH IS ESTIMATED TO BE  MILLION ATMOSPHERES

Height of Atmosphere
In actual practice the value of g and density of air decrease with height, so the atmosphere
extends with decreasing pressure even beyond hundreds of kilometre. Making the following
assumptions:
(i) The density of air, assuming constant r = 1.3 kg/m3.
(ii) Temperature remains constant throughout the atmosphere.
(iii) The value of g does not change with height.
Atmospheric pressure Pa = rgh
Pa 1.013 ¥ 10 3
\ h = = = 7950 mm or 8 km
rg 1.3 ¥ 9.81

Barometric Formula or Variation of Pressure in the Atmosphere


If it is assumed that pressure P and density r vary according to isothermal condition, then
according to the gas law, we have
m
PV = RT
M
Dm
and PDV = RT
M
984പWŚLJƐŝĐƐĨŽƌ//dͲ:͗DĞĐŚĂŶŝĐƐ//

Ê Dm/DV ˆ
or P =Á RT
Ë M ˜¯

Ê RT ˆ
or P = rÁ
Ë M ˜¯

Ê RT ˆ
Also, P0 = r 0 Á
Ë M ˜¯
P r Ê Pˆ
\ = fi r = Á ˜ r0
P0 r0 Ë P0 ¯
where P0 and r0 are the values of the pressure and density of the air at reference level (at
ground level).
We know that dP = – rg dy
y=h P
Ê Pr ˆ
= - Á 0 ˜ gdy
Ë P ¯
0

dP r g
\ = 0 (- dy )
P P0
Integrating the above equation, we have
P
dP r g h
Ú P
= – 0 Ú dy
P0 0
P0
-r0 g h y=0 P0
|lnP|PP0 = |y|0
P0
Ground level
P -r0 gh
ln = Fig. 6.17
P0 P0
or P = P0e–r0gh/P0

WORKED PROBLEMS
Problem 6.6 A barometer tube is inclined at an angle
of 60° with the horizontal direction. What will be the
length of mercury column in a barometer tube, when the 60°
atmospheric pressure is 75 cm of mercury?
(BITSAT, 2016)
5QNWVKQP The barometric height h = 75 cm h 
If l is the length of the mercury column in the tube,
h
then = sin 60°
l
h 75
or l = =
sin 60∞ ( 3/2) Fig. 6.18
= 86.6 cm
,LJĚƌŽƐƚĂƟĐƐŽƌ&ůƵŝĚ^ƚĂƟĐƐപ985

Problem 6.7 A U-tube contains water and methylated spirit


separated by mercury. The mercury columns in the two arms
are in level with 10 cm of water in one arm and 12.5 cm of spirit
in the other. What is the specific gravity of spirit?
Spirit
5QNWVKQP As the mercury column in the two arms of the U-tube
are the same, so 12.5 cm

PA = PB
10 cm
or rw gh w = rs gh s Water

rs h
or = w A B
rw hs

10
= = 0.8
12.5 Mercury

Thus, specific gravity of spirit is 0.8. Fig. 6.19

Problem 6.8 What is the pressure on a swimmer 10 m below the surface of a swimming
pool? Take atmospheric pressure = 1 ¥ 105 N/m2. (EAMCET, 2014)
5QNWVKQP Pressure at any depth h from the free surface of the water is given by
P = rgh + Pa
= (1000) ¥ 9.8 ¥ 10 + 1.0 ¥ 105
= 1.98 N/m2
Problem 6.9 What is the gauge pressure and absolute pressure of the gas above the liquid
surface in the tank shown in Fig. 6.20. Density of oil = 820 kg/m3, density of mercury = 13.6
¥ 103 kg/m3. Given 1 atmospheric pressure = 1.01 ¥ 105 N/m2.
Pa

Gas

1m
0.75 m

1.5 m
Oil
A B

Mercury

Fig. 6.20

5QNWVKQP Suppose Pgas is the pressure of the gas on the oil. As the points A and B are at the
same level in the mercury columns, so
986പWŚLJƐŝĐƐĨŽƌ//dͲ:͗DĞĐŚĂŶŝĐƐ//

PA = PB
or Pgas + roil ghoil = Pa + rHg ghHg
or Pgas + 820 ¥ 9.8 ¥ (1 + 1.50) = Pa + 13.6 ¥ 103 ¥ 9.8 ¥ (1.5 + 0.75)
or Pgas + 20.0.9 ¥ 103 = Pa + 299.88 ¥ 103
\ Pgas – Pa = 299.88 ¥ 103 – 20.09 ¥ 103
or [Pgas]gauge = 279.8 ¥ 103 N/m2
= 2.8 ¥ 105 N/m2
Absolute pressure of gas
[Pgas]absolute = [Pgas]gauge + Pa
= 2.8 ¥ 105 + 1.01 ¥ 105
= 3.81 ¥ 105 N/m2

PA
&KHHGTGPVKCN /CPQOGVGT It is used to measure pressure
difference between two points. z
Pressure difference between the points A and B can be
obtained as: PB
By Pascal’s law
y
PM = PN
r3 r1
or PA = r3g(x + y + z) = PB + r2gx + r1gy
O Q
\ PA – PB = (r2 – r3)gx + (r1 – r3)gy – r3gz
x
Note: According to Pascal’s law the pressure at two points
M N
on the same level in a liquid at rest are equal. So it should r2
be remembered that the two points chosen must be on the
same level of the same liquid. In the above case PM = PN,
but PO π PQ .
Fig. 6.21
Problem 6.10 Find the value of h if the pressure difference between the vessels A and B
is 3 kN/m2.

h Kerosene

Water
20 cm

Water
Fig. 6.22
,LJĚƌŽƐƚĂƟĐƐŽƌ&ůƵŝĚ^ƚĂƟĐƐപ987

5QNWVKQP Let P = Pressure at Q and R and r1, r0 be the densities of kerosene and water
respectively.
Q R

h1
h2
h

A
h0
Water
20 cm

Water
Fig. 6.23

P B = P Q + r 1h 1 + r 0h 0 ...(i)
Ê 20 ˆ
PA = PR + r1h2 + r0 Á h0 - h - ˜ ...(ii)
Ë 100 ¯
PB – PA = 3 ¥ 103 N/m2 (given) ...(iii)
Using Eqs. (i), (ii) and (iii),
h = 0.5 m
Problem 6.11 A manometer reads the pressure To Pump
of a gas in an enclosure as shown in Fig. 6.24(a).
When some of the gas is removed by the pump, 20 cm 18 cm
the manometer reads as shown in Fig. 6.24(b). The B B A
A
liquid used in manometers is mercury and the
atmospheric pressure is 76 cm of mercury. Hg

(i) Give the absolute and gauge pressure of the (a) (b)
gas in the cases (in units of cm of mercury).
Fig. 6.24
(ii) How would the levels change in case (b) if
13.6 cm of water is poured into the right limb of the manometer? (Ignore the change
in volume of the gas.) (NCERT)
5QNWVKQP Here, atmospheric pressure, P = 76 cm of mercury
(i) In Fig. 6.24(a) pressure head, h = + 20 cm
\ Absolute pressure = P + h = 76 + 20 = EOQHOGTEWT[
Gauge pressure = h = EOQHOGTEWT[
In Fig. 6.24(b) pressure head, h = – 18 cm
Absolute pressure = P + h = 76 + (– 18) = EOQHOGTEWT[
Gauge pressure = h = ŌEOQHOGTEWT[
988പWŚLJƐŝĐƐĨŽƌ//dͲ:͗DĞĐŚĂŶŝĐƐ//

13.6
(ii) Here 13.6 cm of water added in right limb is equivalent to = 1 cm of mercury
column. 13.6
i.e., h¢ = 1 cm of mercury column.
Now pressure at A, PA = P + h¢ = 76 + 1 = 77 cm
Let the difference in mercury levels in the two limbs be h1, then pressure at B, PB = 58
+ h1
As, PA = PB, \ 77 = 58 + h1 or h1 = 77 – 58 = EOQHOGTEWT[EQNWOP
Problem 6.12 A manometer tube contains a liquid of density 3 ¥ 103 kg m–3. When connected
to a vessel containing a gas, the liquid level in the other arm of the tube is higher by 10 cm.
When connected to another sample of enclosed gas, the liquid level in the other arm of the
manometer tube falls 7 cm below the liquid level in the first arm. Which of the two samples
exerts more pressure and by what amount? P a

5QNWVKQP For Sample 1


h1 = 10 cm = 0.1 m
P1 = Pa + rgh1 ...(i)
For Sample 2
h1 = 10 cm
In this case, level of the liquid in the left arm is higher than that in
Gas
the right arm by 7 cm.
1
\ Atmospheric pressure Pa is greater than the pressure exerted by P1
the sample
fi Pa = P2 + rgh2
fi P2 = Pa – rgh2 ...(ii)
Fig. 6.25
Comparing Eqs. (i) and (ii), it is clear that P1 > P2. Therefore,
the gas in sample 1 exerts greater pressure than that in sample 2.
The difference in the pressures is
P1 – P2 = (Pa + rgh1) – (Pa – rgh2)
Gas
= rg(h1 + h2)
2
= rg(17 cm) P2
h2 = 7 cm
= (3 ¥ 103 kg m–3) ¥ (9.8 ms–2) (0.17 m) Pa
= 4.99 ¥ 103 Pa
ª 5 kPa
Fig. 6.26

Problem 6.13 At a depth of 500 m in an ocean, what is the absolute pressure? Given that
the density of seawater is 1.03 ¥ 103 kg/m3 and g = 10 ms–2.
5QNWVKQP Absolute pressure P = Pa + rgh
Here, Pa = 1.01 ¥ 105 Pa
r = 1.03 ¥ 103 kg m–3
\ P = 1.01 ¥ 105 Pa + 1.03 ¥ 103 ¥ 10 ¥ 500 Pa
,LJĚƌŽƐƚĂƟĐƐŽƌ&ůƵŝĚ^ƚĂƟĐƐപ989

= 1.01 ¥ 105 Pa + 51.5 ¥ 105 Pa


= 52.5 ¥ 105 Pa
ª 52 atm

Problem 6.14 A U-tube, in which both ends are open to the atmosphere, is partly filled with
water. Oil, which does not mix with water, is poured into one side until it stands a distance
d = 12.3 mm above the water level on the other side, which has meanwhile risen a distance
a = 67.5 mm from its original level (Fig. 6.27). Find the density of the oil.
5QNWVKQP In Fig. 6.27 points C are at the same pressure. (If
this were not true, then the U-shaped fluid element below
d
the CC level would experience a net unbalanced force
and would accelerate, violating the static assumption we
Oil a
make in this problem.) The pressure drop from C to the Initial
surface on the water side is rw g2a, where 2a is the height water
of the water column above C. The pressure drop on the level
a
other side from C to the surface is rg(2a + d), where r is
the unknown density of the oil. Equating the pressures C C
at point C on each side, we obtain Water
p0 + rw g2a = p0 + rg(2a + d),

and so
2a Fig. 6.27 A U-tube is filled partly
r = rw
( 2 a + d) with water and partly with
oil of unknown density.
2(67.5 mm)
= (1.000 ¥ 103 kg/m3)
2(67.5 mm) + 12.3 mm
= 916 kg/m3.
The ratio of the density of a substance to the density of water is called the relative density
(or the specific gravity) of that substance. In this case the specific gravity of the oil is 0.916.
Note that in solving this problem, we have assumed that the pressure is continuous
across the interface between the oil and the water at point C on the left side of the tube. If
this were not so and the pressures were different, then the force exerted by the fluid on one
side of the interface would differ from that of the fluid on the other side, and the interface
would accelerate under the influence of the unbalanced force. Since we are assuming a
static situation, there can be no motion and the pressures must therefore be the same. When
we first pour the oil into the tube, however, there may be a difference in pressure and an
unbalanced force that would cause the system to move until it reached the static situation
shown in Fig. 6.27.
Problem 6.15 A U-tube in which the cross-sectional area of the limb on the left is one third
limb on the right contains mercury (density 13.6 g/cm3). The level of mercury in the narrow
limb extends to a distance of 30 cm from the upper end of the tube. What will be the rise in
the level of mercury in the right limb. If the left limb is filled to the top with water (neglect
surface tension effects).
990പWŚLJƐŝĐƐĨŽƌ//dͲ:͗DĞĐŚĂŶŝĐƐ//

5QNWVKQP
a 3a

30 cm
30 cm

Hg x
B
A Hg

(a) (b)

Fig. 6.28

Suppose area of cross section of the narrow limb is a, then the area of cross-section of the
right limb will be 3a. Let the level of mercury in the left limb fall by x and the rise of level
in the right limb is y, then
ax = (3a)y
or x = 3y
According to Pascal’s law, PA = PB
or (30 + x)rwg = (x + y)rHgg
(30 + 3y) ¥ 1 ¥ g = (3y + y) ¥ 13.6 ¥ g
fi y = 0.58 cm
Problem 6.16 A glass full of water has a bottom of area 20 cm2, top of area 20 cm2, height
20 cm and volume half a litre.
(a) Find the force exerted by the water on the bottom.
(b) Considering the equilibrium of the water, find the resultant force exerted by the sides
of the glass on the water. Atmospheric pressure = 1.0 ¥ 105 N/m2. Density of water =
1000 kg/m3 and g = 10 m/s2.
5QNWVKQP
(a) The pressure intensity at the bottom of the container is
P = h r g = 0.20 ¥ 1000 ¥ 10 = 2 ¥ 103 N/m2
20 cm
Force at the bottom of the container
= P ¥ Base area
= 2 ¥ 103 ¥ (20 ¥ 10– 4) 20 cm2
= 4 N (Force exerted by water only) Ans. Fig. 6.29
(b) Weight of the water in the container
= Volume ¥ Density of water ¥ g
= 0.5 ¥ 103 ¥ 1000 ¥ 10 = 5 N
,LJĚƌŽƐƚĂƟĐƐŽƌ&ůƵŝĚ^ƚĂƟĐƐപ991

P2 > P1

P1 P1

P2 P2 5N

4N
Force exerted by Force exerted by
water on the container container on water

Fig. 6.30

From the diagram, the force exerted by the container is in upward direction (because
(P2 > P1). Let force exerted by sides of container be F, then
\ 4+F =5N
fi F =1N
Problem 6.17 A tube 1 cm2 in cross section is attached to the top of a vessel 1 cm high and
of cross section 100 cm2. Water is filled up to a height of 100 cm from the bottom of the vessel.
(a) What is the force exerted by the water against the bottom of the vessel?
(b) What is the weight of the water in the vessel? 1 cm2
(c) Explain why (a) and (b) are not equal.
5QNWVKQP
(a) Intensity of pressure at the bottom of the container is
P = hrg V2 99 cm
= 1 ¥ 1000 ¥ 10
= 1 ¥ 104 N/m2
Force exerted by water at the bottom
F =P¥A V1 1 cm
= (1 ¥ 10 ) ¥ (100 ¥ 10 )
4 –4
100 cm2
= 100 N
Fig. 6.31
(b) Weight of the water in the container
W = (V1 + V2)rg
fi (A1l1+ A2l2)rg = [100 ¥ 10– 4 ¥ 1 ¥ 10–2 + 1 ¥ 10– 4 ¥ 99 ¥ 10– 2] ¥ 1000 ¥ 10
= 1.99 N
(c) As we have seen in (a) and (b) the thrust of water at the bottom of the vessel is greater
than the weight of the water. It is because of the force exerted by the top face of the
vessel on the water. Water transfers this force to the bottom of the vessel.

HYDROSTATIC FORCES ON SURFACES


Total Pressure and Centre of Pressure
When a static fluid comes in contact with a surface, either plane or curved, a normal force
is exerted by the fluid on the surface. This force is known as total pressure. The point of
application of total pressure on the surface is known as the centre of pressure.
992പWŚLJƐŝĐƐĨŽƌ//dͲ:͗DĞĐŚĂŶŝĐƐ//

 K  6QVCNRTGUUWTGQPCRNCPGJQTK\QPVCNUWTHCEG
Consider a plane surface inside a liquid of density Free surface of liquid
r, such that it is held in a horizontal position at a
depth h below the free surface of the liquid as shown h F
in Fig. 6.32. Since every point on the surface is at the
same depth h, so the intensity of pressure is constant
over the entire surface, being equal to rgh. Thus, if
A is the area of the surface, then the total pressure Fig. 6.32
on the horizontal surface is
F = PA = rgh A.
KK  6QVCNRTGUUWTGKPKPENKPGFXGTVKECNRNCPGUWTHCEG
Consider a plane surface of length a and width
b which is inside a liquid of density r as shown O
in Fig. 6.33. q
y
Choose an element of length (dy) at a distance h
y from O.
The depth of the element h = y = cos q 
dy
The intensity of pressure at the position of
element
dy
P = rgh = rg (y cos q)
The force on the element, dF = P (bdy)
a
b
= r g(y cos q) bdy
= r gb cos q y(dy) Fig. 6.33
(l + a)
The force on the entire surface, F = Úl dF
(l + a)
= r gb cos q Úl y dy
b cos q 2 (l + a)
= rg y l
2
cos q
or F = r gb [(l + a)2 - l 2 ]
2
r gb cos q 2
F = [a + 2 al]
2
Special cases
r gb cos q 2
(i) For l = 0, F = [a + 0]
2
1
= r gba2 cos q
2 F
1
(ii) For l = 0, and q = 0°, F = r gb cos 0° a2 Centre of pressure
2
Fig. 6.34
rgba 2
=
2
,LJĚƌŽƐƚĂƟĐƐŽƌ&ůƵŝĚ^ƚĂƟĐƐപ993

WORKED PROBLEMS
Problem 6.18 Suppose water stands at a depth of H behind the vertical face of a dam of
width L. Find force exerted by the water on the wall and centre of pressure.
5QNWVKQP Choose an element of thickness (dy) at a depth y. The pressure at the position of
the element
P = rgy
The force on the element dF = P(Ldy)

y
H dy Dam dy
O
L
Pressure diagram
(a) (b)

Fig. 6.35

or dF = rgy (Ldy) = r gL (ydy)


The total force on the dam
H H
F = Ú dF = r gL Ú ydy
0 0
rgL 2 H
= y
2 0

rgLH 2
or F =
2
How to Find Centre of Pressure?
The moment of the force dF about an axis through O (see Fig. 6.35(a)).
or Torque dt = dF (H – y)
= rgL (ydy)(H – y)
= rgL(Hy – y2)dy
F Dam
H
Net torque t = rgL Ú ( Hy - y )dy (Hy – y )dy
2 2 H = H /3
0
H
Hy 2 y 3 O
= rgL -
2 3 Fig. 6.36
0

ÊH H 3 3ˆ
= rgL Á -
Ë 2 3 ˜¯
1
t = rgLH 3
6
994പWŚLJƐŝĐƐĨŽƌ//dͲ:͗DĞĐŚĂŶŝĐƐ//

If H is the height above O at which the total force F would have to act to produce this
torque, then (see Fig. 3.36)
FH = t 1
r gLH 3
t 6 H
or H = = 2
=
F r gLH 3
2
Pressure Diagram
Net force (total pressure) as well as point of application of force (centre of pressure) for a
plane surface wholly submerged in a static liquid, either vertically or inclined, may also be
determined by drawing a pressure diagram. A pressure diagram is a graphical representation
of the variation of the pressure intensity over a surface. Such a diagram may be prepared by
plotting to some convenient scale the pressure intensities at various points on the surface.
Free liquid surface

h1 rgh1
h1 h1

rgh1
rgh1
Horizontal plane h2 h2
surface

rgh2
rgh2
Vertical plane Inclined plane
surface surface

Fig. 6.37

Since force at any point acts in the direction normal to the surface, the pressure intensities
at various points on the surface are plotted normal to the surface. Figure 6.38 shows
typical pressure diagrams for horizontal, vertical and inclined plane surfaces. Consider a
rectangular plane surface of depth a and width b, held vertically in static liquid of density r
as shown in Fig. 6.37. Let top and bottom edges of the plane surface be at vertical depth of
h1 and h2 respectively below the free surface of the liquid. The pressure intensity at the top
edge, P1 = rgh1. The pressure intensity at the bottom edge, P2 = rgh2. As the pressure intensity
increases linearly from rgh1 to rgh2, so the average pressure intensity over the entire surface.
P1 + P2
Pav =
2
Êh +h ˆ
= rg Á 1 2 ˜
Ë 2 ¯
,LJĚƌŽƐƚĂƟĐƐŽƌ&ůƵŝĚ^ƚĂƟĐƐപ995

Thus, the net force on the vertical surface


F = Pav ¥ Area of the surface

Êh +h ˆ
or F = rg Á 1 2 ˜ ¥ ab
Ë 2 ¯

WORKED PROBLEMS
Problem 6.19 To what height should a cylindrical vessel of radius R be filled with a
homogeneous liquid to make the force with which the liquid presses on the sides of the
vessel equal to the force exerted by the liquid on the bottom of the vessel?
5QNWVKQP Let h be the height of the liquid in the vessel.
The average pressure at the side of the vessel
(PTop + PBottom ) 0 + r gh r gh
Pav = = =
2 2 2
Force at the side of vessel, Fside = Pav (2pRh)

Ê r gh ˆ
= Á (2p Rh)
Ë 2 ˜¯
The pressure at the bottom of the vessel remains uniform,
P = rgh
Force at the bottom of the vessel,
Fbottom = P(pR2)
rgh
= rgh(pR2)
Given, Fside = Fbottom rgh
Pressure diagram
r gh
or (2p Rh) = rgh(pR2) Fig. 6.38
2
fi h =R
E F
Force on Curved Surface
Consider a curved surface inside a static liquid of density
r. Figure 6.39 shows the trace of the curved surface which
extends in the direction normal to the plane of the paper.
At any point on the curved surface the force acts normal h
to the surface. Choose a small element of area dA of the
D A
curved surface lying at a vertical depth of h below the free
dF dFy
surface of the liquid. The force on the element
q
dF = P(dA) = rgh(dA) ...(i) dA
dFH
The force dF acting on the element can be resolved into B
horizontal and vertical components C

Fig. 6.39
996പWŚLJƐŝĐƐĨŽƌ//dͲ:͗DĞĐŚĂŶŝĐƐ//

dFH = dF sin q = rgh (dA) sin q ...(ii)


and dFV = dF cos q = rgh (dA) cos q ...(iii)
Here q is the inclination of the elementary area with the horizontal. The total horizontal
and vertical components of the force can be obtained by integrating equations (ii) and (iii), so
FH = Ú dFH = r g Ú h(dA)sin q ...(iv)

and FV = Ú dFV = r g Ú h(dA)cos q ...(v)

In Eq. (iv) (dA sin q) is the vertical projection of the area dA and in equation (v), (dA cos q)
is the horizontal projection of the elementary area dA. Thus, rg Ú h(dA) (dA) sin q represents
the total force on the vertical projection of the curved surface, i.e.,

ÈTotal force on the projected area of the ˘


Í ˙
FH = Ícurved surface on the vertical plane, the ˙
ÍÎ trace of which is represented by CD. ˙˚

Further rg Ú h(dA) cos q represents the total force on the horizontal projection of the curved
surface, and it is equal to the weight of the liquid contained in the portion extending above
the curved surface in the portion ABCDEFA, i.e.,

ÈThe weight of the liquid contained in the portion ˘


Í ˙
Fv = Íextending vertically above the curved surface up to˙
ÍÎ the free surface of the liquid. ˙˚

The resultant force on the curved surface F = FH2 + FV2 .

WORKED PROBLEMS
Problem 6.20 Compute the resultant force on the gate AB as a result of hydrostatic pressure
Fig. 6.40. The gate is 1.5 m wide. Determine the line of action of this force.
Pa
O

5m y
5m
3
Water, r = 1000 kg/m
60°
A dy
3m A
Pa
60° dy
B B
1.5
m

(a) (b)

Fig. 6.40
,LJĚƌŽƐƚĂƟĐƐŽƌ&ůƵŝĚ^ƚĂƟĐƐപ997

5
5QNWVKQP From Fig. 6.40(b) distance OA = = 5.77 m
sin 60∞
The distance OB = 3 + 5.77 = 8.77 m
Choose a small strip of the gate of length (dy) at a distance y from O. The pressure intensity
at the position of the strip, P = rg(y sin 60°) = 0.87 rgy
The force of this strip, dF = P(1.5dy)
= 0.87 rgy ¥ 1.5(dy) = 1.305 rgy (dy)
The resultant force on the gate
8.77
OB y2
F = 1.305 rg Ú ydy = 1.305 r g
OA 2
5.77

Ê 8.77 2 - 5.77 2 ˆ
= 1.305 ¥ 1000 ¥ 9.8 ¥ Á ˜
Ë 2 ¯
= 2.78 ¥ 10 N
5

The effect of air pressure gets cancelled out from both sides of the gate.
To find the line of action of this force, assume that resultant force acts at a distance y from
O, now equal the moment of this resultant about O with the moment of the distribution
about the same axis. Thus,
8.77
F
Fy = Ú (dF )y
O
5.77

8.77
= Ú 1.305 rgy(dy )y y = 7.4 m
5.77
8.77

Ú
A
= 1.305 rg y 2 dy F
5.77
Gate
8.77 Centre of
y3 pressure B
= 1.305 rg
3
5.77 Fig. 6.41

1.305 rg Ê 8.77 3 - 5.77 3 ˆ


\ y = Á ˜
F Ë 3 ¯

1.305 ¥ 1000 ¥ 9.8 Ê 8.77 3 - 5.77 3 ˆ


= Á ˜
2.78 ¥ 10 5 Ë 3 ¯
= 7.4 m
Problem 6.21 A conical cup of height b, semivertical angle a rests on the open end down
on a flat surface as shown in Fig. 6.42. The cup is filled to height h with liquid of density r.
What is the upward lifting force on the cup?
998പWŚLJƐŝĐƐĨŽƌ//dͲ:͗DĞĐŚĂŶŝĐƐ//

5QNWVKQP Let W be the weight of the liquid and F is the


thrust of the liquid at the bottom of the container, then a
for equilibrium of the liquid
F = W + Fv
b
where Fv is the vertical force.
r
\ Fv = F – W ...(i) h
dy
F = P ¥ base area y

= (rgh) ¥ p (b tan a)2 b tan a


= prghb2 ¥ tan2 a
Fig. 6.42
W = (r ¥ Volume) g
h F
= gr Ú p r 2 dy
o
h
= p g r Ú [(b - y )tan a ]2 dy
o
h
= p g r tan 2 a Ú (b - y )2 dy
o

Ê h3 ˆ
= pr g tan 2 a Á b 2 h - bh 2 + ˜
Ë 3¯ W

Substituting these values in Eq. (i), we get Fig. 6.43


Ê h3 ˆ
Fv = F – W = prg tan2 a Á bh 2 - ˜
Ë 3¯

Problem 6.22 Compute the horizontal and vertical components of the resultant of the
hydrostatic pressure distribution on the gate AB, which is a quarter of a cylinder (Fig. 6.44).
Assume the gate is 3 m wide.

Water r = 100 kg/m3

10 m
y
13 m
B 13 m
B
3m

C dy

A
A A¢
(a) (b)

Fig. 6.44
,LJĚƌŽƐƚĂƟĐƐŽƌ&ůƵŝĚ^ƚĂƟĐƐപ999

5QNWVKQP
(a) Horizontal component: The horizontal component of the resultant force on the gate is
equal to the force on the vertical projection BA¢ of the gate. The pressure at a depth y
from free surface of the water, P = rgy. The force on the strip of area (3dy) of the gate,
dFH = P(3dy) = rgy(3dy)
= 3rgydy
13
13 y2
Total horizontal force, FH = 3 r g Ú ydy = 3 r g
10 2
10

Ê 132 - 10 2 ˆ
= 3 ¥ 1000 ¥ 9.8 ¥ Á ˜
Ë 2 ¯
= 1.014 ¥ 106 N
(b) Vertical component: It is equal to the weight of the water above the curved surface.
Dividing the volume above the curved surface into a rectangular parallelopiped and
a quarter cylinder, we thus have:
Fv = Weight of water in DEFB + Weight of water in ACBF
= r[V1 + V2]g D E
È p(3)2 ¥ 3 ˘
= 1000 Í10 ¥ 3 ¥ 3 + ˙ ¥ 9.8
Î 4 ˚ 10 m
= 1.09 ¥ 10 N 6
F
The resultant force on the gate B

3m
F = FH2 + FV2 C

= 1.49 ¥ 10 N 6 A

Fig. 6.45
BUOYANT FORCE AND BUOYANCY
Body inside a fluid experiences pressure on its all faces. As the fluid pressure increases with
depth, so the upward thrust at the bottom is more than the downward thrust on the top.
Hence, a net force acts in upward direction. This upward force acting on a body in fluid
is called upthrust orDWQ[CPVHQTEG and the phenomenon is called buoyancy. The point of
application of buoyant force is called centre of buoyancy. It is the centre of gravity of the
displaced fluid.

ARCHIMEDES’ PRINCIPLE
The principle was discovered by the Greek scientist, Archimedes around 225 BC. It states
that when a body is immersed in a fluid, partially or wholly, it experiences an upward force
equal to the weight of the volume of the fluid displaced by the body.
2TQQH To understand easily, take a body height h and area A, lying inside a liquid of density
r. The top face of the body is at a distance y from the free surface of the liquid.
1000പWŚLJƐŝĐƐĨŽƌ//dͲ:͗DĞĐŚĂŶŝĐƐ//

Pressure at the top face of the body, P1 y


P1 = rgy
Pressure at the bottom face of the body, h

P2 = rg(y + h)
P2
Thrust on the top face of the body,
Fig. 6.46
F1 = P1A = rghA acting vertically downwards.
Thrust acting on the bottom face of the body
F2 = P2 A = r g (y + h) A acting vertically upwards.
The resultant force (F2 – F1) acts on the body in the upward direction and is called buoyant
force (Fb). Thus,
Fb = F2 – F1 = rg (y + h) A – r gyA
= (Ah)rg
But Ah = V, the volume of the body, which is equal to the volume of the liquid displaced.
\ Fb = Vrg
Thus, buoyant force is equal to the weight of the fluid displaced.

Apparent Weight of Immersed Body


Apparent weight = Actual weight – Buoyant force
Wapp = W – Fb
For a body completely inside the liquid, Fb = Vrg
\ Wapp = Vs g – Vrg
Ê rˆ
= Vs g Á 1 - ˜
Ë s¯

Ê rˆ
or Wapp = W Á 1 - ˜
Ë s¯
Here s is the density of the material of the body.
Note:
1. For a wholly immersed body of homogeneous composition the centre of buoyancy
will coincide with the centre of gravity.

Fb
Fb y/2 Centre of gravity
Centre of gravity
and centre of y/2 w
buoyancy Centre of buoyancy
w

Fig. 6.47
,LJĚƌŽƐƚĂƟĐƐŽƌ&ůƵŝĚ^ƚĂƟĐƐപ1001

2. When a body lies inside two or more immiscible liquids, the buoyant force on the body
is given by
Fb = Fb1 + Fb2 + ……
For two liquids
F b = V 1r 1g + V 2r 2g

v1
r1
v2
r2

Fig. 6.48

RELATIVE DENSITY
We know that,
Loss in weight = Buoyant force on the immersed body
= Weight of the body in air – Weight of the body in water
or Fb = Wair – Wwater
or Wair = Fb + Wwater ...(1)

Ê W ˆ
or Wair = Fb Á 1 + water ˜ ...(2)
Ë Fb ¯

If s is the density of material of body and rw is the density of the water, then equation
(2) can be written as
Ê F + Wwater ˆ
Vsg = V rw g Á b ˜¯
Ë Fb
In view of Eqs. (1) and(2), we have
s Wair
or =
rw Wair - Wwater

Wair
or R.D. =
Wair - Wwater

PRINCIPLE OF FLOATATION
The principle of floatation states that:
(i) weight of a body floating in a fluid is equal to the buoyant force which in turn is equal
to the weight of the fluid displaced by the body.
1002പWŚLJƐŝĐƐĨŽƌ//dͲ:͗DĞĐŚĂŶŝĐƐ//

(ii) The lines of action of weight of the body (CG) and the buoyant force (centre of
buoyancy) must lie on the same vertical line, so that their moment about any axis is
zero.
Fb
Fb

Fb
W
W

W
(i) W > Fb (ii) W > Fb (iii) W > Fb

Fig. 6.49

Three possible cases:


(i) When W > Fb: In case, when weight of the body is greater than the buoyant force, the
net force (W – Fb) acts in the downward direction and hence the body will sink.
W > Fb fi Vs g > Vrg or s > r.
Thus, the body will sink in a liquid if its density is greater than the density of the
liquid.
(ii) When W = Fb: The weight of the body is just equal to the buoyant force. No net force
acts on the body. The body floats fully immersed.
W = Fb fi Vsg = Vrg or s = r
(iii) When W < Fb: In case when weight of the body is less than the buoyant force on a fully
immersed body, the body will again float with less volume inside liquid. Here s < r.

Fractional Submerged Volume of a Floating Body


When density of the body (s) is less than the density of the liquid (r), the body floats
partially submerged. If V is the volume of the body and V¢ is the submerged volume, then
for a floating body
W = Fb
Vsg = V¢rg
V¢ s
or =
V r

Volume of Cavity Inside Metal


M
Consider a metal piece of mass M and density s. The volume of metal, V = .
s
When metal weight in water, let its weight is W¢. Thus,
Loss in weight in water = Weight in air – Weight in water
or Fb = W – W¢
V Metal
If V¢ is the total volume of the metal body, then
V¢rwg = W – W¢
Cavity
W - W¢
\ V¢ =
rw g
Fig. 6.50
,LJĚƌŽƐƚĂƟĐƐŽƌ&ůƵŝĚ^ƚĂƟĐƐപ1003

The volume of cavity = V¢ – V


Ê W - W¢ˆ M
= Á -
Ë rw g ˜¯ s

WORKED PROBLEM
Problem 6.23 The density of ice is 917 kg/m3. What fraction of ice lies below the water?
The density of seawater is 1024 kg/m3. What fraction of the iceberg do we assuming that it
has the same density as ordinary ice (917 kg/m3)?
5QNWVKQP According to law of floatation:
Weight of the ice = Weight of the water displaced
Vrice g = V¢rw g
rice 917
\ V¢ = V= V = 0.917 V
rw 1000
So, 91.7% of the ice is inside water,
Now if V1 is the volume of ice inside sea water, then
Vrice g = V1rw g
rice 917
\ V1 = V= V = 0.896 V = 0.896 V
rw 1024
The fraction of ice visible to us is
= V – V1 = V – 0.896 V
= 0.105 V = 0.104 V
So, 10.4% of the ice is visible to us.

Applications of Archimedes’ Principle


1. Buoyant force = weight of the liquid displaced by the immersed portion of the body
= Vrg where V is the volume immersed and r is the density of the liquid.
Case (a): If the density (s) of the body is less than that of the liquid r(s < r), then the
body will float with volume (V0 outside liquid and Vi, inside liquid) such that (V =
volume of the body)
Vi rg = Vs g
Vi s V r -s
fi = and 0 = (∵ V = V0 + Vi)
V r V r
Case (b): If s = r, the body will float completely immersed, i.e., V0 = 0.
Case (c): If s > r, the body will sink to the bottom.
2. Tension T in the string tied to a body immersed in a liquid (s > r).
Case (a): When the system is at rest.
T = Vsg – Vr g = V (s – r) g
1004പWŚLJƐŝĐƐĨŽƌ//dͲ:͗DĞĐŚĂŶŝĐƐ//

Case (b): When the system is accelerating, then


T = V (s – r)g¢
T
(i) If the system moves up with acceleration a,
g¢ = g + a and T¢ = V (s – r) (g + a)
(ii) If the system moves down with acceleration a (< g)
g¢ = g – a and T¢ = V (s – r) (g – a) Fig. 6.51
(iii) If the system falls freely,
a=g and T¢ = 0
3. Tension when body is kept immersed in a liquid (s < r) by a string fixed at the bottom
of the beaker
(i) If the system is at rest,
T = V (r – s) g
(ii) If the system moves up with acceleration a,
T¢ = V (r – s) g¢ where g¢ = (g + a) T

(iii) If the system moves down with acceleration a (< g)


T¢ = V (r – s) where g¢ = (g – a) Fig. 6.52

(iv) If the system falls freely, T ¢ = 0


4. A toy boat carrying an object is floating in water in a beaker. The object is dropped
into water.
1. If the object is made of wood (of density less than that of water), it will float and
the level of water in the beaker remains unchanged.
2. If the object is denser than water, it will sink and the water level will fall.
5. A piece of ice is floating in liquid. If the ice melts, the level of water:
(a) remains unchanged if the liquid is water;
(b) rises if the relative density of the liquid is greater than 1; and
(c) falls if the relative density of the liquid is less than 1.
6. A piece of ice with an object embedded in it is floating in water. If all the ice melts,
the water level
(a) falls if the object sinks in water; and
(b) remains unchanged if the object floats in water.

WORKED PROBLEMS
Problem 6.24 A solid floats in water with 3/4 of its volume below the surface of water.
Calculate the density of the solid.
5QNWVKQP If V is the volume and s is the density of the solid, then
Ê3 ˆ
Vsg = Á V ˜ rw g
Ë4 ¯
,LJĚƌŽƐƚĂƟĐƐŽƌ&ůƵŝĚ^ƚĂƟĐƐപ1005

3
or s = rw
4
3
= ¥ 1000 = 750 kg/m3
4

Problem 6.25 A solid weighs 10 N in air. Its weight decreases by 2N when weighed in
water. What is the density of the solid? (BITSAT, 2011)

Weight in air Weight in air


5QNWVKQP R.D. of solid = =
Loss in weight in water Upthurst
10
= =5
2
s s
or = 5 R.D. = solid
rw rwater
\ s = 5rw = 5 ¥ 1000 = 5000 kg/m3
Problem 6.26 A body of density r floats with a volume V1 of its total volume V immersed
in one liquid of density r1 and with the remainder of volume V2 immersed in another liquid
of density r2, where r1 > r2. Find the relative volume immersed in two liquids.
5QNWVKQP For a floating body in two liquids
Weight of the body = Buoyant force of I liquid + Buoyant force of II liquid
Vrg = V 1r 1g + V 2r 2g
or Vr = V 1r 1 + V 2r 2 ...(i)
Also, V = V1 + V2 ...(ii)
After solving Eqs. (i) and (ii), we get
Ê r - r2 ˆ
V1 = V Á apply V – V1 = V2
Ë r1 - r2 ˜¯

Ê r -r ˆ
and V2 = V Á 1 apply V – V2 = V1
Ë r1 - r2 ˜¯

Problem 6.27 A block of wood is floating on water at 0°C with a certain volume V above
the water level. The temperature of water is slowly raised from 0°C to 20°C. How will the
volume V change with the rise in temperature?
5QNWVKQP Suppose V¢ be the volume of the block of wood and W be the weight of the block.
For the floating block:
Weight of the block = Weight of the water displaced.
W = (V¢ – V)rt g
where rt is the density of water at t°C which is
1006പWŚLJƐŝĐƐĨŽƌ//dͲ:͗DĞĐŚĂŶŝĐƐ//

ro
rt =
1+g t
ro
\ W = (V ¢ - V ) g
(1 + g t)

W (1 + g t)
or V = V¢ -
ro g

Clearly V decreases with increase in temperature.


Problem 6.28 A large block of ice 5 m thick has a vertical hole drilled through it and is
floating in the middle of a lake. What is the minimum length of the rope required to scoop
up a bucket full of water through the hole? (Relative density of ice = 0.9)
5QNWVKQP The length of the rope required is equal to the height of the block above the water
level. Let it be y. If A is the area of base of the block, then

Hole

Bucket
5–y

lce block

Fig. 6.53

Weight of ice block = Weight of the water displaced


(A ¥ 5)rice g = A ¥ (5 – y)rwg
or 5rice = (5 – y)rw
or 5 ¥ 900 = (5 – y) ¥ 1000
which after solving gives, y = 0.5 m
Problem 6.29 A ball floats on the surface of water in a container exposed to the atmosphere.
Will the ball remain immersed at its initial depth or will it sink or rise somewhat if the
container is shifted to the moon?
5QNWVKQP The gravity on moon is about one sixth of that on the earth, i.e.,
gm = g/6
On the earth, water and air both exert buoyant force on the ball, but on the moon as there
is no air so weight of the ball is balanced only by buoyant force of the water. Thus,
,LJĚƌŽƐƚĂƟĐƐŽƌ&ůƵŝĚ^ƚĂƟĐƐപ1007

On the earth mg = Vw rw g + Va ra g
or m = Vw rw + Va ra ...(i)

Ê gˆ Ê gˆ
On the moon m Á ˜ = Vw¢ rw Á ˜ ...(ii)
Ë 6¯ Ë 6¯
or m = V¢w rw
From Eqs. (i) and (ii), we have
Va ra
V¢w = Vw +
rw

Clearly, V¢w > Vw, that is, the volume of the ball immersed in water on the moon will be
greater than that on the earth. Hence, the ball will sink slightly more in water on the moon.
Important:
1. If we neglected the effect of density of air on the earth, then
V¢w = Vw.
2. Gravity has equal effect both on weight and buoyant force, so equilibrium of the
floating body is not affected.
Problem 6.30 A balloon filled with air weighed so that it barely floats in water, as shown in
Fig. 6.54. Explain why it sinks to the bottom when it is submerged more by a small distance.

Fig. 6.54

5QNWVKQP When the balloon is submerged slightly into water, the pressure exerted by water
increases and hence the air inside the balloon is slightly compressed (PV = constant). The
buoyant force on the balloon therefore decreases (Fb a V), and so it sinks to the bottom.
Problem 6.31 A beaker containing water of weight W is placed on a spring balance. A
stone weight W¢ is hung and lowered into the water without touching the sides and bottom
of the beaker. Explain how the reading of the balance and tension in the string will change.
5QNWVKQP When stone is submerged into water, water exerts buoyant force Fb on the stone
in upward direction. The stone also exerts an equal downward force on water. So tension
in the string becomes,
1008പWŚLJƐŝĐƐĨŽƌ//dͲ:͗DĞĐŚĂŶŝĐƐ//

T
Fb

W¢ Fb

Fig. 6.55

T = W¢ – Fb and reading of spring balance become = W + Fb.


Note:
1. When bottom of the body is quite plane, and no water enters below it, the buoyant
force on the body will be zero (Fig. 6.56(b)).

(a) (b)

Fig. 6.56

When bottom face of the body is rough, the water enters below it and Fb
exerts buoyant force (Fig. 6.57).
2. Weight of empty balloon and inflated balloon are equal: Suppose weight
of the empty balloon is Mg. Let m amount of air be in inflated the balloon.
On the inflated balloon buoyant force is also there, so net weight of the
balloon
= (M + m)g – Fb
= (M + m)g – Vair rair g
(M + m)g
m
= (M + m)g – rair g
rair Fig. 6.57
= Mg
Problem 6.32 Compute the depth that a 500 N force will submerge the body shown in Fig.
6.58. It is a right circular cone whose weight we shall neglect.
5QNWVKQP Let y be the depth of immersion. By similar triangles the radius of the cone cross
section at the free surface is
,LJĚƌŽƐƚĂƟĐƐŽƌ&ůƵŝĚ^ƚĂƟĐƐപ1009

r 0.3 0.3 m
=
y 3
r
y
\ r =
10
Using the principle of buoyancy, we have 3m
y
Downward force exerted on the cone
= Buoyant force on the submerge portion of the cone
Ê1 ˆ
or 500 = Á p r 2 y˜ rw g
Ë3 ¯ 500 N
2 Fig. 6.58
1 Ê yˆ
= p Á ˜ y ¥ 100 ¥ 9.8
3 Ë 10 ¯
y = 1.69 m
Problem 6.33 A piece of wax weighs 18.03 g in air. A piece of metal weighs 17.03 g in
water. It is tied to the wax and both together weigh 15.23 g in water. What is the specific
gravity of wax?
5QNWVKQP Weight of the wax in air, W1 = 18.03 g-wt
Let Fb1 and Fb2 be the buoyant forces on wax and metal respectively.
Weight of the metal in air, W2 = 17.03g + Fb2
Combined weight of wax + Metel in water = 15.23 g-wt
or (W1 + W2) – (Fb1 + Fb2) = 15.23g
or (18.03g + 17.03g + Fb2) – (Vmaxrw g + Fb2) = 15.23g
or 35.06g – Vmax rw g = 15.23g
mwax
or rw g = 19.83g
rwax
18.03
or rw = 19.83
rwax
rwax
or = 0.909
rw
Thus, Specific gravity of wax = 0.909
Problem 6.34
(i) A piece of ice floats in water in a beaker. What happens to the level of the liquid in
the beaker when the ice melts completely?
(ii) A small metal piece is inside an ice block which is floating in water in a beaker. What
happens to the level in the beaker when ice melts completely?
(iii) Stones are unloaded from the boat into the lake. What happens to the level of water
in the lake?
(iv) A cork piece is inside an ice block which is floating in water in a beaker. What happens
to the level of water in the beaker when ice melts completely?
1010പWŚLJƐŝĐƐĨŽƌ//dͲ:͗DĞĐŚĂŶŝĐƐ//

5QNWVKQP
(i) Suppose M is the mass of the ice block and rw the density of water. For a floating ice
block, let V1 be the volume of water displaced, then
Mg = V1 rw g
M
fi V1 =
rw
After melting of ice, the volume of water forms
M
V2 =
rw
As, V1 = V2, so the level of water in the beaker will not change.
(ii) Suppose m and rmetal are the mass and density of metal, then for a floating ice block,
(M + m)g = V1rwg
M m
fi V1 = + M
rw rw
m Metal
When ice melts, the metal piece sink into the water.
So total volume of water forms + Displaced by metal piece
M m
V2 = + Fig. 6.59
rw rmetal
As rmetal > rw \ V2 < V1
So the level of water in the beaker will decrease.
(iii) As explained in (ii).
(iv) Suppose M is the mass of the cork.
For the floating in block
(M + m)g = V1 rw g
M+m M
fi V1 =
rw
m Cork
After melting of ice, cork piece will float such that,
m
mg = V¢rwg fi V¢ =
rw
M Fig. 6.60
The volume of water forms =
rw
M m
Total volume of water forms + Displaced by cork, V2 = +
rw rw
As V2 = V1, so the level of water in the beaker will not change.

Equilibrium of the Floating Object (Optional)


As already written, the buoyant force acts vertically up at the centre of gravity of the
displaced fluid. This point is called the EGPVTG QH DWQ[CPE[. Now, there are two points of
great importance here, namely:
,LJĚƌŽƐƚĂƟĐƐŽƌ&ůƵŝĚ^ƚĂƟĐƐപ1011

1. Centre of gravity (G) of the floating object.


2. Centre of buoyancy (B), i.e., centre of gravity of the displaced fluid.

Normal Axis of the Body


It is the line joining B and G when the object is floating in equilibrium.

Meta Centre (M)


It is defined as the point about which a body starts oscillating when the body is tilted by
a small angle. It can also be defined as the point at which the line of action of the force
of buoyancy will cut the normal axis of the body when the body is given a small angular
displacement as shown in Fig. 6.61.

W W

G G
B B1

FB FB

Normal Normal
axis axis

Fig. 6.61

Metacentric Height
It is the distance MG, i.e., the distance between the metacentre (M) of a floating body and
its centre of gravity (G).

Conditions of Equilibrium of Floating and Submerged Bodies (Optional)


A submerged or a floating body is said to be stable if it comes back to its original position
after a slight disturbance. The relative position of the centre of gravity (G) and centre of
buoyancy (B1) of a body determines the stability of a submerged body.

Stability of a Submerged Body


The positions of centre of gravity and centre of buoyancy in case of completely submerged
body are fixed. Consider a balloon, which is completely submerged in air. Let the lower
portion of the balloon contain heavier material, so that its centre of gravity is lower than its
centre of buoyancy as shown in Fig. 6.62(a). Let the weight of the balloon be W. The weight
W is acting through G, vertically in the downward direction, while the buoyant force FB is
acting vertically up, through B. For the equilibrium of the balloon W = FB.
1012പWŚLJƐŝĐƐĨŽƌ//dͲ:͗DĞĐŚĂŶŝĐƐ//

If the balloon is given an angular displacement in the clockwise direction as shown in Fig.
6.62(a), then W and FB constitute a couple acting in the anticlockwise direction and brings
the balloon in the original position.
Thus, the balloon in the position, shown by Fig. 6.62(a) is in stable equilibrium.

FB FB
B B
G
G
W
W

(a) Stable equilibrium

W
G
G
W
B B
FB
FB

(a) Unstable equilibrium

B G

(c) Neutral equilibrium

Fig. 6.62

(a) Stable Equilibrium. When W = FB and point B is above G, the body is said to be in
stable equilibrium.
(b) Unstable Equilibrium. If W = FB but the centre of buoyancy (B) is below the centre
of gravity (G), the body is in unstable equilibrium as shown in Fig. 6.63(b). A slight
displacement to the body in the clockwise direction, gives the couple due to W and FB
also in the clockwise direction. Thus, the body does not return to its original position
and hence the body is in unstable equilibrium.
(c) Neutral Equilibrium. If FB = W and B and G are at the same point, as shown in Fig.
6.63(c), the body is said to be in neutral equilibrium.
,LJĚƌŽƐƚĂƟĐƐŽƌ&ůƵŝĚ^ƚĂƟĐƐപ1013

Stability of a Floating Body (optional)


The stability of a floating body is determined from the position of the metacentre (M). In case
of floating body, the weight of the body is equal to the weight of liquid displaced.
(a) Stable Equilibrium. If the point M is above G, the floating body will be in stable
equilibrium as shown in Fig. 6.63(a). If a slight angular displacement is given to the
floating body in the clockwise direction, the centre of buoyancy shifts from B to B1
such that the vertical line through B1 cuts at M. Then the buoyant force FB through B1
and weight W through G constitute a couple acting in the anticlockwise direction and
thus bringing the floating body in the original position.

W W

G G
B B B1

FB FB

Normal Normal
axis axis

(a) Stable equilibrium M is above G

W W
G
G

M
B B B1
FB
FB

Normal Normal
axis axis

(b) Unstable equilibrium M is below G

Fig. 6.63
1014പWŚLJƐŝĐƐĨŽƌ//dͲ:͗DĞĐŚĂŶŝĐƐ//

(b) Unstable Equilibrium. If the point M is below G, the floating body will be in unstable
equilibrium as shown in Fig. 6.63(b).
The disturbing couple is acting in the clockwise direction. The couple due to buoyant
force FB and W is also acting in the clockwise direction and thus overturing the floating
body.
(c) Neutral Equilibrium. If the point M is at the centre of gravity of the body, the floating
body will be in neutral equilibrium.
Condition of equilibrium of a floating and submerged body are:

Equilibrium Floating Body Submerged Body


(i) Stable Equilibrium M is above G B is above G
(ii) Unstable Equilibrium M is below G B is below G
(iii) Neutral Equilibrium M and G coincide B and G coincide

Thus, if G and B do not coincide, it may result in torques, rolling and pitching of ships and
resulting sea-sickness. Top-heavy floating bodies are unstable. The tumbling of the icebergs
as they melt is due to this reason.

Practical Applications of Law of Floatation


1. Ship: Ships are made up of metals whose densities are more than the density of water.
Even then the ships float in water. This is because they are made hollow, hence their
effective density becomes less than the density of water.
2. Submarine: A submarine is a fish-shaped watertight hollow boat which can be
manoeuvred under water. It is provided with blast tanks which can suck in or throw
out seawater by means of pumps. When tanks are filled with water, the weight of
submarine increases. It becomes more than the weight of the water displaced and so
it goes down into the water. When it is desired to raise it upwards, the water from the
tank is forced out, so its weight decreases. Now the weight of water displaced by it
becomes greater than the weight of the submarine and hence the submarine rises up
to the surface of water.
3. .KHG$GNV It is a big hollow tube of rubber as that of a car. When air is filled in it, its
volume becomes much larger. If a man, wearing this belt, jumps in water, sufficient
water is displaced and the weight of (man + belt) becomes less than the weight of
water displaced; therefore, man remains safe.
It is used to save lives of men in steamers and ships; therefore is called the life saving
belt.
4. Iceberg: Iceberg is a large floating mass of ice, detached from a glacier and carried
out to sea. The icebergs float in seawater with most of its part is submerged and only
a little part is outside the water, hence the idiom tip of the iceberg.
The density of seawater is 1.03 g/cm3 while that of ice is 0.92 g/cm3.
By the law of floatation the fraction of the volume of iceberg submerged,
Vin d 0.92 8
= ice = = (nearly)
Vtotal dwater 1.03 9
,LJĚƌŽƐƚĂƟĐƐŽƌ&ůƵŝĚ^ƚĂƟĐƐപ1015

5. (NQCVKPIQH/CPQP9CVGT The body of man is lighter than water except his head. Total
weight of the man happens to be greater than the weight of water displaced; therefore,
the man can sink in water. Hence, for floating in water, the man has to displace more
water by moving his arms and legs. By doings so, when the weight of water displaced
by man becomes equal to the weight of man, the man begins to float. The density of
seawater is 1.03 ¥ 103 kg/m3.

Accelerating Liquid
(i) 2TGUUWTGFKHHGTGPEGYJGPNKSWKFKUCEEGNGTCVKPIKPXGTVKECNFKTGEVKQP
Consider a cylindrical element of height h and area A. The force on
the top face of the element is P1 A and force on the bottom face is P1A
P2 A. If a is the acceleration of the liquid, then mg
P2 A – (mg + P1 A) = ma a

here m is the mass of the element of liquid which is equal hAr. P2A
Thus, we have
Fig. 6.64
P2 A – (hArg + P1 A) = (hAr)a
After simplification, we get
P2 – P1 = r(g + a)h
(ii) $WQ[CPVHQTEG
a
Suppose the body is submerged into a liquid of density r, which is Fb
accelerating upwards. If a is the acceleration of the liquid, then
Fb – mg = ma
mg
or Fb = m(g + a)
here m is the mass of the displaced liquid, which is equal to Vr.
Fig. 6.65
\ Fb = rV(g + a)
(iii) .KSWKFUWDLGEVGFVQJQTK\QPVCNCEEGNGTCVKQP
Consider a liquid in a tank which is mov-
q
ing on a horizontal surface with constant
acceleration a. The free surface of the liq- q
uid takes the shape as shown in Fig. 6.66. y1 y2
F1 F2
Suppose a cylinder of liquid of length l and 
a
area of cross section A. The force on the
left face of the cylinder, F1 = P1A and force
on the right face of the cylinder, F2 = P2 A. Fig. 6.66
Here P1 = rgy1 and P2 = rgy2.
Mass of the liquid cylinder, m = (Alr)
Using Newton’s second law for the liquid cylinder,
F1 – F2 = ma
or P1A – P2 A = ma
or (rgy1 – rgy2)A = (Alr)a
1016പWŚLJƐŝĐƐĨŽƌ//dͲ:͗DĞĐŚĂŶŝĐƐ//

y1 - y2 a
or =
l g
y1 - y2
From Fig. 6.66, = tan q
l
a
\ tan q = .
g
ax
The above equation can be written as: tan q =
ay
a
Alternative Method
q
Surface of liquid stays perpendicular the resultant ma
force. q

ma a
tan q = =
mg g FR
mg
–1 Ê

q = tan Á ˜ Fig. 6.67
Ë ¯
g

(iv) .KSWKFUWDLGEVGFVQEQODKPGFJQTK\QPVCNCPFXGTVKECNCEEGNGTCVKQPU
Resolve the acceleration a into two components, ax and ay.
ay a

ax ax
tan q =
g + ay

Fig. 6.68

In view of the previous analysis, we can write


ax
tan q =
g + ay

(v) .KSWKFKPC8GUUGN4QVCVKPIYKVJ%QPUVCPV#PIWNCT8GNQEKV[
When the vessel is rotated there is no relative motion between fluid elements. The
liquid surface orients itself in permanent position relative to the vessel. We will analyse
a differential element dm in the reference frame of vessel. The forces acting on it are
shown in Fig. 6.69 and 6.70. R is the force exerted on the element due to neighbouring
elements of fluid.
Thus, R cos q = dmg ...(i)
2
R sin q = dmxrw ...(ii)
,LJĚƌŽƐƚĂƟĐƐŽƌ&ůƵŝĚ^ƚĂƟĐƐപ1017

Dividing Eq. (ii) by Eq. (i), we have


w 2x
tan q = ...(iii)
g
w

R R sin q
Tangent

YB q
dm xw2
B R
q R sin q (Pseudo-force)
N r P
ymax dmg
A
ymin
q
x
O

Fig. 6.69

At the position of the element, the slope of the tangent to curve is given by dy/dx.
dy w 2 x w 2x
tan q = = fi dy = dx
dx g g
w 2r 2
which on integration yields y = +c
2g
which represents a parabola. Thus, liquid surface is a paraboloid of revolution.
R2w 2
At x = R and y = ymax = +c
2g
At x = 0, y = ymin = C
R2w 2
Therefore, ymax = + ymin
2g
and the general equation can be written as
x 2w 2
y = + ymin …(i)
2g
Volume of a paraboloid of revolution is half that of the circumscribing cylinder.

( y max - y min )
ymax 2
y0
ymin

Fig. 6.70
1018പWŚLJƐŝĐƐĨŽƌ//dͲ:͗DĞĐŚĂŶŝĐƐ//

( ymax - ymin )
Hence, volume of a paraboloid = A
2
If originally liquid filled up to height y0, then
ymax - ymin ymax + ymin
y0 = ymin + =
2 2
2 2
w R
ymax = y0 +
4g

WORKED PROBLEMS
Problem 6.35 A U-tube rotates with angular velocity w about the vertical axis AB. What
is the difference in fluid level ‘h’ in terms of w, the radii r1 and r2 and the fluid density r?
5QNWVKQP Let ymin be the lowest point on the dotted parabola w
shown in Fig. 6.71. r1 r2
r 2w 2 r 2w 2
y1 = 1 + ymin and y2 = 2 + ymin
2g 2g
B
h
(r 2 - r 2 )w 2 w 2 2 2 y2
\ h = y2 – y1 = 2 1 = (r2 - r1 ) A
2g 2g
y1
ymin
Alternatively, we may apply Bernoulli’s equation between
points A and B. Fig. 6.71
1 1 Ê r22 - r12 ˆ 2
Patm + r(r1w)2 = Patm + r(r2w)2 + rgh fi h = Á w
2 2 Ë 2 g ˜¯

Problem 6.36 A cylindrical vessel of diameter 0.3 m and height 0.6 m is filled two-thirds
with a liquid of specific gravity 0.8. The vessel is rotated about its axis. (a) Determine the
speed of rotation when the liquid just starts spilling. (b) Find the speed of rotation when the
base is just visible. (c) What is the percentage of liquid left in the vessel?
2
5QNWVKQP Initial height of liquid in the vessel y = 0.6 ¥ = 0.4 m
3
Height of the container = 0.6 m
w
w 2 R2
ymax – ymin = …(i)
2g w2R2/4
w2R2/4
w 2 R2 ymax
ymax =y+ …(ii) Y
4g ymin

w 2 R2
ymin = y – …(iii)
4g Fig. 6.72
,LJĚƌŽƐƚĂƟĐƐŽƌ&ůƵŝĚ^ƚĂƟĐƐപ1019

(a) ymax = 0.6 m 2


Ê 0.3 ˆ
w2 ¥ Á
Ë 2 ˜¯
From Eq. (ii), 0.6 = 0.4 +
4 ¥ 9.81

R2w 2
(b) ymax = 0.6 m, ymin = 0 fi ymax =
2g

(0.3)2 ¥ w 2
0.6 = fi w = 22.9 rad/s
8 ¥ 9.81
(c) The volume of paraboloid of revolution is half the volume of the enclosing cylinder.
Volume of the liquid left in the vessel = (1/2) ¥ 0.6 ¥ A, where A is cross-sectional area.
0.3 A
Initial volume was 0.4A m3. Percentage of the liquid left = ¥ 100 = 75%.
0.4 A

Problem 6.37 A cylindrical tank of radius 20 cm and height 50 cm has water up to 30 cm


of height. What will be the rise in level of liquid at the periphery if the cylinder be given
an angular velocity of 10 rad s–1? Also determine the frequency of rotation when water just
starts spilling over the sides of the vessel.
5QNWVKQP At an angular speed of 10 rad s–1, let us assume
that the water does not spill.
We have
w 2r 2 w 2r 2
h2 = H + fi h2 – H =
4g 4g 50 cm

Therefore, rise in the liquid level at the periphery is 10 cm. 30 cm


For water to just spill over the sides, the maximum height
(hmax) is 50 cm. 20 cm
w 2r 2
Again, hmax = y0 + Fig. 6.73
4g

Here, hmax = 50 cm, y0 = 30 cm, R = 20 cm, g = 10 ms–2

w 2 (20 ¥ 10 -2 )2
\ 50 ¥ 10–2 = 30 ¥ 10–2 +
4 ¥ 10
fi w2 ¥ 10–3 = 20 ¥ 10–2
w2 = 200 fi w = 10 2 rad s–1

w 5 2 –1
Hence, frequency of rotation is f = = s
2p p
1020പWŚLJƐŝĐƐĨŽƌ//dͲ:͗DĞĐŚĂŶŝĐƐ//

Problem 6.38 A barometer kept in an elevator accelerating upwards reads 76 cm of Hg.


What will be the possible air pressure inside the elevator?
5QNWVKQP Let a be the acceleration of the elevator, then pressure inside the elevator
P = r(g + a)h = P ¥ (g + a) 0.76 N/m2
Atmospheric pressure, Pa = r g ¥ 0.76 N/m2
Clearly, the air pressure inside the elevator will be greater than Pa, i.e., 76 cm of Hg.
Problem 6.39 A barometer kept in elevator reads 76 cm, when it is at rest. What will be the
barometric reading when the elevator accelerates upwards?
5QNWVKQP Let a be the acceleration of the elevator and h be the barometric height, then
Pa = r(g + a)h ...(i)
For the static elevator with barometric height h0
Pa = rgh0 ...(ii)
From Eqs. (i) and (ii), we get
gh0
h =
( g + a)
Clearly, h < h0, so barometric reading in an accelerating elevator will be less than 76 cm.
Problem 6.40 A barometer accelerating downwards reads 76 cm of Hg. What will be the
possible air pressure inside the jar?
5QNWVKQP If a is the acceleration, then pressure P = r(g – a)h = r(g – a) ¥ 0.76
Clearly, the pressure is less than rg ¥ 0.76, i.e., 76 cm of Hg.
Problem 6.41 Calculate the change in the potential energy of a body raised in water to a
height h. Will the potential energy of the water in the vessel change when the body rises?
What will happen when the density of the body is larger and smaller than the density of
the water? The density of the body is r, the density of the water is rw and the volume of
the body is V.
5QNWVKQP The body in water is simultaneously subjected to the force of gravity and the
buoyant force. We know that the change in potential energy of the body is equal to the work
done by conservative forces, i.e.,
DU = – Wc
= – [– Fg + Fb] ¥ h
= (Fg – Fb)h
For the body completely inside water
Fb = Vrwg and Fb = mg = Vrg
\ DU = Vgh(r – rw)
If r > rw, then DU > 0 and the energy of the body increases. If r < rw, then DU < 0 and
the energy of the body diminishes.
,LJĚƌŽƐƚĂƟĐƐŽƌ&ůƵŝĚ^ƚĂƟĐƐപ1021

When the body moves up to the height h a volume of water V is displaced downwards
by the same distance. In this case the potential energy of this volume in the fluid of the force
of gravity will diminish by Vrwgh and the energy due to the buoyant force will increase by
Vrwgh. Therefore, the total potential energy of the water will remain constant.
DUwater = 0

Note:
w2 2
If a U-tube is rotated about an axis as shown in Fig. 6.74 then y = (r2 - r12 ) .
2g
w

r1 r2

Fig. 6.74

Problem 6.42 A tube of length h, which is wide enough to make surface tension effects
negligible, is closed at one end. It is then lowered into a tank of mercury to a depth h as
shown in Fig. 6.76, so that mercury rises a distance x into the tube. If mercury barometer
stands at h, then find relationship between h and x. ++6,''
Pa

Air

Air

Fig. 6.75 Fig. 6.76

5QNWVKQP Let area of cross-section of tube be A. When it is air the volume of air in it, V1 =
Ah, and pressure P1 = h of mercury
When the tube is lowered into mercury, then volume of air (Fig. 6.76).
V2 = A(h – x);
P2 + x = 2h
fi P2 = (2h – x) of mercury
1022പWŚLJƐŝĐƐĨŽƌ//dͲ:͗DĞĐŚĂŶŝĐƐ//

Now by Boyle’s law


P1V 1 = P 2V 2
or h (Ah) = (2h – x) [A(h – x)]
fi h2 = (2h – x)(h – x)
Problem 6.43 An ornament weighing 36 g in air, weighs only 34 g in water. Assuming that
some copper is mixed with gold to prepare the ornament, find the amount of copper in it.
Specific gravity of gold is 19.3 and that of copper is 8.9.
5QNWVKQP Let mg and mc are the masses of gold and copper in the ornament, then
mg + mc = 36 ...(i)
Loss of weight in water = (36 – 34)g = 2g
\ 2g = Vtotal rw g
2
or Vtotal =
rw
mg mc 2
or + =
19.3 8.9 1
After solving Eqs. (i) and (ii), we get
mc = 2.2g
Problem 6.44 A cubical block of wood of edge 3 cm floats in water. The lower surface of
the cube just touches the free end of a vertical spring fixed at the bottom of the pot. Find
the maximum weight that can be put on the block without wetting it. Density of wood =
800 kg/m3 and spring constant of spring = 50 N/m. Take g = 10 m/s2.
5QNWVKQP

m
M
y
M

(a) (b)

Fig. 6.77

When the block just touches the spring, it floats freely on water. Let y be the portion of
block inside water, then
,LJĚƌŽƐƚĂƟĐƐŽƌ&ůƵŝĚ^ƚĂƟĐƐപ1023

Wt of block = buoyant force


Vrwood g = Vrwg V=A¥l=l¥l¥l
or (3 ¥ 3 ¥ 3 ¥ 10 ) ¥ 800g = (y ¥ 3 ¥ 3 ¥ 10–6) ¥ 1000 ¥ g
–6

y = 2.4 cm.
\ Height of block out of water = 3 – 2.4 = 0.6 cm Fb
Let m be the mass, that can be placed on the m
block without wetting it. y
The additional weight is balanced by the
buoyant force on additional dipped portion +
spring force
i.e., mg = Vrwg + ky ky

= (0.6 ¥ 3 ¥ 3 ¥ 10–6) ¥ 1000 ¥ 10 + 50 Fig. 6.78


¥ (0.6 ¥ 10 )
–2

= 0.354 N
Problem 6.45 A cube of ice of edge 4 cm is placed in an empty cylindrical glass of inner
diameter 6 cm. Assume that the ice melts uniformly from each side so that it always retains
its cubical shape. Remembering that ice is lighter than water, find the length of the edge of
the ice cube at the instant it just leaves the contact with the bottom of the glass.
5QNWVKQP

4 cm
x
Ice y

6 cm 6 cm

Fig. 6.79 Fig. 6.80

Let size of ice block remaining is x3 when it just about to float, then
\ x3 ¥ rice g = (x2 y)rw g wt. of block = Buoyant force ...(i)
fi x3 rice = x2 yrw
Also mass of ice melt = Mass of water forms
(43 – x3) ¥ rice = (p ¥ 32 ¥ y – x2y)rw ...(ii)
or 43 rice – x3 rice = p ¥ 32 yrw – x2yrw as x3 rice = x2 yrw
1024പWŚLJƐŝĐƐĨŽƌ//dͲ:͗DĞĐŚĂŶŝĐƐ//

or 43 rice = p ¥ 32 yrw
Ê 7.11rice ˆ
or y = Á
Ë prw ˜¯
Substituting the value of y in Eq. (i), we get

Ê 7.11rice ˆ
x3 ¥ rice g = x 2 ¥ Á rw g
Ë prw ˜¯
or x = 2.26 cm
Problem 6.46 A wooden plank of length 1 m and uniform cross section is hinged at one
end to the bottom of a tank as shown in Fig. 6.81. The tank is filled with water up to a height
of 0.5 m. The specific gravity of the plank is 0.5. Find the angle q that the plank makes with
the vertical in the equilibrium position (Exclude the case q = 0). ,''
5QNWVKQP Let y be the length of the plank inside water
0.5 Fb
\ y = l/2
cos q
Let A be the cross-sectional area of the plank, then
buoyant force on it 0.5 m
Fb = Vrwg q mg
= (Ay)rwg O
Since the plank is in rotational equilibrium, so Fig. 6.81

 to = 0
l y
or mg ¥ sin q - Fb ¥ sin q =0
2 2 Fb l/2
or mg l – Fb ¥ y =0
(A l ¥ 0.5)gl – (Ay)rwg ¥ y =0
or 0.5l2 = y2 F l/2 mg
2 q
Ê 0.5 ˆ
or 0.5 ¥ (1)2 = Á O
Ë cos q ˜¯
y/2
1
fi cos2 q =
2 Fig. 6.82
1
or cos q =
2
or q = 45° Ans.
Problem 6.47 A closed tank filled with water is mounted on a railroad car. The car moves
with an acceleration a on a plane road. Find the value of the pressure at any point which is
at a depth h and a distance l from the front wall.
,LJĚƌŽƐƚĂƟĐƐŽƌ&ůƵŝĚ^ƚĂƟĐƐപ1025

B
h
P l
C a
l

Fig. 6.83

5QNWVKQP Because of the acceleration of the tank, the pressure increases towards the
backward side of the tank. The equivalent profile is shown in Fig. 6.84. We know that
a
tan q =
g
y y
= q
R
h
al
\ y = l
a
g
The pressure at the point
= (h + y)dg Fig. 6.84
Ê al ˆ
= Á h + ˜ dg
Ë g¯
= (hg + al)d
Problem 6.48 A piece of pure gold (r = 19.3 g/cm3) is suspected to be hollow from inside.
It weighs 38.250 g in air and 33.865 g in water. Calculate the volume of the hollow portion
in gold, if any. (BITSAT, 2009)
38.25
5QNWVKQP The volume of metal V =
19.3
= 1.98 cm3
The loss of weight in water = 38.250 – 33.865
= 4.385 g.
If V¢ is the total volume of the piece of gold then
4.385g = V¢rwg
\ V¢ = 4.385 cm3
Thus, volume of cavity inside piece
= 2.405 cm3
Problem 6.49 A spring balance reads 10 kg when a bucket of water is suspended from it.
What is the reading on the spring balance when (i) an ice cube of mass 1.5 kg is put into the
bucket; and (ii) an iron piece of mass 7.8 kg suspended by another spring is immersed with
half its volume inside the water in the bucket? Relative density of iron = 7.8.
1026പWŚLJƐŝĐƐĨŽƌ//dͲ:͗DĞĐŚĂŶŝĐƐ//

5QNWVKQP
(i) The ice block becomes the part of the system, so reading of spring balance
= 10 + 1.5
= 11.5 kg-f
(ii) The buoyant force on the iron piece
V
= rw g
2

È 7.8/7.8 ˘
= Í ˙ ¥1¥ g
Î 2 ˚
= 0.5 kg-f
Thus, reading of the spring balance become
= 10 + 0.5
= 10.5 kg-f
Problem 6.50 A tank contains water and mercury as shown in
Fig. 6.85. An iron cube of edge 6 cm is in equilibrium as shown. Water
What is the fraction of cube inside the mercury? Given density
of iron = 7.7 ¥ 103 kg m3 and density of mercury = 13.6 ¥ 103
kg/m3. (VITEEE, 2016) Mercury

5QNWVKQP Suppose y depth of the cube is inside the mercury,


then for floating block. Fig. 6.85

Weight of the block = Buoyant force


7.7 ¥ 103 ¥ [0.063]g = [y ¥ (0.06)2] ¥ 13.6 ¥ 103 g + [(0.06 – y) ¥ (0.06)2 ¥ 103]g
After simplifying y = 3.2 cm
y
Fraction f =
6
3.2
=
6
= 0.533
Problem 6.51 A barometer is a device for measuring atmospheric
pressure. If we use a liquid having a specific gravity of 13,600 kg/m3
and invert a tube full of this material as shown in Fig. 6.86. What is the
h
value of h if the vapour pressure of the liquid is 21 kg/m2?
9$,''
5QNWVKQP  The vapour pressure = 21 kg/m2
21
= m
136000 Fig. 6.86
= 0.15 cm
The barometric height = 76 – 0.15
= 75.85 cm
,LJĚƌŽƐƚĂƟĐƐŽƌ&ůƵŝĚ^ƚĂƟĐƐപ1027

Problem 6.52 The vessel has two immiscible fluids, water


and oil are evacuated to a pressure of 0.5 atm. What is the Oil 3m
resultant force on the door from fluids inside the container (the
specific gravity of oil is 0.8)? H2O 9 m 45 m

5QNWVKQP Pressure intensity at a depth y from surface of water 3m

P = [0.5Pa + roil g ¥ 3 + rwgy] …(i)


Fig. 6.87

a
0.5 P 3m
Oil
H2O y

dy

Fig. 6.88

The force on the dy height of gate


dF = P(4.5 dy)
9
Total force F = Ú P(4.5 dy) ...(ii)
6
After solving Eqs. (i) and (ii), we get
F = 8.21 ¥ 105 N
Problem 6.53 The area of cross section of a large tank is 0.5 m2. It has an opening near the
bottom having area of cross section 1 cm2. A load of 50 kg is applied on the water at the top.
Find the velocity of the water coming out of the bottom. Take g = 10 m/s2.
5QNWVKQP The pressure exerted by 50 kg weight is corresponding to h height of water, then
rgh = 50g
50
\ h =
r
50
=
1000
= 0.05 m
Thus, total height of water above opening
H = 0.50 + 0.05
= 0.55 m
The efflux velocity = 2gH

= 2 g ¥ 0.55
= 3.3 m/s
1028പWŚLJƐŝĐƐĨŽƌ//dͲ:͗DĞĐŚĂŶŝĐƐ//

Problem 6.54 A trolley containing a liquid of density r slides down a smooth


inclined plane of angle a with the horizontal. Determine the angle of inclination
q of the free surface with the horizontal.
Solution The acceleration of the trolley along the inclined plane is
a = g sin a
a
ax = a cos a = g sin a cos a
ay = a sin a = g sin2a Fig. 6.89
The inclination of the free surface is
q
ax g sin a cos a q=a
tan q = = = tan a
g - ay g(1 - sin 2 a ) ax
a
or q =a
ay a = g sin a
Note: The free surface becomes parallel to the inclined plane. It is a
because the net effective gravity inside the trolley is perpendicular
Fig. 6.90
to the inclined plane.
Problem 6.55 The Fig. 6.91 shown a semi-cylindrical massless gate pivoted at the point O
holding a stationary liquid of density r. The length of the cylinder is l. Calculate horizontal
force exerted by the liquid on the gate.
Solution The force exerted by liquid on the gate
FH = Pav ¥ vertical projected area of the gate h=R
A
ÊP +P ˆ
= Á A B ˜ ¥ (2Rl) r 3R
Ë 2 ¯ R
O

Ê Rr g + 3 Rr g ˆ
= Á
Ë ˜¯ ¥ 2Rl B
2
= 4rgR2l Fig. 6.91

Problem 6.56 A closed rectangular tank 1.2 m high, 2.4 m long and 1.5 m wide is two-thirds
full of gasoline of relative density 0.8. Calculate the acceleration which may be imparted
to the tank so that the bottom front end of the tank is just exposed. Also calculate the total
forces on each end of the tank. F
2
Solution The height of gasoline = ¥ 1.2 = 0.8 m
3 E
A D
x q
Since the tank is closed, therefore volume of
liquid inside it remains as such. Suppose free
A¢ D¢
surface makes q with the horizontal and x is the 1.2 m
distance of free surface from A, then 0.8 m
Volume of AECBA = Volume of A‘D’CB
B C
2.4 m

Fig. 6.92
,LJĚƌŽƐƚĂƟĐƐŽƌ&ůƵŝĚ^ƚĂƟĐƐപ1029

1
\ (2.4 + x) ¥ 1.2 ¥ 1.5 = 2.4 ¥ 1.5 ¥ 0.8
2
fi x = 0.8 m
CD
From geometry tan q =
DE
Ê 1.2 ˆ
= Á = 0.75
Ë 2.4 - 0.8 ˜¯
ax
Also tan q =
g
= 0.75
or ax = 0.75 ¥ g
= 7.35 m/s2
At A the pressure head is equal to an imaginary column of gasoline of height equal to AE.
AF
= tan q = 0.75
AE
or AF = 0.75AE
= 0.75 ¥ 0.8 = 0.6 m
Therefore, effective height of liquid at end B
= 1.2 + 0.6 = 1.8 m
PA = 0.6rg
PB = 1.8rg
PA + PB (0.6 + 1.8)
\ Pav = = rg
2 2
= 1.2rg
Force acting on the back face
F1 = Pav ¥ Area of face
= (1.2rg) ¥ (1.2 ¥ 1.5)
= (1.2 ¥ 800 ¥ 9.8) ¥ (1.2 ¥ 1.5)
= 16934.4 N
Force on the front face
F2 = 0

Problem 6.57 A liquid stands at the same level in the U-tube


when at rest. If A is the area of cross section and g the acceleration
due to gravity, what will be the difference in height h of the h
a
liquid in the two limbs of U-tube, when the system is given an
acceleration a towards right, as shown in Fig. 6.93.

Fig. 6.93
1030പWŚLJƐŝĐƐĨŽƌ//dͲ:͗DĞĐŚĂŶŝĐƐ//

Solution If h is the difference in height, then


h a q
tan q = = a
L g
La
\ h = . L
g
Fig. 6.94
FLUID DYNAMICS
Flow Characteristics
5VGCF[CPFWPUVGCF[ƀQY Steady flow may be defined as the flow in which at any point in
the flowing fluid various characteristics which describe the behaviour of flow are independent
of time. However, these characteristics may be different at different points in the direction
of flow. Mathematically, steady flow can be expressed as:

Ê ∂v ˆ Ê ∂P ˆ Ê ∂r ˆ
ÁË ˜¯ = 0; ÁË ˜¯ = 0; ÁË ˜¯ = 0
∂t ∂t ∂t
Fluid flow is said to be unsteady if at any point in the flowing fluid any one or all
the characteristics which describe the behaviour of the flow change with time. Thus, for
unsteady flow
Ê ∂v ˆ Ê ∂P ˆ
ÁË ˜¯ π 0 and ÁË ˜¯ π 0
∂t ∂t

7PKHQTOCPFPQPWPKHQTOƀQY If velocity of flow does not change from point to point in


a flowing fluid for any given instant of time, the flow is said to be uniform otherwise non-
uniform. Thus,
Ê ∂v ˆ
for uniform flow, ÁË ˜¯ = 0
∂s

Ê ∂v ˆ
for non-uniform flow, ÁË ˜¯ π 0
∂s

4CVG QH ƀQY Consider a pipe of cross-sectional area a a v


carrying liquid with a velocity v, the volume of liquid
flow in one second, i.e., v

dV Volume of shaded portion Fig. 6.95


Q= = = av
dt 1 second

WORKED PROBLEMS
Problem 6.58 Discuss the flow characteristics in the pipes shown in Figs. 6.96 and 6.97, for
(i) constant flow and (ii) variable flow.
,LJĚƌŽƐƚĂƟĐƐŽƌ&ůƵŝĚ^ƚĂƟĐƐപ1031

v1 v2
v v

Fig. 6.96 Fig. 6.97

Solution
(i) For constant rate of flow, velocity at different points in the direction of flow do not
change with time, so flow in both the pipes is steady. But in pipe of varying cross
sections, the velocity of flow changes with distance, so the flow will be nonuniform.
Thus, in the first pipe flow is steady-uniform and in the second pipe it is steady-
nonuniform.
(ii) When flow rate changes, the velocity at any point will change with time. So in the first
pipe flow is unsteady-uniform but in the second pipe it is unsteady-nonuniform.
.COKPCT(NQY A flow is said to be laminar when different liquid particles move in layers
with one layer of fluid over an adjacent layer. In the laminar flow, the viscosity of fluid plays
an important role. The flow of viscous liquid may, in general, be treated as laminar.

(a) Laminar flow (b) Turbulent flow

Fig. 6.98 Fig. 6.99

6WTDWNGPV(NQY When liquid particles move on zig-zag path or in disorderly manner, the
flow of liquid leads to turbulent flow. The occurrence of turbulent flow is more frequent
than that of laminar flow. Flow in streams, channels, water supply pipes, sewers, etc., are a
few examples of turbulent flow.
8GNQEKV[ 2TQſNG The surface obtained by joining the heads of velocity vectors for the
particles in a section normal to the direction of flow is called velocity profile.
(a) 8GNQEKV[RTQſNGQHCPQPXKUEQWUNKSWKF In this case, the velocity of all the particles
at any section of pipe is same, so the velocity profile is plane as shown in Fig. 6.100.

(a) Non-viscous liquid (b) Viscous liquid

Fig. 6.100

(b) 8GNQEKV[RTQſNGQHXKUEQWUNKSWKF The velocity of fluid particles in the contact of pipe


is zero and maximum at the centre of the pipe. In this case velocity profile will be
parabolic.
1032പWŚLJƐŝĐƐĨŽƌ//dͲ:͗DĞĐŚĂŶŝĐƐ//

5VTGCONKPG(NQY Consider the flow of a liquid along the path ABC as shown in Fig. 6.101.
VB
VA
B
C VC
A
Streamlines

Fig. 6.101

If every successive particle passes through A, B and C with velocities vA , vB and vC


respectively, the flow is said to be steady or streamlined flow. The path ABC along which the
particles move one after other is called streamline. A group of streamlines passing through
a small closed curve, which may or may not be circular, is called stream tube.

Streamlines

Stream tube

Fig. 6.102

Critical Velocity
At low velocity, the flow of liquid is laminar. As the velocity of flow increases, the flow
becomes turbulent after a certain velocity of flow. Thus, the velocity at which flow changes
from laminar to turbulent, is called ETKVKECNXGNQEKV[. The critical velocity vc of a liquid flowing
through a pipe depends on
(i) density of liquid (r),
(ii) diameter of pipe (D), and
kh
(iii) coefficient of viscosity (h) = vc =
rD
Here k is a constant.

REYNOLDS NUMBER
It is a dimensionless parameter which describes the nature of flow of the fluid. It is defined
as:
Inertia per force unit area of fluid( F1 )
Re = ...(i)
Viscous force per unit area( Fv )
DP
Fi = F/A = Dt = Dm v
A Dt A
,LJĚƌŽƐƚĂƟĐƐŽƌ&ůƵŝĚ^ƚĂƟĐƐപ1033

DV v Qv
= r =r = rv 2
Dt A A
Fv = h ¥ Velocity gradient
v
= h
D
Putting the value in Eq. (i)
rv 2
\ Re =
hv/D
rvD
or Re =
h

(QTƀQYQHNKSWKFKPCRKRG If Re is less than equal to 2000, the flow will be laminar. If Re >
3000, the flow is turbulent. If Re lies between 2000 and 3000, the flow is unstable.

WORKED PROBLEMS
Problem 6.59 (a) What is the largest average velocity of blood flow in an artery of radius
2 ¥ 10–3 m if the flow must remain laminar? (b) What is the corresponding flow rate? Take
viscosity of blood to be 2.084 ¥ 10–3 Pa-s and density of blood = 1.06 ¥ 103 kg/m3.
Solution
(a) The maximum value of Reynolds number for blow to be laminar is 2000. If vc is the
rvc D
average velocity of flow, then Re =
h
h Re h Re
\ vc = =
rD r 2r

2.084 ¥ 10 - 3 ¥ 2000
=
1.06 ¥ 10 3 ¥ 2 ¥ 2 ¥ 10 - 3
= 0.98 m/s
(b) The flow rate of blood
Q = vc ¥ Area of cross section of the artery
= 0.98 ¥ pr2
= 0.98 ¥ p (2 ¥ 10–3)2
= 1.23 ¥ 10–5 m3/s
Problem 6.60 The flow rate from a tap of diameter 1.25 cm is 3 litres/min the coefficient
of viscosity of water is 10– 3 Pa-s.
Solution Given, D = 1.25 ¥ 102 m
3 ¥ 10 - 3
Q = 3 litre/min = = 5 ¥ 10–5 m3/s
60
1034പWŚLJƐŝĐƐĨŽƌ//dͲ:͗DĞĐŚĂŶŝĐƐ//

p D2
As Q = vc A = vc ¥
4
Q 5 ¥ 10 - 5
vc = = = 4.08 m3/s
p 2 p
D (1.25 ¥ 10 - 2 )2
4 4
rvc D 1000 ¥ 4.08 ¥ 1.25 ¥ 10 - 2
Reynolds number, Re = = = 5095
h 10 - 3
As Re > 3000, so the flow will be turbulent.

IDEAL FLUID
An ideal fluid has the following characteristics:
(i) Incompressible: The density of fluid does not change with change in fluid pressure.
(ii) Non-viscous: The fluid layers offer no internal resistance and hence total the mechanical
energy of non-viscous fluid remains constant. In practice water can be taken as ideal
fluid.

EQUATION OF CONTINUITY
The equation of continuity is the mathematical statement of the principle of conservation
of mass.
Consider a fluid is flowing in a pipe of varying area of cross section as shown in Fig. 6.103.
Let v1 and v2 be the velocities of flow at cross sections A1 and A2 respectively.
The mass of the fluid enters into section 1 in time Dt,
m 1 = r 1V 1 v2Dt

= r1(A1v1Dt) v2
The mass of the fluid leaving the section 2 in the same A2
interval of time,
m2 = r 2V 2 v1Dt

= r2(A2v2Dt) v1
A1
By conservation of mass
m1 = m2 Fig. 6.103
or r1(A1v1Dt) = r2(A2v2Dt)
or r 1A 1v 1 = r 2A 2v 2
For incompressible fluid r1 = r2
\ A 1v 1 = A 2v 2
or Av = Constant (Q)
The above equation is the equation of continuity. It states that for an incompressible fluid
product of flow velocity and corresponding area of cross section of a pipe remain constant.
,LJĚƌŽƐƚĂƟĐƐŽƌ&ůƵŝĚ^ƚĂƟĐƐപ1035

ENERGY OF A FLOWING FLUID


A flowing fluid may have three kinds of energy.
(i) -KPGVKEGPGTI[ The energy possessed by a fluid by virtue of motion is called its kinetic
energy. Kinetic energy of m mass of the fluid flowing with velocity v is given by
1
KE = mv 2
2
1 mv 2 1 2
KE per unit volume = = rv
2 V 2
1 mv 2 v 2
The kinetic energy per unit weight of the fluid is given by = . It is measured
2 mg 2 g
in metre and called the XGNQEKV[JGCF.
(ii) 2QVGPVKCNGPGTI[ The energy possessed by a fluid by virtue of its position is called its
potential energy.
Potential energy of a fluid at a height h above the earth’s surface is given by
PE = mgh
mgh
PE per unit volume = = rgh
V
mgh
PE per unit weight of the fluid is given by = h and is called potential or
ITCXKVCVKQPCNJGCF. mg

(iii) 2TGUUWTG GPGTI[ The energy possessed by a fluid by


virtue of its pressure is called its pressure energy. To
understand this, consider a liquid in a cylinder fitted with
a piston. Let P be the pressure at the piston. Suppose the
piston moves through a distance x.
The work done against this pressure,
W = Force ¥ Displacement
P
= (PA)x x

= P (Ax) = PV Fig. 6.104


where V is the volume swept by the piston. This work done is stored as the pressure
energy of the fluid of the volume V.
Thus, pressure energy of a fluid of volume V = PV
PV
Pressure energy per unit volume = = P (Excess pressure)
V
PV P
Pressure energy per unit weight is given by = and is called RTGUUWTGJGCF.
mg r g
1036പWŚLJƐŝĐƐĨŽƌ//dͲ:͗DĞĐŚĂŶŝĐƐ//

BERNOULLI’S PRINCIPLE
The Swiss scientist Daniel Bernoulli in 1738 first derived the principle which is based on
the law of conservation of energy and applies to ideal fluids. According to this principle
the sum of pressure energy, kinetic and potential energies of an ideal fluid flowing along a
streamline is a constant. Bernoulli’s equation
1
P + rv 2 + r gh = Constant
2

DERIVATION OF BERNOULLI’S EQUATION


To derive Bernoulli’s equation, we can apply the work-energy theorem to the fluid in a
section of a flow tube. Consider the fluid initially lies between the two sections respectively.
In a time interval Dt, the fluid was initially at a moves to a¢, a distance v1 Dt. In the same
time the fluid initially at b moves to b¢, a distance v2 Dt.
v2Dt

v2
P2

b b¢

v1Dt
Streamline h2
P1 v1

a a¢
h1

Reference level

Fig. 6.105

If A1 and A2 are the cross-sectional areas at the two ends, then by the equation of
continuity, the volume of fluid DV passing any cross section in time Dt is
DV = A1 (v1 Dt) = A2 (v2 Dt)
or DV = A1Ds1 = A2Ds2
If P1 and P2 are the pressures at the two ends, then force at the cross section a is P1A1, and
that at b is P2A2. The net work done on the element during this displacement
W = F1 Ds1 – F2Ds2
= P1A1Ds1 – P2A2Ds2
= P1DV – P2DV = (P1 – P2)DV ...(i)
,LJĚƌŽƐƚĂƟĐƐŽƌ&ůƵŝĚ^ƚĂƟĐƐപ1037

%JCPIGKP-'HTQOCVQD
The mass of the fluid between a and a¢ Dm = Density ¥ Volume
= rDV
1
The kinetic energy of the fluid between a and a¢ K1 = Dmv12
2
1
= rDVv12
2
Similarly, at the end of Dt, the kinetic energy of the fluid between b and b¢,
1
K2 = rDVv22
2
Thus, the change in kinetic energy of the fluid between a and b,
DK = K2 – K1
1
= rDV(v22 – v12) ...(ii)
2
%JCPIGKP2QVGPVKCN'PGTI[
The potential energy of the mass entering at a in time Dt is,
U1 = Dmgh1 = rDVgh1
The potential energy of the mass leaving at b is,
U2 = Dmgh2 = rDVgh2
The change in potential energy between a and b is,
DU = U2 – U1
= rDVg(h2 – h1) ...(iii)
Now using the work-energy theorem
W = DK + DU
1
or (P1 – P2)DV = rDV(v22 – v12) + rDVg(h2 – h1)
2
After rearranging the above expression, we get
1 1
P1 + rv12 + r gh1 = P2 + rv22 + r gh2
2 2
We can write
1
P + rv 2 + r gh = Constant ...(1)
2
Bernoulli’s equation can also be written as
P v2
+ + h = Constant ...(2)
rg 2g
1038പWŚLJƐŝĐƐĨŽƌ//dͲ:͗DĞĐŚĂŶŝĐƐ//

Note:
1. In Bernoulli’s equation P is the absolute pressure, not gauge pressure.
2. In Eq. (i) each term has unit N/m2 and in Eq. (ii) each term has unit metre.
1
3. For horizontal streamline, h1 = h2, so P + rv 2 = Constant
2
4. In Bernoulli’s equation, the term (P + rgh) is called static pressure, because it is the
1
pressure of the fluid even if it is at rest, and the term rv 2 is called dynamic pressure
2
of the fluid. Bernoulli’s equation thus can be written as:
Static pressure + Kinetic pressure = Constant.

WORKED PROBLEMS
Problem 6.61 Water enters a house through a pipe 2.0 cm inside diameter, at an absolute
pressure of 4 ¥ 105 Pa. The pipe leading to the second-floor bathroom 5 m above is 1.0 cm
in diameter. When the flow velocity at the inlet pipe is 4 m/s, find the flow velocity and
pressure in the bathroom.
Solution By the continuity equation the flow velocity
A1v1 p (0.01)2
v2 = = ¥ 4 = 16 m/s Bathroom
A2 p (0.005)2 v2
Using Bernoulli’s equation between 1 and 2, we have
1 1
P1 + rv12 + r gh1 = P2 + rv22 + r gh2
2 2 House
5m
1
or P2 = P1 - r(v22 - v12 ) - r g (h2 – h1)
2
1 v1
= 4 ¥ 105 – ¥ 1000(162 – 42) – 1000 ¥ 9.8 ¥ 5
2
= 2.3 ¥ 105 Pa Fig. 6.106

Problem 6.62 The reading of pressure-meter attached with a closed pipe is 3.5 ¥ 105 N/
m2. On opening the valve of the pipe, the reading of the pressure-meter is reduced to 3.0 ¥
105 N/m2. Calculate the speed of the water flowing in the pipe.
Solution Before opening the valve
P1 = 3.5 ¥ 105 N/m2, v1 = 0
After opening the valve
P2 = 3.0 ¥ 105 N/m2
Let v2 be the speed of the water after opening of the valve, then for the horizontal pipe
1 1
P1 + rv12 = P2 + rv22
2 2
,LJĚƌŽƐƚĂƟĐƐŽƌ&ůƵŝĚ^ƚĂƟĐƐപ1039

1/2
È 2(P1 - P2 ) ˘
\ v2 = Ív12 + ˙
Î r ˚
As v1 = 0
1/2
È 2(P - P ) ˘
\ v2 = Í 1 2 ˙
Î r ˚
1/2
È 2(3.5 ¥ 10 5 - 3.0 ¥ 10 5 ) ˘
= Í ˙
Î 1000 ˚
= 10 m/s

APPLICATIONS OF BERNOULLI’S EQUATION


1. 2TGUUWTG FKHHGTGPEG When v1 and v2 are zero, the Bernoulli’s equation reduces to
P1 – P2 = rg(h2 – h1).
This is the same equation as we have derived in hydrostatics. Thus, the equation of
hydrostatics is special case of Bernoulli’s equation.
2. 8GPVWTKOGVGT It is an ideal device of measuring rate of flow of a liquid in a pipe. It is
also known as venturi tube or flow meter. The basic principle of venturimeter is that by
reducing the cross-sectional area of the flow passage, a pressure difference is created and
the measurement of the pressure difference enables the determination of the rate of flow
through the pipe.
%QPUVTWEVKQP As shown in Fig. 6.107 a venturimeter consists of (1) an inlet section followed
by a convergent cone, (2) a cylindrical throat, and (3) a gradually divergent cone. The inlet
section of the venturimeter is of the same diameter as that of the pipe which is followed by
a convergent cone.
A1

A2
v1
v2

Throat
Inlet h
Liquid of density rm

Differential manometer

Fig. 6.107 Venturimeter placed horizontally

Let area of cross sections of inlet and throat be A1 and A2 respectively. By the continuity
equation
1040പWŚLJƐŝĐƐĨŽƌ//dͲ:͗DĞĐŚĂŶŝĐƐ//

A1v 1 = A 2v 2 = Q (Rate of flow)


Q Q
\ v1 = and v2 = ...(i)
A1 A2

If r is the density of the flowing fluid and P1 and P2 are the pressures of fluid at inlet and
throat, then by Bernoulli’s equation
1 1
P1 + rv12 = P2 + rv22
2 2
2(P1 - P2 )
or v22 – v12 = ...(ii)
r

Substituting values of v1 and v2 from Eqs. (i) into (ii), we have

Q2 Q2 2(P1 - P2 )
2
- 2 =
A2 A1 r

È A2 - A2 ˘ 2(P1 - P2 )
or Q 2 Í 1 2 22 ˙ =
ÍÎ 1 2 ˙˚
A A r
2(P1 - P2 )
\ Q = A1 A2 ...(1)
r( A12 - A22 )
From the manometer P1 – P2 = rmgh
2rm gh
\ Q = A1 A2 ...(2)
r( A12 - A22 )
r h
If simple manometers are inserted in inlet and throat, then rm = r
r
2 gh
\ Q = A1 A2
A1 - A22
2
Fig. 6.108

6JG2KVQVVWDGQTƀQYOGVGT A Pitot tube is a simple device which is used to measure the


velocity of flow in the rivers. It is named in honour of its invertor Henry de Pitot. The basic
principle used in the device is that if the velocity of flow at a particular point is reduced to
zero. Which is known a stagnation point, the pressure there increases due to conversion of
kinetic energy into pressure energy. By measuring pressure head, we can calculate velocity
of flow.
Consider two points A and B as shown in Fig. 6.109. Using Bernoulli’s equation between
these points.
1 2
(Pa + rgho)+ rv + 0 = (Pa) + 0 + rg(ho + h)
2
,LJĚƌŽƐƚĂƟĐƐŽƌ&ůƵŝĚ^ƚĂƟĐƐപ1041

Pa

Pa h

ho
v
A

Fig. 6.109

After simplifying the above expression, we get

v= 2gh
Now applying Bernoulli’s equation between points 1 and 2, we have
1 1
P1 + rv12 + 0 = P2 + rv22 + r gh
2 2
Substituting v1 = ve, v2 = 0 and P1 = Pa, P2 = P in the above expression, we have
1
Pa + rve2 = P + rgh
2
After solving, we get
2(P - Pa )
ve = 2 gh +
r
Special Cases
(1) For large tank with large P, the term 2gh can be neglected

2(P - Pa )
\ ve =
r

(2) When the tank is open to atmosphere


P = Pa \ ve = 2gh
Thus, the speed of efflux of a liquid through a small hole in an open tank is equal to
the velocity which a body acquires in falling freely from the free liquid surface to the
orifice. This is called 6QTTKEGNNKŏUVJGQTGO.
(3) The reaction force on the container due to emerging of the liquid
F = rA1ve2 = rA1 ¥ 2gh
= 2rgh A1
1042പWŚLJƐŝĐƐĨŽƌ//dͲ:͗DĞĐŚĂŶŝĐƐ//

(4) For water in a closed tank without any pressure over its free surface
P =0 and ve ≥ 0
Pa 1.03 ¥ 10 5
fi h≥ =  10.3 m
rg 1000 ¥ 9.8
i.e., water will come out from a closed tank when height of water above the hole is
greater than or equal to 10.3 m.
(5) The distance at which the liquid strikes:
Let H be the height of liquid in the open container.
The time taken by liquid to hit the ground
1 2
(H – h) = 0 + gt
2 h
Ve
2( H - h)
\ t = H
g
H/2
The distance x = ve t
2( H - h)
= 2 gh ¥ x
g H

or x = 2 h( H - h) Fig. 6.110

4. Aerofoil or F[PCOKENKHVQPCGTQRNCPGYKPIU The design of aeroplane wings is made in


such a way that the curvature length of the upper part of the wing is greater than the lower
part. The orientation of the wing relative to the flow direction causes flow lines to crowd
together above the wing, corresponding increased flow velocity and decreased pressure in
this region, while below the wing the pressure remains nearly atmospheric. Because of this
pressure difference, there is a net upward force or lift on the wing.
v1, P1

1
2
v2, P2

Fig. 6.111

Suppose v1 and v2 are the velocities and P1 and P2 are the pressures at 1 and 2 respectively.
Clearly v1 > v2 and P1 < P2.
Lift force on the wing F = Pressure difference ¥ Projected area of wing
or F = (P2 – P1) Projected area of wing
%WTXGFRCVJQTUYKPIQHVJGDCNN/CIPWUGHHGEV
When one side of the ball is made smoother by rubing, it experiences greater drag force by
air on the rough side, and the ball starts spinning due to a net torque.
,LJĚƌŽƐƚĂƟĐƐŽƌ&ůƵŝĚ^ƚĂƟĐƐപ1043

F > F¢

Smooth side of the ball

Fig. 6.112

When such a ball is thrown horizontally with large speed, it deviated from its usual
parabolic path.
v
(v – u)
v
u

Par
abo
Sw lic
ing pa
pa th
(v + u) th

Fig. 6.113 Fig. 6.114

Suppose v is the speed of the air molecules in contact with the ball due to its turning
figure (a). When the ball moves forward, the air ahead of the ball rushes towards the ball
with speed u. The layer above the ball moves in a direction opposite to that of the spinning,
so the net speed becomes (v – u). The layer below the ball moves in the direction of spin, so
the net speed there becomes (v + u). Because of this difference in speed, the net difference
of pressure is created and the ball gets deviated from its usual parabolic path.
#VQOK\GTQT5RTC[GT Figure 6.115 shows an atomizer. When the rubber or sprayer balloon
is pressed, the air rushes out on the horizontal tube. By Bernoulli’s principle, pressure

P2

Rubber balloon

P1

Container

Fig. 6.115
1044പWŚLJƐŝĐƐĨŽƌ//dͲ:͗DĞĐŚĂŶŝĐƐ//

Ê 1 ˆ
there P2 becomes less than the pressure P1 in the container Á P + rv 2 = constant ˜ . As a result,
Ë 2 ¯
the liquid rises up in the vertical tube. The liquid mixed together with high speed air in the
tube produces fine spray.
 $NQYKPIQHHVJGTQQHFWTKPIYKPFUVQTODuring wind storm, high speed wind over the
roof creates low pressure. While pressure below the roof is equal to the atmospheric pressure.
This difference in pressure causes an upward thrust and the roof is lifted up. Once the roof
is lifted up, it will blown off with the wind.
P2

P2 < P1

P1 = Pa

Fig. 6.116 Blowing off the roof during wind storm

SPEED OF EFFLUX OR TORRICELLI’S LAW OF EFFLUX


Consider a tank containing liquid of density r with
P2 = P
a small hole on its side at a depth h from the free 2
surface of the liquid. Let P be the air pressure above P1 = Pa h
the liquid surface. (IIT Roorkee, 1988)
If A1 and A2 are the cross-sectional areas of the v1 Reference level
H
hole and the tank respectively, and v1 and v2 are the
liquid velocities at points 1 and 2, then by equation
of continuity
A 1v 1 = A 2v 2
A1 Fig. 6.117
or v2 = v1
A2
A1
For a small hole in a large container, Æ0 \ v2 = 0
A2
dx
For maximum x, =0
dh
d
or [2 h( H - h)] = 0
dh
H
fi h =
2
,LJĚƌŽƐƚĂƟĐƐŽƌ&ůƵŝĚ^ƚĂƟĐƐപ1045

HÊ Hˆ
and xmax = 2 ÁË H - ˜¯
2 2
or xmax = H

WORKED PROBLEMS
Problem 6.63 In a test experiment known as aerofoil on a model aeroplane in a wind
tunnel, the flow speeds on the upper and lower surfaces of the wing are 70 m/s and 63 m/s
respectively. What is the lift on the wing if its area is 2.5 m2? Density of air = 1.3 kg/m3.
Solution Let P1 and P2 be the pressures on the upper P1
and lower surfaces of the wing. Using Bernoulli’s
equation
1 1 P2
P1 + rv12 = P2 + rv22
2 2 Fig. 6.118
(Neglecting gravitation heat between lower and upper surfaces)
r
or (P2 – P1) = (v12 – v22)
2
1.3
= (70 2 - 632 )
2
= 605.15 N/m2
Lift force on the wing = (P2 – P1) ¥ Projected area of wing
= 605.15 ¥ 2.5
= 1.51 ¥ 103 N
Problem 6.64 A Pitot tube is mounted on an aeroplane wing to measure the speed of the
plane. The tube contains alcohol and shows a level difference of 40 cm. What is the speed
of the plane relative air? (sp. gr of alcohol = 0.8 and density of air = 1 kg/m3)
Solution Let v be the velocity of the plane w.r.t. air which is equal to the velocity of the air
in main pipe (see Fig. 6.119).
Using Bernoulli’s equation between A and B
Wind B
1 1
PA + rvA2 = PB + rvB2
2 2
A v
But vA = v and vB = 0
40 cm
1 2
\ rv = PB – PA
2
(In equilibrium the velocity vB of the fluid at point
B is zero and the pressure in the right arm is PB called Fig. 6.119
stagnant pressure)
1046പWŚLJƐŝĐƐĨŽƌ//dͲ:͗DĞĐŚĂŶŝĐƐ//

If r¢ is the density of alcohol in the tube, then


PB – PA = r¢gh

2 r ¢ gh
\ v =
r

2 ¥ 800 ¥ 9.8 ¥ 0.40


=
1
= 79.18 m/s
Problem 6.65 A fully loaded Boeing 737 aircraft has a mass of 3.3 ¥ 105 kg. Its total wing
area is 500 m2. It is in level flight with a speed of 960 km/h. (a) Estimate the pressure
difference between the lower and upper surface of the wings. (b) Estimate the fractional
increase in the speed of the air on the upper surface of the wing relative to the lower surface.
The density of air is r = 1.2 kg/m3 and g = 9.81 m/s2.
Solution
(a) If P1 and P2 are the pressures at the upper and lower surfaces of the aircraft, the
pressure difference
mg
DP = P2 – P1 =
A
3.3 ¥ 10 5 ¥ 9.81
= = 6.5 ¥ 103 N/m2
500
(b) If v1 and v2 are speeds of air on the upper and lower surfaces of the aircraft, then by
Bernoulli’s equation
1 1
P1 + rv12 = P2 + rv22
2 2
v12 - v22 P -P
or = 2 1
2 r
Êv +v ˆ DP
or (v1 – v2) Á 1 2 ˜ =
Ë 2 ¯ r
DP
or (v1 – v2)vav =
r
( DP/r )
v1 – v 2 =
vav
(v1 - v2 ) ( DP/r )
and =
vav v 2av
Here vav = 960 km/h = 267 m/s

v1 - v2 6.5 ¥ 10 3
\ = 0.08 = 8%
vav 1.2 ¥ 267 2
,LJĚƌŽƐƚĂƟĐƐŽƌ&ůƵŝĚ^ƚĂƟĐƐപ1047

Problem 6.66 Figure 6.120 shows how the stream of water


emerging from a tap “necks down” as it falls. The indicated cross-
sectional areas are A0 = 1.2 cm2 and A = 0.35 cm2. The two levels
are separated by a vertical distance h = 45 mm. What is the volume
flow rate from the tap? ++6,''5
Solution If v0 and v are the speeds at the respective sections, then
by the continuity equation A0, v0

A0v0 = Av ...(i) h
A, v
Using Bernoulli’s equation, we have
Fig. 6.120
1 2 1
Pa + rv0 + r gh = Pa + rv 2 + 0
2 2
or v2 = v02 + 2gh ...(ii)
Solving Eqs. (i) and (ii), we get

2 ghA2
v0 =
A02 - A2

2 ¥ 9.8 ¥ 0.045 ¥ 0.352


=
(1.22 - 0.352 )
= 0.286 m/s
The volume rate of flow
Q = A 0v 0
= (1.2 ¥ 10–4) ¥ 0.286
= 34 ¥ 10–6 m3/s
Problem 6.67 A cylindrical tank has a hole of 1 cm2 at its bottom. If the water is allowed
to flow into the tank from a tube above it at the rate of 70 cm3/s, then find the maximum
height up to which water can rise in the tank. (IIT Roorkee, 1979)
Solution As height of water in the tank increases, the efflux velocity and hence rate of flow
of emerging water also increases. At a certain height h the output becomes equal to input
and the level of water becomes constant.
\ Qin = Qout
Q
Q = a 2 gh
Q2
h =
2 ga 2
(70 ¥ 10 - 6 )2 h
=
2 ¥ 9.8 ¥ (1 ¥ 10 - 4 )2
= 2.5 ¥ 10 – 2 m a

Fig. 6.121
1048പWŚLJƐŝĐƐĨŽƌ//dͲ:͗DĞĐŚĂŶŝĐƐ//

TIME OF EMPTYING A TANK


Consider a large tank of cross-sectional area A, with a small hole of area a at the bottom of
the tank.
At any instant when height of liquid is h, the speed of the efflux through the hole v =
2gh and rate of out flow
q = a 2 gh
Let in time dt, the level of liquid in the tank decrease by dh. Thus, volume of liquid
emerging out from the hole
= volume of the shaded portion of the liquid
or q(dt) = A(dh)
A
or a 2 gh dt = A(dh) dh
A dh
\ dt =
a 2g h
h
With increase in time, h decreases, so
A - dh
dt = v a
a 2g h
Time of emptying the tank from h1 to h2 Fig. 6.122

t h2
A - dh
Ú dt = a 2 g Ú h
0 h1

A 2
or t = ( h1 - h2 )
a g

    ȍ   Ȏ    


Consider a large tank of cross-sectional area A, with a small hole
of area a at its bottom (or at side wall). Let there be a constant Q
inflow of liquid Q m3/s and at the same time the liquid is
discharged through the hole.
dh
Let at any instant liquid surface be at a height h above the
h2
centre of the hole and in time dt the level is increased by dh.
h1 h
Then volume of liquid added to the tank is Adh. Further, in
time dt the volume of the inflow of the liquid into tank is Qdt,
and during the same time the volume of the liquid discharged q = a 2gh
through the hole is qdt, where
Fig. 6.123
q = a 2 gh
Thus, net volume of the liquid added in time dt
= Qdt – qdt = (Q – q)dt
,LJĚƌŽƐƚĂƟĐƐŽƌ&ůƵŝĚ^ƚĂƟĐƐപ1049

= (Q – a 2 gh )dt
Thus, Adh = (Q - a 2 gh )dt
Adh
or dt = …(1)
(Q - a 2 gh )
If the liquid level rises from h1 to h2 in time t, then
t h2
Adh
Ú dt = Ú (Q - a 2 gh )
0 h1

Substituting
Q - a 2 gh = z
(Q - z)2
or h =
2 ga 2
(Q - z)
Also, dh = - dz
ga 2
Substituting these values in Eq. (1), we get

È (Q - z)dz ˘
A Í- ˙
Î ga 2 ˚
t = Ú
z
A ÊQ ˆ
= -
ga 2 Ú ÁË z - 1˜¯ dz
A
= - [Q ln z - z]hh12
ga 2
A
= - 2
[Q ln(Q - a 2 gh ) - (Q - a 2 gh )]hh12
ga
A È ÔÏ Q - a 2 gh2 Ô¸ ˘
= - 2
ÍQ ln Ì ˝ + a 2 g ( h2 - h1 ˙ ...(2)
ga ÍÎ ÓÔ Q - a 2 gh1 Ô˛ ˙˚

  
A siphon is a long bent pipe which is used to drain water from the reservoir at higher level
to a reservoir at lower level. The rising portion of the siphon is known as inlet leg, the
highest point is known as summit and the portion between the summit and lower reservoir
is known as outlet leg (see Fig. 6.124).
From Fig. 6.124
v1 = v2 = 0 and P 1 = P2 = P5 = Pa
1050പWŚLJƐŝĐƐĨŽƌ//dͲ:͗DĞĐŚĂŶŝĐƐ//

For a pipe of uniform cross section v3 = v4 = v5 = v Summit


Applying Bernoulli’s equation between (1) and (5), Inlet 4
we have leg
h1
1 Pa
Pa = Pa + rv22 - r gh2 1 3
2 2

fi v2 = 2 gh2

Thus, for v1 = v2 = 0 and P1 = P5 = Pa h2

Applying Bernoulli’s equation at 1 (or 2), 3, 4, 5


1 1
Also, Pa = P3 + rv 2 = P4 + rv 2 + r gh1 5
2 2
1 Fig. 6.124
= Pa + rv 2 + r gh2
2
From the above equation following conclusion can be made
(i) P4 < P3 < Pa
(ii) rg(h1 + h2) = Pa – P4
Pa - P4
fi (h1 + h2) =
rg
Pa
or h1 + h2 <
rg
Note: Point 3 is just above point 2. v2 = 0 but v3 = v.

WORKED PROBLEMS
Problem 6.68 A liquid is poured into a vessel at rest with the hole in a wall closed by a
valve. It is filled by liquid up to height h above the valve. What horizontal acceleration a
should the vessel be moved, so that liquid does not come out when valve is opened?
Solution
/GVJQF+ Let A be the area of the hole. The liquid which comes out from the hole, exerts
force on the rest part of liquid. Let Dm amount of liquid leave the hole in Dt time with velocity
v. By Newton’s second law
DP Dmv
F = =
Dt Dt
h
DV A
= vr F
Dt
DV
where = Q, rate of liquid coming out of the hole.
Dt
This force acts in backward direction of liquid confined in shaded
portion. The acceleration l

Fig. 6.125
,LJĚƌŽƐƚĂƟĐƐŽƌ&ůƵŝĚ^ƚĂƟĐƐപ1051

F rvQ r Av 2
a = = = (Q = Av)
m r Al r Al
v 2 2 gh
As v = 2gh \ a= =
l l
/GVJQF++ The vessel is given an acceleration of such a value so that
h
level of liquid at the valve become zero. Let a be the acceleration of
the vessel towards right, then q h
a
tan q = x
g
2h a
or =
l g
l
2gh Fig. 6.126
fi a =
l

Problem 6.69 A tank filled with water (density rw = 1000 kg/m3) and oil of (density roil =
900 kg/m3). The height of water is 1.00 m and of the oil is 4.00 m. Find the velocity of efflux
through a hole at the bottom of the tank. (BITSAT, 2008)
Solution
/GVJQF+ Applying Bernoulli’s equation between (1) and (2), we get
1
1 2 1
Pa + rv1 + ( rw g ¥ 1 + roil ¥ g ¥ 4) = Pa + rw v22 + 0
2 2
1 2 Oil
Since v1 << v2, so rv1 can be neglected. 4m
2
2( rw g ¥ 1 + roil ¥ g ¥ 4)
\ v2 =
rw Water 1m
2 RL
2(1000 ¥ 9.8 ¥ 1 + 900 ¥ 9.8 ¥ 4)
=
1000 Fig. 6.127
= 9.5 m/s
/GVJQF++ Height of water which exerts the same pressure on interface, whatever oil exerts,
let it is h.
\ h rw g = 4 ¥ roil ¥ g
4 ¥ 900
or h = = 3.6 m Water 3.6 m
1000
Effective height of water over the hole
H = 1 + 3.6 = 4.6 m
Water 1m
\ ve = 2 gh = 2 ¥ 9.8 ¥ 4.6
= 9.5 m/s
Fig. 6.128
1052പWŚLJƐŝĐƐĨŽƌ//dͲ:͗DĞĐŚĂŶŝĐƐ//

Problem 6.70 A bent tube is lowered into a water stream as shown 2


in Fig. 6.129. The velocity of stream relative to the tube is equal to v h
= 2.5 m/s. The closed upper end of the tube located at the height h0 h0
= 12 cm has a small orifice. To what height h will the water jet spurt? y
v
Solution Consider two points 1 and 2. Point (1) is y0 below the
free surface of the liquid. Applying Bernoulli’s equation between
(1) and (2), we have Fig. 6.129
1
(Pa + y0 rg) + rv2 + 0 = Pa + 0 + r(y0 + h0 + h)g
2
1 2
or rv = r(h0 + h)g
2
1
or ¥ 2.52 = (0.12 + h) ¥ 9.8
2
\ h = 0.2 m
Problem 6.71 In the arrangement shown in Fig. 6.130 a viscous liquid whose density is r
= 1.0 g/cm3 flows along a tube out of a wide tank A. Find the velocity of the liquid flow, if
h1 = 10 cm, h2 = 20 cm, and h3 = 35 cm. All distances l are equal.

h3

h2
h1
A B C D v

l l l

Fig. 6.130

Solution Loss of heat from A to B = h3 – h2 = 35 – 20 = 15 cm from B to C = h2 – h1 =


20 – 10 = 10 cm.
On the similar way from C to D = 5 cm
\ Kinetic head available at D, h = 35 – (15 + 10 + 5) = 5 cm
1 2
\ rv = (0.05)rg
2
or v = 2 ¥ 0.05 ¥ 10
= 1 m/s
Problem 6.72 The pressure in tank A measures 700 kg/m2 gauge. What is the pressure in
the tank B if roil = 800 kg/m3.
,LJĚƌŽƐƚĂƟĐƐŽƌ&ůƵŝĚ^ƚĂƟĐƐപ1053

H2O

0.4 m 0.35 m

Oil
0.2 m
B
Hg

Fig. 6.131

Solution By Pascal’s law, we have


PA + rHg g ¥ 0.2 + rHg ¥ 0.4 = PB + roil g ¥ 0.25
Given PA = 7000 kg/m2.
After simplifying, we get PB = 3620 kg/m2.
Problem 6.73 A U-tube contains water and liquid separated by
mercury. The mercury columns in the two arms are in level with
10 cm of water in one arm and 5 cm of liquid in the other. What Liquid
10 cm
is the specific gravity of the liquid? (BITSAT, 2007) Water 5 cm
Mercury
A B
Solution By Pascal’s law
PA = PB
or rwg ¥ 10 = rsp g ¥ 5 Fig. 6.132

10
\ rsp =
5
=2
Problem 6.74 A large open top container of negligible mass and uniform cross-sectional
area A has a small hole of cross section A/100 in its side wall near the bottom. The container is
kept on a smooth horizontal floor and contains a liquid of density r and mass m0. Assuming
that the liquid starts flowing out horizontally through the hole
A
at t = 0, calculate: ++6,''
(i) the acceleration of the container and
(ii) its velocity when 75% of the liquid has drained out.
Solution h
m0
(i) The height of liquid in the container, h =
rA
Initially, the velocity of efflux v = 2gh
Force acts on the container
F = rvQ = rav2 A
100
= r ¥ A ¥ 2 gh
100 Fig. 6.133
1054പWŚLJƐŝĐƐĨŽƌ//dͲ:͗DĞĐŚĂŶŝĐƐ//

Ê A ˆ
rÁ ¥ 2 gh
F Ë 100 ˜¯
\ Acceleration, ax = =
m0 m0

Ê A ˆ m
rÁ ¥ 2g ¥ 0
Ë 100 ˜¯ rA
=
m0
2g 1
= = m/s2 (Constant value)
100 5
h
(ii) Let in time t the level of liquid fall from h to(75% of the liquid drained out). The
4
time of emptying a tank from h1 to h2 is given by

A 2
t = a g ( h1 - h2 )

A 2 Ê hˆ
= Á h-
A
gË 4 ˜¯
100
100 ◊ 2 Ê 1ˆ
= ÁË 1 - ˜¯ h
g 2

100
= h
2g

100 m0
=
2g rA

Velocity of the container in time t


v = at
1 100 m0
= ¥
5 2g rA

Problem 6.75 A liquid of density 900 kg/m3 is filled in a cylindrical tank of upper radius
0.9 m and lower radius 0.3 m. A capillary tube of length l is attached at the bottom of the
tank as shown in Fig. 6.134. The capillary has outer radius 0.002 m and inner radius a. When
pressure P is applied at the top of the tank volume flow rate of the liquid is 8 ¥ 10–6 m3/s
and if capillary tube is detached, the liquid comes out from the tank with a velocity of 10
m/s. Determine the coefficient of viscosity of the liquid. ++6,''
a2
[Given pa2 = 10– 6 m2, and = 2 ¥ 10– 6 m]
l
,LJĚƌŽƐƚĂƟĐƐŽƌ&ůƵŝĚ^ƚĂƟĐƐപ1055

Solution
0.9 m

P
v1

0.3 m

v2 l
(a) (b)

Fig. 6.134

Without the capillary tube, the liquid comes out from the narrower part of the cylinder
a2v2
\ a 1v 1 = a 2v 2 fi v1 =
a1
Applying Bernoulli’s equation between (1) and (2), we have
1 2 1
(P + Pa) + rv1 + r gH = Pa + rv22 + 0
2 2
1
or (P + rgH = r (v22 – v12)
2
1 È Êa v ˆ ˘
2
= r Ív22 - Á 2 2 ˜ ˙
2 Í Ë a1 ¯ ˙
Î ˚
1 2 È a22 ˘
= rv2 Í1 - 2 ˙
2 Î a1 ˚
where v2 = 10 m/s
1 È È p (0.3)2 ˘ 2 ˘

\ P + rgH = ¥ 900 ¥ (10) 1 - Í ˙ ˙
2 Í Î p (0.9)2 ˚ ˙
Î ˚
1 Ê 1ˆ
= ¥ 900 ¥ 100 Á 1 - ˜
2 Ë 81¯
1 80
= ¥ 900 ¥ 100 ¥
2 81
4
= ¥ 105 N/m2
9
With the capillary tube the rate of flow = 8 ¥ 10– 6 m3/s.
1056പWŚLJƐŝĐƐĨŽƌ//dͲ:͗DĞĐŚĂŶŝĐƐ//

Pressure difference across the tube (Pa + P + Hrg)


DP = (Pa + P + Hrg – Pa)
= P + Hrg
4
¥ 105 N/m2
= Pa
9 Input
Now using Poisulli’s equation, we have Fig. 6.135
4
p ( DP)r
Q =
8hl

Ê4 5ˆ 2
ÁË ¥ 10 ˜¯ (p a ) Ê a 2 ˆ
9
or 8 ¥ 10– 6 = Á l ˜
8h Ë ¯
-6 -6
Ê4 ˆ (10 )(2 ¥ 10 )
or 8 ¥ 10– 6 = Á ¥ 10 5 ˜
Ë9 ¯ 8h

1 N-s
After solving, h =
720 m 2

Problem 6.76 The gate OA shown is hinged at O and is in the form of a quadrant of a circle
of radius 1 m. It supports water on one side as shown Fig. 6.136. If the width of the gate is
3 m, calculate the force required to hold the gate in position
C Fv A
F

1m C
Fh
h/3

O 4R
3p

Fig. 6.136

Solution Given radius R =1m


Width of the gate, b =3m
Fn = Force on vertical projection of AOC
= pav ¥ (OC ¥ b)
rgh
= ¥ (1 ¥ 3)
2
1000 ¥ 9.8 ¥ 1
= (1 ¥ 3)
2
= 14715 n
,LJĚƌŽƐƚĂƟĐƐŽƌ&ůƵŝĚ^ƚĂƟĐƐപ1057

Fv = Weight of the water contained in OAC.


Ê1 ˆ
= Á p R 2 ¥ b˜ r g
Ë4 ¯
1
= p(1)2 ¥ 3 ¥ 1000 ¥ 9.81
4
= 23102.5 N
h 1
The horizontal force Fn acts at a height= m above the hinge whereas force Fv acts
3 3
4R 4 ¥ 1
through the centroid of OAC which is located at a distance = from OC.
3p 3p
Taking moment of forces about the hinge O
1 Ê 4 ¥ 1ˆ
F ¥ 1 = 14715 ¥ + 23102.5 ¥ Á
3 Ë 3p ˜¯
\ F = 14715 N
Problem 6.77 The pressure in a pipe at M is to be measured by the open manometer as
shown. Fluid A is oil of density 800 kg/m3 and fluid B is mercury of density 13.6 ¥ 103 kg/
m3. Estimate the absolute pressure at M.

25 cm
1
2 3

rA rB
75 cm

Fig. 6.137

Solution Gauge pressure at 3, P3 = 0.25 ¥ 13.6 = 3.4 m of water height


Pressure at 1 = Pressure at 3
or P1 = 3.4 m of water
Pm = P1 + 0.75 ¥ 800
= 3.4 + 0.6 = 4.0 m of water
Absolute pressure at M = gauge pressure at M + atmospheric pressure
= 4 m + 10.3 m
= 14.3 m of water
1058പWŚLJƐŝĐƐĨŽƌ//dͲ:͗DĞĐŚĂŶŝĐƐ//

or [Pm] abs = rwgh


= 1000 ¥ 9.81 ¥ 14.3
= 14.05 ¥ 103 N/m2

Problem 6.78 A tube BCA bent at a right angle B


open at B and closed at A, filled with water, is
accelerated to the right as shown. Determine the
acceleration at which the pressure at A becomes
atmospheric. a
H = 30 cm
Solution The pressure at A to be atmospheric,
the force exerted by the water column of height
30 cm and 40 cm long column is to be balanced
by the pseudo force. If a is the acceleration of A
tube, then
Applying Pascal’s law starting from point B C
L = 40 cm
PB + PgH – PaL = PA Fig. 6.138
Since PB = PA = Patm
h
a = g
l
h
\ a = g
l
0.30
= ¥ 9.8
0.40
= 7.36 m/s2

Problem 6.79 A tank and a through are placed on a flat car as shown in Fig. 6.139. Water
issues from the tank through a 5 cm diameter nozzle at 5 m/s and strikes the through which
turns it by 45%. Determine the compression of the spring of stiffness 2000 N/m.

45°

Fig. 6.139
,LJĚƌŽƐƚĂƟĐƐŽƌ&ůƵŝĚ^ƚĂƟĐƐപ1059

Solution The rate of flow through the nozzle


p
Q = Av = (0.05)2 ¥ 5
4
= 9.81 ¥ 10– 3 kg/m3
The reaction force at the mouth of nozzle
F = rvQ
= 1000 ¥ 5 ¥ 9.81 ¥ 10– 3
= 49.05 N
The direction of F is along the direction of nozzle.
The horizontal component of this force
49.05
Fx = F cos 45° = N
2
If x is the compression of spring, then
Fx = kx
49.05
\ x = Fx/k =
2 ¥ 2000
= 0.0173 m
Problem 6.80 A bellmouth entry (which
ensures the flow is uniform) in front of an air-
compressor is to be calibrated for discharge Air 20 cm
through it in terms of height h of water in a
single tube manometer as shown. If h = 0.2 m
and the density of air is 1.2 kg/m3, estimate
the discharge of air through the compressor.
h
Solution If v is the velocity of flow of the
air then
1
rair v 2 = rw gh
2
2rw gh
\ v =
rair Fig. 6.140

2 ¥ 1000 ¥ 9.8 ¥ 0.2


= = 57.14 m/s
1.2
The rate of flow of air or discharge through the compressor
Q = Av
p
= d2 ¥ v
4
p
= (0.20)2 ¥ 57.14
4
= 1.8 m3/s

Problem 6.81 A rod of length 6 m has mass 12 kg. If it is hinged at one end at a distance
of 3 m below a water surface:
1060പWŚLJƐŝĐƐĨŽƌ//dͲ:͗DĞĐŚĂŶŝĐƐ//

(i) What weight should be attached to the other end so that 5 m of the rod is submerged?
(ii) Find the magnitude and direction of force exerted by the hinge on the rod. The specific
gravity of the material of the rod is 0.5.
Q

F w
G

R
B
W
P q

Fig. 6.141

Solution As shown in Fig. 6.142, the forces acting on the


rod are: w
1. The weight of rod 12g N acting downwards through Th
R
the c.g. of the rod, i.e., at a distance of 3 m from the 3m q
hinge. W
2. Force of buoyancy through the c.g. of displaced liquid
vertically upwards. As O
Weight of displaced water Fig. 6.142
R.D. =
Force of buoyancy
Ê 5 ˆ 12 g
Force of buoyancy = Á ˜ ¥ =20 g N
Ë 6 ¯ 0.5
and acts at a distance 2.5 m from the hinge.
3. Extra weight w at the other end of the rod at a distance 6 m from O acting vertically
downwards.
4. Reaction R at the hinge at O will be vertical (as here all the forces are vertical, so for
horizontal equilibrium of the rod RH = 0).
So for translatory equilibrium of rod,
R + Th – W – w = 0
i.e., w – R = 20g – 12g = 8g ...(i)
And for rotational equilibrium of rod (taking moments about O)
6 5
–12g ¥ sin q + 20 g ¥ sin q - w6 sin q = 0
2 2
or w = (14/6)g = 2.33 gN = 2.33 kg ...(ii)
Substituting the value of w from Eqs. (ii) in (i) and solving for R, we get
R = (2.33 – 8)gN = – 5.67 kg
Negative sign implies that R is directed vertically downwards.
,LJĚƌŽƐƚĂƟĐƐŽƌ&ůƵŝĚ^ƚĂƟĐƐപ1061

Problem 6.82 A garden sprinkler with two nozzles

°
30
0.1 cm diameter each at 20 cm and 10 radii is connected A
B
across at tap capable of 6 litres/minute discharged. The
nozzle discharges water upwards and outwards from 10 cm 20 cm
45°
the place of rotation. What torque will the sprinkler
exert on the hand? Fig. 6.143
Solution Assuming the discharge to be divided equally between the two nozzles, so
QA = QB = 3 litre/minute
3 ¥ 10 - 3
= = 50 ¥ 10– 6 m3/s
60
The force exerted by the discharging water on the nozzle is given by
Q2 Q FA

°
30
F = rvQ = r as v= B O A
A A
1000 ¥ (50 ¥ 10 - 6 )2 FB
45° 10 cm 20 cm
Thus, FA = FB =
p
(0.01)2 Fig. 6.144
4

= 31.85 ¥ 10– 3 N
The net torque at the hand (about)
t = FA cos 30° ¥ 0.20 + FB cos 45° ¥ 0.10
3 1
= 31.85 ¥ 10– 3 ¥ ¥ 0.20 + 31.85 ¥ 10 – 3 ¥ ¥ 0.10
2 2
= 0.078 N-m
Problem 6.83 Calculate the minimum force F required
to keep the timber floating horizontally. The timber
cross section is 10 cm ¥ 10 cm and the specific gravity
5m
0.6. Take g = 10 m/s2. Timber float
Solution The weight of the timber float
= mg = r(Al)g Water
String
= 600 ¥ (10 ¥ 10 ¥ 10– 4) ¥ 5 ¥ 10
= 300 N
The buoyant force on the timber float
Fb = rwAg = rw ¥ (Al)g Fig. 6.145

= 1000 ¥ (10 ¥ 10 ¥ 10– 4 ¥ 5) ¥ 10 Fb = 500 N


= 500 N
If F is the minimum force required, then for the rotational O
300 N F
equilibrium, we have 2.5 m

Ât = 0 Fig. 6.146
1062പWŚLJƐŝĐƐĨŽƌ//dͲ:͗DĞĐŚĂŶŝĐƐ//

Taking moment of all the forces acting on the timber about O, we get
300 ¥ 2.5 + F ¥ 5 – 500 ¥ 2.5 = 0
\ F = 100 N
Problem 6.84 Determine the torque required to turn a 10 cm long 5 cm diameter shaft at
500 revolutions per minute in a 5.1 cm diameter concentric bearing flooded with a lubricating
oil of viscosity 100 centipoise.
Solution Angular speed of the shaft w = 2pn dy
Peripheral speed of the shaft v = wr = 2pnr v
dF
500
= 2p ¥ ¥ 0.025 = 1.31
60 r
dq
= 1.31 m/s
O
Since bearing is at rest, so velocity gradient
dv 1.31
=
dy (5.1 - 5) ¥ 10 - 2
Fig. 6.147
= 2620/s
Now choose an element of angular width dq, the area over which viscous force of the
element acts,
dA = (rdq)l.
The viscous force on the element
dv
dF = h(dA)
dy
Torque of this force, dt = (dF)r
The torque required to turn the shaft
2p 2p
dv
t = Ú (dF)r = Ú h(dA) dy r
0 0
2p
dv
= Ú h(rdq l) dy r
0
Ê dv ˆ
= hr 2l Á ˜ ¥ 2p
Ë dy ¯
= 0.1 ¥ (0.025)2 ¥ 0.10 ¥ 2620 ¥ 2p
= 0.1028 N-m
Problem 6.85 Two holes are drilled in the wall of a vessel filled with water. The distances
of the holes from the level of water are h and h + H. Find the distances x and y as shown
in Fig. 6.148 where the streams flowing out of the holes intersect. Assume that the level of
water is maintained in the vessel by regulated supply of water.
,LJĚƌŽƐƚĂƟĐƐŽƌ&ůƵŝĚ^ƚĂƟĐƐപ1063

Solution At the point P, point of intersection of streams, the


A x
range for the two streams are same. As the range of liquid
h
issuing from a depth h from the surface and from the base range B
is same, hence H y
CD = AB = h C
or y = H + h + h = H + 2h D
P
The time taken by water coming out of C to reach P,
Fig. 6.148
2h
t =
g
Velocity of water coming out of C,
v = 2 g( H + 2 h)
Hence, required value of x = vt

2h
= ¥ 2 g( H + 2 h) = 2 h( H + 2 h)
g

Problem 6.86 A small hole is made at a height of h¢= (1 - 2 ) m


from the bottom of a cylindrical water tank and at a depth of h =
2 m from the upper level of water in the tank. The distance where
the water emerging from the hole strikes the ground is
R
(a) 2 2 m (b) 1 m
Fig. 6.149
(c) 2 m (d) None of these
Solution Applying Bernoulli’s Principle between the points 1 and 2
rgh1 + P1 + (1/2) rv12 = P2 + (1/2) rv22 + rgh2
Substituting h1 = h2 = h¢v1 = a/Av2 or, v ª as a << A h

v1= 0, P1 = P0 + rgh, P2 = P0,


we obtain, rgh = 1/2 r v22 v2 = 2gh = v (say), h¢

P0 is the atmospheric pressure. R


The range R = v2 ¥ t, (where t = time of fall) can be given by Fig. 6.150

2 h1 2 h1
h1 = 1/2 gt2 fi t= fi R = v2
g g

Putting v2 = 2gh , we obtain R = 2 h h¢


1
Putting h’ = m and h = 2 m, we obtain R = 2 m
2
Therefore, answer is (c).
1064പWŚLJƐŝĐƐĨŽƌ//dͲ:͗DĞĐŚĂŶŝĐƐ//

Problem 6.87 A light cylindrical vessel is kept on a horizontal surface. Its bases are a is A.
A hole of cross-sectional area a is made just at its bottom side. The minimum coefficient of
friction necessary for sliding of the vessel due to the impact force of the emerging liquid is
(a << A) ++6,'' 1PNKPG 
(a) varying (b) a/A
(c) 2a/A (d) None of these
Solution The velocity of efflux of the liquid is given as
v= 2gy
\ The impact force of the emerging liquid on the vessel + liquid
content is equal to y
dm
F = v = varv = arv 2 v
dt
N
fi F = ar ( 2 gy )2 = 2argy
The force of friction = f = F = 2argy Fig. 6.151
fi mN = 2argh fi m (Argh) = 2a rgy
2a
fi m =
A
\ Therefore, answer is (c).
Problem 6.88 On the opposite sides of a wide vertical vessel filled with water two indentical
holes are opened, each having cross-sectional area S. The height difference between them is
equal to Dh. Find the resultant force of reaction of the water flowing out of the vessel.
Solution The situation is shown in Fig. 6.152.
Discharge per sec through A = SvA
Discharge per sec through B = SvB h

Force of reaction at A = r QA vA = r S vA2 A VA


Similarly, force of reaction at B = r SvB2
Dh
Net force F = rS(vA2 – vB2) ...(1) S
Applying Bernoulli’s theorem at points A and B, we have VB B

1 2 1
Pa + h rg + rvA = Pa + ( h + Dh)r g + rvB2
2 2
Fig. 6.152
Here Pa is atmospheric pressure
1
r[vA2 - vB2 ] = Dh r g ...(2)
2
From Eq. (1) (vA2 – vB2) = (F/r S)
Substituting this value in Eq. (2), we get
1 Ê F ˆ
r = Dh r g
2 ÁË r S ˜¯
\ F = (2Dh r gS)
,LJĚƌŽƐƚĂƟĐƐŽƌ&ůƵŝĚ^ƚĂƟĐƐപ1065

Problem 6.89 Show comparison between accelerated fluid and accelerated pendulum.
Solution Comparison of an Accelerated fluid with an accelerated pendulum.

‘•–ƒ– ‘”‹œ‘–ƒŽ ‡Ž‡”ƒ–‹‘


Under the action of gravity (mg) and pseudoforce (ma) the
geff g
pendulum attains a new equilibrium position with respect ma q a
a y
to the trolley. The pendulum orients itself in the direction of geff mg
x
net effective gravity (geff)
Fig. 6.153(a)
a
tan q =
g

Every fluid element attains an equilibrium position under q


the action of gravity and pseudoforce. The free surface of q a
the liquid orients itself perpendicular to the direction of geff
dy y
net effective gravity. dx x
a Fig. 6.153(b)
tan q =
g
a

Constant Vertical Acceleration


 K  %QPUVCPVWRYCTFCEEGNGTCVKQP T
m
s %QUILIBRIUM POSITION DOES NOT CHANGE IT REMAINS VERTICAL
m(g + a)
s 4ENSION IN THE STRING INCREASES IE geff = g + a
T = m (g + a) Fig. 6.154(a)

s &REE SURFACE REMAINS HORIZONTAL a


s 0RESSURE AT EVERY POINT INCREASES !T THE POINT A,
r h
A geff
p = r(g + a)h
geff = g + a
 KK  %QPUVCPVFQYPYCTFCEEGNGTCVKQP
a Fig. 6.154(b)
s %QUILIBRIUM POSITION DOES NOT CHANGE
s 4ENSION IN THE STRING DECREASES IE
T
T = m(g – a) m
m(g – a)
s 4ENSION BECOMES ZERO AS a = g.
geff = g – a

Fig. 6.155(a)
1066പWŚLJƐŝĐƐĨŽƌ//dͲ:͗DĞĐŚĂŶŝĐƐ//

s &REE SURFACE REMAINS HORIZONTAL a


s 0RESSURE DECREASES AT EVERY POINT !T THE POINT A,
r h
A geff
p = r(g – a)h
geff = g – a
s 0RESSURE BECOMES ZERO EVERYWHERE AS a = g.
Fig. 6.155(b)
 KKK  %QPUVCPVFQYPYCTFCEEGNGTCVKQPITGCVGTVJCPITCXKV[ a > g)
The pendulum gets inverted to align itself in the direction of net geff = a – g
gravity.
The fluid occupies the upper part of the container. a>g

geff
a>g
Fig. 6.156(a)

geff = a – g

Fig. 6.156(b)

 KX  (NWKFUWDLGEVGFVQEQODKPGFJQTK\QPVCNCPFXGTVKECNCEEGNGTCVKQP
ax
tan q =
g + ay

ay

ax

ax
geff
q g + ay
geff

Fig. 6.157(a)
ax
tan q =
g + ay

ay
q
geff ax

q g + ay
geff
ax

Fig. 6.157(b)
,LJĚƌŽƐƚĂƟĐƐŽƌ&ůƵŝĚ^ƚĂƟĐƐപ1067

Problem 6.90 A closed cylinder 0.4 m in diameter and 0.4 m in height is filled with oil of
specific gravity 0.80. If the cylinder is rotated about its vertical axis at a speed or 200 rpm,
calculate the thrust of oil on top and bottom covers of the cylinder.
200 rpm

O
A B

H = 0.4 m

R = 0.2 m

dr

r (Top view)

Fig. 6.158 A rotating closed cylinder filled with oil

Solution In the top plane AB of the cylinder (Fig. 6.158), pressure head at any radial
distance r is given by
p/rg = w2 r2/2g
where, w is the angular velocity of the cylinder.
Considering a thin annular ring of radius r and thickness dr (Fig. 6.158), and summing
up the forces on all such elemental rings. We have thrust on top plane,
R R
prw 2 R 4
Ú p2p r dr = prw Ú r dr =
2 3
FT =
0 0
4
2p ¥ 200
Here, w = = 20.94 rad/sec
60
R (the radius of the cylinder) = 0.2 m

p ¥ 0.8 ¥ 10 3 ¥ (29.94)2 (0.2)4


Therefore, FT =
4
= 440.81 N
The radial distribution of pressure due to rotation will remain same for both the top and
bottom covers. But the bottom cover experiences an additional hydrostatic thrust due to the
weight of liquid above it.
1068പWŚLJƐŝĐƐĨŽƌ//dͲ:͗DĞĐŚĂŶŝĐƐ//

Hence, the thrust at the bottom cover


F = FT + rgH ¥ pR2
= 440.81 + 0.8 ¥ 103 ¥ 9.81 ¥ 0.4 ¥ p ¥ (0.2)2
= 835.29 N

Problem 6.91 A solid hemisphere of radius R is made to


just sink in a liquid of density r. Find the (a) vertical thrust
on the curved surface, (b) vertical thrust on the flat surface,
(c) side thrust on the hemisphere, and (d) total hydrostatic
force acting upon the hemisphere. R

Solution
(a) Vertical thrust of the liquid to weight of the liquid
Fig. 6.159
column above the curved (spherical) surface
Fv = Vrg,
where V = volume of the shaded portion = volume of the cylinder – volume of
hemisphere
3 2 3
= (p R ) - p R
3
p R3
=
3

(i) (ii)

Fig. 6.160

Substituting V in the equation Fv = Vrg, we have


p R3
rg (down)
Fv =
3
(b) The upward thrust on the base of the hemisphere is
F¢v = Vr g,
where V = volume of the liquid column above the base
= (pR2)R = pR3
3
Then, we have F¢v = pR rg (up)
(c) Side thrust Fh = 0 (as discussed earlier)
(d) The total hydrostatic force is
F = F¢v – Fv
,LJĚƌŽƐƚĂƟĐƐŽƌ&ůƵŝĚ^ƚĂƟĐƐപ1069

p R3
= p R3 r g - rg
3
2 3
= p R rg
3
Student Task
s )F THE HEMISPHERE IS INVERTED AND PRESSED IN THE LIQUID SUCH THAT ITS BASE GOES BELOW
the liquid by a distance R, find the vertical thrust on its curved surface.
5p 3
[Ans. R r g ≠]
3

Fig. 6.161

Problem 6.92 A rubber ball of density s (< r) is released from point A at a height h from
the surface of a swimming pool. If the density of water is r and the ball does not strike the
bottom of the swimming pool, find the maximum distance covered by the ball inside water
during its descent. Neglect friction between ball, water and air.
Solution Let the ball accelerate up inside the water with an A
acceleration a. Applying v2 = u2 + 2as for the ball from A to B and
from B to C, we have the following equations.
g
vB2 = vA2 + 2gh …(i) h
vC2 = vB2 + 2(– a)h¢ …(ii)
Substituting vA = vC = 0 as the ball is released from rest from A B
and it remains stationary momentarily at C, O

We have vB2 = 2gh



Then, we have 0 = 2gh – 2ah¢ Êr ˆ
g Á - 1˜
g Ës ¯
This gives h¢ = h
a
Êr ˆ C
where a = g Á - 1˜
Ës ¯ h
Then, h¢ = Fig. 6.162
Êr ˆ
ÁË - 1˜¯
s
1070പWŚLJƐŝĐƐĨŽƌ//dͲ:͗DĞĐŚĂŶŝĐƐ//

Applying work energy theorem from A to C, we have


Wgr + Wbuoyant = DK
where Wgr = mg(h + h¢), Wbuoyant = Fbh¢
and DK = KC – KA = 0
Êr ˆ
This gives h¢ = Á - 1˜
Ës ¯
Êr ˆ
After substituting Fb = mg Á - 1˜
Ës ¯

Student Task
s )N THE ABOVE EXAMPLE lND THE TOTAL TIME OF PERIODIC MOTION FOR ONE CYCLE

2h Ê s ˆ
[Ans. 2 Á 1+ ]
g Ë r - s ˜¯

Problem 6.93 A cylinder of area of length l, cross section A and density s is pushed into a
liquid of density r through a distance x and released. Find the:
(a) net force acting on the cylinder, and
(b) equilibrium distance x, if s < r.
Solution
(a) The forces acting on the body are Fb and mg.
The net force acting upon it is
Fnet = Fb – mg
where Fb = Vrg; V = volume of liquid displaced.
This gives Fnet = Vrg – mg ...(i) l
mg
When the cylinder is pushed by a distance x into the liquid, x
volume of liquid displaced is
V = Ax
Fb
Substituting V = Ax in Eq. (i), We have
Fnet = (Arx – m)g ...(ii) Fig. 6.163
Substituting m = Als in Eq. (ii), we have
Fnet = (rx – sl)Ag
s
(b) If the body floats (remains in equilibrium) substitute Fnet = 0, to obtain x = l
r
Student Task
s )N THE ABOVE EXAMPLE ANALYSE THE MOTION OF THE CYLINDER WHEN r = s.
[Ans. The speed of the cylinder increases till it completely sinks;
then it moves with constant velocity]
,LJĚƌŽƐƚĂƟĐƐŽƌ&ůƵŝĚ^ƚĂƟĐƐപ1071

For any depth of penetration x (< l),


x s
If = , a = 0 (the body remains in equilibrium)
l r
x s
If > , (the body accelerates up)
l r
x s
If < , (the body accelerates down).
l r

Problem 6.94 A hemispherical iron vessel of mass m, R


radius R and density s is floating in liquid of density r.
Find the:
(a) volume of liquid displaced, and
(b) acceleration of the vessel, when it is made to immerse
completely by pushing it into the liquid.
Solution Fig. 6.164
(a) Since the body floats, its acceleration is zero. It means, Fb
Fnet = Fb – mg = 0
Then, Fb = mg
where m = mass of the body (mass of the liquid V1
displaced) mg
Substituting Fb = Vrg,
m
We have V = Fig. 6.165
r
(b) When the body is totally immersed, the net force
acting on the body is given by V2
Êr ˆ
F = mg Á - 1˜
Ës ¯
as derived earlier.
Ê rˆ
Since s > r, we have a = -g Á1- ˜
Ë s¯
which means a downward acceleration. Fig. 6.166
The volumes of liquid displaced in the cases of floatation and total immersion of the
same object by the application of external force need not be equal.

Student Task
s )N THE ABOVE EXAMPLE
(a) find the greatest mass required to keep inside the body (bowl) so as to float on
water.
1072പWŚLJƐŝĐƐĨŽƌ//dͲ:͗DĞĐŚĂŶŝĐƐ//

(b) what will be the minimum force needed to equilibrate the body when it sinks
completely?
Ê2 ˆ Ê rˆ
Ans. (a) Á p R3 r - m˜ ; (b) mg Á 1 - ˜
Ë3 ¯ Ë s¯

You should not falsely assume that when the density of a body is greater than the density
of surrounding liquid, the body sinks completely.
A
E P0
Problem 6.95 (6TCPUNCVKPINKSWKF) A rectangular closed
trough of length l and height h is completely filled with y
a liquid of density r. It accelerates towards right with an x B
acceleration a. If P0 is the atmospheric pressure at the hole C
h
made at the corner A of the trough, find the pressure at
the point C.
l
Solution Directly using the formula F D

P2 = P1 + rgy,
Fig. 6.167
P0
A
y

x B
C

Fig. 6.168

We have
PA = P0
PB = PA + rgy, where PA = P0 (given)
Then, we have PB = P0 + rgy ...(i)
Similarly, using the formula
P2 = P1 – rax,
We have PB = PC – rgx ...(ii)
Equating PB from Eqs. (i) and (ii), we have
PC = P0 + rax + rgy

Student Task
s &IND THE PRESSURES AT THE POINTS D, E and F.
[Ans. PD = P0 + rgh, PE = P0 – ral, PF = P0 – ral + rgh]
The pressure at any point inside a horizontally accelerating liquid is given as
P = (PC) = P0 + rgx + rgy,
,LJĚƌŽƐƚĂƟĐƐŽƌ&ůƵŝĚ^ƚĂƟĐƐപ1073

where P0 = Atmospheric pressure


rax = Pressure due to the acceleration
rgy = Pressure due to gravity
Since pressure is a scalar quantity, we can add all the above three pressures to get the
total pressure P.
Hence, the pressure at any point or pressure difference between two points in a horizontally
accelerating liquid does not depend on the path we follow. It solely depends on the horizontal
and vertical distance between the points.
Fb
Problem 6.96 What volume of helium is needed to lift a load of 180 kg
(including weight of empty balloon)? Density of air rair = 1.29 kg/m3,
density of helium rhelium = 0.18 kg/m3.
Solution From Newton’s second law, for the equilibrium of system,
Fb = mHeg + mload g mHeg

The buoyant force is equal to weight of displaced air.


rair Vg = (rHeg Vg + 180)g
Solving for V, we find
180 180
V = =
rair - rHe (1.29 - 0.18)
= 162.16 m3
This is the volume required to keep the system just in equilibrium, a mloadg
slightly greater volume is needed. As the density of air decreases with
altitude, volume required is still more. Fig. 6.169

Problem 6.97 A container having a hole at the bottom is free


to move on a horizontal surface. As the liquid comes out, the
container moves in backward direction with an acceleration a and
finally acquires a velocity v (when all the liquid has drained out).
Neglect the mass of container. The correct option out of the
following is
(a) only v depends on h
(b) only a depends on h Fig. 6.170
(c) both v and a depend on h
(d) neither v nor a depends on h
Solution The reaction of outflowing liquid = mass coming out per sec ¥ velocity of container.
\ (Ahr)a = rav ◊ v = ra ◊ 2gh
with a and A as area of hole and container
2ag
\a= and the time taken for the tank to be empty.
A
A 2h
t= ; \ v = at = 2gh
a g
1074പWŚLJƐŝĐƐĨŽƌ//dͲ:͗DĞĐŚĂŶŝĐƐ//

Problem 6.98 A vessel contains two immiscible liquids of density r1


= 100 kg/m3 and r2 = 1500 kg/m3. A solid block of volume V = 10 – 3
m3 and density d = 800 kg/m3 is tied to one end of a string and the r1
other is tied to the bottom of the vessel as shown in Fig. 6.171. The
block is immersed with 2/5th of its volume in the liquid of higher
density and 3¢5th in the liquid of lower density. The entire system is
r2
kept in an elevator which is moving upwards with an acceleration of
a = g/2. Find the tension in the string. (g = 10 m/s2)
Solution We will analyse this problem from the reference frame of
elevator.
Total buoyant force on the block, Fig. 6.171

Ê2 3 ˆ
Fb = Á V r2 + V r1 ˜ ( g + a) 2 3
Ë5 5 ¯ r V (g + a ) r V (g + a )
5 2 5 1
From the condition of equilibrium,
Fb = T + Vd(g + a)
T = Fb – Vd(g + a)
È2 3 ˘
= ( g + a)V Í r2 + r1 - d ˙
Î5 5 ˚ Vda
(Pseudo force) T Vdg
È2 3 ˘
= 15 ¥ 10 - 3 Í ¥ 1500 + ¥ 1000 - 800 ˙
Î5 5 ˚ Fig. 6.172
=6N
Problem 6.99 A wooden rod weighing 25 N is mounted
on a hinge below the free surface of water as shown
in Fig. 6.173. The rod is 3 m long and uniform in cross
section and the support is 1.6 m below the free surface.
m

At what angle a will it come to rest when allowed to


3

drop from a vertical position. The cross section of the 1.6 m


rod is 9.5 ¥ 10– 4 m2 in area. Density of water is 1000 kg/
m3. Assume buoyancy to act at the centre of immersion. a
g = 9.8 m/s2. Also find the reaction on the hinge in this
position.
Fig. 6.173
Solution Let G be the mid-point of AB and E the mid-
point of AC (i.e., the centre of buoyancy)
AC = 1.6 cosec a
Volume of AC = (1.6 ¥ 9.8 ¥ 10– 4)cosec a
Weight of water displaced by AC
= (1.6 ¥ 9.8 ¥ 10– 4 ¥ 103 ¥ 9.8) cosec a
= 14.896 cosec a
Hence, the buoyant force is 14.896 cosec a acting vertically upwards at E. While the weight
of the rod is 25 N acting vertically downwards at G. Taking moments about A,
,LJĚƌŽƐƚĂƟĐƐŽƌ&ůƵŝĚ^ƚĂƟĐƐപ1075

(14.896 cosec a) (AE cos a) = (25)(AG cos a) B

Ê 1.6 cosec a ˆ 3
or (14.896 cosec a) Á ˜ = 25 ¥
Ë 2 ¯ 2 C

or sin2 a = 0.32 G
1.6 m
\ sin a = 0.56 E

or a = 34.3° a

Further, let F be the reaction at hinge in vertically A


downward direction. Then, considering the translatory
Fig. 6.174
equilibrium of rod in vertical direction we have,
F + weight of the rod = upthrust
\ F = upthrust – weight of the rod
= 14.896 cosec (34.3°) – 25
= 26.6 – 25
\ F = 1.6 N (downwards)
Problem 6.100 A spring is attached to the bottom of an empty swimming pool, with the
axis of the spring oriented vertically. An 8.00 kg block of wood (r = 840 kg/m3) is fixed
to the top of the spring and compresses it. Then the pool is filled with water, completely
covering the block. The spring is now observed to be stretched twice as much as it had been
compressed. Determine the percentage of the block’s total volume that is hollow. Ignore any
air in the hollow space.
Solution Initially, kx = mg ...(i) kx F
In the second case,
F = mg + 2kx
F - mg
or kx = ...(ii)
2
From Eqs. (i) and (ii) mg mg + 2kx
(a) (b)
F - mg
mg =
2 Fig. 6.175
or F = 3 mg
Let V be the total volume then,
Vrwg = 3mg
3mg
\ V =
rw g
(3)(8) 3
= m
10 3
= 0.024 m3
1076പWŚLJƐŝĐƐĨŽƌ//dͲ:͗DĞĐŚĂŶŝĐƐ//
Mass
Volume of wood =
Density
8
=
840
= 0.0095
\ Volume of cavity = 0.024 – 0.0095
= 0.0145
\ Pressure volume of cavity
0.0145
= ¥ 100
0.024
= 60.41%
Problem 6.101 A cube of iron 6 cm along each edge is in equilibrium at the interface of
two liquids, water and mercury. Find the length of the cube in each Patm
liquid. The densities of iron and mercury are 7.7 ¥ 10 kg/m and 13.6
3 3

¥ 103 kg/m3 respectively. Water

Solution Let x be the submerged depth in mercury, the (0.6 – x) P1 h


length protrudes in water. If P1 and P2 are the pressures on the upper
and lower faces of the cube, the net upward force = (P2 – P1)A, where l
x
A is the cross-sectional area of cube. In equilibrium it must be equal
P2
to weight of the cube.
W = (P2 – P1)A ...(1)
Mercury
P1 = rw gh + Patm
P2 = rw gh + Patm + rw g (l – x) + rHg gx Fig. 6.176

(P2 – P1) = r g (l - x ) A + rHg gxA ...(2)


w  
weight of displaced weight of displaced
water mercury

fi Archimedes’ principle can be used for two or more than two liquids for any shape of
object.
From Eqs. (1) and (2),
riron Viron g = rwg(l – x) A + rHg gxA
(7.7 ¥ 103)(0.06)3 = (1.0 ¥ 103) (0.06 – x)(0.06)2 + (13.6 ¥ 103)x(0.06)2
which on solving for x yields x = 0.032 m.
Depth submerged in mercury is 32 mm and in water 28 mm.
Problem 6.102 A cylinder of radius R, height H and density s has a hemispherical cut at
its bottom. The top of the cylinder is kept at depth h from the liquid surface. If the density
of liquid is r, find the hydrostatic force acting upon the curved (hemispherical) surface of
the cylinder.
,LJĚƌŽƐƚĂƟĐƐŽƌ&ůƵŝĚ^ƚĂƟĐƐപ1077

Fig. 6.177

Solution Let us assume that the liquid presses the top and bottom of the cylinder with
forces F1 and F2 respectively. Since the net side thrust is zero, the net hydrostatic force, that
is, buoyant force is
F b = F 2 – F 1, F1
where Fb = Vrg
This gives F2 = Vrg + F1
Fb
2
Substituting V = volume of the body = pR2H – p R3
3
and F1 = P1A = (rgh)(pR2)
F2
in the above equation, we have
Ê 2 ˆ
F2 = Á p R2 H - p R3 ˜ r g + p R2 r gh Fig. 6.178
Ë 3 ¯

Ê 2 ˆ
This gives F2 = p R2 r g Á H + h - R˜
Ë 3 ¯

Student Task
s )N THE ABOVE EXAMPLE IF WE HOLD THE BODY HOW MUCH MINIMUM FORCE DO WE HAVE TO
apply?
Ê 2 ˆ
[Ans. F = p R2 Á H - R˜ (s - r ) g ]
Ë 3 ¯

Problem 6.103 A tank, initially at rest, is filled with water to


a height H = 4 m. A small orifice is made at the bottom of the
Water
wall. Find the velocity attained by the tank when it becomes
completely empty. Assume mass of the tank to be negligible. H=4m
Friction is negligible.

Fig. 6.179
1078പWŚLJƐŝĐƐĨŽƌ//dͲ:͗DĞĐŚĂŶŝĐƐ//

Solution Let v be the instantaneous velocity of the tank


A
and c be the instantaneous velocity of the efflux with
respect to the tank.
h Water
Thrust exerted on the tank is
v
F = rac2 a
c
where a is the cross-sectional area of the orifice, c = 2gh
where h is the instantaneous height of water in the tank. Fig. 6.180
Mass of the tank at any time t is, m = rAh
A = cross-sectional area of the tank.
Using Newton’s second law,
dv dv dv
F=m = r Ah \ r Ah = r ac 2 = 2rgah
dt dt dt
dv Ê aˆ
or = 2g Á ˜ ...(1)
dt Ë A¯
In a time dt if the water level falls by dh, then according to the conservation of mass.
dh ac
– rAdh = racdt or =-
dt A
Equation (1) can be written as
dv dh Ê aˆ dv Ê ac ˆ Ê aˆ
= 2g Á ˜
Ë A¯
or ÁË - ˜¯ = 2g ÁË ˜¯
dh dt dh A A

dv 2g 2g 2g
or =- =- =-
dh c 2 gh h
On integrating
v 0
dh
Ú dv = - 2 g Ú h
, v = 2 2gH
0 H

Since H = 4 m, therefore v = 2 2(10)( 4) = 17.9 m/s.


Problem 6.104 A rectangular tank of height 10 m filled with water is placed near the bottom
of a plane inclined at an angle 30° with the horizontal. y
At height h from bottom a small hole is made (as shown
in Fig. 6.181) such that the stream coming out from
hole, strikes the inclined plane normally. Calculate h.

Solution v= 2 g(10 - h) ...(i)


10 m
Component of its velocity parallel to the plane is h
v cos 30°
Let the stream strikes the plane after time t. Then 30°
x
O
0 = v cos 30° – g sin 30° t
Fig. 6.181
,LJĚƌŽƐƚĂƟĐƐŽƌ&ůƵŝĚ^ƚĂƟĐƐപ1079

v cot 30∞
\ t =
g
v 2 cot 30∞
Further x = vt =
g
= 3y
2
v cot 30∞ Ê 1 ˆ
or = 3 Á h - gt 2 ˜
g Ë 2 ¯

3v 2 Ê g v 2 cot 2 30∞ ˆ
\ = 3Áh- ˜
g Ë 2 g2 ¯

v2 3 v2
or = h-
g 2 g
5 v2
\ =h
2 g
or 5(10 – h) = h
\ h = 8.33 m
Problem 6.105 An engine pumps water continuously through a hose. Water leaves the hose
with a velocity v and m is the mass per unit length of the water jet. What is the rate at which
kinetic energy is imparted to water
1
(a) mv3 (b) mv3
2
1 1 2 2
(c) mv2 (d) mv
2 2
2 2
dk d Ê1 ˆ v dM v Ê dM dl ˆ
Solution = Á Mv 2 ˜ = ◊ = Á ¥ ˜
dt dt Ë 2 ¯ 2 dt 2 Ë dl dt ¯
dk 1 dl 1
fi = mv 2 ¥ = mv 3
dt 2 dt 2

Problem 6.106 Mercury is poured into a U-tube in which the cross-sectional area of the
left-hand limb is three times smaller than that of the right one. The level of the mercury in
the narrow limb is a distance l = 30 cm from the upper end of the tube. How much will the
mercury level rise in the right-hand limb if the left one is filled to the top with water?
Solution PQ is original level of mercury. Equating the pressures at the level of M from
left-hand side and right-hand side.
1080പWŚLJƐŝĐƐĨŽƌ//dͲ:͗DĞĐŚĂŶŝĐƐ//

A 3A

30 cm
30 cm

x
P M 3x Q

Fig. 6.182

p0 + rwghw = p0 + rHg gh Hg
or rw hw = rHg hHg
\ (1) (30 + 3x) = (13.6)(4x)
Solving we get, x = 0.58 cm
Problem 6.107 Water is flowing smoothly through a closed pipe system. At one point the
speed of the water is 3.0 m/s, while at another point 1.0 m higher the speed is 4.0 m/s. If the
pressure is 20 kPa at the lower point, what is the pressure at the upper point? What would
the pressure at the upper point be if the water were to stop flowing and the pressure at the
lower point were 18 kPa?
Solution
1 2 1
(i) p1 + rv1 + rgh1 = p2 + rv22 + rgh2
2 2
1 1
(20 ¥ 103) + ¥ 103 ¥ (3)2 + 0 = p2 + ¥ 103 ¥ (4)2 + 103 ¥ 10 ¥ 1
2 2
\ p2 = 6.5 ¥ 103 N/m2 = 6.5 kPa
(ii) Again applying the same equation, we have:
(18 ¥ 103) + 0 + 0 = p2 + 0 + (103)(10)(1)
fi p2 = 8 ¥ 103 N/m2
= 8 kPa
Problem 6.108 A balloon of volume 4000 m3 is filled with helium of density 0.18 kg/m3.
The total mass of the balloon and equipment is 3 tonnes. At an altitude where the density
of air is 1.2 kg/m3, helium completely fills the balloon. Find the maximum mass of the load
that can be lifted by the balloon.
Solution Let m be the mass of load that can be lifted by balloon.
rHe Vg + Mg + mg = rair Vg
\ m = (rair – rHe)V – M
= (1.2 – 0.18) 4000 – 3000 = 1080 kg.
,LJĚƌŽƐƚĂƟĐƐŽƌ&ůƵŝĚ^ƚĂƟĐƐപ1081

Problem 6.109 A tank having a small circular hole contains oil Initial
Air level
on top of water. It is immersed in a large tank of the same oil.
Water flows through the hole. What is the velocity of this flow
Oil 5m
initially? When the flow stops, what would be the position of
the oil-water interface in the tank from the bottom. The specific Oil
gravity of oil is 0.5.
Water 10 m
Solution
(a) Dp = h(rw – r0)g = (10)(1000 – 500) 9.8
= 49000 N/m2
Fig. 6.183
1
Now, DP = rw v 2
2
2DP
\ v =
rw

2 ¥ 49000
=
1000
= 9.8 m/s
(b) The flow will stop when,
(10 + 5)r0g = 5r0g + hrwg
10r0 = hrw

10 ¥ 500
\ h =
1000
=5m
i.e., flow will stop when the water-oil interface is at a height of 5.0 m.
Problem 6.110 A tank with a square base of area 1.0 m2 is divided by a vertical partition
in the middle. The bottom of the partition has a small hinged door of area 20 cm2. The tank
is filled with water and an acid (of relative density 1.7) in the other, both to a height of 4.0
m. Compute the force necessary to keep the door closed.
Solution For compartment containing water, h1 = 4 m, r1 = 103 kg m– 3.
The pressure exerted by water at the door provided at bottom,
P1 = h1r1 g = 4 ¥ 103 ¥ 9.8 = 3.92 ¥ 104 Pa Water Acid

For compartment containing acid, h2 = 4 m, r2 = 1.7 ¥ 10 kg /m


3 3

The pressure exerted by acid at the door provided at bottom,


4m
P2 = h2r2 g = 4 ¥ 1.7 ¥ 103 ¥ 9.8 = 6.664 ¥ 104 Pa.
\ Difference of pressure = P2 – P1 Door
= 6.664 ¥ 104 – 3.92 ¥ 104 = 2.744 ¥ 104 Pa
Fig. 6.184
Given, area of door, A = 20 cm2 = 20 ¥ 10– 4 m2
1082പWŚLJƐŝĐƐĨŽƌ//dͲ:͗DĞĐŚĂŶŝĐƐ//

Force on the door = Difference in pressure ¥ Area


= (P2 – P1) ¥ A = (2.744 ¥ 104) ¥ (20 ¥ 10– 4) = 54.88 N = 55 N
To keep the door closed, the force equal to 55 N should be applied horizontally on the
door from compartment containing water to that containing acid.
Problem 6.111 A horizontally oriented bent thin tube ABC of c

length 1.4 m rotates with a constant angular velocity w about a w
stationary vertical axis OO¢ passing through the end A as shown 0.4 m
in Fig. 6.185. The portion BC of tube is parallel to OO¢ and its
length is equal 0.4 m. At t = 0, AB part of the tube is filled with A B
an ideal fluid. The end A of tube is open, the closed end C has l1 = 1 m
O
a very small orifice. Find the maximum height reached by fluid
relative to end C. (take g = 10/s2 and w = 10 rad/s) Fig. 6.185

Solution Let any time t fluid just fills the portion BC and comes out with velocity v.
l = 1 m, x = 0.4 m
In rotating frame, centrifugal force on portion A¢B,
h
l 2 2 2 C
r Aw (l - x )
F = r AÚ w 2 r ◊ dr =
x
2 x

Due to pressure difference between C and A¢ B


A –x
1 x A¢
DP = rw2 (l2 – x2 – rgx)
2 Fig. 6.186
1 rw 2 2
Now, rV 2 = DP = (l - x 2 ) - r gx ...(1)
2 2
From Bernoulli’s theorem,
Let it reach up to a height h, then
1 2 rw 2 (l 2 - x 2 )
rgh = rv = - r gx
2 2
(10)2 2
r ¥ 10 ¥ h = r ¥ (1 - (0.4)2 ) - 0.4 ¥ 10 ¥ r fi h = 3.8 m.
2

Problem 6.112 A long cylindrical tank of cross-sectional


area 0.5 m2 is filled with water. It has an opening at 20 kg
y
a height 50 cm from the bottom, having area of cross
section 1 ¥ 10– 4 m2. A movable piston of cross-sectional
x
area almost equal to 0.5 m2 is fitted on the top of the tank
such that it can slide in the tank freely. A load of 20 kg is
applied on the top of the water by piston, as shown in Fig.
6.187. Find the speed of the water jet with which it hits
the surface when piston is 1 m above the bottom. (ignore
the mass of the piston)
Fig. 6.187
,LJĚƌŽƐƚĂƟĐƐŽƌ&ůƵŝĚ^ƚĂƟĐƐപ1083

Solution With respect to the opening the height of the piston is 0.5 m.
20 ¥ 10 N
Pressure at the top is P1 = P0 +
0.5 m 2
where P0 is the atmospheric pressure.
Pressure at the opening is P2 = P0. From Bernoulli’s equation, we have
1 2 1
P1 + rgh + rv1 = P2 + rv22
2 2
On solving, we get v2 = 3.3 m/s = vx (say)
vy = 2gh

v = vx2 + vy2 = 4.56 m/s

Problem 6.113 A solid semi-cylinder of uniform density r1


rests in equilibrium on a rough inclined plane with a liquid
of density r on its right as shown in Fig. 6.188(a). Determine
the minimum coefficient of friction to ensure equilibrium; and
(b) the ratio r1/r2. Assume that flat surface of the cylinder is
parallel to the inclined plane.
Solution The hydrostatic pressure force acts radially inwards, 45°
towards the axis of the cylinder that passes through C and is
normal to plane of paper. The torque due to this force about C Fig. 6.188
is zero. Secondly, the submerged portion is symmetrical about
the line CA. Therefore,
Fx = 0

C
Fp
45°
C y
A N
f G
x
45° mg
(a) (b)

Fig. 6.189

From the diagram of pressure force it can be deduced that:


(i) the torque of pressure force about C to zero; and
(ii) the resultant pressure force acts vertically (by symmetry), i.e., Fx = 0.
Calculation of y-component of the pressure force,

Fy = Ú PdA sin q
1084പWŚLJƐŝĐƐĨŽƌ//dͲ:͗DĞĐŚĂŶŝĐƐ//

3p/4 È Ê p ˆ˘
= Úp/4 Í r gR ÁË sin q - sin 4 ˜¯ ˙(LR dq )sin q
Î ˚
3p/4 Ê 1 ˆ
= r gR2 LÚ 2
ÁË sin q - sin q ˜ dq
p/4 2 ¯
Ê p 1ˆ
= r gR2 L Á - ˜
Ë 4 2¯
Balancing force in x-direction.
f N
=
2 2
or f =N ...(1)
f
ms ≥
N
Hence, the minimum coefficient of static friction = 1.
1
In y-direction, Fp + ( f + N ) = mg
2
Ê p 1ˆ 2
ÁË - ˜¯ LR r g + 2 f = mg [using (1)] ...(2)
4 2
Balancing torque about C,
4 p Ê 4ˆ
fR = mg R sin ÁË∵GC = ˜
3p 4 3p ¯
4 mg
f =
3 2p
Putting in (2),
Ê p 1ˆ 2 4 mg
ÁË - ˜¯ LR r g + = mg
4 2 3p
m 3p (p - 2)
or =
2
LR r 4(3p - 4)
The required ratio,
r1 2m 3(p - 2)
= =
r2 Lrp R 2
4(3p - 4)

Problem 6.114 The ancient water clock clepsydra shown in y


Fig. 6.190 has such a shape that the water level descended at a Water x(y)
constant rate at all times. If the water level falls by 4 cm every hour,
determine the shape of the jar, i.e., specify x as a function of y. The x
Drain hole
drain hole diameter is 2 mm and can be assumed to be very small
compared to x.
Solution Rate of change of volume in the jar
= Rate of water flowing out through drain hole
Fig. 6.190
,LJĚƌŽƐƚĂƟĐƐŽƌ&ůƵŝĚ^ƚĂƟĐƐപ1085

dy
px2 = (vA)drain hole ...(1)
dt
Now we use Bernoulli’s equation to get velocity of water at the hole.
1 2
rgy = rv
2
or v = 2gy
and A = p(1 ¥ 10–3)2
Putting the values in Eq. (1), we get

4 ¥ 10 - 2
p x2 = 2 g y ◊ p(1 ¥ 10 - 3 )2
60 ¥ 60
fi x2 = 0.3984 y

or x = 0.631y1/4
Problem 6.115 A large tank filled with water to a height h is to be emptied through a small
h
hole at the bottom. The ratio of time taken for the level of water to fall from h to and from
2
h
to zero is
2
1
(a) 2 (b)
2
1
(c) 2 -1 (d)
2 -1
Solution Time taken for the level to fall from H to H¢
A 2
t= [ H - H¢]
A0 g
h
According to the problem, the time taken for the level to fall from h to t1 =
2
A 2È h˘ h
Í h- ˙ and similarly time taken for the level to fall from to zero
A0 gÎ 2˚ 2

A 2È h ˘
t2 = Í - 0˙
A0 gÎ 2 ˚
1
1-
t1 2 = 2 -1
\ =
t2 1
-0
2
1086പWŚLJƐŝĐƐĨŽƌ//dͲ:͗DĞĐŚĂŶŝĐƐ//

Problem 6.116 Consider a horizontally oriented syringe


containing water located at a height of 1.25 m above the D = 8 mm
d = 2 mm
ground. The diameter of the plunger is 8 mm and the
diameter of the nozzle is 2 mm. The plunger is pushed with
a constant speed of 0.25 m/s. Find the horizontal range of 1.25
water stream on the ground. (Take g = 10 m/s2)
++6,''
Solution From equation of continuity, Ground R¢

A1 Ê p r12 ˆ Fig. 6.191


A 1v 1 = A 2v 2 fi v1 = Á p r 2 ˜ v1
A2 Ë 2¯
2
Ê Dˆ Ê 8 ¥ 10 - 3 ˆ
2
or v2 = Á ˜ v1 = Á -3 ˜
¥ 0.25 m/s = 4 m/s (horizontal)
Ë d¯ Ë 2 ¥ 10 ¯

Vertical component of the velocity is zero.


1 2 2H
Now, H= gt fi t =
2 g
Range is given by R = v2t = v2

2H 2 ¥ 1.25
= 4¥ =2m
g 10

Problem 6.117 A cylindrical vessel has an opening of cross-sectional area a near the bottom.
A disc is held against the opening to prevent liquid of density r from coming out.
(a) What is the force on the disc?
(b) Now the disc is moved a short distance away. The liquid gushes out, strikes the disc
inelastically and drops vertically downward. What is the force exerted on the disc?
Solution
(a) The disc experiences a hydrostatic pressure from inside
P1 = Patm + rgh h

The air pressure on the outside of disc,


P2 = Patm
Fig. 6.192
The net outward force = (P1 – P2)a
= rgha
(b) When the disc is moved away, the liquid gushes out with speed v = 2gh , according
to Torricelli’s theorem.
The mass flux (mass per second) = rav = ra 2 gh
Momentum flux (momentum per second) rightward = (rav)v
= rav2 = 2rgha
,LJĚƌŽƐƚĂƟĐƐŽƌ&ůƵŝĚ^ƚĂƟĐƐപ1087

The change in momentum after striking the disc, +ve


pi = rgha
  
Dp = p f - pi
pf = 0
= 0 – (+2rgha)
= – 2rgha
Taking rightward direction positive, therefore force on the liquid is
towards left and its reaction with the disc is towards right. The force
obtained is twice the hydrostatic force. The force due to atmospheric
pressure is cancelled out on both sides. Fig. 6.193
Problem 6.118 A garden hose delivers water at the speed of 18 m/s, and rate of 0.1 m3/s.
If a flat plate is moved normally towards the discharging jet of water at a speed of 3 m/s.
Find the force on the plate due to the jet. Compare the force if the plate were stationary.
Solution We assume that plate stops forward flow of water, water does not splash back.
Then the force on the plate is equal to rate of change of momentum along the direction of
water jet. Hence, ++6,''
F = (rav)v = rav2
= 1000 ¥ 0.1 ¥ 18 N
= 1800 N
Here av is flow rate, 0.1 m3/s.
When plate is moving towards the stream at 3 m/s the change in velocity of water jet is
(18 + 3) m/s, secondly the mass of water hitting the plate per second is ra(v + 3). As area
of cross section of stream is a = (0.1/v) m2, we have
Ê 0.1ˆ
F = r Á ˜ (v + 3)(v + 3)
Ë v ¯
1000 ¥ 0.1 ¥ (21)2
= N
18
= 2450 N
Problem 6.119 A nonviscous liquid of constant density 1000 kg/m3 flows in a streamline
motion along a tube of variable cross section. The tube is kept inclined in the vertical plane as
shown in Fig. 6.194. The area of cross section of the tube at two points P and Q at heights of
2 metres and 5 metres are respectively 4 ¥ 10– 3 m2 and 8 ¥ 10– 3 m2. The velocity of the liquid
at point P is 1 m/s. Find the work done per unit volume by the pressure and the gravity
forces as the liquid flows from point P to Q. ++6,''

P
5m

2m

Fig. 6.194
1088പWŚLJƐŝĐƐĨŽƌ//dͲ:͗DĞĐŚĂŶŝĐƐ//

Solution The work done per unit volume by the force of gravity, Wg/vol. = r g Dh,
= 103 ¥ 9.8 ¥ 3 = 2.94 ¥ 104 J/m3
From the equation of continuity,
A1v1 = A2v2 or v2 = A1v1/A2

4 ¥ 10 - 3 ¥ 1
\ v2 = = 0.5 m/s
8 ¥ 10 - 3
Now, increment of kinetic energy of fluid per unit volume
1 Ê1 ˆ È1 ˘
= r(v22 - v12 ) = Á ¥ 10 3 ˜ Í - 1˙
2 Ë2 ¯ Î4 ˚
= – 3.75 ¥ 102 J/m3
We know that net energy of the fluid per unit volume, in a streamline flow remains the
same and hence the net work done by pressure per unit volume of the fluid is given by
Wp/vol = (2.94 ¥ 104 – 3.75 ¥ 102)
= 29025 J/m3
Problem 6.120 A side wall of a wide open tank is provided
with a narrowing tube through which water flows out. The
cross-sectional area of the tube decreases from S = 3.0 cm2
to s = 1.0 cm2. The water level in the tank is h = 4.6 m
higher than in the tube. Neglecting the viscosity of water, h2
find the horizontal component of the force tending to pull S s
the tube out of the tank.
Solution Let P and r be the pressures at cross section S h1
and s respectively. Then from the Bernoulli’s theorem
1 1 Fig. 6.195
P+ rV 2 + r g h2 = r + r v 2 - r g h1
2 2
Hence, the pressure difference between S and s is
r 2
P–r = (v – V2) + rgh ...(i)
2
where h = |h1 – h2|is the head loss causing liquid flow.
Now, consider water coming out of area s in unit time. The rate of increase in momentum
of water is
rvs (v – V)
This must be equal to the force on water = (P – r)v. Therefore,
P – r = rv (v – V) ...(ii)
From Eqs. (i) and (ii)
r 2
(v – V2) + hrg = rv(v – V)
2
,LJĚƌŽƐƚĂƟĐƐŽƌ&ůƵŝĚ^ƚĂƟĐƐപ1089

1 1
or h = [+ v2 – 2vV + V2] = (v – V)2 ...(iii)
2g 2g
But from the equation of continuity
V s
SV = sv fi =
v S
2 2
1 2 Ê V ˆ v2 È s ˘
\ h = v Á1 - ˜ 1- ...(iv)
2g Ë v ¯ 2 g ÍÎ S ˙˚
2
v2 Ê sˆ 2 v2
= r gS◊ ÁË 1 - ˜¯ = r g(s - S) ◊
2g S 2gS

(S - s)2
= rgh as v= 2gh
S
=6N (Putting values)
Problem 6.121
(a) Consider a stream of fluid of density r with speed
v1 passing abruptly from cylindrical pipe of cross-
sectional area a1 into a wider cylindrical pipe of v1 P1 v2
P2
cross-sectional area a2 (see Fig. 6.196). The jet will a1
mix with the surrounding fluid and, after the
mixing, will flow on almost uniformly with an a2
average speed v2. Without referring to the details Fig. 6.196
of the mixing, use momentum ideas to show that
the increase in pressure due to the mixing is approximately
P2 – P1 = r v2 (v1 – v2)
(b) Show from Bernoulli’s principle that in a gradually widening pipe we would get
1
P2 – P1 = r(v12 – v22)
2
1
and explain the loss of pressure [the difference is ◊ r(v1 – v2)2] due to the abrupt
2
enlargement of the pipe. Can you draw an analogy with elastic and inelastic collisions
in particle mechanics?
Solution
(a) At the beginning of the large pipe eddy motions are set up which cause a loss of
energy. At inlet and outlet the flow is steady. Applying Bernoulli’s equation to section
a1 and a2
1 1
P1 + rv12 = P2 + rv22 + Q
2 2
where Q represents the energy loss.
1090പWŚLJƐŝĐƐĨŽƌ//dͲ:͗DĞĐŚĂŶŝĐƐ//

Further v 1a 1 = v 2a 2
Momentum of the quantity of water which passes a1 in unit time = mass flux ¥ velocity
= (a1v1r)v1 = a1v12 r
For section a2 = (a2v2r)v1 = a1v1v2 r
Change in momentum in unit time
= a1 r v1 (v1 – v2)
Net force = P2 a2 – P1a1 – P1(a2 – a1)
\ P2a2 – P1a1 – P1(a2 – a1) = a1 rv1 (v1 – v2)
a2 (P2 – P1) = ra2 v2 (v1 – v2)
P2 – P 1 = rv2 (v1 – v2) ...(1)
(b) In case of steady flow
1 2 1
r v1 = P2 + r v22
P1 +
2 2
1
P2 – P1 = r (v12 - v22 ) ...(2)
2
The difference of the two results is
1 1
r v2 (v1 – v2) – r (v12 – v22)2 = r(v1 - v2 )2
2 2
This quantity represents the internal energy arising from the eddy motion. This is
analogous to inelastic collision.
Problem 6.122 A cylindrical tank having cross-sectional
area A = 0.5 m2 is filled with two liquids of densities r1 =
900 kg/m3 and r2 = 600 kg/m3 to height h = 60 cm each as
r2
shown in Fig. 6.197. h
A small hole having area a = 5 cm2 is made in right vertical
wall at a height y = 20 cm from the bottom. Calculate:
(a) velocity of efflux, r1
(b) horizontal force F to keep the cylinder in static equilib- h A B
rium, if it is placed on a smooth horizontal plane, and F y
(c) minimum and maximum values of F to keep the cyl-
inder in static equilibrium, if the coefficient of friction
Fig. 6.197
between the cylinder and the plane is m = 0.01. g = 10
m/s2.
Solution
(a) Let v be the velocity of efflux.
Applying Bernoulli’s theorem at points A and B in the same horizontal line, we have
1
P0 + h r2 g + (h – y) r1 g = P0 + r1 v 2
2
1
or h r2 g + (h – y)r1 g = r1 v 2
2
,LJĚƌŽƐƚĂƟĐƐŽƌ&ůƵŝĚ^ƚĂƟĐƐപ1091

0.60 ¥ 600 ¥ 10 + (0.60 – 0.20) ¥ 900 ¥ 10


1
= ¥ 900 ¥ v2
2
1
3600 + 3600 = ¥ 900 ¥ v 2
2
2 ¥ 7200
v2 = = 16
900
v = 4 m/s
(b) The horizontal force F required to keep the cylinder stationary is equal to the horizontal
thrust exerted by water jet. So
F = mass flowing per second ¥ change of velocity
= avr ¥ (v – 0) = arv2
= (5 ¥ 10– 4) ¥ 900 ¥ 16 = 7.2 N
mg
(c) The different forces are shown in Fig. 6.200.
m = Ahr1 + Ahr2 = Ah(r1 + r2)
= 0.5 ¥ 0.60 ¥ 1500 r2
= 450 kg
Limiting friction = mN = mmg
= 0.01 ¥ 450 ¥ 10 = 45 N r1
7.2 N
So, limiting friction > F. Hence, minimum force re- Fmax
quired is zero.
The maximum value of the force can be calculated mN
with the help of Fig. 6.198. N

Fmax = F + mN = 7.2 N + 45 N
= 52.2 N Fig. 6.198
Problem 6.123 A container of large uniform cross-sectional
area A resting on a horizontal surface holds two immiscible,
non-viscous and incompressible liquids of densities d and 2 d
H/2
d, each of height H/2 as shown Fig. 6.199. The lower density
liquid is open to the atmosphere having pressure P0. A tiny s
hole of area s (s << A) is punched on the vertical side of the
H/2 2d
container at a height h (h < H/2). Determine: h

(i) the initial speed of efflux of the liquid at the hole; x


(ii) the horizontal distance x travelled by the liquid
initially; and Fig. 6.199
(iii) the height hm at which the hole should be punched so
that the liquid travels the maximum distance xm initially. Also calculate xm.
(Neglect the air resistance in these calculations.) ++6,''
1092പWŚLJƐŝĐƐĨŽƌ//dͲ:͗DĞĐŚĂŶŝĐƐ//

Solution
(i) Here the effective height of orifice cannot be taken as (H – h) due to the presence of
two liquids. Using Bernoulli’s theorem, just inside and just outside the orifice, we have
H ÊH ˆ 1
P0 +dg + Á - h˜ (2d) g = P0 + (2d)v2
2 Ë2 ¯ 2
where v is the velocity of efflux
H H 1 2
\ dg + (2 d) g - h(2 d) g = (2 d)v
2 2 2
3
or Hg – 2 hg = v2
2
Èg ˘
or v = Í ( 3 H - 4 h )˙ ...(1)
Î2 ˚
(ii) Let t be the time taken by the liquid to hit the ground. Then
1 2 Ê 2h ˆ
h=g t or t = ÁË g ˜¯
2
The horizontal range x = v ¥ t

Èg ˘ Ê 2h ˆ
\ x= Í 2 (3 H - 4 h ˙ ¥ ÁË g ˜¯
Î ˚
= [(3 H - 4 h)h] ...(2)
(iii) To find out the maximum height hm, we differentiate x with respect to h and equate it
to zero for maximum h. Hence
dx d
= [(3 H - 4 h)h] = 0
dh dh
\ 3H – 8h = 0
3
or h= H ...(3)
8
ÈÊ 4 ¥ 3 ˆ 3 ˘ 3H
\ xm = ÍÁË 3 H - 8 H ˜¯ 8 H ˙ = 4 ...(4)
Î ˚

Problem 6.124 A cylinder tank of height 0.4 m is open at the top and has a diameter 0.16
m. Water is filled in it up to a height of 0.16 metre. Calculate how long will it take to empty
the tank through a hole of radius 5 ¥ 10– 3 m in its bottom. (IIT Roorkee, 1990)
Solution See Fig. 6.200.
Let rate of drop of water is – dh/dt, then
dh
– pR2 = p r 2 v = p r 2 ◊ ( 2 g h)
dt
[∵v = (2 g h)]
,LJĚƌŽƐƚĂƟĐƐŽƌ&ůƵŝĚ^ƚĂƟĐƐപ1093

dh (0.005)2
- = ¥ (2 ¥ 9.8 h)1/2 R
dt (0.08)2
dh
or - = 0.0173 dt
h1/2
Integrating this expression, we get
0 dh
-Ú = 0.0173 t h
0.16 h1/2
or – 2[h1/2] 00.16 = 0.0173 t
r
2[0.16]1/2 = 0.0173 t or 0.8 = 0.0173 t
0.8 Fig. 6.200
\ t = = 46.265 seconds
0.0173

Problem 6.125 Water flows through a horizontal tube as


shown in Fig. 6.201. If the difference of heights of water
column in the vertical tube is 2 cm, and the areas of cross
section at A and B are 4 cm2 and 2 cm2 respectively, find
the rate of flow of water across any section. B
A
Solution The rate of flow in the horizontal venturimeter
is given by Fig. 6.201

A1 A2 2 gh
Q =
A12 - A22
Here, h = 2 cm
A1 = 4 cm2
A2 = 2 cm2
\ Q = 146 cm3/s
Problem 6.126 A horizontally oriented tube AB of length l rotates with a constant angular
velocity w about a stationary vertical axis OO¢ passing through the end A (Fig. 6.202). The
tube is filled with an ideal fluid. The end A of the tube is open, the closed end B has a very
small orifice. Find the velocity of the fluid relative to the tube as a function of the column
height h. (IIT Roorkee, 1977)
O O

w A B A dx B
w

h x
O¢ l O¢ l
(a) (b)

Fig. 6.202
1094പWŚLJƐŝĐƐĨŽƌ//dͲ:͗DĞĐŚĂŶŝĐƐ//

Solution Consider an element dx at a distance x from the axis OO¢. Here, the water will be
thrown out due to the centrifugal force acting on water.
Ê dv ˆ Ê dv ˆ
Force acting on water = dm Á v ˜ Á∵ a = v ˜
Ë dx ¯ Ë dx ¯
Centrifugal force = dmw2 x
where dm is the mass of water in element dx.
Ê dv ˆ
\ dmw2 x = dm Á v ˜
Ë dx ¯
or w2x dx = vdv
Integrating this expression, we get
l v
w2Ú x dx = Ú0 v dv
(l - h)
l v
È x2 ˘
2 È v2 ˘
w Í ˙ =Í ˙
Î 2 ˚( l - h ) Î 2 ˚ 0
w2 2 v2
[l - (l - h)2 ] =
2 2

or v = w [l 2 - (l - h)2 ]

= w [l 2 - l 2 - h 2 + 2 l h]

Ê 2l ˆ
= w h Á - 1˜
Ë h ¯

Problem 6.127 Two very large open tanks, A and F both contain the same liquid. A
horizontal pipe BCD, having a constriction at C, leads out of the bottom of tank A, and a
vertical pipe E open into the constriction at C and dips into the liquid in tank F. Assume
streamline flow and no viscosity. If the cross section at C is one half that at D, and if D is at
distance h1 below the level of the liquid in A, to what height h2 will liquid rise in pipe E?
Express your answer in terms of h1. Neglect changes in atmospheric pressure with elevation.

h1
A B C D

h2 E

Fig. 6.203
,LJĚƌŽƐƚĂƟĐƐŽƌ&ůƵŝĚ^ƚĂƟĐƐപ1095

Solution Using Bernoulli’s equation between top of tank A and B, we get

vD = 2 gh1
Now by equation of continuity
ACv C = A Dv D
AD
\ vC = vD
AC

= 2 2 gh1

Now using Bernoulli’s equations between C and free surface of liquid in A, we have
1
Pa + 0 + rgh1 = Pc + rv22 + 0 ...(i)
2
From tank F, Pc + rgh2 = Pa
\ Pc = Pa – rgh2 ...(ii)
From Eqs. (i) and (ii), we have
v22
h2 = - h1
2g

(2 2 gh1 )2
= - h1
2g
= 3h1
Problem 6.128 Water flows steadily from a reservoir as shown 1
in Fig. 6.204. The elevation of point 1 is 10 m; of points 2 and 3
it is 1 m. The cross section at point 2 is 0.04 m2 and at point 3 it
is 0.02 m2. The area of the reservoir is very large compared with
the cross sections of the pipe.
(a) Compute the gauge pressure at point 2. 2 3
(b) Compute the discharge rate in cubic metre per second.
Fig. 6.204
Solution
(a) Using Bernoulli’s theorem between points 1 and 3, we have
1 1
P1 + rv12 + r gh1 = P3 + rv32 + r gh3
2 2
Here P1 = P3
= Pa
v1 << v2 and h1 = 10 m,
h3 = 1m
1
\ rg ¥ 9 = r ¥ v32
2
\ v3 = 2g ¥ 9
= 13.3 m/s
1096പWŚLJƐŝĐƐĨŽƌ//dͲ:͗DĞĐŚĂŶŝĐƐ//

Now using equation of continuity between sections 2 and 3, we have


A2v 2 = A 3v 3
0.02 ¥ 19.8
\ v2 =
0.04
= 6.64 m/s
Again using Bernoulli’s equation between 2 and 3, we have
1 1
P2 + rv22 = P3 + rv32
2 2
As P3 = P a
1
\ P2 – Pa = r(v32 – v22)
2
1
= ¥ 1000(13.302 – 6.642)
2
= 6.62 ¥ 104 N/m2
(b) The rate of flow Q = A 3v 3
= 0.02 ¥ 13.3
= 0.266 m3/s
Problem 6.129 W-tube system as shown is rotated about an B A
axis AA at 10 rpm. Determine the levels in columns C1, C2 and
C3 in the new position of equilibrium. Determine also the levels
in the tube if the axis of rotation was BB instead of AA. Assume
the tubes to be long enough not to allow spillage. C1 C2 C3
Solution The rise in the tubes c1 or c3 rises with respect to 30 cm
c2, is
w2r 2 50 cm
25 cm
50 cm
y =
2g B¢ A¢
2
È Ê 10 ˆ ˘ 2 Fig. 6.205
Í2p ÁË 60 ˜¯ ˙ ¥ 0.5
= Î ˚
2 ¥ 9.8
= 1.4 ¥ 10 – 2 m y
If h1 and h2 be the heights of the liquid in tubes, then h1 h1
h2
h1 – h2 = 1.4 ...(i)
and 2h1 + h2 = 30 + 30 + 30
Fig. 6.206
= 90 ...(ii)
After solving the above equations, we get
h1 = 30.47 cm
and h2 = 29.06 cm
Do the other part similarly.
,LJĚƌŽƐƚĂƟĐƐŽƌ&ůƵŝĚ^ƚĂƟĐƐപ1097

Problem 6.130 A vessel with a hole in its bottom is fastened l


on a railroad car of the vessel and the car is M and cross-
b
sectional area A. What force F is required to pull the car so
that a maximum amount of water remains in the vessel? The c
dimensions of the vessel are shown in Fig. 6.207. F

Solution For maximum amount of water in the vessel, the


situation is shown in Fig. 6.208. Fig. 6.207
If q is the angle made by free surface from the horizontal,
then
a
tan q =
g
b
From the geometry, tan q = b
c
a b
\ =
g c q
bg c
fi a =
c
Fig. 6.208
The total mass of the system
È 1 A ˘
= Í M + bc r ˙
Î 2 l ˚
È 1 Ar ˘
Thus, force needed F = Í M + bc a
Î 2 l ˙˚
È bcAr ˘ bg
= ÍM + ˙
Î 2l ˚ c

w
Problem 6.131 A small tank practically filled with liquid is z
mounted at the end of a long rotating arm of radius R which
rotates steadily in the horizontal plane at an angular velocity
w. The remaining space in the tank is at uniform pressure. q
The free surface of liquid, which may be taken as a plane,
from an angle q with the horizontal. Show that q is given by
tan q = (w2R/g).
Solution If a is the centripetal acceleration, then R

a x = w 2R Fig. 6.209
If a is the inclination, then
ax
tan a =
g

w 2R a ax
or tan(180° – q) =
g
Fig. 6.210
1098പWŚLJƐŝĐƐĨŽƌ//dͲ:͗DĞĐŚĂŶŝĐƐ//

w 2R
or – tan q =
g
w 2R
\ tan q =
g

Problem 6.132 A Pitot tube is used to determine


the air speed of an airplane. It consists of an outer rair
B
tube with a number of small holes B (four are Air
shown) that allow air into the tube; that tube is HoleA
B
connected to one arm of the U-tube. The other arm
of the U-tube is connected to hole A at the front
end of the device, which points in the direction h
the plane is headed. At A the air becomes stagnant Liquid
so that vA = 0. At B, however the speed of the air
presumely equals the air speed, v of the aircraft.
Fig. 6.211
2r gh
(a) Use Bernoulli’s equation to show that v = . where r is the density of the liquid
rair
in the U-tube and h is the difference in the fluid levels in that tube.
(b) Suppose that the tube contains alcohol and indicates a level difference h of 26.0 cm.
What is the plane’s speed relative to the air? The density of the air is 1.03 kg/m3 and
that of alcohol is 810 kg/m3.
Solution
(a) By using Bernoulli’s equation between A and B, we have
1 1
PA + rair vA2 = PB + rair vB2
2 2
As vA = 0.
1
\ rair vB2 = (PA – PB) = rgh
2 2r gh
\ vB = v =
rair
2 ¥ 810 ¥ 9.8 ¥ 0.26
(b) v =
1.03
= 63.3 m/s
Problem 6.133 The horizontal bottom of a wide vessel with an ideal
fluid has a round orifice of radius R1 over which a round closed cylinder
R2
is mounted, whose radius R2 > R1. The clearance between the cylinder
and the bottom of the vessel is very small, the fluid density is r. Find the
h
static pressure of the fluid in the clearance as a function of the distance
r from the axis of the orifice (and the cylinder), if the height of the fluid
is equal to h.
Solution Using equation of continuity between sections A and B, we R1
have Fig. 6.212
,LJĚƌŽƐƚĂƟĐƐŽƌ&ůƵŝĚ^ƚĂƟĐƐപ1099

(2prd) vB = (2pR1d) vA C

or r vB = R 1v A ...(i)
Now using Bernoulli’s equation between A and C and between
A and B, we have
1
Pa + rvA2 = Pa + rgh ...(ii)
2 r
1 1 vB ¨ B
and Pa + rvA2 = P + rvB2 ...(iii)
2 2
vA
After simplifying, we get
Ê R2 ˆ Fig. 6.213
P = P0 + r gh Á 1 - 21 ˜
Ë r ¯

Problem 6.134 A metal rod of length 80 cm and mass 1.6 kg has a


uniform cross-sectional area of 6.0 cm2. Due to a nonuniform density,
the centre of mass of the rod is 20 cm from one end of the rod. The
rod is suspended in a horizontal position in water by ropes attached
to both ends.
(a) What is the tension in the rope closer to the centre of mass?
(b) What is the tension in the rope further from the centre of mass? Fig. 6.214
(*KPV The buoyant force on the rod effectively acts at the rod’s centre)
Solution Buoyant force on the rod Fb = Vrg T1 Fb T2
= (6 ¥ 80 ¥ 10– 6) ¥ 1000 ¥ 9.8 0.4 m
= 4.7 N 0.2 m 0.6 m
We have
1.6 g
T 1 + T2 + Fb = W
\ T1 + T2 = W – Fb = 1.6g – 4.7 = 10.98 N. Fig. 6.215

Taking moment of all the forces acting on the rod about the left end and put equals to zero,
– (1.6 ¥ 9.8) ¥ 0.2 + 4.7 ¥ 0.4 + T2 ¥ 0.8 = 0
\ T2 = 1.6 N
and T1 = 9.4 N

Problem 6.135 A broad pipe having a radius 10 cm


branches into two pipes of radii 5 cm and 3 cm. If the
velocity of flowing water in the pipe of radius 3 cm is 5

h
cm/s, determine the velocities of water in the remaining
two pipes. Given that the rate of discharge through the
main branch is 600p cm3/s.
Solution Consider any three sections (1), (2) and (3) in
the three pipes of different radii as shown in Fig. 6.216.
If v1 and v2 be the velocities of water at sections (1) and Fig. 6.216
(2), respectively; then
1100പWŚLJƐŝĐƐĨŽƌ//dͲ:͗DĞĐŚĂŶŝĐƐ//

a1 = p (10)2 cm2, a2 = p (5)2 cm2


and a3 = p (32) cm2
v1 = ?, v2 = ?, v3 = 5 cm/s
The rates of discharge through the three pipes are
a1v1 = 100pv1 cm3/s
a2v2 = 25pv2 cm3/s
a3v3 = 9p ¥ 5 = 45p cm3/s
Now, Q = a1v1 = a2v2 + a3v3 fi 600p = 100pv1 = 25pv2 + 45p
Solving we get v1 = 6 cm/s and v2 = 22.2 cm/s
Problem 6.136 The flow of blood in a large artery of an anesthetized dog is diverted
through a venturimeter. The wider part of the meter has a cross-sectional area equal to that
of the artery, A = 8 mm2. The narrower part has an area a = 4 mm2. The pressure drop in the
artery is 24 Pa. What is the speed of the blood in the artery?
Solution If v1 is the speed of the blood in artery, then
1 1
P1 + rv12 = P2 + rv22 ...(i)
2 2
Also, A 1v 1 = A 2v 2
or 8v1 = 4 ¥ v2
\ v2 = 2 v1
Now from Eq. (i),
1 1
P1 + rv12 = P2 + rv22
2 2
3 2
or rv1 = P1 – P2 PBlood = 1.06 ¥ 103 kg m–3
2
2(P1 - P2 )
\ v1 =
3r

2 ¥ 24
=
3 ¥ 1000
= 0.125 m/s
Problem 6.137 Figure 6.217 shows a siphon in action. The B C

liquid flowing through the siphon has a density of 1.5 g/cc. h1 = 20 cm


Calculate the pressure difference between points: A
h2 = 200 cm
(a) (i) A and D (ii) B and C, neglecting density of air.
(b) (i) A and D (ii) B and C, taking into account the density
D
of air = 1.3 kg/m3.

Fig. 6.217
,LJĚƌŽƐƚĂƟĐƐŽƌ&ůƵŝĚ^ƚĂƟĐƐപ1101

Solution
(a) (i) As both the points are exposed to atmosphere, so
PA = PD
= Pa
and hence, PA – PD = 0
(ii) PB – PC = (PA – h1r g) – (PD – h2 rg)
As PA = PD
= Pa
\ PB – PC = (h2 – h1)rg
= (2.0 – 0.2) ¥ 1.5 ¥ 103 ¥ 9.8
= 2.646 ¥ 104 N/m2
(b) (i) PD – PA = (h2 – h1) rair g
= (2.0 ¥ 0.20) ¥ 1.3 ¥ 9.8
= 22.93 N/m2
(ii) PB – PC = (PA – PD) + (h2 – h1)rg
= – (h2 – h1) rair g + (h2 – h1)rg
= – 22.93 + 2.646 ¥ 104 N/m2
= 2.644 ¥ 104 N/m2.
Problem 6.138 The side wall of a wide vertical cylindrical vessel of height h = 75 cm has
a narrow vertical slit running all the way down to the bottom of the vessel = The length of
the slit is l = 50 cm and the width b = 1.0 mm. With the slit closed, the vessel is filled with
water. Find the resultant force of reaction of water flowing out the vessel immediately after
the slit is opened.
Solution Consider an element of length dx of the slit as shown in Fig. 6.218.
Area of the slit = bdx
Discharge per second through this area = r(bdx)v
Force of reaction due to element dx
dF = – r(bdx)v2 …(1) h
A dx l
Negative sign is used because this is opposite to v.
Applying Bernoulli’s theorem at point A, we have x

1
Pa = Pa + rg(h – x) + rv2 Fig. 6.218
2
\ v2 = – rg(h – x) ...(2)
2
Substituting the value of v from Eq. (2) in Eq. (1), we get
dF = r(bdx) 2rg(h – x)
l
F = 2 rgb Ú ( h - x )dx
0

È l2 ˘
or F = 2 r g b Í h l - ˙ = r g b l [2 h - l] = rgbl [2h – l]
Î 2˚
1102പWŚLJƐŝĐƐĨŽƌ//dͲ:͗DĞĐŚĂŶŝĐƐ//

Substituting the given values, we get


F = (1000) (9.8) (1 ¥ 10–3) (0.5) [2 ¥ 0.75 – 0.5]
= 5 N.
Problem 6.139 For the arrangement shown in Fig. 6.219, find the time interval after which
the water jet ceases to cross the wall. Area of the tank is A and area of orifice is a.

Orifice

Wall

Fig. 6.219

Solution Let h be the instantaneous head of water in the tank. For this head of water, the
velocity of efflux v is given by
v = (2gh) ...(1)
Consider a point P (x1, y1) on the trajectory as shown in Fig. 6.220.
1
Now x1 = vt and y1 = - g t 2
2 y
Eliminating t, we get
1 h
y1 = – g(x12/v2) = – x12/4h ...(2) x
2
Let minimum water head be hmin when x1 = y
x and y1 = y. Then from Eq. (2) A P (x1, y1)

- x2 x2
hmin = = ...(3)
4(- y ) 4 y Wall

Let in the interval dt, the water level


in the tank fall by dh. Applying the law of x
conservation of mass, we have
Fig. 6.220
,LJĚƌŽƐƚĂƟĐƐŽƌ&ůƵŝĚ^ƚĂƟĐƐപ1103

– A dh = avdt
or – A dh = a (2 gh) dt
dh a
or – = (2 g ) dt
h A
Integrating this expression within proper limits, we have
dh hmina t
-Ú = (2 g ) Ú dt
H h A 0

a
2[ H - hmin ] = (2 g ) t
A
A Ê 2ˆ
\ t = [ H - hmin ] …(4)
a ÁË g ˜¯
where hmin is given by Eq. (3).
Problem 6.140 Water flows out of a big tank along a tube bent at
right angles: the inside radius of the tube is equal to r Fig. 6.221. The
length of the horizontal section of the tube is equal to l. The water
flow rate is Q litres/s. Find the moment of reaction forces of flowing
water, acting on the tube’s walls, relative to the point O.
l
Solution The velocity v of water in the reaction at tube is given by
O
Q Q
v= =
A p r2
Force of reaction at bent Fig. 6.221

È Ê Q ˆ
2 ˘ Q2 r
F = Ír p r 2 Á 2 ˜ - 0˙ =
Í Ëp r ¯ ˙ p r2
Î ˚
The direction of force of reaction is upward.

Ê Q2 r ˆ
Moment of the force about O = Á 2 ˜ l .
Ëpr ¯

Problem 6.141 A metal piece of mass 1600 g lies in equilibrium


inside a glass of water. The piece touches the bottom of the glass at
a small number of points. If the density of the metal is 8000 kg/m3,
find the normal force exerted by the bottom of the glass on the metal
piece. ++6,''
Solution For the equilibrium of the block
Fig. 6.222
mg = Fb + N
\ N = mg – Fb
= 1.4 N
1104പWŚLJƐŝĐƐĨŽƌ//dͲ:͗DĞĐŚĂŶŝĐƐ//

Problem 6.142 A cylindrical tank 1 m in radius rests on a platform 5 m high. Initially, the
tank is filled with water to a height of 5 m. A plug, whose area is 10– 4 m2, is removed from
the orifice on the side of the tank at the bottom. Calculate the: (IIT Roorkee, 1995)
(i) initial speed with which the water flows from the orifice,
(ii) initial speed with which the water strikes the ground, and
(iii) time taken to empty the tank to half its original value.
Solution See Fig. 6.223.
(i) Let v2 be the initial speed with which the water flows a1
from the orifice. Now

v2 = (2 g h0 ) = [2 ¥ (10) ¥ (5)]
h0
= 10 m/sec a2
Plug
(ii) Let v3 be the velocity with which the water strikes v2
the ground. Now

v3 = [2 g ( h0 + H )] = [2 ¥ (10) ¥ (10)]
H=5m
= 10 2 = 14.1 m/sec
v3
(iii) Let t be the time taken to empty the tank to half its
original value. Platform
The rate of empty the tank = (dh/dt) Fig. 6.223
dh
\ -r a1 = r a2 v2
dt
a dh
or dt = - 1
a2 2 gh

Integrating this expression, we get


a1 1 h0/2 dh
t= -
a2 2g Úh
0 h

a1 Ê 2 ˆ ÈÊ h0 ˆ ˘
1/2
1/2
= - Á ˜ ÍÁË ˜¯ - ( h0 ) ˙
a2 Ë g ¯ ÍÎ 2 ˙˚

2 ÈÊ 5 ˆ
1/2 ˘
= - 3140 ÍÁ ˜ - (5)1/2 ˙
10 ÍÎË 2 ¯ ˙˚
= 942 sec.
Problem 6.143 A laboratory beaker having mass 390 g and an interior volume of 500 cm3
floats on water when it is less than half filled with water. What is the density of the material
of the beaker?
,LJĚƌŽƐƚĂƟĐƐŽƌ&ůƵŝĚ^ƚĂƟĐƐപ1105

Solution As the laboratory beaker floats in water when less than


V = 500 cc
half filled with water, it will float just fully submerged when half
filled. In this situation, Water
Mass of beaker + Mass of water in it = Vr
i.e., 390 + 250 = V ¥ 1 [as r = 1 g/cc] Water
i.e., outer volume of beaker
V = 640 cc Fig. 6.224
Now as the inner volume of the laboratory beaker is given to be 500 cc, so the volume of
the material of beaker = 640 – 500 = 140 cc. But as the mass of the beaker is 390 g, so density
of the material of the laboratory beaker
m 390 g
s= = = 2.79
V 140 cc
Problem 6.144 A block of wood floats in water with two-thirds of its volume submerged.
In oil the block floats with 0.90 of its volume submerged. Find the density of (a) wood, and
(b) oil, if density of water is 103 kg/m3.
Solution In case of floatation W = Th, i.e., Vs = Vin r
So,
(a) Vs = (2/3)VrW [as Vin = (2/3)V]
2 2 kg
or s = rW = ¥ 10 3 = 667 3
3 3 m
(b) For oil, Vs = 0.9Vroil
or 0.9Vroil = (2/3)VrW) [as Vs = (2/3)VrW]
2 2 kg
or roil = rW = ¥ 10 3 = 740 3 .
3 ¥ 0.9 2.7 m
Problem 6.145 Water flows into a horizontal pipe whose one end is closed with a valve
and the reading of a pressure gauge attached to the pipe is 3 ¥ 105 N/m2. This reading of
the pressure gauge falls to 1 ¥ 105 N/m2 when the valve is opened. Calculate the speed of
water flowing into the pipe.
Solution If v is the speed, then,
1 2
rv = (P1 – P2)
2
2(P1 - P2 )
or v =
r

2(3 ¥ 10 5 - 1 ¥ 10 5 )
=
1000
= 20 m/s Ans.
1106പWŚLJƐŝĐƐĨŽƌ//dͲ:͗DĞĐŚĂŶŝĐƐ//

Problem 6.146 A block of wood weighs 12 kg and has a relative density 0.6. It is to be in
water with 0.9 of its volume immersed. What weight of a metal is needed (a) if the metal is
on the top of the wood, (b) if the metal is attached below the wood? (RD of metal = 14)
Solution
(a) When the metal is on the top of the wood,
M È M˘
M + m1 = 0.9VWr = 0.9 s r ÍasVW = ˙
W Î s W˚

È r ˘ È 0.9 ˘
m1 = M Í0.9 - 1˙ = 12 Í - 1 = 6 kg
Î sW ˚ Î 0.6 ˙˚
m1

VW sW VW sW
r r
M M
sM m2
VM

(a) (b)

Fig. 6.225

(b) When the metal is attached at the bottom of the wood,


(M + m2) = (0.9VW + VM)r
È M m2 ˘ È Mass ˘
or M + m2 = Í0.9 + ˙r ÍÎas s = Vol. ˙˚
Î rW r M ˚
È r ˘ È r ˘
or m2 Í1 - ˙ = M Í0.9 - 1˙
Î sM ˚ Î sW ˚

È 1˘ È 0.9 ˘
or m2 Í1 - ˙ = 12 Í - 1 = 6 kg
Î 14 ˚ Î 0.6 ˙˚
14 ¥ 6
or m2 = 6.5 kg
13
Problem 6.147 A cubical block of iron 5 cm on each side is floating on mercury in a vessel.
(a) What is the height of the block above mercury level? (b) Water is poured into the vessel
so that it just covers the iron block. What is the height of water column?
[R.D. of Hg = 13.6 and Fe = 7.2]

Solution
(a) In case of floatation W = Th, i.e., Vs = Vin r, so if h is the height of iron block above
the mercury and a is the side of the iron cube,
,LJĚƌŽƐƚĂƟĐƐŽƌ&ůƵŝĚ^ƚĂƟĐƐപ1107

a
h 2.35 h 2.54
2.65 2.46

a–h
Hg Hg

(a) (b)

Fig. 6.226

(a ¥ a ¥ a)s = (a – h) ¥ a ¥ a ¥ r
s
i.e., (a – h) = a
r
Ê sˆ
or h = aÁ1 - ˜
Ë r¯
Ê 7.2 ˆ
So, h = 5Á1 - = 2.35 cm
Ë 13.6 ˜¯
(b) Here upthrust is provided by both mercury and water and if h is the height of water level,
Vsg = Th Hg + ThW
i.e., a ¥ a ¥ a ¥ s = a ¥ a(a – h)rHg + a ¥ a ¥ hrW
i.e., as = (a – h)rHg + hrW
or h(rHg – rW) = a(rHg – s)
( r Hg - s ) (13.6 - 7.2)
or h = a = 5¥ = 2.54 cm
( r Hg - rW ) (13.6 - 1)

Problem 6.148 A body of density r is released gently on the surface of a layer of a liquid
of depth d and density s (r > s). Show it will reach the bottom of the liquid after a time
1
È 2dr ˘ 2
Í ˙ .
Î g( r - s ) ˚
Solution The acceleration of the body
mg - Fb
a =
m
Ê r -s ˆ
= Á g
Ë r ˜¯
1
The distance d = 0 + at 2
2
2d
\ t =
a
2dr
=
g( r - s )
1108പWŚLJƐŝĐƐĨŽƌ//dͲ:͗DĞĐŚĂŶŝĐƐ//

Problem 6.149 A cubical tank of side 2 m is filled with 1.5 m of glycerine of specific gravity
1.6. Find the force acting on the side of the tank when:
(a) it is accelerated vertically upwards at 5 m/s2, and
(b) it is accelerated vertically downwards at 5 m/s2.
Solution The pressure intensity at the bottom of the tank
P = r(g + a)h
0+P
The average pressure intensity Pav =
2
r( g + a)h
=
2
(a) The force on the side of the tank.
F = Pav ¥ Wetted area of wall
r( g + a)h
= ¥ (bh)
2
r( g + a)bh 2
=
2
1600(9.8 + 5) ¥ 2 ¥ 1.52
=
2
 53.316 kN
(b) In case of acceleration downward
r( g - a)bh 2
F =
2
1600(9.8 - 5) ¥ 2 ¥ 1.52
=
2
= 17.316 kN
Problem 6.150 A solid ball density half that of water falls freely under gravity from a height
of 19.6 m and then enters water. Up to what depth will the ball go? How much time will it
take to come again to the water surface? Neglect air resistance and viscosity effects in water.
Solution The velocity of the ball after falling from height of 19.6 m

v = 2 g ¥ 19.6

= 19.6 m/s
The retardation of the ball in water
Fb - mg
a =
m
,LJĚƌŽƐƚĂƟĐƐŽƌ&ůƵŝĚ^ƚĂƟĐƐപ1109

r
Vrg - V g
= 2 19.6 m
V r/2

= g m/s2 h

If h is the depth up to which ball goes, then


Fig. 6.227
o = v2 – 2ah
v 2 19.6
\ h = = = 19.6 m Ans.
2a 2 ¥ g
v 19.6
Time taken t = = 2s
g 9.8
Total time taken to return to the surface = 4 s.
Problem 6.151 A balloon filled with hydrogen has a volume of 1000 litres and its mass 1
kg. What would be the volume of the block of a very light material which it can just lift?
One litre of the material has mass of 91.3 g. (Density of air = 1.3 g/litre)
Solution To just lift off the block
(m + M)g = Fb1 + Fb2
(1000 + V ¥ 9.1.3)g = 1000 ¥ 1.3 g + V ¥ 1.3 ¥ g

Fb = Fb1 + Fb2

(m + M)g

Fig. 6.228

V = 3.333 litres
Problem 6.152 A hemispherical tank of radius R has an orifice of cross-sectional area a at
its base. Determine the time required to empty the tank from initial height H1 of the liquid
to final height H2.
Solution Volume of liquid in layer of thickness
dv = – px2 dh ...(1)
2
where px is the area of liquid surface at height h.
1110പWŚLJƐŝĐƐĨŽƌ//dͲ:͗DĞĐŚĂŶŝĐƐ//

R
x B H1
dh A
h H2

Fig. 6.229

Volume of liquid flowing out of the orifice


= a 2 gh dT ...(2)
Equating Eqs. (1) and (2), we have
– px2 dh = a 2 gh dT

-p x 2 dh
dT = ...(3)
a 2 gh
We have two variables x and h. We can relate x and h by applying Pythagoras’ Theorem
to triangle OAB.
x = R 2 - ( R - h )2 R
(R – h)
x
2
= 2Rh - h dh

Thus, the expression for dT becomes


- p(2Rh - h 2 )dh h
dT =
a 2 gh
T H2 -p
Ú0 dT = ÚH 1 a 2g
(2Rh1/2 - h 3/2 )dh

H Fig. 6.230
È ˘ 2
1/2+1 3/2+1
-p Í h h ˙
= Í 2R 1 - ˙
a 2g Í 3
+1 +1 ˙
Î 2 2 ˚ H1
-p È 4 3/2 3/2 2 5/2 5/2 ˘
= Í R( H 2 - H1 ) - ( H 2 - H1 )˙
a 2g Î 3 5 ˚
p È4 3/2 3/2 2 5/2 5/2 ˘
= Í R( H1 - H 2 ) - ( H1 - H 2 )˙
a 2g Î 3 5 ˚
For completely emptying the tank, H2 = 0.
p È4 3/2 2 5/2 ˘
T = Í RH1 - H1 ˙
a 2g Î 3 5 ˚
,LJĚƌŽƐƚĂƟĐƐŽƌ&ůƵŝĚ^ƚĂƟĐƐപ1111

Problem 6.153 Two liquids which do not react chemically are placed in a bent tube as
shown in Fig. 6.231. Find out the displacement of the liquid in equilibrium position.

2r r
 


Fig. 6.231

Solution The pressure at the interface must be same, calculated via either tube. Since both
tube are open to the atmosphere, we must have
2rg(l – x) = rg(l + x) fi x = l/3

x x

 

Fig. 6.232

Problem 6.154 Three liquids which do not react chemically are placed in a bentule as
shown in Fig. 6.233 (initially) then fluid out the displacement of the liquid in equilibrium
position.

r 3r
 
2r

Fig. 6.233
x
Solution Let us assume that level of liquid having
x
density 3r displaced below by x as shown in Fig. 6.234
below. 
–x
fi rlg + 2rgx = 3r(l – x)g
x = 2l/5 Fig. 6.234

Problem 6.155 Figure 6.235 shows a tube in which liquid is filled at the level. It is now
rotated at an angular frequency w about an axis passing through arm A find out pressure
difference at the liquid interfaces.
1112പWŚLJƐŝĐƐĨŽƌ//dͲ:͗DĞĐŚĂŶŝĐƐ//

w B
A

Fig. 6.235

Solution To solve the problem we take a small mass dm B


from the axis at a distance x in displaced condition.
Net inward force = (P + dP) A – PA P + dP
= AdP dm = rAdx x0
P
This force is balanced by centripetal force in equilibrium
l xw dx
Ú dP = Ú rw
2
fi 2
AdP = dm w x = rAdxw x 2
fi xdx x
x0
l  – x0
DP = rw 2 Ú xdx = x0 r g
Fig. 6.236
x0

Problem 6.156 A triangular tube of uniform cross section has r2


three liquids of densities r1, r2 and r3. Each liquid column has
length l equal to length of sides of the equilateral triangle. Find
the length x of the liquid of density r1 in the horizontal limb of the
tube, if the triangular tube is kept in the vertical plane. r1

Solution Let us consider two points 1 and 2 in the horizontal 60°


r3
60°
limb.
Pressure at 1 is P1 = r1gh1 + r2gh2 x
l
Pressure at 2 is P2 = r2gh¢2 + r3gh3
Fig. 6.237
r2
x

(l –
x)

h2 h¢2

r1
30°
x)
(l –

h1
x r3
60° h3

1
x (l – x) 2

Fig. 6.238
,LJĚƌŽƐƚĂƟĐƐŽƌ&ůƵŝĚ^ƚĂƟĐƐപ1113

Since P1 = P2 for non-accelerating liquid, we have


r1h1 + r2h2 = r2h¢2 + r3h3
Substituting, we have
h1 = h¢2 = (l – x)sin 60°
h2 = h3 = x sin 60°
( r2 - r1 )l
x =
2 r2 - ( r1 + r3 )

Student Task
r1 + r2
s )N THE ABOVE EXAMPLE IF r3 = , find x.
2 2l
[Ans. ]
3
A
Problem 6.157 Figure 6.239 shows a glass tube in the
form of an equilateral triangle of uniform cross section. It x
d
lies in the vertical plane, with base horizontal. The tube is
l–x F
filled with equal volumes of three imiscible liquids whose
densities are in arithmetic progression. Determine the length
x as shown in Fig. 6.239. D l–x
Solution Let the densities of liquids in DAF, FCE and EBD x 60° d +y
be d, d + y and d + 2y respectively. Pressure at a point inside
a fluid depends on the vertical height of the liquid above B d + 2y E C
x
that point. We will calculate pressure at E from left arm and
right arm, these values must be the same for equilibrium. Fig. 6.239
PE = dg(l – x)sin 60° + (d + 2y)gx sin 60° [left arm]
PE = dgx sin 60° + (d + y)g (l – x) sin 60° [right arm]
Equating the two values of pressures,
3 x 3 x 3 3
dg(l - x ) + (d + 2 y) g = dg + (l - x ) (d + y) g
2 2 2 2
(l – x) 3d + x 3 (d + 2 y ) = x 3d + (l - x ) 3 (d + y )
(l – x) 3d + x 3d + 2 yx 3 = x 3d + (l - x ) 3d + y(l - x ) 3
l
2yx = y(l – x); 2x = l – x; x =
3
Problem 6.158 A large slab rests on a soft ground at the bottom
F
of a lake. The slab has sunk into the ground a slight distance so 17 m
that water cannot reach the under surface. If the slab of mass
1000 kg, has a cross-sectional area parallel to the free surface of
1 m2, what initial force is required to start moving it up? When 1000 kg
it has moved a slight distance up so that water reaches the lower
surface, what is the force required to hold the slab? Fig. 6.240
1114പWŚLJƐŝĐƐĨŽƌ//dͲ:͗DĞĐŚĂŶŝĐƐ//

Solution In the first case, there is no thrust of water below the slab, so
F = mg + (rgh) ¥ A
= 1000 ¥ 9.8 + (1000 ¥ 9.8 ¥ 17) ¥ 1
= 17658 N
In the second case
F = mg – Fb
If Fb is small enough, then
F  mg
= 9800 N
Problem 6.159 Find the depth of penetration of a cube of mass m cross section A floating
in a liquid of density r, if the liquid accelerates vertically?
Solution The forces acting on the cubical block are Fhydro ≠ and mg Ø. The net force acting
on the body (cube) is
F = Fhydro – mg
A
where Fhydro = Ú PdA = r(g + a)yA
This gives F = Ar(g + a)y – mg a
This force accelerates the block with an acceleration a. y
mg
Applying Newton’s 2nd law, we have
F = Ar(g + a)y – mg = ma
m Fhydro
This gives y =
Ar Fig. 6.241
The above expression tells us that the depth of immersion remains
constant and does not depend upon the acceleration of the liquid.
2TQDNGO  0CTTQYKPI YCVGT LGV A stream of liquid jet comes out of a water tap of
radius R with a downward velocity v0. Find the radius of the tube of liquid at a vertical
distance y from the water tap.
Solution Here, the tube of flux has length y. We have
Water tap
chosen the points 1 and 2 in the tube of flux to apply the 1
following two equations. Since, both points are subjected
to atmosphere, we have P1 = P2 = Patm. Then, Bernoulli’s
equations can be given as
v1
y
v2 v2
Patm + r 1 + r gh1 = Patm + r 2 + r gh2
2 2 h1
2
Substituting v1 = v0, v2 = v, h1 – h2 = y

we have v = v02 + 2 gy …(i) h2


v2

Fig. 6.242
,LJĚƌŽƐƚĂƟĐƐŽƌ&ůƵŝĚ^ƚĂƟĐƐപ1115

Equation of continuity gives


A1v 1 = A 2v 2
where A1 = pR2, v1 = v0, A2 = pr2

and v2 = v02 + 2 gy

R v0
This gives r =
(v02 + 2 gy )1/4

Student Task
s )N THE FOREGOING EXAMPLE IF THE LIQUID JET STRIKES THE GROUND AFTER FALLING A DISTANCE h,
find the thrust exerted by the liquid in the process of collision. Assume that the liquid
does not bounce just after collision.
[Ans. p R2 ( v02 + 2 gh)rv0 ]
When pressure does not vary between two points of a tube of flux,
rv 2
+ r gh = constant. It means, v is more where height h is less and vice-versa.
2

Problem 6.161 Water flows through a horizontal tube as


shown in Fig. 6.243. If the difference of heights of water
column in the vertical tubes is 2 cm and the areas of cross
section at A and B are 4 cm2 and 2 cm2 respectively. Find the
rate of flow of water across any section. A
B
1 2 1 2
Solution rvB - rvA = rg(hA – hB)
2 2
Fig. 6.243
or vB2 – vA2 = 2g (hA – hB)
= 2 ¥ 10 ¥ 0.02
or vB2 – vA2 = 0.4 m2/s2 ...(i)
vA AA = v BA B
or 4vA = 2vB
\ vB = 2vA ...(ii)
Solving Eqs. (i) and (ii), we get
vA = 0.363 m/s
Volume flow rate = vAAA
= (0.365)(4 ¥ 10– 4)
= 1.46 ¥ 10– 4 m3/s
= 146 cm3/s
Problem 6.162 Water flows through the tube as shown in Fig. 6.244. The areas of cross
section of the wide and the narrow portions of the tube are 5 cm2 and 2 cm2 respectively.
The rate of flow of water through the tube is 500 cm3/s. Find the difference of mercury
levels in the U-tube.
1116പWŚLJƐŝĐƐĨŽƌ//dͲ:͗DĞĐŚĂŶŝĐƐ//

Solution In this case, the rate of flow is given by

2rm gh
Q = A1 A2
r( A12 - A22 )

2 ¥ 13.6 ¥ 981 ¥ h
or 500 = 5 ¥ 2
1(52 - 22 )
Fig. 6.244
After simplifying, we get h = 1.97 cm

Problem 6.163 A spray gun is shown


in the figure where a piston pushes
air out of a nozzle. A thin tube of
uniform cross section is connected to
the nozzle. The other end of the tube
is in a small liquid container. As the
piston pushes air through the nozzle,
the liquid from the container rises into
Fig. 6.245
the nozzle and is sprayed out. For the
spray gun shown, the radii of the piston and the nozzle are 20 mm and 1 mm respectively.
The upper end of the container is open to the atmosphere.
1
Solution PA – PB = ra n a2
2

PC – PB = 1 r n 2
l l
2
But PC = PA

1 1 2 ra
\ rl n l2 = 2 ra n a fi nl = ¥ na
2 rl

ra
\ Volume flow rate μ = Ans.
rl
60 rpm

Problem 6.164 A vertical U-tube with the two limbs 0.75 m apart is
filled with water and rotated about a vertical axis 0.5 m from the left
limb, as shown in Fig. 6.246. Determine the difference in elevation of
the water levels in the two limbs when the speed of rotation is 60 rpm.
Solution Consider a small element of length dr at a distance r from the
0.5 m
axis of rotation. Considering the equilibrium of this element,
0.75 m
(p + dp) – p = rw 2 r dr Fig. 6.246
or dp = rw2r dr
,LJĚƌŽƐƚĂƟĐƐŽƌ&ůƵŝĚ^ƚĂƟĐƐപ1117

On integrating between 1 and 2,


r1
p1 – p2 = rw 2 Ú rdr
- r2
h1
rw 2 2 2 r h2
p1 – p2 = (r1 - r2 ) dr
2 1 2
p + dp
2 2
w 2 2 (2p ) r1 r2
or h1 – h2 = [r1 - r2 ] = [0.5)2 - [0.25)2 ] = 0.37 m.
2g 2(10) Fig. 6.247

Problem 6.165 A block of mass m is kept


over a fixed smooth wedge. Block is attached P1 P2
to a sphere of same mass through fixed
massless pullies P1 and P2. The sphere is m
dipped in the water as shown in Fig. 6.248. m
If specific gravity of the material of sphere is 30°
2. Find the acceleration of sphere.
Solution Writing equation of motion for the Fig. 6.248
block
T – mg sin 30° = ma ...(i)
For the sphere
Weight – Buoyant force – T = ma ...(ii)
mg
or mg – – T = ma
2
Solving, we get a =0
Problem 6.166 A cubic body floats on mercury with 0.25 fraction of its volume below the
surface. What fraction of the volume of the body will be immersed in the mercury if a layer
of water poured on top of the mercury covers the body completely?
Solution W = (0.25)VrHg g ...(i)
Let x fraction of volume be immersed in mercury in the second case. Then,
W = xVrHg g + (1 – x)Vrwg ...(ii)

(a) (b)
Fig. 6.249
Equating Eqs. (i) and (ii), we have
rHg
= xrHg + (1 – x)rw
4
\ 12.6x = 2.4
or x = 0.19
1118പWŚLJƐŝĐƐĨŽƌ//dͲ:͗DĞĐŚĂŶŝĐƐ//

Problem 6.167 A wooden stick of length L, radius R and density r has a small metal piece of
mass m (of negligible volume) attached to its one end. Find the minimum value for the mass
m (in terms of given parameters) that would make the stick float vertically in equilibrium in
a liquid of density s, where s (> r).
Solution For the stick to be vertical for rotational equilib-
rium, centre of gravity should be below in a vertical line
through the centre of buoyancy. For minimum m, the two
will coincide.
Let h be the length of immersed portion. For translatory
equilibrium,
1 h
Wt. of rod + Mass attached = Force of buoyancy 2 C
(M + m)g = pR2 hsg ...(i)
where M = pR Lr
2 m

The height of the centre of mass from bottom Fig. 6.250

m1 y1 + m2 y2 ( M )L/2 + m ¥ 0 ML
yCM = = =
m1 + m2 m+ M 2(m + M )
For rotatory equilibrium and for minimum m, this should be equal to h/2.
h ML
\ = ...(ii)
2 2(m + M )
ML
\ h =
(m + M )
Substituting for h in Eqs. (i), we get
ML
(M + m)g = p R2s g ◊
(m + M )
(M + m)2 = pR2s ◊ ML

(M + m) = Mp R2s L = p R2 Lr ◊ p R2s L

m = p R2 L sr - p R2 Lr

È s ˘
= p R 2 Lr Í - 1˙
Î r ˚
The mass m cannot be at the top, because for stable equilibrium, the centre of mass should
be the lowest possible.
Problem 6.168 What is the pressure drop (in mm Hg) in the blood as it passes through a
capillary 1 mm long and 2 mm in radius if the speed of the blood through the centre of the
capillary is 0.66 mm/s? (The viscosity of whole blood is 4 ¥ 10– 3 P1.)
,LJĚƌŽƐƚĂƟĐƐŽƌ&ůƵŝĚ^ƚĂƟĐƐപ1119

Solution See the expression of maximum velocity at the centre of the tube,
( p1 - p2 )R2
vmax =
4hL
4hL vmax
\ (P1 – P2) =
R2
( 4 ¥ 4 ¥ 10 - 3 )(10 - 3 )(0.66 ¥ 10 - 3 )
=
(2 ¥ 10 - 6 )2
= 2.64 ¥ 103 N/m2 = hrg

2.64 ¥ 10 3
h =
rg
2.64 ¥ 10 3
= m of Hg
13.6 ¥ 10 3 ¥ 9.81
= 0.0195 m of Hg
= 19.5 mm of Hg
Problem 6.169 Two equal drops of water are falling through air with a steady velocity v.
If the drops coalesce, what will be the new velocity? ++6,''
Solution ∵ Volume of bigger drop
= n ¥ Volume of smaller drop
4 4 R
p R3 = 2 ¥ p r 3 fi = 21/3
3 3 r
Terminal velocity of smaller drop = V1
As terminal velocity μ r2
\ Terminal velocity of bigger drop = V2
200 rpm

O
A B

H = 0.4 m

= (21/3)2
= (22/3)
Problem 6.170 Velocity of a viscous liquid in long cylinder of radius R at a distance R1
from centre is v1. Find the velocity of the liquid as a function of the distance r from the axis
of the cylinder.
Solution See the expression of v.
p1 - p2 2 2
v = (R - r ) ...(i)
4hL
P1 - P2 2
v1 = (R - R12 ) ...(ii)
4hL
1120പWŚLJƐŝĐƐĨŽƌ//dͲ:͗DĞĐŚĂŶŝĐƐ//

Dividing Eq. (i) by Eq. (ii) we get


Ê R2 - r 2 ˆ
v = Á 2 ˜ v1
Ë R - R12 ¯
Problem 6.171 A ball of density d is dropped onto a horizontal solid surface. It bounces
elastically from the surface and returns to its original position in a time t1. Next, the ball
is released and it falls through the same height before striking the surface of a liquid of
density dL. ++6,''
(a) If d < dL, obtain an expression (in terms of d, t1 and dL) for the time t2 the ball takes to
come back to the position from which it was released.
(b) Is the motion of the ball simple harmonic?
(c) If d = dL, how does the speed of the ball depend on its depth inside the liquid? Neglect
all frictional and other dissipative forces. Assume the depth of the liquid to be large.
Solution In elastic collision with the surface, direction of velocity is reversed but its
magnitude remains the same.
Therefore, time of fall = time of rise.
t1
or Time of fall =
2
Hence, velocity of the ball just before it collides with liquid is
t1
v = g ...(i)
2
Retardation inside the liquid
Upthrust - Weight
a =
Maass
VdL g - Vdg
=
Vd
Ê d - dˆ
=Á L g ...(ii)
Ë d ˜¯

Time taken to come to rest under this retardation will be


v
t =
a
gt1
=
2a
gt1
=
Ê dL - d ˆ
2Á g
Ë d ˜¯
dt1
=
2(dL - d)
Same will be the time to come back on the liquid surface.
,LJĚƌŽƐƚĂƟĐƐŽƌ&ůƵŝĚ^ƚĂƟĐƐപ1121

Therefore,
(a) t2 = time the ball takes to come back to the position from where it was released
= t1 + 2t
dt1
= t1 +
dL - d
È d ˘
= t1 Í1 + ˙
Î dL - d ˚
t1dL
or t2 =
dL - d
(b) The motion of the ball is periodic but not simple harmonic because the acceleration
Ê d - dˆ
of the ball is g in air and Á L g inside the liquid which is not proportional to the
Ë d ˜¯
displacement, which is necessary and sufficient condition for SHM.
(c) When dL = d, retardation or acceleration inside the liquid becomes zero (upthrust =
gt
weight). Therefore, the ball will continue to move with constant velocity v = 1 inside
2
the liquid.
Problem 6.172 A beaker in a cylinder form is fitted with a piston,
beneath which is a spring, as in Fig. 6.251. The cylinder is open at the
top. Friction is absent. The spring constant of the spring is 3600 N/m.
The piston has a negligible mass and a radius of 0.025 m. (a) When
air beneath the piston is completely pumped out, how much does the
atmospheric pressure cause the spring to compress? (b) How much work
the atmospheric pressure do for compressing the spring?
Solution
Fig. 6.251
(a) P0A = kx
2
p0 A ( p0 )(p r )
\ x = =
k k
(1.01 ¥ 10 5 )(p )(0.025)2
=
3600
= 0.055 m = 5.5 cm
(b) Work done by atmospheric pressure

1 2 1
W = kx = (3600)(0.055)2
2 2
= 5.445 J
1122പWŚLJƐŝĐƐĨŽƌ//dͲ:͗DĞĐŚĂŶŝĐƐ//

Problem 6.173 A metal sphere of radius 1 mm and mass 50 mg falls vertically in glycerine.
Find (a) the viscous force exerted by the glycerine on the sphere when the speed of the
sphere is 1 cm/s, (b) the hydrostatic force exerted by the glycerine on the sphere, and (c)
the terminal velocity with which the sphere will move down without acceleration. Density
of glycerine = 1260 kg/m3 and its coefficient of viscosity at room temperature = 8.0 poise.
Solution
(a) Fr = 6phrv
= 6p (0.8)(10– 3)(10– 2)
= 1.5 ¥ 10– 4 N
(b) Hydrostatic force = Upthrust

Ê4 ˆ
= Á p r3 ˜ rg
Ë3 ¯
4
= p (10– 3)3 ¥ 1260 ¥ 9.8
3
= 5.2 ¥ 10 – 5 N
(c) At terminal velocity
W = Upthrust + Viscous force
or (50 ¥ 10 –3
¥ 9.8) = (5.2 ¥ 10– 5) + 6p (0.8)(10 – 3) vT
Solving we get
vT = 32.5 m/s
Problem 6.174 A cubical block of wood, 40 40 cm
cm along each side, floats in water with its
lower surface 5 cm below the free surface of
water as shown in Fig. 6.252.
(a) What is the mass of the block in kg? Free surface
(b) What is the pressure of water in Pa on
the lower surface of block? 5 cm
(g = 10 m/s2, Density of water = 103
kg/ m3)
Fig. 6.252
Solution
(a) Volume of immersed portion of block
v = (40 cm ¥ 40 cm ¥ 5 cm)
= 8 ¥ 103 cm3 = 8 ¥ 103 ¥ 10– 6 m3 = 8 ¥ 10 –3 m3
By law of floatation,
Weight of block = Weight of water displaced
Mg = vdw g (here rw = Density of water)
\ M = vrw = 8 ¥ 10– 3 ¥ 103 = 8 kg
,LJĚƌŽƐƚĂƟĐƐŽƌ&ůƵŝĚ^ƚĂƟĐƐപ1123

(b) Pressure of water at lower surface of block


= hrw g = 5 ¥ 10 – 2 ¥ 103 ¥ 10 = 500 Pa.
Problem 6.175 A wide vessel with a small hole in the bottom is filled with water and
kerosine. The thickness of water layer is 30 cm and that of kerosine is 20 cm. Neglecting the
viscosity, find the velocity of water flow. Density of kerosine oil = 800 kg/m3.
(IIT Roorkee, 1983)

r2

h2
Kerosine

r1

h1

Water

Water
A
B
Fig. 6.253

Solution Let P0 = atmospheric pressure. The total energy of flowing liquid is given by
1 2
E =P+ rv + rgh
2
At B, v = 0
Total energy at B
= P0 + 0 + r1gh1 + r2gh2 ...(i)
Total energy at A
1
= P0 + r1 v12 + 0 ...(ii)
2
According to Bernoulli’s theorem, equating Eqs. (i) and (ii), we get
1
P0 + r1 v12 = P0 + r1g h1 + r2 g h2
2
1
r1 v12 = r1 g h1 + r2g h2
2
2 g( r1h1 + r2 h2 )
\ v1 =
r1

Ê r ˆ
= 2 g Á h1 + 2 h2 ˜
Ë r1 ¯
1124പWŚLJƐŝĐƐĨŽƌ//dͲ:͗DĞĐŚĂŶŝĐƐ//

r1 = 1000 kg/m3, r2 = 800 kg/m3,


h1 = 30 cm = 0.30 m, h2 = 20 cm = 0.20 m

Ê 800 ˆ
\ v1 = 2 ¥ 9.8 Á 0.30 + ¥ 0.20˜ ª 3.00 m/s
Ë 1000 ¯

Problem 6.176 A solid sphere of mass m = 2 kg and density


r = 500 kg/m3 is held stationary relative to a tank filled with
water. The tank is accelerating upwards with acceleration
2 m/s2. Calculate:
a = 2 m/s2
(a) Tension in the thread connected between the sphere and
the bottom of the tank.
(b) If the thread snaps, calculate the acceleration of sphere
with respect to the tank. Fig. 6.254
[Density of water = 1000 kg/m3, g = 10 m/s2]
Solution
(a) Upthrust – Weight – T = ma
\ T = Upthrust – Weight – ma

Ê 2 ˆ
= ÁË ˜ (1000) (10 + 2) – 20 – 4
500 ¯
= 48 – 20 – 4 = 24 N
(b) Downward force T suddenly becomes zero.
Therefore,
Upthrust - Weight
a=
m
48 - 20
=
2
= 14 m/s2
\ Acceleration w.r.t. tank
= 14 – 2
= 12 m/s2
Problem 6.177 An irregular piece of metal weighs 10.00 g in air and 8.00 g when submerged
in water.
(a) Find the volume of the metal and its density.
(b) If the same piece of metal weighs 8.50 g when immersed in a particular oil, what is
the density of the oil?
,LJĚƌŽƐƚĂƟĐƐŽƌ&ůƵŝĚ^ƚĂƟĐƐപ1125

Solution
(a) Relative density of metal

Weight in air 10
= = =5
Change in weight in water 2

\ Density of metal = 5rw = 500


Mass
Now, Volume =
Density

10 ¥ 10 - 3
= = 2 ¥ 10– 6 m3
5000
(b) Change in weight
= Upthrust on 100% volume of solid
or DW = Vsrl g
\ DW μ rl
DWl r
\ = l
DWw rw

Ê DWl ˆ Ê 1.5 ˆ
or rl = Á rw = Á ˜ (1000)
Ë DWw ˜¯ Ë 2 ¯

= 750 kg/m3
Piston
Problem 6.178 The pressure gauge shown in Fig. 6.255 has a
spring for which k = 60 N/m and the area of the piston is 0.50
cm2. Its right end is connected to a closed container of gas at a Spring
gauge pressure of 30 kPa. How far will the spring be compressed F
if the region containing the spring is (a) in vacuum and (b) open
to the atmosphere? Atmospheric pressure is 101 kPa.
Solution
Area = A
(a) Force from left-hand side
Fig. 6.255
= Force from right-hand side
\ kx = DPA
( DP) A
or x =
k

(30 + 101) ¥ 10 3 ¥ 0.5 ¥ 10 - 4


=
60
= 0.109 m or 10.9 cm
1126പWŚLJƐŝĐƐĨŽƌ//dͲ:͗DĞĐŚĂŶŝĐƐ//

( Dp) A 30 ¥ 10 3 ¥ 0.5 ¥ 10 - 4
(b) x = =
k 60
= 0.025 m = 2.5 cm Ans.
Problem 6.179 A cubical vessel filled with a liquid of density r is kept at rest on an inclined
plane of angle of inclination q. Find the pressure difference between A and B in terms of b.

h1

A
h2
b B

Fig. 6.256

Solution First we will determine pressure at A and B respectively.

y1 h1
y2

h2
y2 – y1 q

q
B

Fig. 6.257

The pressure at A and B are


PA = P0 + rgy1
PB = P0 + rgy2
Then, PB = PA = P0 + rgy2 – (P0 + rgy1)
= rg(y2 – y1)
where y2 – y1 = b sin q
Then, we have PB – PA = rgb sin q
,LJĚƌŽƐƚĂƟĐƐŽƌ&ůƵŝĚ^ƚĂƟĐƐപ1127

Problem 6.180 A circular tube of uniform cross section is filled with two liquids of densities
r1 and r2 such that half of each liquid occupies a quarter of volume of the tube. If the line
joining the free surface of the liquids makes an angle q with horizontal, find the value of q.
,''

q r2

r1

Fig. 6.258

Solution Let us find the pressure at the lowest point


1. Since, the liquid of density r2 and height of liquid R
column h¢2 is there, in right-hand side of the point 1, q
we have q h¢2
q
P1 = r1gh1 ...(1) h1
q r2
Since two liquid columns of heights h1 and h2 and
h1 h2
densities r1 and r2 are situated above the point 1, in
left-hand side, we have 1

P2 = r1gh2 + r2gh¢2 ...(2)


Fig. 6.259
Equating P1 and P2 from Eqs. (1) and (2), we have
r1h2 + r2h¢2 = r1h1 ...(3)
Substituting h¢2 = R sin q + R cos q
h2 = R(1 – cos q)
and h1 = R(1 – sin q) in Eq. (3) we have
r1R(1 – cos q) + r2 R(sin q + cos q) = r1R(1 – sin q)
cos q + sin q r
This gives = 1
cos q - sin q r2
r1 - r2
Then tan q =
r1 + r2
Problem 6.181 A tank contains water on top mercury. A cube of iron, 60 mm along each
edge, is sitting upright in equilibrium in the liquids. Find how much of it is in each liquid.
The densities of iron and mercury are 7.7 ¥ 103 kg/m3 and 13.6 ¥ 103 kg/m3 respectively.
1128പWŚLJƐŝĐƐĨŽƌ//dͲ:͗DĞĐŚĂŶŝĐƐ//

Water

Iron

Mercury

Fig. 6.260
Patm
Solution Let x be the submerged depth in mercury, the (0.6 – x)
length protrudes in water. If P1 and P2 are the pressure on the upper Water
and lower faces of the cube, the net upward force = (P2 – P1)A, where
A is the cross-sectional area of cube. In equilibrium it must be equal P1 h
to the weight of the cube.
l
W = (P2 – P1)A ...(1) x
P1 = rwgh + Patm P2
P2 = rwgh + Patm + rwg(l – x) + rHg gx
Mercury
(P2 – P1) = r w g (l - x ) A + rHg gxA ...(2)
  
weight of displaced weight of displaced Fig. 6.261
water mercury

fi Archimedes’ principle can be used for two or more than two liquids for any shape of
object.
From Eqs. (1) and (2)
riron Viron g = rwg(l – x)A + rHggxA
(7.7 ¥ 103)(0.06)3 = (1.0 ¥ 103) (0.06 – x)(0.06)2 + (13.6 ¥ 103)x(0.06)2
which on solving for x yields x = 0.032 m.
Depth submerged in mercury is 32 mm and in water 28 mm.
Problem 6.182 Figure 6.262 shows a container having liquid of
Ê 3h ˆ
variable density. The density of liquid varies as r = r0 Á 4 - ˜ .
Ë h0 ¯
h0
Here h0 and r0 are constants and h is measured from bottom of the
5
container. A solid block of small dimensions whose density is
2
r0 and mass m is released from bottom of the tank. Prove that the Fig. 6.262
block will execute simple harmonic motion. Find the frequency of
oscillation.
,LJĚƌŽƐƚĂƟĐƐŽƌ&ůƵŝĚ^ƚĂƟĐƐപ1129

Solution Net force on the block at a height h from the bottom


is
Fnet = Upthrust – Weight (upwards) x
h0
h=
Ê ˆ 2
Á m ˜ Ê 3h ˆ
= Á ˜ r0 Á 4 - ˜ g - mg
5 Ë h0 ¯
ÁË r0 ˜¯
2
h Fig. 6.263
Fnet = 0 at h = 0
2
h0
So, h = is the equilibrium position of the block.
2
h0
For h > , weight > upthrust
2
h
i.e., net force is downwards and for h < 0
2
Weight < Upthrust
For upward displacement x from mean position, net downward force is
ÈÊ ˆ ˘
ÍÁ m ˜ Ï 3( h + x ) ¸ ˙Ê h0 ˆ
F = - ÍÁ
5 ˜ r0 Ì 4 - h ˝ g - mg ˙ ÁË h = ˜¯
2
ÍÁ r0 ˜ Ó 0 ˛ ˙
ÍÎË 2 ¯ ˙˚
6 mg
\ F = - x ...(i)
5 h0
h0
(because at h = upthrust and weight are equal)
2
Since F μ–x
Oscillations are simple harmonic in nature.
Rewriting Eq. (i)
6 mgx
ma = -
5 h0
6g
or a = - x
5 h0
1 a 1 6g
\ f = f= Ans.
2p x 2p 5 h0

Problem 6.183 A cubical block of side a and density r slide over a


fixed inclined plane with constant velocity v. There is a thin film of
viscous fluid of thickness rt between the plane and the block. Then
the coefficient of viscosity of the film will be ++6,''
3 ragt 4 ragt
(a) (b)
5v 5v qq =
= 37°
30°
ragt
(c) (d) None of these
v Fig. 6.264
1130പWŚLJƐŝĐƐĨŽƌ//dͲ:͗DĞĐŚĂŶŝĐƐ//

Solution Viscous force = mg sin q


v 3
\ h(a2) = mg sin 37° = mg
t 5
3 3 3 ragt
= (a r)g \ h =
5 5v

Problem 6.184 A spring balance reads 10 kg when a bucket of water is suspended from it.
What will be the reading of the balance when an iron piece of mass 7.2 kg suspended by a
string is immersed with half its volume inside the water in the bucket? Relative density of
iron is 7.2.
(a) 10 kg (b) 10.5 kg
(c) 13.6 kg (d) 17.2 kg
Solution Reading = Weight of bucket of water + Magnitude of upthrust on block
1 Ê 7.2 ˆ
= (10 g ) + Á rw g = 10.5 g = 10.5 kg
2 Ë 7.2 rw ˜¯
Problem 6.185 Figure 6.265 shows a pipe of uniform cross
H
section inclined in a vertical plane. A U-tube manometer is
connected between the points A and B. If the liquid of density
H
r0 flows with velocity v0 in the pipe. Then the reading h of
B
the manometer is
v2
(a) h = 0 (b) h = 0
2g
h
r Ê v2 ˆ r H
(c) h = 0 Á 0 ˜ (d) h = 0
r Ë 2g ¯ r - p0
r
Solution From continuity equation,
vA = vB = v0 Fig. 6.265
\ PA + rgh = PB + 0
\ PB – PA = rgh ...(i)
Now let us make pressure equation from manometer.
PA + rg (h + H) – rHg gh = p3
Putting PB – PA = rgh we get h=0
Problem 6.186 A horizontal tube of uniform cross- v
sectional area A is bent in the form of U as shown in
F
Fig. 6.266. If the liquid of density r enters and leaves
the tube with velocity v, then the external force F
required to hold the bend stationary is v

(a) F = 0 (b) rAv2 Fig. 6.266

1
(c) 2rAv2 (d) rAv 2 (BITSAT, 2009)
2
,LJĚƌŽƐƚĂƟĐƐŽƌ&ůƵŝĚ^ƚĂƟĐƐപ1131

Dp Ê Dm ˆ
Solution F = =Á ˜ ( Dv)
Dt Ë Dt ¯

Ê DV ˆ
= rÁ ( 2v )
Ë Dt ˜¯
= r (Av) (2v) = 2rAv2
Problem 6.187 A plate moves normally with the speed v1 towards a horizontal jet of water
of uniform area of cross section. The jet discharges water at the rate of volume V per second
at a speed of v2. The density of water is r. Assume that water splashes along the surface of
the plate at right angles to the original motion. The magnitude of the force acting on the
plate due to the jet of water is (IIT Roorkee, 1974)
ÊVˆ rV
(a) rVv1 (b) r Á ˜ (v1 + v2 )2 (c) (v1 )2 (d) rV (v1 + v2)
Ë v2 ¯ v1 + v2
Dp Ê Dm ˆ
Solution F= =Á ˜ ( Dv)
Dt Ë Dt ¯
Ê DV ˆ
= rÁ (v + v ) = rV (v1 + v2)
Ë Dt ˜¯ 1 2

Problem 6.188 A U-tube having horizontal arm of length 20 cm, has uniform cross-sectional
area = 1 cm2. It is filled with water of volume 60 cc. What volume of a liquid of density 4g/
cc should be poured from one side into the U-tube so that no water is left in the horizontal
arm of the tube?
(a) 60 cc (b) 45 cc (c) 50 cc (d) 35 cc
Solution

Water

60 cm
(x) cm

60 cm3
x cm3

20 cm3
Fig. 6.267

r1gh1 = r2gh2
\ (4)(g)(x) = (1)(g)(60)
or x = 15 cm
Total volume of liquid = (20 + 15) cm3 = 35 cm3
1132പWŚLJƐŝĐƐĨŽƌ//dͲ:͗DĞĐŚĂŶŝĐƐ//

Problem 6.189 An iron casting has a number of cavities in it. It weights 6000 N in air and
4000 N in water. Determine the total volume of all the cavities in the casting. The density of
iron (without cavities) is 7.87 g/cm3.
Solution Volume of the cavities Vcav can be determined by taking difference between the
volume Vcast of the casting as a whole and the volume of the iron in the casting,
Vcav = Vcast – Viron
W
Viron = , W is weight of casting
riron g
Effective weight of casting in water,
Weff = W – rwgVcast
W - Weff
Vcast =
rw g
W - Weff W
Thus, Vcav = -
rw g g riron
6000 - 4000 6000
= -
(0.998 ¥ 10 )(9.8) (9.8)(7.87 ¥ 10 3 )
3

= 0.127 m3
Problem 6.190 What work should be done in order to
squeeze all water from a horizontally located cylinder (Fig.
6.268) during time t by means of a constant force acting on
the piston? The volume of water in the cylinder is equal to F
V, the cross-sectional area of the orifice is s, with s being
considerably less than the piston area. The friction and Fig. 6.268
viscosity are negligibly small. Density of water is r.

F 1
Solution = rv 2
A 2 1
or F = rAv 2 ...(i)
2
Here, v is the velocity of liquid, with which it comes out of the hole.
Further V = Ax ...(ii) A

V
t= ...(iii)
sv V
F
and W=F◊x ...(iv)
From the above four equations, x

Ê1 ˆÊVˆ Fig. 6.269


W = Á Arv 2 ˜ Á ˜
Ë2 ¯ Ë A¯
1 V2 1 rV 3
= r 2 2 ◊V =
2 st 2 s 2t 2
,LJĚƌŽƐƚĂƟĐƐŽƌ&ůƵŝĚ^ƚĂƟĐƐപ1133

Problem 6.191 A level controller is shown in Fig. 6.270. It consists


of a thin circular plug of diameter 10 cm and a cylindrical float of
diameter 20 cm tied together with a light rigid rod of length 10 Float
cm. The plug fits in snugly in a drain hole at the bottom of the
tank. As water fills up and the level reaches the height h, the plug h
opens. Find h. Determine the level of water in the tank when the 10 cm
plug closes again. The float has a mass of 3 kg and the plug may Plug
be assumed to be massless.
Fig. 6.270
Solution The plug opens because of the buoyancy on the float.
Consider the FBD of the assembly
The forces acting are: Buoyancy
(i) Weight of the float 3g downwards
(ii) Pressure force on the plug
= rgh (p/4) (0.1)2 downwards
3g
(iii) Buoyancy on the float
= rg(h – 0.1) (p/4) (0.2)2 upwards
The plug opens when
rg(h – 0.1)(p/4)(0.2)2 = gh(p/4)(0.1)2 + 3g
fi 10p(h – 0.1) = 2.5ph + 3 Fig. 6.271
3+p
fi h = = 0.261 m = 26.1 cm
7.5p
The plug closes again when
(h¢ – 0.1)(p/4)(0.02)2 ¥ 1 ¥ 103 ¥ g = 3g
where h¢ is the level of water in the tank when it happens
3
fi h¢ – 0.1 =
10p
fi h¢ = 0.195 m = 19.5 cm
Problem 6.192 A barrier AB of length 12 m is hinged at A. At the
A
lower end a horizontal spring keeps the barrier closed. The height 4m
of the water is 6 m and the width of the barrier is 5 m. Water level 12 m
is 4 m below the hinge A. If minimum elongation of spring to keep
6m
barrier closed is 1 m, find the spring constant. Neglect atmospheric
pressure? B

Solution Torque applied by pressure force about A, Fig. 6.272


h
È h 3 dh 2 d 3 ˘
t= Ú ( x - d ) r gwdx ◊ x = r gw Í -
Î3 2
+ ˙

d
1134പWŚLJƐŝĐƐĨŽƌ//dͲ:͗DĞĐŚĂŶŝĐƐ//

Putting values = t = 72 ¥ 105 Nm.

A
d x
l
h
dx

Fig. 6.273

Taking moments about point A,


72 ¥ 105 = (k ¥ 1)(12)
fi k = 600 kN/m.

DAILY PRACTICE PROBLEMS FOR JEE MAIN AND ADVANCE

‡˜‡Žͳ౨ 1PN[QPGQRVKQPKUEQTTGEV 13d 23d


(c) (d)
1. From the adjacent figure, the correct 9 18
observation is 3. A vertical U-tube of uniform inner
cross section contains mercury in both
sides of its arms. A glycerine (density
Water Water
= 1.3 g/cm3) column of length 10 cm
(b)
is introduced into one of its arms. Oil
(a)
of density 0.8 gm/cm3 is poured into
(a) The pressure on the bottom of the other arm until the upper surfaces
tank (a) is greater than at the of the oil and glycerin are in the same
bottom of (b) horizontal level. Find the length of the
(b) The pressure on the bottom of oil column. Density of mercury = 13.6
the tank (a) is smaller than at the g/cm3
bottom of (b)
(c) The pressure depends on the Oil
shape of the container 10cm 10 cm
(d) The pressure on the bottom of (a) Glycerin
and (b) is the same
Mercury
2. Three liquids of densities d, 2d and
3d are mixed in equal proportions of
(a) 10.4 cm (b) 8.2 cm
weights. The relative density of the
(c) 7.2 cm (d) 9.6 cm
mixture is
4. A closed rectangular tank is completely
11d 18 d filled with water and is accelerated
(a) (b)
7 11 horizontally with an acceleration a
,LJĚƌŽƐƚĂƟĐƐŽƌ&ůƵŝĚ^ƚĂƟĐƐപ1135

towards right. Pressure is (i) maximum a mass of 3.5 kg. When we release the
at, and (ii) minimum at lump, it will move
(a) upward (b) downward
A C a (c) remain in place (d) none of these
9. A ball of radius r and density r falls
B D freely under gravity through a distance
(a) (i) B (ii) D (b) (i) C (ii) D h before entering water. Velocity of
(c) (i) B (ii) C (d) (i) B (ii) A ball does not change even on entering
5. Density of ice is r and that of water is water. If viscosity of water is h, the
s. What will be the decrease in volume value of h is given by
when a mass M of ice melts
1,''
M s -r h
(a) (b)
s -r M

È1 1˘ 1 È1 1˘
(c) M Í - ˙ (d) +
Îr s ˚ M ÍÎ r s ˙˚
2 2 Ê 1- rˆ
6. A siphon in use is demonstrated in (a) r g
9 ÁË h ˜¯
the following figure. The density of the
liquid flowing in siphon is 1.5 gm/cc. 2
2 2 Ê r - 1ˆ
(b) r g
81 ÁË h ˜¯
The pressure difference between the
point P and S will be
Q R 2
2 4 Ê r - 1ˆ
(c) r g
10 cm 81 ÁË h ˜¯
20 cm
P
2
S 2 4 Ê r - 1ˆ
(d) r g
9 ÁË h ˜¯

10. A cylindrical vessel of 90 cm height is


(a) 105 N/m (b) 2 ¥ 105 N/m kept filled up to the brim. It has four
(c) zero (d) infinity holes 1, 2, 3, 4 which are respectively
7. A block floats in water in a stationary at heights of 20 cm, 30 cm, 45 cm and
elevator. When the elevator moves 50 cm from the horizontal floor PQ.
upwards with an acceleration, the The water falling at the maximum
block floats now with volume inside horizontal distance from the vessel
the water (BITSAT, 2013) comes from
(a) same (b) less
(c) greater 4
(d) any of the above depending on 3
the density of the block 2
8. We fully submerge an irregular 5 kg 1
lump of material in a certain fluid.
The fluid that would have been in the P Q
space now occupied by the lump has
1136പWŚLJƐŝĐƐĨŽƌ//dͲ:͗DĞĐŚĂŶŝĐƐ//

(a) hole number 4


g
(b) hole number 3
3
(c) hole number 2
(d) hole number 1
11. Water is moving with a speed of 5.18
ms–1 through a pipe with a cross-
sectional area of 4.20 cm2. The water 15. A given shaped glass tube having
gradually descends 9.66 m as the pipe uniform cross section is filled with
increases in area to 7.60 cm2. The speed water and is mounted on a rotatable
of flow at the lower level is shaft as shown in the figure. If the
(a) 3.0 ms–1 (b) 5.7 ms–1 tube is rotated with a constant angular
–1
(c) 3.82 ms (d) 2.86 ms–1 velocity w, then
12. A manometer connected to a closed tap
reads 3.5 ¥ 105 N/m2. When the valve A B
is opened, the reading of manometer
falls to 3.0 ¥ 105 N/m2, then velocity
of flow of water is
(a) 100 m/s
(b) 10 m/s
(c) 1 m/s L 2L

(d) 10 10 m/s
(a) Water levels in both sections A
13. An incompressible liquid flows
and B go up
through a horizontal tube shown in
(b) Water level in section A goes up
the following figure. Then the velocity
and that in B comes down
v of the fluid is (BITSAT, 2016)
(c) Water level in section A comes
v2 = 1.5 m/s down and that in B it goes up
A (d) Water levels remain same in both
v1 = 3 m/s A
sections
1.5 16. An application of Bernoulli’s equation
A for fluid flow is found in
v
(a) dynamic lift of an aeroplane
(a) 3.0 m/s (b) 1.5 m/s (b) viscosity meter
(c) 1.0 m/s (d) 2.25 m/s (c) capillary rise
14. A cubical block is floating in a liquid (d) hydraulic press
with half of its volume immersed in 17. In the figure shown, water drains out
the liquid. When the whole system through a small hole of a large tank.
accelerates upwards with acceleration A
of g/3, the fraction of volume immersed H/2
H B
in the liquid will be C
1 3
(a) (b)
2 8
(a) Pressure at the point C is greater
2 3 than the atmospheric.
(c) (d) H
3 4 (b) Pressure at the point B is rg .
2
,LJĚƌŽƐƚĂƟĐƐŽƌ&ůƵŝĚ^ƚĂƟĐƐപ1137

(c) Pressure at the point B is less than (a) r1 > r3 > r2 (b) r1 > r2 > r3
H (c) r1 > r3 > r2 (d) r3 < r1 < r2
rg . 21. Consider the following statements. In
2
a streamline flow of a liquid,
(d) Velocity head at the point B is
1. The kinetic energies of all particles
negligible.
arriving at a given point are same.
18. Consider the following statements:
2. The momenta of all particles
There is a small hole near the bottom
arriving at a given point are same.
of an open tank filled with water. The
3. The speed of particles is less than
speed of water ejected depends on
the critical velocity.
1. area of the hole
Which of the statements given above
2. density of liquid
are correct:
3. height of liquid from the hole
(a) 1 and 2 only
4. acceleration due to gravity
(b) 2 and 3 only
Which of the statements given above
(c) 1 and 3 only
are correct
(d) 1, 2 and 3
(a) 1 and 2 only
22. For cylindrical pipes, the Reynolds
(b) 1, 3 and 4 only
(c) 3 and 4 only number corresponding to the critical
speed is 2000. Water (viscosity
(d) 2, 3 and 4 only
coefficient 10–3 N s/m2) flows through
19. Consider the following statements:
a pipe of diameter 2 cm. What is the
1. Magnus effect is a consequence of
critical speed
Bernoulli’s principle
2. A cricketer, while spinning a ball (a) 4 cm/s (b) 10 cm/s
makes it to experience magnus (c) 12 cm/s (d) 100 cm/s
effect. 23. A lead shot of 1 mm diameter falls
Which of the statements given above through a long column of glycerine.
is/are correct The variation of its velocity v with
(a) 1 only distance covered is represented by
(b) 2 only
(c) Both 1 and 2
v v
(d) Neither 1 nor 2
20. A jar is filled with two non-mixing
liquids 1 and 2 having densities r1 and (a) Distance covered (b) Distance covered
r2 respectively. A solid ball made of a
material of density r3 is dropped in
v v
the jar. It comes to equilibrium in the
position shown in the figure. Which of
the following is true for r1, r2 and r3 (c) Distance covered (d) Distance covered
++6,'' 24. A block of ice floats on a liquid of
density 1.2 in a beaker, then level of
liquid when ice completely melt
Liquit 1 r1 (a) remains same
r3 (b) rises
r2
Liquit 2 (c) lowers
(d) (a), (b) or (c)
1138പWŚLJƐŝĐƐĨŽƌ//dͲ:͗DĞĐŚĂŶŝĐƐ//

25. A body floats in a liquid contained in 28. A multitube manometer is employed


a beaker. The whole system as shown to determine the pressure in a pipe.
falls freely under gravity. The upthrust For the levels in the manometers as
on the body due to the liquid is shown, compute the pressure in the
1,'' pipe. What would be the length of a
single mercury filled U-tube to record
this pressure?
Water Sp Gr = 1.0
Sp Gr = 0.00012
Air

40 cm
S S4 S3
50 cm
(a) zero
(b) equal to the weight of the liquid S2 S1
displaced
(c) equal to the weight of the body in
air
(d) equal to the weight of the Mercury
immersed position of the body Sp Gr = 13.6
26. A vessel contains oil (density = 0.8 gm/
cm3) over mercury (density = 13.6 gm/ (a) 193 cm (b) 165 cm
cm3). A homogeneous sphere floats (c) 121 cm (d) 153 cm
with half of its volume immersed in 29. A cup of water is placed in a car
mercury and the other half in oil. The under constant acceleration to the
density of the material of the sphere in left as shown. Inside the water there
gm/cm3 is ++6,'' is a small air bubble. The following
(a) 3.3 (b) 6.4 figures show the shape of the water
(c) 7.2 (d) 12.8 surface and the direction of motion of
27. A U-tube in which the cross-sectional the bubble as indicated by the arrow
area of the limb on the left is one on the bubble. Choose the correct one.
quarter, the limb on the right contains
mercury (density 13.6 g/cm3. The level a a
of mercury in the narrow limb is at a (a) (b)
distance of 36 cm from the upper end
of the tube. What will be the rise in the
level of mercury in the right limb if the a a
left limb is filled to the top with water (c) (d)
Water
30. An open water tanker moving on a
horizontal straight road has a cubical
block of cork floating over its surface.
Mercury If the tanker has an acceleration a,
then the acceleration of the cork w.r.t.
(a) 1.2 cm (b) 2.35 cm container is
(c) 0.56 cm (d) 0.8 cm
,LJĚƌŽƐƚĂƟĐƐŽƌ&ůƵŝĚ^ƚĂƟĐƐപ1139

F2
(a) F1 = pF2 (b) F1 =
a p
(c) F1 = p F2 (d) F1 = F2
34. The area of cross section of the wider
tube shown in figure is 800 cm2. If a
(a) a (b) a2/g mass of 12 kg is placed on the massless
piston, the difference in heights h in
(c) a2 + g 2 (d) zero the level of water in the two tubes is

31. The pressure at the bottom of a tank of 12 kg


h
water is 3P where P is the atmospheric
pressure. If the water is drawn out till
the level of water is lowered by one
fifth, the pressure at the bottom of the
tank will now be
(a) 2P (b) (13/5)P (a) 10 cm (b) 6 cm

(c) (8/5)P (d) (4/5)P (c) 15 cm (d) 2 cm

32. A cylindrical block of area of cross 35. By sucking through a straw, a student
section A and of material of density can reduce the pressure in his lungs
r is placed in a liquid of density one- to 750 mm of Hg (density = 13.6 gm/
third of density of block. The block cm3). Using the straw, he can drink
compresses a spring and compression water from a glass up to a maximum
in the spring is one-third of the length depth of
of the block. If acceleration due to (a) 10 cm (b) 75 cm
gravity is g, the spring constant of the (c) 13.6 cm (d) 1.36 cm
spring is
36. A triangular lamina of area A and
height h is immersed in a liquid of
density r in a vertical plane with its
base on the surface of the liquid. The
thrust on the lamina is
1 1
(a) Argh (b) Argh
(a) rAg (b) 2rAg 2 3
(c) 2rAg/3 (d) rAg/3 1 2
(c) Argh (d) Argh
33. Some liquid is filled in a cylindrical 6 3
vessel of radius R. Let F1 be the force
applied by the liquid on the bottom ‡˜‡Žʹ౨ 1PN[QPGQRVKQPKUEQTTGEV
of the cylinder. Now the same liquid 1. Two capillary tubes of the same length
is poured into a vessel of uniform but different radii r1 and r2 are fitted in
square cross-section of side R. Let F2 parallel to the bottom of a vessel. The
be the force applied by the liquid on pressure head is P. What should be the
the bottom of this new vessel (neglect radius of a single tube that can replace
atmospheric pressure). Then
1140പWŚLJƐŝĐƐĨŽƌ//dͲ:͗DĞĐŚĂŶŝĐƐ//

the two tubes so that the rate of flow is Ê h - 1ˆ


(a) Á mg (b) hmg
Ë h ˜¯
same as before
(a) r1 + r2 (b) r12 + r22
mg
(c) (d) (h – 1)mg
(c) r14 + r24 (d) None of these h-1
2. Two spheres of volume 250 cc each 5. A candle of diameter d is floating on
but of relative densities 0.8 and 1.2 a liquid in a cylindrical container of
are connected by a string and the diameter D (D > > d) as shown in the
combination is immersed in a liquid figure. If it is burning at the rate of 2
in vertical position as shown in the cm/hour, then the top of the candle
figure. The tension in the string is (g = will
10 m/s2)

Relative
density 0.8
L

d L
Relative
density 1.2
D
(a) 5.0 N (b) 0.5 N
(c) 1.0 N (d) 2.0 N (a) remain at the same height
3. A large tank filled with water to a (b) fall at the rate of 1 cm/hour
height h is to be emptied through a (c) fall at the rate of 2 cm/hour
small hole at the bottom. The ratio of (d) go up the rate of 1 cm/hour
time taken for the level of water to fall 6. A concrete sphere of radius R has
h h a cavity of radius r which is packed
from h to and to zero is with sawdust. The specific gravities of
2 2
1 concrete and sawdust are respectively
(a) 2 (b) 2.4 and 0.3 for this sphere to float with
2
its entire volume submerged under
1 water. Ratio of mass of concrete to
(c) 2 -1 (d)
2 -1 mass of sawdust will be
(a) 8 (b) 4
4. A solid sphere of density h (> 1) times
(c) 3 (d) zero
lighter than water is suspended in a
7. The volume of an air bubble becomes
water tank by a string tied to its base
three times as it rises from the bottom
as shown in the figure. If the mass of
of a lake to its surface. Assuming
the sphere is m, then the tension in the
atmospheric pressure to be 75 cm of
string is given by
Hg and the density of water to be 1/10
of the density of mercury, the depth of
the lake is
(a) 5 m (b) 10 m
(c) 15 m (d) 20 m
8. A body of density d1 is counterpoised
by Mg of wights of density d2 in air of
,LJĚƌŽƐƚĂƟĐƐŽƌ&ůƵŝĚ^ƚĂƟĐƐപ1141

density d. Then the true mass of the Water


body is h
Mercury
Ê dˆ
(a) M (b) M Á 1 - ˜
Ë d2 ¯

Ê dˆ M(1 - d/d2 ) n2 h h
(c) M Á 1 - ˜ (d) (a) (b)
Ë d1 ¯ (1 - d/d1 ) (n + 1)2 s 2
(n + 1)s
9. Two evacuated brass hemispheres
h h
of thin walls, each of radius 1 m are (c) (d)
pulled apart by exerting force F on (n + 1)2 s n2 s
each of them. Taking inside pressure 13. There are two identical small holes of
0.5 atm and the outside pressure as 1.0 area of cross section a on the opposite
atm, the value of F is sides of a tank containing a liquid of
density r. The difference in height
between the holes is h. The tank is
resting on a smooth horizontal surface.
F F
Horizontal force which will has to
be applied on the tank to keep it in
equilibrium is
(a) 1 ¥ 105 N
(b) 1.57 ¥ 105 N
(c) 3.14 ¥ 105 N
(d) 6.28 ¥ 105 N h
10. A hemispherical bowl just floats
without sinking in a liquid of density
1.2 ¥ 103 kg/m3. If the outer diameter
2gh
and the density of the bowl are 1 m (a) ghra (b)
and 2 ¥ 104 kg/m3 respectively, then ra
the inner diameter of the bowl will be
(c) 2 ragh (d) rgh
(a) 0.94 m (b) 0.97 m a
(c) 0.98 m (d) 0.99 m
14. A large open tank has two holes in
11. A log of wood of mass 120 kg floats in
the wall. One is a square hole of side
water. The weight that can be put on
L at a depth y from the top and the
the log to make it just sink, should be
other is a circular hole of radius R at
(density of wood = 6000 kg/m3)
a depth 4y from the top. When the
(a) 80 kg (b) 50 kg
tank is completely filled with water
(c) 60 kg (d) 30 kg
the quantities of water flowing out
12. Two communicating vessels contain
per second from both the holes are the
mercury. The diameter of one vessel
same. Then R is equal to
is n times larger than the diameter of
the other. A column of water of height L
h is poured into the left vessel. The (a) 2p L (b)
2p
mercury level will rise in the right-
hand vessel (s = relative density of L
(c) L (d)
mercury and r = density of water) by 2p
1142പWŚLJƐŝĐƐĨŽƌ//dͲ:͗DĞĐŚĂŶŝĐƐ//

15. Water is filled in a cylindrical container


to a height of 3 m. The ratio of the
cross-sectional area of the orifice and 3L/4 H/2
l
the beaker is 0.1. The square of the
speed of the liquid coming out from 2d H/2
the orifice is (g = 10 m/s2)

18. A hollow sphere of mass M and radius


R is immersed in a tank of water
3m (density rw). The sphere would float if
52.5 cm it were set free. The sphere is tied to the
bottom of the tank by two wires which
makes angle 45° with the horizontal as
(a) 50 m2/s2 (b) 50.5 m2/s2
2 2 shown in the figure. The tension T1 in
(c) 51 m /s (d) 52 m2/s2
the wire is
16. A wooden block, with a coin placed
on its top floats in water as shown in
figure. The distance l and h are shown
R
there. After some time the coin falls M
into the water. Then ++6,'' T1
45°45°
Coin

4
l p R3 rw g - Mg
(a) 3
h 2
2
(b) pR3rwg – Mg
3
(a) l decreases and h increases 4
(b) l increases and h decreases (c) pR3rwg – Mg
3
(c) Both l and h increase
4
(d) Both l and h decrease (d) pR3rwg + Mg
17. A homogeneous solid cylinder of 3
length L (L < H/2). Cross-sectional 19. A slender homogeneous rod of length
area A/5 is immersed such that it 2 L floats partly immersed in water,
floats with its axis vertical at the liquid- being supported by a string fastened to
liquid interface with length L/4 in the one of its ends, as shown. The specific
denser liquid as shown in the figure. gravity of the rod is 0.75. The length of
The lower density liquid is open to rod that extends out of water is
atmosphere having pressure P0. The
density D of solid is given by
,''
5 4
(a) d (b) d
4 5
2L
d
(c) d (d)
5
,LJĚƌŽƐƚĂƟĐƐŽƌ&ůƵŝĚ^ƚĂƟĐƐപ1143

1 water column being 200 mm. Find the


(a) L (b) L fall in height (in mm) of water level
2
1 due to opening of the orifice.
(c) L (d) 3L [Take atmospheric pressure = 1.0 ¥ 105
4
N/m2, density of water = 1000 kg/m3
20. A small spherical solid ball is dropped
and g = 10 m/s2. Neglect any effect of
from a great height in a viscous
surface tension]
liquid. Its journey in the liquid is best
(a) 5 mm (b) 6 mm
described in the diagram given below
(c) 2 mm (d) 1 mm
by the
23. Water is filled up to a height h in a
beaker of radius R as shown in the
A
Velocity (v)

B figure. The density of water is r, the


C surface tension of water is T and the
D atmospheric pressure is P0. Consider
Time (t) a vertical section ABCD of the water
column through a diameter of the
(a) curve A (b) curve B beaker. The force on water on one side
(c) curve C (d) curve D of this section by water on the other
21. A cone of radius R and height H is side of this section has magnitude
hanging inside a liquid of density r ++6,''
by means of a string as shown in the
figure. The force, due to the liquid 2R
acting on the slant surface of the cone B
is (neglect atmosphere pressure) A
h
C

(a) |2P0Rh + pR2rgh – 2RT|


H
(b) |2P0Rh + Rrgh2 – 2RT|
R
(c) |P0pR2 + Rrgh2 – 2RT|
(d) P0pR2 + Rrgh2 + 2RT|
(a) rpgHR2 (b) prHR2
24. An L-shaped glass tube is just
4 2 immersed in flowing water such that
(c) prHR2 (d) prgHR2
3 3 its opening is pointing against flowing
22. A cylindrical vessel of height 500 water. If the speed of water current is
mm has an orifice (small hole) at its v, then
bottom. The orifice is initially closed
and water is filled in it up to height H.
Now the top is completely sealed with h
a cap and the orifice at the bottom is
opened. Some water comes out from
the orifice and the water level in the v
vessel becomes steady with height of
1144പWŚLJƐŝĐƐĨŽƌ//dͲ:͗DĞĐŚĂŶŝĐƐ//

(a) the water in the tube rises to 28. Equal volumes of two immiscible
2 liquids of densities r and 2r are filled
v
height in a vessel as shown below. Two small
2g holes are punched at depth h/2 and
(b) the water in the tube rises to 3h/2 from the surface of lighter liquid.
g If v1 and v2 are the velocities of a flux
height
2v 2 at these two holes, then v1/v2 is
(c) the water in the tube does not rise
at all
h
(d) None of these r v1
25. A cubical block of wood 20 cm on a
side and density of 500 kg/m3 floating h 2r v2
on water. From its equilibrium floating
position, it is pushed further by 4 cm
into the water. What is the force needed 1 1
to keep the block in this new position? (a) (b)
2 2 2
g = 10 m/s2. '#/%'6
(a) 16 N (b) 32 N 1 1
(c) 40 N (d) 56 N (c) (d)
4 2
26. A uniform rod of density r is placed
29. A vessel has the shape shown in the
in a wide tank containing a liquid of
figure. Water, which has density of
density s (s > r). The depth of liquid
103 kg/m3, is filled in the vessel. The
in the tank is half the length of the
pressure due to liquid column at the
rod. The rod is in equilibrium, with its
bottom will be (take g = 10 m/s2)
lower end resting on the bottom of the
(area of bottom = 1 m2)
tank. In this position, the rod makes an
angle q with the horizontal. Then sin q
is equal to
1m
1 s 1s
(a) (b) 0.5 m
2 r 2r
0.1 m
r s Area = 1 m2
(c) (d)
s r
(a) 1.6 ¥ 103 N/m2
27. A cubical block is floating in a liquid
(b) 1.5 ¥ 104 N/m2
3
with of its volume immersed in (c) 1 ¥ 104 N/m2
4
(d) 1.6 ¥ 104 N/m2
the liquid. When the whole system 30. A large tank is filled with water
accelerates downwards with a net (density = 103 kg/m3). A small hole
acceleration of g/4, the fraction of is made at a depth 10 m below water
volume immersed in the liquid will be surface. The range of water issuing
1 1 out of the hole is R on the ground.
(a) (b)
2 4 What extra pressure must be applied
on the water surface so that the range
3 3
(c) (d) becomes 2R (take 1 atm = 105 Pa and g
4 8 = 10 m/s2)
,LJĚƌŽƐƚĂƟĐƐŽƌ&ůƵŝĚ^ƚĂƟĐƐപ1145

33. A body having volume V and density r


10 m is attached to the bottom of a container
as shown. Density of the liquid is d(>r).
Container has a constant upward
acceleration a. Tension in the string is

R
a
(a) 9 atm (b) 4 atm
(c) 5 atm (d) 3 atm
31. A fire hydrant delivers water of den- (a) V[dg – r(g + a)]
sity r at a volume rate Q. The water (b) V[(g + a)(d – r)
travels vertically upwards through (c) V(d – r)g
the hydrant and then does 90° turn to (d) None
emerge horizontally at speed v. The 34. A bent tube is of uniform cross section
pipe and nozzle have uniform cross has a liquid of density r (see figure).
section throughout. The force exerted The height of liquid in each limb of
by the water on the corner of the hy- the tube is h. The acceleration of the
drant is tube so that pressure due to liquid at
v A becomes zero is (BITSAT, 2013)

v
l h
B A
(a) rvQ (b) zero
(c) 2rvQ (d) 2rvQ (a) gh/l towards right
(b) 2gh/l towards right
32. A cubical box of wine has a small spout (c) gh/2l towards left
located in one of the bottom corners. (d) none of these
When the box is full and placed on a 35. A bent tube of uniform cross section
level surface, opening the spout results is mounted on a cart, which is
in a flow of wine with a initial speed accelerating towards right with
of v0 (see figure). When the box is half constant acceleration a. The total length
empty, someone tilts it at 45° so that of the liquid in the tube is 2 2l . The
the spout is at the lowest point (see level difference between two limbs is
figure). When the spout is opened the
wine will flow out with a speed of
45° 45°

v0 al 2al
(a) g (b)
g
al
(a) v0 (b) v0/2 (c) (d) none of these
2g
(c) v0/ 2 (d) v0/4 2
1146പWŚLJƐŝĐƐĨŽƌ//dͲ:͗DĞĐŚĂŶŝĐƐ//

36. A right circular cone of density r,


floats just immersed with its vertex
downwards in a vessel containing
two liquids of densities s1 and s2
respectively, the planes of separation h
of the two liquids cut off from the axis r
of the cone a fraction z of its length.
Find z. 2R
1/3 1/3
Ê r +s2 ˆ Ê r -s2 ˆ 39. The limbs of a glass U-tube are lowered
(a) Á (b) Á
Ë s 1 + s 2 ˜¯ Ë s 1 - s 2 ˜¯ into vessels A and B as shown in the
figure. Some air is pumped out through
1/2
Ê r -s2 ˆ Ê r -s2 ˆ a value, placed at the top of tube and
(c) Á (d) Á
Ë s 1 + s 2 ˜¯ Ë s 1 - s 2 ˜¯ then the value is closed. The liquid in
the left hand limb then rises to h1 = 20
37. A uniform wooden stick of length L, cm and in the right hand it rises to a
cross-sectional area A and density d is height h2 = 20 cm. Vessel A has water,
immersed in a liquid of density 4d. A then the density of liquid in vessel B is
small body of mass m and negligible
Valve
volume is attached at the lower end of
the rod so that the stick floats vertically
in stable equilibrium then
h1
h2
d

l 4d A B

m
(a) 2 ¥ 103 kg/m3
(b) 3 ¥ 103 kg/m3
(c) 4 ¥ 103 kg/m3
(a) m > dAL (b) m < dAL
(d) 4.5 ¥ 103 kg/m3
(c) m < dAL/2 (d) m < dAL/4
40. The pressure of air in the tank as
38. A hemispherical portion of radius
shown in figure is: (oil has density r1,
R is removed from the bottom of a
mercury has density rHg, density of air
cylinder of radius R. The volume of the
negligible small in comparison to that
remaining cylinder is V and its mass M.
of liquid and mercury. Atmospheric
It is suspended by a string in a liquid
pressure is P0.
of density r where it stays vertical. The
Air
upper surface of the cylinder is at a P0
depth h below the liquid surface. The
force on the bottom of the cylinder by
the liquid is ,'' Air l1
l2
(a) Mg Oil l3
(b) Mg – Vrg
(c) Mg + pR2hrg
Mercury
(d) rg(V + pR2h)
,LJĚƌŽƐƚĂƟĐƐŽƌ&ůƵŝĚ^ƚĂƟĐƐപ1147

(a) rgl1 + rHgl3 + P0 and its contents put together is M.


(b) rgl2 + rHggl3 If now all the fish start accelerating
upwards with an acceleration a, then
(c) r1g(l2 – l1) + rHggl3 + P0
the incorrect option(s) is/are
(d) none of these
(a) the weight recorded will be equal
41. A bird of mass 1.23 kg is able to hover to Mg
by imparting a downward velocity 10 (b) the weight reading will be less
m/s uniformly to air of density r kg/ than Mg
m3 over an effective area 0.1 m2. If the (c) the weight reading will be more
acceleration due to gravity is 10 m/s2, than Mg
then the magnitude of r in kg/m3 is (d) no conclusion can be drawn from
(a) 0.0123 (b) 0.123 the given information
(c) 1.23 (d) 1.32 3. A body of weight W volume V is
floating in liquid with V¢ of its volume
Multiple Correct Options inside liquid. When the container of
1. The spring balance A reads 2 kg with a the liquid is accelerated, then
block m suspended from it. A balance (a) buoyant force on the body is equal
B reads 5 kg when a beaker with liquid to W
is put on the pan of the balance. The (b) buoyant force on the body is
two balances are now so arranged that greater than W
the hanging mass is inside the liquid (c) body floats with volume V¢ inside
in the beaker as shown in the figure. liquid
In this situation ,'' (d) body floats with volume greater
than V¢ inside liquid
4. A cylindrical container has water and
A closed at the top. If v1, v2, v3 represent
the efflux velocities, then
m
Vacuum
B 6m
v1
3m
(a) the balance A will read more than v2
2 kg 3m
v3
(b) the balance B will read more than
5 kg
(c) the balance A will read less than (a) v1 = 2g ¥ 6 (b) v1 = 0
2 kg and B will read more than
5 kg (c) v2 = 2g ¥ 9 (d) v3 π 2 g ¥ 12
(d) the balance A and B will read 2 kg
and 5 kg respectively 5. An upright U-tube manometer with
2. A completely filled closed aquarium its limbs 0.6 m high and spaced
is kept on a weighing machine. It can 0.3 m apart contains a liquid to a height
be assumed that the density of the of 0.4 m in each limb. If the U-tube is
fish is greater than the density of the rotated at 10 radians/second about a
water. The total mass of the aquarium vertical axis at 0.1 m from one limb.
Choose the correct options
1148പWŚLJƐŝĐƐĨŽƌ//dͲ:͗DĞĐŚĂŶŝĐƐ//

10 rad/s

z2
0.4m
z1
zmin 12

0.1m 0.2m
9. A piston is pressed with a force F on
(a) z1 = 0.324 (b) z2 = 0.477 a hydroulic cylinder containing water
(c) zmin = 0.273 (d) z1 + z2 = 0.8 (r 103 kg/m3). The cross-sectional area
6. In a streamline flow of the cylinder is A = 100 cm2. The
water is forced into pipe with a cross-
(a) the speed of a particle always
sectional area of 1 cm2 that rises to a
remains same
height h = 50 cm (atmospheric pressure
(b) the velocity of a particle always
P0 = 105 N/m2)
remains same
v
(c) the kinetic energies of all the
particles arriving at a given point
h
are the same F
(d) the momenta of all the particles
arriving at a given point are the
same (a) Force F required to make water
7. In this figure, an ideal liquid flows eject with a speed of 10 m/s is
through the tube, which is of uniform 550 N.
cross section. The liquid has velocities (b) Force F required to make water
vA and vB and pressure PA and PB at eject with a speed of 10 m/s at the
points A and B respectively top is 5500 N.
(c) The pressure inside the cylinder
A at the bottom when water ejects
at 10 m/s is 1.55 ¥ 105 Pa.
(d) none of the above
10. A tank is filled up to a height h with
B a liquid and is placed on a platform
of height h from the ground. To get
(a) vA = vB (b) vB > vA maximum range xm a small hole is
(c) PA = PB (d) PB > PA punched at a distance of y from the
free surface of the liquid. Then
8. Ideal fluid flows along a tube of
uniform cross section, located in a
horizontal plane and bent as shown h
y
in figure. The flow is steady; 1 and 2
are two points and v1 and v2 are the
velocities of flow at these points, then
h
(a) P1 < P2 (b) P1 > P2
(c) v1 < v2 (d) v1 > v2
xm
,LJĚƌŽƐƚĂƟĐƐŽƌ&ůƵŝĚ^ƚĂƟĐƐപ1149

(a) xm = 2h (b) xm = 1.5 h levels of the liquid in the two vertical


(c) y = h (d) y = 0.75 h tubes is h
11. Water coming out of a horizontal tube
at a speed v strikes normally a vertical h
wall close to the mouth of the tube and
falls down vertically after the impact. A1 v1 A2
When the speed of water is increased v2
to 2v.
(a) the thrust exerted by the water on (a) the volume of the liquid flowing
the wall will be doubled through the tube in unit time is
(b) the thrust exerted by the water on A 1v 1
the wall will be four times (b) v2 – v1 = 2gh
(c) the energy lost per second by
water strikes the wall will also be (c) v22 – v12 = 2gh
four times (d) the energy per unit mass of the
(d) the energy lost per second liquid is the same in both sections
by water striking the wall be of the tube
increased eight times
12. Figure shows a siphon. Choose the Passages and Matrix Matching
correct statement: 2CUUCIGHQT3Ō
Water stands at a depth of 1 m in an enclosed
h1
tank whose side walls are vertical. The space
above the water surface contains air at a
h=0 2
h2 gauge pressure of 8 ¥ 105 Pa. The tank rests
1 h3 on a platform 2 m above the floor. A hole of
3 cross-sectional area 1 cm2 is made in one of
the side walls just above the bottom of the
tank.
(a) Siphon works when h3 > 0
1. The horizontal distance at which
(b) Pressure at point 2 is P2 = P0 – rgh3
stream of water strike the floor is
(c) Pressure at point 3 is P0
(a) 12.3 m (b) 25.2 m
(d) None of the above
(c) 30.80 m (d) 35.5 m
(P0 = atmospheric pressure)
2. The vertical force exerted by the stream
13. A beaker, filled with water, is
on the floor is
accelerated a m/s2 in +x direction. The
(a) 25.2 N
surface of water shall make an angle
(b) 50.4 N
(a) tan–1 (a/g) backwards
(c) 75.6 N
(b) tan–1 (a/g) forwards
(d) none of these
(c) cot–1 (g/a) backwards
3. The horizontal force exerted on
(d) cot–1 (g/a) forwards
the tank is (Assume the water level
14. A liquid flows through a horizontal
and pressure in the tank to remain
tube. The velocities of the liquid in
constant.)
the two sections, which have areas of
(a) 80 N (b) 123 N
cross sections A1 and A2, are v1 and
(c) 160 N (d) 182 N
v2 respectively. The difference in the
1150പWŚLJƐŝĐƐĨŽƌ//dͲ:͗DĞĐŚĂŶŝĐƐ//

2CUUCIGHQT3Ō (a) 1.68 cm (b) 3.16 cm


A common method for testing gold for purity (c) 4.20 cm (d) 5.20 cm
is to measure its density by weighing it in air 2CUUCIGHQT3Ō
and then in water. In an era of rising gold
When an object moves in a viscous fluid,
prices, a swindler proposed to make a fake
it experiences a viscous force, which is
gold ingot by using a hollow slab of iridium
proportional to the speed of the object.
(density 22.5 g/cm3) and plating it with a
According to the stoke, the viscous force on
thin layer of gold (density 19.3 g/cm3)
a spherical body of radius r1, moving with
4. To make fake ingot of total mass 0.5 kg. a speed is, 6 phrv. According to Poisualli the
What should be the total volume? rate of flow given by
(a) 12.4 cm3 (b) 25.9 cm3
3 p r 4 DP
(c) 18.6 cm (d) 27.3 cm3 Q=
5. The volume of interior air space 8hL
(cavity) in the ingot is Here h is the coefficient of viscosity and
(a) 3.68 cm3 (b) 4.24 cm3 DP is the pressure drop in a length L of the
3
(c) 8.24 cm (d) 12.20 cm3 pipe, and r is the radius of the pipe. For the
6. Select the correct one laminar flow in a pipe, the Reynolds number
(a) iridium is cheaper than gold is given by
(b) gold is cheaper than iridium rvD
(c) cost cannot be decided by these R=
h
observations
(d) none of the above Here r is the density of the liquid, D is the
diameter of the pipe and v is the average
2CUUCIGHQT3Ō velocity of flow. For laminar flow in a pipe,
The section of pipe shown in figure has a the Reynolds number should be less than or
cross section of 40 cm2 at the wider portion equal to 2000.
and 10 cm2 at the constriction. The discharge 10. The viscous force on a glass sphere of
of water from the pipe is 3000 cm3/s. radius r = 1 mm falling through water
(h = 10–3 Pa-s) with a speed of 3 m/s
is
40 cm2 10cm2
(a) 2.5 ¥ 10–5 N
(b) 2.7 ¥ 10–2 N
h
(c) 3.7 ¥ 10–3 N
(d) 5.6 ¥ 10–5 N
11. Blood vessel is 0.10 m in length and
7. The velocity at the wide portion is has a radius of 1.5 ¥ 10–3 m. Blood
(a) 0.75 m/s (b) 3.0 m/s flows at the rate of 10–7 m3/s through
(c) 3.75 m/s (d) 4.50 m/s this vessel. The pressure difference
8. The pressure difference between wider that must be maintained in this flow,
and narrow sections is between the two ends of the vessel
(a) 4000 N/m2 is 20 Pa. What is the coefficient of
(b) 4219 N/m2 viscosity of the blood?
(c) 4872 N/m2 (a) 1 ¥ 10–3 Pa-s
(d) 5200 N/m2 (b) 2 ¥ 10–3 Pa-s
9. The difference in height of mercury (c) 4 ¥ 10–3 Pa-s
column in the U-tube is (d) 5 ¥ 10–4 Pa-s
,LJĚƌŽƐƚĂƟĐƐŽƌ&ůƵŝĚ^ƚĂƟĐƐപ1151

12. Calculate the highest average speed water. This works because fat tends to float
that blood (r = 1000 kg/m3) could on water as it is less dense than water. On
have and still remain in laminar flow the other hand muscle and bond tend to
when it flows through the aorta (r = 8 sink as they are more dense. Knowing your
¥ 10–3 m). Take h = 4 ¥ 10–3 Pa-s “weight” under water as well as your real
(a) 0.5 m/s (b) 1.0 m/s weight out of water, the percentage of your
(c) 1.5 m/s (d) 2.0 m/s body’s volume that is made up of fat can
easily be estimated. This is only an estimate
2CUUCIGHQT3Ō
since it assumes that your body is made up
A uniform solid cylinder of density 0.8 g/ of only two substances, fat (low density) and
cm3 floats in equilibrium in a combination of everything else (high density). The “weight”
two non-mixing liquids A and B with its axis is measured by spring balance both inside the
vertical. The densities of liquids A and B are outside the water. Quotes are placed around
0.7 g/cm3 and 1.2 g/cm3 respectively. The weight to indicate that the measurement
height of liquid A is hA = 1.2 cm. The length read on the scale is not your true weight, i.e.,
of the part of the cylinder in liquid B is hB = the force applied to you body by gravity, but
0.8 cm. ,'' a measurement of the net downward force
on the scale.
Air
h 16. Ram and Shyam have the same weight
when measured outside the water.
When measured under water, it is
hA found that weight of Ram is more than
A that of Shyam, then we can say that
(a) Ram has more fat content than
B hB Shyam
(b) Shyam has more fat content than
Ram
13. The total force exerted by liquid A on (c) Ram and Shyam both have the
the cylinder is same fat content
(a) 200 N (b) 350 N (d) None of these
(c) 450 N (d) zero 17. A person of mass 165 kg having one-
14. The length h of the cylinder in air is fourth of his volume consisting of
(a) 0.25 cm (b) 0.50 cm fat (relative density 0.4) and rest of
(c) 0.75 cm (d) 1.20 cm the volume consisting of everything
15. The cylinder is depressed in such a else (average relative density 4/3) is
way that its top surface is just below weighed under water by the spring
the upper surface of liquid A and balance. The reading shown by the
then released. The acceleration of the spring balance is
cylinder just after release is (a) 15 kg
g
(a) g m/s2 (b) m/s2 (b) 65 kg
2
g g (c) 150 kg
(c) m/s2 (d) m/s2 (d) None of these
6 8
18. Suppose that Ram is floating in
2CUUCIGHQT3Ō water with two-thirds of his volume
One way of measuring a person’s body immersed. Now the system is taken
fat content is by “weighing” them under in a lift which is accelerating upward
1152പWŚLJƐŝĐƐĨŽƌ//dͲ:͗DĞĐŚĂŶŝĐƐ//

with acceleration g/3. The new fraction a1


immersed is h0 a2Plug
(a) one-third v2
(b) half
(c) two-thirds
(d) three-fourths H = 5m Platform

2CUUCIGHQT3Ō
v3
A cylinder of radius R is kept embedded
along the wall of a dam as shown. Take 22. Initial speed with which the water
density of water as r. Take length as L. flows from the orifice is
(a) 5 m/s (b) 10 m/s
R (c) 12 m/s (d) 15 m/s
23. Initial speed with which the water
R strikes the ground is
(a) 10 m/s (b) 14.1 m/s
(c) 16.4 m/s (d) 18.2 m/s
24. Time taken to empty the tank to half
its original value is
19. The vertical force exerted by water on
the cylinder is (a) 444 s (b) 512 s
(a) rpR2Lg (c) 628 s (d) 942 s
(b) rpR2Lg/2 2CUUCIGHQT3Ō
(c) zero
A container of large uniform cross-sectional
(d) None of these
area A resting on a horizontal surface
20. The net torque exerted by liquid on the
holds two immiscible, non-viscous and
cylinder is
incompressible liquids of densities d and 2
2 rR3 Lg rR3 Lg d, each of height H/2 as shown. The lower
(a) (b)
3 3 density liquid is open to the atmosphere
having pressure P0. A tiny hole of area
rR3 Lg s(s << A) is punched on the vertical side of
(c) (d) 0
2 the container at a height h (h < H/2).
21. The force exerted by liquid on the Determine:
cylinder in horizontal direction is
[neglect atmospheric pressure]
d
(a) 2R2rgL (b) R2rgL H/2

(c) 4R2rgL (d) 1.61 R2rgL S

2CUUCIGHQT3Ō H/2 2d h
A cylindrical, tank 1 m in radius, rests on a x
platform 5 m high. Initially, the tank is filled
with water to a height of 5 m. A plug whose 25. The initial speed of efflux of the liquid
area is 10–4 m2, is removed from an orifice on at the hole is
the side of the tank at the bottom. Calculate
the following.
,LJĚƌŽƐƚĂƟĐƐŽƌ&ůƵŝĚ^ƚĂƟĐƐപ1153

(a) 2gH 28. The value of PB – PA is


(a) rga (b) 2rga
(b) 2 g( 3 H - 4 h)
(c) 3rga (d) 4rga
g 29. The force exerted by liquid r on liquid
(c) ( 3 H - 4 h)
2 2r at the interface is
(d) g( H - 3 h)
Ê 9ˆ Ê 29 ˆ
(a) rSag Á ˜ (b) rSag Á ˜
26. The horizontal distance x travelled by Ë 4¯ Ë 3¯
the liquid initially is
(a) ( 3 H - 4 h )h Ê 9ˆ Ê 29 ˆ
(c) rSag Á ˜ (d) rSag Á ˜
Ë 10 ¯ Ë 10 ¯
(b) ( 4 H - 3 h)h
(c) 150 kg 30. The value of w is
(d) None of these
27. The height h at which the hole should 9g 9g
be punched so that the liquid travels (a) (b)
4a 32 a
the maximum distance xm is
(a) 3H/4
9g 9g
(b) H/2 (c) (d)
(c) 3H/8 8a 16 a
(d) None of these
2CUUCIGHQT3Ō 2CUUCIGHQT3Ō
A U-tube containing two different liquids A cubical beaker contains two immiscible
of densities r and 2r is fixed vertically liquids of density r and 2r each, filled upto
on a rotating table about a vertical axis height h as shown in figure.
passing through the centre of the table. The
interface of two liquids of densities r and 2r
r h
respectively lies at the point A in a U-tube
8 2r h
at rest height of liquid column above A is
3
a, where AB = 2a. The cross-sectional area of 31. The distance of point of application
the tube is S. Now the table is whirled with of the force due to the liquids on the
angular velocity w about a vertical axis as vertical wall of the beaker from the
shown in figure such that the interface of bottom is
2 (a) h (b) h/3
the liquids shifts towards B by a, then at
3 (c) 3h/2 (d) 3h/5
this instant
32. The beaker is given a horizontal
w acceleration a. The liquid of density
r finally makes an angle q1 with
r 2r the horizontal while the liquid of
A B
a 2a density 2r makes an angle q2 with the
horizontal, then
1154പWŚLJƐŝĐƐĨŽƌ//dͲ:͗DĞĐŚĂŶŝĐƐ//

q1 such that the height of water


above the top of the block is h1
a (measured from the top of the
q2
block).
2TQEGUU The water is removed from the
(a) q1 < q2 tank to a height h2 (measured
(b) q1 > q2 from the bottom of the block), as
(c) q1 = q2 shown in the figure. The height
(d) none of these h2 is smaller than h (height of
33. Two holes are made in the beaker as the block) and thus the block is
shown. Find the initial acceleration of exposed to the atmosphere.
the beaker. The area of cross section of 34. Find the minimum value of height h1
beaker is A0 and that of each hole is A. (in process 1), for which the block just
Neglect the mass of the beaker. starts to move up?
2h 5h
(a) (b)
h/2 3 4
r
h/2
h/2 5h 5h
2r (c) (d)
h/2 3 2
3A 35. Find the height of the water level h2 (in
(a) g
A0 process 2), for which the block remains
Ag in its original position without the
(b)
A0 application of any external force
5A h 4h
(c) g (a) (b)
3 A0 3 9
(d) none of these 2h
(c) (d) h
3
2CUUCIGHQT3Ō
36. In Process 2, if h2 is further decreased,
A cylindrical tank has a hole of diameter then
2r in its bottom. The hole is covered with
a wooden cylindrical block of diameter 4r, (a) cylinder will not move up and
height h and density r/3. ,'' remains at its original position
h
(b) for h2 = , cylinder again starts
h1 3
4r
moving up
h2 h
h
(c) for h2 = , cylinder again starts
4
r
moving up
2r
h
(d) for h2 = , cylinder again starts
2TQEGUU Initially, the tank is filled with 5
water of density r to a height moving up
,LJĚƌŽƐƚĂƟĐƐŽƌ&ůƵŝĚ^ƚĂƟĐƐപ1155

Matrix Matching
37. Match Column I (property at fluid) with Column II (law of fluid motion) and select
the correct answer:
Column I Column II
(a) Volume rate of flow (p) Bernoulli’s theorem
(b) Viscous drag (q) Torricelli’s theorem
(c) Speed of efflux (exit) (r) Stokes’ law
(d) Pressure difference between two points (s) Poisuilli’s law points in a
flow tube
38. The vessel has two sections of areas of cross sections A1 and A2. A liquid of density
fills both the sections, up to a height in each. Neglect atmospheric pressure.
Column I Column II
(a) The pressure at the base of the vessel (p) 2hrgA2
(b) The force exerted by the liquid on the base of the vessel (q) 2hrg
(c) The weight of the liquid is less than (r) hrg (A2 – A1)
(d) Downward force on the liquid by the walls of the (s) 2hrgA1
vessel at the level x
A1

A2 x

h
r

39. Figure shows a siphon. It is a long pipe which is used to drain water from the reservoir
at higher level to a reservoir at lower level. Regarding the siphon match the following
columns:
Summit

Inlet leg 4
h1
Pa
3

2
1
h2

5
1156പWŚLJƐŝĐƐĨŽƌ//dͲ:͗DĞĐŚĂŶŝĐƐ//

Column I Column II
(a) Pressure is more than the atmospheric pressure at (p) 1
(b) Pressure is less than the atmospheric pressure at (q) 2
(c) Pressure is highest of all the five points at (r) 3
(d) Pressure is least of all the five points (s) 4
(t) 5
40. Column II shows five systems in which two objects are labelled as X and Y. Also in
each case a point P is shown.
Column I gives some statements about X and/or Y. Match these statements to the
appropriate system(s) from Column II. ++6,''
Column I Column II
A. The force exerted (p) Y Block Y of mass M left on a fixed
by X and Y has a x inclined plane X, slides on it with a
magnitude Mg constants velocity.
P

B. The gravitational (q) P Two ring magnets Y and Z, each of


potential energy of mass M, are kept in frictionless ver-
X is continuously Z tical plastic stand so that they repel
y
increasing each other. Y rests on the base X and
X
Z hangs in air in equilibrium. P is
the topmost point of the stand on the
common axis of the two rings. The
whole system is in a lift that is going
up with a constant velocity.
C. Mechanical energy of (r) A pulley Y of mass m0 is fixed to a table
the system X + Y is P through a clamp X. A block of mass
continuously decreas- Y M hang from a string that goes over
ing X the pulley and is fixed at point P of
the table. The whole system is kept
in a lift that is going down with a
constant velocity.
D. The torque of the (s) A sphere Y of mass M is put in a non-
weight of Y about viscous liquid X kept in a container
Y
point P is zero at rest. The sphere is released and it
X moves down in the liquid.
P

(t) A sphere Y of mass M is falling with


its terminal velocity in a viscous
Y liquid X kept in a container.
X

P
,LJĚƌŽƐƚĂƟĐƐŽƌ&ůƵŝĚ^ƚĂƟĐƐപ1157

Answers Key
.GXGN
1. (d) 2. (b) 3. (d) 4. (a) 5. (c) 6. (c) 7. (a) 8. (a)
9. (c) 10. (b) 11. (d) 12. (b) 13. (c) 14. (a) 15. (a) 16. (a)
17. (c) 18. (c) 19. (c) 20. (c) 21. (d) 22. (b) 23. (a) 24. (b)
25. (a) 26. (c) 27. (c) 28. (d) 29. (d) 30. (d) 31. (b) 32. (b)
33. (d) 34. (c) 35. (c) 36. (a)
.GXGN
1. (c) 2. (b) 3. (c) 4. (d) 5. (b) 6. (b) 7. (c) 8. (d)
9. (b) 10. (c) 11. (a) 12. (b) 13. (c) 14. (b) 15. (a) 16. (d)
17. (a) 18. (a) 19. (a) 20. (c) 21. (d) 22. (b) 23. (b) 24. (a)
25. (a) 26. (a) 27. (c) 28. (d) 29. (d) 30. (d) 31. (d) 32. (d)
33. (b) 34. (b) 35. (b) 36. (b) 37. (a) 38. (d) 39. (a) 40. (a)
41. (c)
/WNVKRNG%QTTGEV1RVKQPU
1. (b, c) 2. (a, b, d) 3. (b, c) 4. (b, d)
5. (a, b, c, d) 6. (c, d) 7. (a, c) 8. (b, c)
9. (a, c) 10. (a, c) 11. (b, d) 12. (a, b, c)
13. (a, c) 14. (a, c, d)
2CUUCIGUCPF/CVTKZ/CVEJKPI
1. (b) 2. (a) 3. (c) 4. (b) 5. (a) 6. (a) 7. (a) 8. (b)
9. (b) 10. (d) 11. (c) 12. (a) 13. (d) 14. (a) 15. (c) 16. (b)
17. (a) 18. (c) 19. (b) 20. (d) 21. (c) 22. (b) 23. (b) 24. (d)
25. (c) 26. (a) 27. (c) 28. (b) 29. (a) 30. (b) 31. (d) 32. (c)
33. (c) 34. (c) 35. (b) 36. (a) 37. A Æ (s); B Æ (r); C Æ (q); D Æ (p)
38. A Æ (q); B Æ (p); C Æ (p); D Æ (r) 39. A Æ (p); B Æ (q, r, s); C Æ (p); D Æ (r)
40. A Æ (p. t); B Æ (q, s, t); C Æ (p, r, t); D Æ (q)

  


Level 1
1. (d) As, P = rgh, so pressure in both the tanks at the bottom is same.
3m 18 d
2. (b) rmit = =
Êm m mˆ 11
ÁË + + ˜
d 2d 3d ¯

3. (d) 10 h
v
A B

h + y = 10 cm ...(i)
PA = PB
1158പWŚLJƐŝĐƐĨŽƌ//dͲ:͗DĞĐŚĂŶŝĐƐ//

or 1.3g ¥ 10 = 13.6 ¥ g ¥ y + 0.8 ¥ g ¥ h


or 13 = 13.6 y + 0.8 h ...(ii)
After solving above equations, we get
h = 9.6 cm.
4. (a) Because of rightward acceleration, the water level at the backward sides tends to
increase and so pressure at B will increase and at C will decrease.
M M
5. (c) The volume of ice V = and that of water . Therefore, decrease in volume
r s
M M È1 1˘
= - = MÍ - ˙
r s Îr s ˚
6. (c) Both the points are exposed to atmosphere, so they have atmospheric pressure.
7. (a) For floating
mg = Vrg
m
\ V=
r
Thus, submerged volume is free from acceleration due to gravity.
8. (a) Previously buoyant force, Fb = 5g. When 3.5 kg is put in that place, it has an
unbalanced force in upward direction.
9. (c) The velocity of ball just penetration the water v = 2gh . This to be constant.

Ê 4 3ˆ Ê 4 3ˆ
ÁË p r ˜¯ r g = 6phr( 2 gh ) + ÁË p r ˜¯ rw g
3 3
2
2 4 Ê r - 1ˆ
h= r g
81 ÁË h ˜¯
10. (b) For maximum range,
H 90
h= = = 45 cm
2 2
11. (d) A1v1 = A2 v2
A1v1 4.20 ¥ 5.18
\ v2 = = = 2.86 m/s
A2 7.60
12. (b) Decrease in pressure energy is equal to increase in kinetic energy of water, so
1 2
rv = (P1 – P2)
2
1
or ¥ 1000 ¥ v 2 = (3.5 – 3) ¥ 105
2
\ v = 10 m/s
13. (c) Av = A1v1 + A2v2
or A ¥ 3 = A ¥ 1.5 + 1.5A ¥ v
\ v = 1 m/s
,LJĚƌŽƐƚĂƟĐƐŽƌ&ůƵŝĚ^ƚĂƟĐƐപ1159

m
14. (a) The volume of liquid displaced V = , does not depend on g so it remains as
such. r
15. (a) The water levels in A and B goes up. But level in B rises greater than A.
16. (a) For dynamic lift on aeroplane,
1
P + rv 2 = constant
2
17. (c) For the static liquid, pressure at B will be, P = rgH/2. Due to velocity of liquid at
1
B, the pressure at the point will decrease, so that P + rv 2 = constant.
2
18. (c) Velocity of efflux, ve = 2gh , clearly ve depends on g and h.
19. (c) Go to theory.
V V
20. (c) Vr3g = r1 g + r2 g
2 2
r1 + r2
or r3 = ,
2
Clearly r1 < r3 < r2.
21. (d) For streamline flow, all are correct
rvd
22. (b) RN =
h
È 1000 ¥ v ¥ (2 ¥ 10 - 2 ) ˘
or 2000 = Í 10 - 3
˙
ÍÎ ˙˚

\ v = 0.1 m/s
23. (a) The velocity of the body becomes constant after travelling for some distance.
24. (b) mg = Vrlg
m m
or V= =
rl 1.2
When ice melt, the volume of water formed
m m
V¢ = = =m
rw 1
Clearly V¢ > V, so level of liquid will rise.
25. (a) If V is the volume of liquid displaced then,
Fb = Vr(g – a) = Vr(g – g) = 0

V/2 roil = 0.8


26. (c) rHg = 13.6
V/2

V V
mg = roil g + rHg g rHgg
2 2
1160പWŚLJƐŝĐƐĨŽƌ//dͲ:͗DĞĐŚĂŶŝĐƐ//

0.8 ¥ g V
or Vrg = V ¥ + ¥ 13.6 ¥ g
2 2
\ r = 7.2 g/cm3
27. (c) Ax = (4A)y
or x=4y y

From Pascal’s law A
x
B
PA = PB
or rwg(36 + x) = rHgg(x + y)
or 1 ¥ g(36 + 4y) = 13.6 ¥ g (4y + y)
\ y = 0.56 cm
50
28. (d) Ps1 = 50 cm mercury = ¥ 13.6 = 6.8m WG
100
(WG Æ water gauge)
Ps2 = Ps1 = 6.8m WG
Ps3 = 6.8 – 0.5 = 6.3m WG
Equating Ps4 to Ps3 etc.
Ps = 6.3 ¥ 2 = 12.6m WG
50 90
Ppipe = 12.6 + ¥ 13.6 - 0.0012 ¥
100 100
= 19.4 m WG
= 19.4 ¥ 1000 ¥ 9.81 = 190 ¥ 103 N/m2
= 190 kN/m2 (gauge)
The pressure, 19.4 m WG equals 1.43 m mercury gauge. Allowing, say 10 cm to
stay in the bottom U-space, the single U-tube mercury manometer would be 153
cm long.
29. (d) The pressure on right side of the bubble is greater than the left, and so bubble will
have the shape as in case (d).
30. (d) The acceleration of the cork is also a w.r.t. the ground observer, so acceleration
w.r.t. water becomes zero.
31. (b) The pressure of liquid = 3P – P = 2P
4 13P
Thus, in the next case pressure, P¢ = ¥ 2P + P =
5 5
32. (b) For the equilibrium of the block
mg = kx + Vrg
r
r(Ah)g = k(h/3) + (Ah) g
\ k = 2rAg 3

33. (d) pR2h1 = R2h2


\ ph1 = h2 h1 h2
,LJĚƌŽƐƚĂƟĐƐŽƌ&ůƵŝĚ^ƚĂƟĐƐപ1161

F1 r gh1 ¥ p R2
F2 = r gh2 ¥ R2
p h1
= =1
h2
34. (c) From Pascal’s law
mg
rwg ¥ h =
A
Ê 12 ˆ
or h= Á = 0.15 m
Ë 800 ¥ 10 - 4 ¥ 10 3 ˜¯

35. (c) The pressure difference between the lungs and atmosphere,
= 760 – 750 = 10 mm of Hg
\ hrwg = 1 ¥ rHg ¥ g
rHg
or h= = 13.6 cm.
rw
rgh
36. (a) The thrust, F = Pav ¥ A = A
2

Level 2
1. (c) Q1 + Q2 = Q
p pr14 p pr24 p pr 4
+ =
8hl 8hl 8hl

or r4 = r14 + r24
2. (b) T + 0.8 ¥ 250 ¥ 10–3g = 250 dlg
T + 250 dlg = 1.2 ¥ 250 ¥ 10–3 g
Solving, T = 0.5 N
3. (c) t = k( h1 - h2 )

\ t1 = k( h - h/2 )

and t2 = k( h/2 - 0)
t1
\ = 2 -1
t2
4. (d) T + mg = Fb
\ T = Fb – mg
= Vrwg – mg
1162പWŚLJƐŝĐƐĨŽƌ//dͲ:͗DĞĐŚĂŶŝĐƐ//

Fb

T
mg

m
= rw g - mg
( rw/h)

= (h – 1)mg.
5. (b) Initially, mg = (AL)Pwg
or r(A ¥ 2L)g = (AL)rwg ...(i)
After one hour,
r[A ¥ (2L – 2)]g = (Al)rwg ...(ii)
Dividing (ii) by (i), we get
l = (L – 1)

6. (b)

r Concrete

(mconcrete + msawdust)g = Vrwg


4 4 4
2.4 ¥ p (R3 - r 3 ) + 0.3 ¥ p r 3 = p R3 ¥ 1
3 3 3
3 3
or R3 = r
2
4
rconcrete p (R3 - r 3 )
mconcrete 3
\ =
msawdust 4
rsawdust p r 3
3
Ê 3 3 3ˆ
r -r ˜
2.4 ÁË 2 ¯
= 3
=4
0.3 r
,LJĚƌŽƐƚĂƟĐƐŽƌ&ůƵŝĚ^ƚĂƟĐƐപ1163

7. (c)

3V, Pa

V, (rgh + Pa)

3V ¥ 75 = V(75 + h)
\ h = 150 cm of Hg
or h = 1.50 ¥ 10 = 15 m of water
8. (d) mg – Vdg = Mg – V¢dg
m M
or mg - dg = Mg - dg
d1 d2
Ê dˆ
MÁ1- ˜
Ë d2 ¯
\ m =
Ê dˆ
ÁË 1 - d ˜¯
1

9. (b) F = P ¥ pR2 = (1 – 0.5) ¥ 105 ¥ p(1)2


= 1.57 ¥ 105 N
10. (c)

mg = Vrlg
4 4
or p(R3 – r3)rg = pR3 ¥ rlg
3 3
or (R3 – r3) ¥ 2 ¥ 104 = 1.2 ¥ 103 R3
or (0.53 – r3) ¥ 20 = 1.2 ¥ (0.5)3
or r = 0.98 m.
11. (a) If m is the mass on the log, then
mg + 120g = Vrwg
120
or m + 120 = ¥ 1000
600
\ m = 80 kg
1164പWŚLJƐŝĐƐĨŽƌ//dͲ:͗DĞĐŚĂŶŝĐƐ//

12. (b)

h y
x
A B

p d2 p(nd)2
x¥ = y
4 4
\ x = n 2y
From Pascal’s law
rwgh = rHgg(x + y)
or 1 ¥ gh = sg(n2y + y)
h
\ y =
s(1 + n2 )
13. (c)

y
v1

v2

Force F = rQv = rav2


= ra ¥ 2gh
Thus, net force = F2 – F1
= rav22 – rav12
= ra(v22 – v12) ra[2g(y + h) – 2gy]
= ra ¥ 2gh
14. (b) Rate of flow, Q = av
= L2 ¥ 2 gy = p R2 ¥ 2 g( 4 y )
L
\ R =
2p
15. (a) If v2 is the required speed then
A 1v 1 = A 2v 2
A2
or v1 = v2 = 0.1 v2 ...(i)
A1
,LJĚƌŽƐƚĂƟĐƐŽƌ&ůƵŝĚ^ƚĂƟĐƐപ1165

From Bernoulli’s equation, we have

1 1
Pa + rv12 + r g(3 - 0.525) = Pa + rv22 + 0 ...(ii)
2 2
After solving above equations, we get
v22 = 50 m2/s2
16. (d) Suppose m and M are the masses of coin and block respectively, then
(M + m)g = Vrwg

M+m
\ V =
rw
When coin falls into water, volume of water displaced
M m
V¢ = +
rw rcoin

As rcoin > rw, \ V¢ < V.


17. (a) mg = F1 + F2
L 3L
or (AL)Dg = A ¥ ¥ 2dg + A ¥ ¥ dg
4 4
5d
\ D =
4
18. (a) For the equilibrium of the sphere,
Mg + 2T1 cos 45° = Fb

Ê4 ˆ
= Á p R 3 ˜ rw g
Ë3 ¯

È4 3 ˘
Í 3 p R rw g - Mg ˙
\ T1 = Í ˙
Í 3 ˙
Î ˚
19. (a)

Fh
P
q
y
W

W = (2LA) ¥ 0.75g, Fb = (Ay) ¥ 1 ¥ g


1166പWŚLJƐŝĐƐĨŽƌ//dͲ:͗DĞĐŚĂŶŝĐƐ//

As rod is in equilibrium, so StP = 0


or W ¥ L cos q – Fb ¥ (2L – y/2)cos q = 0
After substituting and simplifying, we get
y =L
So, the length of the rod out of water is L.
20. (c) When ball falls from great height, its initial velocity before entering into the liquid
is quite enough. So, viscous force together with buoyant force becomes greater
than the weight of the cone. So first ball retartes and thereafter will move with
constant velocity
21. (d) The buoyant force
Fb = Fbottom + Fvertical force on slint force
Ê p R2 H ˆ
Á 3 ˜ r g = pR ¥ rgH + Fvertical force on slint face
2
or
Ë ¯
2
fi Fvertical force on slint face = – prgHR2 (downward)
3
22. (b) P1 = Pa, V1 = (500 – H)A
and P2 + 200 = Pa ¥ P2 = (Pa – 200),
V2 = (500 – 200) A = 300 A
From Boyle’s law, we have
Pa ¥ (500 – H)A = (Pa – 200) ¥ 300 A
After substituting values and simplifying, we get
H = 206 mm.
23. (b)

2TR

Ê rgh ˆ
ÁË Pa + ˜ ¥ 2Rh
2 ¯

The net force


Ê r gh ˆ
F = Á P0 + ˜ ¥ 2Rh - 2TR
Ë 2 ¯
= [2P0Rh + Rrgh2 – 2TR].
24. (a) Using Bernoull’s equation, we have

v
,LJĚƌŽƐƚĂƟĐƐŽƌ&ůƵŝĚ^ƚĂƟĐƐപ1167

1
Pa + rv 2 + r gv = Pa + 0 + rg(h + y)
2
v2
\ h =
2g
25. (a) The extra buoyant force,
Fb = Vrg
= (0.20 ¥ 0.20 ¥ 0.04) ¥ 1000 ¥ 10 = 16 N.
26. (a)

Fb
l

l/2 W
q
P

W = Alr, Fb = (Ay)sg
Ê l ˆ
= ÁA sg
Ë 2 sin q ˜¯
Taking moment of all the forces acting on the rod about P, and put equal to zero,
we get
l y
W ¥ cos q - Fb ¥ cos q = 0
2 2
After substituting and simplifying, we get

1 s
sin q =
2 r
m
27. (c) The volume of liquid displaced, V = , does not depend on acceleration due to
r
gravity, so it remains as such.
28. (d)

h/2
h
h v2

v1 = 2 g( h/2)
For v2 we can replace the liquid of density r and height h to a liquid of density 2r
and height h/2. Thus, v2 = 2 gh = 2v
1168പWŚLJƐŝĐƐĨŽƌ//dͲ:͗DĞĐŚĂŶŝĐƐ//

29. (d) P = rgh


= 1000 ¥ 10 ¥ (1 + 0.5 + 0.1)
= 1.6 ¥ 104 N/m2
30. (d) Range, R = vt = 2gh ¥ t ...(i)

Also 2R = 2gh ¢ ¥ t ...(ii)


From above equation, we get h¢ = 4h = 40 m
\ Extra height of water = 30 m = 3 atm.
31. (d)
F

The resultant force at the corner of the hydrant

F¢ = F 2 + F 2 = 2F
= 2rvQ
32. (d) v0 = 2gh

h
h¢ =
2

h 2 gh
Now, v = 2 gh ¢ = 2 g = 4
2 2
v
= 40
2
33. (b)
Fb
a

T
mg

Fb = Vd(g + a)
Fb – (T + mg) = ma
,LJĚƌŽƐƚĂƟĐƐŽƌ&ůƵŝĚ^ƚĂƟĐƐപ1169

or T = Vd(g + a) – Vr(g + a)
= V(d – r)(g + a).
a 2h
34. (b) tan q = =
g l 2h
l
2gh B A
\ a =
l
35. (b) Total horizontal length of liquid tube,
x = 2 2l sin 45∞
= 2l
h = difference in height of liquid in ends, then
a h
tan q = =
g 2l

2al
\ h =
g

36. (b) VAB is the given cone. Let its height be h and semi-vertical angle a. Let the base
AB of the cone be in the surface. CD is the surface of separation of two liquids, O
and O¢ are the centres of the base AB and surface of separation CD.

A O B

(h–z)
s2

C O¢ D
z s1
a

Let VO¢ = x then OO¢ = h – z


\The weight of the cone = (vol. of the cone) rg
1
ph3 tan2a r g
=
3
Volume of liquid (of density s1) displaced
1
pz3 tan2a
= Volume of cone (VCD) =
3
and volume of liquid of density s2 displaced = volume of the frustum ABCD =

Ê1 3 2 ˆ Ê1 3 2 ˆ
ÁË p h tan - a ˜¯ - ÁË p z tan - a ˜¯
3 3
1170പWŚLJƐŝĐƐĨŽƌ//dͲ:͗DĞĐŚĂŶŝĐƐ//

\ For equilibrium,
Weight of the cone = (weight of liquid of density s1 displaced) + (weight of liquid
of density s2 displaced)
1 1 1
or ph3 tan2 arg = pz3 tan2 as1g + p(h3 – z3)tan2 as2g
3 3 3
or h3r = z3s1 + (h3 – z3)s2
or h3(r – s2) = z3(s1 – s2)
1/3
Ê r -s2 ˆ
or z = hÁ
Ë s 1 - s 2 ˜¯
37. (a) For equilibrium
4dA(g = (dAL + m)g
l
and > Ycm (for rotational equilibrium)
2
m ¥ 0 + dAL(L/2) dAL2
Yc.m. = =
m + dAL 2(m + dAL)
AL2 d m + dAL AL2 d
l > fi >
( ALd + m) 4 dA ( ALd + m)
m > ALd
38. (d) The buoyant force,
Fb = Pbottom A – PtopA
fi PbottomA = Fb + PtopA
= Vrg + rgh ¥ pR2
39. (a) P + r1gh1 = P + r2gh2
r1h1 1000 ¥ 20
\ r2 = = = 2000 kg/m3
h2 10
40. (a) Pair – rgl1 = rHggl3 + P0
or Pair = rgl1 + rHggl3 + P0
41. (c) For floating, we have mg = rAv2
mg 1.23 ¥ 10
\ r = =
Av 2 0.1 ¥ 10 2
= 1.23 kg/m3

Multiple Correct Options


1. (b, c)
If Fb is the buoyant force on the block, then
Reading of balance A = 2g – Fb,
and reading of balance B = 5g + Fb
,LJĚƌŽƐƚĂƟĐƐŽƌ&ůƵŝĚ^ƚĂƟĐƐപ1171

2. (a, b, d)
When all the fish start accelerating up, the weight W¢ = M(g + a).
3. (b, c)
Buoyant force on the body, Fb = Vr(g + a) > W
m
The volume of liquid displaced, V¢ = ,
r
which is constant.

4. (b, d)
1 1
0+ rv2 + rgh = Pa + rve2 + 0
2 2
For large container v << ve, and so can be neglected.
\ ve = 2( rgh - Pa )
For ve >> 0, or (rgh – Pa) ≥ 0
Pa 1 ¥ 10 5
or h ≥ = 10 m
rg 1000 ¥ 10
Thus, v1 = 0 and v2 = 0
5. (a, b, c, d)
Let zmin be the minimum reference level of the dotted parabola and z1 and z2 the liquid
levels above the base.
6. (c, d)
In a streamline flow, particles arriving at a given point must have same velocity and
so KE and momenta.
7. (a, c)
For uniform cross section
avA = avB fi vA = vB
From Bernoulli’s equations, we have
1 1
PA + rvA2 = PB + rvB2
2 2
As vA = vB , \ PA = PB
8. (b, c)
At point 1, the space available for the fluid is large and so the speed becomes slower.
Thus, v1 < v2 and P1 < P2

9. (a, c)
500
P = = 5 ¥ 104 N/m2
100 ¥ 10 - 4
Using Bernoull’s equations, we have
1
P + 0 + 0 = Pa + rv2 + rgh
2
1172പWŚLJƐŝĐƐĨŽƌ//dͲ:͗DĞĐŚĂŶŝĐƐ//

1
= 1 ¥ 105 + ¥ 103 ¥ 102 + 103 10 ¥ 0.50
2
= 1.55 ¥ 105 N/m2
If F is the force applied then
F
Pa + = 1.55 ¥ 105
A
F
or 1 ¥ 105 + = 1.55 ¥ 105
A
\ F = 0.55 ¥ 105 ¥ 100 ¥ 10–4 = 550 N
10. (a, c)

H = 2h; x = y( H - y )

dx H
For maximum, = 0, which give y = = h.
dy 2
Also xm = H = 2h.
11. (b, d)
F = rvQ = rAv2
Clearly, when velocity becomes two times, the thrust becomes four times.
Energy lost per second, P = Fv = rAv3, so it becomes eight times.
12. (a, b, c)
Using Bernoulli’s equation between points 2 and 3, we have
1 1
P2 + rv2 + rgh3 = P0 + rv 2 + 0
2 2
\ P2 = (P0 – rgh3)
Now between free surface on container and point 2, we have
1
P0 + 0 + 0 = P2 + rv 2 + 0
2
1
or P0 = P0 – rgh3 + rv 2
2
fi v = 2 gh3 . Clearly, h3 > 0

13. (a, c)
The free surface of water makes an angle
a
tan q = backward
g
or tan q = g/a
14. (a, c, d)
The rate of flow, Q = A 1v 1 = A 2v 2
,LJĚƌŽƐƚĂƟĐƐŽƌ&ůƵŝĚ^ƚĂƟĐƐപ1173

From Bernoulli’s equation,


1 1
P1 + rv12 = P2 + rv22
2 2
2
or v22 – v12 = (P1 – P2) = 2gh
r
Passages and Matrix Matching
Passage (Q. 1–3)
1. (b)

2m

2(P - Pa )
The velocity efflux, v =
r

2 ¥ 8 ¥ 10 5
=
1000
= 40 m/s
The time to fall from height 2 m,

2h 2¥2
t = = = 0.63 s
g 9.8

The distance, x = vt = 40 ¥ 0.63 = 25.2 m


2. (a) the vertical component of velocity
vy2 = 0 + 2g ¥ 2
\ vy = 6.32 m/s
The vertical component of force
Fy = rAvy2 + rghA
= 25.2 N
3. (c) The horizontal component of force on the tank
Fx = rAvx2 = 1000 ¥ 10–4 ¥ (40)2
= 160 N.
1174പWŚLJƐŝĐƐĨŽƌ//dͲ:͗DĞĐŚĂŶŝĐƐ//

Passage (Q. 4–6)


4. (b) The volume of 0.5 kg of gold
500
V = = 25.91 cm3
19.3
5. (a) The volume of 0.5 kg of iridium
500
V¢ = = 22.22 cm3
22.5
To appear this size like gold, the cavity left inside
DV = V – V¢ = 25.91 – 22.22
= 3.68 cm3
6. (a) Iridium should be cheaper.
Passage (Q. 7–9)
Q 3000 ¥ 10 - 6
7. (a) v1 = = = 0.75 m/s
A1 40 ¥ 10 - 4

Q 3000 ¥ 10 - 6
8. (b) v2 = = = 3 m/s
A2 10 ¥ 10 - 4
Using Bernoulli’s equation, we have
1 1
P1 + rv12 = P2 + rv22
2 2
1
\ P1 – P 2 = r(v22 – v12)
2
1
= ¥ 1000 (32 – 0.75)2
2
= 4219 N/m2
4219
9. (b) h =
13.6 ¥ 10 3 ¥ 9.8
= 0.0316 = 3.16 cm
Passage (Q. 10–12)
10. (d) f = 6rhrv = 6p ¥ 10–3 ¥ 10–3 ¥ 3
= 5.65 ¥ 10–5 N
pR 4 (P2 - P1 )
11. (c) From Q =
8hL
pR 4 (P2 - P1 )
We have, h =
8 LQ
Substituting the value, we get h ª 4 ¥ 10–3 Pa-s
,LJĚƌŽƐƚĂƟĐƐŽƌ&ůƵŝĚ^ƚĂƟĐƐപ1175


2vrR
12. (a) From Re =
h
 hRe
v =
2rR
Flow remains laminar till Re = 2000
 4 ¥ 10 - 3 ¥ 2000
\ v = = 0.5 m/s
2 ¥ 1000 ¥ 8 ¥ 10 - 3
Passage (Q. 13–15)
13. (d) Liquid A exerts all round horizontal force on the cylinder and so net on it is zero.
14. (a) If A is the area of cross section of the cylinder, then for the floating cylinder weight
of the cylinder = buoyant force by the liquids
(hA + hB + h)A ¥ 0.8 g = (hAA)rAg + (hBA)rBg
(1.2 + 0.8 + h) A ¥ 0.8 g = (1.2 A) ¥ 0.7 g + (0.8 A) ¥ 1.2 g
\ h = 0.25 cm.
15. (c) The extra buoyant force
Fb = (A h) rB g
Fb
Acceleration a =
m
AhrB g
=
m
Here m = (hA + hB + h)A ¥ 0.8
= (1.2 + 0.8 + 0.25)A ¥ 0.8
A ¥ 0.25 ¥ 1.2 g
\ a =
1.8 A
g
= m/s2
6
Passage (Q. 16–18)
16. (b) Weight of Ram is more than that of Shyam in water means upthrust on Ram is less
hence less volume and less fat content.
17. (a) Let fat mass = m1, other mass = m2,
Total volume = V¢
m1 V m2 3V
Given: = , =
0.4 dw 4 ( 4/3)dw 4
and m1 + m2 = 165
1650
Solving, V =
11dw
1650
Spring balance reading = 165 – dw = 15 kg
11dw
1176പWŚLJƐŝĐƐĨŽƌ//dͲ:͗DĞĐŚĂŶŝĐƐ//

18. (c) Upthrust and effective weight changes by same factor hence fraction immersed
remains same.
Passage (Q. 19–21)
19. (b), 20. (d) , 21. (c)
p
Fv = Ú [rg (2R – R cos q)] ¥ [Rdq ¥ L] cos q
0
p
Fv = rgR2L Ú cos q ( 2 – cos q) dq
0

q q

p
p
= rgR2L Ú (2 cos q – cos2 q) dq = – rgR2L 2
0
p
\ Force = rgR2L upwards
2
As all forces are radial and all pass through axis and hence torque is zero.
p
Fx = Ú rg (2R – R cos q) (Rdq ¥ L) sin q = 4R2rgL
0

Passage (Q. 22–24)


22. (b) Speed of efflux
v2 = 2gh

= 2 ¥ 10 ¥ 5
= 10 m/s
23. (b) By using conservation of mechanical energy, we have
1 2 1
rv2 + r gH = rv32 + 0
2 2

\ v3 = v22 + 2 gH

= 10 2 + 2 ¥ 10 ¥ 5
= 14.14 m/s.
24. (d) Time to empty the tank is given by
A 2
t = ( h1 - h2 )
a g
,LJĚƌŽƐƚĂƟĐƐŽƌ&ůƵŝĚ^ƚĂƟĐƐപ1177

where A = p(1)2
= 3.14 m2
a = 10–4 m2
h1 = 5m
h2 = 2.5 m.
After substituting the values and simplifying, we get
t = 942 s.

Passage (Q. 25–27)


25. (c) The velocity of efflux v = 2gy
( 3 H - 4 h)
= 2g
4
g
= ( 3 H - 4 h)
2
26. (a) The horizontal distance x = v t
2h
= 2 gv ¥
g

= ( 3 H - 4 h )h
27. (c) For maximum x,
dx
=0
dh
1
or (3Hh – 4h2)1/2 ¥ (3h – 8h) = 0
2
- 5h
or (3Hh – 4h2)1/2 = 0
2
Thus, 3Hh – 4h2 = 0
3H
or h=
4
Passage (Q. 28–30)
28. (b) The level in tube A becomes,
8 a 2a
hA = - = 2a
3 3
The level in tube B becomes,
4 a 2a
hB = + = 2a
3 3
Thus, PB – PA = (2r)ghB – rghA
= 2rg ¥ 2a – rg ¥ 2a = 2arg
1178പWŚLJƐŝĐƐĨŽƌ//dͲ:͗DĞĐŚĂŶŝĐƐ//

29. (a)
( a+ 2 a/3 )
30. (b) S(Px – PA) = Ú (dm)w2x
a

( 5 a/3 )
= Ú (rsdx)w2x
a

8
= rsw 2 a 2 ...(i)
9
56
Similarly, S(PB – Px) = rsw2a2 ...(ii)
9
From the above equations
9
Fx = P xs = rsag
4

9g
and w= .
32 a
Passage (Q. 31–33)
31. (d)
a
32. (c) tan q = ; which does not depend on density so, q1 = q2
g

33. (c)
r h

2r h/2
v2

h
The velocity of efflux at hole 1, v1 = 2g = gh .
2
\ F1 = rAv12 = rAgh.
Using Bernoulli’s equation between a point on free surface and 2, we have
Ê hˆ 1
Pa + 0 + Á r gh + 2 r g ¥ ˜ = Pa + (2r)v22
Ë 2¯ 2
\ v2 = 2gh .
Force F2 = (2r)Av22 = 2rA ¥ 2gh = 4 rAgh
The mass of liquid,
m = rA0h + 2rA0h = 3rA0h
,LJĚƌŽƐƚĂƟĐƐŽƌ&ůƵŝĚ^ƚĂƟĐƐപ1179

The initial acceleration of tank


F1 + F2 5 r Agh
a= =
m 3 r A0 h
5 Ag
=
3 A0
Passage (Q. 34–36)
34. (c) Consider the equilibrium of wooden block. Forces acting in the downward direction
are
P1 ¥ p(4r)2

P2 P0 W P2

(i) Weight of wooden cylinder (W)


r hr
= p (4r)2 ¥ h ¥ ¥ g = p ¥ 16r2 g
3 3
(ii) Force due to pressure (P1) created by liquid of height h1 above the wooden
block is
= P1 ¥ p(4r)2 = [P0 + h1rg] ¥ p (4r)2
= [P0 + h1rg] p ¥ 16r2
Force acting in the upward direction due to pressure P2 exerted from below the
wooden block and atmospheric pressure is
= P2 ¥ p [(4r)2 – (2r)2] + P0 ¥ (2r)2
= [P0 + (h1 + h)rg] ¥ p ¥ 12r2 + 4r2P0
At the verge of rising
[P0 + (h1 + h) rg]p ¥ 12r2 + 4r2P0
r
= p ¥ 16r2h ¥ g + [P0 + h1rg] ¥ p ¥ 16r2
3
16 h
fi 12h1 + 12h = + 16h1
3
16 h 5h
fi 12h – fi = h1
3 3
1180പWŚLJƐŝĐƐĨŽƌ//dͲ:͗DĞĐŚĂŶŝĐƐ//

35. (b) Again considering equilibrium of wooden block.


Total downward force = Total upward force
Wt. of block + Force due to atmospheric pressure = Force due to pressure of liquid
+ Force due to atmospheric pressure
r
\ p (16r2) hg + P0p ¥ 16r2
3
= [h2rg + P0]p [16r2 – 4r2] + P0 p 4r2
r
fi p (16r2) h g = h2rg ¥ p ¥ 12r2
3
h 4
fi 16 = 12h2 fi h2 = h
3 9
36. (a) When the height h2 of water level is further decreased, then the upward force
acting on the wooden block decreases. The total force downward remains the
same. This difference will be compensated by the normal reaction by the tank wall
on the wooden block. Thus, the block does not move up and remains at its original
position.
Matrix Matching
37. A Æ s; B Æ r; C Æ q; D Æ p
(A) Poisulli’s law gives,
p Pr 4
Q=
8hl
(B) Viscous change on a spherical body is given by Stokes’ law,
Fb = 6phrv.

(C) Speed of efflux is related to Toricelli’s theorem v = 2gh


(D) Pressure difference between two points can be obtained by Bernoulli’s equation
38. A Æ q; B Æ p; C Æ p; D Æ r
39. A Æ p; B Æ q, r, s; C Æ p; D Æ r
For pipe of uniform cross section v3 = v4 = v5 = v
Applying Bernoulli’s equation between (1) and (5), we have
Summit
Intet
4
leg h1
Pa
3

2
1
h2

5
,LJĚƌŽƐƚĂƟĐƐŽƌ&ůƵŝĚ^ƚĂƟĐƐപ1181

1
Pa = Pa + rv22 - r gh2
2
fi v2 = 2 gh2
Thus, for v2 > 0, h2 > 0
Also
1 1
Pa = P 3 + rv2 = P4 + rv2 + rgh1
2 2
1
= Pa + rv2 – rgh2
2
From the above equation following conclusion can be made
(i) P4 < P3 < Pa
(ii) rg(h1 + h2) = Pa – P4
Pa - P4
fi (h1 + h2) =
rg
Pa
or h1 + h2 <
rg
40. A Æ p, t; B Æ q, s, t; C Æ p, r, t; D Æ q
N
Y f
v
X
(p) q
sin q
Mg q Mg Mg cos q
P

As the velocity is constant,


f = Mg sin q ...(i)
But f = mN = mMg cos q ...(ii)
From Eqs. (i) and (ii),
mMg cos q = Mg sin q fi m tan q
The force by X on Y is the resultant of f and N and is equal to

f 2 + N2 = m 2 N 2 + N 2 = ( m 2 + 1 )N

= ( tan 2 q + 1 ) Mg cos q = sec q Mg cos q = Mg


= Weight of Y.
Therefore, option (A) is correct
Now, due to the presence of frictional force between Y and X, the mechanical
energy of the system (X + Y) decreases continuously as Y slides down.
1182പWŚLJƐŝĐƐĨŽƌ//dͲ:͗DĞĐŚĂŶŝĐƐ//

Therefore, option (C) is correct.

Z v
(q)
Y
Mg
X

As the lift moves up, X also moves up and therefore the gravitational energy of X
is continuously increasing.
\ Option (B) is correct.
The torque of the weight of Y about P is zero as the perpendicular distance of the
line of action of force from the point P is zero.
\ Option (D) is correct.
The force exerted by X on Y will be equal to Mg + Mg = 2 Mg where Mg is weight
of Y and Mg is the force on Y due to Z.
Option (A) is incorrect.

(r)
P
Y

X
v
m0g

In this case the force exerted by X on Y is same as the force exerted by Y on X.


The force on X due to Y is

R= ( Mg )2 + [(m0 + M ) g ]2 π Mg

Mg

m0g

R
Mg

Therefore, option (A) is incorrect.


The mechanical energy of the system (X + Y) is continuously decreasing as the
system is coming down and its potential energy is decreasing, the kinetic energy
remaining the same. Therefore, option (C) is correct and (B) is incorrect.
The torque of the weight of Y about P is not zero.
,LJĚƌŽƐƚĂƟĐƐŽƌ&ůƵŝĚ^ƚĂƟĐƐപ1183

(s) Y

The force on Y by X is equal to the weight of liquid displaced which cannot be


equal to Mg as the density of Y is greater than density of X (as Y is sinking).
Therefore, option (A) is incorrect.
The gravitational potential energy of X increases continuously because as Y moves
down. the centre of mass of X moves up.
Therefore, option (B) is correct.

(t) Y

Sphere Y is moving with terminal velocity. Therefore, the net force on Y is zero,
i.e.,
B Fv

Mg

Mg = B + Fv
where B = buoyant force and Fv = viscous force.
B + Fv are exerted by X on Y.
Therefore, option (A) is correct.
The gravitational potential energy of X is continuously increasing because as Y
moves down, the centre of mass of X moves up.
Option (B) is correct.
The mechanical energy of the system (X + Y) is continuously decreasing to
overcome the viscous forces.
Option (C) is correct.
es
ludst
TM

nc
I at nse

IIT-JEE Qu
L stio
e

II
II I IT - J E E &
Physics for IIT - JEE (Mechanics-II) is designed for the aspirants of JEE and
other engineering examinations. All Other Engineering Examinations
The volumes contain to the point theory and numerous solved numericals along
with practice tests having subjective type, objective type, matrix matching and
integer type questions which provide the aspirants a thorough understanding of
the subject and help prepare them for any type of problem asked in IIT - JEE.
II
Key Features
Develop and hone the problem Mechanics I 20%
solving technique
Abundant tips, short-cut methods
for solving specific problems Oscillations, Waves, Heat
and Thermodynamics 17%
Plenty of conceptual and
application-based numericals with Electrostatics and
detailed solutions and free-body Magnetism 23%
diagrams for better understanding
Optics and Modern Physics 20%

978-93-89307-24-5
Sharma Ashwani Kumar Sharma
Distributed by:
9 789389 307245 TM

You might also like